You are on page 1of 473

Super Course in Mathematics

CALCULUS
for the IIT-JEE
Volume 3

Trishna Knowledge Systems


A division of
Triumphant Institute of Management Education Pvt. Ltd

&KDQGLJDUK‡'HOKL‡&KHQQDL
, 0

7,0( 7,0(LVWKHQDWLRQDOOHDGHULQWHVWSUHSVHJPHQWDQGRQHRIWKHELJJHVW
7

(

DQGWKHPRVWVXFFHVVIXOFODVVURRPEDVHGHQWUDQFHH[DPLQDWLRQWUDLQLQJ
7ULXPSKDQW,QVWLWXWHRI
0DQDJHPHQW(GXFDWLRQ3YW/WG LQVWLWXWHLQ,QGLD)RUPRUHGHWDLOVYLVLWZZZWLPHHGXFDWLRQFRP

The aim of this publication is to supply information taken from sources believed to be valid and reliable. This is not an
attempt to render any type of professional advice or analysis, nor is it to be treated as such. While much care has been taken
to ensure the veracity and currency of the information presented within, neither the publisher nor its authors bear any
responsibility for any damage arising from inadvertent omissions, negligence or inaccuracies (typographical or factual) that
may have found their way into this book.

Copyright © Trishna Knowledge Systems, 2012


Licensees of Pearson Education in South Asia

No part of this eBook may be used or reproduced in any manner whatsoever without the publisher’s prior written consent.

This eBook may or may not include all assets that were part of the print version. The publisher reserves the right to remove
any material present in this eBook at any time.

ISBN 9788131759806
eISBN 9789332511880

Head Office: A-8(A), Sector 62, Knowledge Boulevard, 7th Floor, NOIDA 201 309, India
Registered Office: 11 Local Shopping Centre, Panchsheel Park, New Delhi 110 017, India
Contents
Preface iv

Chapter 1 Functions and Graphs 1.1—1.94


sTUDY MATERIAL
•  Set Theory  •  Cartesian Product of Two Sets  •  Relations  •  Functions 
•  Composition of Functions  •  Inverse of a Function  •  Even and Odd
Functions  •  Periodic Functions  •  Some Real Valued Functions  •  Parametric form of
Representation of a Function  •  Graphs of Conic Sections  •  Graphs of a Few Composite
Functions  •  Transformation of Functions  •  Some Special Curves
Chapter 2 Differential Calculus  2.1—2.196
sTUDY MATERIAL
•  Introduction  •  Limit of a Function  •  Laws on Limits  •  Standard
Limits  •  Continuity of a Function  •  Types of Discontinuities of a Function  •  Concept
of Derivative—Differentiation  •  Differentiability of Functions  •  Derivatives of
Elementary Functions  •  Differentiation Rules  •  Concept of Differential  •  Successive
Differentiation  •  Higher Order Derivatives  •  Tangents and Normals  •  Mean Value
Theorem and its Applications  •  Rolle’s Theorem  •  L’ Hospital’s Rule  •  Extension
of the Mean Value Theorem  •  Increasing and Decreasing Functions  •  Maxima and
Minima of functions  •  Convexity and Concavity of a curve
Chapter 3 Integral Calculus  3.1—3.178

sTUDY MATERIAL
•  Introduction  •  Definite Integral as the Limit of a Sum  •  Anti-Derivatives 
•  Indefinite Integrals of Rational Functions  •  Integrals of the form ∫ dx
,  
a + b cos x
dx a cos x + b sin x
∫ , ∫ dx   •  Integration By Parts Method  •  Integrals of the
a + b sin x c cos x + d sin x

form ∫ ax 2 + bx + c dx  •  Evaluation of Definite Integrals  •  Properties of Definite


Integrals  •  Improper Integrals  •  Differential Equations  •  Formation of a Differential
Equation  •  Solutions of First Order First Degree Differential Equations
Preface
The IIT-JEE, the most challenging amongst national level engineering entrance examinations, remains on the top of the
priority list of several lakhs of students every year. The brand value of the IITs attracts more and more students every year,
but the challenge posed by the IIT-JEE ensures that only the best of the aspirants get into the IITs. Students require thorough
understanding of the fundamental concepts, reasoning skills, ability to comprehend the presented situation and exceptional
problem-solving skills to come on top in this highly demanding entrance examination.
The pattern of the IIT-JEE has been changing over the years. Hence an aspiring student requires a step-by-step study
plan to master the fundamentals and to get adequate practice in the various types of questions that have appeared in the
IIT-JEE over the last several years. Irrespective of the branch of engineering study the student chooses later, it is important
to have a sound conceptual grounding in Mathematics, Physics and Chemistry. A lack of proper understanding of these
subjects limits the capacity of students to solve complex problems thereby lessening his/her chances of making it to the top-
notch institutes which provide quality training.
This series of books serves as a source of learning that goes beyond the school curriculum of Class XI and Class XII
and is intended to form the backbone of the preparation of an aspiring student. These books have been designed with the
objective of guiding an aspirant to his/her goal in a clearly defined step-by-step approach.
• Master the Concepts and Concept Strands!
This series covers all the concepts in the latest IIT-JEE syllabus by segregating them into appropriate units. The theories
are explained in detail and are illustrated using solved examples detailing the different applications of the concepts.
• Let us First Solve the Examples—Concept Connectors!
At the end of the theory content in each unit, a good number of “Solved Examples” are provided and they are designed
to give the aspirant a comprehensive exposure to the application of the concepts at the problem-solving level.
• Do Your Exercise—Daily!
Over 200 unsolved problems are presented for practice at the end of every chapter. Hints and solutions for the same are
also provided. These problems are designed to sharpen the aspirant’s problem-solving skills in a step-by-step manner.
• Remember, Practice Makes You Perfect!
We recommend you work out ALL the problems on your own – both solved and unsolved – to enhance the effective-
ness of your preparation.
A distinct feature of this series is that unlike most other reference books in the market, this is not authored by an in-
dividual. It is put together by a team of highly qualified faculty members that includes IITians, PhDs etc from some of the
best institutes in India and abroad. This team of academic experts has vast experience in teaching the fundamentals and
their application and in developing high quality study material for IIT-JEE at T.I.M.E. (Triumphant Institute of Manage-
ment Education Pvt. Ltd), the number 1 coaching institute in India. The essence of the combined knowledge of such an
experienced team is what is presented in this self-preparatory series. While the contents of these books have been organized
keeping in mind the specific requirements of IIT-JEE, we are sure that you will find these useful in your preparation for
various other engineering entrance exams also.
We wish you the very best!
chapter functions and
graphs

1
nnn  Cha p t e r O u t l i n e
Preview
sTUDY MATERIAL topic grip
Set Theory • Subjective Questions (10)
• Concept Strands (1-8) • Straight Objective Type Questions (5)
• Assertion–Reason Type Questions (5)
Cartesian Product of Two Sets • Linked Comprehension Type Questions (6)
Relations • Multiple Correct Objective Type Questions (3)
Functions • Matrix-Match Type Question (1)
Composition of Functions iit assignment exercise
• Concept Strands (9-10) • Straight Objective Type Questions (80)
Inverse of a Function • Assertion–Reason Type Questions (3)
• Concept Strand (11) • Linked Comprehension Type Questions (3)
Even and Odd Functions • Multiple Correct Objective Type Questions (3)
• Matrix-Match Type Question (1)
Periodic Functions
Some Real Valued Functions Additional Practice Exercise
Parametric form of Representation of a Function • Subjective Questions (10)
• Straight Objective Type Questions (40)
Graphs of Conic Sections
• Assertion–Reason Type Questions (10)
Graphs of a Few Composite Functions • Linked Comprehension Type Questions (9)
• Concept Strands (12-13) • Multiple Correct Objective Type Questions (8)
Transformation of Functions • Matrix-Match Type Questions (3)
Some Special Curves
Concept connectors
• 35 Connectors
1.2  Functions and Graphs

Set Theory

The concept of a set is usually the starting point in the de- If the set contains only one element, it is called a sin-
velopment of basic Mathematics and its applications. gleton set. A set that contains no element is called a ‘null
A set is a well-defined collection of objects. Objects form- set’ and is denoted by f or { }. If the number of elements in
ing part of a set are called its ‘elements’. a set is not finite, it is called an infinite set.
The following are some examples of sets: The following examples of sets illustrate the above defi-
nitions clearly.
(i) Set of people living in a particular town
(ii) Set of English alphabets (i) Set R is an infinite set.
(iii) Set of students in a school whose weights are less than (ii) X = {x, a real number between 10 and 50} also written
45 kg as X = {x ∈ R/10< x < 50} is an infinite set.
(iv) Set of cities in India whose population is greater than (iii) A = {x ∈ N/1 ≤ x ≤ 10} is a finite set and n(A) = 10.
10 lakhs. (iv) {1}, {f} are singleton sets.
(v) If A = {x/x is a prime number and 3 < x < 5} is a null set
We require that the collection of objects, which forms
⇒ A = f.
the elements of the set, be well defined. This means that we
should be able to decide without ambiguity whether an ele-
ment is or is not in a given set. Subset
Sets are usually denoted by capital letters A, B, C, X,
Y etc. If x belongs to a set A, we write x ∈ A. If x does not If every element of a set B is also an element belonging to an-
belong to a set A, we write x ∉ A. other set A, then B is said to be a subset of A and is written as
B ⊆ A. If there exists atleast one element in A not in B, then
we write B ⊂ A.
Representation of sets For example, Q ⊂ R, Z ⊂ R, N ⊂ Q.
(a) Roster form: the elements of the set are listed inside It is the usual convention that null set and the set itself
set brackets. are subsets of a given set.
For example, A = {1, 2, 3, 4, 5, 6, 7}
(b) Set builder form: the elements of the set are represent-
ed by a variable satisfying certain well-defined condi- Number line and Intervals
tions, for example, The elements of the set R of real numbers can be repre-
{ x/x is a counting number less than 5} sented by points on a line called the Number line.

Universal set í 2 

The set of all elements that are of interest in a study is called The point O on this line represents the number zero.
Universal set and is denoted by S. For example, if we are All positive numbers are represented by points on the line
discussing about certain books in a library, then the Uni- to the right of O and all negative numbers are represented
versal set is the collection of all books available in that li- by points on the line to the left of O.
brary. It may be noted that we can move to the right of O
indefinitely without end. We say that there are infinite
number of positive numbers (denoted by ∞). Similarly, we
Finite and Infinite sets can move to the left of 0 indefinitely without end and we
A set consisting of a definite number of elements is a finite say that there are infinite number of negative numbers (de-
set. noted by -∞).
If A is a finite set, the number of elements in A is called The set R of real numbers may be represented by
the cardinal number of A denoted by n(A). (-∞, ∞).
For example A = {3, 7, 9, 11, 13, 19} is a finite set. For
this set, n(A) = 6. D 2 E
Functions and Graphs  1.3

Let a and b represent two elements of R. The set of set is represented by points inside a rectangle and its sub-
points (or the set of real numbers) lying between a and b, sets are represented by points inside closed curves.
inclusive of the two extreme points a and b, may be repre-
sented by [a, b] (called the closed interval).
If the set of points does not include the extreme points Algebra of sets
a and b, we represent this by (a, b) (called the open interval).
(i) Union of sets
[a, b) – set includes a but does not include b.
If A and B are any two sets, the set of all elements that be-
(a, b] – set includes b but does not include a. long to either A or B is called the union of A and B and is
The set of points to the left of a may be represented by denoted by A ∪ B.
(-∞, a] or (-∞, a), according as this set includes or does not
include a. The set of points to the right of b may be repre- 6
sented by [b, ∞) or (b, ∞), according as this set includes or
does not include b. $ %

Power set of a set


$‰%VKDGHGUHJLRQ
The set of all subsets of a given finite set A is called the Power
Fig.1.2
set of A, denoted by P(A).
The number of subsets of a given finite set A is 2n, where, A ∪ B = { x/x ∈ A or x ∈ B}
n is the cardinal number of A. Therefore, n((P(A)) = 2n.
For example, if A = {1, 2}, then P(A) = {f, {1}, {2}, A}. Consider the following examples:
Note that n(P(A)) = 22.
(i) Let A = {a, b, c, d, e}; B = {b, x, c, d, y, f} Then A ∪ B =
{a, b, c, d, e, x, y, f}.
Equal sets (ii) Let A represent the set of points {x/–1 ≤ x ≤ 5} i.e., x
lies in the closed interval [–1, 5]; B represents the set
Two finite sets A and B are said to be equal and we write of points {x/–3 < x < 4} i.e., x lies in the open interval
A = B if every element in A is in B and every element in B is (–3, 4). Then, A ∪ B = {x/–3 < x ≤ 5}
in A i.e., if A ⊆ B and B ⊆ A.
If the numbers of elements of two sets are equal, then
the sets are called equivalent sets. (ii) Intersection of sets
For example, if A = {1, 2, 3, 4} and B = {2, 3, 4, 5}, then
If A and B are any two sets, the set of all elements that be-
A and B are equivalent sets but A ≠ B.
long to both A and B is called intersection of A and B and
is denoted by A ∩ B.
Venn diagrams
S
Sets and operations on sets can be geometrically illustrated
by means of Venn diagrams. In such diagrams, Universal
A B

6 A ∩ B-shaded region
$ %
' Fig.1.3

A ∩ B = { x | x ∈ A and x ∈ B}

If A ∩ B = f, i.e., there are no elements common to


& both A and B, we say that A and B are disjoint sets.
Examples are

Fig.1.1 (i) Let A = {1, 2, 3, 4, 5} , B = {3, 4, 5, 6, 7, 8}, then A ∩ B


= {3, 4, 5}
1.4  Functions and Graphs

(ii) The set of rational numbers and the set of irrational (v) Symmetric difference of two sets
numbers are disjoint sets.
(iii) Let A = {x ∈ Z/x ≥ 0} and B is the set of natural Let A and B be any two sets. The symmetric difference of A
numbers. Then A ∩ B = B. and B is the set of elements that belong only to A or only to
B and is denoted by A D B.
(iii) Complement of a set S
If A is any subset of the universal set S, the set consisting
of the elements in S that do not belong to A is called the A B
complement of A and is denoted by A’ or AC.

S A ∆ B-shaded
region
A Fig.1.6

A D B = (A – B) ∪ (B – A) or (A\B) ∪ (B\A)
A’-shaded region For example, if A = {a, b, c, d, e, f} and B = {c, d, e, f, g, h}
Fig.1.4 then A – B = {a, b} and B – A = {g, h}.\A D B = {a, b, g, h}
Also note that A D B = A ∪ B – A ∩ B.
For example, if S = {1, 2, 3, 4, 5, 6, 7, 8, 9} and
A = {3, 7, 9}, then A’ = {1, 2, 4, 5, 6, 8} (vi) Fundamental laws of set operation
(i) Identity law: A ∪ f = A; A ∩ f = f; A ∪ S = S;
(iv) Difference of two sets
A ∩ S = A.
If A and B are any two sets, the set A – B (or A\B) is the set (ii) Complement law: A ∪ A’ = S; A ∩ A’ = f; (A’)’ = A.
of all elements that belong to A but not to B. (iii) Idempotent law: A ∪ A = A; A ∩ A = A.
S (iv) Commutative law: A ∪ B = B ∪ A, A ∩ B = B ∩ A.
(v) Associative law: (A ∪ B) ∪ C = A ∪ (B ∪ C);
A B (A ∩ B) ∩ C = A ∩ (B ∩ C).
(vi) Distributive law: A ∪ (B ∩ C) = (A ∪ B) ∩ (A ∪ C);
A ∩ (B ∪ C) = (A ∩ B) ∪ (A ∩ C).
A / B-shaded region (vii) De Morgan’s laws : (A ∪ B) ’ = A’ ∩ B’;
Fig.1.5 (A ∩ B) ’ = A’ ∪ B’
(viii) n (A ∪ B) = n(A) + n(B) – n(A ∩ B)
A – B (or A\B) = {x | x ∈ A, x ∉ B} (ix) n(A ∪ B ∪ C) = n(A) + n(B) + n(C) – n(A ∩ B) –
For example, if A = {x ∈ R/1 ≤ x ≤ 2} = [1, 2] and B = n(B ∩ C) – n(C ∩ A) + n(A ∩ B ∩ C)
{x ∈ R/1 < x < 2} = (1, 2), then A - B = {1, 2} and B – A = f. (x) n(A’) = n(S) – n(A)

C o nce p t S t r a n ds
Concept Strand 1 Given n(A) = 400, n(B) = 12000, n(A ∩ B) = 350 and
n(S) = 40000.
A town has total population of 40000 out of which 400 We have to find n(A’ ∩ B’) or n[(A ∪ B)’].
people own cars, 12000 people own motorcycles and Now, n (A ∪ B) = 400 + 12000 – 350 = 12050.
350 people own both cars and motorcycles. How many in \ n [(A ∪ B)’] = 40000 – 12050 = 27950.
the town do not own either?

Solution Concept Strand 2


Let A = people owning cars; B = people owning motor Let A = {1, 2, 3, 4, …., 20}. Find the number of subsets of A
cycles; S = people in the town which contain 5, 6, 7, 8, 9 and 10.
Functions and Graphs  1.5

Solution We are given:


(1) + (2) + (3) + (4) + (5) + (6) + (7) + (8) = 3000
Since 5, 6, 7, 8, 9 and 10 are to be there in all the subsets,
(2) + (3) + (5) + (6) = 2270;
the number of subsets is clearly the number of subsets that
(2) + (5) = 750; (4) + (5) = 450;
can be formed with the remaining 14 elements of A.
(5) = 400; (5) + (6) = 1000;
The answer is 214.
(1) = 250; (7) = 200
Answer (i) = (1) + (2) + (4) + (5) = 250 + 750 + 450
Concept Strand 3 – 400 = 1050.
In an examination, 75% students passed in English, 65% Answer (ii) = (8) = 3000 – (2270 + 450 – 400 + 200
passed in Hindi and 10% failed in both. Find the percent- + 250) = 230.
age of students who passed in both subjects. Answer (iii) = (3) = 2270 – 1000 – 750 + 400 = 920.

Solution
Concept Strand 5
Let A = Set of students who passed in English;
B = Set of students who passed in Hindi In a survey of 100 students in a music school, the num-
Given n (A) = 75, n (B) = 65, n (A’ ∩ B’) = 10. ber of students learning different musical instruments was
⇒ n(A ∪ B) = 90 found to be: Guitar: 28, Veena: 30, Flute: 42, Guitar and
We have, 90 = 75 + 65 – n (A ∩ B) or n (A ∩ B) = 50 or Veena: 8, Guitar and Flute: 10, Veena and Flute: 5, All mu-
The required answer is 50%. sical instruments: 3
(i) How many students were learning none of these three
Concept Strand 4 musical instruments?
(ii) How many students were learning only the flute?
In a competitive examination consisting of three tests viz.,
general knowledge, arithmetic and English, the number of
participants was 3000. Only 2270 participants were able to Solution
get through the arithmetic test, 750 through both arith-
(i) n(G) = 28, n(V) = 30, n(F) = 42
metic and general knowledge, 450 through both general
knowledge and English, 400 through all the three tests ⇒ n (G ∪ V ∪ F) = 28 + 30 + 42 – 8 – 10 – 5 + 3 = 80
and 1000 through arithmetic and English. There were 250 Hence n (G ’ ∩ V ’ ∩ F ’) = 100 – 80 = 20.
participants who got through general knowledge alone but (ii) n(Flute only) = 42 – 10 – 5 + 3 = 30
not through the other two and 200 passed in English only.
(i) How many were in a position to get through general Concept Strand 6
knowledge?
(ii) How many participants failed in all the three subjects? Out of 500 students who appeared at a competitive exami-
(iii) How many got through arithmetic only? nation from a centre, 140 failed in Mathematics, 155 failed
in Physics and 142 failed in Chemistry. Those who failed
in both Mathematics and Physics were 98, in Physics and
Solution Chemistry were 105, and in Mathematics and Chemistry
Set G : Candidates who got through general knowledge. 100. The number of students who failed in all the three
Set A : Candidates who got through arithmetic subjects was 85. Assuming that each student appeared in
Set E : Candidates who got through English all the 3 subjects, find
(i) the number of students who failed in at least one of
the three subjects.
G A
(ii) the number of students who passed in all the subjects.
1 2 3 (iii) the number of students who failed in Mathematics
5 only.
4 6
7 8 Solution
E
Let M: failed in Mathematics; P: failed in Physics; C: failed
Fig.1.7 in Chemistry
1.6  Functions and Graphs

Given n(M) = 140; n(P) = 155; n(C) = 142; n(M ∩ P) Let y ∈ (A\B) ∩ (A\C). Then, y ∈ (A\B) and y ∈
98; n(P ∩ C) = 105; n(C ∩ M) = 100 and n(M ∩ P ∩ C) (A\C)
= 85 ⇒ (y ∈ A and y ∉ B) and (y ∈ A and y ∉ C)
⇒ y ∈ A and (y ∉ B and y ∉ C)
We have,
⇒ y ∈ A and y ∉ (B ∪ C)
 (M ∪ P ∪ C) = n(M) + n(P) + n(C) - n(M ∩ P) -
n
⇒ y ∈ A\(B ∪ C)  — (2)
n(P ∩ C) - n(C ∩ M) + n(M ∩ P ∩ C) = 140 + 155 +
From (1) and (2) we have, A\(B ∪ C) = (A\B) ∩
142 - 98 - 105 - 100 + 85 = 219
(A\C)
Number of students who failed in at least one of the
subjects = n(M ∪ P ∪ C) = 219 (ii) Let x be an element of A\(B ∩ C). Then, x ∈ A\(B ∩ C)
Number of students who passed in all the subjects = ⇒ x ∈ A and x ∉ (B ∩ C)
M’ ∩ P’ ∩ C’ = 500 - n(M ∩ P ∩ C) = 281 ⇒ x ∈ A and (x ∉ B or ∉ C)
⇒ (x ∈ A and x ∉ B) or (x ∈ A and x ∉ C)
Number of students who failed in Mathematics only
⇒ (x ∈ A\B) or (x ∈ A\C)
= n(M) - n(M ∩ P) - n(M ∩ C) + n(M ∩ P ∩ C)
= 140 - 98 - 100 + 85 = 27 ⇒ x ∈ (A\B) ∪ (A\C)  — (3)
Let y ∈ (A\B) ∪ (A\C). We have, y ∈ (A\B) ∪
Aliter: (A\C)
⇒ (y ∈ A\B) or (y ∈ A\C)
Let M: Passed in Mathematics; P: Passed in Physics; C:
Passed in Chemistry. Given: ⇒ (y ∈ A and y ∉ B) or (y ∈ A and y ∉ C)
⇒ y ∈ A and (y ∉ B or y ∉ C)
n(M’) = 140; n(P’) = 155; n(C’) = 142; n(M’∩P’) = ⇒ y ∈ A and y ∉ (B ∩ C)
98; n(P’∩C’) = 105; n(C’∩M’) = 100; ⇒ y ∈ A\(B ∩ C)  — (4)
n(M’∩P’∩C’) = 85; From (3) and (4), A\(B ∩ C) = (A\B) ∪ (A\C)
\ n
 (M’ ∪ P’ ∪ C’) = 140 + 155 + 142 – 98 – 105 – 100 +
85 = 219
Concept Strand 8
n(M∩P∩C) = n[(M’ ∪ P’ ∪ C’)’] = 500 – 219 = 281
Prove that if A and B are any two sets,
The number of students who passed in all the subjects
= 281 (i) A ⊆ B implies B’ ⊆ A’ and B’ ⊆ A’ implies A ⊆ B
The number of students who failed in at least one of (ii) A\B = B’\A’
the subjects = 219
The number of students who failed in Mathematics
only = 140 – 98 – 100 + 85 = 27. Solution
(i) Let A ⊆ B. Let x be an element of B’.
Then, x ∈ B’ ⇒ x ∉ B
Concept Strand 7
⇒ x ∉ A (since A ⊆ B)
Prove that if A, B, C are any three sets, ⇒ x ∈ A’ ⇒ B’ ⊆ A’
(i) A\(B ∪ C) = (A\B) ∩ (A\C) Suppose B’ ⊆ A’
(ii) A\(B ∩ C) = (A\B) ∪ (A\C) Let x be an element of A. Then, x ∈ A
⇒ x ∉ A’ ⇒ x ∉ B’ ⇒ x ∈ B ⇒ A ⊆ B
(ii) Let x be an element of A\B. Then, x ∈ A\B ⇒ x ∈ A
Solution and x ∉ B
(i) Let x be an element of A/(B ∪ C), i.e., x ∈ A\(B ∪ C) ⇒ x ∉ A’ and x ∈ B’ ⇒ x ∈ B’ and
by definition of the set A\B x ∉ A’ ⇒ x ∈ B’\A’  — (1)
⇒ x ∈ A and x ∉ (B ∪ C), Let y be an element of B’\A’. Then, y ∈ B’\A’ ⇒ y ∈ B’
⇒ x ∈ A and (x ∉ B and x ∉ C) and y ∉ A’
⇒ (x ∈ A and x ∉ B) and (x ∈ A and x ∉ C) ⇒ y ∉ B and y ∈ A ⇒ y ∈ A and
⇒ (x ∈ A\B) and (x ∈ A\C) y ∉ B ⇒ y ∈ A\B  — (2)
⇒ x ∈ (A\B) ∩ (A\C) — (1) From (1) and (2), A\B = B’\A’
Functions and Graphs  1.7

Cartesian product of two sets

Let A and B be any two sets. The Cartesian product of A and If n (A) = N 1, n (B) = N 2, then n(A × B)
B denoted by A × B is the set of all ordered pairs (x, y) where = N 1 N 2 = n(B × A).
x ∈ A and y ∈ B.
For example, Let A = {1, 4, 7}; B = {a, p, q, r}
Then, A × B = {(1, a), (1, p), (1, q), (1, r), (4, a), (4, p),
A × B = {(x, y) | x ∈ A and y ∈ B}
(4, q), (4, r), (7, a), (7, p), (7, q), (7, r)}
B × A = {(y, x) | y ∈ B and x ∈ A} B × A = {(a, 1), (a, 4), (a, 7), (p, 1), (p, 4), (p, 7),
(q, 1), (q, 4), (q, 7), (r, 1), (r, 4), (r, 7)}
In general, A × B need not be equal to B × A. Note that A × B ≠ B × A

Relations

Suppose A and B are two sets. Then, a relation from A to B Let another relation R 2 be defined on A as: (l1, l2) ∈ R2
(written as A R B) is defined as a subset of A × B. if line l1 is perpendicular to line l2.
For example, let A = {3, 5, 6, 7, 9} and B = {4, 8, 10}. Clearly, R 2 is not reflexive.
Consider a relation from A to B which is defined as the
set of ordered pairs (x, y) where x ∈ A and y ∈ B such that x
and y are co-primes to each other. (i.e., x and y do not have
a common factor). Symmetric relations
If R 1 represents this relation, it can be seen that
A relation R from A to A is said to be symmetric, if (x, y) ∈ R
R 1 = {(3, 4), (3, 8), (3, 10), (5, 4), (5, 8), (7, 4), (7, 8),
implies (y, x) ∈ R, where x, y ∈ A.
(7, 10), (9, 4), (9, 8), (9, 10)}
Another relation, say R 2 may be defined as set of all For example, let A represent a set of positive integers.
(x, y) such that x + y is a multiple of 2. Let a relation R 1 be defined on A as: (x, y) ∈ R 1 if
We see that R 2 = {(6, 4), (6, 8), (6, 10)}. (x + y) is an even integer.
Clearly, R 1 is symmetric.
Results Let another relation R2 be defined on A as: (x, y) ∈ R 2
y
(i) If R is a relation from A to B, then RC (called if x divides y (i.e., is an integer).
complement of R) is the set (A × B) \ R. x
(ii) If (x, y) ∈ R, then any element of R-1 will be of the form Clearly, R 2 is not symmetric.
(y, x).
(iii) A relation from A to A is a subset of A × A.
Transitive relations
Reflexive relations
A relation R from A to A is said to be transitive if (x, y) and
A relation R from A to A is said to be reflexive if (x, x) ∈ R (y, z) ∈ R imply (x, z) ∈ R.
for all x ∈ A. For example, let A represent a set of positive integers.
For example, let A represent a set of lines in a plane. We define a relation R1 on A as: (x, y) ∈ R 1 if x + y is an even
The elements of A may be denoted by l1, l2, l3, l4,…... integer. Clearly, R 1 is transitive.
A relation, say R 1 on A may be defined as: (l1, l2) ∈ R1 Let another relation R 2 be defined on A as: (x, y)
if line l1 is parallel to line l2. ∈ R 2 if (x 2 + y 2) is a perfect square. Clearly, R 2 is not
Clearly, R 1 is reflexive. transitive.
1.8  Functions and Graphs

Antisymmetric relations Equivalence relations


A relation R from A to A is said to be anti-symmetric, if both A relation R on A is said to be an equivalence relation if R is
(x, y) and (y, x) ∈ R ⇒ x = y. reflexive, symmetric, and transitive.
For example, let A be the set of real numbers and R For example, let A represent a set of positive integers.
be a relation on A defined as; (x, y) ∈ R if x ≤ y. It is clear Define a relation R on A as: (x, y) ∈ R if x + y is an even
that both x ≤ y and y ≤ x are possible only if x = y, i.e., R is integer. Clearly, R is an equivalence relation on A.
anti-symmetric.

Functions

Functions the fundamental boiling blocks for the study of Let f: X → Y be a function
Calculus which is an important branch of mathematics (i) Every x ∈ X is related to some y ∈Y
A relation (or rule) f, which associates to each element of a (ii) One x ∈ X is related to only one y ∈ Y (only one image).
set X a unique element of another set Y is called a function Or if (x, y1) ∈ f and (x, y2) ∈ f imply y1 = y2
from X to Y and is denoted by f: X → Y (read as X maps (iii) Two or more elements of X can have the same image in
into Y under f). Y under a function f.
If x is any element of X, and a rule or a function (iv) There may be elements in Y, which are not images of
f assigns the element y ∈ Y for this x, we say that y is the any elements of X under f.
image of x under f. (v) If n(X) = p and n(Y) = q, then the number of functions
X is called the domain of f. The set of all images (or map- from X to Y is q p.
pings) under f is called the range of f and it is a subset of Y.
And we write y = f(x) where x ∈ domain of f, y ∈ range All relations (or rules) that associate a set of elements
of f. X with set of elements Y cannot be called as functions if
Note the following important characteristics of a (i) or (ii) of the above is not satisfied. This can be pictorially
function: represented as shown in Fig. 1.8.

I I < I I
; < ; ; < ; <

[ \ [ \ [ \ [ \


[ \ [ \ [ \ [ \
[ \ [ \ [ \ [ \
[ \ [ \ [ \ [ \
[ \ [ \ [ \ [ \
\ \ \ \
L LL LLL  LY 
)XQFWLRQV 1RW)XQFWLRQV
Fig. 1.8

One one functions (or injective functions) I


; <
A mapping or function f: X → Y is called one one (injective)
if distinct elements in X have distinct images in Y. [  \
\
Or in other words, if x1 and x2 are two elements in X [ 
\
such that x1 ≠ x2 then f(x1) ≠ f(x2) or f(x1) = f(x2) ⇒ x1 = x2. [  \
An injective function may be represented diagram- [  \
matically as shown in Fig. 1.9. [  \
For example, let X = {3, 4, 9, 11, 16} and Y= {-3, -2, \
-1, 0, 1, 2, 5, 6}
Fig. 1.9
Functions and Graphs  1.9

Let f: X → Y where f = {(3, -3), (4, -1), (9, 6), (11, 0), Bijective functions
(16, -2)}
i.e., image of the element 3 under f is –3, image of the A function, which is both one one and onto (i.e., both injec-
element 4 under f is –1 and so on. tive and surjective) is called a bijective function.
Then f is a one one or injective function. A bijective function may be represented diagrammati-
cally as shown in Fig. 1.11.
Consider another example:
Let X = {1, 2, 3, 4} and Y = {10, 11, 14, 15} For example, let X = {-1, 2, 3, 6} and Y = {p, q, r, s}
Let g: X → Y be such that g = {(1, 14), (2, 14), (3, 15), Let f: X → Y be such that f = {(-1, r), (2, p), (3, s), (6, q)}
(4, 11)}. g is not a one one function. f is a bijective function.
A function, which is not one one is called a many one If f: X → Y is bijective, then n(X) = n(Y)
function.
X Y
f
Onto function (or surjective function)
A function f: X → Y is called onto function (or surjective
function) if every element of Y is an image of at least one x1 y1
element of X or f is surjective (or onto) if for each y ∈ Y there x2 y2
x3 y3
exists at least one x ∈ X such that f(x) = y.
x4
x5 y4
y5
X Y
f
Fig. 1.11
x1 y1
x2 y2
x3 y3
Real valued functions
x4 y4 Functions, which are defined on subsets of real numbers
x5
and whose images are also real numbers, are called real val-
ued functions.
Fig. 1.10
Examples of real valued functions
A surjective function may be represented diagram- (i) Speed of a particle moving on a straight line at differ-
matically as shown in Fig. 1.10. ent times is recorded. If t represents the time and v
For example, let X = {1, 4, 9, 16}, Y = {3, 7, 8} represents the speed at t, v is a function of t and we
And f: X → Y such that f = {(1, 3), (4, 7), (9, 7), (16, 8)}. write v = f(t). (We may use x for t and y for v also.)
f is an onto function. (ii) Area A of a circle depends on the radius r of the
circle. A is a function of r. We know that the rule that
connects r and A is A = p r2.
Remarks (iii) The human population P of the world depends on
the time t. Suppose we have a table giving the human
(i) In the case of surjective functions, Y is the range
population of the world for different years t, then P is a
of f.
function of t.
(ii) A function which is not onto or which is not surjective
is called an into function. In this case there exists We may think of the function f as a machine. (Refer
at least one y ∈ Y which is not the image of any Fig. 1.12).
x ∈ X.
If f is an into function the range of f is a proper subset
of Y. x (input) f y (output)
In the above example, if a function g is defined as g:
X → Y such that g = {(1, 3), (4, 3), (9, 7), (16, 7)}, then
g is an into function. Fig.1.12
1.10  Functions and Graphs

If x is the domain of f, then when x enters the machine


it is accepted as an input by the machine and the machine y
produces the output y(= f(x)) according to the rule of the
function. Graph of y = f(x)

Representation of real valued functions O x


There are three possible ways to represent a real valued
function. Fig. 1.13
(i) Numerical representation by a table of values (called
tabular representation) ( iii) Analytical representation
x x1 x2 x3 x4 …… Suppose the rule or function f is such that image y
of x equals thrice the square of x. We write this rule
y = f(x) y1 y2 y3 y4 …… or function in the form y = 3x2. In this case, we say
(ii) Visual representation (graphical representation) that we have represented the function in the analytical
The most common method for visualizing a function form or in an explicit form as a formula or in a closed
is its graph. form.
If f is a function with domain X then its graph is the It may be noted that from the analytical representation
set of ordered pairs (x, f(x)) i.e., {x, f(x)/x ∈X}. In other of a function we can easily draw its graph and we can
words, the graph of f consists of all points (x, y) in also prepare a table of values of y for different values
the rectangular Cartesian coordinate plane such that of x in the domain. On the other hand, the tabular
y = f(x). form and the graphical form tell us that the variables
We can read the value of y or f(x) for a given x from x and y are related, but the explicit formula or rule
the graph. Moreover, the graph of f(x) gives us a useful is not known from these representations. In many of
picture of the behaviour or life history of a function. It the applications, analytical form (or a closed form)
is also possible to know about the domain and range representation of a function will help us to gain an
as well as where f attains its maximum and minimum insight into the behaviour of the function, its domain,
and what are their values. range and other characteristics.

Composition of functions

Similar to operations of addition and multiplication among Let f: X → Y and g: Y → Z be two functions and let
numbers we can define an operation called ‘composition’ x ∈ X. Then the image of x under f which may be denoted
connecting two functions. by f(x) is in Y.
Since f(x) ∈ Y we can find the image of f(x) under g.
This mapping or function is called a composition of f and
g and is denoted by g o f.
; < = OR
I J If f: X → Y and g: Y → Z are two functions then the
composite function g o f is a function from X to Z such that
I [ g o f (x) = g(f(x)) for every x ∈ X
[ We may represent g o f diagrammatically (Refer
J I [ Fig. 1.14)
J I
Remarks
(i) The composite function or the composition g o f
Fig. 1.14 is defined only if the range of f is a subset of the
domain of g.
Functions and Graphs  1.11

(ii) If the range of g is a subset of the domain of f then we Even if both g o f and f o g are defined g o f need not
can define the composite function or the composition be identically the same as f o g. In other words, g o f
f o g also. ≠ f o g always, i.e., composition of mappings is not
f o g (y) = f(g(y)) for every y ∈Y. commutative.
(iii) If both f and g are bijective functions g o f is also a (v) If f: X → Y, g: Y → Z and h: Z → W are functions then,
bijective function. f o (g o h) = (f o g) o h, i.e., composition of mappings is
(iv) It may be noted that g o f may be defined but f o g is associative.
not defined or f o g is defined but g o f is not defined.

C o nce p t S t r a n ds
Concept Strand 9 Solution
Let f = {(1, 0), (2, 1), (3, 2), (4, 3)} and g = {(0, 1), (1, 1), Both g o f and f o g exist in this case, since both the func-
(2, 2), (3, 2), (4, 3), (5, 3)}. Find g o f and f o g, if they exist. tions are from R → R.
g o f(x) = g(f(x)) = g(2x + 1) = (2x + 1)2
Solution f o g(x) = f(g(x)) = f(x2) = 2x2 + 1
Range of f = (0, 1, 2, 3} and Domain of g = {0, 1, 2, 3, 4, 5}. \ g o f ≠ f o g
Clearly range of f ⊂ domain of g. So we can determine
the composition g o f = g o f(x). (ii) Let f and g are real valued functions defined by f(x) =
We have g o f(1) = g(f(1)) = g(0) = 1 and so on. 3x + 4 and g(x) = x2 - 1. Then find f o g(x2 - 1) and g o
\ We get g o f = {(1, 1), (2, 1), (3, 2), (4, 2)} f(3x + 4)
Now, range of g = {1, 2, 3}. So range of g ⊂ domain of f
We can therefore determine the composite function Solution
fog Both f o g and g o f exist
f o g = {(0,0), (1, 0), (2, 1), (3, 1), (4, 2), (5, 2)}
Note that g o f ≠ f o g f o g(x2 - 1) = f((x2 - 1)2-1) = f(x4 - 2x2)
= 3(x4 - 2x2) + 4
= 3x4 - 6x2 + 4
Concept Strand 10
g o f(3x + 4) = g(3(3x + 4) + 4)
(i) Given that f: R → R defined by f(x) = 2x + 1 and g: R = g(9x + 16)
→ R defined by g(x) = x2, verify if g o f = f o g. = (9x + 16)2 - 1 = 81x2 + 288x + 255

Identity function (ii) Identity function on A is different from identity


function on B since the domains of A and B are
A function IA: A → A defined by IA(x) = x for all x ∈ A different. So we use the notations IA for identity
is called an identity function on A. (i.e., a function which function on A and IB for identity function on B.
maps an element onto itself.) (iii) If f: A → B be any function, then (IB o f) x = IB f (x) =
Note that f(x) = (f o IA)x
(i) Identity function is one one and onto i.e., it is a
This means that IB o f = f o IA = f
bijective function.
1.12  Functions and Graphs

Inverse of a function
If f: X → Y is a bijective function, one can think of a map- (iii) If f-1 is the inverse of f: X → Y then f-1 o f = identity
ping (or function), f -1: Y → X i.e., x will be the image of y. function on X.
(where y = f(x)) under f-1. i.e., f-1 o f(x) = x and f o f-1 = identity function on Y
The function f-1 is called inverse function of f. The do- i.e., f o f-1(y) = y.
main of f-1 is Y and the range of f-1 is X. (iv) If f and g are two bijective functions then f-1 and g-1 are
their respective inverses, then it can be easily verified
Remarks
that (g o f)-1= f-1 o g-1.
(i) Inverse of a function is defined only if it is bijective.
(ii) Inverse of a bijective function is unique and is also a
bijective function.

C o nce p t S t r a n d
Concept Strands 11 (2 x + 5) − 5
For, (f-1o f )(x) = f-1(2x+5) = =x
2
(i) f(x) = {(1, 2), (2, 3), (3, 4), (4, 5)} is a function on Similarly, f o f-1(y) = y.
X = {1, 2, 3, 4} to Y = {2, 3, 4, 5}
(ii) f: R → R defined by f(x) = 2x + 5 Note that the inverse of a function is different from
Find the inverses of these functions, if they exist. the inverse of an element under a function.
Let f: X → Y be any function and let y ∈ Y. Then,
Solution inverse of the element y under f is denoted by f-1(y) and
(i) Its range is Y = {2, 3, 4, 5}. Clearly, f is a bijection (or f it is the set of elements of X which are mapped to y
is a bijective function) from X → Y by the function f. (it is the set of all pre images of y
Now, if we define g: Y → X by g = {(2, 1), (3, 2), (4, 3), under f)
(5, 4)}, then g is the inverse of f. i.e., f −1 (y) = {x | f(x) = y}
(Since g o f(1) = 1, g o f(2) = 2 and so on)
i.e., f-1 = g. For example, consider f: R → R defined by f(x) = x2.
(ii) Consider f: R → R defined by f(x) = 2x + 5. Clearly, f Then f-1(4) = {elements that are mapped to 4 under
is a bijection. f(x) = x2}
y −5
Let f(x) = 2x + 5 = y. Then, x = = {x/f(x) = 4} = {2, -2}.
2
So, the inverse of an element under a function is
y −5
If we define g: R → R by, g(y ) = a set. It may be a singleton set (i.e., a set having one
2 element only) or the set may have more than one element
y −5 as in the above example. It depends on the nature of the
Then g is the inverse of f or f-1(y) =
2 function f.

Even and odd functions

A function f(x) is said to be even if f(-x) = f(x) for all x in The graph of an even function is shown in Fig. 1.15.
its domain. Examples are
The graph of an even function is symmetrical about (i) f(x) = 3x4 - 4x2 + 7
y axis. (ii) f(x) = x6
Functions and Graphs  1.13

Y Examples are
(i) f(x) = x
(ii) f(x) = 2x5 - x3 + 3x
y = f(x)
(even function
Y
X
O

Fig. 1.15 X
O
y = f(x)
(odd function)
A function f(x) is said to be an odd function if f(-x) = -f(x)
for all x in its domain.
Fig. 1.16
The graph of an odd function is symmetrical about the
origin (refer Fig. 1.16)

Periodic functions

A function f(x) is said to be periodic if there exists a non


zero positive number T such that for all x in the domain of Y
f, f(x + T) = f(x).
The smallest positive number T such that f(x + T) =
f(x) is called the period of f(x).
For example in Fig. 1.17, the period of the given func- –6 O 6
1 X
tion is 6, i.e., f(x + 6) = f(x). −11 −7 −5 −1 5 7 11
The best examples for periodic functions are the
circular or trigonometric functions.
In what follows, we take up the study of a few
well-known real valued functions, which are represented Fig. 1.17
in analytical or closed form.

Some Real Valued Functions

Constant functions The graphs of constant functions are lines parallel to x axis.
The range of a constant function is a singleton set { k }.
A function of the form y = f(x) = k (a constant) for all x
represents a constant function.
Polynomial functions
Y
A function of the form y = f(x) = a 0 x n + a1 x n −2 ..... + an–1 x
k y=k
+ an, where a0, a1, a2, a3, …..an are real numbers and n is a
positive integer is called a polynomial function of degree n.
O X The domain of a polynomial function can be R. (The
set of real numbers).
Fig. 1.18
1.14  Functions and Graphs

Case (i) (ii) with one real root


n = 1, y = ax + b is called a linear function. Graph of a linear
function is a straight line (refer Fig. 1.19 (i)) a>0
A special case of linear function is the identity func- y
tion y = x. The graph of the identity function is the line
through the origin making an angle 45° with the positive
direction of x-axis. (i.e., the slope of the line y = x is tan
45° = 1) (refer Fig. 1.19 (ii))
α O x

y y

y = ax + b y=x a<0
x 45° x y
O O

(i) (ii)
Fig. 1.19 α
O x
Case (ii)
If n = 2, y = ax2 + bx + c is called a quadratic function.
If we consider the graph of y = ax2 + bx + c, the real
roots of the quadratic equation ax2 + bx + c = 0 are the
Fig. 1.21
x-coordinates of the points of intersection of the graph
with the x-axis. The graphs of quadratic polynomials are
shown in Fig.1.20, 1.21 and 1.22 and correspond to qua-
dratic functions with two real roots, one real root and no (iii) with no real roots
real roots respectively.
a>0
Case (iii) y

n = 3, y = ax3 + bx2 + cx + d is called a cubic function. The


graph of a cubic function will be as shown in Fig. 1.20.
The range of a cubic function is the set R of real numbers.
O x

Graphs of a quadratic polynomial

(i) with two real roots


a<0
y
D! D
\ \

O x
2
D ȕ [ 2 D ȕ  [

Fig. 1.20 Fig. 1.22


Functions and Graphs  1.15

Graph of a cubic polynomial and cosec x are positive while all the other circular
functions are negative.
 3p 
y (iii) if x lies in the third quadrant,  i.e., if p < x < ,
y  2 
i.e., if the angle x is between 180° and 270°; tan x and
cot x are positive while all the other circular functions
x x are negative.
O O
 3p 
(iv) if x lies in the fourth quadrant,  i.e., if < x < 2p  ,
 2 
Fig. 1.23 i.e., if the angle x is between 270° and 360°; cos x and
sec x are positive while all the other circular functions
Rational functions are negative.
sinx, cosx, cosec x and sec x are periodic functions
P(x) with period 2p while tanx and cot x are periodic func-
A function of the form y = f(x) = where P(x) and
Q(x) tions with period p.
Q(x) are polynomial functions in x is called a rational

function. The domain of this rational function is the set Modulus function
of all real numbers R excluding those numbers for which
Q(x) = 0 − x, x ≤ 0
f(x) = | x |, or f(x) = 
Some examples of rational functions are given  x, x > 0
below:
Domain of the modulus function is R and the range of
(x − 3) the function is [0, ∞).
(i) y = The graph of the modulus function is given in Fig. 1.24
(x + 1) (x − 5)
Domain of the above function is R excluding the
Y
points -1 and 5.
(x 2 + x − 5)
(ii) y =
x(x + 5) (x − 2) (x − 3)
Domain of the above function is R excluding the X
points -5, 0, 2 and 3. O

Circular or Trigonometric functions Fig. 1.24


The functions represented by y = sin x; y = cos x; y = tan x;
y = cosec x; y = sec x; y = cot x are called circular or trigo- Observe that modulus function is an even function. It is
nometric functions. symmetric about y-axis.
Domain of the sine function y = sin x is R and its range
is [-1, 1].
Domain of the cosine function y = cos x is R and its Signum function
range is [-1, 1].
 x
 , x≠0
 p f(x) = | x |
(i) if x lies in the first quadrant,  i.e., if 0 < x <  , i.e.,
 2  0, x=0

if the angle x is between 0° and 90°; all the circular
The signum function can also be expressed as
functions are positive.
−1, x < 0
 p  
(ii) if x lies in the second quadrant,  i.e., if < x < p  , f(x) =  0, x = 0
 2 
 1, x > 0
i.e., if the angle x is between 90° and 180°; sin x 
1.16  Functions and Graphs

The domain of this function is R, while its range is Logarithmic function


{-1, 0, 1}
Graph of the signum function is shown in Fig. 1.25 f(x) = loga x, a > 0, a ≠ 1.
In the case of the logarithmic function, depending on
Y the value of a, the graph will be different, i.e., if a > 1 the
graph is as shown in (i) of Fig. 1.27 and if a < 1, the graph is
1 as shown in (ii) of Fig. 1.27.
Domain of the logarithmic function is (0, ∞) and the
O X range is R.

−1
Y Y

Fig. 1.25 a>1 0<a<1

Observe that signum function is an odd function. It is sym- O1 X X


O1
metric about the origin.

Exponential function
(i) (ii)
f(x) = ex , where e is the exponential number. Fig. 1.27

Y
Remarks
(i) loge x and ex are inverse functions of each other, i.e.,
1 loge(ex) = e loge x = x.
X (Refer Fig. 1.28)
O

Fig. 1.26 Y

To draw the graph of y = ex, we note that when x = 0, y = 1. 1


e being greater than 1, ex increases as x increases through y = ex y = loge x
positive values. This is expressed as ‘As x tends to infinity, X
O 1
ex tends to infinity’.
When x is negative, ex < 1. Since ex is positive for all x
(positive or negative), we have 0 < ex < 1 for x < 0. Also, there
is no x for which ex = 0. Therefore, as x decreases through
negative values, ex decreases and approaches the value zero.
Fig. 1.28
This is expressed “as x tends to - ∞, y tends to zero.
The graph of the function for x < 0 is such that it con-
tinually approaches the negative x-axis, yet never quite
meets it. The graph is said to approach the negative part of Note that the graphs of these two functions reflect each
the x-axis ‘asymptotically’. (Refer Fig. 1.26) other over the line y = x.
We therefore infer that the domain of ex is R and its (ii) In general, we can say that loga x and ax are inverse
range is (0, ∞). functions of each other. (where, a > 0 and ≠ 1)
Functions and Graphs  1.17

Greatest integer function through (1, 0). [the coordinates of (1, 0) satisfy the equation
y = x - 1].
f(x) = greatest integer less than or equal to x, denoted by [ x] In other words, the graph of y = x - 1 (or f(x) = x - 1)
or f(x) = [ x ] represents the greatest integer function. is obtained by translating the graph of y = x through 1 unit
For example: [ 4.3 ] = 4; [ -2.9 ] = -3; [ 5 ] = 5; [ -7 ] = -7. along the positive side of the x-axis.
Similarly, the graph of y = x - 2 is obtained by translat-
Y ing the graph of y = x through 2 units along the positive
3 side of the x-axis and so on. Again, the graph of y = x + 1
is obtained by translating the graph of y = x through 1 unit
2 along the negative side of the x-axis. We are now in a posi-
1 tion to draw the graph of the function f(x) = x - [ x ].
Graph of f(x) = x - [ x ] is shown in Fig. 1.30.
X This function is also called ‘saw tooth wave function’.
− 4 −3 −2 −1 0 1 2 3 4 5
1 Domain of the function is R and its range is [0, 1).
−2 Also note that this function is periodic with period 1.
−3
Unit step function u(x –a)
The unit step function is
Fig. 1.29 Y
defined as
The domain of the greatest integer function is R and its 0, x < a 1
u(x - a) = 
range is the set of all integers. Graph of f(x) = [x] is shown 1, x ≥ a
in Fig. 1.29. a X
Graph of the unit step O
The greatest integer function is also known as staircase function is shown in
function. Fig. 1.31.
f(x) = x – [ x ] Fig. 1.31
If { x } denotes the fractional part of a real number x, Domain of the function is R and its range is the set
it is clear that { x } = x - [ x ], where, [ x ] is the greatest {0, 1}
integer function. Therefore, the above function may also be
expressed as f(x) = { x }.
Note that when x is an integer positive or negative, Catenary function
f(x) = 0.
For 0 < x < 1, f(x) = x; c x −x

The catenary function is defined as f (x) =  e c + e c  , c > 0
1 < x < 2, f(x) = x - 1; 2 
2 < x < 3, f(x) = x - 2 and so on. It is the curve in which a uniformly heavy chain hangs
Also, for -1 < x < 0, f(x) = x + 1; when suspended freely under gravity.
-2 < x < -1, f(x) = x +2 and so on. The function is an even function. Graph of the func-
tion is symmetrical about y-axis. (refer above Fig. 1.32)
Y The domain of the function is R and its range is [c, ∞).
2
1 Y

X
−3 −2 −1 0 1 2 3 4 5

Fig. 1.30 A OA = c
X
O
Recall that f(x) = x or y = x is the identity function whose
graph is the line through the origin making an angle 45°
with the x-axis. Now, y = x - 1 is the line with slope 1
Fig. 1.32
(i.e., making an angle 45° with the x-axis) and which passes
1.18  Functions and Graphs

Parametric form of representation of a function

Consider the circle centered at the origin and whose radius


is r. Let P be any point on this circle where ∠AOP = q. If Y
(x, y) are the coordinates of P referred to a coordinate sys-
tem, then x = OM = r cos q; y = MP = r sin q

Y
B X
O A
P (p, q)

θ (i)
X’ X
A’ O M A
P

B’ P
×
Y’ A B
2πa
Fig. 1.33
(ii)
As q varies from 0 to 2p, P moves along the circle. q = 0 Fig. 1.34
corresponds to A;
p
q= corresponds to B; q = p corresponds to A’ and
2 We have seen that the equation of a locus is the relation
3p between the x and y coordinates of a point on the locus.
q= corresponds to B’. For example, the equation x2 + y2 = r2 is obtained as
2
This means that any point on the above circle can be the locus of a point which moves in a plane such that its
represented as distance from the origin is always a constant r. Therefore,
the equation x2 + y2 = r2 is said to represent a circle cen-
x = r cos q tered at the origin with radius r and its graph is shown in
y = r sin q, where 0 ≤ q < 2p. Fig. 1.35.
This is called the representation of the circle in para-
metric form, where q is the parameter. Y
Another example of a function expressed in paramet-
ric form is P(x, y)
r
x = a(q - sin q)
X
y = a(1 - cos q), where 0 ≤ q < 2p. O
Here, q is the parameter. (We may also use the letter
t instead of q for the parameter). The graph of the above
function is known as Cycloid (refer Fig. 1.34 (i)).
This graph may also be obtained as follows: Consider Fig. 1.35
a circular disc. Mark a point P on its rim (or its circumfer-
ence). Roll the disc on a straight line starting from a point
A such that P coincides with A. The locus of P as the disc is
rolled from A to B on the line such that AB = 2pa, where a From the equation y = ± r 2 − x 2 , for every x (-r ≤ x ≤ r),
is the radius of the disc, is a cycloid. there are two values for y (equal in magnitude but oppo-
The domain of the cycloid function is [0, 2pa] and its
range is [0, 2a]. site in sign). This means that y = f(x) = ± r 2 − x 2 is not a
Functions and Graphs  1.19

function. If we take f(x) = + r 2 − x 2 or f(x) = − r 2 − x 2 , A very important family of curves, which find appli-
then, each represents a function. The circle (or the curve) cations in many problems in real life, is that of the ‘Conic
drawn is that of the locus x2 + y2 = r2, is not that of a sections’. These curves are obtained as sections of a right cir-
function. cular cone by planes. They are also obtained as the locus of
The conclusion is that although every function can be points, which move in a plane satisfying a specific condition.
represented by its graph, every graph (or curve) need not be We give below the equations and graphs of these
that of a function. However, we can always represent any curves. A detailed study of these curves is being undertak-
locus by its graph. en in a later unit.

Graphs of Conic Sections

Parabola (iii) We note that a point whose coordinates are (at2, 2at)
where -∞ < t < ∞ satisfies the equation y2 = 4 ax. Thus,
parametric form of representation of a point on the
Y
parabola
Y
x = at2
2
y 2 = 4ax y = −4ax y = 2at , - ∞ < t < ∞
(a > 0) (a > 0)
(Here, t denotes the parameter)
(iv) These curves are called parabolas. Recall that the
O
X
O
X graphs of quadratic polynomials y = ax2 + bx + c and x
= ay2 + by + c are both parabolas.

(i) (ii)
Ellipse
Y Y The graph of the curve is shown in Fig. 1.37.
2
x = −4ay
(a > 0)
Y x2 y2 Y x2 y2
X O 2
+ 2
=1 2
+ =1
O X a b a b2
x 2 = 4ay
(a > b) (a < b)
(a > 0)
O X O X

(iii) (iv)
Fig. 1.36
Fig. 1.37

Observations Observations
(i) The curves in Fig. 1.36 (i) and (ii) are symmetrical
(i) Ellipse is a closed curve
about x-axis, while the curves in Fig. 1.36 (iii) and (iv)
are symmetrical about y-axis. (ii) The curve is symmetrical about both the axes of
(ii) In the case of the parabola y2 = 4ax, no part of the coordinates.
curve lies to the left of the y-axis; in the case of y2 =
(iii) Domain: -a ≤ x ≤ a and Range: -b ≤ y ≤ b
- 4ax, no part of the curve lies to the right of the y-axis,
in the case of x2 = 4ay, no part of the curve lies below (iv) Parametric form of representation of a point on the
the x-axis, and in the case of x2 = -4ay, no part of the x2 y 2
ellipse 2 + 2 = 1 (a > b) is
curve lies above the x-axis. a b
1.20  Functions and Graphs

x = a cos q c2
Graph of the curve y = (or xy = c2)
y = b sin q, 0 ≤ q < 2p x
(Here, q denotes the parameter) Since division by zero is not defined, the function y = f(x)
c2
= is not defined at x = 0. For all other values of x, y is
x
c2
Hyperbola defined. Hence the domain of f(x) = is R – {0}. Also, y =
x
The graph of the curve is shown in Fig. 1.38 (i). 0 does not correspond to any value of x in the domain. The
range of f(x) is therefore, R – {0}.
As x approaches zero from either side, the point (x,
Y Y y) on the graph moves further and further away from the
x-axis (i.e., y increases indefinitely). The distance from
x2 y2 xy = c2 any point on the graph to the y-axis becomes smaller and
2
− 2
=1
a b smaller on either side. In such a case the graph is said to
X O X approach the ends of y-axis asymptotically.
O
The y-axis is said to be an asymptote of the curve
c2
y = . Similarly, the x-axis is another asymptote.
x c2
(ii) The graph of y = is a rectangular hyperbola and is
(i)
Fig. 1.38 x
shown in Fig. 1.38 (ii)

Remark
Observations
If x represents volume V and y represents pressure P of an
(i) The curve is symmetrical about both the axes of ideal gas, then
coordinates. PV = a constant = c2 (say)
(ii) | x | ≥ a c2 c2
or P = corresponds to y =
(iii) Parametric form of representation of a point on the V x
x2 y 2 (iv) Parametric form of representation of a point on the
hyperbola 2 − 2 = 1 is rectangular hyperbola xy = c2 is
a b
x = ct
x = a sec q c
y = b tan q, 0 ≤ q < 2p y = , − ∞ < t < ∞
t
(Here, q denotes the parameter) (Here, t denotes the parameter)

Graphs of a few composite functions

We discuss below the graphs of a few composite functions.

C o nce p t S t r a n ds
Concept Strand 12 x + 6 x < 0
g(x) = 
 x + 3, − 4 ≤ x < −2 2x + 6 x ≥ 0

Let f(x) = 1 , −2 ≤ x <2
3 − x, 2 ≤ x < 4 Discuss the composite functions g o f(x) and f o g(x)

Functions and Graphs  1.21

Solution y

To discuss g o f(x), we need to ascertain intervals for


f(x) > 0 and f(x) < 0
\


0 x
−4 −3 −2 −1 1 2 3 4

       [


Graph of y = g  f(x)

We know that g(x) = x + 6 when x < 0.
\ I [  For g(x) = - 4, x = - 10,
\ g(x) = -2 x = -8,
g(x) = 2 x = -4,
g(x) = 4 x = -2,
\ [
Hence,

(x + 6) + 3 , −10 ≤ x < −8
\ [ 
f o g(x) =  1 , −8 ≤ x < −4
 [ 3 − ( x + 6) , −4 ≤ x ≤ −2

\ J [ 
 x + 9, − 10 ≤ x < −8

f(x) < 0 in (-4, -3) and in (3, 4) ⇒ f o g(x) = 1, − 8 ≤ x < −4
−3 − x, − 4 ≤ x ≤ −2
Therefore, 

 (x + 3) + 6 = x + 9, −4 ≤ x < −3 The graph of y = f o g(x) is shown below


g o f(x) = 
(3 − x) + 6 = − x + 9, 3 < x ≤ 4
y
f(x) > 0 in - 3 < x < 3 :
Therefore, g o f(x) =
1
2(x + 3) + 6 = 2x + 12, −3 ≤ x < −2

 2 × 1 + 6 = 8, −2 ≤ x < 2
2(3 − x) + 6 = 12 − 2x, 2 ≤ x < 3
−10 −4 −2 0
 −8
 x + 9, −4 ≤ x < −3 −1
2x + 12, −3 ≤ x < −2

Consolidating, g o f(x) =  8, −2 ≤ x < 2
12 − 2x,
Graph of f  g(x)
2≤x<3 Graph of f o g(x)

 9 − x, 3≤x≤4

The graph of y = g o f(x) is shown below Concept Strand 13


 hen we consider f o g(x), we should first note that
W f(x) = sin x , x ∈ R; g(x) = 3[x] where, [ ] denotes the greatest
the domain of f(x) is [-4, 4]. We also note that g(x) integer function
takes values in [-4, 4] only when x < 0. Determine the function g o f(x)
1.22  Functions and Graphs

Solution  p
1 0 ≤ x < 2
We consider the function f(x) in [0, 2p], since f(x) is a 
periodic function with period 2p. 3 x = p
 2
g o f(x) = 3[sin x] and therefore it is convenient to  p
p 1 <x≤p
consider the function in intervals of . ⇒ g o f(x) =  2
2 1 3p
 p<x≤
x f(x) = sin x [sin x] g o f(x)  3 2
 1 3 p
x=0 f(x) = 0 0 g o f(x) = 1 < x < 2p
3 2
1 x = 2p
p g o f(x) = 1 
0<x< 0 < f(x) < 1 0
2 since f(x + 2p) = f(x)
p g o f(x) = 3
x= f(x) = 1 1 g o f(x + 2p) = f(x)
2
⇒ g o f(x) is periodic with period 2p
p <x<π g o f(x) = 1
1 > f(x) > 0 0
2
y
x=π f(x) = 0 0 g o f(x) = 1
3
3p 1
π<x< –1 < f(x) < 0 –1 g o f(x) =
2 3
3p 1 1
x= f(x) = - 1 –1 g o f(x) = ⅓
2 3 x
0 π π 3π 2π
3p 1 2 2
< x < 2π –1 < f(x) < 0 –1 g o f(x) =
2 3
Graph of g  f(x) is [0, 2π]
x = 2π f(x) = 0 0 g o f(x) = 1

Transformation of functions
By applying certain transformations to the graph of a Vertical and Horizontal shifts:
given function, we can obtain the graphs of certain related
Suppose k > 0.
functions. This will give us the ability to sketch the graphs
Table 1.1
of many functions quickly by hand.
To obtain the graph of To do
(i) Let us first consider translations. If k is a positive y = f(x) + k shift the graph of y = f(x) a
number, then the graph of y = f(x) + k is just the distance k units upwards (↑)
graph of y = f(x) shifted upwards a distance of k units
(because each y coordinate is increased by the same y = f(x) - k shift the graph of y = f(x) a
number k). distance k unit downwards (↓)
Likewise, if g(x) = f(x - k) where, k is a positive num- y = f(x + k) shift the graph of y = f(x) a
ber, then the value of g(x) at x is the same as the value distance k units to the left (←)
of f(x) at (x - k) (the point (x - k) is k units to the left of y = f(x - k) shift the graph of y = f(x) a dis-
the point x). Therefore, the graph of y = f(x - k) is just tance k units to the right (→)
the graph of y = f(x) shifted k units to the right. Our Figure 1.39 gives the graphical representations of these
findings are presented in table 1.1. transformations.
Functions and Graphs  1.23

(ii) Again, if k > 1, then the graph of y = kf(x) is the (iii) We use the functions y = sin x and y = cos x for the
graph of y = f(x) stretched by a factor of k in the graphical illustrations of the transformations y = f(kx)
vertical direction (because for each x coordinate the
1 
corresponding y coordinate is multiplied by the same and y = f  x  , k > 1.
1 k 
number k). Clearly, the graph of y = f(x), k > 1, is the
k
graph of f(x) compressed by a factor of k in the vertical Y
direction.
Also note that in both of the above cases, the intercepts y = sin 2x y = sin x
1
made by the graphs y = f(x) and y = kf(x) with the
x-axis remain the same. 3π/2 4π
X
O π π

Y 2
−1
y = f(x) + k
(i) Graph of y = sin2x
Y

k
y = f(x + k) y = f(x) y = f(x− k) y = sin x
1
k k
3π/2 4π
X
k O π π

X 2
O
−1 x
y = sin
2
y = f(x)− k x
(ii) Graph of y = sin
2
Y
Fig. 1.39
2

When k is negative, the graph of y = kf(x) is the y = 2sin x


1 y = sin x
reflection of the graph y = |k| f(x) in the x-axis.
In particular if k = -1, y = -f(x) is the graph of y = 3π/2 4π
X
O π π
f(x) reflected in the x-axis because the point (x, y) is 2π
2
replaced by the point (x, -y). −1
It easily follows that y = f(-x) is the graph of y =
−2
f(x) reflected in the y-axis.
(iii) Graph of y = 2sinx
y = -f(x) → reflection of the graph of y = f(x) in Y
the x-axis
y = f(-x) → reflection of the graph of y = f(x) in
y = |sin x|
the y-axis. 1
Fig. 1.40 illustrates the above.
3π/2 4π
X
O π π

Y 2
k>1 −1 y = sin x

y = k f(x) (iv) Graph of y = |sinx|

Fig. 1.41
1
y = f(−x) y= f(x)
y = f(x) k
Observation
X
O
From Fig. 1.41, we observe that period of the function
y = −f(x)
y = sin x is 2p. Period of the function y = sin 2x is p and
x
Fig. 1.40 period of the function y = sin is 4p.
2
1.24  Functions and Graphs

Similar is the case for cosine function. (Refer Fig. 1.42) We therefore infer that if n is a rational number (n > 0), the
2p
period of the functions sin nx or cos nx is .
Y n
Note:
y = cos 2x From the graphs (i) and (ii) we infer that the range of y =
y = cos x
1 x
sin2x and y = sin remain same as that of y = sin x, i.e., y
2
3π/2 X ∈ [-1, 1]. From graph (iii) we infer that range of y = 2sin x
O π π 2π 4π
is y ∈ [-2, 2] but the period of the graph is 2p (same as that
2
−1 x of y = sin x). Graph (iv) shows us that the range of y = |sin
y = cos x| is y ∈ [0, 1] and the period is p.
2

Fig. 1.42

Some Special Curves

x3 The graph of the curve is shown in Fig. 1.43 (ii). Curve


Graph of the curve y 2 = ,a > 0 is said to have a loop between 0 and a. Before we conclude,
2a − x
we mention about the curve whose equation is
If (x, y) is a point on the above curve, it is clear that (x, -y) is
2 2 2
also a point on it. This means that the curve is symmetrical x 3
+y 3
= a 3 , a > 0.
about x-axis. i.e., suppose y is changed to -y in the equation
of a curve and the equation remains unaltered. Then, the
curve is symmetrical about x-axis. Y Y
We note the following:

(i) (0, 0) satisfies the equation. Therefore, origin is a point x = 2a


A
on the curve. 0 X O X
(ii) x cannot be negative, since y2 in this case becomes
negative. Therefore, no part of the curve lies to the left
of the y-axis.
(iii) x cannot be greater than 2a. (as y2 becomes negative
(i) (ii)
in this case). As x is increased from 0 to 2a, the
denominator (2a – x) becomes smaller and smaller. Y
Consequently, y increases indefinitely. We say, “as x
increases from 0 to 2a, y tends to infinity”. Hence, x = B
2a is an asymptote of the curve.
The graph of the curve is shown in Fig. 1.43 (i)
A’ O A X
Graph of the curve y2 = x(x – a)2, a > 0
We note that the curve passes through the origin and
(a, 0) is a point on the curve. The curve is symmetrical B’
about x-axis and x cannot be negative. This means that no (iii)
part of the curve lies to the left of y-axis. As x increases
from a, y increases or as x tends to infinity, y tends to
Fig. 1.43
infinity.
Functions and Graphs  1.25

2 2 2 The parametric form of representation of a point on


Graph of the curve x 3
+y 3
=a 3
,a>0 the above hypocycloid is
This closed curve, whose graph is as shown in Fig. 1.32 (iii), x = a cos3 q,
is known as astriod or hypocycloid. This curve intersects y = a sin3 q, where 0 ≤ q ≤ 2p.
the x-axis at (a, 0) and (-a, 0) and the y-axis at (0, a) and
(0, - a). The curve is symmetrical about both the coordi-
nate axes. Also, -a ≤ y ≤ a.

Summary
1. Relations
Let A and B are two sets then a relation from A to B is defined as subset of A × B
(i) Reflexive relation–A relation R from A to A is said to be reflexive if x R x for all x ∈ A.
(ii) Symmetric relations–A relation R from A to A is said to be symmetric, if x R y
⇒ y R x for x, y ∈ A.
(iii) Transitive relations–A relation R from A to A is said to be transitive if x R y and y R z
⇒ x R z for all x, y, z ∈ A.
(iv) Antisymmetric relations–A relation R from A to A is said to be antisymmetric if
x R y and y R x ⇒ x = y
(v) Equivalence relations–A relation R on A is said to be an equivalence relation if R is
reflexive, symmetric, and transitive.
2. Functions
(i) A relation f which associates to each element of a set X a unique element of another set Y is called a function
from X to Y and denoted by f: X → Y, X is called domain Y is called co-domain and set of all images of under
f is called range.
(ii) Number of relation from set A to set B when n
(A) = m and n (B) = n is 2mn.
(iii) Similarly, number of functions from A to B = nm
3. Composition of functions
Let f : X → Y and g : Y → Z are two functions then the composite function g  f is a function from X to Z such that
g  f(x) = g(f(x)) for every x ∈ X.
(i) g  f is defined only when range of f is subset of domain of g.
(ii) f  g is defined only when range of g is subset of domain of f.
(iii) f  g ≠ g  f.
(iv) f and g are bijective function g  f is also bijective function.
(v) f  (g  h) = (f  g)  h i.e., composition of mappings is associative.
4. Types of functions
(i) Identity function
A function IA: A → A is called an identity function of I(x) = x for all x ∈ A
(ii) Inverse function
If f : X → Y is a bijective function, then f -1: Y → X is called inverse function of f.
f  f−1 = identity function
(g  f)−1 = f−1  g−1
1.26  Functions and Graphs

(iii) Even function and odd function


A function f(x) is said to be even if f(−x) = f(x) for all x in its domain.
Graph of even function is symmetric about y-axis. A function f(x) is said to be odd if f(−x) = −f(x) for all x in
its domain. Graph of odd function is symmetric about the origin.
(iv) Periodic functions
A function f(x) is said to be periodic if there exists a positive number T such that f(x + T) = f(x) for all x in its
domain.
The least positive value T is called the period. If f(x) is periodic with period T, then f(ax + b) is periodic with
T
period , a > 0, b ∈ R.
a
If f1(x), f2(x), f3(x) are periodic with periods T1, T2, T3 respectively then a1f1(x) + a2f2(x) + a3f3(x) is periodic
with period equal to L.C.M of T1, T2 and T3 where a1, a2, a3 are non zero real numbers.
If f(x) is periodic function with period T and g(x) is any function such that domain of f is a proper subset of
domain of g, then g  f is periodic with period T.
(v) Rational function
P(x)
A function is of the form where, P(x) and Q(x) are polynomial function, are called rational function.
Q(x)
(vi) Modulus function
x , x>0
Function f(x) = |x| = 
− x , x < 0
Domain of modulus function is R range is [0, ∞). It is an even function.
(vii) Signum function
 x −1, x < 0
 , x≠0 
f(x) = | x | =  0, x = 0
 0 , x=0  1, x > 0
 
Domain of signum function is R range = {−1, 0, 1}
(viii) Exponential function
f(x) = ex, where e is the exponential number.
Domain = R and range = (0, ∞)
(ix) Logarithmic function
f(x) = logax, a > 0, a ≠ 1
Domain = (0, ∞) range = R
(x) Greatest integer function.
f(x) = [x] = greatest integer less than or equal to x
Domain = R range = Z
(xi) Fractional part functions
f(x) = {x} = x − [x]
Domain = R range = [0, 1)
It is periodic function with period = 1
(xii) Unit Step function
0, x < a
u(x − a) = 
1, x ≥ a
Domain = R range = {0, 1}
Functions and Graphs  1.27

(xiii) Catenary function


c  xc −x

f(x) =  e + e c , c > 0
2 
Domain = R range [c, ∞)
It is an even function.
5. Parametric form representation of some curves
(i) Parametric representation of circle x2 + y2 = r2 is
x = r cosθ
y = r sinθ, θ ≤ θ < 2π
(ii) Parametric form of parabola y2 = 4ax is
x = at2
y = 2at, −∞ < + < ∞
x2 y 2
(iii) Parametric form of ellipse 2 + 2 = 1 is
a b
x = a cosθ
y = b sinθ, θ ≤ θ < 2π
x2 y 2
(iv) Parametric form of hyperbola − = 1 is
a 2 b2
x = a sec θ
y = b tan θ, 0 ≤ θ < 2π
that of rectangular hyperbola xy = c2 is
x = ct
c
y= , −∞<t<∞
t
6. Transformation of functions

To obtain the graph of To do


y = f(x) + k shift the graph of y = f(x) a distance k units upwards (↑)
y = f(x) − k shift the graph of y = f(x) a distance k unit downwards (↓)
y = f(x + k) shift the graph of y = f(x) a distance k units to the left (←)
y = f(x − k) shift the graph of y = f(x) a distance k units to the right (→)

y = −f(x) → reflection of the graph of y = f(x) in the x-axis


y = f(−x) → reflection of the graph of y = f(x) in the y-axis.
1.28  Functions and Graphs

CONCEPT CONNEC TORS

Connector 1: A represents the set of positive integers > 1. A relation R is defined on A as: (x, y) ∈ R if x = y (mod 4).
Examine whether R is reflexive, symmetric, and transitive.
Solution: x = y (mod 4) means that both x and y leave the same remainder when they are divided by 4. or, in other
words, x = y (mod 4) means that (x – y) is divisible by 4.
Clearly, (x – x) is divisible by 4.Therefore, R is reflexive.
Also, if (x – y) is divisible by 4, (y – x) is also divisible by 4. Therefore, R is symmetric.
If (x – y) is divisible by 4 and (y – z) is divisible by 4,
x – z = (x – y) + (y – z) is divisible by 4. Therefore, R is transitive.
We conclude that R is an equivalence relation.

Connector 2: Let A = [Rectangles of the chess Board]. Define a relation R in A: Say (R 1, R2) ∈ Relation R, if rectangles
R1 and R2 have a common side. Examine R.
Solution: Clearly, R is reflexive. Also, if (R1, R2) ∈ R, (R2, R1) will be in R. i.e., R is symmetric.
However, if R1 and R2 have a common side and R2 and R3 have a common side, it does not imply R1 and R3
have a common side. In other words, R is not transitive.
Connector 3: A represents the set of real numbers. Examine the relation R in A:
(x, y) ∈ R, if | x – y | ≤ 5 where, x, y ∈ A.
Solution: x ∈ R. Since | x - x | = 0 ≤ 5, R is reflexive.
Also, | x – y | ≤ 5 implies | y – x | ≤ 5 or R is symmetric.
| x – y | ≤ 5 and | y – z | ≤ 5 does not imply | x – z | ≤ 5. Therefore, R is not transitive.

Connector 4: A represents the set {2, 3, 5}. Obtain the number of symmetric relations that are possible in A.
Solution: A × A = {(2, 2), (3, 3), (5, 5), (2, 3), (3, 2), (2, 5), (5, 2), (3, 5), (5, 3)}.
Since, in any symmetric relation both (x, y) and (y, x) are to be present, we treat {(2, 3), (3, 2)}, {(2, 5),
(5, 2)}, {(3, 5), (5, 3)} as three members. Together with (2, 2), (3, 3), (5, 5), we have 6 elements.
The number of symmetric relations possible is clearly the number of subsets that can be formed with these
6 elements. And it is equal to 2 6 or 64. The answer is therefore 64.
x
Connector 5: Examine the nature of the function f(x) = 3 , x ∈ R.
x
Solution: (i) If we take f(x) = 3 , R →R, we see that f(x) is not one one. Also, it is not surjective.
In fact, the range of f (x) is [1, ∞). The function is even.
x
(ii) If we take f(x) = 3 , R → [1, ∞), f is surjective.
Connector 6: Find the periods of the following functions.
(i) f(x) = k (a constant)
(ii) f(x) = sin2x
(iii) f(x) = 5 cos3x -2
(iv) f(x) = a cos nx, n is a rational number.
(v) f(x) = a sin nx, n is a rational number.
Functions and Graphs  1.29

Solution: (i) Since f(x) is a constant for all x, f(x + T) = f(x) = k for any positive number T.
Therefore, a constant function is periodic and the period can be taken to be any positive number.
(ii) Now, sin (2x + 2p) = sin 2x, since all circular functions are periodic functions with period 2p.
⇒ sin (2x + 2p) = sin [2(x + p)] = sin 2x Or f(x + p) = f(x) where f(x) = sin2x
⇒ sin 2x is periodic with period p.
(iii) We have 5 cos (3x + 2p) - 2 = 5 cos 3x - 2
  2p  
⇒ 5cos 3  x + − 2 = 5cos3x − 2
  3  
 2p 
⇒ f  x + = f(x)
 3 
2p
⇒ f(x) is periodic with period
3
(iv) and (v)
  2p  
We have cos n  x + = cos(nx + 2p) = cos nx
  n  
  2p  
and sin n  x + = sin(nx + 2p) = sin nx.
  n  
2p
⇒ Both cos nx and sin nx are periodic functions with period
n
 x , 0 < x < 2
2
f(11) − f(−11)
Connector 7: Given f(x) =  and f(x+5) = f(x) for all x. Compute
 x + 2, 2 ≤ x ≤ 5 f(11) + f(−11)
Solution: f(x + 5) = f(x) ⇒ f(x) is periodic with period 5.
Now f(11) = f(10 + 1) = f(1) = 1; f(-11) = f(-11 + 15) = f(4) = 4 + 2 = 6
f(11) − f(−11) 1 − 6 −5
\ = =
f(11) + f(−11) 1 + 6 7

 1 + x3 
Connector 8: If f(x) = log e  , find f(2x) + f(-2x)
 1 − x 3 
 1 + 8x 3   1 − 8x 3 
Solution: f(2x) + f(-2x) = log e  3 
+ log 
 1 − 8x   1 + 8x 3 

 (1 + 8x 3 ) (1 − 8x 3 ) 
= log e  = log e 1 = 0
 (1 − 8x 3 ) (1 + 8x 3 ) 
25x
Connector 9: If f(x) = , show that f(x) + f(1 - x) = 1
25x + 5
251− x 25
Solution: f(1- x) = 1− x
= ,
25 + 5 25 + 5 × 25x
(on multiplication of numerator and denominator by 25x)
5
= x
25 + 5
25x 5
\ f(x) + f(1 - x) = x + =1
25 + 5 25x + 5
1.30  Functions and Graphs

Connector 10: Find the domains of the following functions.


(i) f(x) = log(x − 3) + x 2 − 5x + 6
1
(ii) f(x) = + sin x
(x − 2)(x + 1)
(iii) f(x) = log N , where N > 0
x
Solution: (i) Observe that x – 3 > 0 or x > 3 and x2 - 5x + 6 ≥ 0 i.e., x ≤ 2 or x ≥ 3. Domain is: x > 3.
(ii) x should not be equal to –1 or 2 and also x ≥ 0. This means that the domain of the function is [0, ∞)
excluding x = 2.
(iii) The base of the logarithm should be positive and not equal to 1.The domain is therefore x ∈ (0, ∞)
excluding x = 1.
Connector 11: A function f(x) is defined for x ∈ [0, 1]. What is the domain of f (2x + 3)?
−3  −3 
Solution: We must have 0 ≤ 2x + 3 ≤ 1 or ≤ x ≤ − 1 . Or the domain is  , − 1 .
2 2 
Connector 12: Find the domain of the function y = sin 1 − 2 [x] + log e (1 − [ x]) where, [x] represents the greatest
integer function.
Solution: We have f(x) = sin 1 − 2 [x] + log e (1 − [ x])
Logarithm and square root in f can be defined only if
1
1- 2[ x ] ≥ 0 and (1- [ x ] ) >0 i.e., [ x ] ≤ and [ x ] < 1
2
Combining, [ x ] = 0, -1, -2, -3, …….. ⇒x ∈ (-∞, 1)
\ Domain of f = (-∞, 1)

Connector 13: Find the range of the function: f(x) = 3x 2 + 5x + 7 for real x.
Solution: Note that the discriminant of the quadratic is < 0.
f(x) has the same sign as that of the coefficient of x 2 for all real values of x or f(x) is positive for all real x.
 5x 7    5
2
59 
Again, 3x 2 + 5x + 7 = 3  x 2 + +  = 3 x +  + 
 3 3   6 36 
−5 59
The minimum value of f (x) is obtained for x = and it is equal to .
6 12
 59 
The range of the function is  , ∞  .
 12 
x
Connector 14: Find the range of f(x) = 2 .
(x − 5x + 9)
Solution: Observe that since the discriminant of the denominator of f(x) is negative, (x 2 – 5x + 9) will never become
zero for any real x.
x
If y represents an element in the range, y = 2 ,
x − 5x + 9
giving yx – (5y + 1) x + 9y = 0
2

Since x ∈ R, the above quadratic equation must yield real roots.


\(5y + 1)2 – 36y 2 ≥ 0.
−1
Simplification gives 11y 2 – 10y – 1 ≤ 0 or y should lie between and 1.
11
 −1 
The range of f(x) is therefore  , 1 .
 11 
Functions and Graphs  1.31
5
Connector 15: Find the range of the function f(x) = , x ∈ R.
3 + 2sin x
Solution: Observe that (3 + 2 sin x) will not reduce to zero for any real x.
We have – 1 ≤ sin x ≤ 1
⇒ – 2 ≤ 2 sin x ≤ 2 or 1 ≤ (3 + 2 sin x) ≤ 5
1 1
⇒1≥ ≥
3 + 2sin x 5
5
or 5 ≥ ≥ 1.
(3 + 2sin x)
The range of f(x) is [1, 5].
(x − 1)
Connector 16: If f(x) = , for all real x except x = 0 and g(x) = x 2 + 1 for all real x, find g o f (1) and f o g (–1)
x
Solution: f(1) = 0, g o f(1) = g(0) = 1.
1
g( –1) = 2, f o g(–1) = f(2) = .
2
Connector 17: Find the range of the function f(x) = |x - 1| + |x - 2|, x ∈ R Y
Solution: For x < 1, f(x) = 1 - x + 2 - x = 3 - 2x
For 1 ≤ x < 2, f(x) = x - 1 + 2 - x = 1
For x ≥ 2, f(x) = x - 1 + x - 2 = 2x - 3 1
The graph of f(x) is shown in the figure.
X
The range of f(x) is easily seen as [1, ∞) 0 1 2
2x + 3, 0 ≤ x ≤1
 4x, 1 ≤ x ≤ 2
Connector 18: Let f(x) =  x 2 + 4, 1 < x ≤ 3 and g(x) =  x
4x +1, x > 3 e , x > 2

Obtain the composite functions g o f and f o g
Solution: For 0 ≤ x ≤ 1, the range of f is [3, 5]
For 1 ≤ x ≤ 3, the range of f is [5, 13]
For x > 3, the range of f is (13, ∞)
We are now in a position to write the composite function g o f.
e2x + 3 , 0 ≤ x ≤1
 2
g o f(x) = e x + 4 , 1 < x ≤ 3
 e 4 x +1 , x > 3

Again, For 1≤ x ≤ 2, the range of g is [4, 8]
For x > 2, the range of g is (e2, ∞)
4(4x) + 1 = 16x + 1, 1 ≤ x ≤ 2
f o g(x) =  x
4e + 1, x >2

2x + 3, 0 < x < 1 x + 1 , 2 < x < 4


Connector 19: If f(x) =  and g(x) = 
2 + 3x, 1 ≤ x < 2 4[x + 1], x ≥ 4
when [x] represents the greatest integer function. Determine g o f(x)

1
Solution: If 0 < x < , then 3 < f(x) = 2x + 3 < 4 ⇒ g(f(x)) = (2x +3) + 1
2
1
If ≤ x < 1 , then 4≤ f(x) = 2x + 3 < 5 ⇒ g(f(x)) = 4[2x + 3 + 1]
2
1.32  Functions and Graphs

If 1 ≤ x < 2, then 5 ≤ f(x) = 2 + 3x < 8 ⇒ g(f(x)) = 4[2 + 3x + 1]

 1
2x + 4, 0 < x < 2

 1 20, 1
 2x + 4 , 0 < x < ≤ x <1
2  2
 
 1 4
\ g(f(x)) = 4[2x + 4], ≤ x < 1 = 24, 1≤ x <
 2  3
 4[3x + 3], 1 ≤ x < 2 28, 4 5
≤x<
  3 3
 5
32, ≤x<2
 3
Connector 20: The parametric form of representation of a curve is given by x = 2t - t3, y = 1 + 3t. Find the equation of the
curve as a relation between x and y.
y −1
Solution: y = 1 + 3t ⇒ =t
3
3
2(y − 1)  y − 1  2y 2 1 3
x = 2t - t3 ⇒ x = −  = − −  y − 3y(y − 1) − 1
3  3  3 3 27 
2y 2 1 3 1 2 1 1
= − − y + y − y+
3 3 27 9 9 27
⇒ 27x = 18y - 18 - y3 + 3y2 - 3y + 1
⇒ y3 - 3y2 - 15y + 27x + 17 = 0
Connector 21: Sketch the curve y
{ y } = sin x, x ∈ R
where, { x } denotes the fractional part
x
of x −2π 0 2π 4π
Solution:
Graph of y = sinx
y
Since {y} is always lying [0, 1), in the
graph of {y} = sin x is as shown below.
No part of the curve {y} = sinx lies in (p, 2p), (3p,
4p), (5p, 7p), …….
−2π −π 0 π 2π 3π 4π x
as well as (-p, 0), (-3p, -2p), …..
Connector 22: Sketch the curve
y = {ex}
where, {ex} denotes the fractional part of ex. y
Solution:
Since y = {ex},
1
between x = 0 and x = log 2, ex lies between 1 and 2
0 x
between x = log2 and x = log3, ex lies between 2 and 3
Hence, the graph of y = {e } will be as shown below.
x
Graph of y = ex
Functions and Graphs  1.33

0 log2 log3 log4 x

Breaks are there in the graph at x = 0, log2, log 3, log 4, ……….


Connector 23: Sketch the graph of
y = |2 - |x - 2||
Solution: Stage 1 Stage 2
Graph of y = |x| Graph of y = |x - 2|
y y

0 x 0 2 x

Stage 3 Stage 4
Graph of y = - |x - 2| Graph of y = 2 - |x - 2| = - |x - 2| + 2

y y
2
0 2 x

4 x
0 2

Stage 5 y
Graph of y = |2 - | x - 2||
OR
2
y = |2 - |x - 2||
when x < 2, y = |2 - (2 - x)| = |x|
O 2 4 x
when x > 2, y = |2 - (x - 2)| ± |4 - x|
when x = 2, y = |2 - 0| = 2
The graph of the function is as shown above
Connector 24: Parametric form of representation of the equation of a curve is given as
1 1
x = (t + 1) , y = (t − 1) . Represent the equation of the curve in the form f(x, y) = 0.
2 2

4 4
Solution: We have to eliminate t from the relations
1
(t + 1)
2
x=
4
1.34  Functions and Graphs

1
(t − 1)
2
y=
4
4x - 4y = (t +1)2 - (t - 1)2 = 4t
t=x-y
Hence the equation of the curve is
4y = (x - y - 1)2 = x2 + y2 + 1 - 2xy - 2x + 2y
OR
x2 + y2 - 2xy - 2x - 2y + 1 = 0
t − t2 t2 − t3
Connector 25: Parametric form of representation of the equation of a curve is given as x = and y = . Rep-
resent the equation of the curve in the form f(x, y) = 0. 1 + t2 1 + t2
Solution: We have
t (1 − t ) t 2 (1 − t )
x= ; y =
(
1 + t2 ) 1 + t2

y
⇒ =t
x
y y
1+ 
x  x y 1 x2 y (x + y )
⇒x= = ( x + y ) × × = 2
y 
2
1+  2 
x 2
x x +y 2
x + y2 ( )
x 
or y(x + y) = x(x2 + y2)
x
Connector 26: Find the domain of f(x) = sec −1 where [x] represents the greatest integer function.
x − [x]
x
Solution: We note that for integer values of x (positive or negative), x - [x] = 0 consequently, is not defined
for integer values of x. x − [x]
x
When x lies between -1 and +1, < 1 . Which means that the function is not defined for x lying
x − [x]
between -1 and +1. Therefore, the domain of f is given by |x| ≥ 1 excluding integer values of x.
p 
Connector 27: Find the range of the function f(x) = sin  [x 2 ] where, [x2] represents the greatest integer function.
2 
Solution: When 0 < x < 1, [x2] = 0, and therefore, f(x) = 0. When x > 1, [x2] will be natural numbers. In this case, f(x)
= - 1, 0 or 1.
\ Range of f(x) is {- 1, 0, 1}.
Functions and Graphs  1.35

TOPI C GR I p

Subjective Questions

1. Find the Domain of the following functions.


(i) f(x) = log log x

(ii) f(x) = sin x − 1


x+2
(iii) f(x) = + sin− 1x.
log10 (1 − x)

4 − x2
(iv) f(x) = , where, [x] is the greatest integer function.
[x]+ 2
 3 
(v) f(x) = sin−1  4 + 2sin x 

( )
(vi) f(x) = 1 − log 4 x 2 + 6x − 5 + 1 − x 2 .

(vii) f(x) = [x]2 − 4[x] + 3 where, [x] represents the greatest integer function.

(viii) f(x) = sin {log 2 log 3 x}

 4 
( )
(ix) f ( x ) = log10 x 2 − 1 + sin 
 x + 2 
(
− log e 1 + x 2 )
2. Find the range of the following functions.
ex + e− x
(i) f(x) =
2
(ii) f(x) = − x 2 − 6x − 5
−p p
(iii) f(x) = cos[x], < x < where, [x] is this greatest integer function.
2 2
(iv) f(x) = cos2x + sin2x
(v) f(x) + g(x) where, f(x) = x + 5, x ≤ 0 and g(x) = 5, x ≥ 0
3. If f(x) = – 1 + | x – 1|, – 1 ≤ x ≤ 3, g (x) = 2 – | x + 1|, –2 ≤ x ≤ 2. Draw their graphs, find g o f(x) and f o g(x)
1 1
4. If f(x) = ; g(x) = f (f(x)) ; and h(x) = f ( f (f(x))) , find .
2+x f(x) g(x) h(x)
5. f(x) satisfies the relation, f (x) + f (y) = f (x + y). Show that f(x) is an odd function. Also find f(10) if f(1) = 2
 x + 1
6. If f(x) is an even function defined in the interval (– 5, 5) find 4 real values of x satisfying the equation f(x) = f 
 x + 2 
7. Plot the graphs of the following functions and write its range
(i) f(x) = x2 − 1.
(ii) f(x) = |x2 − 1|, where x ∈ R
1.36  Functions and Graphs

8. Sketch the following curves. From the sketches drawn explain the procedures for getting the graphs of the curves
y = f (x + k), y + k = f(x) and y + h = f (x + k) from the graph of the curve y = f(x).
(i) y = | x |, y = | x – 1 |, y + 3 = | x |, y – 2 = | x – 3 |
(ii) y = x + 2 – [x + 2], y – 1 = x – [x], where, [ ] represents the greatest integer function.
9. Find the periods of the following functions.
(i) y = 4 cos (2x + 1)
(ii) y = 2 sin x + cos 3x
(iii) y = sin 4x + 3 cos (x/2)
(iv) y = e x −[x] , where, [x] represents the greatest integer function.
(v) y = | sin x |
 x
( )
10. If f(x) = cos (p[x]) + cos [p2 ]x + sin [− p2 ]  where, [x] represents the greatest integer function, find the values of
 3
p p
(i) f   (ii) f (− p) (iii) f   (iv) f(p)
2 4

Straight Objective Type Questions

Directions: This section contains multiple choice questions. Each question has 4 choices (a), (b), (c) and (d), out of
which ONLY ONE is correct.
2x
11. Domain of the function f(x) = is
1+ x
(a) (−∞, ∞) (b) (−∞, −1) ∪ [0,∞) (c) (−1, 1) (d) (−1, ∞)
12. The graph of f(x) = 2(x − 1)2+ 3, f : R → R is symmetric about the
(a) line x = 1 (b) line y = x (c) y-axis (d) line y = −x
3
13. If f(x) = , g(x) = f f(x) and h(x) = f f f(x), then the value of f(x) × g(x) × h(x) is
1+ x
27 27 3 3
(a) (b) (c) (d)
4x + 7 (1 + x)3 (1 + x) (x + 4) (1 + x)2
1
14. Let f(x) = the domain of f is the set of all real numbers. Then the set of possible values of p is
2
x − px + 4
(a) (−4, 4) (b) p can take any real value except −4 and 4
(c) (−∞, −4) ∪ (4, ∞) (d) {4, −4}
15. Which of the following functions can be represented by the given graph? Y
(a) f(x) = max {|x|, x2}
(b) f(x) = min {|x|, x2}
(c) f(x) = |x| + (x −1)2 (− 1, −1) (1, 1)

 x 2 , − 1 ≤ x ≤ 1
(d) f(x) =  O X
 x , x ∉[−1, 1]
Functions and Graphs  1.37

Assertion–Reason Type Questions

Directions: Each question contains Statement-1 and Statement-2 and has the following choices (a), (b), (c) and (d), out
of which ONLY ONE is correct.
(a) Statement-1 is True, Statement-2 is True; Statement-2 is a correct explanation for Statement-1
(b) Statement-1 is True, Statement-2 is True; Statement-2 is NOT a correct explanation for Statement-1
(c) Statement-1 is True, Statement-2 is False
(d) Statement-1 is False, Statement-2 is True
16. Statement 1
A relation R is defined on the set of real numbers.
x R y if x − y is positive. Then, R is neither reflexive nor symmetric.
and
Statement 2
A relation R on a set A is reflexive if (x, x) ∈ R for all x ∈ A and R is symmetric if (x, y) ∈ R implies (y, x) ∈ R.
17. Statement 1
x 2 − 5x − 9
f(x) = 2 , x ∈ R is not a one one function.
3x + 2x + 7
and
Statement 2
f(x) is not one one, if for any x1, x2 ∈ domain of f(x) where x1 ≠ x2, then f(x1) = f(x2).
18. Statement 1
Let f : R → [2, 4] where f(x) = 3 + cos2x. Then, f(x) is not bijective.
and
Statement 2
Let I1 and I2 denote intervals ∈ R and the domain of f(x) be I1. If f(x) is one one and the range of f(x) be I2, f(x) is a
bijective function.
19. Statement 1
2p
Period of f(x) = sin3x cos[3x] − cos3x sin[3x] where [ ] denotes the greatest integer function, is .
3
and
Statement 2
Period of {x} where { } denotes the fractional part of x, is 1.
20. Statement 1
If f(x) = sin− 1 (log x), then f(1) = 0.
and
Statement 2 O 1
graph of f(x) = logax, 0 < a < 1 is.
1.38  Functions and Graphs

Linked Comprehension Type Questions

Directions: This section contains 2 paragraphs. Based upon the paragraph, 3 multiple choice questions have to be
answered. Each question has 4 choices (a), (b), (c) and (d), out of which ONLY ONE is correct.
Passage I
A hostel of a school has designed a menu table to be followed throughout the year. This is to be followed on all days from
Sunday to Saturday of the week. They have the menu 0 to 6 to be followed on Sunday to Saturday in that order. A particular
year, which is not a leap year, has started on a Sunday. Denote the number of the day from the beginning of the year as t; w
is the number of weeks from the beginning of the year and x is the remainder when t is divided by 7.
21. The equation to find out the correct table on any day of the year has the graph, which is
(a) a straight line (b) periodic
(c) a parabola (d) a set of discrete points
22. The functional equation is
(a) t = 7w + x (b) x = 7w + t
(c) t2 = 7 x (d) (t1,x1), (t2,x2)……
23. What is the first day of July on which Sunday table is to be followed?
(a) 8 July (b) 1 July (c) 15 July (d) 2 July
Passage II
y
0 , x < a
The unit step function u(x − a) is defined as u(x − a) = 
1 , x ≥ a 1
The graph of y = u(x − a) is as shown below:
Domain of u(x − a) is R and its range is {0, 1}.
0 a x
Answer the following questions.

24. Let f(x) = [x] 0 ≤ x < 3


Where [ ] denotes the greatest integer function. The representation of f(x) in terms of unit step function is
(a) f(x) = u(x) + u(x − 1) +u(x − 2) + (x − 3), 0 ≤ x ≤ 3 (b) f(x) = u(x − 1) + u(x − 2) + u(x − 3), 0 ≤ x < 3
(c) f(x) = u(x − 1) − u(x − 2) + u (x − 3), 0 ≤ x < 3 (d) f(x) = u (x − 1) + 2u(x − 2) + 3u (x − 3), 0 ≤ x < 3

25. Graph of y = f(x), 0 ≤ x < 3 is shown below


2
Representation of f(x) in terms of the unit step function is given by
(a) f(x) = x{u(x) − u(x − 1)} − 2u (x − 2) 1
(b) f(x) = x{u(x) − u(x − 1)} − u(x − 2) + 2u(x − 3), 0 ≤ x < 3
(c) f(x) = x {u(x) − u(x − 1)} − u(x − 1) + 3u(x − 2), 0 ≤ x < 3 0 1 2 3 x
(d) f(x) = xu(x) − u(x − 1) + 2u(x − 2), 0 ≤ x < 3
−1
26. Representation of the function
 x3 , 0≤ x <1

f(x) =  x − 1, 1≤ x <3
0, x≥3

interms of the unit step function is
(a) x3[u(x) − u(x − 1)] + (x − 1) [u(x − 1) − u(x − 3)] (b) x3u(x) + (x − 1) u(x − 1)
(c) x3[u(x) − u (x − 1)] + (x − 1) u(x − 2) + u(x − 3) (d) x3u(x) + (x − 1) [u(x) − u(x − 1)] + u(x − 2)
Functions and Graphs  1.39

Multiple Correct Objective Type Questions


Directions: Each question in this section has four suggested answers of which ONE OR MORE answers will be correct.
1
27. Given f(x2 − 1) = x4 − 5x2 + 6 and g(x) =
x
1
(a) Domain of (g  f)x is R − {0} and (g  f) (1) =
2
1
(b) Domain of (g  f)x is R − {1, 2} and (g  f) (0) =
2
(c) Domain of (f  g)x is R − {0} and (f  g) (1) = 0
(d) Domain of (f  g)x is R − {1, 2} and (f  g) (0) = 0
10 x − 1
28. If f(x) = , g(x) = sinx and h(x) = cosx
10 x + 1
(a) h(x) . (f  g)x is an odd function (b) g(x) . (f  h)x is an odd function
(c) f(x) . (h  g)x is an odd function (d) f(x) . h(x) . g(x) is an odd function
29. If f(x + 2) + f(x − 2) = f(x) and f(0) = 0, then
(a) f(x) is a periodic function with period 6 (b) f(x) is periodic function with period 12
12 6
(c) ∑ f(r) = 0 (d) ∑ f(2r) = 0
r =0 r =0

Matrix-Match Type Questions


Directions: Match the elements of Column I to elements of Column II. There can be single or multiple matches.
30.
Column I Function Column II Graph

(a) y = −e−x (p)

1
(b) y = e −|x| − (q)
3

(c) |y| = sinx (r)

(d) |y| = |log |x|| (s)


1.40  Functions and Graphs

I IT AS S I GNMENT E XER C I SE

Straight Objective Type Questions


Directions: This section contains multiple choice questions. Each question has 4 choices (a), (b), (c) and (d), out of
which ONLY ONE is correct.
31. Domain of the function f(x) = −(x 2 − 1) is
(a) [−1, 1] (b) {x ∈ R/x ≤ 0} (c) null set (d) {x ∈ R/x ≥ 0}
2
x − x +1
32. Domain of the function f (x) = is
x2 + x + 1
(a) R (b) (0, ∞) (c) R – {1} (d) {−1, 1}
x+3
33. The range of the function f(x) = , x ≠ −3 is
x+3
(a) {3, −3} (b) R − {−3} (c) all positive integers (d) {−1, 1}
1
34. Range of the function y = x + , x > 0, is
x
(a) R (b) (−∞, ∞) (c) [2, ∞) (d) [1, ∞)
35. f : x → y is a given function. Then f exists if
-1

(a) f is one-one (b) f is onto


(c) f is one-one but not onto (d) f is one-one and onto
2x − 1
36. If f (x) = , then f –1 (x) =
2x + 1
1+ x 2x + 1 1 + 2x 2+x
(a) (b) (c) (d)
2 (1 − x ) 2x − 1 1 − 2x 2−x
37. If f(x) = 2x2 – 2x + 4 and f (2α) = 4f (α), then α is equal to
(a) 4 (b) 3 (c) 0 (d) −2
x −1
38. If f(x) = , then f(2x) in terms of f(x) is
x +1
f (3x ) + 1 3f ( x ) + 1 2f ( x ) + 1 2f ( x ) +1
(a) (b) (c) (d)
3f ( x ) − 3 f (x ) + 3 2f ( x ) − 1 3f ( x ) + 2

39. If f(x + 2) = (x + 3)2 − 2x, then f(x) =


(a) x2 − 2 (b) x2 + 5 (c) x2 + 4x + 9 (d) (x + 5)2 −2(x + 2)
40. Let f(x) = 5x2 + 3x – 4, x ∈ R. Then f(x) is a/an
(a) periodic function (b) odd function
(c) even function (d) neither even nor odd
41. Which of the following is an odd function?
ex + e− x ex − e− x ex − e− x ex − e− x
(a) f1(x) = (b) f2(x) = (c) f3(x) = (d) f4(x) =
2 2e x 2e − x 2
Functions and Graphs  1.41

42. If f (x) = 5x – 7, then f-1 (x) is


1 5x x+7 x+5
(a) (b) (c) (d)
5x − 7 7 5 7
43. Which among the functions is inverse of itself?
2
+2 1 + x2 1− x
(a) y = a 2 log x (b) y = 5x (c) y = (d) y =
1 − x2 1+ x
44. If log 4  log 3 [ log 2 x ]  = 1 , then x is
4 2
(a) 23 (b) 9 (c) 24 (d) 43
 1
 3 | x | +5, x≤
2

 1
45. If : R → R is defined by f(x) = log x , < x ≤ 2.
 2 2
 x + 4, x<2


 1 1 2 3


The value of f  −  + f   + f   + f   − f (3) is
 3 3 3 2
(a) 0 (b) −1 (c) 19 (d) 1+ log 2
46. If f = {(1, 3), (2, 5), (3, 7)} and g = {(3, 10), (4, 13), (5, 16), (6, 19), (7, 22)}, then g  f is
(a) {(1, 10), (2, 16), (3, 22)} (b) {(3, 3), (5, 5), (7, 7)}
(c) {(1, 10), (2, 13), (3, 5)} (d) {(1, 16), (2, 10), (3, 22)}
 7
47. If f(x) = [x] where, [ ] represents the greatest integer function and g(x) = |x|, then the value of (g  f)  −  − (f  g)
 3
 7
 − 3  is
(a) 0 (b) 2 (c) 1 (d) –1
48. If f(x) = x3 and g(x) = 3x are two real valued functions, then the number of solutions of g o f(x) = f o g(x) is
(a) 0 (b) 1 (c) 2 (d) 3
1, if x is rational
49. Let f : R → R be defined by f(x) =  then f f f(x) is
0, if x is irrational

(a) x (b) f(x) (c) 0 (d) 1


50. The function f : R → R defined by f(x) = (x2 − 1) (x2 − 2) is
(a) bijective (b) not injective
(c) injective but not surjective (d) not injective or surjective
6x − 3
51. If f(x) = , then f –1(x) is
2x + 4
2x + 4 6x − 4 4x + 3
(a) (b) (c) (d) Does not exist
6x − 3 2x + 3 6 − 2x
x
52. The inverse of the function f(x) = 22 is
1
2
(a) log2 (log2 (x)) (b) log2 x (c) [log 2 (x)]2 (d) 2 x
1.42  Functions and Graphs

53. Let f and g be two bijective functions defined on a set A, such that f(x) = 2x + 1 and (g of ) (x) = 3x + 2. Then g(x) is
3 5 3 1 3
(a) x− (b) x+ (c) 2x − (d) 5x + 3
2 2 2 2 2
54. The range of the function f(x) = |x − 1| + |x − 2| + |x + 1| + |x + 2| where, x ∈ [−2, 2] is
(a) [6, 8] (b) [2, 4] (c) [0, 4] (d) {1, 2}
1
55. Two real valued functions f and g are given by f(x) = and g(x) = x3. The domain of f  g is
x − 27
(a) R − {27} (b) R − {3} (c) R (d) R − {9}
56. f(x) = 2 , then
x

(a) f(x + y) = f(x) (1+ f(y)) (b) f(2x) = 2 f(x)


(c) f(xy) = f(x) f(y) (d) f(x) + f(y) = f(x) + f(x)f(y − x)
p
57. f(x) = sin[x] + [sin x], 0 < x <
, where [ ] represents the greatest integer function can also be represented as
2
 1 p
0 ,0<x<1  , 0< x <
  2 4
(a)  p (b) 
1+ sin1 , 1 ≤ x < 2 1 + 1 + 1 + 3 , p ≤ x < p
 2 2 2 4 2
 p
0 , 0<x<
4
0 , 0<x<1 
  p
(c)  p (d)  1 , <x<1
 sin1 , 1 ≤ x < 2  4
sin1 , 1 ≤ x < p
 2
58. If f(x) = ax + b and f f(x) = x, then
(i) a = 1, b = 0 (ii) a = −1, b = 0 (iii) a = −1, b = 1 (iv) a = −1, b = 2
(a) (i) and (ii) are true (b) only (i) is true (c) (i), (ii), (iii) are true (d) all the four are true
3x 2 + 6x + 14
59. If f is a function defined on the set of rational numbers such that f(x) = , then f−1(2) is equal to
x 2 − 3x − 3
 1 1  1
(a) {0, 1} (b) − ,  (c) {−10, −2} (d) −2, − 
 2 3  7
60. The graph of y = (x + 1) − 1 is
2

Y Y

(a) (b)
X X
O O

Y Y

(c) (d)
X X
O O
Functions and Graphs  1.43

a 1 b 1
61. A curve has parametric equation x =  t +  , y =  t −  . The equation of the curve is
2  t  2  t

x2 y 2 x y x y x2 y 2
(a) + = 1 (b) + = − (c) − = 1 (d) ax + by = bx + ay
a 2 b2 a b a b a 2 b2

n(n + 1)x
62. Period of the function f(x) = [x] + [2x] + [3x] + [4x] + … + [nx] − ,
2
where, n ∈ N and [ ] denotes the greatest integer function, is
1
(a) 1 (b) n (c) (d) 2n
n
 1 1
63. A function f satisfies f  x +  = x 3 + 3 . Then (f o f )( x ) is
 x x
(a) x3 – 3x (b) x3 + 3x
(c) x9 – 9x7 + 27x5 – 30x3 + 9x (d) x3 − 3x2 + x − 1

64. The domain of the function f(x) = 10 − x 4 − 21x 2 is


(a) [5, ∞) (b)  − 21, 21 

(c)  − 5, − 21  ∪  21, 5 ∪ {0} (d) (−∞, −5]

2x + 3
65. If f(x) = , then f o f (x) is
x−2
2x + 3 x 3x + 2
(a) (b) (c) (d) x
3x + 2 (x − 2) 2−x
1
66. The range of the function f(x) = is
2 + cos x
1   1
(a)  , 1 (b) [0, 1] (c) [−1, 1] (d) 0, 
3   3
67. Let A = {1, 2, 3} and let R, S be relations on A defined as R = {(1, 1), (1, 2), (1, 3), (2, 2), (3, 3)},
S = {(1, 1), (1, 2), (1, 3), (2, 2), (3, 2), (3, 3)}. Consider the statements
P: R ⊆ S
Q : R −1 ⊆ S −1
(a) both P and Q are true (b) only P is true
(c) only Q is true (d) both P and Q are false
cos (sinnx )
68. If the period of f(x) = ,n ∈ N is 6π, then n is equal to
x
tan  
n
(a) 2 (b) 3 (c) 6 (d) 1
69. The function f(x) satisfying f(x + y) = f(x) – f(y) is
(a) an odd function (b) an even function
(c) neither an odd nor an even function (d) an even periodic function
70. If f(x) = x + x3 + cos x, then f(x) + f(– x) is
(a) an odd function (b) an even function
(c) neither an odd nor an even function (d) an even periodic function
1.44  Functions and Graphs

1
71. The domain of the function f(x) =
x − [x]
(a) N (b) (0, ∞) (c) R − {0,±1, ±2, ±3,..} (d) R−N

( )
1
72. The inverse of f(x) = 5 − ( x − 8)
5
3
is

(a) 5 − ( x − 8)
5
(b) 8 + 5 − x 3 ( )
1/5
(
(c) 8 − 5 − x 3 )
1/5
(
(d) 5 − ( x − 8) )
1/5 3

73. If f(x − 2) = x2 − 5x + 11, f(x) + f(−x) is


(a) 2x2 + 11 (b) 2(x2 + 7) (c) 2(x2 + 11) (d) (2x2 + 10)
74. The domain of f(x) = log 2 log 1 (5x − 1) is
3

1 2 1 2   1 2 
(a)  ,  (b)  ,  (c)  −∞,  (d)  , ∞ 
5 5 5 5   5 5 

 x + 2, x < 0  x + 1, x<0
75. If f(x) =  and g(x) =  then (g o f ) ( x ) is
( x − 1) + 4, x ≥ 0
2
| x − 3 |, x ≥ 0

 x + 3, x < −2
 2  x − 3, x < −2
 x + 2x + 5, − 2 ≤ x < 0  2
(a)  2 (b)  x − 2x + 5, − 2 ≤ x < 0
 x − 4x + 8, 0 ≤ x < 3  x 2 − 8x + 20, x ≥ 0
 x 2 − 8x + 20, x ≥ 3 

 x 2 + 2x + 5, x < 0
(c)  2 (d) not defined
 x − 4x + 1, x ≥ 0
1
76. Given f(x) = , g(x) = f o f (x) and h(x) = f o f o f(x), f(x) g(x) h(x) is given by
3−x
x 3−x 1
(a) (b) (c) (d) x
(3 − x) (21 − 8x) 21 − 8x
77. Let A and B be two sets such that n(A × A, B × A) = 384 and n(A × B, B × B) = 864. Then n (A) and n (B) are respectively
(a) 2, 10 (b) 4, 6 (c) 12, 24 (d) 4, 24
78. Let A = {x/x ∈ N, x ≤ 5} and B = {x/x is an odd positive integer less than 10}. Let R1 = {(x, y)/x ∈A, y ∈ B, y = x + 3},
R2 = {(1, 1), (1, 2), (1, 3)}
(a) both R1 and R2 are relations from A into B
(b) R1 is a relation from A into B but R2 is not a relation from A into B
(c) R2 is a relation from A into B but R1 is not a relation from A into B
(d) both R1 and R2 are not relations from A into B
79. Domain of the function sin−1 [2x − 3] where [ ] denotes the greatest integer function is
 3  5  −1 
(a) [−1, 1] (b) 1,  (c) 1,  (d)  , 1
 2  2 2 
80. Domain of the function log4 log2 log3 (x2 + 4x − 23) is
(a) (−8, 4) (b) (−∞, −8) ∪ (4, ∞) (c) (−4, 8) (d) (−∞, −4) ∪ (8, ∞)
81. If f : A → B is a bijection, then
(a) n (A) = n (B) (b) n (A) > n (B) (c) n (A) < n (B) (d) n (A) = n (b) + 1
Functions and Graphs  1.45

82. The total number of distinct functions that can be formed from A to A, where A is a set with m elements, is
(a) 2m (b) 2m (c) mm (d) m2
83. Which among the following relations is a function?
x2 y 2
(a) x2 + y2 = r2 (b) 2 + 2 = r 2 (c) y2 = 4ax (d) x2 = 4ay
a b
84. Let A = { x/f(x) = 0} and B = {x/g(x) = 0}. Then A ∪ B =
(a) {x/f o g(x) = 0} (b) {x/g o f(x) = 0} (c) {x/f(x) +g(x) = 0} (d) {x/f(x). g(x) = 0}
x+2
85. If f (x) = ; then f (x) is
x−3
(a) even function (b) odd function
(c) neither even function nor odd function (d) periodic function
86. A function f : R → R is defined as f(x) = 3x2 + 1. Then f−1(x) is
y −1 1 y −1
(a) (b) x − 1 (c) f−1 does not exist (d)
3 3 3
87. f : R → R is defined by f(x) = x2 − 2x − 17. The number of elements in the domain which are mapped to −2 by the
function f is
(a) 0 (b) 1 (c) 2 (d) 3
88. The range of the function y = sin x is
 p
(a) 0,  (b) [−1, 1] (c) [0, 1] (d) [−2π, 2π]
 2
89. The parametric equation of a curve is given by x = 2 sint − 2, y = 2 cost + 1. Which of the following equation represents
the given curve?
(a) x2 + y2 = 9 (b) x2 + y2 + 4x − 2y + 1 = 0
(c) x2 +y2 + 4x = 4 (d) (x + 2)2 − (y − 1)2 = 4
 1 1
90. If f  x +  = x 2 + 2 ; then f (x) is
 x x
 1  1
(a)  x +  (b) x2 – 2 (c) x2 + 2 (d)  x − 
 x  x
x +1
91. If f (x) = , then f  f (f ( 1 ))  is
x  x 
1 3 + 2x 1 3 − 2x
(a) 1 − (b) (c) 1 − (d)
x 2+x x2 2+x
 1
 − x + 2
,−1 ≤ x ≤ 0
92. A function f is defined on [−1, 1] as f(x) =  . The value of |f(x)| + f( |x| ) is
x + 1 ,0 < x < 1
 2
 1 1
x − 2 , − 1 ≤ x < 2
 1
 1 −1  ,−1 ≤ x ≤ 0
(a) f(x) (b) − x + , ≤ x ≤ 0 (c) 2f(x) (d)  4
 2 2 2x + 1, 0 < x ≤ 1
x + 1 , 0 < x < 1
 2
1.46  Functions and Graphs

93. If : N → N is defined by f(n) = n − (−1)n, then


(a) f is one- one but not onto (b) f is both one-one and onto
(c) f is neither one-one nor onto (d) f is onto but not one-one
2n − 1 if n is even
94. If f and g are two functions defined on N, such that f(n) = 
and g(n) = f(n) + f(n +1). Then range of g is 2n + 2 if n is odd
(a) { m∈ N/m = multiple of 4}
(b) { set of even natural numbers}
(c) { m∈ N/m = 4k + 1, k is a natural number}
(d) { m∈ N/m = multiple of 3 or multiple of 4}
95. If f(x) = x(x − 1) is a function from R to R, then {x ∈ R/f−1 (x) = f(x)} is
(a) null set (b) {1}
(c) {0, 2} (d) a set containing 3 elements.
f (x )
96. If Df and Dg denote the domains of the functions f(x) and g(x) respectively, then the domain of the function is
g (x )
(a) Df ∪ Dg − {x ∈ R : g (x) = 0} (b) Df ∩ Dg
(c) Df ∪ Dg (d) Df ∩ Dg – {x ∈ Dg : g (x) = 0}
97. Domain of the function f(x) = 5 | x | − x 2 − 6 is
(a) (−∞, 2) ∪ (3, ∞) (b) [−3, −2] ∪ [2, 3] (c) (−∞, −2) ∪ (2, 3) (d) R − { −3, −2, 2, 3}

2x − 2− x
98. Range of the function y = is
2x + 2− x
(a) R (b) (−1, 1) (c) [−1, 1] (d) (0, 1)
99. Let R be a reflexive relation on a set A. Then
(a) R-1 is reflexive (b) R-1 is not reflexive (c) RC is reflexive (d) None of the above
2 + x, x ≥ 0
100. If f(x) =  , then f(f(x)) is given by
 2 − x, x < 0
4 + x, x ≥ 0 4 + x, x ≥ 0
(a) f(f(x)) =  (b) f(f(x)) = 
4 − x, x < 0  x , x <0

4 − x, x ≥ 0 4 + 2x, x ≥ 0
(c) f(f(x)) =  (d) f(f(x)) = 
 x, x < 0 4 − 2x, x < 0

101. If f(x) = x and g(x) = 2 − x , the composition f o g(x) is


1 1
(a) (2 − 2x) (b) (2 − x) 4 (c) x 4 (d) 2− x

1
102. The domain of f(x) = log is
| cos x |

(a) R (b) R – [ –π, π]


 (2n + 1) p 
(c) R −  x / x = ,n ∈ Z  (d) R – [ –π/2, π/2]
 2 
Functions and Graphs  1.47

103. Let R be a relation from A into B and let x ∈ A. We define the R relative of x, denoted by R(x), as R(x) = {y∈B/x R y}.
If A1 ⊆ A, then the R relative set of A1 denoted by R(A1) is defined as R(A1) = {y∈B/x R y for some x ∈ A1}. If A1 ⊆ A2
⊆ A and R is a relation from A into B, then
(a) R(A1) ⊆ R(A2) (b) R(A2) ⊆ R(A1)
(c) R(A1) = R(A2) (d) R(A1) and R(A2) are not comparable
104. Let R be a relation from A to B. Let A1 and A2 be subsets of A. Let R(x) denote the R-relative of a set X. Then
(a) R(A1 ∪ A2) ⊆ R(A1) ∪R( A2) (b) R(A1 ∪ A2) ⊇ R(A1) ∪R( A2)
(c) R(A1 ∪ A2) = R(A1) ∪R( A2) (d) none of the these
105. Let R be a relation from A to B and S, T be relations from B to C. Then
(a) (S ∪ T) o R ⊆ (S o R ) ∪ (T o R ) (b) (S ∪ T) o R ⊇ (S o R ) ∪ (T o R )

(c) (S ∪ T) o R = (S o R ) ∪ (T o R ) (d) none of the above

3x 2 + 3x − 4
106. The function f : R → R defined as f(x) = is
3 + 3x − 4x 2
(a) one to one but not onto (b) onto but not one to one
(c) both one to one and onto (d) neither one to one nor onto
107. The domain of f(x) = log|x| 8 is
(a) R – {0} (b) R – {– 1, 0, 1} (c) R − {– 1, 1} (d) (0, ∞)

108. If (0.5)
log 3 log 1
5
( x2 − 4 5 ) ≥ 1 then | x | belongs to

 2   2 
(a) [1, ∞) (b)  ,1 (c)  , ∞  (d) [0, 1]
 5   5 

109. The number of solution of the equation ex − log x = 0 is


(a) 0 (b) 1 (c) 2 (d) 3

110. The set of all numbers x satisfying the inequality x3 (x + 1) (x – 2) ≥ 0 is


(a) 0 ≤ x < ∞ (b) 2 ≤ x < ∞ (c) –1 ≤ x < ∞ (d) None of the above

Assertion–Reason Type Questions

Directions: Each question contains Statement-1 and Statement-2 and has the following choices (a), (b), (c) and (d), out
of which ONLY ONE is correct.
(a) Statement-1 is True, Statement-2 is True; Statement-2 is a correct explanation for Statement-1
(b) Statement-1 is True, Statement-2 is True; Statement-2 is NOT a correct explanation for Statement-1
(c) Statement-1 is True, Statement-2 is False
(d) Statement-1 is False, Statement-2 is True
111. Statement 1
p
f(x) = |sinx| + |cosx| + 5 is periodic with period
2
and
Statement 2
If the function f(x) = cosλx + sinλx is periodic then λ must be rational.
1.48  Functions and Graphs

112. Statement 1
The equation logex = e−x has a root between 1 and 2.
and
Statement 2
If f(x) is continuous in an interval I and a, b ∈ I such that f(a)f (b) < 0 then, there exists a number c where, a < c < b
such that f (c) = 0.
113. Statement 1
 x 2 − x + 1 , x > 0
f(x) =  2 is an odd function.
− x − x − 1 , x < 0
and
Statement 2
If f(x) is continuous and an odd function, f(0) = 0.

Linked Comprehension Type Questions


Directions: This section contains 1 paragraph. Based upon the paragraph, 3 multiple choice questions have to be
answered. Each question has 4 choices (a), (b), (c) and (d), out of which ONLY ONE is correct.
A cellular phone service provider charges Rs 20/- for the first 30 minutes service and Rs 0.75 per minute thereafter, minutes
being rounded off to the nearest higher integer
114. If c(t) is the charge for t minutes
 20 + 0.75t, t ≤ 30  20, t ≤ 30
(a) c(t) =  (b) c(t) = 
20 + 0.75(t − 30), t > 30 20 + 0.75t, t > 30
 20, t ≤ 30  20, t ≤ 30
(c) c(t) =  (d) c(t) = 
20 + 0.75(t − 30), t > 30 20 + 0.75(t + 30), t > 30
115. A customer begins the talk at 8 a.m and ends at 8 a.m 40 minutes 30 seconds. The charges he has to pay
(a) Rs 8.25 (b) Rs 30.75 (c) Rs 28.25 (d) Rs 11.75
116. The range and domain of the function is
(a) Range : (0, ∞) (b) Range : [20, ∞)
Domain : [20, ∞) Domain : (0, ∞)
(c) Range : (0, 20] (d) Range : [20, ∞)
Domain : (0, ∞) Domain : [20, ∞)

Multiple Correct Objective Type Questions


Directions: Each question in this section has four suggested answers of which ONE OR MORE answers will be correct.
 6p 
117. Given f(x) = cos   x + cos
[6p] x , where [.] denotes the greatest integer function. Then
5 5
2p 10 p
(a) Period of f(x) is (b) Period of f(x) is
3 3
(c) f(x) is an odd periodic function (d) f(x) is an even periodic function.
Functions and Graphs  1.49

118. f(x) = sin3xcos2x and g(x) = 2x + 3


(a) (f  g) x is not periodic (b) (g  f) x is not periodic
(c) (f  g) x is periodic with period π (d) (g  f)x is periodic with period 2π

{ }
1
119. If f(x) = a + 2a 3 − 3a 2 [ f(x − b) − c ] + 3a (f(x − b))2 + c2  − (f(x − b))3 − c3 
3
, then

(a) f(a + b) = f(a − b) (b) f(a + 2b) = f(2a + b) (c) Period of f(x) is 2a (d) Period of f(x) is 2b

Matrix-Match Type Questions

Directions: Match the elements of Column I to elements of Column II. There can be single or multiple matches.
120.
Column I Column II
(a) f(x) = −ex−{x} + |sinπx| + |sin2πx| (p) odd function
(b) f(x) = x + sinx (q) periodic with period π
(c) f(x) = sin4x + cos12x (r) periodic with period 1
(d) f(x) = cos−1(cos2πx) (s) even function
1.50  Functions and Graphs

ADDIT IONAL PRA CT I C E E XERCIS E

Subjective Questions

121.
(i) Determine in each case, whether the curve drawn below is the graph of a function of x. If it is, state the domain
and range of that function.
Y Y

10
2

6 X
−3 O 3
−5 O X
−3.5
−2
−4

Fig (i) Fig (ii)

Y Y

X X
−3 O 5 −5 −4 O 1 4
−2

Fig (iii) Fig (iv)

(ii) Drawn below are the graphs of a few functions. Identify which one is an even function, which one is odd, which
one is periodic.

Y Y

4
2
O
−4 4 X X
−3 O 3 −6 −3 −1 1 3 6 9
−2 −2

Fig (i) Fig (ii)


Functions and Graphs  1.51

Y Y

3
1
X X
−2 −1 O 1 2 −8 −4 O 4 8
−1 −1

Fig (iii) Fig (iv)

(iii) Sketch the graph of the function f(x) = x + |x – 1| + |x – 2| + |x – 3| in the interval [– 5, 5]. What is the range
of f(x)?
2x, 0≤x≤2

(iv) Sketch the graph of f ( x ) = 4, 2 < x ≤ 4 and f(x + 6) = f(x)
12 − 2x, 4 < x ≤ 6

122. Find the domain of the following functions.


1− | x |
(i) f(x) = .
2− | x |

1
(ii) f(x) = where, [x] represents the greatest integer function.
x − [x]
1
(iii) f(x) = + 3x − 1 − 2x 2
2
x−x
1
(iv) where, [ ] denotes the greatest integer function
[| x − 9 |] + [| x − 1 |] −8

123. Find which of the following functions are even and which are odd and which are neither even nor odd.
(i) y = x + sin x
(ii) y = | x – 2 |
 x 2 , x > 0
(iii) y =  2
− x , x ≤ 0

−1 x < 0

(iv) y = 0 x = 0
1 x>0

(v) y = 2x + x2
(vi) y = x (3x)

 0, x ≤ −4
 4 + x, −4 < x < −1

(vii) y =  3, −1 ≤ x ≤ 1
 4 − x, 1 < x < 4

 0, x≥4
1.52  Functions and Graphs

124. Check whether the following relations are reflexive, symmetric, transitive
(i) x R y if x + y is a multiple of 3 where x, y ∈ set of positive integers.
(ii) x R y if y = 1 − x where, x, y ∈ set of real numbers.
(iii) x, y ∈ set of reals and x R y if x2 + 3x = y2 + 3y
125. Given the relation R = {(1, 3), (3, 2)} on the set of natural numbers, add minimum number of ordered pairs so that
the enlarged relation is an equivalence relation.
126. Sketch the graph of f(x) = {x} where, { } denotes the fractional part of x. Also, find the domain and range of f(x).
Prove that f(x) is periodic with period 1.
127. Sketch the graph of the function
 1 1

 4 2
( )
f(x) = log 1  x −  + log 4 16x 2 − 8x + 1 find its domain and range.
4

128. Represent geometrically, the region represented by the inequalities


x + 2y −1 ≥ 0
5x + 4y − 20 ≤ 0
4x − 5y + 20 ≥ 0 and x, y > 0
129. If a function f is such that f(0) = 2, f(1) = 3 and for any x
f(x + 2) = 2f(x) − f(x + 1).
Determine the value of f(5).
130. If g(x) = x2 + 2x − 5 and g  f(x) = 25x2 + 50x + 19, determine f(x).

Straight Objective Type Questions


Directions: This section contains multiple choice questions. Each question has 4 choices (a), (b), (c) and (d), out of
which ONLY ONE is correct.
131. Least value of the function f(x) = |x + 1| + |x + 2| + |x + 3| + |x + 4| is
(a) 6 (b) 4 (c) 2 (d) 1
132. Let f : [5, ∞) → [5, ∞) where f(x) = x2 − 7x + 15. Then, f is
(a) one one but not onto (b) not one one but onto
(c) one one and onto (d) not one one and not onto
x x
133. The range of the function f(x) = cos2+ sin , x ∈ R is
4 4
 5  5  5  5
(a) 0,  (b) 1,  (c)  −1,  (d)  −1, 
 4  4  4  4
x+3
134. Range of the function y = is
x−3
(a) [3, -3] (b) R (c) R – {1} (d) [3, ∞]
135. If the domain of the function f(x) = −x + 8x − 13 is R, then the maximum value of f is
2

(a) 4 (b) 3 (c) 13 (d) 8


136. The non-periodic function from among the following is
f1(x) = x – [x]; f2(x) = log (2 + cos 2x),
f3(x) = tan (3x + 2); f4(x) = x2 – x + tan x
(a) f1(x) (b) f2(x) (c) f3(x) (d) f4(x)
Functions and Graphs  1.53

137. If f(x) = k x, then f(x1 + x2) is equal to


(a) f(x1) + f(x2) (b) f(x1x2) (c) f(x1) − f(x2) (d) None of these
138. Let f(x) = c be a constant function from A to B. Then f is a bijection if and only if
(a) A = {a} and B is any set (b) A is any set and B = {b}
(c) A = {a} and B = {b} (d) A and B are nonempty sets

139. The domain of the function f(x) = log(x2 + x +1) + sin x − 1 is


(a) (−2, 1) (b) (−2, ∞) (c) [1, ∞) (d) None of these
140. The domain and range of the relation R defined as
R = {(x, y)/x, y ∈ N and 2x + y = 7} are respectively
(a) {1, 2, 3}, {1, 3, 5} (b) {0, 1, 2, 3}, {1, 3, 5, 7}
(c) {1, 2, 3}, {3, 5, 7} (d) {0, 1, 2, 3}, {1, 1, 3, 5}
141. Which of the following is not injective (one to one)?
x 2x 5x
(a) 9 − x 2 , x ∈ 0,3 (b) ,x ≠ 0 (c) , x ≠ −1 (d)
|x| 1+ x 2+ | x |

142. If f: A → B and g: B → C are onto functions then g o f is


(a) onto (b) one to one (c) bijection (d) not defined

143. If f: A → B and g: B → C are functions and if g o f : A → C is one to one then


(a) f is one to one (b) g is one to one (c) f is onto (d) g is onto
x
144. The equations of the lines parallel to the x–axis between which the graph of y = lies are
1 + x2
−1 1 −1 1 −1
(a) y = ,y = (b) y = , y = −1 (c) y = ,y = 1 (d) y = ,y = 1
2 2 2 2 2

145. The domain and range of the function f(x) = 4 − x + x − 2 are respectively

(a) [2, 4],[ 2, 2] (b) [2, 4],[ 2, 4] (c) [ 2, 2 2],[4, 4 2] (d) [ 2, 2],[2, 4]

146. Suppose y = f(x) = x − 1, x ≥ 0 . If g(x) is the reflection of y = f(x) about y = x, then g(x) is equal to

( x + 1)
2
(a) (b) x2 (c) x 2 + 1 (d) x 2 − 1
1
147. Domain of the function f(x) = log e is
sin x − 1
 p p
(a) nπ + (−1)nα where n is any integer and α ∈ 0,  (b) nπ + (−1)n , n = 1, 2, 3,…..
 2 2


 p
(c) 2nπ − α where α ∈  0,  , n any integer (d)
(2n + 1) p , n any integer
 2 2

3x + 5 3
148. The inverse of f(x) = ,x≠ is
8x − 3 8

3x + 5 3x + 5 8x + 3 8x − 3
(a) (b) (c) (d)
3 − 8x 8x − 3 3x + 5 3x + 5
1.54  Functions and Graphs

3x + 1 1
149. If f(x) = , then roots of the equation f(x) + f   = 0 are
3x − 1 x
−1 1
(a) x = −3 (b) x = (c) x = (d) both (a) or (b)
3 3

x + y x − y
150. For the real valued function f(x) satisfying the relation f(x) + f(y) = 2f  f , x, y ∈ R, then
 2   2 
(a) f(0) = 0 (b) f(0) = −1 (c) f(0) = 1 (d) (a) or (c)

f (x ) + f (y )
151. The real valued function f(x) satisfying the relation f(x + y) = , x, y ∈ R,
1 − f (x ) f (y )

(a) f(x) is an even function (b) f(x) is an odd function


(c) f(x) is neither even nor odd (d) f(0) = 1
1
152. If f(2 + x) = a + 1 − (f(x) − a ) 
4 4
for all x ∈ R, then f(x) is periodic with period
 
(a) 1 (b) 2 (c) 4 (d) 8
153. A linear function that map the set {−2, 2} onto the set {0, 4} is
(a) f(x) = (x − 2) (b) f(x) = (2 − x) (c) f(x) = (2 + x) (d) (b) and (c)
154. Let f(x) = x2 − 3x + 2, x ∈ [2, 10]. Then f−1(6) (where, f−1 denotes the inverse of f) is equal to
11
(a) 2, 5 (b) (c) 3 (d) 4
3
155. Let f(x) = ax + b, g(x) = cx + d where a, b, c, d are constants. The number of sets of values for a, b, c, d so that the
compositions f  g(x) = g  f(x) for x ∈ R is
(a) 2 sets of values (b) 4 sets of values
(c) 8 sets of values (d) infinite many sets of values
3+ x , 0 ≤ x < 3
156. A single formula that gives f(x) for all x ≥ 0 where, f(x) =  is
3x − 3 , x≥3
(a) f(x) = |2x − 1| + 4x (b) f(x) = |x − 3| + 2x (c) f(x) = |3x − 9| − x (d) f(x) = |x − 3| + 3x
157. Let f(x) = −1 + |x − 1|, −1 ≤ x ≤ 3; g(x) = 2 − |x + 1|, −2 ≤ x ≤ 2. Then

− x − 1 , −1 ≤ x < 0  x + 1 , −1 ≤ x < 0
(a) f  g(x) =  (b) f  g(x) = 
 x +1 , 0 < x ≤ 2 x − 1 , 0 ≤ x < 2

− x − 1 , −1 ≤ x < 0 1 + x , −2 < x < −1


(c) f  g(x) =  (d) f  g(x) = 
 x −1 , 0 ≤ x ≤ 2  x − 2 , −1 < x < 2
158. If a function f is such that f(0) = 2, f(1) = 3 and for any x f(x + 2) = 2f(x) − f(x + 1), f(5) equals
(a) 13 (b) −9 (c) −13 (d) 23
x e− x
159. Let f(x) = , g(x) = where, [ ] denotes the greatest integer function. Then, domain of {f(x) + g(x)} is
(1 + x )2
1 + [x]

(a) R − {−1} (b) R − [−1, 0) (c) R − {−2, −1, 0} (d) R − [−2, 0)


Functions and Graphs  1.55

 x + 3, − 3 < x < 0

160. Let f(x) =  3, 0 ≤ x < 1 and g(x) = x2 + 4, x > 3. Then,
4 − x, 1 ≤ x ≤ 4

(a) g  f(x) is not defined (b) f  g(x) is not defined (c) g  f(x) is defined (d) both (a) and (b)

ex + e− x
161. Let f(x) = and f  g(x) = x, then g(x) is
2
(a) logex ( 2
(b) loge x + x − 1 ) (
(c) loge x − x 2 − 1 )
(d) both (b) and (c)

162. Let f(x) = (sinx)(sinx + sin3x), x ∈ R. Then, f(x)


(a) ≥ 0 for all real x (b) ≤ 0 for all real x (c) ≥ 0 only for x ≥ 0 (d) ≤ 0 only for x ≤ 0
x − [x]
163. Let f(x) = , x ∈ R, where [ ] denotes the greatest integer function. Then, the range of f is
1 + x − [x]
 1  1
(a) (0, 1) (b) 0,  (c) [0, 1] (d) 0, 
 2  2
1
 2
( x − 2) 2 x −1 
4
164. The domain of the function f(x) = 9 x + 27 3 − 219 − 3 ( ) 
 
5 
(a) [−3, 3] (b) [3, ∞) (c)  , ∞  (d) [0, 1]
2 
kx
165. Let f(x) = , x ≠ −1. Then, the value of k for which f f(x) = x is
x +1
1
(a) 1 (b) (c) −1 (d) 2
2
 p 
166. Let f(x) = [x]cos   where, [ ] denotes the greatest integer function. Then, the domain of f is
  x + 2 
(a) x ∈ R, x not an integer (b) (−∞, −2) ∪ [−1, ∞) (c) x ∈ R, x ≠ −2 (d) (−∞, −1]
167. Suppose f(x) = (x + 2) for x ≥ −2. If g(x) is the function whose graph is the reflection of the graph of f(x) in the line
2

y = x, then g(x) equals


1
(a) − x − 2, x ≥ 0 (b) x − 2, x ≥ 0 (c) , x > 2 (d) x + 2 , x > −2
( x + 2)
2

p p
168. Let f(x) = cosx −x(1 + x), < x < . The range of f(x) is
6 3
p  p  p  p  1 p  p  3 p  p 
(a)  1 +  , 1 +   (b)  − 1+  , − 1 +  
3  3  6  6   2 3  3  2 6 6  
 3  1 p  p 3 p p 
(c)  , 1 (d)  − 1 +  , − 1 +  
 2   2 6  6 2 3 3  

10 x − 10 − x
169. If f(x) = , x ∈ R, and g(x) is the inverse of f(x), then g(x) is
10 x + 10 − x

1  2x  1 1 +x  1
(a) log10   (b) log10   (c) log10(2 − x) (d) log (2x − 1)
4 1 − x  2 1 − x  2 10
1.56  Functions and Graphs

170. Which one of the following statements is false?


(a) f(x) = x2 − [x2] where, [ ] denoted the greatest integer function, is periodic with period 1
 −3  3
(b) If f(x) = |x| + [x] where, [ ] denotes the greatest integer function, f   + f   = 2
 2  2
p
(c) For the set of all non zero real numbers, if f(x) = 2sin and g(x) = x , then f  g(4) − g  f(6) = 1
x
(d) Range of the function f(x) = 2 + 3cos4x + 4sin4x is [−3, 7]

Assertion Reason Type Questions

Directions: Each question contains Statement-1 and Statement-2 and has the following choices (a), (b), (c) and (d), out
of which ONLY ONE is correct.
(a) Statement-1 is True, Statement-2 is True; Statement-2 is a correct explanation for Statement-1
(b) Statement-1 is True, Statement-2 is True; Statement-2 is NOT a correct explanation for Statement-1
(c) Statement-1 is True, Statement-2 is False
(d) Statement-1 is False, Statement-2 is True
 −1
171. f : R −   → R
2
x +1
f(x) =
2x + 1
Statement 1
f(x) is invertible
and
Statement 2
f(x) is one one
172. Statement 1
Period of the function
x
f(x) = sin3x − tan + cos5x is 2π
2
and
Statement 2
Period of sinx is 2π
 x + y  f (x ) + f (y )
173. Let f(x) be a nonconstant function satisfying the relation f  = for all x, y∈R.
 2  2
Statement 1
f is one one
and
Statement 2
f(x) is of the form ax + b where a and b are constants and a ≠ 0.
Functions and Graphs  1.57

174. Let the domains of two functions f(x) and g(x) be D1 and D2 respectively.
Statement 1
f(x)
Domain of is D1 ∩ D2
g(x)
and
Statement 2
Domain of f(x) g(x) is D1 ∩ D2
175. Statement 1
f(x) = cos (x − [x])
where, [ ] denotes the greatest integer function, is periodic with period 1.
and
Statement 2
The function g(x) = x − [x] is periodic with period 1.
176. Statement 1
 1   −25 
f :  ,∞→  ,∞
2   4 
f(x) = x − x − 6 is invertible
2

and
Statement 2
A function f : R → R is invertible if it is one to one.
177. Statement 1
Range of the function
f(x) = 2 + sin3x is [1, 3]
and
Statement 2
sin 3x is an odd function.
178. Statement 1
( )
The function f(x) = cos log x + x 2 + 1 is an even function.

and
Statement 2
If f(x) is an even function, then the curve y = f(x) is symmetrical about y-axis.
2x − 1 , −3 < x < −2
179. Let f(x) =  and g(x) = 2x, x > 0
 5x + 5 , x ≥ − 2

Statement 1
g(f(x)) is defined for all x∈(−3, ∞)
and
Statement 2
The function g(f(x)) is defined for all x only if the range of f(x) is a subset of the domain of g(x).
1.58  Functions and Graphs

180. Let f : (0, ∞) → R : f(x) = 2x + logex


Statement 1
f−1(2) = 1 where, f−1 denotes the inverse of f.
and
Statement 2
f(x) is bijective.

Linked Comprehension Type Questions


Directions: This section contains 3 paragraphs. Based upon the paragraph, 3 multiple choice questions have to be
answered. Each question has 4 choices (a), (b), (c) and (d), out of which ONLY ONE is correct.
Passage I
y
A straight line is said to be an asymptote of an infinite branch of a curve, if as the point
P on the curve recedes to infinity along the branch, the perpendicular distance of P
from the straight line tends to zero.
The lines x = 2 and x = −2 are asymptotes of the above curve. −2 O 2
If, for a curve, as x → a (where, a is finite), y → ±∞, then, x = a is an asymptote
of the curve.
If, for a curve, as y → b (where, b is finite), x → ±∞, then, y = b is an asymptote
of the curve.
1
x = 2 as an asymptote of the curve y =
x−2
1 1+ y
y = 1 is an asymptote y = of the curve x2 =
( x − 2 ) 1− y

181. The asymptotes of the curve y(x2 −9) + 27 = 0 are


(a) x = ± 3 (b) y = 0, x = ±3 (c) x = 0, y = 3 (d) x = 3, y = 3
25 16
182. The asymptotes of the curve + = 1 are
x2 y 2
(a) The curve has no asymptotes (b) x = ± 5
(c) y = ± 5 (d) both (b) and (c)
x
183. The asymptotes of the curve y = 2
+ 5 are
x −9
(a) x = ± 3 (b) y = 5
(c) x = 0 (d) both (a) and (b)
Passage II
The authorities of an amusement park are planning to have a central fountain area, surrounded
by a tiled circular area as shown in the figure in a reserved circular ground of 100 π m2. A is the
area in m2 of the tiled region. They have only a limited budget. They want to find the radius r 3m
that allows maximum area within the budget, where r is the radius of the circular portion including
the tile portion.
r
Functions and Graphs  1.59

184. r as a function of A is

A + 9p
(a) r = (b) A = πr2 + 9π
p
A − 9p
(c) r 2 = (d) r is not a function of A
p
185. If the area of the tiled portion is to be 27π m2, the radius is
(a) 9π m (b) 6π m (c) 6 m (d) 3 m
186. If the area of the tiled portion is to be 27π m2, how wide is the tiled portion?
(a) 3 m (b) 3π m (c) 6π m (d) 6 m
Passage III
A consumer research society finds that the percentage of homes in a metropolis with at least one VCR is given by
80
P(t) = where, t is the number of years after the year 1980. The city had 5000 homes.
1 + 63e −0.63t
187. The domain of the function is
(a) (0,5000) (b) (0,∞) (c) (0,100) (d) (1.25,80)
188. The percentage of homes with VCR is in the year 2000 is (It is given that e−12.6 = .33 x 10−5)
(a) 60 (b) 80 (c) 100 (d) 20
189. Find the years when the % of homes with VCRs is > 70.
100log 441
(a) after long years. (b)
63
100log 441 100log 441
(c) 1980 − (d) 1980 + year onwards
63 63

Multiple Correct Objective Type Questions


Directions: Each question in this section has four suggested answers of which ONE OR MORE answers will be correct.
1− x
190. If f(x + y) = f(x) + f(y) and f(5) = 50 and g(x) = 1 + sin−1(1 − x) − tan−1 in (0, 2). Then
2x − x 2
3 3 1 1
(a) (g  f)   = 1 (b) (f  g)   = 10 (c) (g  f)   = 1 (d) (f  g)   = 10
2 2 2 2
1
191. Let f(x) = Then,
3+ x
 1
(a) domain of f is R − {−3} (b) range of f is  0, 
 3
(c) f is an even function (d) x – axis is an asymptote of the curve y = f(x)
192. The graph of a function f(x) is as shown below.
Then,
(a) f(x) = −2 x < 0 (b) f(0) is not defined
 2 0< x <1

(c) f(x) = 3 − x, 1 ≤ x < 3 (d) range of f(x) is [−2, 2]
 x − 3, x≥3

1.60  Functions and Graphs

2
π
4
o 1 3 x

−2

193. Consideration the function


x−3
f(x) = loge (x2 − x − 2) and g(x) =
x−2
Then,
(a) domain of f(x) + g(x) is (−∞, −1) ∪ (2, ∞)
f(x)
(b) domain of is R − [−1, 2] − {3}
g(x)
(c) g(x) <1 for all x in the domain of g
x−3
(d) x = 2 and y = 1 are asymptotes of the curve y =
x−2
194. f(x) is an even periodic function with period 10. In [0, 5],
 2x, 0≤x<2
 2
f(x) = 3x − 8 2 ≤ x < 4 . Then,
 10 x 4≤x≤5

f (−13) − f (11) 17
(a) f (−4) = 40 (b) =
f (13) + f (−11) 21

(c) f(5) is not defined (d) range of f(x) is [0, 40]

195. Let f(x) = x − [x] where, [ ] denotes the greatest integer function
Then,
(a) g(x) = x + 2 − [x + 2] is periodic with period1 (b) h(x) = x2 − [x2] is periodic with period1
(c) (x) = loge (x −[x]) is periodic with period 1 (d) range of l(x) is (−∞, 0)
1
196. Let f(x) = . Then,
( x)
1 −
3
(a) domain of f o f(x) is R − {0, 1} (b) f o f (−2) =
2
(c) domain of f o f o f(x) is R − {1} (d) f o f o f(5) = 5
197. If the domain of

1   x2 
f(x) = cos −1  log 3    where, x > 0 is [a, b] and the range of f(x) is [c, d], then
p   3 

(a) a, b are the roots of the equation x4 − 3x3 − x + 3 = 0


(b) a, b are the roots of the equation x4 − x3 + x2 − 2x + 1 = 0
(c) a3 + d3 = 1
(d) a2 + b2 + c2 + d2 = 11
Functions and Graphs  1.61

Matrix-Match Type Questions

Directions: Match the elements of Column I to elements of Column II. There can be single or multiple matches.
p p
198. Let A = {−1, 0}, I1 = (0, 1), I2 = [−1, 0], I3 =  , 
4 2 
Column I Column II
Function Domain
sin −1 x
(a) y = where, {.} denotes the fractional part function (p) R+
{x}
1
(b) y = where, [.] denotes the greatest integer function (q) R+ − I1
 tan x 
−1

(c) |y| = log10x (r) I1UI2 − A


(d) |y| = e − logx
x
(s) I3
199.
Column I Column II
(a) Domain of the function
 −1 1 
f(x) = 4 + x + 4 − x + x − x 2 is (p)  , 
 2 2
(b) Domain of the function


(
f(x) = log e x 2 − 6x + 6 is ) (q) (−∞, 1]

(c) Domain of the function


p
f(x) = + sin −1 (2x ) is (r) [0, 1]
5
(d) Domain of the function
f(x) = log(x − 3) (x2 − 25) is (s) (5, ∞)
200.
Column I Column II
1
(a) Range of the function f(x) = is (p) (−∞, ∞)
3 + sin5x
(b) Range of the function
 11 
f(x) = (x + 1) (x − 3) (x − 5) is (q) 1, 
 3
(c) Range of the function
 2p   4p  1 1
f(x) = sin  + x  + 2cos  − x  is (r)  , 
 3   3  4 2

x2 + x + 3
(d) Range of the function f(x) = is (s)  − 5, 5 
x2 + x + 1
1.62  Functions and Graphs

Solutions
a ns w e r K e ys

Topic Grip 9. (i)  p (ii) 2p 73. (d) 74. (a) 75. (a)
(iii)  4p (iv) 1 76. (c) 77. (b) 78. (b)
1. (i) (0, 1) ∪ (1, ∞) 79. (c) 80. (b) 81. (a)
(v)  p
π 82. (c) 83. (d) 84. (d)
(ii) 2np + n∈Z 3 85. (c) 86. (c) 87. (c)
2 10. (i)  −1 +
2 88. (b) 89. (b) 90. (b)
(iii) [-1, 1) - {0} 91. (b) 92. (c) 93. (b)
(ii)  0
(iv) (-∞, -2) ∪ [-1, 2] 94. (c) 95. (c) 96. (d)
1 1 97. (b) 98. (b) 99. (a)
 π 7π  (iii)  +
(v)  − ,  2 2 100. (b) 101. (b) 102. (c)
 6 6  103. (a) 104. (c) 105. (a)
(iv)  -2
(
(vi) −3 + 14,1 11. (b)
14. (a)
12. (a)
15. (b)
13. (a)
16. (a)
106.
109.
(b) 107. (b)
(b) 110. (b)
108.
111.
(a)
(b)
(vii) (-∞, 1] ∪ [3, ∞) 17. (a) 18. (a) 19. (d) 112. (a) 113. (b) 114. (c)
20. (a) 21. (b) 22. (a) 115. (c)
(viii) 3,32π  23. (b) 24. (b) 25. (c) 116. (b)
26. (a) 117. (b), (d)
(ix) (-∞, -2) ∪ (-2, -1) ∪ (1, ∞) 27. (b), (c) 118. (c), (d)
28. (a), (b), (c) 119. (a), (d)
2. (i) [1, ∞) 120. (a) → (r)
29. (b), (d)
(ii) [0, 2] 30. (a) → (s) (b) → (p)
(b) → (p) (c) → (q), (s)
(iii) {1, cos1, cos2} (d) → (r), (s)
(c) → (q)
(iv)  − 2, 2  (d) → (r)

(v) {10}
Additional Practice Exercise
1 + x , −1 ≤ x < 1
IIT Assignment Exercise 121. (i)  Fig (i) → Domain : (-∞, 6]
3. g o f(x) =  Range : [-4, 10]
3 − x , 1 ≤ x ≤ 3 31. (a) 32. (a) 33. (d)
34. (c) 35. (d) 36. (a)   Fig (iv) → Domain : [-5 ∞)
1 + x, − 2 ≤ x ≤ −1 37. (b) 38. (b) 39. (b) Range : [-2, ∞)
 (ii) (i) even
f o g(x) = −1 − x, − 1 ≤ x ≤ 0 40. (d) 41. (d) 42. (c)
 x − 1, 0 ≤ x ≤ 2 43. (d) 44. (a) 45. (b) (ii) odd
 (iii)  even
46. (a) 47. (c) 48. (d)
49. (d) 50. (b) 51. (c) (iv)  periodic
4. (12 + 5x)
52. (a) 53. (b) 54. (a) (iii)  [4, 16]
5. 20 55. (b) 56. (d) 57. (c) 122. (i)  (-∞, -2) ∪ [-1, 1] ∪ (2, ∞)
−3 ± 5 −1 ± 5 58. (d) 59. (c) 60. (d) (ii)  R excluding 0, ±1, ±2, ….
6. and . 61. (c) 62. (a) 63. (c)
2 2 1
64. (c) 65. (d) 66. (a) (iii)  ≤ x <1
7. (i) [-1, ∞) 67. (a) 68. (b) 69. (d) 2
(ii) [0, ∞) 70. (d) 71. (c) 72. (b) (iv)  (-∞, 0] ∪ [10, ∞)
Functions and Graphs  1.63

123. (i)  odd 131. (b) 132. (c) 133. (d) 188. (b) 189. (b) 190. (b), (d)
(ii)  neither odd nor even 134. (c) 135. (b) 136. (d) 191. (b), (c), (d)
(iii)  odd 137. (a) 138. (c) 139. (c) 192. (a), (b), (c)
(iv)  odd 140. (a) 141. (b) 142. (a) 193. (a), (b), (d)
(v) neither odd nor even 143. (a) 144. (a) 145. (a) 194. (a), (b), (d)
(vi) neither odd nor even 146. (a) 147. (a) 148. (b) 195. (a), (c), (d)
(vii) even 149. (d) 150. (d) 151. (b) 196. (a), (b), (c), (d)
124. (i)  symmetric 152. (c) 153. (d) 154. (d) 197. (a), (c), (d)
(ii)  symmetric 155. (d) 156. (b) 157. (c) 198. (a) → (r)
(iii) reflexive, symmetric and 158. (a) 159. (b) 160. (d) (b) → (s)
transitive (c) → (q)
161. (d) 162. (a) 163. (b)
125. {(1, 1), (2, 2), (3, 3), (3,1), (d) → (p)
164. (c) 165. (c) 166. (b)
199. (a) → (r)
(2, 3), (1, 2), (2, 1)} 167. (b) 168. (b) 169. (b)
(b) → (q), (s)
126. Domain : R 170. (a) 171. (d) 172. (a) (c) → (p)
Range : [0, 1] 173. (a) 174. (d) 175. (a) (d) → (s)
1  176. (c) 177. (b) 178. (a) 200. (a) → (r)
127. Domain :  , ∞ 
4  179. (d) 180. (a) 181. (a) (b) → (p)
Range : {1} 182. (d) 183. (d) 184. (a) (c) → (s)
129. 13 130. (5x + 4) 185. (c) 186. (a) 187. (b) (d) → (q)
1.64  Functions and Graphs

H INT S AND E X P LANAT I ON S

Topic Grip ( )
log 4 x 2 + 6x − 5 ≤ 1 ⇒ x 2 + 6x − 5 ≤ 4
1.
−6 − 72 −6 + 72
(i) f(x) is defined when log x > o must lie between &
2 2
⇒ x > 0
⇒ x 2 + 6x − 9 ≤ 0
\ i.e., x ≠ 1 and x > 0 (\ log 1 = 0)
\ Domain = (0,1) ∪ (1, ∞). ⇒ x ∈ −3 − 3 2, −3 + 3 2   (1)

(ii) f(x) is defined only when sinx - 1 ≥ 0 x 2 + 6x − 5 > 0 ⇒ x must lie beyond
⇒ sin x ≥ 1
−6 − 56 −6 + 56
⇒ sin x = 1 &
2 2
p
⇒ x = np + (−1)n ,n∈Z i.e., beyond −3 − 14 and − 3 + 14   (2)
2
1 – x2 ≥ 0 ⇒ – 1 ≤ x ≤ 1   (3)
p
⇒ x = 2np + , n ∈ Z. Combining (1), (2), (3), the domain of f is
2
(iii) x + 2 is defined when x > - 2. (−3 + 14,1

log10 (1 - x) defined when 1 - x > 0 and x ≠ 0 (vii) [x]2 − 4[x] + 3 ≥ 0


⇒ x < 1 and x ≠ 0. ⇒ [x] must lie beyond 1 and 3
sin- 1 x defined when - 1 ≤ x ≤ 1 Domain of f is (−∞,1] ∪ [3, ∞ )
\ Domain of f(x) = [ - 1, 1) - {0}
(viii) sin {log 2 log 3 x} ≥ 0
4 − x2
(iv) f(x) is defined when ≥0 ⇒ 0 ≤ log 2 log 3 x ≤ p
[x] + 2
⇒ 4 - x2 ≥ 0 and [x] + 2 > 0 log 2 log 3 x ≥ 0 ⇒ log 3 x ≥ 1 ⇒ x ≥ 3   (ii)
or 4 - x ≤ 0 and [x] + 2 < 0
2
log 2 log 3 x ≤ p ⇒ log 3 x ≤ 2p
⇒ x ≤ 4 and [x] > - 2
2

⇒x≤3 
(2 )
p

 (ii)
or x2 ≥ 4 and [x] < - 2.
Combining (i) and (ii)
⇒ x ≤ 2 and [x] > - 2
Domain of f is 3,32 
p

or x ≥ 2 and [x] < - 2

⇒ x ∈ [- 1, 2] or x ∈ (- ∞, - 2) (ix) x 2 − 1 > 0 ⇒ x 2 > 1 ⇒ x > 1 or x < −1


\ Domain = (-∞, - 2) ∪ [- 1, 2]
x + 2 ≠ 0 ⇒ x ≠ −2
3
(v) - 1 ≤ ≤1
4 + 2sin x Also 1 + x 2 > 0 for all real x.
⇒ - 7 ≥ 2sinx ≥ - 1 The domain of f(x) is ( − ∞, −2) ∪ (−2, −1) ∪ ( 1, ∞ )
1 7
sin x ≥ − or sin x ≤ - (not possible) 2.
2 2 1
(i). ex > 0, >0
ex
 −p 7p 
\x∈  ,  \ ex + e-x ≥ 2 (A.M ≥ G.M).
 6 6
ex + e− x
(vi) We must have \ ≥1
2
( ) (
log 4 x 2 + 6x − 5 ≤ 1, x 2 + 6x − 5 > 0 and 1 − x 2 ≥ 0 ) \ range = [1, ∞).
Functions and Graphs  1.65

(ii) Domain of f(x) = [- 5, - 1] J [ 



Let y = − x 2 − 6x − 5 ⇒ y ≥ 0 
⇒ y2 = - x2 - 6x - 5 
⇒ x + 6x + 5 +y = 0
2 2

± ±   
⇒ 36 - 4 (5 + y2) ≥ 0, y ≥ 0 (x ∈ R). 
⇒ 36 - 20 - 4y2 ≥ 0
⇒ y2 ≤ 4, y ≥ 0 f o g(x) = (3 + x) – 2, –2≤x≤–1
⇒ 0 ≤ y ≤ 2 = (1 – x) – 2, –1≤x≤0
\ Range = [0, 2]. = – (1 – x), 0≤x≤1
= – (1 – x), 1≤x≤2
 p p
(iii) When x ∈  − ,  or
 2 2
[x] = - 2, - 1, 0, or 1 1 + x, − 2 ≤ x ≤ −1

\ cos [x] = cos (- 2), cos (- 1), cos 0 or cos 1 f o g(x) = −1 − x, − 1 ≤ x ≤ 0
 x − 1, 0 ≤ x ≤ 2
⇒ cos2, cos1, 1 
\ Range = {1, cos1, cos2} gof(x)

 1 1 
(iv) f(x) = 2 cos2x + sin2x  Since f is not defined in – 2 ≤ x ≤ – 1, gof is also un-
 2 2  defined in that interval.
p  – 1 ≤ x ≤ 0 → gof = 1 – (– x) = 1 + x
= 2 cos  − 2x 
4  0 ≤ x ≤ 1 → gof = 1 – (– x) = 1 + x
p  1 ≤ x ≤ 2 → gof = 3 + (– x) = 3 + (– x) = 3 – x
sin C = cos  − 2x  lies between -1, and 1
4  2 ≤ x ≤ 3 → gof = 1 – (x – 2) = 3 – x
1 2+x
\ Range f(x) = [ − 2, 2 ] 4. g(x) = =
1 5 + 2x
(v) Domain f + g = Df ∩ Dg = {0} 2+
2+x
\ Range f + g = f(x) + g(x) at x = 0 1 5 + 2x
h(x) = f(g(x)) = =
= {5 + 5} 2+x 12 + 5x
2+
= {10} 5 + 2x
3. Range of f(x) is [– 1, 1] 1 (2 + x ) (5 + 2x )
f(x)g(x)h(x) = × ×
Range of g(x) is [–1, 2] (2 + x ) (5 + 2x ) (12 + 5x )
f(x) = – 1 + 1 – x = – x, – 1 ≤ x ≤ 1
1
= – 1 + x – 1 = x – 2, 1 ≤ x ≤ 3 =

(12 + 5x )
g(x) = 2 – (– 1 – x) = 3 + x, – 2 ≤ x ≤ – 1
1
= 2 – x – 1 = 1 – x, – 1 ≤ x ≤ 2 i.e., = 12 + 5x
f(x)g(x)h(x)
The graphs of f and g are
5. f(x) + f(y) = f(x + y);
I [ 
Putting x = 0, y = 0, 2 f(0) = f(0) ⇒ f(0) = 0
Replacing y by – x,

f(x) + f(– x) = f(0) = 0 ⇒ f(x) = – f (– x)

±    ⇒ f(x) is an odd function
Since f(1) = 2,
1.66  Functions and Graphs

f(2) = f(1) + f(1) = 2f(1) y


Again, f(3) = f(2) + f(1) = 3 f(1)
f(4) = 4 f(1); f(5) = 5 f(1); f(10) = 10 f(1) = 20
1
6. Since f(x) is an even function,
f(x) = f(– x) o 1 x
−1
− ( x + 1)
or x =
x+2
Range of the function is [0, ∞).
x + 2x = − x − 1 ⇒ x 2 + 3x + 1 = 0
2

−3 ± 5 8.
⇒x=
2 (i) \
Both roots are in (–5, 5)
 x + 1
Also, since f(x) = f  ,
 x + 2 
 [
x +1 \ _[_
x= ⇒ x 2 + 2x = x + 1 ⇒ x 2 + x − 1 = 0
x+2
\
−1 ± 1 + 4 −1 5
x= = ±
2 2 2
Both roots are in (–5, 5)
  [
The four values of x satisfying
\ _[±_
 x + 1 −3 ± 5 −1 ± 5
f(x) = f   are and .
x + 2 2 2
(Translation of y = | x | by 1 unit to the right)
7. (i) f(x) = x - 1
2

We note that the function is not one to one [as f(k) \


= k2 - 1 = f(-k)].
When x = ± 1, y = 0.
Also, when x = 0, y = -1. When x assumes a value  [
between -1 and +1, (x2 -1) is negative. The graph is
as shown below.
± \ _[_
y
2
y =f(x) = x −1

(Translation of y = | x | by 1 unit to the right)


x
−1 o 1
(move the curve y = | x | by 3 units down parallel to
y – axis)

Range of the function is [-1, ∞) \


(ii) f(x) = |x2 -1|
The position of the graph of f(x) in (i) between
x = -1 and x = +1 is below the x-axis since f(x) is 
negative in this interval.
But, in the case of f(x) = |x2 -1|, f(x) is positive for all
x and so, its graph will be completely above the x-axis   [
as shown below.
Functions and Graphs  1.67

Translating the graph of y = | x | by 3 units to the y


right and then, moving the graph by 2 units up
parallel to itself.
(ii) The graph of y = x – [ x ] is drawn below.

R R R R R R 0 1 2 3 x

[
    The function may be represented as
e x + k , x ∈ ( − k, − k + 1)
f(x) =  x − k , k positive integer
e , x ∈ (k − 1,k )
y = x + 2 – [ x + 2 ] = x + 2 – [ x ] – 2 = x – [ x ], Period of f(x) = 1
which means that the graph of this function is (v) We have, f(x + p) = | sin (x + p) | = sin x = f(x)
the same as that of y = x – [ x ].
Period = p
y–1=x– [x] p  p  p p
10. (i) f   = cos p    + cos [p2 ]  + sin [− p2 ] 
2   2   2  6
y
9p  −10p 
= cos p + cos + sin 
 6 
o o o o o o

1 2
5p p 3
x = −1 + 0 − sin = −1 + sin = −1 +
O 3 3 2
(ii)
{ } {
f (− p) = cos (p[− p]) + cos [p2 ]( − p) + sin [− p2 ](− p) }
Move the graph of y = x – [ x ] by 1 unit up parallel
to itself. = cos (−4 p) + cos (−9p) + sin (−10p) = 1 − 1 + 0 = 0
(iii)
9.
2p p   p    p    p 
(i) Replacing x by x + , f   = cos p    + cos [p2 ]    + sin [− p2 ]   
2 4   4    
4    12  

  2p    9p   −10p 
y = 4cos 2  x +  + 1 = 4cos (2x + 2p + 1) = cos0 + cos   + sin 

2   4   12 
 
= 4cos (2x + 1) 1 5p 1 1 1 1
= 1+ − sin = 1+ − = +
2 6 2 2 2 2
Period is therefore p
(ii) Period of sin x is 2p, while the period of cos 3x {
(iv) f(p) = cos (p[p]) + cos [p2 ](p) + sin [− p2 ]p } ( )
is
2p
. = cos3p + cos9p + sin (−10p)

3 = −1 + (−1) + 0 = −2
Period of y = 2 sin x + cos 3x is 2p
2p p 2x
(iii) Period of sin 4x is or 11. f(x) =
4 2 1+ x
x 2p For domain of f,
Period of cos = = 4p 2x
2 (1 / 2) ≥ 0 and 1 + x ≠ 0
1+ x
Hence, period of y = sin 4x + 3 cos (x/2) is 4p. ⇒ x ≥ 0 and x > -1
(iv) The graph of y = e x −[x] is shown below. or x ≤ 0 and x < -1⇒ x ∈ ( -∞, -1) ∪ [0, ∞)
1.68  Functions and Graphs

12. Let x - 1 = u ⇒ R is not symmetric


f(u) = 2 u3 + 3 ⇒ Statement 2 is true
We know that, the graph of f(u) is a parabola sym- Choice (a)
metric about u = 0 i.e., y axis
17. Statement 2 is true
u = 0⇒ x - 1 = 0 ⇒ x = 1
Consider Statement 1
\ f(x) is symmetric about the line x = 1.
Let a and b denote the roots of the quadratic
3 x2 - 5x - 9 = 0
13. f(x) =
1+ x Then, a ≠ b, but f(a) = f(b) = 0
 3  ⇒ f(x) is not one one
g(x) = f 
 1 + x  ⇒ Statement 1 is true

3 Choice (a)
=
3 18. Statement 2 is true
1+
1+ x Consider Statement 1
3 × (1 + x) 3(1 + x) f(x) is not one one, since f(-x) = f(x)
= =
4+x 4+x Range of f(x) is [2, 4]
h(x) = f( g(x)) Using Statement 2, Statement 1 is true
 3(1 + x)  3 3(4 + x) ⇒ Choice (a)
= f = =
 4 + x  3(1 + x) 7 + 4x 19. {x} = x - [x] which is periodic with period 1
1+
4+x Statement 2 is true
3 3(1 + x) 3(4 + x) Consider Statement 1
\ f(x) . g(x) . h(x) = × ×
(1 + x) (4 + x) (4x + 7) f(x) = sin(3x - [3x] = sin({3x})
1
Using Statement 2, period of f(x) is
27 3
= Statement 1 is false
4x + 7
Choice (d)
15..
20. \ statement 2 is true.
y = x2 y = |x| f(x) = sin -1( log x)
1 log x defined only for positive number.
\ x = 1 f(1) = sin- 1 (log1)
−1 1
= sin-1(0) log1 = 0
= 0 (from this graph)
The graphs of two functions y = |x| and y = x 2
\ Statement -1 follows from statement – 2
are drawn above. Now, in (-1, 1) minimum of \ option (a).
{x2, |x| } is x2
21. t
For |x| >1, minimum of {x2, |x| } is |x| 6
\ Correct choice is (b)

16. Statement 2 is true


Consider Statement 1 0 7 14 t
x -x=0
The graph is periodic
⇒ R is not reflexive
22. The equation to find the correct menu table for any day
If x – y > 0, then y - x< 0
is the function t = 7w + x, where w is the number of
Functions and Graphs  1.69

weeks from the beginning of the year; t = the number \ Domain of (f o g)x is R - {0}
of the day from the beginning of the year, x gives the Domain of (g o f)x is R - {1, 2}
number, from which we can find the exact table.
1−3+2
23. Number of days till 30th June Now, (f o g) (1) = =0
1
= 31 + 28 + 31 + 30 + 31 + 30
1
= 181 = 7 x 25 + 6 and (g o f) (0) =
2
Since 30th is a Saturday, in which table 6 is to be
followed. 28. (a) h(x) . (f o g) (x) = cosx .
(10 sin x
−1 ) = ϕ(x) (say)
\ July 1st corresponds to 0,which is Sunday table (10 sin x
+ 1)
sin ( − x )
24. 10 −1
y \ f(-x) = cos(-x) . sin ( − x )
2 10 +1
 10 − 1  sin x
1 = - cos(x) .  sin x = −ϕ(x)
 10 + 1 
0 1 2 3 x ⇒ h(x) (fog) x is odd
10cos x − 1
(b) g(x) . (foh)x = sinx = ϕ (x) (say)
10cos x + 1
using the definition of u(x), we can write f(x) as f(x)
= u(x -1) + u(x - 2) + u(x -3), 0 ≤ x < 3
f(-x) = −
(
sin x. 10cos x − 1 ) = −ϕ(x)
25. We have (10 cos x
+1 )
x 0 ≤ x <1 ⇒ g(x) (f o h)x is odd.
 10 x − 1
f(x) = −1 1 ≤ x < 2 (c) f(x). (h o g)x = .cos (sin x ) = ϕ(x) (say)
2 2 ≤ x < 3 10 x + 1

 10− x − 1 
using the definition of u(x), f(x) can be represented as \ f(-x) =  − x cossin(− x)
 10 + 1 
f(x) = x{u(x) - u(x - 1)} - {u(x - 1) -u(x -2)} +
2u(x - 2)  10 x − 1 
=- x cos (sin x ) = − ϕ(x)
= x {u(x) - u(x -1)} - u (x - 1) + 3u(x - 2)  10 + 1 
0≤x<3 ⇒ f(x) . (h o g) x is odd
26. using the definition of u(x), we may write f(x) as
10 x − 1
f(x) = x3 [u(x) - u(x -1)] + (x -1)[u(x -1) - u(x - 3) (d) f(x) . h(x). g(x) = . sin x.cos x = ϕ(x)
10 x + 1
27. Given f(x2 -1) = x4 - 5x2 + 6
(say)
Take y = x2 - 1 ⇒ x2 = y + 1
 10− x − 1 
\ f(y) = (y + 1)2 - 5(y + 1) + 6 ϕ(− x) =  − x sin ( − x ).cos ( − x )
 10 + 1 
= y2 + 2y + 1 - 5y - 5 + 6
= y2 - 3y + 2  10 x − 1 
=  x . sin x . cos x = ϕ(x)
\ f(x) = x2 - 3x + 2 ⇒ f : R → R  10 + 1 
1
g(x) = ⇒ f(x) . h(x). g(x) is an even function
x
\ g : R - {0} → R 29. Given f(x + 2) + f(x - 2) = f(x)
1 − 3x + 2x 2 Take x = x + 2
(f o g)x =
x2 f(x + 4) + f(x) = f(x + 2)   (1)
1 1 Again take x = x + 2
(g o f)x = 2 =
x − 3x + 2 ( x − 1)( x − 2) ⇒ f(x + 6) + f(x + 2) = f(x + 4)   (2)
1.70  Functions and Graphs

(1) + (2) ⇒ f(8) + f(4) = f(8)


f(x + 4) + f(x) + f(x + 6) + f(x + 2) = f(x + 2) ………………
+ f(x + 4) ………………
\ f(x + 6) + f(x) = 0 f(12) + f(8) = f(10)
\ f(x + 6) = -f(x) Adding we get
\ f(x + 12) = -f(x + 6) = f(x) f(0) + f(4) + f(6) + … f(12) = f(10)
\ f is periodic with period 12. 6
∴ ∑ f(2r) = f(2) + f(10)
Again f(x + 2) + f(x - 2) = f(x) r =0
Setting x = 2, 3, ….. 10, we get
= f(2) + f(−2) = f(0) = 0
f(4) + f(0) = f(2)
6
f(5) + f(1) = f(3) ∴ ∑ f(2r) = 0
f(6) + f(2) = f(4) r =0

f(7) + f(3) = f(5)


30. (a) Transformation of f(x) to f(-x).
………………..
→ Take the image of f(x) about the y-axis
……………….
Transformation of f(x) to -f(x)
……………….
→ Take the image of f(x) about the x-axis
f(12) + f(8) = f(10)
Adding all, we get
f(0) + f(1) + f(4) + f(5) + f(6) + f(7) + f(8) + f(11) y = ex
+ f(12) = 0
12
⇒ ∑ f(r) = f(2) + f(3) + f(9) + f(10)
r =0

Since f(x) is periodic with period 12


f(x + 12) = f(x)
\ f(-2 + 12) = f(-2) Image of ex about the y-axis → e-x
i.e., f(10) = f(-2)
f(9) = f(-3)
y = e−x
12
\ ∑ f(r) = f(2) + f(3) + f(−3) + f(−2)
r =0

Again f(x + 2) + f(x - 2) = f(x)


⇒ f(2) + f(−2) = f(0) = 0 (given )
12
\ ∑ f(r) = 0 + f(3) + f(−3)
r =0
Image of e-x about the x-axis → -e-x

12
\ ∑ f(r) ≠ 0 because f(3) and f(-3) are not
r =0

specified.
Again setting x = 2, 4 …. 10, we get y = −e−
x

f(4) + f(0) = f(2)


f(8) + f(2) = f(4)
Functions and Graphs  1.71

(b) |y| ≥ 0
\ f f(x) < 0; graph of |y| = f(x) does not exist and if
f(x) > 0,
y = e−
x graph of |y| = f(x) is y = ±f(x)
1
Thus region where, f(x) < 0 is neglected and the
region where, f(x) > 0 is retained together with its
reflection on the x-axis.

1
2 y = +sinx
3 log3

−2
3 1
y = e−x − 3
The graph below the x-axis is neglected.

1
→ Shift y = e-x downwards through units
3

2
3
|y| = sinx
log3
−log3

2
− 3 −|x| Take the image about the x-axis
y=e −1
3 (d)
2
3

1
−log3 log3 y = logx
y = e− | x | − 1
3

(c) Reflection about the y-axis

−3π −2π −π 0 π 2π 3π −1 1
−1 y = log|x|
1.72  Functions and Graphs

Region below the x-axis is rejected after taking its 3x − 3 + ( x + 1) 4x − 2


reflection about the x-axis = =
x − 1 + 3x + 3 4x + 2
2x − 1
= = f (2x ) .
2x + 1
39. f(x + 2) = (x+3)2 - 2x
Take x + 2 = u ⇒ x = u - 2
⇒ f(u) = (u+1)2 - 2(u -2)

|y| = |log|x||
= u2 + 2u + 1 - 2u + 4 = u2 + 5
\ f(x) = x2 + 5
Reflection of y = |log|x|| about the x-axis
40. f (-x) ≠ f (x) and f (-x) ≠ - f(x) So, f is neither even
nor odd .
f (a + x) ≠ f (x) ⇒ not periodic.
IIT Assignment Exercise
41. Out of the choices,
31. For domain of f, (d) gives
e− x − ex  ex − e− x 
-(x2 - 1) ≥ 0 ⇒ x2 - 1 ≤ 0 ⇒ x2 ≤ 1 ⇒ -1 ≤ x ≤ 1 = −
f4(-x) =
2  2  = -f(x)
Domain of f = {x∈R/-1 ≤ x ≤ 1} ⇒ f4 is odd
2
 1 3 42. y = 5x – 7 ⇒ 5x = y + 7
32. Since x2+ x + 1 =  x +  + ≠ 0 for all real x,
 x 4 y+7 x+7
x= ⇒ f -1(x) = .
therefore domain of f = R. 5 5
33. f(x) = 1 when x + 3 > 0 1− x
f(x) = -1 when x + 3 < 0 43. Out of 4 choices, if f(x) =
1+ x
Range = {-1, 1}. (1 − x)
1 1−
34. x can have any value but x + will be greater than (1 + x)
f[f(x)] = =x
x (1 − x)
2. So range is [2, ∞). 1+
(1 + x)
35. f is invertible if f is one-one and onto 1− x
\ is the inverse of itself
2x − 1 1+ x
36. y =
2x + 1
⇒ 2yx + y = 2x – 1= 2yx – 2x = - (1 + y) 44. log 4  log 3 log 2 x  = 1 ⇒ log3 log2 x = 4

2x (y – 1) = -(1 + y) 4
⇒ log2 x = 34 ⇒ x = 23 .
(1 + y )
x= .  1
2 (1 − y ) 3 | x | +5, x ≤
 2
 1
37. f(2a) = 4f(a) 45. f(x) =  log x , ≤ x ≤ 2
⇒ 2(2a)2 – 2(2a) + 4 = 4(2a2 - 2a + 4)  2
 x2 + 4 , x > 2
⇒ 8a2 - 4a + 4 = 8a2 - 8a + 16 
4a = 12 \ a = 3.
 1 1 2 3
 x − 1 f  −  + f   + f   + f   − f (3)
3 +1  3 3 3 2
3f ( x ) + 1  x + 1 
38. =
f (x ) + 3 x −1 1 1
+3 = 3− +5+3 +5
x +1 3 3
Functions and Graphs  1.73

2 3 x −1
+ log   + log   − (32 + 4) \ f −1 (x) =
3 2 2

2 3 \ g(x) = (g o f ) f −1 (x)
= 12 + log  ×  − 13 = -1 + log1= -1
3 2  (x − 1) 
= (g o f ) 
46. f (1) = 3  2 
g (f(1)) = g (3) = 10 3(x − 1) 1
= + 2 = (3x + 1)
f (2) = 5 2 2
g (f(2)) = g (5) = 16 and so on. 54. f(x) = |x - 1| + |x - 2| + |x + 1 | + |x + 2|
 7  7  7  7 when x ∈ [-2, -1]
47. g o f  −  − f o g  −  = g   −   − f  − 
 3  3  3  3 f(x) = -(x - 1) + - (x-2) - (x+1) + x + 2 = -2x + 4
( )  7 
= g(-3) - f 7 = |-3| -   = 3 - 2 = 1
3 3
When x ∈ [-1, 1]
f(x) = -(x - 1) - (x - 2) + x + 1 + x + 2
48. f(x) = x , g(x) = 3x
3

f o g(x) = f(3x) = (3x)3 = 33x =-x+1-x+2+x+1+x+2=6

g o f(x) = g(x3) = 3x
3
When x ∈ [1, 2]

f o g(x) = g o f(x) ⇒ 33x = 3x


3
f(x) = (x - 1) - (x - 2) + x + 1 + x + 2 = 2x + 4

⇒ 3x = x3 ⇒ x = 0, 3, − 3 Plotting the graph of the function, range of


f(x) = [6, 8]
\ No. of solutions = 3
1
55. (f o g) (x) = f(x3) = 3
49. If x is rational, f(x) = 1 -a rational x − 27
If x is irrational, f(x) = 0 , - a rational This function is not defined when x = 3
\ f(x) is rational for all x. So domain of f o g = R - {3}
\ f f(x) = 1, a rational for all x 56. f(x + y) = 2x + y = 2x × 2y = f(x) × f(y)
⇒ f f f(x) = 1 for all x. ≠ f(x) [1+f(y)]
50. f(x) is a polynomial function f(2x) = 22x = (2x)2
But, f(0) = f(1) = f(-1) = f ( 2 ) = f (− 2 ) = 0 = [f(x)]2 ≠ 2 f(x)
\ f is not injective. f(xy) = 2xy ≠ f(x) × f(y)
6x − 3  f(y) 
51. Let y = ⇒2yx + 4y = 6x – 3 f(x) + f(y) = f(x) 1 + = f(x) [ 1 + 2 y - x ]
2x + 4  f(x) 
⇒ 2yx – 6x = - (4y + 3) = f(x) [1+f(y - x)] = f(x) + f(x) f(y - x)
⇒ x (2y – 6) = - (4y + 3) p
− (4y + 3) 57. 0 < x <
4y + 3 4x + 3 2
x= = ⇒ f-1 (x) = .
2y − 6 6 − 2y 6 − 2x  0 if 0 < x < 1
52. f(x) = 22
x \ [ ] 1 if 1 ≤ x < p
x =
 2
log2 f(x) = 2x log2 2 = 2x
⇒ log2 (log2 f(x)) = x log2 2 = x sin0 = 0 if 0 < x < 1

Inverse of f is f (x) = log2 [log2 (x)]
-1 ⇒ sin[x] =  p
sin1 if 1 ≤ x <
2
53. f(x) = 2x+ 1
p
y −1 We have 0 < sin x < 1 when 0 < x <
y = 2x + 1 ⇒ x = 2
2
1.74  Functions and Graphs

p  1 1  1  1
3
\ [sin x] = 0 for 0< x < 63. Given f  x +  = x 3 + 3 =  x +  − 3  x + 
2  x x  x  x
 0 if 0 < x < 1 1
\ sin [x] + [sin x] =  p Let y = x + Then f(y) = y 3 − 3y
 sin1 if 1 ≤ x < x
 2
∴ f(x) = x 3 − 3x
58. f(x) = ax + b
f (f(x)) = a[f(x)] + b = a(ax + b) + b (
Hence (f o f )( x ) = f ( f ( x )) = f x 3 − 3x )
= a x + ab + b
2

( ) ( )
3
f(f(x)) = x = x 3 − 3x − 3 x 3 − 3x
⇒ a2x + ab + b = x ⇒ a2 = 1 and (a + 1)b = 0 = x 9 − 9x 7 + 27x 5 − 30x 3 + 9x

If a = 1 then b = 0, and a = -1 ⇒ 0 × b = 0
⇒ b can take any value.
64. We must have x4 - 21x2 ≥ 0 and 10 − x 4 − 21x 2 ≥ 0
59. f (2) = the set of values of x such that f(x) = 2
-1
⇒ x2 (x2 - 21) ≥ 0   (1)
3x 2 + 6x + 14 and 100 ≥ x4 - 21x2   (2)
⇒ =2
x 2 − 3x − 3 (1) gives x = 0 or x ≤ − 21 or x ≥ 21
⇒ 3x2 + 6x + 14 = 2(x2 – 3x – 3) (2) ⇒ x - 21x - 100 ≤ 0
4 2

⇒ x2 + 12x + 20 = 0 ⇒ (x +10) (x + 2) = 0 ⇒ (x2 - 25) (x2 + 4) ≤ 0


⇒ x = -2 or x = -10
⇒ x2 - 25 ≤ 0 (as x2 + 4 ≥ 0 always)
\ f-1 (2) = {-2, -10}
⇒ -5 ≤ x ≤ 5
60. Let f(x) = (x + 1)2 - 1 Domain is given by  −5, − 21  ∪  21, 5 and
f(0) = 1 - 1 = 0
\ The graph of f(x) passes through the origin. x=0
So right choice is (d)
 2x + 3 
2 +3
a 1 2x 1  x − 2  4x + 6 + 3x − 6
61. x =  t +  ⇒ =t+   (1) 65. f o f(x) = =
2  t  a t  2x + 3  2 + 3 − 2x + 4
 x − 2  − 2
b 1 2y 1
y =  t− ⇒ =t−   (2)
2 t b t
7x
= =x
x y 7
(1) + (2) ⇒ 2t = 2  + 
a b

66. We have -1 ≤ cos x ≤ 1
2 x y
(1) - (2) ⇒ = 2 −  Adding 2, 1 ≤ 2 + cos x ≤ 3
t a b
1 1
1≥ ≥
x2 y 2 2 + cos x 3
Multiplying 2 − 2 = 1.
a b 1 
Range is  ,1
62. f(x) = [x] + [2x] +……+[nx] - (x + 2x +…..+nx) 3 
= [x] - x + [2x] - 2x +……+ [nx] - (nx) 67. We see that R ⊆ S. Hence statement P is true.
= -[{x} + {2x} +……+{nx}] Now R-1 = {(1, 1), (2, 1), (3, 1), (2, 2), (3, 3)}
1 S-1 = {(1, 1), (2, 1), (3, 1), (2, 2), (2, 3), (3, 3)}
Period of {rx} =
r We note that R-1 ⊆ S-1. Hence statement Q is also
 1 1 1 true.
\ Period of f(x) = LCM 1, , , ......,  = 1
 2 3 n \ Both P and Q are true
Functions and Graphs  1.75

2p f(-x) = x2 + x + 5
68. Period of cos(sin nx) =
n f(x) + f(-x) = 2x2 + 10
x 74. log1/3 (5x − 1) > 0
Period of tan   = np
n
0 < (5x − 1) < 1
cos(sinnx) 2p
\ Period of f(x) = is the LCM of , 1 2
x n 1 < 5x < 2 ⇒ <x<
tan  
n 5 5
and np which is equal to 2np 1 2
Domain is  , 
5 5

But given that 2np = 6p f(x) + 1,f(x) < 0


75. g(x) = 
( f(x) − 1) + 4,f(x) ≥ 0
2
⇒ n = 3

69. Given f(x + y) = f(x) – f(y)   (1) We note that f(x) < 0 if x+2 < 0 if x + 2 < 0 (i.e.,) if x< – 2
Put y = – x in (1) we have f(0) = f(x) – f(– x)   (2) When x < – 2, f(x) = x + 2 &
Put y = x = 0 in (1) g(f(x)) = (x + 2) + 1 = x + 3
We have f(0) = f(0) – f(0) = 0   (3) When – 2 ≤ x < 0, f(x) = x + 2 &
Using (3) in (2) we have 0 = f(x) – f(– x) g(f(x)) = (x+2 – 1)2 + 4 = (x + 1)2 + 4
⇒ f(x) = f(– x) ⇒ f(x) is an even function. When 0 ≤ x < 3, f(x) = |x – 3| = 3 – x &
70. x and x3 are odd functions. However, cos x is an even g(f(x)) = (3 – x – 1)2 + 4 = (2 – x)2 + 4
function. When 3 ≤ x f(x) = x – 3 and
\ f(x) + f(-x) = 2 cos x which is an even periodic g(f(x)) = (x – 3 – 1)2 + 4 = (x – 4)2 + 4
function.
 x + 3, x < −2
71. Observe that when x is an integer x = [x]. Hence, 
( x + 1) + 4, − 2 ≤ x < 0
2
f(x) is not defined when x is an integer. Domain is R
∴ (g o f )( x ) = 
( x − 2) + 4,0 ≤ x < 3
excluding 0, ±1, ±2,… 2

(
72. Let y = f(x) = 5 − ( x − 8)
5
) 1/3
. Then

( x − 4 ) + 4,3 ≤ x
2

1 (3 − x)
y 3 = 5 − ( x − 8) ⇒ ( x − 8) = 5 − y 3
5 5
76. g(x) = f o f(x) = =
 1  (8 − 3x)
3−
()  3 − x 
1/5
⇒ x = 8 + 5 − y3
1 (8 − 3x)
Let z = g(x) = 8 + (5 − x ) 3 1/5
h(x) = f o f o f(x) = =
 3 − x  (21 − 8x)
3−
Now f (g(x)) = 5 − ( x − 8) 
5 1/3
 8 − 3x 
 
1/3
f(x) g(x) h(x)
 1/5 5
 

(
=  5 −  5 − x 3

)   =
1
×
(3 − x)
×
(8 − 3x)
=
1
(3 − x) (8 − 3x) (21 − 8x) (21 − 8x)
( )
1/3
= 5 − 5 + x3 =x
77. Let n(A) = m and n(B) = n. Then
Similarly, we can show that g (f(x)) = x
n(A × A, B ÷ × A) = n(A)3. n(B) = m3n = 384   (1)
( )
1/5
Hence g(x) = 8 + 5 − x 3 is the inverse of f(x). and n(A × B, B × B) = n(A). n(B)3 = mn3 = 864 (2)
m2 4
73. f(x - 2) = x2 - 5x + 11 = (x - 2)2 - (x - 2) + 5 (1) ÷ (2) gives =
⇒ f(x) = x2 - x +5 n2 9
1.76  Functions and Graphs

m and n being the cardinalities are both positive 81. For bijection n (A) = n (B).
n (A )
m 2 3 82. Number of functions from A to B is  n (B)
⇒ = ⇒n= m   (3)
n 3 2 i.e., mm.
⇒ R2 is not a relation from A into B 83. If y2 is related to an expression in x, then corresponding
Multiplying (1) and (2) we have, to one value of x, we get two values for y. Such relations
are not functions. So a, b and c are not functions.
m 4 n 4 = 384 × 864
4 84. {x/f(x) × g(x) = 0} = {x/f(x) = 0 or g(x) = 0}
3 
4
⇒ m  m  = 384 × 864 = {x/f(x) = 0} ∪ {x/g(x) = 0} = A ∪ B
2 
384 × 864 × 16 85. f (-x) ≠ ± f(x)
⇒ m8 = = 65536
81 So f (x) is neither even nor odd.
3 86. f : R → R, f(x) = 3x2 + 1
⇒ m = 4 ⇒ n = m = 6 (As we are interested only
2 f(1) = f(-1) = 4
in positive integral value)
\ This function is not one-one. Hence f has no
78. A = {1, 2, 3, 4, 5} B = {1, 3, 5, 7, 9} inverse.
R1 = {(x, y)/x ∈A, y ∈ B, y = x + 3}
87. The elements, which are mapped to -2 are obtained
= {(2, 5); (4, 7)} by solving
R2 = {(1, 1), (1, 2), (1, 3)} f(x) = -2 ⇒ x2 - 2x - 17 = -2
R1 being a subset of A × B is a relation from A into B. x2 - 2x - 15 = 0
R2 ⊄ A × B [Q(1, 2) ∉ A × B]
(x - 5) (x + 3) = 0 x = 5, x = -3
79. We can have [2x - 3] = -1 or 0 or 1 \ No: of such elements = 2.
3 p
⇒ x = 1, or 2   (1) 88. The maximum value of sin q = 1 when q = ;
2 2
[2x - 3] = -1 ⇒ -1 < 2x - 3 < 0 Minimum value of sin q = -1 when q = 270°
2 < 2x < 3 So range = [-1, 1].
3
1 < x <   (2) 89. x = 2 sin t - 2 ⇒ x + 2 = 2 sin t
2
y = 2 cos t + 1 ⇒ y - 1 = 2 cos t
[2x - 3] = 0 ⇒ 0 < 2x - 3 < 1
\ (x + 2)2 + (y - 1)2 = 4 sin2 t + 4 cos2 t = 4
3 < 2x < 4
3 ⇒ x2 + 4x + 4 + y2 - 2y + 1 = 4
< x < 2   (3)
2 x2 + y2 + 4x - 2y + 1 = 0
[2x - 3] = + 1⇒ 1 < 2x - 3 < 2  1 1  1
2

90. f  x +  = x 2 + 2 =  x +  − 2
4 < 2x < 5  x x  x
5 f(x) = x – 2.
2
2 < x <   (4)
2 1
combining (1), (2), (3) and (4), domain of the func- 91. f (x) = 1 +
x
 5
tion is 1,  1
 2 f   = 1+ x
x
80. The given function is defined when
log2 log3 (x2 + 4x - 23) > 1   1  1 2+x
f f   = 1 + =
i.e., when log3(x2 + 4x - 23) > 2   x  1+ x 1+ x
i.e., when x2 + 4x - 23 > 32
   1   1+ x 3 + 2x
i.e., when x2 + 4x - 32 > 0 f f  f     = 1 + = .
i.e., when x < - 8 or x > 4    x   2+x 2+x
Functions and Graphs  1.77

92. f(x) is positive; \|f(x)| = f(x) 99. Let R be a reflexive relation on a set A. Then
Also f(-x) = f(x) R-1 = {(b, a)/(a, b) ∈ R}
\ f(|x|) = f(x) As R is reflexive, for every a ∈ A, (a, a) ∈ R
\ |f(x)| + f(|x|) = 2 f(x) i.e., for every a ∈ A, (a, a) ∈ R-1

93. This function f maps ⇒ R-1 is reflexive


1→ 2, 2→ 1 As R is reflexive, for every a ∈A, (a, a) ∈ R
3 → 4, 4→ 3 i.e., (a, a) ∉ RC for every a ∈ A ⇒ RC is not re-
flexive.
5 → 6, 6 → 5
i.e., 2m - 1 → 2m and 2m → 2m -1  2 + x; x ≥ 0
100. f(x) = 
So f is one-one and onto. 2 − x; x < 0
94. g(n) = f(n) + f(n+1) 2 + (2 + x); x ≥ 0
f(f(x)) = 
If n is even, n + 1 is odd.  2 − (2 − x) ; x < 0
\ g(n) = 2n - 1 + 2n + 2 = 4n + 1 4 + x ; x ≥ 0
=
If n is odd, n+1 is even  x ;x< 0
\ g(n) = 2n + 2 + 2n - 1 = 4n + 1 1
101. f o g(x) = ( 2 − x) = (2 − x) 4
95. {x∈R/f (x) = f(x)} = {x∈R/f f (x) = x}
-1

f(f(x)) = f(x(x - 1)) = [x(x -1)] [x(x - 1)-1] 1


= x(x - 1) [x2 - x - 1] 102. f(x) is defined if log ≥0
| cos x |
f f(x) = x ⇒ (x - 1) (x2 - x - 1) = 1 i.e., if − log | cos x |≥ 0
⇒ x3 - 2x2 = 0 ⇒ x = 0, 2.
i.e., if log | cos x |≤ 0
96. Domain of f/g = Df ∩ Dg, provided g(x) ≠ 0 i.e., if 0 < | cos x |≤ 1
So Domain of f/g = Df ∩ Dg - { x ∈ Dg/g(x) = 0}
i.e., if ( −1 ≤ cos x < 0) ∪ (0 < cos x ≤ 1)
97. 5|x| - x2 - 6 ≥ 0 ⇒ x2 - 5|x| + 6 ≤ 0
⇒ f(x) is defined at all points of R except at those
When x < 0, x2 +5x + 6≤ 0 , -3 ≤ x ≤ -2 points where cos x = 0
When x > 0, x2 - 5x + 6 ≤ 0, 2 ≤ x ≤ 3 ⇒ f(x) is defined at all points of R except at
x = 0 will not satisfy the condition.
(2n + 1) p
Domain is [-3, -2] ∪ [2, 3] x= ,n ∈ Z
2
98. 2x + 2-x is always > 0 i.e., domain is R ⇒ Domain of
2x − 2− x 22x − 1  (2n + 1) p 
y= = f(x) = R −  x / x = ,n ∈ Z 
2x + 2− x 22x + 1  2 
1 + y 2 ⋅ 22x 103. Given A1 ⊆ A2 ⊆ A and R be a relation on A.
⇒ = Componendo Dividendo
1− y 2 Let y ∈ R(A1). Then there exists an x ∈ A1 such that
(x, y) ∈ R. As A1 ⊆ A2, x ∈ A2 and (x, y) ∈ R
= 22x > 0
⇒ y ∈ R(A2) ⇒ R(A1) ⊆ R(A2)
1+ y
⇒ >0 104. As A1 ⊆ A1 ∪ A2, R(A1) ⊆ R(A1 ∪ A2) (using the result
1− y
of the previous problem)
(1 + y)2 As A2 ⊆ A1 ∪ A2, R(A2) ⊆ R(A1 ∪ A2)
i.e., >0
1 − y2 ⇒ R(A1) ∪ R(A2) ⊆ R(A1 ∪ A2)   (1)
⇒ 1 - y > 0⇒ -1 < y < 1
2
Let y ∈ R(A1 ∪ A2)
1.78  Functions and Graphs

⇒ There exists an x ∈ A1 ∪ A2 such that x R y 7 ( x1 − x 2 )( x1 + x 2 − 3x1 x 2 − 3) = 0


⇒ There exists an x ∈ A1 such that x R y or there
exists an x ∈ A2 such that x R y 3 − x1
⇒ x1 = x 2 or x 2 =
1 − 3x1
⇒ y ∈ R(A1) or y ∈ R(A2)
⇒ y ∈ R(A1) ∪ R(A2) 3 − x1
i.e., f(x1 ) = f(x 2 ) ⇒ x1 = x 2 or x 2 =
Thus, R(A1 ∪ A2) ⊆ R(A1) ∪ R(A2)   (2) 1 − 3x1

From (1) and (2) we have, 3 − x1


⇒ Function is not one to one as x1 , are both
R(A1∪A2) = R(A1)∪R(A2) 1 − 3x1
mapped onto the same image onto:
105. Let ( x, y ) ∈ (S ∪ T) o R
3x 2 + 3x − 4
⇒ There is an z∈B such that (x, z)∈R and (z, y) ∈ Let y =
3 + 3x − 4x 2
S∪T
⇒ x 2 (3 + 4y ) + x (3 − 3y ) − 4 − 3y = 0
⇒ There is an z∈B so that (x, z)∈R and (z, y)∈S or
(z, y)∈T If f(x) is to be onto, it has to take all real value for real x.
⇒ There is an z∈B so that (x, z)∈R and (z, y)∈S For real x, we should have
Or (x, z)∈R and (z, y)∈T
(3 − 3y ) − 4 (3 + 4y )(−4 − 3y ) ≥ 0
2

⇒ (x, y)∈ S o R or (x, y)∈ T o R


⇒ 57 + 74y + 57y 2 ≥ 0
⇒ ( x, y ) ∈ (S o R ) ∪ (T o R )
74
Thus, (S ∪ T) o R ⊆ (S o R ) ∪ (T o R )   (1) ⇒ y2 + y +1 ≥ 0
57
Let ( x, y ) ∈ (S o R ) ∪ (T o R ) 2
 37  1880
⇒ ( x, y ) ∈ (S o R ) or ( x, y ) ∈ (T o R ) ⇒ y +  + ≥ 0 which is true.
 57  3249
⇒ There is an z1∈B so that (x, z1)∈R and (z1, y)∈Sor Hence f is onto
there is an z2∈B so that (x, z2)∈R and (z2, y)∈T
Thus f is onto but not one to one.
⇒ There is an z1∈B so that (x, z1)∈R and (z1, y)∈S
∪ T or there is an z2∈B so that (x, z2)∈R and (z2, 107. f ( x ) = log x 8 ⇒ We must have x ≠ –1, 0, 1
y)∈S∪T
⇒ ( x, y ) ∈ (S ∪ T) o R 108. (0.5)
log 3 log 1
5
(x 2
− 45 ) ≥1
Thus, (S o R ) ∪ (T o R ) ⊆ (S ∪ T) o R   (2) log 3 log 1 (x 2
− 45 ) 0

From (1) and (2) we have 1 5 1


 2  ≥  
2
(S ∪ T) o R = (S o R ) ∪ (T o R )
 4
106. Let f(x1) = f(x2) \ log3 log 15  x 2 −  ≤ 0
 5
3x12 + 3x1 − 4 3x 22 + 3x 2 − 4 4
⇒ = 
3 + 3x1 − 4x12 3 + 3x 2 − 4x 22 log 15  x 2 −  ≤ 30
 5

=
( )
3 x12 − x 22 + 3 ( x1 − x 2 )
(using dividend rule)
 4
log 15  x 2 −  ≤ 1
(
3 ( x1 − x 2 ) − 4 x − x 2
1
2
2 )  5
1

( )(
⇒ 3x12 + 3x1 − 4 3 ( x1 − x 2 ) − 4 x12 − x 22 ( )) x2 -
4 1

5  5 
( ) ( )
= 3 + 3x1 − 4x12 3 x12 − x 22 + 3 ( x1 − x 2 ) x2 ≥ 1
|x| ≥ 1
On simplification we have
Functions and Graphs  1.79

109. ex - log |x| = 0 20 t ≤ 30


⇒ ex = log |x| 114. c(t) = 
20 + 75(t − 30) t > 30
let y = ex, y = log |x|
115. Talk time = 40 m 30 s
y = ex = 41 m rounded off
First 30 minutes Rs 20
Next 11 minutes Rs 11 × 0.75 = Rs 8.25
x=1 \ Total charges = Rs 28.25
loge|x| 116. 0 < t < ∞
20 ≤ c(t) < ∞

117. Obviously f(x) is an even periodic function.


The number of points of intersection is one
 
110. x3 (x + 1) (x – 2) ≥ 0  2p 2p.5 
Period of f(x) = LCM  , 
if x3 ≥ 0 and (x + 1) (x – 2) ≥ 0 or x3 ≤ 0 and  6p 5  [6p] 
 
(x + 1) (x – 2) ≤ 0
For the first one, x ≥ 0 and x lies beyond –1 and 2 or  2p 2p.5   2p 5p  10p
= LCM  ,  = LCM  ,  =
x ≥ 2.  3 18  3 9 3
For the second one, x ≤ 0 and x lies between –1 and 2 118. (f o g)x = sin(6x + 9).cos(4x + 6)
or –1 ≤ x ≤ 0. \ x ∈ [-1, 0] ∪ [2, ∞).
1
= sin (10x + 15) + sin(2x + 3)
111. Statement 2 – f(x) = coslx + sinlx is periodic 2
⇒ l must be rational.  2p 2p 
It is periodic and the period is LCM of  , 
\ Statement 2 is true  10 2 
2p
 p p  p  = =p
f  x +  = sin  + x  + cos  + x  + 5 2
 2 2  2 
(gof)x = 2sin3x . cos2x + 3
= |cosx| + |sinx| + 5 = f(x)
p = sin 5x + sinx + 3 is also periodic
\ f(x) is periodic with period
,
2  2p  2p
\ Statement 1 is true but not follows from Period = LCM  ,2p = = 2p
Statement 2 5  1
\ option (b) {
119. Given f(x) = a + 2a 3 − 3a 2 [ f(x − b) − c ]
112. Statement 2 is true
}
1
+3a (f(x − b))2 + c2  − (f(x − b))3 − c3 
3
f(x) = logex - e-x is continuous for x > 0
f(1) < 0 and f(2) > 0 ∴ (f(x) - a)3 = 2a3 - 3a2[f(x - b) - c]
Using Statement 2, we infer that f(x) vanishes at some + 3a[(f(x - b))2 + c2 ] - [f(x - b)3 - c3]
point between x = 1 and x = 2 = (a + c)3 - [f(x - b) - a]3
Choice (a) ∴ (f(x) - a)3 + (f(x - b) - a)3 = (a + c)3  —(1)
113. Statement 2 is true Put x = x + b ⇒
Statement 2 is also true (f(x + b) - a)3 + (f(x + b - b) - a)3 = (a + c)3
But, the function in Statement 1 is not continuous (f(x + b) - a)3 + (f(x) - a)3 = (a + c)3  —(2)
at x = 0
(2) - (1) ⇒
Choice (b)
(f(x + b) - a)3 - (f(x - b) - a)3 = 0
1.80  Functions and Graphs

i.e., (f(x + b) - a)3 = (f(x - b) - a)3 Additional Practice Exercise


f(x + b) - a = f(x - b) - a
121. (i) Fig (ii) and Fig (iii) → are not graphs of functions.
f(x + b) = f(x - b)
For a given x, the value of y (= f(x)) must be unique
Put x = a ⇒ f(a + b) = f(a - b) in the case of a function. This is not so for graphs in
Put x = a + b ⇒ f(a + 2b) = f(a) figures (ii) and (iii). Fig (i) corresponds to the graph
of a function. Domain is (-∞, 6] and range [-4, 10].
∴ f(a + 2b) ≠ f(2a + b)
Fig. (iv) corresponds to the graph of a function.
Again taking x = x + b
Domain is [-5, ∞) and range is [-2, ∞)
⇒ f(x + b + b) = f(x + b - b)
(ii)
∴ f(x + 2b) = f(x)
Even function
∴ f(x) is periodic with period 2b. Domain = R
Fig (i) (symmetry
Range = (-∞, 4]
120. (a) Period of x - {x} is 1 about y-axis)
Period of |sinpx| is 1 Odd function Domain = R
1 Fig(ii) (symmetry
Period of |sin2px| is Range = [-2,2]
2 about origin)
 1 Even function
∴ period of f(x) is the LCM of 1,1,  = 1 Domain = R
 2 Fig (iiii) (symmetry
Range = (-∞, 3]
Also f(x) is neither even nor odd about y-axis)
(b) x + sinx is non-periodic. Its graph is given below Periodic func- Domain = R
Fig (iv) tion with period
Range = [0, 1]
y=x+1 4
y=x
(iii)
y=x−1
π
π 5 1 2 3 −5
−2
2
f(x) = x + | x – 1| + | x – 2 | + | x – 3 |
Region R1 : (– 5, 1)
Region R2 : (1, 2)
Region R3 : (2, 3)
Region R4 : (3, 5)
Also f(-x) = -x + sin(-x)
Region R1
= -(x + sinx) = -f(x)
f(x) = x + 1 – x + 2 – x + 3 – x = 6 – 2x
∴ f is an odd function
Region R2
(c) We know that sinnx, cosnx are periodic with pe-
riods p or 2p depending on whether n is even or f(x) = x + x – 1 + 2 – x + 3 – x = 4
odd. Region R3
∴ period of f(x) is p. f(x) = x + x – 1 + x – 2 + 3 – x = 2x
Also f(-x) = f(x)
⇒ f is even f(5) = 14
f(– 5) = 16
(d) cos (cosx) is periodic with period 2p
-1
6
∴ cos-1(cos2px) is periodic with period 1 4
Also f(-x) = cos-1(cos(-2px)) = cos-1(cos2px)
= f(x) –5 –4 –3 –2 –1 1 2 3 4 5
∴ f is even
Functions and Graphs  1.81

Region R4 [9 - x] + [ x - 1] - 8 > 0
f(x) = x + x – 1 + x – 2 + x – 3 = 4x – 6 9 + [ - x] + [x] - 1 - 8 > 0
f(1) = 1 + 1 + 2 = 4 [-x] + [x] > 0
f(2) = 2 + 1 + 1 = 4 But, [-x] + [x] can be either 0 or -1 according as x is
f(3) = 3 + 2 + 1 = 6 an integer or non integer. Hence, there is no value of
x between 1 and 9 satisfying the inequality.
Range of the function is [4, 16]
Case iii x > 9
(iv) Since the function is periodic the graph repeats
itself. [x - 9] + [x - 1] - 8 > 0
[x] - 9 + [x] -1 - 8 > 0
y
2[x] > 18 ⇒ [x] > 9
x ≥ 10
y = 4 (4,4)
y = 2x y = −2x + 12 when x =1,[|x - 9|] + [|x - 1|] - 8
=8+0-8=0
x
–6 0 2 4 (6, 0) 12 ⇒ x = 1 is not in the domain
when x = 9
122. (i) We must have [|x - 9|] + [|x - 1|] - 8 = 0 + 8 - 8 = 0
1 – | x | ≥ 0 and 2 – | x | > 0  —(i) ⇒ x = 9 is not in the domain
or 1 – | x | ≤ 0 and 2 – | x | < 0  —(ii) combining all the above results,
(a) | x | ≤ 1 and | x | < 2 ⇒ x ∈ [ –1, 1] domain of the function is
(b) x | ≥ 1 and | x | > 2 ⇒ | x | > 2 (-∞, 0] ∪ [10, ∞)
Combining, the domain of definition of the 123. (i) f(x) = x + sin x
function is (−∞, −2) ∪  −1,1 ∪ (2, ∞ ) f(– x) = – x + sin (– x) = – x – sin x = – f(x)
(ii) We must have x – [ x ] > 0 f(x) is odd.
It is clear that x cannot be 0, ± 1, ± 2, …
2 − x, x < 2
For all other values of x, x – [ x ] > 0 (ii) f(x) = 
 x − 2, x ≥ 2
Domain of f is R excluding 0, ± 1, ± 2, …
The graph of f(x) is sketched as shown.
(iii) We must have x − x 2 > 0 and 3x − 1 − 2x 2 ≥ 0
y
2
⇒ x (1 - x) > 0 and 1 + 2x − 3x ≤ 0
⇒ x should lie between 0 and 1 and x should
satisfy 0.5 ≤ x ≤ 1. Combining, the domain of 0 2 x
f(x) is 0.5 ≤ x < 1.
(iv) [|x - 9|] + [|x - 1|] - 8 ≠ 0
and [|x - 9|] + [|x - 1|] - 8 > 0 f(x) is neither odd, nor even
Case i x < 1 (Observe that it is symmetrical about the line
x = 2)
Then, [9 - x] + [1 - x] - 8 > 0
(iii) 
⇒ 9 + [-x] + 1 + [-x] - 8 > 0
y
2[-x] > - 2
[-x] > - 1
-x ≥ 0 x
0
⇒ x ≤ 0
Case ii 1 < x < 9
1.82  Functions and Graphs

The graph of f(x) is sketched. f(x) is clearly an odd ∴ R is not reflexive.


function y=1-x;z=1-y⇒z=1-y
For, f(– x) = (– x)2 = x2, = 1 - (1 - x) = x
– x > 0 = – (– x)2 = – x2, – x ≤ 0 ∴ R is not transitive.
 x , x < 0
2

or f(− x) =  2 = – f(x) (iii) x2 + 3x = x2 + 3x ⇒ R is reflexive


− x , x ≥ 0 x2 + 3x = y2 + 3y ⇒ y2 + 3y = x2 + 3x
(iv) ⇒ R is symmetric
y
If x2 + 3x = y2 + 3y and y2 + 3y = z2 + 3z, it is clear
that x2 + 3x = z2 + 3z
x ⇒ R is transitive.
0
125. The ordered pairs to be added to R are {(1, 1), (2, 2),
f(x) is odd (3, 3), (3,1), (2, 3), (1, 2), (2, 1)}
126. {x} = x - [x]
f(x) is odd
Suppose x ∈(-3, -2) ⇒ [x] = -3
(v) f (– x) = – 2x + x2
{x} = x - (-3) = x + 3 where x∈(-3, -2)
It is neither odd nor even
(vi) f(x) = x × 3x
Hence in (-3, -2) }, f(x) = { x} = x+3

−x Similarly, in (-2, -1)}, f(x) = x+2


f(− x) = (− x) × 3− x =
3x
in (-1, 0), f(x) = x +1
It is neither odd nor even.
in (0, 1), f(x) = x
(vii)
y The graph of f(x) = {x} is as given below.
3
y

x
–4 0 4

−3 −2 −1 0 1 2 x

From the graph of f(x), we note that f(x) is an


even function.
Domain of f(x) is R
124.
Range of f(x) is [0, 1]
(i) x + y = 3a ⇒ y + x = 3a
Clearly, f(x) is periodic with period 1.
\ R is symmetric
x + x ≠ 3a for all positive integers x. 1
127. We note that x >
\ R is not reflexive 4
x + y = 3a; y + z = 3b; x + z need not be a multiple 1 
⇒ Domain of f(x) is  , ∞ 
of 3. 4 
Eg: x = 2, y = 1, z = 5 Now,
\ R is not transitive  1 1
f(x) = log 1  x −  + log 4 ( 4x − 1) 
2

(ii) y = 1 - x ⇒ x = 1 - y  4  2  
4

∴ R is symmetric  1 1   1 
2
1 = log 1  x −  + log 4  42  x −  
x ≠ 1 - x for all real x, except for x =  4 2
2 4   4  
Functions and Graphs  1.83

 1 1 Put x = 2 in (1)
= log 1  x −  + log 4 (16) f(4) = 2f(2) - f(3)
4
4 2
= 2 × 1 - 5 = -3
1  1
+ × 2log 4  x −  Put x = 3 in (1)
2  6
f(5) = 2f(3) - f(4) = 2 × 5 - (-3) = 13
 1  1
= log 1  x −  + 1 − log 1  x − 
 4   4 130. g o f (x) = (f(x))2 + 2 f(x) - 5
4 4
= 25x2 + 50x + 19
(since logxN = - log 1 N )
x
(f(x))2 + 2f(x) = 25x2 + 50x + 24
1 = (5x + 4) (5x + 6)
f(x) = 1 for all x >
4 = (5x + 4) [(5x + 4) + 2]
Range of f(x) is {1} ⇒ f(x) = 5x + 4
128. x + 2y ≥ 1
131.
x y −4 −3 −2 −1
+ ≥1
1 1
2 -∞ < x < -4 → f(x) = -4 - x - 3 - x - 2 - x - 1 - x
= -10 - 4x
4x – 5y + 20 = 0 -4 < x < -3 → f(x) = x + 4 - 3 - x - 2 - x - x - 1
y B
= -2 - x
C 5x + 4y – 20 -3 < x < -2 → f(x) = x + 4 + 3 + x - 2 - x - 1 - x
=4
-2 < x < -1 → f(x) = x + 4 + 3 + x + 2 + x - 1 - x
D
= 2x + 8
O x
x > -1 → f(x) = x + 4 + x + 3 + x + 2 + x + 1
x + 2y – 1 = 0 = 4x + 10
A
y

5x + 4y ≤ 20
x y
+ ≤1 4
4 5
x
-4x + 5y - 20 ≤ 0 −4 −3 −2 −1
-4x + 5y ≤ 20
x y
+ ≤1
−5 4
Since, x, y > 0, the region is in the first quadrant. Least value of f(x) = 4
The region is in the first quadrant bounded by 132.
ABCDA.

129. f(x + 2) = 2f(x) - f(x + 1)  —(1)


Put x = 0 in (1)
f(2) = 2f(0) - f(1) = 4 - 3 = 1
O
Put x = 1 in (1) x=
7 x=5
2
f(3) = 2f(1) - f(2) = 2 × 3 - 1 = 5
1.84  Functions and Graphs

y = x2 - 7x + 15 p
2 2 f3 (x) = tan (3x + 2) is periodic with period
 7 49  7 11 3
=  x −  + 15 − = x −  + f4 (x) = x2 – x + tan x is not a periodic function
 2 4  2 4
When x = 5 → f = 25 - 35 + 15 = 5 137. f(x1) = kx1, f(x2) = kx2
In [5, ∞), from the graph, we note that f(x) is f(x1 + x­2) = k(x1 + x2) = f(x1) + f(x2)
one - one in [5, ∞) and as x→ ∞, f(x) → ∞
138. A constant function is one to one if and only if the
Therefore, f(x) is on to also domain contains only one element.
i.e., A = {a}
x x
133. f(x) = 1 - sin2 + sin A constant function is onto if and only if the co
4 4
domain contains exactly one element (i.e.,) if and only
 x x if B = {b}
= - sin2 − sin  + 1
 4 4
Hence a constant function is bijective if and only
  x 1  1 
2
if A = {a} and B = {b}
= -   sin −  − +1
  4 2  4  139. We must have x - 1 ≥ 0.
2
5  x 1 Note that (x2 + x + 1) is always positive combining,
= −  sin −  the domain is [1, ∞)
4  4 2
5 140. The positive integral solution of 2x + y = 7 are
Maximum f(x) = x = 1, y = 5; x = 2, y = 3, x = 3, y = 1
4
5  1 5 9
2 \ The relation R = {(1, 5), (2, 3), (3, 1)}
Minimum f(x) = − −1 −  = − = −1
4  2 4 4 \ Domain of R = {1, 2, 3}
 5 Range of R = {1, 3, 5}
Range of f(x) =  −1, 
 4 x −1, x < 0
141. f(x) = , x ≠ 0 ⇒ f(x) = 
x+3 |x| 1, x > 0 
134. y = ⇒ yx – 3y = x + 3
x−3 This function is not injective.
yx – x = 3 + 3y × (y – 1) x = 3 + 3y
3 + 3y 142. As g: B → C is onto, for every c ∈ C there is a b ∈ B
x= so that g(b) = c
y −1
As f: A → B is onto for each b ∈ B there is an a ∈ A
y can have any value except 1.
such that f(a) = b
∴ R – {1}. Combining, we have for every c ∈ C there is an a ∈ A
135. f(x)= -x + 8x - 13 is a quadratic expression with
2 such that of (f(a)) = c
a = -1 < zero, b = 8 and c = -13. ⇒ (g o f ) : A → C is onto
This expression attains maximum when
b 143. Given that g o f : A → C is one to one.
x= − = 4 \ maximum value = f(4) = 3 Thus we have
2a
Or (g o f )(a ) = (g o f )(b) ⇒ a = b 
 —(1)
f(x) = -x + 8x - 13 = -[x - 8x + 13]
2 2
or a ≠ b ⇒ (g o f )(a ) ≠ (g o f )( b)
= -[(x - 4)2 - 3] Suppose that, if possible f is not one to one.
= 3 - (x - 4)2 Then there exist a, b, (b ≠ a) ∈ A such that f(a) = f(b)
Maximum value of f(x) is 3, when x =4.  –(2)
As a, b, b (≠ a) ∈ A and g o f is one to one,
136. f1 (x) = x – [x] is a periodic function of period 1
f2 (x) = log (2 + cos 2x) is periodic with period
2p
=p (g o f )(a ) ≠ (g o f )(b) ⇒ g (f (a )) ≠ g (f (b))
2
Functions and Graphs  1.85

⇒ g (f (a )) ≠ g (f (a )) a contradiction 147. sin x − 1 ≠ 0


Hence f has to be one to one sin x ≠1  —(i)
sin x − 1 ≥1
144. Any line parallel to x – axis is of the form y = c.
Equation of the lines parallel to the x – axis between - 1 ≤ sin x - 1 ≤ 1
x 0 ≤ sin x ≤ 2
which the graph of y = lies are given by y =
1 + x2 ⇒ 0 ≤ sin x ≤ 1  —(ii)
minimum of y and y = maximum of y (i.e.,). In other
from (1) and (2)
x
words, we have to find the range of y = sin x ∈ [0, 1)
1 + x2
sinx = 0 → x = np
x p
y = ⇒ x2 y − x + y = 0 sinx = 1 → x = np +(-1)n
1 + x2 2
⇒ Domain of f(x) is
1
x is real if 1 − 4y 2 ≥ 0 i.e., if y 2 ≤ x = np (n any integer)
4
−1 1 sin x ≤ 1
i.e., if ≤y ≤
2 2  p
x ∈ 0, 
\ The equation of the required lines are  2
General solution is
−1 1
y = and y = x = np + (-1)n a
2 2
 p
where, a ∈ 0, 
145. The function f1 (x) = 4 − x is defined if 4 – x ≥ 0  2
i.e., if x ≤ 4 ⇒ Domain of f1 (x) = (−∞,4  3x + 5
148. y =
8x + −3
The function f2 (x) = x − 2 is defined if x – 2 ≥ 0
8y x - 3y = 3x + 5
i.e., if x ≥ 2 ⇒ Domain of f2 (x) = [2, ∞ ) x (3 - 8y) = - 3y - 5
As f(x) = f1 (x) + f2 (x) we have, Domain of f(x) 3y + 5
x=
= [Domain of f1(x)] ∩ [Domain of f2(x)] 8y − 3
= (−∞,4 ] ∩ 2, ∞ ) = [2,4] 3x + 5
Inverse of f is
To find the range 8x − 3
( )
2
Consider (f(x)) =
2
4−x + x−2 3
+1
 1  3x + 1
149. f(x) + f   = + x
= 2 + 2 1 − ( x − 3)
2
 x  3x − 1  3 
 x − 1
We observe that greatest value of 
(f(x))
2
= 2 + 2 1 − 0 = 4 and it occurs when x = 3 (3x + 1) (3 + x)
= ×
3x − 1 (3 − x)
Least value of (f(x)) = 2 + 2 1 − 1 = 2 and it occurs
2

when x – 3 = 1 (i.e.,) when x = 4. 9x + 3 + 3x 2 + x


=
9x − 3 − 3x 2 + x
We have, the greatest value of f(x) = 2 and the least
value of f(x) = 2 3x 2 + 10x + 3
=
(−3x 2 + 10x − 3)
\ Range of f(x) = [ 2,2]
3x 2 + 10x + 3
146. The graph of the reflection of y = f(x) about y = x is =0
−3x 2 + 10x − 3
the graph of the inverse function of y = f-1(x).
Here f-1(x) = (1 + x)2 = g(x) −1
x = - 3,
3
1.86  Functions and Graphs

y
x + y x − y
150. f(x) + f(y) = 2f  f
 2   2 
Setting x = 0, y = 0
2f(0) = 2 [f(0) ]2
x
f(0) = 0 or 1 o 1 2

151. Putting x = 0 = y
2f(0)
f(0) = [2, 10] is one to one.
1 − (f(0))2
Clearly, f(0) cannot be equal to 1 I were f - 1 (x) exists f - 1 (6) means, we have to find,
that x0 whose image is under f is 6
We have
f(0) - [f(0)]3 = 2 f(0) or x 20 − 3x 0 + 2 = 6

f(0) + [f(0)]3 = 0 ⇒ f(0) = 0 x 20 − 3x 0 − 4 = 0 ⇒ x 0 = 4


[(f(0)]2 ≠ -1 ] (x0 = -1 is not acceptable)
Replacing x by - x is 155. Since x ∈ R is the domain for both f and g, both fog
f(x) + f(y) and gof are defined
f(x + y) =
1 − f(x)f(y) fog(x) = a(cx + d) + b
f(x) + f(− x) gof(x) = c(ax + b) + d
we get f(0) =
1 − f(x)f(− x) ⇒ acx + ad + b ≡ acx + (bc + d)
⇒ ad + b = bc + d
⇒ f(x) + f( - x) = 0, minimum f(0) = 0
There are infinitely many sets of values of a, b, c,
⇒ f (x) is an odd function
d satisfying the above relation.

}
1

152. f(2 + x) -a = {1 - [f(x) - a ]


4 4
156. Consider (b)
when x < 3, f(x) = 3 - x + 2x = x + 3

[f(2 + x) - a]4 = 1 - [f(x) - a]4
when x ≥ 3, f(x) = x - 3 + 2x = 3x - 3
[f(2 + x) - a]4 + [f(x) - a]4 = 1  —(1)
⇒ choice (b)
(1) is true for all x
Replace x by (x + 2) in (1) 157.
y y
[f(x + 4) - a] a] + [f(x + 2) - a] = 1 
4 4 4
—(2)
(1) and (2) gives 1
1
[f(x) - a] = [f(x + 4) - a]
4 4
0 x 0 x
−1 −2 −1
⇒ f(x + 4) - a = f(x) - a
⇒ f(x + 4) = f(x)
y = f(x) y = g(x)
153. Let the linear function be
f(x) = ax + b 1 − x − 2, −1 < x < 0
Let f(- 2) = 0 and f(2) = 4 ⇒ f(x) = x + 2 fog(x) = 
 −(1 − x), 0 < x < 2
Let f(- 2) = 4 and f(0) = 0 ⇒ f(x) = - x + 2
The two linear function as are f(x) = (x + 2) −1 − x, −1 < x < 0

x − 1, 0 ≤ x < 2
and f(x) = (2 - x)

154. 158. f(x + 2) = 2f(x) - f(x + 1)  —(1)


Since f(x) Is defined in [2, 10], from the figure we Put x = 0 in (1)
observe that the mapping defined by f(x) from → f(2) = 2f(0) - f(1) = 4 - 3 = 1
Functions and Graphs  1.87

Put x = 1 in (1)
2y ± 4y 2 − 4
→ f(3) = 2f(1) - f(2) ⇒ ex =
2
=2×3-1=5
=y± y2 − 1
Put x = 2 in (1)
→ f(4) = 2f(2) - f(3) Since both y + y 2 − 1( )
( )
= 2 - 5 = -3
and y − y 2 − 1 are > 0,
Put x = 3 in (1)
→ f(5) = 2f(3) - f(4) (
x = loge y ± y 2 − 1 )
= 10 - (-3) = 13

159. Domain of f(x) is R


⇒ g(x) = log x ± x 2 − 1 ( )
g(x) is not defined for values of x for which
162. f(x) = (sinx) {2 sin2x cosx}
1 + [x] = 0 or when [x] = -1 or when x∈[-1, 0)
= (2 sinx cosx) sin 2x
Domain of f(x) + g(x) is R - [-1, 0)
= sin2 2x
160. ≥ 0 for all real x
y
choice (a)
3
163. The graph of y = x -[x] is as shown below

−3 O 1 4 x 

   2     [

y = f(x)

y When x is an integer, x - [x] = 0


Hence, f(x) = 0 when x is an integer
x → [x] as x tends to an integer.
O 3 x x 1
As x → 1, →
1+ x 2
y = g(x)
 1
Hence, the range of f(x) is 0, 
 2
Domain of g(x) is (3, ∞) and the range of f(x) is [0, 3]
⇒ gof(x) is not defined. 164. We must have
Domain of f(x) is [-3, 4] and the range of g(x) is (13, 2
(x − 2)

∞) ⇒ fog(x) is not defined. 9 x + 27 3 − 219 − 32(x −1) ≥ 0


⇒ choice (d) (3x)2 + 32 (x-2) - 219 - 32x -2 ≥ 0
32x 32x
ex + e− x 32x + − 219 − ≥0
161. Let y = 81 9
2
 1 1  2x
1
x
2y = e + x 1 + 81 − 9  3 ≥ 219
e
73
⇒ (ex)2 - 2yex + 1 = 0 × 32x ≥ 219
81
1.88  Functions and Graphs

32x ≥ 3 × 81 = 35 −p  p −p  p
2x ≥ 5 ⇒ 1 +  ≥ − x (1 − x ) ≥ 1+   —(4)
6  6  3  3
5 (3) + (4) gives
x≥
2
5  3 p p 1 p p
⇒ Domain is  , ∞  − 1 +  ≥ cos x − x(1 − x) ≥ − 1 + 
2  2 6  6  2 3  3

k2 x Range of f(x) is
( x + 1) k2 x 1 p  p 3 p p 
165. f f(x) = =  − 1 +  , − 1 +  
+1 (
kx k + 1) x + 1  2 3  3 2 6 6  
x +1
Since f f(x) must be = x y 10 x − 10 − x
169. =
1 10 x + 10 − x
⇒ k + 1 = 0, k = -1
y + 1 10 x
166. [x + 2 ] ≠ 0 = = 102x
1 − y 10− x
[x] + 2 ≠ 0
[x] ≠ -2 1 + y 
2x = log10 
 1 − y 
x should not belong to [-2, -1)
Domain of f is(-∞, -2) ∪ [-1, ∞) 1 1 + y 
x= log10 
2  1 − y 
167. y = (x + 2)2
Equation of the reflection curve in y = x is obtained 1 1 + x
⇒ g(x) = log10 
by interchanging x and y in y = (x + 2)2 2 1 − x 
⇒ reflection curve is
 x2 0< x <1
x = (y + 2)2 
y+2= x 170. (a) f(x) =  x 2 − 1 1< x < 2
 2
y= x − 2, x ≥ 0 x − 2 2 <x< 3
f(x) = is not periodic
Since x is always ≥ 0
(a) is false
p p
168. As x increases from to , cos x decreases from  −3   3   3  3 
6 3 (b) f   + f   =  − 2  +  + 1 = 2
 2  2 2  2 
3 1
to
2 2 (b) is true
p p p p (c) f o g(4) = 2, g o f(6) = 1
⇒ cos > cos x > cos x∈  , 
6 3 6 3 f o g(4) - g o f(6) = 1
(c) is true
3 1
⇒ > cos x >  —(1) (d) Range of f(x)
2 2
p p is 2 − 32 + 42 , 2 + 32 + 42 
Again, ≤ x ≤  —(2)  
6 3
i.e., [-3, 7]
p p
1+ ≤1+x ≤ 1+  — (3) (d) is true
6 3
x1 + 1 x 2 + 1
Therefore, 171. Let x1, x2 ∈ the domain of f =
2x1 + 1 2x 2 + 1
p p p  p if 2x1 x2 + 2x2 + x1 + 1
1 +  ≤ x ( 1+ x) ≤ 1+ 
6  6 3  3 = 2x1 x2 + 2x1 + x2 + 1
Functions and Graphs  1.89

⇒ if x2 = x1 f(x) = cosx : 0 < x < 1


⇒ Statement – 2 is true cos(x -1) : 1 < x < 2
Consider Statement – 1 cos(x - 2) : 2 < x < 3
1
As x → ∞, f(x) → In the interval : (-∞, 0)
2
f(x) is one one but not onto f(x) = cos(x + 1) : -1 < x < 0
⇒ Statement is false = cos(x + 2) : -2 < x < -1
Choice (d) ⇒ f(x) is periodic with period 1
Choice (a)
172. Statement 2 is true
Consider Statement 1 176. Statement 2 is not always true.
2p Infact, the function has to be one to one and onto for
Period of sin 3x is
3 the existence of the inverse.
x Consider Statement 1
Period of tan is 2p
2  1  −25
Note that f   =
2p 2 4
Period of cos5x is
5
⇒ Period of f(x) is 2p y

⇒ Statement 1 is true
1
choice (a) 2
−2 0 3
173. f(x) = ax + b
f(y) = ay + b
f(x) + f(y) a(x + y) + 2b
= The quadratic function
2 2
1
x + y f(x) is minimum at x = and the minimum value
= a +b 2
 2  is
−25
4
x + y
= f  1   −25 
 2  f :  ,∞→  ,∞
2   4 
⇒ Statement 2 is true f is one one and onto
Since f(x) = ax + b represents a straight line not
Statement 1 is true
parallel to the x – axis (since a ≠ 0) f is one one
Choice (c)
Statement 1 is true
Choice (a) 177. Statement 2 is true
Consider Statement – 1
174. Statement 2 is true
max (f(x)) = 2 + max (sin 3x) = 2 + 1 = 3
Consider Statement 1
min (f(x)) = 2 + min (sin 3x) = 2 -1 = 1
Statement 1 need not be always true for, suppose x0 ∈
D1 × D2 and g(x0) = 0, then x0 cannot be in the domain Statement 1 is true

f(x) However, it does not follow from Statement 2


of Choice (b)
g(x)
Choice (d) 178. Statement 2 is true
175. Statement 2 is true ( (
f(x) = cos log x + x 2 + 1 ))
Consider Statement 1
In the interval (0, ∞ ) ( )
log x + x 2 + 1 is an odd function
1.90  Functions and Graphs

( (
But cos log x + x 2 + 1 )) is even

Hence, the asymptotes are
x = ± 3, y = 5
∴ Statement 1 is true and follows from 2 II
Option (a)
184. p r2 = p (3)2 + A = 9p + A
179. Statement 2 is true 9p + A
Consider Statement – 1 r2 = and r is positive only.
p
f(x) < 0 for -3 < x < -1 185. A = p r2 - 9p
f(-1) = 0 27p = p r2 - 9p
and f(x) > 0 for x > -1 r2 = 36 ⇒ r = 6
Since the domain of g(x) is (0, ∞) 186. r = 6
g(f(x)) is defined only for x∈(0, ∞) \ Width = 6 - 3 = 3
Statement 1 is false III
Choice (d)
187. t varies from 0 to ∞
180. Statement 2 is true 80
188. P(t) = corresponding to t = 20
Consider Statement 1 1 + 63e −12.6
When x = 1, f(1) = 2 + loge1 = 2 80
⇒ f-1(2) = 1 = where, e is negligible
1+ e
Statement 1 is true ≈ 80
Choice (a)
80
I 189. ≥ 70
1 + 63e −0.63t
181. We have 8 ≥ 7 + 441 e-.63t
−27 e.63t ≥ 441
y= 2
x −9
log 441
It is clear that as x → ± 3, y → ∞ t≥ years after 1980.
63
⇒ x = ± 3 are asymptotes of the curve.
190. f(x + y) = f(x) + f(y)
16 25 f(1 + 1) = f(1) + f(1) 1 1−x
182. 2 = 1 − 2
y x ∴ f(2) = 2f(1) θ
As x → ± 5 , y → ∞ f(3) = f(2) + f(1) = 3f(1) 2x − x 2
Again,
f(4) = f(3) + f(1) = 4f(1)
25 16 f(5) = f(4) + f(1) = 5f(1) = 50
2
= 1− 2
x y
∴ f(1) = 10  —(1)
As y → ± 4, x → ∞
1− x
Hence, the asymptotes of the curve are tan-1 ⇒
x = ± 5, y = ± 4 2x − x 2

x

= sin-1(1 - x)
183. y = 2 +5  1− x 
x −9 g(x) = 1 + sin-1(1 - x) - tan-1  
 2x − x 2 
x
⇒ y - 5 = ⇒ g(x) = 1, x ∈ [0, 2]
x2 − 9
As y → 5, x → ∞ \(fog) (x) = f(g(x)) = f(1) = 10
Again, as x → ± 3, y → ∞ Since (fog)(x) = 10
Functions and Graphs  1.91

⇒ x-axis is an asymptote of the curve.


3
(f o g)   = 10 and
2 (d) is true.

1 192. (a) is true


(f o g)   = 10 (b) is true
2
clearly f(x) = 2 for 0 < x < 1
3  1 1 1
f   = f 1 +  = f(1) + f   = 10 + f  In 1 < x < 3, graph of f(x) is a straight line passing
2  2 2 2
through (1, 2) and (3, 0)
1 1 1 1 ⇒ f(x) = 3 - x in 1 ≤ x < 3
Again, f  +  = f   + f  
2 2 2 2 In 3 < x < ∞, graph of f(x) is a straight line passing
through (3, 0) and having Slope 1.
1
⇒ f(1) = 2f   ⇒ f(x) = x - 3 in x ≥ 3
2
⇒ (c) is true
1
⇒ f   = 5 From the graph, we note that the range of f(x) is
2
(-2, ∞)
3 ⇒ (d) is false.
Hence f   = 10 + 5 = 15
2
193. x2- x - 2 = (x - 2) (x + 1)
  3  x2 - x - 2 > 0 ⇒ x∈(-∞, -1) ∪ (2, ∞)
g  f    is not defined, since domain of g is
[1, 2]   2   Domain of f is (-∞, -1) ∪ (2, ∞)
g(x) is not defined at x = 2
1 Domain of g is R - {2}
g o f   is also not defined.
2 ⇒ Domain of [f(x) + g(x)] is (-∞, -1) ∪ (2, ∞)
Note that g(3) = 0
191. Clearly, 3 + |x| ≠ 0 for any x∈R f(x)
⇒ Domain of is (-∞, -1) ∪ (2, 3) ∪ (3, ∞)
⇒ Domain of f is R g(x)
(a) is false (b) is true
we have f(-x) = f(x) for all x x−3 1
g(x) = = 1−
⇒ f(x) is an even function x−2 x−2
(c) is true It is clear that g(x) cannot be < 1 for all x in its
1 domain
f(o) =
3
As x → ∞, f(x) → 0 y
 1
Range of f is  0, 
 3
y=1

y
O 1 2 x
1
3 x=2

O x

3
For example, g(o) = >1
(b) is true 2
graph of y = f(x) is as shown below. (c) is false
1.92  Functions and Graphs

we may draw the graph of y = g(x) l(x) = logex 0 < x < 1


1 loge(x - 1) 1 < x < 2
As x → -∞, y → | = 1 -
x−2 and similar case for negative valves of x.
As x → 2, y → ∞
The graph of loge(x - [x]) is as shown below.
y = g(x) passes through (3, 0)
l(x) is periodic with period 1.
As x → ∞, y → 1
range of l(x) is (-∞, 0)
The lines x = 2 and y = 1 are asymptotes of the both (c) and (d) are true.
curve y = g(x)
(d) is true. 196. Domain of f(x) is R - {1}
1
194. Since f(x) is even, f o f(x) = ,x ≠1
1
1−
f(-x) = f(x) (1 − x )
⇒ f(-4) = f(4) = 40 1− x x −1 1
= = = 1−
(
f(-13) = f(13) = 3x 2 − 8 ) x =3 = 19 −x x x
⇒ domain of f o f (x) is R - {0, 1} and
f(11) = f(1) = (2x )x =1 = 2 3
f (−13) − f (11) 19 − 2 17 f o f (-2) =
= = 2
f (13) + f (−11) 19 + 2 21 1
Again, f o f o f (x) = =x
(c) is false, since f(5) = f(-5) = 40  1
1 − 1 − 
 x
and so, f(5) is defined
Domain of f o f o f (x) is the same as that of f(x)
f(x) ≥ 0 for all x∈R
⇒ (c) is true
max f(x) = f(5) = 40
Range of f(x) is [0, 40] ⇒ f o f o f (5) = 5
(d) is true
195. g(x) = x + 2 - (2 + [x])
= x - [x] = f(x) 197. We have
 x2 
-1 ≤ log3   ≤ 1
3
y
1 x2
≤ ≤3
3 3
1 ≤ x2 ≤ 9
−3 −2 −1 0 1 2 3
x x2 ≥ 1 and x2 ≤ 9
O

−3 −1 1 3

-3 ≤ x ≤ -1 or 1 ≤ x ≤ 3
since x > 0, we have 1 ≤ x ≤ 3
⇒ g(x) is periodic with period 1 ⇒ a = 1, b = 3
2
x 0< x <1 Range of cos-1 x is [0, p]
h(x) = x 2 − 1 1< x < 2 ⇒ 0 ≤ f(x) ≤ 1
2
x −2 2 <x< 3 c = 0, d = 1

h(x) is not a periodic function x4 - 3x3 - x + 3 = 0
(b) is false is satisfied by x = 1 and x = 3
Functions and Graphs  1.93

⇒ (a) is true
sin-12x ≥ − p
(b) is false 5
c3 + d3 = 0 + 1 = 1 −1 1
which will be satisfied if ≤x≤
(c) is true 2 2
a2 + b2 + c2 + d2 = 1 + 9 + 0 + 1 = 11 −1 1
Hence, the domain of f(x) is ≤x≤
2 2
(d) is true

198. (a) Domain of sin-1x is [-1, 1] (d) x - 3 > 0 and x - 3 ≠ 1


Domain of {x} is R. But {x} = 0 for x ∈ Z x > 3 and x ≠ 4
1 x2 - 25 > 0
\ Domain of =R−Z
{x} -5 < x < 5
−1
sin x ⇒ Domain is (5, ∞)
\ Domain of is
{x} (a) → (r)
(b) → (q), (s)
[-1, 1] ∩ (R - Z) = (-1, 0) ∪ (0, 1)
(c) → (p)
 p
(b) tan-1x > 0 ⇒ x ∈  0,  (d) → (s)
 2
Also [tan-1 x] > 0 ⇒ [tan-1x] ≥ 1 200. (a) Minimum value of 3 + sin 5x = 2
Maximum value of 3 + sin 5x = 4
⇒ x ∈ p , p
 4 2 ) 1 1
Range of f(x) is  , 
(c) |y| = log10x ⇒ log10x ≥ 0 4 2
⇒ x ≥ 1 (b) Range is (-∞, ∞)
⇒ x ∈ [1, ∞) 2p 2p
(c) f(x) = sin cos x + cos sin x
(d) |y| = ex - logx ⇒ ex - logx ≥ 0 3 3
⇒ ex ≥ logx which is always true  4 p 4p 
+ 2 cos cos x + sin sin x 
Also x > 0  3 3 
∴ x ∈ (0, ∞) 3 1
cos x − sin x
=
2 2
199. (a) 4 + x ≥ 0;  −1 3 
+ 2  cos x − sin x 
4 - x ≥ 0;
 2 2 
x - x2 ≥ 0
 3   1
x ≥ - 4 a n d x ≤ 4 a n d x 2 - x ≤ 0 =  − 1 cos x −  3 +  sin x
⇒ x ∈ [0, 1]  2   2
Hence, the domain of f(x) is [0, 1]  3 
2 2
 1
 − 1 +  3 + 
(b) x2 - 6x + 6 ≥ 1  2   2
⇒ x2 - 6x + 5 ≥ 0 3   1 
=  + 1 − 3  +  3 + 4 + 3 
⇒ x lies beyond 1 and 5 4 
⇒ x ∈ [5, ∞) and (-∞, 1]
= 4 + 1 = 5
(c) -1 ≤ 2 x ≤ 1
Maximum value = 5
−1 1
⇒ ≤x≤
2 2 Minimum value = − 5
p x2 + x + 3 2
sin-12x + ≥0 (d) 2
= 1+ 2
5 x + x +1 x + x +1
1.94  Functions and Graphs

2
 x2 + x + 3 
x2 + x + 1 =  x + 1  + 3 min  2 = 1+0 = 1
 2 4  x + x + 1 
3
Minimum value of (x2 + x + 1) =  11 
4 Range of f(x) is 1, 
Maximum value of (x2 + x + 1) is infinite  3
 2  4 8 (a) → (r)
Hence, max  2  =2× =
 x + x + 1 3 3 (b) → (p)
 2  (c) → (s)
min  2 =0
 x + x + 1  (d) → (q)
 x2 + x + 3  8 11
Hence, max  2  = 1+ =
 x + x + 1 3 3
chapter DIFFERENTIAL
CALCULUS

2
nnn  Cha p t e r O u t l i n e
Preview
sTUDY MATERIAL Extension of the Mean Value Theorem
Introduction Increasing and Decreasing Functions
• Concept Strands (40-42)
Limit of a Function
• Concept Strands (1-4) Maxima and Minima of functions
• Concept Strands (43-47)
Laws on Limits
Convexity and Concavity of a curve
Standard Limits
• Concept Strands (48-49)
• Concept Strand (5)
Continuity of a Function Concept connectors
Types of Discontinuities of a Function • 56 Connectors
Concept of Derivative—Differentiation topic grip
• Concept Strands (6-7a, b) • Subjective Questions (15)
Differentiability of Functions • Straight Objective Type Questions (15)
• Concept Strands (8-9) • Assertion–Reason Type Questions (10)
• Linked Comprehension Type Questions (6)
Derivatives of Elementary Functions
• Multiple Correct Objective Type Questions (3)
Differentiation Rules • Matrix-Match Type Question (1)
• Concept Strands (10-19)
iit assignment exercise
Concept of Differential
• Straight Objective Type Questions (100)
• Concept Strands (20 -21)
• Assertion–Reason Type Questions (3)
Successive Differentiation • Linked Comprehension Type Questions (3)
• Concept Strands (22) • Multiple Correct Objective Type Questions (3)
Higher Order Derivatives • Matrix-Match Type Question (1)
• Concept Strand (23)
Additional Practice Exercise
Tangents and Normals • Subjective Questions (30)
• Concept Strands (24-29) • Straight Objective Type Questions (77)
Mean Value Theorem and its Applications • Assertion–Reason Type Questions (10)
Rolle’s Theorem • Linked Comprehension Type Questions (12)
• Concept Strands (30-31) • Multiple Correct Objective Type Questions (8)
• Matrix-Match Type Questions (3)
L’ Hospital’s Rule
• Concept Strands (32-39)
2.2  Differential Calculus

Introduction

Calculus owes its origin to the English mathematician maximum or minimum, convexity, concavity of the graph
Newton (1642–1727) and the German mathematician of a function and so on. In short, Differential Calculus
Leibniz (1646–1717). Calculus differs from elementary enables us to study the above geometrical properties of
algebra and geometry in the sense that a new operation functions analytically.
is introduced in Mathematics, that of ‘Limits’. As we will In the first part of this chapter, the concept of the limit
see later when we develop the concept of ‘Limit’, it is an of a function is introduced through examples. We then find
operation defined on a continuum unlike the case of opera- a few standard limits and then explain the techniques of
tions of addition, subtraction, multiplication, division and the computation of limits of functions, by using these stan-
exponentiation, defined in algebra, which are defined on a dard limits. Concepts of ‘Continuity’ and ‘Differentiability’
discrete set of real or complex numbers. of functions are then introduced. Derivative of a function
The fundamental objects that we deal with in is defined and derivatives of elementary functions are then
‘Calculus’ are functions. In the chapter unit on ‘Functions found. Applications of derivatives are taken up followed by
and graphs’, we have already dealt with the basic ideas of other important theorems.
functions. Calculus is a powerful tool for solving many ‘real The whole topic is covered in the following order:
life’ problems in engineering. In addition, techniques of
(i) Differentiation rules
Calculus are being used very extensively in all branches of
(ii) Concept of a differential
knowledge like Economics, Biology, Medicine, Behavioural
(iii) Successive differentiation
sciences and so on.
(iv) Tangents and Normals
The three important parts of Calculus are:
(v) Angle between two curves
(i) Differential Calculus (vi) Rolle’s theorem, Mean value theorem and extensions
(ii) Integral Calculus (vii) Cauchy’s theorem, L’ Hospital’s rule for evaluation of
(iii) Differential Equations limits of indeterminate forms
(viii) Increasing and Decreasing functions, Monotonic
The central idea contained in Differential Calculus
functions
is the concept of ‘Derivative of a function’. It is a limiting
(ix) Necessary and sufficient conditions for a function to
operation which enables one to find the rate of change of
have an extremum at a point
one quantity with respect to another. This means that Dif-
(x) Convexity and Concavity of a function, Points of
ferential Calculus comes into play in any situation, where
inflexion
change with respect to a variable is involved. The derivative
of a function helps one to study various geometrical prop- Worked out examples are given to illustrate the basic
erties of functions like intervals in which the function is results contained in each of the above sections. In the end,
increasing or decreasing, points where a function attains its a large number of solved examples is given.

Limit of a function

The concept of the “limit of a function” underlines the vari- common ratio r is numerically less than 1 and we express
ous branches of Calculus. It is therefore appropriate to be- a
this idea symbolically by writing nlim Sn = where, Sn
gin our study of Calculus by investigating limits and their →∞ 1−r
properties. represents the sum of the first n terms of the geometric
We have already encountered the term ‘limit’ when series.
we considered an infinite geometric series (Refer Unit M4 We have also introduced the concept of “limit” in
Sequences and Series). We found that the sum of the series Chapter 1 under the heading ‘Basics of Calculus’, where we
a + ar + ar2 + ar3 + … ∞ as the number of terms is increased dealt with the problems of finding the tangent to a curve
a and the velocity of a moving object. In general, in order
indefinitely will approach the number provided the
1−r to solve certain real life problems, we must be able to find
Differential Calculus  2.3

limits. Now, we already know that physical quantities are Observation


expressed in the form of functions. In what follows, we
study methods for computing limits of functions. The value of the function at x = 3 is (3 × 32 - 4 × 3 - 10) = 5
Let us investigate the behaviour of the function f(x) = which is the limit of f(x) as x tends to 3. This means that,
3x2 - 4x - 10 for values of x near 3. Table 2.1 gives the values in the case of this function, the limit can be evaluated by
of f(x) for values of x close to 3, but not equal to 3. direct substitution. However, not all limits can be evaluated
by direct substitution (we will see it in later examples).
Table 2.1
Definition 1
x f(x) x f(x)
Let f(x) be a function of x. We write,
2.5 -1.25 3.5 12.75
lim f(x) = L
2.6 -0.12 3.4 11.08 x → x0

2.8 2.32 3.3 9.47 and say “the limit of f(x) as x approaches x0 equals L” if we
2.9 3.63 3.1 6.43 can make the values of f(x) arbitrarily close to L (as close to
L as we like) by taking x sufficiently close to x0 but not equal
2.95 4.3075 3.05 5.7075 to x0.
2.99 4.8603 3.01 5.1403 Roughly speaking, this says that the values of f(x) be-
2.999 4.986003 3.001 5.014003 come closer and closer to the number L as x approaches the
number x0 (from either side of x0­) but x ≠ x0. Notice the
2.9999 4.99860003 3.0001 5.00040003 phrase ‘but x ≠ x0’ in the definition of the limit. This means
that in finding the limit of f(x) as x approaches x0, we never
From the above table we see that when x is close to 3 consider x = x0. In fact, f(x) need not even be defined at
(on either side of 3), f(x) is close to 5. In fact, we can make x = x0. The only thing that matters is how f(x) is defined
the values of f(x) as close to 5 as we like by taking x suffi- near x0 (i.e., in a neighbourhood of x0).
ciently close to 3. We express this by saying “the limit of the We consider a few more examples of finding limits of
function f(x) = 3x2 - 4x - 10 as x approaches 3 is equal to functions so as to have the concept of the limit understood
5”. The notation for this, is lim(3x 2
− 4x − 10) = 5. well.
x →3

C o nce p t S t r a n ds
Concept Strand 1 sin x
for values of x near x = 0 on either side. Since is an
sin x x
Find lim , where x is measured in radians. even function, it is enough if we do the computations for
x →0 x
positive values of x near 0. From the table we infer that
 sin x 
Solution lim 
x →0  x 
= 1.

sin x
First, note that the function is not defined at x = 0. sin x
x y x
1.001 x
sin x 0
[when x is put equal to zero, reduces to
which is 1 0.6 0.941070789
x 0 0.5 0.958851077
0.999
called an indeterminate quantity. This is because arithme- 0.4 0.973545855
0.998
tic operations involving division by zero are not defined. 0.3 0.985067355
∞ 0.997 0.2 0.993346654
Similarly, we may encounter indeterminate quantities , 0.1 0.998334166
∞ 0.996
∞ × 0, ∞ - ∞, 0 , 1 , ∞ which are not defined in arithmetic
0 ∞ 0
0.995
0.05 0.999583385
operations]. 0.02 0.999933334
0.994
0.01 0.999983333
sin x 0.993 0.005 0.999995833
We construct the table of values of the function
x x 0.001 0.999999883
–0.2 –0.1 0.992 0 0.1 0.2 0.0001 0.999999998
2.4  Differential Calculus

We can arrive at the above result using a geometric We hasten to add that this does not mean that we are
argument (we do this later in this section). In this example, regarding ∞ as a number. Nor does it mean that the limit
though the value of the function is not defined at x = 0, exists. It simply expresses the particular way in which the
limit exists. limit does not exist.
In general, if a function f(x) is defined on both
sides of a point say x0, except possibly at x0 itself, then
Concept Strand 2 lim f(x) = ∞ means that f(x) can be made arbitrarily
x→x
1 0

Find lim , if it exists. large (as large as we please) by taking x sufficiently close to
x →1 (x − 1)2
x0, but not equal to x0.
Solution
As x comes close to 1, (x - 1)2 comes close to zero and, Concept Strand 3
1 Find lim[ x] where, [ ] represents the greatest integer
consequently, becomes very large. x →2
(x − 1)2
function.
y
Solution
We note that as x approaches 2 from the left, [ x ] ap-
proaches 1 (since [ x ] = 1 for 1 ≤ x < 2) and as x approaches
1 2 from the right, [ x ] approaches 2 (since [ x ] = 2 for 2 ≤
y=
( x − 1)2 x < 3). (refer Figure).
1
x y
O 1

1 1
x (x − 1)2 x (x − 1)2

0.8 25 1.2 25
x
0.9 10 2
1.1 102 O 1 2 3
0.95 4 × 102 1.01 104
0.99 104 1.001 106
0.999 106 1.0001 108 Thus, there is no single number that [ x ] appro-
aches as x approaches 2. Therefore, lim [ x] does not
x →2
We infer from the graph of the function and also from
exist.
1
the table above, that the values of f(x) = can be We indicate the above situation symbolically by writing
(x − 1)2
made arbitrarily large by taking x close to 1. Thus, the lim [ x] = 1 and lim+ [ x] = 2
x → 2− x →2
1 The symbol x → 2- indicates that we consider
values of f(x) do not approach a number and so, lim
x →1 (x − 1)2
only values of x that are less than 2. Likewise, x → 2+
does not exist. indicates that we consider only values of x that are
To indicate this kind of behaviour exhibited by greater than 2.
the function in the above example, we use the notation We now define the left hand limit and right hand limit
1 of a function.
lim = ∞.
x →1 (x − 1)2
Differential Calculus  2.5

Definition 2 from the right, or x approaches x0 through values of x greater


than x0) is equal to L2 if we can make the values of f(x) ar-
We write lim f(x) = L1 and say that the left hand limit of

x → x0 bitrarily close to L2 by taking x sufficiently close to x0 and x
f(x) as x approaches x0 (or the limit of f(x) as x approaches greater than x0.
x0 from the left, or x approaches x0­ through values of x less By comparing Definitions 1, 2 and 3 we infer that
than x0) is equal to L1 if we can make the values of f(x)
arbitrarily close to L1 by taking x sufficiently close to x0 and lim f(x) = L if and only if lim− f(x) = L and
x → x0 x → x0
x less than x0.
lim f(x) = L
Definition 3 x → x0+

We write lim+ f(x) = L2 and say that the right hand limit of (i.e., L1 = L2 = L).
x → x0

f(x) as x approaches x0 (or the limit of f(x) as x approaches x0

C o nce p t S t r a n d
Concept Strand 4 1
When x is very close to 0, but to the right of 0, is
1 x
Find lime x , if it exists.
x →0 of the form ek, where k is a very large positive number.
1
Solution Consequently, e x becomes larger and larger as x comes
1
closer and closer to 0 from the right. We write lim e x
=∞
We note that when x is very close to 0, but to the left of x → 0+

1 1
0, is of the form -k where k is a very large positive (i.e., the right hand limit of e x as x tends to zero does
x not exist). As the right limit does not exist although the
1
number. Consequently, e x
approaches zero as x ap- left limit exists, lime
1
x does not exist.
1 x →0
proaches zero from the left or lim e x
= 0 or the left limit
x → 0−

of the function as x tends to zero is 0.

Limits at Infinity Let f(x) be a function of x defined for all x < x0 where,
x0 is a negative number. Then, we say that lim f(x) = L
Let f(x) be a function of x defined for all x > x0 where x0­ is x→ −∞

a positive number. Then, we say that lim f(x) = L where L where L is a finite number, if f(x) comes closer and closer
x →∞
is a finite number, if f(x) comes closer and closer to L as x is to L as x is made smaller and smaller through negative
made larger and larger through positive values. values.

Laws on Limits

Suppose xlim
→x
f1(x) and xlim
→x
f2(x) exist. Then, (ii) lim (f1 (x) f2 (x)) = lim f1 (x) × lim f2 (x)
0 0 x → x0 x → x0 x → x0

(i) xlim (k1 f1 (x) ± k 2 f2 (x)) = k1 lim f1 (x) ± k2 lim f2 (x) lim f1 (x)
→x x → x0 x → x0 f1 (x) x → x0
0
(iii) lim = provided lim f2 (x) ≠ 0.
x → x 0 f (x) lim f2 (x) x → x0
where k1 and k2 are finite constants. 2
x → x0
2.6  Differential Calculus

( )
lim f2 (x)
x → x0
This result is called squeeze theorem or Sandwich
(iv) xlim (f1 (x))f2 (x) = lim f1 (x) provided theorem or Pinching theorem. The result is illustrated
→x 0 x→x 0
in Fig. 2.1.
lim f1 (x) and lim f2 (x) are not zero simultaneously.
x → x0 x → x0

(v) If lim f(x) exists, < I [


x → x0

( )
n
(a)  lim (f(x))n = lim f(x) where, n is a positive /
x → x0 x → x0 I [ 
integer. I [ 

( )
1
1 n ;
(b) xlim (f(x)) n
= lim f(x) where, n is a positive 2 [
→x 0 x → x0

integer.
Fig. 2.1
If n is even, we assume that lim f(x) > 0.
x → x0

(vi) If f1(x) ≤ f2(x) when x is near x0 (i.e., in a neighbour- (viii) If f(x) is a polynomial or a rational function and x0 is
hood of x0) (except possibly at x0), and the limits of
in the domain of f(x), then lim f(x) = f(x 0 )
both f1(x) and f2(x) exist as x approaches x0, x → x0

lim f1 (x) ≤ lim f2 (x) Functions with this direct substitution property (for
x → x 0 x→x 0
getting limits), are called continuous at x0. We will deal with
(vii) If f1(x) ≤ f(x) ≤ f2(x) when x is near x0 (except possibly this concept in a later section.
at x0) and We give below a few standard limits. These standard
limits and the laws on limits help us to evaluate limits of
lim f1 (x) = lim f2 (x) = L, then lim f(x) = L .
x → x0 x → x0 x → x0 functions.

Standard Limits

sin x Let A and P are points on a circle of radius r centered


(i) lim = 1 (x measured in radians) at O such that ∠AOP = x radians. PT is the tangent P
x →0 x
to the circle meeting OA produced in T (see Fig. 2.2).
xn − an
(ii) lim = na n −1 , n a rational number. Since PT is the tangent at P to the circle, ∠OPT =
x →a x−a
p
 1
n
(or 90o ) .
(iii) nlim 1 + n  = e (e representing the exponential 2
→∞ 

1
number) or lim (1 + x ) x
=e
x →0 P
ex − 1 r
(iv) lim =1
x →0 x x T
O A
a −b x
a x r
(v) lim = log e   (a, b ≠ 0)
x →0 x b
We consider the proofs of the results (i) and (ii) above.
sin x
(i) lim = 1 , when x is measured in radians. Fig. 2.2
x →0 x
Differential Calculus  2.7

We have, area of DOAP < area of sector OAP < area


( )
n
a 1 + h  − an
of DOPT xn − an (a + h)n − a n  a 
= =
x−a (a + h) − a h
1 1 1
⇒ r 2 sin x < r 2 x < PT × r
2 2 2

( ) 
n
an  1 + h − 1
PT  a 
But, from DOPT, = tan x =  —(1)
r h

1 2 1 1 Since h is very small, we can always take h such that


Therefore, we get r sin x < r 2 x < r 2 tan x
2 2 2 h
<1
1 2 a
Dividing throughout by r sin x (which is > 0)
2
( ) . (As h
n
By using the binomial series for 1 + h a <1
x 1 a
1 < <
sin x cos x and n is rational), the expansion

sin x
(1 + h a ) ( a ) + n(n2!− 1) ha n(n − 1)(n − 2) h 3
n 2
⇒ 1> > cos x = 1+ n h + + .......∞
x 2
3! a3

sin x is valid. Substituting the above in (1),


⇒ cos x < <1
x

n h n(n − 1)  h 2  
But the functions cos x and 1 approach 1 as x a  n   +  a 2  + .......∞
n
x −a n
  a  2 
=
approaches zero. By squeezing theorem, x−a h
sin x
lim = 1. n(n − 1) n − 2
x →0 x = na n −1 + a h + terms involving h2 and higher
2!
xn − an powers of h.
(ii) lim = na n −1 , n is a rational number
x →a x−a
Clearly, as x → a, h → 0. We have, therefore,
Let x = a + h where h is very small positive or negative xn − an  n(n − 1) n − 2 
represent a point near x = a. lim = lim  na n −1 + a h + ........ = na n −1
x →a x−a h→0
 2 

C o nce p t S t r a n d
Concept Strand 5  1
n
1
(vii) lim 1 −  (viii)  lim(1 + 4x) x
n →∞  n
Find the following limits: x →0

5
n+9
2x + 5 1 − tan x  1 x 3
−1
(i) lim (ii)  lim (ix) lim 1 +  (x)  lim 2
x →1 3x − 7 n →∞  n x →1
x→
p sin x x 7
−1
4

tan x sin3q Solution


(iii) lim (iv)  lim
x →0 x
q→ 0 sin7q
2x + 5 2 × 1 + 5 −7
(i) lim = = , on substitution.
x
a −1 4
x − 625 x →1 3x − 7 3 × 1 − 7 4
(v) lim (a > 0) (vi)  lim
x →0 x x →5 x−5
2.8  Differential Calculus

1 − tan x 1−1 −1
   −1   
n −n
(ii) lim = = 0, on substitution.  1
p sin x 1 (vii) nlim 1 −  = lim  1 +    
x→ →∞  n  n  
 
n →∞
4

2
1 1
tan x  sin x   1  = −n
=
= lim   −1  e
(iii) lim x →0  x    lim 1 + 
x →0 x   cos x  n →∞ n

 sin x   1 
( )
1 1 4

= lim × lim  (viii) lim(1 + 4x) x


= lim (1 + 4x) 4x

x →0    = 1 × 1 = 1. x →0 x →0
 x  x → 0
 cos x 
4
=  lim (1 + 4x) 4 x  = e 4
1
 sin3q 
 4 x → 0 
sin3q  3q 
  3
(iv) lim = lim × n+9 n 9
q→ 0 sin7q q→ 0  sin7q  7  1  1  1
 7q  (ix) nlim 1 + n 
→∞ 
= lim 1 +  × lim 1 + 
n →∞ 
  n n →∞  n
 sin3q 
lim  = e × 19 = e.

3  3q 
3q→ 0

= 7 ×
 sin7q  x
5
3
−1 x
5
3
−1
5
3
lim (x) lim = lim
7 q → 0  7q  2 2 2
  x →1
x 7
−1
x →1
x 7
−1 7

3 1 3
= × = . x
5
−1 3
3
5

7 1 7 lim
x →1 (x − 1)
ax − 1 = 2 2
(v) lim (a > 0) x −1 7
7

x →0 x lim
x →1 (x − 1)
x log a x log a
e −1 e −1
= lim × log a = log a × lim 5 5 −1
x →0 x log a x log a → 0 x log a ×1 3
3
= 2 2 −1
= log a × 1 = log a. ×1 7
7
(Here, log a means logea)
5 7 35
x 4 − 625 x 4 − 54 = × =
(vi) lim = lim = 4 × 54 −1 = 500 3 2 6
x →5 x−5 x →5 x − 5

Continuity of a Function

We observed that in the section on limits, the limit of a i.e., a function is continuous at x = x0, if and only if the
function as x approaches x0 in a few cases could be found limit of the function as x tends to x0 is equal to the value of
simply by computing the value of the function at x0. [Refer the function at x = x0.
Concept strand 5 (i) and (ii)]. Functions with this property If f(x) is not continuous at x0­we say that f(x) is discon-
are said to be continuous at x = x0. tinuous at x­0­or f(x) has a discontinuity (or singularity) at x0.
In Fig. 2.3, f(x) is continuous at x = x0 while, in Fig. 2.4, f(x)
Definition is discontinuous at x = x0.
Geometrically, we can think of a function that is
A function f(x) is said to be continuous at a point x0 ­, if continuous at a point as a function whose graph has no
lim f(x) = f(x 0 ) break at x0.
x → x0
Differential Calculus  2.9

Y x4 − 1
(ii) f(x) = is not continuous at x = 1 (or f(x) has a
x −1
discontinuity at x = 1)

y = f(x) Here, lim f(x) = 4 × 13 = 4. However, f(1) is not defined.


f(x0) x →1
P
(iii) f(x) = [ x ] where, [ ] represents the greatest integer
function is not continuous at all the integer points
x0 because lim[ x ] where n is an integer does not exist,
O X x→n
as the left limit is different from the right limit.
1
Fig. 2.3 (iv) f(x) =  x 2 , x ≠ 0 is not continuous at x = 0, since
2, x=0
1
lim 2 does not exist. Here, f(0) = 2 (i.e., the value
x →0 x
Y of f(x) at x = 0 exists, but the limit does not exist.
3x 2 + x − 1
(v) f(x) = has discontinuities
(x + 5) (x + 1) (x − 3) (x − 6)
at the points x = -5, -1, 3, 6. (as f(x) is not defined at
these points)
(vi) f(x) = tan x has discontinuities at
p 3p 5p
x = ± , ± , ± ,.....
O x0 2 2 2
X
(vii) f(x) = cosec x has discontinuities at x = np where n is
any integer.
( viii) f(x) = sec x has discontinuities at
Fig. 2.4 p 3p 5p
x = ± ,± ,± ,.....
2 2 2
(ix) f(x) = cot x has discontinuities at x = np, n any integer.
1
A function is continuous at a point x0 if and only if (x) f(x) = e x
is discontinuous at x = 0.
(i) f(x0) is defined (i.e., x­0 is in the domain of f(x))AND
(ii) lim f(x) exists (which means that f(x) must be
x → x0
Right continuity and left continuity of a function
defined on an open interval that contains x0 or f(x) Definition
must be defined in a small neighbourhood of x0 on
either side)  AND A function f(x) is continuous from the right at x = x0 if
(iii) lim f(x) = f(x 0 ) lim+ f(x) = f(x 0 ) and f(x) is continuous from the left at x
x → x0 x → x0

= x0 if lim− f(x) = f(x 0 ).


A function is discontinuous at a point x0 if x → x0

(i) f(x) is not defined at x = x0 OR


x0− x0+
(ii) lim f(x) does not exist  OR
x → x0 x0
(iii) lim f(x) ≠ f(x 0 ) Consider the following examples:
x → x0

Consider the following examples: (i) f(x) = [ x ], where [ ] represents the greatest in-
teger function is continuous from the right for
(i) f(x) = 2x 2 + x - 5 is continuous at x = 1 since
all integer values of x but discontinuous from the
lim f(x) = 2 × 12 + 1 − 5 = −2 = f(1) left at these points, since lim[ x] = n = f(n), but,
x →1
x → n+
2.10  Differential Calculus

lim [ x] = (n − 1) where, n is an integer. (Refer graph This result says that a limit operation can be moved
x → n−
through a function symbol if the function is continu-
in Concept strand 3) ous and the limit exists OR, in other words, the order
(ii) Suppose we consider the function: f(x) = least integer of these operations can be interchanged.
≥x −1
1 − x 
For -3 < x ≤ -2, f(x) = - 2 For example, suppose we want to find limsin  1− x .
x →1
 
-2 < x ≤ -1, f(x) = - 1
sin -1x is a continuous function in its domain.
-1< x ≤ 0, f(x) = 0 0 < x ≤ 1,
Therefore,
f(x) = 1
1 < x ≤ 2, f(x) = 2 ……….……..…………. 1 − x   x − 1
limsin −1
= sin −1  lim
 1− x  x →1 x − 1 
The graph of this function is as shown in Fig. 2.5. x →1
   

Observe that lim− f(x) = n and lim f(x) = n + 1 where 1 p


x→n x → n+ = sin −1   =
n is an integer. i.e., f(x) is continuous from the left for 2 6

all integer values of x. 2. f(x) is said to be continuous in [a, b] (i.e., in


a ≤ x ≤ b) if it is continuous at every point in [a, b]. If
Y f(x) is continuous in [a, b] then the graph of y = f(x)
will not have any break in [a, b].
  It may be that f(x) is not defined for x < a and or
f(x) is not defined for x > b. That is, f(x) is defined
2
only on one side of an end point of the interval.
1
Continuity at an endpoint in this case, means
continuous from the right or continuous from
0
the left.
−3 −2 −1 1 2 3 X
−1

−2
a b
lim f(x) = f(a) lim f(x) = f(b)
x→a+ x→b–
Fig. 2.5

Results 3. If f(x) is continuous in [a, b] and f(a) ≠ f(b), then


f(x) assumes every value between f(a) and f(b). As a
1. If f(x) and g(x) are two functions which are continuous consequence of this result, we have the following:
at x0 (where, x0 is a point in the domains of the two (i) Let k be a number between f(a) and f(b). Then, there
functions), exists a number c in (a, b) such that f(c) = k. This result
(i) k1 f(x) ± k2 g(x) where k1 and k2 are finite is continuous is known as ‘Intermediate value Theorem’.
at x0. (ii) If f(x) is continuous in [a, b] and f(a) and f(b) are of
(ii)  f(x) g(x) is continuous at x0. opposite signs (i.e., f(a) f(b) < 0), there exists a point
f(x) x0 ∈ (a, b) such that f(x0) = 0.
(iii)  is continuous at x0 provided g(x0) ≠ 0.
g(x) 4. (i) Any polynomial function is continuous every-
(iv) If g(x) is continuous at x0 and f(x) is continuous at where, i.e., it is continuous in (-∞, ∞). OR any
x0, then the composite function f o g(x) = f(g(x)) is polynomial function P(x) is continuous for all
continuous at g(x0). This result is sometimes expressed finite values of x.
as ‘a continuous function of a continuous function is (ii) Any rational function is continuous wherever it
a continuous function’. is defined i.e., it is continuous in its domain.
(v) If f(x) is continuous at x = a and lim g(x) = a, then (iii) Root functions, trigonometric or circular
lim f(g(x)) = f(a)
x → x0
functions,inverse trigonometric functions,
x → x0 exponential functions, logarithmic functions
In other words, xlim
→x
f(g(x)) = f lim g(x)
0x→x ( 0
) are continuous at all points in their respective
domains.
Differential Calculus  2.11

Types of discontinuities of a function

sin x sin x y
(i) We know that lim = 1. However, does not
x →0 x x
exist at x = 0.
 sin x
 , x≠0
By redefining f(x) as f(x) =  x
1, x=0

f(x) becomes continuous at x = 0. a x1 x2 x3 x4 b y


O
We say that f(x) has a removable discontinuity at x
= x0 if lim f(x) exists, but f(x0) does not exist. Fig. 2.7
x → x0

2x + 5, x < 1
2

(ii) Consider the function f(x) = 


 x + 4, x > 1 (iii) If lim f(x) is infinite (i.e., lim f(x) does not exist)
x → x0 x → x0

xlim f(x) = 2 × 1 + 5 = 7 and lim f(x) = 1 + 4 = 5.


− +
and f(x0) is not defined, we say that f(x) has an infinite
→1 x →1
discontinuity at x = x0.
Therefore, lim f(x) does not exist. 1
x →1 For example, for f(x) = , f(0) is not defined. The
We say that f(x) has a finite discontinuity at x = 1. x2
above function has an infinite discontinuity at x = 0.
Again, for f(x) = tan x, it has infinite discontinuities
Y y = 2x2 + 5 p 3p 5p
at x = ± , ± , ± ,......
2 2 2
y=x+4 The following examples illustrate the above:
3x + 5, x < −1
(i) For the function f(x) =  2
 x + 1, x ≥ −1
3 lim− f(x) = 3 × −1 + 5 = 2 and lim+ f(x) = (−1)2 + 1 = 2
x →−1 x → −1
2
Also, f(-1) = (-1) + 1 = 2. 2

1
We therefore find that lim f(x) = 2 = f(−1) which
X x →1
O 1 means that f(x) is continuous at x = -1. Since (3x +
5) and (x2 + 1) are polynomials, these functions are
continuous for all x. Hence f(x) is continuous in R.
Fig. 2.6
 x2 – 9
 x – 3 ,0 < x < 3
In general, if a function f(x) is such that lim f(x) = L1
x → x0−
(ii) For the function f(x) =  x + 2, 3 ≤ x ≤ 6


and lim+ f(x) = L2 , where L1 and L2 are finite, but
x → x0

L1 ≠ L2. Then, lim f(x) does not exist. We say that x 2 − 32


xlim f(x) = lim = 2 × 32 −1 = 6
x → x0 →3 −
x →3 −
x−3
f(x) has a finite discontinuity or a jump discontinuity
lim f(x) = lim x + 2 = 5
at x = x0. x → 3+ x → 3+

Suppose f(x) is continuous in [a, b] except at points


Since lim f(x), does not exist, f(x) is not continuous
x1, x2, x3, x4,…… where the discontinuities at these x →3

points are finite discontinuities (refer Fig. 2.7) we say at x = 3. We say that f(x) has a jump discontinuity at
that f(x) is piece wise continuous in [a, b]. x = 3.
2.12  Differential Calculus

 sin5x  sin5x 
 , x≠0 lim 
(iii) For the function f(x) =  sin9x x → 0  5x   5 1 5 5
k, x=0 = × = × =
 sin9x  9 1 9 9
lim 
 sin5x  x → 0  9x  
 5x  5 f(0) is given as k.
lim f(x) = lim ×
x →0 x →0  sin9x  9 5
If we assign the value to k, f(x) is continuous at
 9x  9
x = 0.

Concept of derivative—Differentiation

Neighbourhood of a point (h positive or negative) if it exists, is called the derivative of


f(x) at x = x0 and is denoted by f ’(x0).
If d is a small positive number, then the interval
f(x 0 + h) − f(x 0 )
(x0 - d, x0 + d) defines a neighbourhood of x0. i.e., f '(x 0 ) = lim if this limit exists.
h→0 h
Definition of the derivative If we write x0 + h as x, then as h approaches zero, x
approaches x0. We may therefore write
Let y = f(x) be a function of x which is continuous in a
f(x + h) − f(x 0 ) f(x) − f(x 0 )
neighbourhood of a point x0. Then, lim 0 f '(x 0 ) = lim , if this limit exists.
h→0 h x → x0 x − x0

C o nce p t S t r a n d
Concept Strand 6 (2 + h)2 − 7(2 + h) + 5 − 22 − 7 × 2 + 5
= lim
Find the derivative of the function f(x) = x2 - 7x + 5 at x = 2. h→0 h

Solution h 2 − 3h
= lim = lim(h − 3), since h ≠ 0
h→0 h h→0
f(2 + h) − f(2)
f '(2) = lim
h→0 h = -3

Remarks corresponding change in y for a change Dx in x). We may


therefore write
In the notation y = f(x), x is called the independent variable
and y is called the dependent variable. f(x 0 + Dx) − f(x 0 )  Dy 
In the definition of the derivative, x0 + h (h > 0 or < 0) f '(x 0 ) = lim = lim  
Dx → 0 Dx Dx → 0  Dx 
represents a point in a neighbourhood of x0. h may be
termed as a change in x (or an increment in x) and we de- if the limit exists.
note it by Dx (pronounced as ‘delta x’). Similarly, f(x0 + h) In general, if f(x) is continuous in a neighbourhood of a
- f(x0) = f(x0 + Dx) - f(x0) is denoted by Dy (i.e., Dy is the point x (here, x denotes a specified point),
Differential Calculus  2.13

f(x + h) − f(x) f(x + Dx) − f(x) A tangent to a curve at a point P is the line that touches
f '(x) = lim = lim the curve at that point (Refer Fig. 2.9).
h→0 h Dx → 0 Dx
 Dy 
= lim  , if the limit exists.
Dx → 0  Dx 
Some common alternative notations for the derivative are
P Curve
as follows:
dy df d Tangent at P to the curve
f '(x) = y ' = = = f(x) = Df(x).
dx dx dx θ x

dy d
is simply a synonym for f ’(x). The symbols Fig. 2.9
dx dx
and D are called differentiation operators because they
indicate the operation of differentiation (which is the pro- We are now in a position to define the slope of a curve
cess of calculating the derivative). at a point.
We can now rewrite the definition of the derivative in    The slope of a curve at a point P on the curve is
the form defined as the slope of the tangent to the curve at P.
Suppose, q is the angle made by the tangent to the
dy  Dy  f(x + Dx) − f(x)
f '(x) = y ' = = lim = lim curve with the positive direction of the x-axis,
dx Dx → 0  Dx  Dx → 0 Dx
slope of the curve at P = slope of the tangent to the
We say that we differentiate y (or f(x)) with respect to x to curve at P = tanq.
dy     Since the tangents to the curve at different points
get the derivative or differential coefficient or f ’(x). If
dx make different angles with the x-axis, the slope of
we want to indicate the value of the derivative at a specified a curve at a given point depends on the x and y
 dy  coordinates of that point.
point, say x0, we write f '(x 0 ) =  
 dx  x = x
0

Curve y = f(x)
Q(x 2, f(x 2))
Interpretations of the Derivative
Q1
(i) Let us introduce the concept of the slope of a curve.
We cannot define the slope of a curve in the same
way as we defined the slope a straight line, because, a Q2
curve is slanted to the x-axis at different points with Tangent to the
different angles (Refer Fig. 2.8). Therefore, we see that P(x 1, f(x 1) curve at P
the slope of a curve changes at every point.
Fig. 2.10

   Let the slope of the curve y = f(x) at the point P


P5
whose x coordinates is x1 be m, i.e., slope of the tangent
P3 (or tangent line) to the curve at P(x1, f(x1)) equals m.
   If we take a neighbouring point Q(x2, f(x2)) on
P4 the curve (Refer Fig. 2.10) the slope of the secant line
P1 Curve
f(x 2 ) − f(x1 )
PQ = . It is clear that this cannot be equal
P2 x 2 − x1
Fig. 2.8 to m as PQ is not the tangent to the curve at P.
2.14  Differential Calculus

f(x 2 ) − f(x1 ) f(t 0 + h) − f(t 0 )


Therefore, slope of PQ i.e., gives only an at time t = t0 is obtained as , as h →
x 2 − x1 h
approximation to m. However, as we move Q closer 0. We know that velocity is the rate of change of dis-
and closer to P along the curve, the slopes of the placement with respect to time.
  This means that we can interpret the derivative f ’(x0)
secants PQ1, PQ2,…, come closer and closer to m or
as the rate of change f(x) with respect to x at x = x0.
the angles made by the secants PQ1, PQ2,.. come closer
and closer to the angle made by the tangent line to the Rate of change of f(x) with respect to x at x = x0
curve at P. We have how shown that the slope m of
f(x 0 + h) − f(x 0 )
the tangent at a point x1 on the curve y = f(x) can be = lim = f ’(x0).
obtained through a limiting process as x2 → x1, of the
h→0 h
f(x 2 ) − f(x1 )   Similarly, we encounter rate of change of work with
expression .
x 2 − x1 respect to time (which is called power), rate of change
in the concentration of a reactant with respect to time
This means that we can interpret the derivative f ’(x0)
(which is called the rate of reaction), rate of change of
as the slope of the tangent at the point x0 on the curve
the population of a colony of bacteria with respect to
y = f(x), i.e.,
time. All these rates of change can be interpreted as
f(x + h) − f(x 0 ) slopes of tangents. This is a very significant observation.
Slope of the tangent at x0 = lim 0
h→0 h In other words, whenever we solve a tangent prob-
= f ’(x0) lem, we are not just solving a problem in geometry.
We are implicitly solving a great variety of problems
(ii) In the light of the aboce said, we can how define that involving rates of change in various branches of sci-
the velocity of a particle moving along a straight line ence and engineering.

C o nce p t S t r a n ds
Concept Strand 7 (a) Slope of the tangent to the curve y = x3 at x = 2 is given by
Find the slope of the tangent to the curve y = f(x) = x3, at  dy 
f '(2) =   = 3 × 22 = 12
x=2  dx  x = 2

Solution Concept Strand 7 (b)


3 3
dy (x + Dx) − x
f(x) = x3 ⇒ f ‘(x) = = lim The position of a particle moving along a straight line is
dx Dx → 0 Dx
t2 1
3 3 given by the equation of motion s = f(t) = + 3t −
(x + Dx) − x 9 2+t
= lim
Dx → 0 (x + Dx) − x where t is measured in seconds and s is measured in
metres. Find the velocity of the particle at time t = 3.
(x + Dx)3 − x 3
= x +lim = 3 × x 3 −1 ,
Dx → x (x + Dx) − x
Solution
xn − an
using the result lim = na n −1 .
x →a x − a
f(t + h) − f(t)
Derivative f ‘(t) = lim
= 3x . 2 h→0 h
Differential Calculus  2.15

 (t + h)2 1   t2 1   
 + 3(t + h) −  −  + 3t −   
 9 2 + t + h  9 2 + t  2t + h + 3 + 1  2t 1
= lim = lim
h→0 h h→0  9 9 (2 + t + h)(2 + t)  = 9 + 3 + (2 + t)2
2
 
2th h h  
+ + 3h +
9 9 (2 + t + h)(2 + t) Velocity at t = 3 is given by f ‘(3) which is
= lim
h→0 h 2×3 1 278
+3+ = metres/sec.
9 25 75

Differentiability of functions

Definition f(x) − f(x 0 )


Now, f(x) − f(x 0 ) = (x − x 0 )
A function f(x) is said to be differentiable at x = x0­ if f ’(x0) (x − x 0 )
exists. ⇒ lim  f(x) − f(x 0 )
It is said to be differentiable in (a, b) [or (−∞, a) or x → x0

(b, ∞) or (−∞, ∞)] if it is differentiable at every point x in this f(x) − f(x 0 )


interval. = lim × lim(x − x 0 )
x → x0 (x − x 0 ) x → x0

Theorem = f ’(x0) × 0, since f(x) is differentiable at x0. = 0

If f(x) is differentiable at a point x0, it is continuous at x0. We infer from the above theorem that a function cannot
We outline below the proof of the above theorem. To have a derivative at points of its discontinuity.
prove that f(x) is continuous at x0 we have to show that Note that the converse of the above theorem is false.
lim f(x) = f(x 0 ) . That is, there are functions that are continuous at a point
x → x0 but are not differentiable there. Let us illustrate this by two
This is equivalent to showing that lim  f(x) − f(x 0 ) = 0 . examples.
x→x 0

C o nc e p t S t r a n ds
Concept Strand 8 Now, f(1) = 5 × 1 = 5; xlim f(x) = 5 × 1 = 5 and

→1

5x, −4< x ≤1 lim f(x) = 3 × 1 + 2 = 5


Let f(x) =  . Verify if f(x) is differentia- x →1+

3x + 2, 1 < x < 4


⇒ lim f(x) = 5 = f(1) , which means that f(x) is con-
ble in the interval (-4, 4). x →1

tinuous in (-4, 4).


In (-4, 1),
Solution
f(x + h) − f(x) 5(x + h) − 5x
f '(x) = lim = lim = lim5 = 5
First, we check whether f(x) is continuous in (-4, 4) since h→0 h h→0 h h→0

continuity of a function is a necessary condition for dif-


3(x + h) + 2 − (3x + 2)
ferentiability. In (1, 4), f '(x) = lim = lim3 = 3
h→0 h h→0
It is clear that in -4 < x < 1, f(x) = 5x which is a poly-
nomial function. Therefore, f(x) is continuous in (-4, 1). For finding the derivative of f(x) at x = 1 we have to
Similarly, f(x) is continuous in (1, 4). We have to check the f(1 + h) − f(1)
continuity of f(x) at x =1. find lim
h→0 h
2.16  Differential Calculus

f(1 + h) − f(1) 5(1 + h) − (5) Solution


If h < 0, lim = lim = 5 and
3 − x, x ≤ 3
h→0 h h→0 h
We may write f(x) = 
f(1 + h) − f(1) 3(1 + h) + 2 − 5  x − 3, x > 3
if h > 0, lim = lim = 3. It can be easily verified that f(x) is continuous for all x
h→0 h h → 0 h
and that for x < 3, f ’(x) = -1 and for x > 3, f ’(x) = 1.
f(1 + h) − f(1) We have to check the differentiability of the function at
We see that lim does not exist.
h→0 h f(3 + h) − f(3)
x = 3 for which we have to see whether lim
i.e., f(x) is not differentiable at x = 1 although it is con- h→0 h
tinuous there. Note that the above function is differentia- exists. We have, f(3) = 0
ble at all x in (-4, 4) except at x = 1. The graphs of f(x) and [3 − (3 + h)] − 0 = −1
f ’(x) are shown below. Now, when h < 0, this limit = lim
h→0 h
and
f(x) when h > 0, limit = lim
[(3 + h) − 3] − 0 = 1
h→0 h
We infer that the left limit and the right limit are dif-
f(3 + h) − f(3)
ferent. Or, lim does not exist. This means
h→0 h
that f(x) is not differentiable at x = 3 although it is continu-
X ous there.
−4 0 1 4 The graphs of f(x) and f ’(x) are shown below.

f(x)

Graph of f(x)

3
f'(x)

5 X
0 3

3
Graph of f(x)

0 X
−4 1 4 f '(x)

Graph of f’(x)
1
X
0 3
Concept Strand 9 −1
Let f(x) = | x–3 |. Verify differentiability of the function at Graph of f ’(x)
x = 3.
Differential Calculus  2.17

Left and right derivatives of a function We can say that a function f(x) fails to be differentiable
at a point x0 if
Let f(x) be continuous at x = x0. The left derivative of f(x) at
(i) f(x) is not continuous at x0. [Refer Fig. 2.11] OR
f(x 0 + h) − f(x 0 )
x = x0 is defined as the limit of as h tends (ii) the graph of f(x) changes direction abruptly at x0 or
h in other words, it has no unique tangent at x0.
to zero from the left.
[This is because f ’(x0-) ≠ f ’(x0+)]
Or left derivative of f(x) at x = x0 which may be denoted
[Refer Fig. 2.12]  OR
by
(iii) the graph of y = f(x) has a vertical tangent at x0. i.e.,
f(x 0 + h) − f(x 0 ) f(x) is continuous at x0 and | f ’(x) | tends to infinity as
f '(x 0− ) = lim− , if it exists.
h→0 h x tends to x0. [Refer Fig. 2.13]
f(x 0 − h) − f(x 0 )
= lim ,h > 0
h→0 −h
f(x)
The right derivative of f(x) at x = x0 is defined as the limit of
f(x 0 + h) − f(x 0 )
as h tends to zero from the right or right
h
derivative of f(x) at x = x0 which may be denoted by
f(x 0 + h) − f(x 0 )
f '(x 0+ ) = lim+ , if it exists.
h→0 h
f(x 0 + h) − f(x 0 )
= lim ,h>0
h→0 h X
O x0
It is clear that f ’(x0) exists if and only if f '(x 0− ) = f '(x 0+ ) .
In Concept strand 8 above f ’(1-) = 5 and f ’(1+) = 3 Fig. 2.12
and in Concept strand 9 above, f ‘(3-) = –1 and f ’(3+) = +1
Summing up,

f(x)
f(x)

X X
O x0 O x0

Fig. 2.11 Fig 2.13

Derivatives of Elementary Functions

On the basis of the general definition of a derivative, in or- (i) The independent variable x is changed to x + Dx
der to find the derivative of a given function y = f(x), it is (i.e., we say that we give an increment Dx to x where
necessary to carry out the following operations. Dx can be positive or negative).
2.18  Differential Calculus

(ii) Dy represents the corresponding change in the (iii) Derivative of f(x) = ex


dependent variable y. i.e., Dy = f(x + Dx)–f(x).
f(x + Dx) − f(x) e x + Dx − e x e Dx − 1
Dy f(x + Dx) − f(x) f '(x) = lim = lim = e x × lim
(iii) We evaluate lim = lim which is Dx → 0 Dx Dx → 0 Dx Dx → 0 Dx
Dx → 0 Dx Dx → 0 Dx
= ex × 1 = ex
 dy 
f ’(x)  or or y '  . d x
 dx  (e ) = e x
dx
This method of finding the derivative of a function (iv) Derivative of f(x) = loge x (written as ln x)
by using the definition may be termed as obtaining the
derivative from first principles. We shall apply the above f(x + Dx) − f(x)
general method for evaluating the derivatives of certain f '(x) = lim
Dx → 0 Dx
elementary functions. However, it would be tedious if
log e (x + Dx) − log e x
we always have to use this approach for finding the = lim
Dx → 0 Dx
derivative of a function. For overcoming this problem, in the
next section, we develop rules termed as differentiation  x + Dx   Dx 
log e  log e 1 +
rules.  x   x 
= lim = lim
By making use of the derivatives of elementary Dx → 0 Dx Dx → 0 Dx
functions and the differentiation rules, we will be able Dx

to obtain the derivative of any function directly without  x  Dx   1 1  Dx  x


= Dlim  log e 1 + x   × x = x Dlim log e 1 +
having to use the first principles approach. x → 0 Dx
  x →0  x 

(i) Derivative of a constant function: y = f(x) = c where,  Dx


  Dx

1   Dx  x  1   Dx  x 
c is a constant. = log e  Dlim 1 + = log e lim 1 +
x x →0  x   x  Dx → 0  x  
f(x + Dx) − f(x) c−c   X 
f '(x) = Dlim = lim = lim 0 = 0
x →0 Dx Dx → 0 Dx Dx → 0
1 1 d 1
= × log e (e) = (log e x) =
d x x dx x
or (c) = 0
dx Remark: Unless otherwise mentioned, log x means loge x.
(ii) Derivative of f(x) = xn where, n is a rational number.
Derivatives of Trigonometric functions
f(x + Dx) − f(x)
f '(x) = lim (i) Derivative of f(x) = sin x
Dx → 0 Dx
(x + Dx)n − x n (x + Dx)n − x n f(x + Dx) − f(x) sin(x + Dx) − sin x
= lim = lim f '(x) = lim = lim
Dx → 0 Dx Dx + x → x (x + Dx) − x Dx → 0 Dx Dx → 0 Dx
xn − an  Dx   Dx 
= n × xn-1, using the limit lim = na n −1 2cos  x + sin
x →a x−a  2   2 
= lim
= nxn-1 Dx → 0 Dx
 Dx 
d n sin
or (x ) = nx n −1 , n a rational number  Dx   2 
dx = lim cos  x +
Dx → 0  2   Dx 
For example,  2 

dy 3 −1 2   Dx  
For y = x3, y ' = dx = 3 × x = 3x  sin  2  
 Dx 
= lim cos
Dx → 0  x + × lim   = cos x × 1 = cos x
1 1 2  Dx → 0   Dx  
For y = x , y ' = x (1/2)−1 = 2
  2  
2 2 x  
1 1 d
For y = , y ' = (−1)x −1−1 = − 2 (sin x) = cos x
x x dx
Differential Calculus  2.19

In a similar way, we can show that the derivative of In a similar way we can show that the derivative of
f(x) = cos x is - sin x. f(x) = sec x is sec x tan x
d d
or (cos x) = − sin x or (sec x) = sec x tan x
dx dx
(ii) Derivative of f(x) = tan x
f(x + Dx) − f(x) tan(x + Dx) − tan x Derivatives of inverse trigonometric functions
f '(x) = lim = lim
Dx
Dx → 0 Dx → 0 Dx
sin(x + Dx) sin x If y = f(x)   (1)

cos(x + Dx) cos x
=
Dx → 0
lim dy Dy
Dx we have = lim
dx Dx → 0 Dx
sin(x + Dx)cos x − cos(x + Dx)sin x
= lim Let us assume that (1) has an inverse given by

Dx → 0 [cos(x + Dx)cos x](Dx)
x = g(y)   (2)
1 sin(x + Dx − x) In (1), x is the independent variable and y is the depen-
= Dlim ×
x →0 cos(x + Dx)cos x Dx dent variable, while, in (2), y is the independent variable
1  sin Dx  and x is the dependent variable.
= Dlim × lim We therefore have
x →0 cos(x + Dx)cos x Dx → 0  Dx 
1 dx Dx 1
= × 1 = sec2 x g '(y) = = lim = lim ,
cos2 x dy Dy → 0 Dy Dx → 0  Dy 
 Dx 
d
(tan x) = sec2 x since Dx → 0 when Dy → 0
dx
In a similar way, we can show that the derivative of 1 1 dy 1 dx
≠0
= or f '(x) = = , provided
f(x) = cot x is -cosec2 x.  dy f '(x) dx  dx  dy
d  dx   dy 
or (cot x) = −cosec2 x
dx
at the point (x, y) under consideration.
(iii) Derivative of f(x) = cosec x
1
f (x + ∆x) − f (x) \ f '(x) = , g’(y) ≠ 0   (3)
f ′(x) = lim g '(y)
∆x → 0 ∆x
cos ec(x + ∆x) − cos ec x (In g’(y), f(x) must be substituted for y)
= lim We use this important result to obtain derivatives of
∆x → 0 ∆x
the functions sin-1 x, cos-1 x and tan-1 x.
1 1
− p p
sin(x + Dx) sin x (i) y = f(x) = sin-1 x, where -1 ≤ x ≤ 1 and − ≤y ≤
= Dlim 2 2
x →0 Dx
sin x − sin(x + Dx) We have x = sin y = g(y)
= lim
Dx → 0 (Dx)sin(x + Dx)sin x dx p p
= g '(y) = cos y, − < y <
dy 2 2
 Dx  Dx
−2cos  x + sin
 2  2 dy 1 1 1 1
= lim \ = = = = ,x≠±1
Dx → 0 (Dx)sin(x + Dx)sin x
dx  dx  cos y 1 − sin2 y 1 − x2
 dy 
 Dx  Dx
− cos  (x + sin
 2  2 p p
= lim × lim
Dx → 0 sin(x + Dx)sin x Dx Dx Positive sign is taken since cos y > 0 in − <y <
2
→0 2 2
2
cos x (ii) y = f(x) = cos-1 x where -1 ≤ x ≤ 1 and 0 ≤ y ≤ p
= − × 1 = − cosecx cot x We have x = cos y = g(y)
sin2 x
d dx
(cosecx) = − cosecx cot x = g '(y) = − sin y , 0 < y < p
dx dy
2.20  Differential Calculus

dy 1 1 −1 Function Derivative Function Derivative


\ = = = , since sin y > 0
dx  dx  − sin y 1 − cos 2 y y = f(x) dy y = f(x) dy
 dy  f ’(x) = f'(x) =
dx dx
−1 ex ex cot x -cosec2 x
=
2
,x≠±1
1− x loge x (or ln x) sin-1 x
1 1
p p x 1 − x2
(iii) y = f(x) = tan-1 x where, -∞ < x < ∞ and − < y <
2 2
sin x cos x cos-1 x −1
We have x = tan y = g(y)
1 − x2
dx
= g '(y) = sec2 y
dy cos x -sin x tan-1 x 1
dy 1 1 1 1 1 + x2
\ = = = =
dx  dx  sec2 y 1 + tan2 y 1 + x 2 tan x sec2 x
 dy 
It may be noted that the above formulas for the de-
The results obtained above are consolidated in the rivatives of elementary functions were obtained in a formal
table below: way by making use of standard limits. The validity of these
derivatives depends on whether the function at the point of
Table 2.2 Table giving derivatives
interest is differentiable or not.
of elementary functions
d 1
Function Derivative Function Derivative For example, ( x) = is not valid at x = 0.
dx 2 x
y = f(x) dy y = f(x) dy
f ’(x) = f'(x) = p
dx dx d
Again, (tan x) = sec2 x is not valid at x = ± since tan x
constant c 0 cosec x -cosec x dx 2
cot x is not defined at these points. Similarly, there is no ques-
x (n rational) nx
n n–1
sec x sec x tan x tion of finding the derivative of cosec x at x = 0 or π (since
cosec x is not defined at these points).

Differentiation rules

Suppose f(x), g(x), h(x), … are given functions. When new For, let g(x) = k f(x)
functions are formed from these functions by addition, g(x + Dx) - g(x)
subtraction, multiplication or division, their derivatives g'(x) = lim
Dx → 0 Dx
can be expressed in terms of the derivatives of the given
kf(x + Dx) - kf(x)
functions. In what follows, we give these rules. = lim
Dx → 0 Dx

Rule 1 f(x + Dx) − f(x)


= k lim = kf '(x)
Dx → 0 Dx
If k is a constant and f(x) is a differentiable function,
d d
dx
[ kf(x)] = k dx [ f(x)]
Differential Calculus  2.21

C o nce p t S t r a n d
Concept Strand 10 Solution
Find the derivatives of dy d
(i) = 4 (x 3 ) = 4 × 3x 2 = 12x 2
dx dx
(i) y = 4x3
(ii) y = 7 tan x dy d
(ii) = 7 (tan x) = 7sec2 x
(iii) y = -3 ln x dx dx
dy −3
(iii) =
dx x

Rule 2 R(x + Dx) - R(x)


R '(x) = lim
If f(x) and g(x) are both differentiable, Dx → 0 Dx
d d d = lim
{ f(x + Dx) + g(x + Dx)} - {f(x) + g(x)}
dx
[ f(x) + g(x)] = dx f(x) + dx g(x) . This rule may be Dx → 0 Dx
called sum rule. f(x + Dx) + f(x)
= lim +
We outline the proof below. Dx → 0 Dx
Let R(x) = f(x) + g(x) g(x + Dx) - g(x)
lim = f '(x) + g '(x)
Dx → 0 Dx

C o nce p t S t r a n d
Concept Strand 11 Solution
Find the derivatives of the following functions: dy d 3 d
(i) = (x ) + 4 (sec x) = 3x 2 + 4sec x tan x
dx dx dx
(i) y = x + 4secx
3

(ii) y = x8 + 5ex dy d 8 d
(ii) = (x ) + 5 (e x ) = 8x 7 + 5e x
(iii) y = 4 cos x + 9 dx dx dx
dy d d
(iii) = 4 (cos x) + (9) = −4sin x + 0 = −4sin x
dx dx dx

Remarks By replacing g(x) by -g(x) in the sum rule,


The sum rule can be extended to the sum of any finite num- d d
ber of functions. If f(x), g(x), h(x) are three functions, f(x) - g(x) = f(x) + (-1)g(x) =

dx dx
d d d d
f(x) + g(x) + h(x) = f(x) +
 f(x) + (-1) g(x) = f '(x) - g '(x)
dx dx dx dx
d d
g(x) + h(x) = f '(x) + g '(x) + h'(x)
dx dx
2.22  Differential Calculus

C o nce p t S t r a n d
Concept Strand 12 Solution
Find f ’(x) if (i)  f(x) = 4x5 + 3 cot x - 5ex + 2 and (i) f ’(x) = 4 × 5x4 + 3(-cosec2x) - 5 × ex + 0
(ii) f(x) = ax2 + bx + c, where a, b, c are = 20x4 - 3 cosec2 x–5ex
constants (ii) f ’(x) = 2ax + b

Rule 3 on subtracting and adding the term f(x + Dx) g(x) in


the numerator.
If f(x) and g(x) are both differentiable,
d d d f(x + Dx)g(x + Dx) - g(x) +
dx
[ f(x)g(x)] = f(x) [ g(x)] + g(x) [ f(x)] .
dx dx g(x)f(x + Dx) - f(x)
= lim
This rule is called product rule for differentiation. Dx → 0 Dx
Product rule may be expressed as:  g(x + Dx) - g(x) 
Derivative of the product of two functions equals first = lim f(x + Dx)  +
Dx → 0
 Dx 
function × derivative of the second function plus second func-
tion × derivative of the first function.  f(x + Dx) - f(x) 
lim g(x)  
If we denote f(x) g(x) by y,
Dx → 0
 Dx 
Dy = f(x + Dx) g(x + Dx)–f(x) g(x)
 g(x + Dx) - g(x) 
d dy = lim f(x + Dx) × lim  +
f(x)g(x) = Dx → 0 Dx → 0
 Dx 
dx  dx
f(x + Dx)g(x + Dx) - f(x)g(x)  f(x + Dx) - f(x) 
= lim g(x) lim  
Dx
Dx → 0
 Dx 
Dx →0
f(x + Dx)g(x + Dx) - f(x + Dx)g(x) + = f(x) g’(x) + g(x) f ’(x)
f(x + Dx)g(x) - f(x)g(x)
= lim ,
Dx → 0 Dx

C o nce p t S t r a n d
Concept Strand 13 dy  2 1 
(iii) = x × + (tan −1 x) × 2x  +
Find the derivatives of the following functions: dx  (1 + x 2 ) 
(3x 4 + 1)cos x + (sin x)(3 × 4x 3 + 0) −
(i) y = x3 ex
(ii) y = x log x  xe x + e x × 1
(iii) y = x2 tan-1 x + (3x4 + 1) sin x–x ex
(iv) y = x2 ex sin x x2
= + 2x tan −1 x + (3x 4 + 1)
(v) 2 −1 2 x
y = (1 − x )sin x + x e log x 1 + x2
cos x + 12x 3 sin x − (x + 1)e x
Solution
(iv) We may take f(x) as representing ex and g(x) as rep-
dy d d
(i) = x 3 (e x ) + e x (x 3 ) = x 3 e x + e x × 3x 2 = (x 3 + 3x 2 )e x resenting x2 sin x.
dx dx dx
dy d 1 dy d 2 d
=x× (log x) + (log x) × 1 = x × + log x = 1 + log x = ex × (x sin x) + (x 2 sin x) (e x )
(ii) dx dx dx
dx dx x
Differential Calculus  2.23

= e  x × cos x + (sin x) × 2x  + (x sin x) × e


x 2 2 x
d 2 d 
 ex × (x log x) + (x 2 log x) (e x )
dx dx 
= e  x (cos x + sin x) + 2xsin x 
x 2

2 −1 x 2  1 
dy d d = 1 − x − 2xsin x + e  x × + (log x) × 2x + (x 2 log x)
(v) =  (1 − x 2 )sin −1 x  +  x 2 e x log x   x 
dx dx  dx 
 1 = 1 − x 2 − 2x sin −1 x + e x  x + 2x(log x) + x 2 log x 
= (1 − x 2 ) × + (sin −1 x)(0 − 2x) +
2
 1 − x

Rule 4 Note that we can find the derivative of tan x, cosec x,


sec x, cot x using the derivatives of sin x and cos x. As an
If f(x) and g(x) are differentiable,
illustration,
d d
g(x) [f(x)] − f(x) [g(x)] d d  1  (cos x) × 0 − 1 × (− sin x)
d  f(x)  dx dx (sec x) = = , by
 = dx dx  cos x  cos2 x
dx  g(x)  [g(x)]2
g(x)f '(x) − f(x)g '(x) using the Quotient rule
= sin x
[g(x)]2 = = sec x tan x
cos2 x
This is called the quotient rule for differentiation.
Quotient rule may be expressed as: d d  cos x 
Again (cot x) =
Derivative of the quotient is equal to the denominator dx dx  sin x 
times the derivative of the numerator minus the numerator sin x(− sin x) − (cos x)(cos x) −(cos2 x + sin2 x)
times the derivative of the denominator, all divided by the = 2
=
sin x sin2 x
square of the denominator. = - cosec2 x

C o nce p t S t r a n d
Concept Strand 14 (7x 2 + 6)(12x 3 + 3x 2 − 2x) − (3x 4 + x 3 − x 2 + 5)(14x)
=
Find the derivatives of the following functions: (7x 2 + 6)2

3x 4 + x 3 − x 2 + 5 (42x 5 + 7x 4 + 72x 3 + 18x 2 − 82x)


(i) y = =
(7x 2 + 6) (7x 2 + 6)2
a cos x + b
(ii) y = (a, b constants) (bcos x + a)(−a sin x) − (a cos x + b)(− bsin x)
bcos x + a (ii) dy =
2
dx (bcos x + a)
Solution =
(b2 − a 2 )sin x
d (bcos x + a)2
(7x + 6) (3x 4 + x 3 − x 2 + 5) −
2

dx
d
(3x 4 + x 3 − x 2 + 5) (7x 2 + 6)
dy dx
(i) dx = (7x 2 + 6)2
2.24  Differential Calculus

Rule 5  dy   du   dv 
=  , provided all the three
Differentiation of composite functions  du   dv   dx 
Before giving the rule, let us consider two examples. limits exist.
(i) Let y = tan x3 dy 1
Since y = sin-1u, = ;
x3 being a function of x may be denoted by u. Then, du 1 − u2
we have du dv
y = tan u where u = x3   (1) u = 5ev ⇒ = 5e v ; v = x4 ⇒ = 4x 3
dv dx
i.e., y is a function of u where u is a function of x or y
is a composite function. We are interested in obtaining dy 1 1
\ dx = × 5e v × 4x 3 = × 5e v × 4x 3
dy 1 − u2 1 − (5e v )2
.
dx
4
For a change Dx in x, let the change in u be Du and let 1 20x 3 e x
= × 5e x4
× 4x =3
the corresponding change in y be denoted by Dy. 4
1 − (5e x )2 1 − 25e2x
4

As Dx → 0, both Du and Dy → 0
dy Dy  Dy Du  We now give the chain rule for finding the derivatives
= lim = lim × , on multiplication and
dx Dx → 0 Dx Dx → 0  Du Dx  of composite functions.
If y is a function of u where, u is a function of x i.e.,
division by Du.
y → u → x,
 Dy   Du 
= Dlim   × Dlim
x → 0  Du 
 
x → 0  Dx 

dy  dy   du 
 Dy   Du  =
= Dlim
u→0
 × lim
 Du  Dx → 0  Dx 
dx  du   dx 

dy  dy   du  The above rule can be extended.


⇒ = , provided both limits exist(2)
dx  du   dx  Suppose y is a function of u where, u is a function of v
dy du and v is a function of x i.e., y → u → v → x,
Since y = tan u, = sec2 u = sec2 x 3 and = 3x 2
du dx
dy dy  dy   du   dv 
\ = (sec2 x 3 ) × 3x 2 = 3x 2 sec2 x 3 =
dx dx  du   dv   dx 
(2) is known as a chain rule for differentiation
( )
(ii) Let y = sin −1 5e x
4
Result 1
x4 being a function of x may be denoted by v. Again, Derivative of an even function is an odd function and deriva-
x4 tive of an odd function is an even function.
5e which is a function of v may be denoted by u.
Then, we have Let f(x) be an even function. This means that f(-x) =
y = sin-1 (u) where, u = 5ev and v = x4 f(x) for all x   (1)
or y is a function of u where u is a function of v where, Differentiating both sides with respect to x,
v is a function of x. d
f '(− x) × (− x) = f '(x), using the differentiation rule
or y is a function of a function of a function. dx
For a change Dx in x, let the corresponding changes in for composite functions.
v, u and y be denoted respectively by Dv, Du and Dy. i.e., f ’(-x) × -1 = f ’(x)
As Dx → 0, Dv, Du, Dy → 0. or f ’(-x) = - f ’(x)
dy Dy Dy Du Dv We conclude that f ’(x) is an odd function. Proceeding
= lim = lim × ×
dx Dx → 0 Dx Dx → 0 Du Dv Dx in the same manner, we can show that the derivative of an
odd function is an even function.
 Dy   Du   Dv 
= lim      
Dx → 0  Du   Dv   Dx 
Result 2
 Dy   Du   Dv  Derivative of a periodic function is also periodic with the
= lim   lim   lim  
Du → 0  Du  Dv → 0  Dv  Dx → 0  Dx 
same period.
Differential Calculus  2.25

Let f(x) be periodic with period T. We therefore have i.e., f ’(x + T) × 1 = f ‘(x), since T is a constant.
f(x + T) = f(x) for all x. or f ’(x + T) = f ’(x)
Differentiating the above relation with respect to x, Result follows.
d We illustrate the use of the chain rule by three
f ‘(x + T) × (x + T) = f '(x) examples.
dx

C o nce p t S t r a n d
 x  1 + x2 (1 + x 2 ) × 1 − x × 2x
Concept Strand 15 = cos  1 + x 2  × 2 x ×
(1 + x 2 )2
Find the derivatives of the following expressions:
 x  (1 − x 2 )
 ax + b 
−1 = cos  2 
×
(i) tan  , a, b, c, d constants. 3
 1 + x  (2 x )(1 + x 2 ) 2
 cx + d 

(ii) sin 
 x 
−1 2
(iii) Let y = (sin x) × log x + 1 − x
2
( )
2 
 1+ x 

dy −1 2
= (sin x) ×
1
×
d
x + 1 − x2 ( )
−1 2
(iii) (sin x) × log x + 1 − x ( 2
) dx x + 1 − x2 dx
( )
Solution (
+ log x + 1 − x 2 × 2sin −1 x × ) d
dx
(sin −1 x)

 ax + b  (sin −1 x)2  1 d 
(i) Let y = tan −1  , a, b, c, d constants. = × 1 + × (1 − x 2 ) +
 cx + d 
(x + 1 − x2 )  2 1− x 2 dx 

x)log ( x + )
dy 1 d  ax + b 
= 2
×  cx + d  (2sin −1 1 − x2
dx  ax + b  dx
1+ 
 cx + d  1− x 2

2
(cx + d)
= (sin −1 x)2  x 
(cx + d)2 + (ax + b)2 = × 1 − +
×
(cx + d) × a − (ax + b) × c (x + 1− x ) 2
 1 − x2 


(cx + d)2
(2sin −1 x)log ( x + 1 − x2 )
(ad − bc)
= 1 − x2
(ax + b)2 + (cx + d)2

(ii) Let y = sin  2 


 1+ x 
 x 
=
(sin −1 x)2 ( 1 − x − x) + 2

 x  d  x 
(x + 1− x ) 1− x2 2

x)log ( x + 1 − x )
dy
= cos  2 
×  1 + x2  (2sin −1 2
dx  1 + x  dx  

 x  1 d  x 
1 − x2
= cos  × ×
 1 + x 2  x dx  1 + x 2 
2
1 + x2
2.26  Differential Calculus

Differentiation of implicit functions In the case of (2), we could express y explicitly in terms
of x (see (3)). But, not every implicitly defined function can
Let the two variables x and y be related by some equation
be represented explicitly in the form y = g(x).
symbolised as F(x, y) = 0   (1)
For example, the equation F(x, y) = x3 + 3xy2 - sin y +
As an example, consider the relation
x2 + 2y2 = 5   (2) 5 = 0 is an implicitly defined function (or an implicit func-
1 tion). However, it will not be possible for us to express y
From (2), we can get y = ± 5 − x2   (3)
2 explicitly in terms of elementary functions of x in the above
(2) is said to define implicitly the functions case.
1 1 dy
y = 5 − x 2 and y = − 5 − x 2 Our problem is to find if the function is given in
2 2 dx
Suppose we are in a position to solve (1) for y and we an implicit form (1) without transforming it into an explicit
get y = g(x). Then we say that y = g(x) is ‘implicitly’ defined form i.e., without representing it in the form y = g(x).
by F(x, y) = 0. We illustrate the procedure by two examples.

C o nce p t S t r a n ds
Concept Strand 16 Concept Strand 17
If F(x, y) = x3 + 3xy2 - sin y + 5 = 0, find y’. dy
Given F(x, y) = y3 + 3yx + 3ey - 2 = 0, find .
dx
Solution
Solution
The given relation may be treated as an identity in x and
y in some region of the xy plane. We differentiate the rela- On differentiating the given expression implicitly, with re-
tion with respect to x and noting that y is a function of x. spect to x,
 dy  dy dy  dy  dy
2 2
We get 3x + 3  x × 2y + y × 1 − (cos y) + 0 = 0 3y 2 + 3  y × 1 + x  + 3 × ey =0
 dx  dx dx  dx  dx

dy dy
i.e., (6xy - cos y) = -3(x2 + y2) (y2 + x + ey) = -y
dx dx
dy −3(x 2 + y 2 ) dy −y
or = =
dx (6xy − cos y) dx (y 2 + x + e y )

Differentiation of composite exponential functions 1 dy


= v(x) ×
1
u '(x) +  log u(x) × v '(x)
y dx u(x)
A composite exponential function is a function in which
both the base and the exponent are functions of x. dy  u '(x)v(x) 
2
For example, y = xx, y = (sin x)x , y = (3x 4 + 5)cos x are OR = y + v '(x)  log u(x) 
dx  u(x) 
composite exponential functions.
The general form of a composite exponential function The above procedure may be termed as logarithmic
will be y = [u(x)]v(x), where u(x) and v(x) are functions of x. dy
differentiation. We illustrate this method to find in the
dy dx
To find the derivative we take logarithms (to the
dx case of three examples below.
base e) on both sides.
We have log y = v(x) log u(x)   (1) (i) y = xx
Treating the above as an implicit function, we differentiate Taking logarithms on both sides,
both sides of (1) with respect to x and get log y = x log x
Differential Calculus  2.27

Differentiating both sides with respect to x, dy


Using product rule or quotient rule for finding is
1 dy 1 dx
y dx = x × x + log x × 1 = 1+ log x
a very cumbersome process. Suppose we take logarithms
dy
on both sides,
⇒ = y(1 + log x) = xx (1 + log x).
dx 5 3
log y = log(4x 2 + 9x − 1) + 5log sin x + x 3 − log(3x 4 + 8x 2 + 6)
x2
2 7
(ii) y = (sin x)
Taking logarithms on both sides, we get log y = x2 log Note that differentiation of the right side of the above
(sin x) involves application of sum rule only.
Differentiating both sides with respect to x, Differentiating both sides with respect to x,
1 dy  1  1 dy 5 1 1
= x2  × cos x  + log sin x × (2x) = x2 cot x + = × × (8x + 9) + 5 × ×
y dx  sin x  y dx 2 (4x 2 + 9x − 1) sin x
3 1
2x log sin x cos x + 3x 2 − × (12x 3 + 16x)
7 (3x 4 + 8x 2 + 6)
dy
\ = y [x2 cot x + 2x log sin x]
dx dy  5(8x + 9) 3(12x 3 + 16x) 
= y + 5cot x + 3x 2 − 
(iii)
y = (3x + 5) 4 cos x
dx 2
 2(4x + 9x − 1) 7(3x 4 + 8x 2 + 6) 
Taking logarithms on both sides,
log y = (cos x) log (3x4 + 5)
Differentiating both sides with respect to x, Derivative of a function represented parametrically
1 dy  1  We have already introduced ‘parametric representation of a
= (cos x) × 4 × 3 × 4x 3  +
y dx  3x + 5  function’ in the unit ‘Functions and Graphs’.

log(3x 4 + 5) × (− sin x) Let a function y of x be represented by the parametric
equations x = f(t) , y = g(t) where t1 ≤ t ≤ t2 [Here, t is the
12x 3 cos x parameter]. Let us assume that both f(t) and g(t) are differ-
= − sin x log(3x 4 + 5)
3x 4 + 5 entiable in t1 ≤ t ≤ t2 and that x = f(t) has an inverse t = h(x)
which is also differentiable. We may say that y is a function
dy 12x 3 cos x 
\ = y 4
− sin x log(3x 4 + 5) . of t where, t is a function of x. i.e., y is a composite function.
dx  3x + 5  Applying chain rule,
Logarithmic differentiation method may be convenient dy dy dt
= ×
in certain problems where we have to find the derivatives dx dt dx
of expressions involving exponents and very complicated
fractions. For example, let us consider  dy 
dt 1 dy  dt  g '(t)
( ) But, = ⇒ = =
5 3
(4x 2 + 9x − 1) 2 (sin x)5 e x dx  dx  dx  dx  f '(t)
y = 3  dt   dt 
(3x 4 + 8x 2 + 6) 7

C o nce p t S t r a n ds
dy
Concept Strand 18 = a × 3sin2 t × cos t
dt
dy
Find if x = a cos3 t, y = a sin3 t dx
= a × 3cos2 t × (− sin t)
dx
dt

Solution  dy 
dy  dt  3a sin2 t cos t
For the function x = a cos3 t, \ = =– = –tan t
dx  dx  3a cos2 t sin t
 dt 
y = a sin3 t
2.28  Differential Calculus

Concept Strand 19 dy
= a(0 + sin q) = a sin q and
dy p dq
Find at q = for the function x = a(q - sin q),
dx 3 dx
= a(1 − cos q)
y = a(1 - cos q) dq

dy sin q 2sin q cos q


= = 2 2 = cot q
Solution dx (1 – cos q) 2sin2 q 2
2
Here, q is the parameter.
 dy  p
 dy   dx  p = cot 6 = 3
dy  dq  q=
3
We have =
dx  dx 
 dq 

Concept of Differential

We introduce this concept through an example. Dy


If the side of a cube is of length x, its volume, say y, will be very small. We may write = f '(x) + l where,
Dx
is given by y = x3. Suppose the actual length of a side is
l → 0 as Dx → 0
(x + Dx) units while it is measured as x units, there is an
error Dx in the measurement of x. Due to this error Dx in or Dy = f ‘(x) Dx + l(Dx)   (1)
x, the error in the computation of the volume is given by
Dy = (x + Dx)3 - x3 = 3x2(Dx) + 3x(Dx)2 + (Dx)3 Thus, the change Dy in y (or in f(x)) consists of two
Since Dx is very small, 3x(Dx)2 and (Dx)3 will be very terms. The product f’ (x) Dx is called the differential of y
much smaller. We may take 3x2(Dx) (by omitting the last and is denoted by dy (or df(x)).
two terms on the right side) as an approximation to the or we have the definition dy = differential of
error in the volume i.e., in y. This approximation to Dy is y = f ‘(x) Dx   (2)
called the “differential” of y and is denoted by dy. If y = x, dy = 1 Dx = dx. Thus, the differential dx of the
We have dy = 3x2(Dx) independent variable x coincides with Dx. (we may also re-
As an illustration, suppose x = 5 cms, Dx = 0.001 cm. gard the quantity dx = Dx as a definition for the differential
An approximation to the error in the computation of the of the independent variable x.) This does not contradict the
volume is given by 3 × 52 × 0.001 = 0.075 c.c. definition of the differential of a function.
In general, if y = f(x) is differentiable in an
We can then write (2) as dy = f’ (x) dx
dy
interval, at some point x in this interval is given by dy
dx From the above, it follows that f '(x) = , i.e., the de-
dx
dy Dy rivative f ‘(x) may be regarded as the ratio of the differential
f '(x) = = lim .
dx Dx → 0 Dx of y to the differential of the independent variable x.
Dy
As Dx → 0, approaches f ‘(x) or for very small values dy Differential of y
Dx Or f ’(x) = = .
dx Differential of x
 dy  Dy
of Dx, the difference between f ‘(x)  or  and
 dx  Dx
Differential Calculus  2.29

C o nce p t S t r a n d
Concept Strand 20 Solution
dy x = t2, y = 2t
If x = t2, y = 2t, find
dx ⇒ dx = 2t dt and dy = 2 dt (as t is the independent vari-
able in each case)
dy 2dt 1
\ = = after cancelling out 2dt.
dx 2t dt t

Geometrical significance of the differential SR = PR tan ∠RPS = PR × slope of the tangent at P


= (dx) × f’ (x) = f’ (x) dx = dy
and QR = QN - RN = QN - PM = f(x + Dx) - f(x) = Dy
The concept of ‘differential’ finds applications in many
Q
problems in engineering and technology.
Y
x may be length or temperature or any measurable
S
quantity and we may like to compute y which is a function
of x. When x is measured using a measuring instrument,
y = f(x) P R there is a likelihood of an error committed in this measure-
ment (we may call it as zero error of the instrument). If
dx denotes this error, an approximation to the error in the
computation of y (= f(x)) is given by dy = f ’(x) dx. We may
therefore get a rough idea of the error in the computation
O X of y. Sometimes, we may be interested in the relative error
T M N
in y or the percentage error in y.
dx
For this purpose we define as the relative error in
Fig. 2.14 x
dx
x and × 100 as the percentage error in x.
x
Referring to Fig. 2.14, P is the point (x, y) and Q is the point dy f '(x)
If y = f(x) ⇒ dy = f ‘(x) dx ⇒ = dx = relative
(x + Dx, y +Dy). PT is the tangent to the curve y = f(x) at P y f(x)
error in y;
meeting the x-axis in T and ordinate QN in S. PR is perpen-
dy dy  f '(x) 
dicular to QN. Since represents the slope of the tangent × 100 =  dx × 100 = relative error in y ex-
dx y  f(x) 
to the curve, from DPSR, pressed in percentage.

C o nce p t S t r a n d
Concept Strand 21 Solution
dy 3x 2 dx
Find the percentage error in y if y = x . 3
Differentiating, dy = 3x2 dx ⇒ = 3 dx = 3 .
y x x
The above relation means that an error of 1% in the
measurement of x will result in an error of 3% in the com-
puted value of y. i.e., it is magnified 3 times.
2.30  Differential Calculus

Successive differentiation

Let y = f(x) be a function of x differentiable on some inter- Again, if f ”(x) (being a function of x) is differentiable,
dy we differentiate this function with respect to x and get the
val. Then the derivative or f ’(x) is also a function of
dx third order derivative of the original function. It is denoted
x. Suppose f ’(x) is differentiable. Then, differentiating this d3 y
by or y ’’’(x)
function with respect to x we get what is called the second dx 3
order derivative of the original function f(x) and it is de- d3 y d  d2 y  d
d2 y = = [f "(x)] = f ’’’(x)
noted by or f ”(x). dx 3
dx  dx 2  dx
dx 2
We can continue this process of differentiating with
d2 y d  dy  d respect to x successively and we get 4th order derivative of
=   = [f '(x)] = f ''(x).
dx 2
dx  dx  dx f(x), 5th order derivative of f(x) and so on.

C o nce p t S t r a n d
Concept Strand 22 d3 y
4
3
= 72 × 7x 6 = 504 x 6 ,
d y dx
Find if
dx 4 d4 y
(i) y = x9 = 504 × 6x 5 = 3024 x 5
dx 4
(ii) y = sin 4x
(ii) y = sin 4x
dy d2 y
Solution ⇒ = 4cos 4x, = −16 sin 4x,
(i) y = x9 dx dx 2
dy d2 y d3 y d4 y
⇒ = 9x 8 , = 9 × 8x 7 = 72 x 7 , = −64cos 4x, = 256 sin 4x .
dx dx 2 dx 3 dx 4

Notations for the different order


derivatives of a function y = f(x)

Table 2.3

Function First order Second order Third order Fourth order ………….. nth order
derivative derivative derivative derivative derivative
y y’ y’’ y ‘’’ y (4) ………….. y(n)
y dy d2 y d3 y d4 y ………….. dn y
dx dx 2 dx 3 dx 4 dx n
y Dy D2 y D3 y D4 y ………….. Dn y
y = f(x) f ‘(x) f ‘’(x) f ’’’(x) f (4) (x) ………….. f (n) (x)
Differential Calculus  2.31

Higher order derivatives

dy  m!
We have already seen that the derivative (or f ’(x)) of m−n
 (m – n)! x , n < m
dx 
a function f(x) gives the slope of the tangent to the curve y
(n)
= m!, n=m
y = f(x) at x. The slope of the curve gives us an idea of the 0, n>m
Direction: of the curve at that point. There are other geo- 

metrical properties of the curve like ‘intervals in which
the function is increasing or decreasing’, ‘where the func- 1
tion attains its maximum or minimum values’, in which (ii) y = where a, b are constants
(ax + b)
interval it is convex or concave and so on. Using the
higher order derivatives, it will be possible for us to study −1 (−1)a
y ' = 2
×a =
these geometrical properties analytically. (ax + b) (ax + b)2

Remarks (−1)a × (−2) (−1)(−2)a 2


y '' = ×a =
(ax + b)3 (ax + b)3
(i) It may be remembered that even if a function f(x) is
continuous at a point, it need not be differentiable …………………………………………
there. Similarly, even though f(x) is differentiable, (−1)(−2)(−3).....(−n)a n
f ’(x) need not be differentiable. Again, even if f ’(x) y (n) =
(ax + b)n +1
is differentiable, f ”(x) need not be differentiable
(−1)n n!a n
and so on. ⇒ y (n) =
(ii) Polynomial functions, ex, sin x, cos x are differentiable (ax + b)n +1
any number of times in R, while tan x is differentiable (iii) y = e where a is a constant
ax

−p p y ‘ =a eax
any number of times in 2 < x < 2 only.
⇒ y’’ = a2eax,
…….……..
We observe that higher order derivatives of a function
y (n) = an eax.
are obtained by successive differentiation. As any function
(iv) y = sin (ax + b)
of x is a composite function of elementary functions, our
next task is to obtain formulas for the nth derivatives of a y ‘ = cos (ax + b) × a = a sin ax + b + p 2 ( )
few standard elementary functions.
It may be mentioned that it is not possible to develop ( )
y ‘’ = a cos ax + b + p 2 × a = a 2 cos ax + b + p 2 ( )
= a sin (ax + b + p 2 + p 2 )
such formulas for all elementary functions.
2

nth derivatives of a few elementary functions = a sin (ax + b + 2 × p 2 )


2

(i) y =xm where, m is a positive integer. ………………………………….


dy  np 
= m x m −1 y (n) = a n sin  ax + b + .
dx  2 
d2 y (v) y = cos (ax + b)
= m(m − 1)x m − 2
dx 2
3
y ’ = - sin (ax + b) × a = a cos ax + b + p
2 ( )
dy
= m(m − 1)(m − 2)x m − 3 Proceeding as in (iv) we can obtain
dx 3
 np 
y (n) = a n cos  ax + b + .
………………………………………..  2 
dn y
y (n) = = m(m − 1)(m − 2).......(m − n + 1)x m − n , n < m We can consolidate the above results in the form of a
dx n
table.
2.32  Differential Calculus

Table 2.4 y= y(n) =


y= y =
(n)
eax, where a is a constant an eax
x , where m is a positive
m
 m!
integer
m−n
 (m – n)! x , n < m sin (ax + b)  np 
a n sin  ax + b + .
  2 
m!, n=m
0, n>m
 cos (ax + b)
  np 
a n cos  ax + b + .
 2 
1 (–1)n n!a n
where a, b are
(ax + b) (ax + n)n +1
constants

C o nce p t S t r a n d
Concept Strand 23
1
y (7) =
1 7
2
( 2 )
13 sin 13x + 7 p + sin x + 7 p 
2 ( )
(i) Find the 4th derivative of
(3x + 4)
(ii) Find the 7th derivative of y = cos6x sin 7x
=
1 7
2
( )
13 sin 13x – p + sin x – p 
2 2  ( )

Solution
= =
1 7
2
(
2 )
–13 sin p – 13x – cos x 

4
d 4 y (−1) × 4! × 3
4
8 × 35 1
(i) = = = –137 cos 13x – cos x 
dx 4
(3x + 4) 5
(3x + 4)5 2
1
(ii) y = sin13x + sin x 
2

Tangents and Normals

Y Consider a curve whose equation is y = f(x). Let P(x1, y1)


y = f(x) represent a point on this curve (i.e., y1 = f(x1)). We have
already shown that the slope of the tangent at P to the curve
dy
y = f(x) is given by the value of the derivative or f ’(x)
P dx
at x = x1.
90° If m is the slope of the tangent at P, we have
normal at P
 dy 
m=  = f '(x1 )   (1)
O G X  dx  x = x
T 1

tangent The equation of the tangent at P to the curve y = f(x)


at P is given by
Fig. 2.15 y - y1 = m(x - x1)  (2), where m is given by (1).
Differential Calculus  2.33

The normal to a curve at a point is defined as the In cases, where f ’(x1) = 0, the tangent at that point
straight line passing through the point and perpendicular will be parallel to the x-axis and therefore the equation of
to the tangent at this point. the tangent will take the form y = y1. Also, when f ’(x1)
−1 −1 = 0, normal at the point x1 is perpendicular to the x-axis
Slope of the normal at P = =
f '(x1 ) m and therefore, the equation of the normal takes the form
x = x1.
The equation of the normal at P to the curve y = f(x)
is given by
−1
y - y1 = (x - x1)   (3)
m

C o nce p t S t r a n d
Concept Strand 24 Equation of the tangent at (2, 16) is y - 16 = 27(x - 2)
Find the equations of the tangent and normal at the point or 27x - y = 38
(2, 16) on the curve y = x3 + 4x2 - x - 6. −1
Slope of the normal at (2, 16) is
27
Solution The equation of the normal at (2, 16) is y - 16 =
−1
dy 2 27
y = x3 + 4x2 - x - 6 ⇒ dx = 3x + 8x − 1 (x - 2)
 dy  or 27y - 432 = - x + 2
Slope of the tangent at (2, 16) =  dx  = 3 × 4 + 16 − 1 = 27
(2,16) ⇒ x + 27y = 434

Sub tangent, sub normal, length Let P(x1, y1) represent a point on the curve y = f(x). PT is
of tangent and length of normal the tangent at P to the curve intersecting the x-axis at T.
PG is the normal at P to the curve intersecting the x-axis
at G.
Y
y = f(x) The portion of the tangent intercepted between the
curve and the x-axis i.e., PT, is called the ‘length of the
tangent’ corresponding to P. The portion of the nor-
P mal intercepted between the curve and the x-axis i.e.,
tangent at P PG, is called the ‘length of the normal’ corresponding
to P.
The projection of PT on the x-axis i.e., TM is called the
normal at P
‘sub tangent’ corresponding to P and the projection of PG
O T M G X on the x-axis i.e., MG is called the ‘sub normal’ correspond-
ing to P.
Let a be the angle made by the tangent at P with the
Fig. 2.16 x-axis. (refer Fig. 2.16)
2.34  Differential Calculus

Then, tan a = slope of the tangent at P = f ‘(x1) or y1


( )(x , y )
2
y ' ⇒ Length of tangent = PT = 1 + y'
1 1
y' at (x1 , y1 )

Also, PM = y1 2 2
From DPGM, PG2 = PM2 + MG2 = y 1 + (y 1 y ′) at (x , y )
PM  dy  1 1

From DPTM, = tan a =  


TM  dx  (x , y )
⇒ Length of normal =PG =  y 1 + y ′ 2 
1 1

  at (x1 , y1 )

PM y1 y For the curve y = f(x), corresponding to a point x on


⇒ Sub tangent = TM = = =
tan a  dy y ' at (x the curve,
 dx  1 , y1 )
(x 1 , y1 ) y
Length of tangent = 1 + y'2
MG  dy  y'
From DPGM, PM = tan a =  dx 
(x1 , y1 )
Length of normal = y 1 + y ' 2
⇒ Sub normal = MG = | PM tan a| = | y y ' |at (x , y ) 1 1

y
Again, from DPTM, Length of sub tangent =
y'
PT2 = PM2 + TM2
Length of sub normal = | y y’|
  y1  
2
 y12 
=  y1 +   
2
=  2 (1 + y ' 2 ) dy
  y '    y'  at (x1 , y1 ) where y ' = = f '(x).
at (x1 , y1 ) dx

C o nce p t S t r a n d
Concept Strand 25 y 2 4 4 5
Length of tangent = y ' 1 + y ' =
2
1+ 4 =
2
For the curve y2 = 16x, find the length of tangent, length at (1, 4)
of normal, length of sub tangent and length of sub normal
corresponding to the point (1, 4). =2 5

Length of normal = y 1 + y ' 2 =4 5


Solution at (1, 4)

We have, on differentiating the equation of the curve with y 4


respect to x, Length of sub tangent = = =2
y' at (1, 4)
2
dy dy 8
2y = 16 or =
dx dx y Length of sub normal = (y y’)at (1, 4) = 4 × 2 = 8.

 dy  8
\   = =2
 dx  at (1, 4) 4
Differential Calculus  2.35

Angle between two curves Let y = f(x) and y = g(x) be two curves intersect-
ing at P. (refer Fig. 2.14). PT1 and PT2 are the tangents
Angle between two curves at a point of their intersection to the curves y = f(x) and y = g(x) respectively at P. If
is defined as the acute angle between the tangents to the ∠T1PT­2 = q, we say that the two curves intersect or
curves at that point of intersection. cut at an angle q or the angle between the two curves at
P is q.
If m1 and m2 denote the slopes of the tangents PT1 and
PT2,
m1 = [f ’(x)] at P; m­2 = [g ’(x)] at P,

P y = g(x)
m1 − m 2
we then have tan q = which gives q.
1 + m1 m 2
θ
Results
y = f(x) T2
(i) The two curves are said to intersect orthogonally at P
T1 p
if q = . i.e., if 1 + m1 m2 = 0 or m1 m2 = -1.
2

(ii) The two curves are said to touch at P, if q = 0, i.e., if


Fig. 2.17 m1 = m2.

C o nce p t S t r a n d
Concept Strand 26 3 × 82
m1 = slope of tangent at (8, 16) to (1) = 4 × 16 = 3
Find the angle of intersection of the curves 2y2 = x3 and
y2 = 32 x at their point of intersection (8, 16). y2 = 32 x   (2)
Differentiating both sides with respect to x,
Solution dy dy 16
2y = 32 giving =
dx dx y
2y2 = x3   (1) 16
m2 = slope of tangent at (8, 16) to (2) =
= 1.
Differentiating both sides with respect to x, 16
If q is the angle between the two curves at (8, 16),
dy
4y = 3x 2 3−1 2 1
dx tan q = = =
1+3×1 4 2
dy 3x 2 1
or = ⇒ q = tan −1   .
dx 4y 2

Velocity, Acceleration and Rate of Change ds


We have already seen that represents the rate of
For the motion of a particle along a straight line, if s is the dt
distance traversed by the particle in time t, s is a function ds
change of s with respect to t or gives the velocity of the
of t. Let s = f(t) dt
2.36  Differential Calculus

ds dv d  ds  d 2 s
particle at time t or v = dt where, v represents the velocity Acceleration = =  = 2
dt dt  dt  dt
of the particle at time t.
dv dv  dv   ds  dv
Again, represents the rate of change of velocity We may also write as     = v
dt  
ds dt  ds
dt
with respect to time, which therefore represents the accel-
eration of the particle at time t.

C o nce p t S t r a n ds
Concept Strand 27 Since 6y = x3 + 2, differentiating both sides with re-
spect to t,
A particle is moving along a straight line starting from a
dy dx dy 3x 2 dx  x 2  dx
point O. If P is the position of the particle after time t the 6 = (3x 2 + 0) or = =
dt dt dt 6 dt  2  dt
distance OP traversed by the particle in time t denoted by
s is given by s = 10 + 27t + 2t3. Find the initial velocity and dy dx
the velocity and acceleration at the end of 4 seconds. Since we have to find the points x at which =8 ,
dt dt
we have
Solution 8
dx x 2 dx
= or x2 = 16 giving x = ± 4.
ds dt 2 dt
We have = 0 + 27 + 6t 2
dt 1
When x = 4, y = (64 + 2) = 11 and when x = -4,
Initial velocity of the particle is obtained by putting t = 0 in 6
ds 1 −31
the expression for and it is equal to 27 units y = (−64 + 2) = .
dt 6 3
d2s  31 
Acceleration at time t = = 12t The required points are (4, 11) and  −4, − .
dt 2  3 
Velocity at the end of 4 seconds
 ds  Concept Strand 29
=   = 27 + 96 = 123 units/sec
 dt  t = 4
The side of a square sheet of metal is increasing at the rate
Acceleration at the end of 4 seconds of 2.5 cm per minute. How fast is the area of the sheet in-
 d2s  creasing when the side is 9 cm long?
=  2 = 12 × 4 = 48 units/sec2 .
 dt  t = 4
Solution
Concept Strand 28
Let x denote a side of the square sheet and A denote the
A particle moves along the curve 6y = x3 + 2. Find the area of the sheet, A = x2   (1)
points on the curve at which the y coordinate is changing dx dA
We are given, = 2.5. we have to determine .
8 times as fast as the x coordinate. dt dt
Differentiating the relation (1) with respect to t,
Solution dA
= 2x
dx
= 2x × 2.5 = 5x ;
dx dt dt
If t represents time, represents the rate of change of x
dt dA
When x = 9, dt = 5 × 9 = 45
dy
with respect to time and represents the rate of change
dt ⇒ Rate of increase of the area when x = 9 is given by
of y with respect to time. 45 sq. cm/minute.
Differential Calculus  2.37

Mean Value Theorem and its Applications

Many of the results of this section depend on Mean Value Theorem. We therefore start this section with Rolle’s
Value Theorem and Rolle’s Theorem is pivotal to Mean Theorem.

Rolle’s Theorem

Let f(x) be a function of x defined in the closed interval Referring to the 3 figures given (Figures 2.18, 2.19,
[a, b] satisfying the following conditions: 2.20) it appears that when f(a) = f(b) = 0 there is at least
one point x = c on the graph of f(x) where the tangent is
(i) f(x) is continuous in the closed interval [a, b]
parallel to the x-axis which means that the derivative f ’(x)
(ii) f(x) is differentiable (i.e., f ’(x) exists) in the open
at x = c is zero.
interval (a, b) and
Suppose f(x) = k in [a, b]. Then it is obvious that f ’(x)
(iii) f(a) = f(b) = 0
= 0 everywhere in [a, b] and therefore, c can be taken any-
Then there exists at least one point c in (a, b), where between a and b. Suppose f(x) is not a constant func-
(a < c < b) such that f ’(c) = 0. tion [Figures 2.15 - 2.17]. Since f(x) is continuous in [a, b] it
Before outlining the proof of this theorem, let us look has a maximum, say M and a minimum, say m in this inter-
at the graph of a function that satisfies the three conditions val. Then at least one of these numbers is not equal to zero.
above:
Y
Y

y = f(x)

A B
X A B
a O c b X
O a c b

Fig. 2.18

Fig. 2.20
<

Let us assume that M > 0 and that f(x) takes on its


maximum value at x = c so that f(c) = M. Note that c cannot
coincide with a or b (since it is given that f(a) = f(b) = 0)
or, in other words, c lies between a and b. Since f(c) is the
$ F % maximum value of the function, f(c + h) - f(c) ≤ 0 both
; when h > 0 and when h < 0. This means that
2 D E
f(c + h) − f(c)
≤ 0 for h > 0  (1) and
h

Fig. 2.19 f(c + h) − f(c)


≥ 0 for h < 0   (2)
h
2.38  Differential Calculus

f(c + h) − f(c) It is easily verified that f(x) is continuous in [0, 4].


being ≤ 0 when h > 0 will tend to a Since f(x) has no unique tangent at x = 2 (Refer Fig.
h
2.22) it is not differentiable there (although f(x) is
f(c + h) − f(c)
limit ≤ 0 and being ≥ 0 when h < 0 will differentiable at all other points in (0, 4)).
h
tend to a limit ≥ 0. But, since f(x) is given to be differen- f(0) = f(4) = 0.
tiable in (a, b), f ’(c) exists. Therefore, these two limits must
be identically same and must represent f ’(c).This unique But, since the differentiability condition is not satisfied
limit can be negative and positive simultaneously only if in (0, 4), it is clear from the figure that f ’(x) does not
f ’(c) = 0. This completes the proof. vanish anywhere in (0, 4) or Rolle’s Theorem does not
hold good for this function.
Remarks
Let us illustrate the application of Rolle’s Theorem by some
(i) Rolle’s theorem holds good for a differentiable function
examples:
(i.e., for which the first two conditions are satisfied) where
the third condition is altered as f(a) = f(b) (instead of (i) Let the position of a moving object at time t be rep-
f(a) = f(b) = 0). resented by s = f(t). If the object is in the same place
It can be easily seen that it is equivalent to applying at two different instants t = t1 and t = t2, then, we have
Rolle’s Theorem to the function F(x) = f(x) - f(a) f(t1) = f(t2). Rolle’s Theorem says that there is some
(refer Fig. 2.21) instant of time t0­ between t1 and t2 (i.e., t1 < t0 < t2)
where f ’(t0) = 0 i.e., the velocity is 0 at t = t0.
(ii) Consider the 3rd degree algebraic equation x3 - 2x2 +
Y
5x - 1 = 0.

Let f(x) = x3 - 2x2 + 5x - 1


A Since f(x) is a polynomial, it is continuous. Also,
B y = f(x)
f(0) = -1 < 0 and f(1) = 1 - 2 + 5 - 1 > 0. By interme-
diate value theorem, there is a number c between 0
and 1 such that f(c) = 0. Thus the given equation has
a c b X a root between 0 and 1.
O
To show that the equation has no other real root we
use Rolle’s Theorem and argue by contradiction. Sup-
pose it had two roots a and b. Then f(a) = 0, f(b) =
Fig. 2.21
0, and since f(x) is a polynomial, it is continuous in
[a, b] and differentiable in (a, b). Thus, by Rolle’s
(ii) The differentiability of f(x) in (a, b) is essential for the Theorem there is a number x0 between a and b such
Rolle’s Theorem to hold good. that f ’(x0) = 0.
For example let us consider the function But f ’(x) = 3x2 - 4x + 5 > 0, since the discriminant
 x, 0 ≤ x ≤ 2 of this quadratic is less than zero. i.e., f ’(x) will never
f(x) =  vanish, which is a contradiction. The inference is that
4 − x, 2 < x ≤ 4
there cannot be two real roots for the given equa-
y
tion. Therefore, the given equation has one real root
between 0 and 1 and two complex roots.

P
Mean Value Theorem
[or Lagrange’s theorem]
A Let f(x) be a function of x satisfying the following condi-
x
O 2 4 tions:
(i) f(x) is continuous in the closed interval [a, b]
Fig. 2.22 (ii) f(x) is differentiable in the open interval (a, b).
Differential Calculus  2.39

Then there exists a number c in (a, b) (i.e., a < c < b)


Y
f(b) − f(a)
such that f '(c) =
(b − a)
or f(b) - f(a) = (b - a) f ’(c) P
B y = f(x)
This theorem is also called the first Mean Value Theo-
rem. Before proving the theorem, let us examine the geo-
metrical significance of the mean value theorem. A Q
The coordinates of the points A and B on the
curve (refer Fig. 2.23, 2.24) are (a, f(a)) and (b, f(b)) re-
spectively. The slope of the secant line AB is given by X
O M c1 c2 N
f(b) − f(a)
.
(b − a) Fig. 2.24
The tangent at P (corresponding to a point c between a
and b) is parallel to AB. The slope of the tangent at P is f ‘(c).
Mean Value theorem states that there is at least one
f(b) − f(a) point c between a and b such that the slope of the tangent at
\ f '(c) =
(b − a) c is equal to that of the secant line joining A and B.
We now prove the theorem.
Let f(x) be a function of x defined by
Y y = f(x) F(x) = f(x) - f(a) - (x - a)l  (1)
f(b) − f(a)
B
where, l =
(b − a)
Since f(x) is continuous in [a, b], F(x) is also continu-
P ous in [a, b]. Again, f(x) being differentiable in (a, b), F(x)
A
is also differentiable in (a, b).
From (1), F(a) = 0, F(b) = f(b) - f(a) - (b - a)l = 0
By applying Rolle’s Theorem to F(x), we can say that
T there exists a point c lying between a and b such that
X
O M R N
F’(c) = 0
From (1), F’(c) = f ’(c) - 0 - l
But F’(c) = 0
Fig. 2.23 f(b) − f(a) which proves the mean value
⇒ f ’(c) = l =
theorem. (b − a)

C o nce p t S t r a n ds
Concept Strand 30 f(3) − f(1) 8 + 2
= =5
Verify the mean value theorem for the function f(x) = 3x2 3−1 2
- 7x + 2 in the interval [1, 3]. f ’(x) = 6x - 7
f(3) − f(1)
and f '(c) = =5
Solution 3−1

f(x) being a polynomial in x is continuous and differen- ⇒ 6c - 7 = 5, giving c = 2 which lies in (1, 3).
tiable in R. It is interesting to note that the point c is the arithme-
f(1) = 3 - 7 + 2 = - 2; f(3) = 27 - 21 + 2 = 8 tic mean of 1 and 3.
2.40  Differential Calculus

Concept Strand 31 f ‘(x) = 2 sin x cos x = sin 2x and by Mean Value theorem
Verify Mean Value theorem for the function f(x) = sin2 x
f '(c) =
( 2 ) − f(0) = 2 .
f p
and a = 0, b = p 2 .
(p 2 − 0) p
Solution 2
\ sin 2c =
 p p
f(x) is continuous and differentiable in 0,  . f(0) = 0
 2 1 -1  2 
 p  ⇒ c = sin  p 
and f   = 1. 2
2
( )
f p − f(0) 2
2
= .
Note that c is about 36° and therefore c lies between
p
0 and .
( )
p −0
2
p 2

Results constant function. This does not contradict the result


above because the domain here is not an interval. The
(i) If f ‘(x) = 0 for all x in an interval (a, b), then f(x) is a
domain in this case is (-∞, 0) ∪ (0, ∞). However, in
constant in (a, b).
We outline the proof below: each of the constituent intervals the result is true.
Let x1 and x2 be any two numbers in (a, b) with (ii) As a byproduct of the above result, we have the fol-
x1 < x2. Since f(x) is continuous in [x1, x2] and differen- lowing result.
tiable in (x1, x2), by applying the Mean Value Theorem    If f ’(x) = g’(x) for all x in an interval (a, b), then
to f(x) in the interval [x1, x2], we have a number c f(x) - g(x) is a constant in (a, b).
f(x 2 ) − f(x1 ) For, let H(x) = f(x) - g(x). Then, H ‘(x) = f ‘(x) - g ‘(x)
between x1 and x2 such that f '(c) = or = 0 for all x in (a, b). We immediately conclude that
(x 2 − x1 )
H(x) is a constant in (a, b).
f(x2) - f(x1) = [f ‘(c)] (x2 - x1)
Proof is complete.
   Since f ‘(x) = 0 for all x in (x1, x2), we have f ‘(c) =
0 and this means that f(x2) = f(x1). Note:
   Therefore, f(x) has the same value at any two points Let x = a be a zero of order n of f(x), (or, in other words, a
x1, x2 in (a, b) or, in other words, f(x) has the same value is a root of the equation f(x) = 0 repeated n times) i.e., f(x)
at every point in (a, b) or f(x) is a constant in (a, b). = (x - a)n g(x), where g(a) ≠ 0
⇒ f ’(x) = (x - a)n g’(x) + n (x - a)n - 1 g(x)
Remarks
= (x - a)n - 1 [(x - a) g’(x) + n g(x)]
A word of caution is appropriate before we want to
= (x - a)n - 1 G(x), where G(a) ≠ 0
apply the above result.
⇒ x = a is a zero of order (n - 1) of f ’(x) (or a is a root of
 1, x > 0 the equation f ’(x) = 0 repeated (n - 1) times)
Suppose f(x) = 
 −1, x < 0 Consolidating,
   The domain of f(x) is {x \ x ≠ 0} and f ’(x) = 0 Let x = a be a zero of order n of f(x), then x = a is a
for all x in this domain. But f(x) is obviously not a zero of order (n - 1) of f ’(x).

L’ Hospital’s Rule

In this section we explain L’ Hospital’s rules which help called “Cauchy’s Theorem”. We therefore explain this
us in the evaluation of limits of functions. This rule theorem first and then proceed to take up L’ Hospital’s
comes as a direct application of an important theorem rules.
Differential Calculus  2.41

Cauchy’s theorem Suppose both f(x) and g(x) are continuous at x = a,


f(x) f(a)
If f(x) and g(x) are two functions continuous in [a, b] and lim = provided g(a) ≠ 0.
x → a g(x) g(a)
differentiable in (a, b) and g ‘(x) does not vanish anywhere
It may sometimes happen that both f(a) and g(a) are
in (a, b), then, there exists a point c where a < c < b such
zero. Then we land up in what is called the indeterminate
f '(c)
that f(b) − f(a) = 0
g(b) − g(a) g '(c) form .
0
The proof of the above theorem is outlined below. For example, this happens when we consider the prob-
f(b) − f(a) sin x sin x
Let us define the number l where l = lem lim . However, lim = 1.
g(b) − g(a) x →0 x x →0 x
  (1) In general, although both f(a) and g(a) are zero,
Note that [g(b) - g(a)] cannot be zero, since, if f(x)
g(b) = g(a), by Rolle’s Theorem, the derivative g’(x) will lim may exist.
x → a g(x)
vanish at some point between a and b and this contradicts
the hypothesis that g ’(x) does not vanish anywhere in We may encounter this form if lim f(x) = 0 and
x →a

(a, b). limg(x) = 0 , but lim


f(x)
may exist.
Consider the function H(x) = f(x) - f(a) - x →a x → a g(x)

l[g(x) - g(a)] For evaluation of limits which reduce to the indeter-


We note that H(a) = 0 and H(b) = 0 and that H(x) satis- 0
fies the continuity and differentiability conditions. Apply- minate form , L’ Hospital’s rule can be used. We now state
0
ing Rolle’s Theorem to H(x), we deduce that there exists a the rule.
point c between a and b such that H ’(c) = 0.
Now, H ’(x) = f ’(x) - 0 - l g ’(x) and H ’(c) = 0 gives
f ’(c) = l g ’(c) L’ Hospital’s Rule
f '(c) Let the functions f(x) and g(x), in some interval contain-
or l = , which proves the theorem.
g '(c) ing the point a satisfy the conditions of Cauchy’s theorem
You may recall that in the section on limits, we use the f(x)
and suppose f(a) = 0 = g(a). Then, if lim exists, then,
x → a g(x)
f(x) lim f(x)
f '(x)
result lim = x →a , provided that limg(x) ≠ 0. f(x)
x → a g(x) limg(x) x →a lim = lim
x →a x → a g(x) x → a g '(x)

C o nce p t S t r a n ds
Concept Strand 32 5cos5x 5cos0 5
= lim = =
sin 5x x →0 9 cos9x 9cos0 9
Evaluate : lim
x →0 sin 9x (It may be mentioned that the above limit can be ob-

Solution  sin5x 
sin 5x  5x  5
f(x) = sin 5x, g(x) = sin 9x and we have f(0) = 0, g(0) = 0. tained by writing sin9x as  sin9x  × 9 and, by using the
sin 5x 0  9x 
Therefore, reduces to the indeterminate form
sin 9x 0
when x takes the value zero.
Therefore, by applying L’ Hospital’s rule, sin x 5
limit lim =1 we obtain the limit as
x 9 
(sin5x )
x→ 0
sin5x d
lim = lim dx
dx (
x → 0 sin9x x→0 d
sin9x )
2.42  Differential Calculus

Concept Strand 33 By applying L’ Hospital’s rule,


d −x
ex − e− x (e − e − x )
Evaluate lim ex − e− x dx
x →0 sin x lim = lim
x → 0 sin x x →0 d
(sin x)
dx
Solution
ex + e− x 1 + 1
Here, f(x) = ex - e-x and g(x) = sin x. = lim = = 2.
x → 0 cos x 1
ex − e− x
f(0) = 0 = g(0) and therefore lim reduces to
x →0 sin x
0
the indeterminate form .
0

Remarks f '(x)
L’ Hospital’s rule to the ratio , we arrive at the
(i) The rule holds also for the case when the func- g '(x)
tions f(x) or g(x) are not defined at x =a, but f '(x) f "(x)
lim f(x) = 0 = limg(x). formula lim = lim
x →a x →a x →a g '(x) x →a g "(x)
(ii) The rule is also applicable if lim f(x) = 0, lim g(x) = 0.    The above rule can be extended. That is if f “(a) =
x →∞ x →∞ 0 = g “(a), and f “(x) and g”(x) satisfy the conditions
(iii) If f ’(a) = 0, g ’(a) = 0 and the derivatives f ’(x) and that were imposed by the rule on the functions f ‘(x)
g ’(x) satisfy the conditions that were imposed by the and g ‘(x), we have, on applying L’ Hospital’s rule to
rule on the functions f(x) and g(x), then, applying f "(x) f "(x) f '''(x)
the ratio ,lim = lim and so on.
g "(x) x → a g "(x) x → a g '''(x)
The following example demonstrates this concept very
clearly.

C o nce p t S t r a n d
Concept Strand 34  0 
 Again, 0 form 
x − sin x  
Find lim
x →0 5x 3 0 + sin x  0 
= lim , by L’ Hospital’s rule  Again, form 
Solution x →0 30x  0 
x − sin x  0  cos x
lim form  = lim , by L’ Hospital’s rule.
3  30
x →0 5x 0  x →0

1 − cos x 1
= lim , by applying L’ Hospital’s rule = .
x →0 15x 2 30

∞ a, excluding a) and g ‘(x) does not vanish in this neigh-


Limits involving indeterminate form of the type ∞ bourhood. Let lim f(x) = ∞ and limg(x) = ∞ and let
x →a x →a

f '(x)
Theorem lim exist.
x →a g '(x)
Let the functions f(x) and g(x) be continuous and dif- f(x) f '(x)
Then, lim = lim .
ferentiable for all x ≠ a (i.e., in a neighbourhood of the point x →a g(x) x → a g '(x)
Differential Calculus  2.43

Remark
finity and lim f(x) = ∞ and lim g(x) = ∞ . We have
The above theorem can be extended to the case x →∞ x →∞

f(x) f ′(x)
where we want the limit of f(x) as x tends to in- lim
x →∞ g(x)
= lim
x →∞ g ′(x)
g(x)

C o nce p t S t r a n d
Concept Strand 35 1
× (sec2 10x) × 10
tan 10x
log tan10x = lim , by L’ Hospital’s rule.
Find lim .
x →0 1
× (sec2 3x) × 3
x →0 log tan3x tan 3x

10 sec2 10x tan 3x


= × lim × lim
Solution 2
3 x → 0 sec 3x x → 0 tan 10x
d
log tan10x  ∞  (tan3x)
lim from  10 1 dx
x → 0 log tan3x  ∞ = × × lim , by L’ Hospital’s rule,
  3 1 x →0 d
(tan10x)
dx
d
(log tan 10x) tan3x 0
= lim dx , by L’ Hospital’s rule. since lim is of the form .
x →0 d
x →0 tan10x 0
(log tan 3x)
dx 10 (sec2 3x) 3 10 1 × 3
= × lim × = × = 1.
3 x → 0 (sec2 10x) 10 3 1 × 10

Limits involving indeterminate forms 0 × ∞, ∞ – ∞


0 ∞
The indeterminate forms of the above type can be evaluated by transforming them as a quotient of the form or to
0 ∞
which L’ Hospital’s rule is applied.

C o nce p t S t r a n ds
Concept Strand 36 −2e2x
= lim , by L’ Hospital’s rule
Find lim(1 − e2x )cot3x .
x →0 3sec2 3x
x →0
−2 × 1 −2
= 3×1 = 3 .
Solution
lim(1 − e2x )cot3x (= (1 - 1) × ∞ = 0 × ∞ form )
x →0
Concept Strand 37
 1 1
(1 − e2x )  0  Find lim  − 2
= lim  0 form  x → 0  sin 2 x x 
x → 0 tan3x
2.44  Differential Calculus

Solution 2 sin2x
= lim 2
The given limit takes (∞ - ∞) form. Rearranging,
x →0 (−2x sin2x + 2x cos2x + 4x
x 2 − sin2 x  0
cos2x + 2 sin2x + 2 sin x cos x)

Limit = lim  0 form 
x → 0 x 2 sin 2 x
(by L’ Hospital’s rule)
2x − 2sin x cos x 2 sin2x
= lim , by L’ Hospital’s = lim
x →0 x 2 × 2sin x cos x + 2x sin2 x x →0 (−2x 2 sin2x + 6x cos2x + 3 sin 2x)
rule.
0 
2x − sin2x  0 form 
0 
= lim 2  form 
x → 0 x sin 2x + 2x sin 2 x  0
4 cos2x
2 − 2cos2x lim 2
lim x →0 (−4x cos2x − 4x sin2x + 6
x →0 (x 2 × 2 cos2x + 2x sin 2x + 2x × =
= cos2x − 12x sin 2x + 6 cos 2x)
2 sin x cos x + 2 sin2 x)
(by L’ Hospital’s rule)
(by L’ Hospital’s rule)
1 − cos2x 0  4 1
= lim 2 = lim = .
 form 
x → 0 x cos2x + 2x sin 2x + sin 2 x)  0
x →0 (6 + 6) 3

Limits involving indeterminate forms 0 0, 1 ∞, ∞0 by taking logarithms and reducing them to one of the
0 ∞
Limits involving exponential expressions which reduce indeterminate forms or . L’ Hospital’s rule can then
0 ∞
to any of the indeterminate forms above are evaluated be applied.

C o nce p t S t r a n ds
Concept Strand 38 log (x − 1)  0 
= lim  0 form 
1 x →2 (x − 2)
Evaluate lim(x − 1)(x − 2) 1
x →2

(x − 1)
= lim , by L’ Hospital’s rule
Solution x →2 1
1

= 1⇒ L = e1 = e.
Note that lim(x − 1)(x − 2) is of [1 form]
x →2

Let lim(x − 1)(x − 2) = L . Concept Strand 39


x →2
p
−x
Taking logarithms on both sides, Evaluate lim (cos x) 2
x→p
1 2

log L = log lim(x − 1) x − 2


x →2
Solution
 1
 p
= lim
x →2
log (x − 1) x −2
 (interchanging the limit Note that lim (cos x) 2
−x
is of 00 form.
  x→p
2
operation and logarithm operation) p
−x
Let L = limp (cos x) 2 . Taking logarithms on both
1 x→
= lim log (x − 1) 2
x → 2 (x − 2)
sides, we get
Differential Calculus  2.45

p
log L = log lim (cos x) 2
−x p 
–  – x2 
x→p 2  0 
2 = lim  form 
x→ p cot x 0 
 p
−x  p  2
= lim log (cos x) 2  = lim  − x  log cos x
x→p x→p  2
2   2 p 
–2  – x  (−1)
log cos x  ∞  2 
= lim = lim , by L’ Hospital’s rule.
x→p 
 ∞ form  –cosec2 x
2  x→p
2
 1 
p  p 
 − x = (-1) = limp 2  − x  sin2 x
2  x→
2
2 
1
× –sin x =2×0×1=0
= lim cos x ,
x→ p
2   by L’ Hospital’s rule log L = 0 ⇒ L = e0 = 1.
 1 
 2

p  
  − x  
 2 

Extension of the Mean Value Theorem

Lagrange’s theorem or the first Mean Value Theorem states Again, if f(x) is differentiable any number of times in
that if a function f(x) is continuous in [a, b] and differen- a neighbourhood of a point a, it is proved that the infinite
tiable in (a, b), there exists a number c between a and b h2 h3
such that series f(a) + h f ’(a) + f ”(a) + f ’’’(a) + …∞   (4)
2! 3!
f(b) − f(a) converges to f(a + h) in an interval containing the point ‘a’
= f '(c) or f(b) = f(a) + (b - a) f ’(c)
(b − a) provided some conditions are satisfied.
By writing b = a +h, (where h > 0 or < 0 depending on This means that the sum of the first r terms of the series
the position of b with respect to a on the real line), we may (4) will approach f(a + h) as r is increased indefinitely OR
write the Mean Value Theorem in the form we can make the difference between the sum of the first r
f(a + h) = f(a) + h f ’(a + qh)  (1), where h = (b - a) terms of the series (4) and f(a + h) as small as we please by
and 0 < q < 1. taking sufficiently large values for r.
Suppose f ’(x) is continuous in [a, b] and f ”(x) exists in We write,
(a, b) (i.e., f ’(x) is differentiable in (a, b)), we can prove that h2 h3
f(a + h) = f(a) + h f ’(a) + f ”(a) + f ’’’(a) + …∞
2! 3!
h2
f(a + h) = f(a) + h f ‘(a) + f ”(a + qh)   (2),   (5) ,
2!
 where h = (b - a) and 0 < q < 1. valid in some interval containing the point a.
If a is replaced by 0 and h is replaced by x in (5), we get
(2) is an extension of the Mean Value Theorem.
We can extend the above result further as follows. x2 x3
f(x) = f(0) + x f ’(0) + f ”(0) + f ’’’(0) + …∞   (6)
Suppose f(x), f ’(x), f ”(x),…..,f (n - 1)x are continuous in 2! 3!
[a, b] and f (n)(x) exists in (a, b) (i.e., f (n - 1)x is differentiable provided f(x) is differentiable any number of times
in (a, b)), we can show that in a neighbourhood of the point 0 (i.e., in an interval
h2 h3 -l < x < l, (l > 0) where l is the distance of the nearest point
f(a + h) = f(a) + h f ’(a) + f ”(a) + f ’’’(a) + …+ of discontinuity of f(x) from 0) and some other conditions
2! 3!
hn n are satisfied.
f (a + qh)   (3), We note that (6) helps us to represent f(x) as a series
n!
where, h = (b - a) and 0 < q < 1. in ascending powers of x or we say that f(x) has the power
2.46  Differential Calculus

series representation (6) about x = 0. Using (6), we can cos x 1


write the power series representation of the functions ex, = lim , by L’ Hospital’s rule = .
x →0 2 2
sin x, cos x, loge(1 + x), loge(1 - x).
We list below power series expansions of a few impor- OR
tant elementary functions. 1 − cos x 2sin2 x
lim = lim 2
2 3
x x x x x →0 x 2 x →0 x2
(i) e = 1 + + + + ......∞ , valid for all finite x.
1! 2! 3! 2 2
 sin x   x 
x3
x x 5 7
= lim  2 × 1 1  sin 2 
= lim
(ii) sin x = x − + −
3! 5! 7!
+ −......∞ , valid for all finite x. x →0 

x ( )

2 
2 x
2
→0 2 

x 
2  ( )
2 4 6
x x x 1 1
(iii) cos x = 1 − + − + −......∞ , valid for all finite x. = × 12 = .
2! 4! 6! 2 2

x2 x3 x 4 1 + log x − x
(iv) log e (1 + x) = x − + − + −......∞ , -1 < x ≤ 1 (ii) Consider another problem: lim
2 3 4 x →1 1 − 2x + x 2

x2 x3 x 4 Let x = 1 + h. As x → 1, h → 0,
(v) log e (1 − x) = − x − − − − ......∞ , -1 ≤ x < 1.
1 + log x − x
2 3 4
substituting x = 1 + h in , we get
1 − 2x + x 2
1 1+ x x3 x5 x7
(vi) log e =x+ + + + ......∞ , -1 < x < 1.
2 1− x 3 5 7 1 + log x − x 1 + log x − x 1 + log(1 + h) − (1 + h)
= =
1 − 2x + x 2 (x − 1)2 h2
Remarks
 h2 h3 
Power series expansions above can be used for evaluating  h − 2 + 3 − +....  − h
limits of functions. We illustrate this procedure by working = (since h → 0 we may assume
h2
out two examples.
1 − cos x | h| < 1)
(i) Consider the problem of evaluating lim .
x →0 x 2
−1
= + terms involving h and higher powers of h →
Using the expansion of cos x, 2
 x2 x 4 x6 
1 − 1 − + − + −..... −1
1 − cos x  2! 4! 6! 
as h → 0.
= 2
x2 x2
−1
Hence, the limit equals .
x2 x 4 x6 2
− + + −.....
2! 4! 6!
= OR
x2
We can also evaluate the above limit by applying L’
1 x2 x 4 1 Hospital’s rule.
= − + − ..... → as x → 0.
2 4! 6! 2
1 + log x − x  0 
1 lim  form 
Therefore, the required limit = . x →1 1 − 2x + x 2  0
2
 1 
OR −1
 x  0 
= lim  , by L’ Hospital’s rule  form 
We can also evaluate the above limit by applying L’ x →1 −2 + 2x   0 
 
Hospital’s rule.  
1 − cos x  0  −1
lim .  form  x 2 , by L’ Hospital’ rule
x →0 x2 0  = lim
x →1 2
sin x 0  −1
= lim , by L’ Hospital’s rule  form  = .
x →0 2x 0  2
Differential Calculus  2.47

Increasing and Decreasing Functions

Definition 1 Definition 2
A function defined in an interval Ι is said to be increasing in Ι A function defined in an interval Ι is said to be decreasing in
if f(x1) ≤ f(x2) whenever x1 < x2 where x1, x2 are in Ι. Ι if f(x1) ≥ f(x2) whenever x1 < x2 where x1, x2 are in Ι
(Refer Fig. 2.25 and 2.26) (Refer Fig. 2.27 and 2.28)
Consider the following examples:
(i) f(x) = x2, 0 ≤ x < ∞ is an increasing function in [0, ∞). Y
 2x + 5, 0 < x ≤ 1

(ii) f(x) =  7, 1 < x ≤ 2 is an increasing function in
y = f(x)
 x + 5, 2 < x ≤ 10

[0, 10].

y = f(x)
X
O
x1 x2

Fig. 2.27

Y
X
O x1 x2
y = f(x)
Fig. 2.25

X
O x1 x2

y = f(x) Fig. 2.28

Consider the following examples:


(i) f(x) = x2 + 3, 1 ≤ x ≤ 6 is an increasing function in
X [1, 6]
O x1 x2 1
(ii) f(x) = 2x2 - x + 3, ≤ x ≤ 2 is an increasing function
4
Fig. 2.26 1 
in  , 2
4 
2.48  Differential Calculus

2 Geometrically speaking, if, in an interval, the func-


(iii) f(x) = 2
, 0 ≤ x ≤ 5 is a decreasing function in [0, 5]. tion f(x) is increasing, the tangent to the curve y = f(x) at
x +1
each point of this interval makes acute angle with the x-
6 − 4x, 0 ≤ x ≤ 1
 axis (or the tangent is horizontal at certain points) (refer
(iv) f(x) =  2, 1 < x ≤ 4 is a decreasing function in Fig. 2.29) or the slope of the tangent at each point of this
6 − x, 4 < x ≤ 8
 interval is non-negative or f ’(x) ≥ 0 at every point in this
[0, ∞). interval.
If in an interval the function f(x) is decreasing the
tangent to the curve y = f(x) at each point of this interval
Monotonic Functions makes obtuse angle with the x-axis (or the tangent is hori-
zontal at certain points) (refer Fig. 2.30) or the slope of the
A function f(x) defined in an interval I is said to be mono-
tangent at each point of this interval is non positive or f ’(x)
tonic increasing (or strictly increasing) if f(x1) < f(x2) when-
≤ 0 at every point in this interval
ever x1 < x2, where x1, x2 are in I.
A function f(x) defined in an interval I is said to be
monotonic decreasing (or strictly decreasing) if f(x1) > f(x2) Y
whenever x1 > x2, where x1, x2 are in I.
A function f(x) which is strictly increasing or strictly
decreasing is called a monotonic function. y = f(x)
Examples (i), (ii) (iii) and (iv) above are monotonic
functions. P1
(i) and (ii) are monotonic increasing while (iii) and
(iv) are monotonic decreasing. T2 P2
P3
Remark
If a function f(x) is monotonic in an interval I, it is injective
(or it is a one one mapping) in I. X
O
Results T1 T3
(i) If f(x) is continuous and differentiable in (a, b) and
f ’(x) ≥ 0 in this interval, then f(x) is increasing in (a, b). Fig. 2.30
(ii) If f(x) is continuous and differentiable in (a, b) and
f ’(x) ≤ 0 in this interval, then f(x) is decreasing in
(a, b). We now prove the above result.
Suppose f(x) is increasing in (a, b).
Then, f(x + h) > f(x) for h > 0 and f(x + h) < f(x) for
Y h < 0.
f(x + h) − f(x)
y = f(x) In both cases is positive and conse-
P4 h
P3 f(x + h) − f(x)
T3 quently lim ≥ 0 which means that f ’(x) ≥ 0
P2 x →0 h
in this interval.
P1 Suppose f(x) is decreasing in (a, b).
T4 Then, f(x + h) < f(x) for h > 0 and f(x + h) > f(x) for
h < 0.
a X f(x + h) − f(x)
O b In both cases is negative and conse-
T2 h
T1 f(x + h) − f(x)
quently lim ≤ 0 which means that f ’(x) ≤ 0
x →0 h
Fig. 2.29 in this interval.
Differential Calculus  2.49

Theorem Suppose f ’(x) > 0 in (a, b). Then f ’(l) > 0. This means
that f(x2) - f(x1) > 0 or f(x2) > f(x1) or f(x) is an increasing
Let a function f(x) be continuous in an interval [a, b] and be
function in this interval.
differentiable in (a, b).
The proof for (ii) follows in a similar way.
(i) If f’(x) > 0 for a < x < b, then f(x) increases in For example,
(a, b). (i) f(x) = ex is monotonic increasing in (-∞, ∞).
(ii) If f’(x) < 0 for a < x < b, then f(x) decreases in For, f ’(x) = ex > 0 for x ∈(-∞, ∞)
(a, b). (ii) f(x) = x3 - 6x2 - 36x + 7
Proof ⇒  f ’(x) = 3x2 - 12x - 36 = 3(x + 2) (x - 6)
   Note that f ’(x) > 0 if x lies beyond -2 and 6 and
Let x1, x2 (x1 < x2) denote two points in [a, b]. Since f(x) f ’(x) < 0 if x lies between -2 and 6
satisfies the conditions of the Mean Value Theorem, ⇒ f(x) is increasing in (-∞, -2) and (6, ∞) and is
f(x2) - f(x1) = f ’(l) (x2 - x­1) where x1 < l < x2. decreasing in (-2, 6).

C o nce p t S t r a n ds
Concept Strand 40 1  sin x  1
= + sin x − tan x
x 2  cos2 x  2
Find the interval in which f(x) = is increasing.
(x 2 + 4) sin x + sin x cos2 x − 2sin x cos x
=
Solution 2 cos2 x
x (sin x)(1 + cos2 x − 2 cos x)
f(x) = =
(x 2 + 4) 2cos 2 x

(x 2 + 4) − 2x 2 4 − x2
(sin x)(1 − cos x)2 p
⇒ f '(x) = = 2 = > 0 for 0 < x < .
2
(x + 4) 2
(x + 4)2 2cos2 x 2

f(x) is increasing if f ’(x) > 0 p


\ f(x) is increasing in 0 < x < . Since, f(0) = 0, f(x) > 0
4−x 2 2
i.e., when > 0 ⇒ 4 - x2 > 0 p
(x 2 + 4)2 in 0 < x <
,
2
⇒ x2 - 4 < 0 ⇒ result follows.
⇒ f(x) is increasing, when x lies between -2 and 2.

Concept Strand 42
Concept Strand 41
p
p Prove that sin x < x, for 0 < x ≤ .
Prove that for 0 < x < , the function, 2
2
f(x) =
1
sin x tan x − log sec x is positive and increasing. Solution
2
Consider the function f(x) = sin x - x.
p
Solution f(0) = 0 and f ’(x) = cos x - 1 < 0, for 0 < x ≤ .
2
1 \ f(x) is decreasing in this interval.
f(x) = sin x tan x − log sec x
2 p
Since f(0) = 0, f(x) < 0 in 0 < x ≤
We have, f(0) = 0 2
1 1 1 p
f '(x) = sin x sec2 x + tan x cos x − × tan x sec x ⇒ in x < x in 0 < x ≤ .
2 2 sec x 2
2.50  Differential Calculus

Maxima and Minima of functions

Definition 1 y
The function f(x) is said to have a maximum (or we say that
f(x) attains a maximum) at a point x0 if the value of f(x) at y = f(x)
x0 is greater than its values for all x in a small neighbour-
hood of x0. In other words, f(x) has a maximum at x = x0, if
f(x0 + h) < f(x0) for any h (positive or negative) that are
sufficiently small in absolute value (Refer (i) and (ii) of P
Fig. 2.31)

Y x
P O x0
y = f(x)
(i)
y

y = f(x)
x0 X
O
(i)
x0
Y P x
O
y = f(x)

P
x0 X
O (ii)
(ii) Fig. 2.32
Fig. 2.31

(ii) If a function f(x) has a maximum or a minimum at a


Definition 2
point x0, f(x) is said to have an extremum at x0. Also,
f(x) is said to have a minimum (or we say that f(x) attains a the point where the function is an extremum is called
minimum) at a point x0 if the value of f(x) at x0 is less than a ‘stationary point’ or a ‘turning point’ or a ‘critical
its values for all x in a small neighbourhood of x0. In other point’ of the function.
words, f(x) has a minimum at x = x0, if f(x0 + h) > f(x0) for
any h (positive or negative) that are sufficiently small in ab- y
solute value (Refer (i) and (ii) of Fig. 2.32). P3 y = f(x)
Remarks P5
(i) In an interval, say [a, b], f(x) may attain maximum and P1
minimum at a number of points. Referring to Fig. 2.33,
f(x) has a maximum at the points P1, P3 and P5 and f(x) P6
has a minimum at the points P2, P4 and P6. When we say
P4
that f(x) is a maximum at a point, say x0, it only means P2
that f(x0) has the largest value in comparison with those
values of f(x) at all points x sufficiently close to x0. For x0 x
O a b
this reason, we may say that f(x) has a local maximum
at P1 (or at P3 or at P5). Similarly, we may say that f(x) Fig. 2.33
has a local minimum at P2 (or at P4 or at P6).
Differential Calculus  2.51

Necessary condition for a function to have an This means that f ’(x) is negative for x < x0 and f ’(x) is
extremum positive for x > x0, where x is a point close to x0. Since, f ’(x)
exists (or f ’(x) is continuous) in this small interval contain-
Let f(x) be differentiable in a small interval containing x0, ing x0, f ’(x) must become zero at x = x0. Observe that the
including x0. This means that f ’(x0) exists. tangent at x0 to the curve is parallel to the x-axis.
Suppose f(x) is a maximum at x0. Note that the tan- We therefore infer that, if at the point x = x0, a differ-
gents at points x, to the left of x0, which are close to x0 make entiable function f(x) has an extremum, its derivative must
acute angles with the x-axis and the tangents at points x vanish at x0, i.e., f ’(x0) = 0.
to the right of x0, which are close to x0 make obtuse angles From the above discussions, we conclude that, if f(x)
with the x-axis (refer Fig. 2.34). is differentiable in an interval, then it can have a maximum
or a minimum only at those points x where the derivative
Y f ’(x) vanishes or at those points where f ’(x) = 0. We hasten
P to add that this does not mean that f(x) has an extremum at
“every point x” where f ’(x) = 0.
P1 P2 For example, consider the function f(x) = 2x3. We have,
f ’(x) = 6x2 and f ’(x) = 0 when x = 0.

Y
T1 T2

O X
x0 y = 2x3

Fig. 2.34
O X
This means that f ’(x) is positive for x < x0 and f ’(x)
is negative for x > x0, where x is a point close to x0. Since,
f ’(x) exists (or f ’(x) is continuous) in this small inter-
val containing x0, f ’(x) must become zero at x = x0. Ob-
serve that the tangent at x0 to the curve is parallel to the
x-axis. Fig. 2.36
Suppose f(x) is a minimum at x0.
It can be noted that the tangents at points x, to the left
However, as can be seen from Fig. 2.36, f(x) has neither
of x0, which are close to x0 make obtuse angles with the x-
a maximum nor a minimum at x = 0. In fact, for x < 0, f(x)
axis and the tangents at points x to the right of x0, which are
< 0 and x > 0, f(x) > 0 where x is close to 0.
close to x0 make acute angles with the x-axis (refer Fig. 2.35).
The above analysis was for functions which are differ-
entiable in the intervals under consideration. Suppose f(x)
Y is differentiable in a small interval containing x0 except at
T1 x0. i.e., f ’(x) exists for all x in this interval but f ’(x0) does not
exist. Referring to (ii) of Fig. 2.31, although f ’(x) does not
T2 exist at x0, f(x) is maximum at x0.
P1 Similar is the case with (ii) of Fig. 2.32. Here, f(x) is
a minimum at x0 although f ’(x0) does not exist. Thus, we
P P2 see that f(x) may have an extremum at points where f(x) is
not differentiable. That is, it is a critical point of f(x). Here
also, not all points where f(x) is not differentiable are criti-
O x0 X
cal points of f(x). (We will illustrate this case by an example
later).
Winding up our discussions, we can now say the fol-
Fig. 2.35
lowing:
2.52  Differential Calculus

A function can have an extremum only in two cases: (ii) If f ’(x) changes sign from negative to positive as one
moves from left to right through x0, f(x) has a mini-
(i) At points where the derivative exists and is zero
mum at x0.
OR
(ii) At points where the derivative does not exist.
In both cases above
Suppose f ’(x) does not change sign as one moves from left
Sufficient conditions for a function to have an to right through x0, then f(x) has neither a maximum nor a
extremum minimum at x0.
The conditions above for testing a function for maxi-
Case 1 mum or minimum are called first derivative tests.
Let f(x) be a differentiable function in an interval I and let    We present the above said results in a tabular form below:
x0 ∈ I. x0 is a critical point of f(x)
Then we have seen that if i.e.,  (i)  f ’(x0) = 0 OR
(ii)  f ’(x) does not exist at x0.
(i) f ’(x0) = 0 and
(ii) f ’(x) > 0 for x < x0 and f ’(x) < 0 for x > x0, where Table 2.5
x is a point close to x0, then f(x) has a maximum Sign of the derivative f ’(x)
at x0. when passing through Nature of the
Or, in other words, as one moves from left to right the critical point x0 critical point
through x0, if f ’(x) changes sign from positive to negative, x < x0 x = x0 x > x0
and f ’(x­0) = 0, then f(x) has a maximum at x0. + f ’(x0) = 0 – f(x) is a maximum
Again if at x0.
or f ’(x) does not
(i) f ’(x0) = 0 and exist at x0­
(ii) f ’(x) < 0 for x < x0 and f ’(x) > 0 for x > x0, where x is – f ’(x0) = 0 + f(x) is a minimum
a point close to x0, then f(x) has a minimum at x0. at x0.
or f ’(x) does not
Or, in other words, as one moves from left to right exist at x0.
through x0, if f ’(x) changes sign from negative to positive + f ’(x0) = 0 + f(x) is neither a
and f ’(x0) = 0, then f(x) has a minimum at x0. maximum nor a
or f ’(x) does not
Case 2 exist at x0. minimum at x0.

Let f(x) be a differentiable function in an interval I contain- – f ’(x0) = 0 – f(x) is neither a


ing the point x0 except at x0. i.e., f ’(x) exists for all x in I or f ’(x) does not maximum nor a
except at x0 (i.e., f ’(x0) does not exist). exist at x0. minimum at x0.

(i) If f ’(x) changes sign from positive to negative as one The following examples are worked out to illustrate
moves from left to right through x0, f(x) has a maxi- how the first derivative test is applied to investigate the
mum at x0­. maxima, minima of functions.

C o nce p t S t r a n ds
Concept Strand 43 Differentiating once we get, y ’ = x2 - 4x + 3 = (x - 1)
(x - 3)
Find the maximum and minimum points and values of the \ The stationary points are x = 1 and x = 3.
x3
function y = f(x) = − 2x 2 + 3x + 1 at these points. In a neighbourhood of 1
3
(y ’)1– is > 0 and (y ’)1+ is < 0.
Solution Therefore, f(x) is a maximum at x = 1 and the maxi-
Since f(x) is a polynomial, it is differentiable any number 1 7
mum value is f(1) = − 2 + 3+1 = .
of times. 3 3
Differential Calculus  2.53

In a neighbourhood of 3 4
(y ’)3– is < 0 and (y ’)3+ is >0. Considering the point x = ,
5
Therefore, f(x) is a minimum at x = 3 and the mini-
mum value is f(3) = 9 - 18 + 9 + 1 = 1. (y ')(4 5) −
< 0 and ( y ') 4
( 5)
+ >0

4
⇒ f(x) is minimum at x = .
Concept Strand 44 5
1 Minimum value of
Find the extreme points of y = f(x) = 3 - 2 ( x + 1) 3
2 2

1 −2 4 4  4 3
−6  4  3
−2
f(x) = f   =  − 2    =
y’ = −2 × (x + 1) 3 =
3 2 5 5  5 5  5 
3(x + 1) 3
Considering the point x = 0,
f ’(x) ≠ 0 and f ’(x) does not exist at x = -1. ⇒ The
only critical point is x = -1.
(y ')0− > 0 and (y ')0+ < 0
In a neighbourhood of –1 ⇒ f(x) is maximum at x = 0.

(y ’) -1– < 0 and (y ’)-1+ < 0 Maximum value of f(x) = f(0) = 0.


The graph of the function is shown below.
which means that f(x) has neither a maximum nor a mini-
mum at x = -1. Y

Concept Strand 45
4
2
O 5
Find the maxima and minima of y = f(x) = (x − 2)x 3
X
2
2 −1 2 2(x − 2) 2
y’ = (x − 2) × x 3 + x 3 = 1
+x 3
3 3x 3

2(x − 2) = 3x 5x − 4
= 1
= 1
3x 3
3x 3

dy 4 Testing a function f(x) for maxima or minima was


= 0 at x = . We note that y’ is not defined at based on the changes in the sign of the derivative f ’(x) as
dx 5
we pass through a critical point x0 of f(x). Suppose f ’(x)
x = 0. is differentiable in a neighbourhood of x0 including x0. In
4 other words, we assume that f ”(x) exists in this neighbour-
\ x = 5 and x = 0 are critical points of f(x).
hood (which includes x0).

When f(x) is a maximum at x = x0, we have f ’(x0) =


0 and f ’(x) changes sign from positive to negative as one
passes through x0 from left to right. This means that f ’(x) Y Y
is a decreasing function in this neighbourhood of x0 (refer y = f ’(x)
(i) of Fig. 2.37) in which case f ”(x0) must be negative. y = f ’(x)
Similarly, when f(x) is a minimum at x = x0, we have
f ’(x0) = 0 and f ’(x) changes sign from negative to positive X X
as one passes through x0 from left to right or f ’(x) is an in- O x0 O x0
creasing function in this neighbourhood of x0­ (refer (ii) of
Fig. 2.34) in which case f ”(x0) must be positive. (i) (ii)
We are now in a position to give the second derivative Fig. 2.37
test for investigation of extrema of a function.
2.54  Differential Calculus

Second Derivative Test Remark

Let f(x) be twice differentiable in an interval I. Also let x0 ∈ If f ”(x) = 0, we may not be in a position to say anything
I be a critical point of f(x). Then, about the extrema of f(x), or the test fails in this case.
f(x) is a maximum at x0 if f(x) is a minimum at x0 if We can then use the first derivative test and solve the
(i) f ‘(x0) = 0 and (i) f ‘(x0­) = 0 and problem.
(ii) f “(x0) < 0. (ii) f “(x0) > 0.    A few examples are worked out below illustrating the
use of the second derivative test.

C o nce p t S t r a n ds
Concept Strand 46 (iii) Let y = 3x4
⇒ y ’ = 12x3 and y ” = 36x2
Find the maximum and minimum values of the following y ’ = 0 gives x = 0. However, (y ’’)x = 0 = 0.
expressions:    We are therefore not in a position to draw any
(i) x3 - 9x2 + 15x + 3 inference regarding the extremum (if any) of the
x function at x = 0.
(ii)    We now use the first derivative test.
(1 + x 2 )
(iii) 3x4 (y ’)0- < 0 and (y ’)0+ > 0

Solution    y is therefore minimum at x = 0 and the minimum


value = 0.
(i) Let y = x3 - 9x2 + 15x + 3
⇒ y ’ = 3x2 - 18x + 15 and y ’’ = 6x - 18
Critical points of the function are obtained by solving Concept Strand 47
y ’ = 0.
This gives x2 - 6x + 5 = 0 ⇒ x = 1, 5 Find two positive numbers x and y such that their sum is
60 and xy3 is a maximum.
(y ’’)x = 1 < 0 and (y ’’)x = 5 > 0
   Therefore, the function is maximum at x = 1 and Solution
minimum at x = 5.
Maximum value of y = f(1) = 10 We are given that x + y = 60 and we have to find the values
Minimum value of y = f(5) = -22. of x and y for which xy3 is maximum.
x (1 + x 2 ) − 2x 2 (1 − x 2 ) Since x + y = 60, x = 60 - y
(ii) Let y = ⇒ y' = 2 2
= Let u = xy3; Then u = (60 - y)y3 = 60y3 - y4
2
(1 + x ) (1 + x ) (1 + x 2 )2
   To maximise xy3 ⇒ to maximise u
(1 + x 2 )2 × (−2x) − (1 − x 2 ) × 2(1 + x 2 ) × 2x du d2u
y “ = u is a maximum if = 0 and 2 < 0 .
(1 + x 2 )4
3
dy dy
(2x − 6x)
=
(1 + x 2 )3 du d2 u
= 180y 2 − 4y 3 and = 360y − 12y 2
dy dy 2
y ’ = 0 gives x = ± 1
du
The critical points of the function are x = -1 and = 0 ⇒ y = 0 or 45.
dy
x = + 1.
We find (y ’’)x =-1 > 0 and (y ’’)x = +1 < 0 y ≠ 0, since y is positive. \ y = 45.
   Therefore, f(x) is maximum at x = 1 and minimum d2 u
at x = -!. When y = 45, = 360 × 45 - 12 × 452 < 0.
dy 2
1 −1 Therefore, u is maximum when y = 45 and x = 15.
Maximum value = ; Minimum value = .
2 2 \ The maximum value of u = xy3 = 15 × 453
Differential Calculus  2.55

Convexity and Concavity of a curve

Let P be a point on a curve and PT be the tangent at P to the Generally, it is important to consider the convexity or
curve. Let the tangent meet a given line L at A. concavity of a curve at a point with respect to the x-axis
rather than an arbitrary line.
Definition 1
The curve is said to be ‘convex’ at P with respect to the line L
Y
if a sufficiently small arc of the curve in a neighbourhood of P
T
lies entirely outside the region bounded by AL and AT (Refer
(i) of Fig. 2.38). P y = f(x)

T
X
P O

Fig. 2.40
L
A
(i)
Referring to Fig. 2.40, the curve y = f(x) is concave
T with respect to the x-axis while it is convex with respect to
P the y-axis at the point P.

Y y = f(x)
L T
A
(ii)
P
Fig. 2.38
X
O
Definition 2
Fig. 2.41
The curve is said to be ‘concave’ at P with respect to the line L
if a sufficiently small arc of the curve in a neighbourhood of
P lies entirely inside the region bounded by AL and AT (Refer Referring to Fig. 2.41, the curve y = f(x) is convex with
(ii) of Fig. 2.38) respect to the x-axis while it is concave with respect to the
y-axis at the point P. and
Definition 3 The curve y = f(x) has a point of inflexion at the point
The point P on the curve that separates the convex part of the P (refer Fig. 2.42)
curve from the concave part of the curve is called a point of
inflexion (Refer Fig. 2.39). Y
y = f(x)
T
T
P
P

O X

Fig. 2.39 Fig. 2.42


2.56  Differential Calculus

Results (ii) The curve y = f(x) has a point of inflexion at x0 if


f ”(x) changes sign (from positive to negative or from
Let f(x) be twice differentiable in an interval I and let x0 be
negative to positive) when passing through x0 and
a point belonging to I. Then,
f ”(x0) = 0.
(i) y = f(x) is convex with respect to the x-axis at the
point x0 if f(x0).f ”(x0) > 0. Again, y = f(x) is con- Note
cave with respect to the x-axis at the point x0 if Convexity, concavity in this result means convexity, con-
f(x0).f ”(x0) < 0. cavity with respect to the x-axis.

C o nce p t S t r a n ds
Concept Strand 48 \ f(x).f "(x) = − sin2 x < 0 ∀ x
Verify the following statements: Result follows.
(Refer Figure)
(i) The function f(x) = ex is convex in R.
 p (iii) f(3) = -2, f "(3) = 2
(ii) f(x) = sin x is concave in  0, 
 2 \ f(3) f "(3) < 0
(iii) f(x) = x2 - 7x + 10 is concave at x = 3
(iv) f(x) = x2 - 7x + 15 is convex at x = 3 \ f(x) is concave at x = 3, with respect to x axis

Solution
(i) True. The function f(x) = ex is convex in R. y = x2 –7x + 10
For, f ’(x) = ex, f ”(x) = ex > 0 for x ∈R.
f(x) f "(x) = e .e = (e ) > 0 ∈ x
x x x 2

\ f(x) is convex in R (Refer Figure)


Y 1 2 3 4 5 6

y = ex
1 (iv) f(3) = 3, f "(3) = 2

X \ f(3) f "(3) > 0


O
\ f(x) is convex at x = 3, with respect to x axis
 p
(ii) True. f(x) = sin x is concave in  0,
 2 
 p y = x2 –7x + 15
f ’(x) = cos x and f ”(x) = - sin x < 0 in  0, 
 2
Y
y = sin x
1

O X
π
2
1 2 3 4 5 6
Differential Calculus  2.57

Concept Strand 49 \ The curve is convex for x ≠ 0 at x = 0,


Prove that f(x) = x3 is neither convex nor concave at x = 0. f '(x) = 0 and f "(x) = 0 , but f '(x) > 0 for x ≠ 0
\ f(x) is increasing.
\ The function f(x) = x3 has a point of inflexion at
Solution
x = 0.
f(x) = x3 ⇒ f ’(x) = 3x2 and f ”(x) = 6x. (If f '(x 0 ) = 0, f "(x 0 ) = 0 and f '''(x 0 ) ≠ 0 then f(x) has
Note that both f ’(x) and f ”(x) vanish at x = 0.
We observe that f(x).f ”(x) > 0 ∀ x, x ≠ 0 a point of inflexion at x = x0)

s U M M AR Y
1. Limit of a function
Definition  Let f(x) be a function of x. We write lim f(x) = L and say “the limit of f(x) as x approaches x0 equals
x → x0

L” if we can make the values of f(x) arbitrarily close to L (as close to L as we like) by taking x sufficiently close to
x0 but not equal to x0.
Results:
sin x tan x xn − an
(i)  lim = 1 (ii)  lim =1 (iii)  lim = na n −1 ,
x →0 x x →0 x x →a x − a

n a rational number (a > 0)


1 x
 1  ax −1
(iv)  lim (1 + x ) = e
x
(v)  lim 1 +  = e (vi)  lim  = log e a
x →0 x →∞  x x →0  x 

(1 + x )
n
 ex − 1  −1  a x − bx  a 
(vii)  lim  =1 (viii)  lim =n (ix)  lim   = log e  b 
x →0  x  x →0 x x →0  x
(a, b > 0)
Left hand limit  We write lim− f(x) = L1 and say that the left hand limit (L.H.L) of f(x) as x approaches x0 from
x → x0
the left is equal to L1 if we can make the values of f(x) arbitrarily close to L1 by taking x sufficiently close to x0 and
x less than x0.
Right hand limit  We write lim+ f(x) = L2 and say that the right hand limit (R.H.L) of f(x) as x approaches x0
x → x0

from the right is equal to L2 if we can make the values of f(x) arbitrarily close to L2 by taking x sufficiently close
to x0 and x greater than x0.
Method to find left hand and right hand limits
L.H.L = lim− f(x) = lim f ( x 0 − h )
x → x0 h→0

R.H.L = lim+ f(x) = lim f ( x 0 + h )


x → x0 h→0

where h > 0 in both cases


2. Continuity of a function
(i)  A function f(x) is said to be continuous at a point x0 if lim f(x) = f ( x 0 )
x → x0

(ii)  If the function f(x) is not continuous at x = x0, then it is said to be discontinuous at x = x0
(iii) A function f(x) is continuous from the right at x = x0 if lim+ f(x) = f ( x 0 ) and f(x) is continuous from left
x → x0
at x = x0 if lim− f(x) = f(x0).
x → x0
2.58  Differential Calculus

(iv) If g(x) is continuous at x0 and f(x) is continuous at x0, then the composite function f o g(x) = f(g(x)) is con-
tinuous at g(x0).
(v) f(x) is continuous in [a, b] if it is continuous at every point in [a, b].
(vi) Intermediate value theorem If f(x) is continuous in [a, b] and f(a) and f(b) are of opposite signs, then there
exists a point x0 ∈ (a, b) such that f(x0) = 0
(vii) Polynomial function is continuous every where. Rational functions, root functions, trigonometric functions,
inverse trigonometric functions, logarithmic functions, exponential functions are continuous at all points
in their respective domains.
(viii) f(x) is said to have a removable discontinuity at x = x0 if lim f(x) exists but f(x0) does not exist. By redefin-
x → x0
ing the function f(x) at x = x0 as f(x0) = lim f(x) , f(x) becomes continuous at x = x0
x → x0
3. Differentiation
f ( x 0 + h ) − f(x 0 )
(i) Derivative of a function y = f(x) at x = x0 denoted by f ’(x0) is defined as f ’(x0) = lim
h→0 h
(ii)  If f(x) is differentiable at a point x0, it is continuous at x0
(iii)  A function f(x) fails to be differentiable at a point x0 if
(a)  f(x) is not continuous at x0
OR
(b)  the graph of f(x) changes direction abruptly at x0 or in other words, it has no unique tangent at x0
OR
(c) the graph of y = f(x) has a vertical tangent at x0 i.e., f(x) is continuous at x0 and |f ’(x)| tends to infinity as
x tends to x0.
4. Derivatives of functions

d d n
(i) 
dx
(c) = 0 , c is a constant (ii) 
dx
( )
x = n x n −1 (n rational)

d x d 1
(iii) 
dx
( )
e = ex (iv) 
dx
(log x ) =
x
d x d
(v) 
dx
( )
a = ax loga, a > 0 (vi) 
dx
(sin x ) = cos x
d d
(vii)  (cos x ) = - sinx (viii)  (tan x ) = sec2 x
dx dx
d d
(ix)  (cosec x) = - cosec x cotx (x)  (sec x ) = secx tanx
dx dx
d d 1
(xi) 
dx
(cot x) = − cos ec2x (xii) 
dx
( )
sin −1 x = , x <1
1 − x2

d −1 d 1
(xiii) 
dx
(
cos −1 x = ) , x <1 (xiv) 
dx
( )
tan −1 x =
1 + x2
1 − x2

d 1 d dv du
(xv) 
dx
( )
sec −1 x =
2
, x >1 (xvi) 
dx
(uv ) = u + v
dx dx
x x −1
du dv
v −u
d u dx dx
(xvii)  = (xviii)  Chain Rule
dx  v  v2
Differential Calculus  2.59

dy dy du d x
If y = f(u) , u = g(x), then = .
dx du dx
(xix) 
dx
( )
x = x x (1 + log x )

(xx) Parametric differentiation


dy du
dy du dx
If x = f(t), y = g(t) then = dt (xxi) u = f(x), v = g(x) then =
dx dx dv dv
dt dx
d
5. Successive differentiation y = f(x) be a function of x. ( y ) or f ’(x) is the first order derivative of f(x).
dx
dy d2 y d3 y
Differentiating with respect to x again we get 2 , differentiating again we get etc. Differentiating f(x)
dx dx dx 3
dn y
n times with respect to x we get the nth order derivative of y = f(x) denoted by y(n) or
dx n
Results
(i)  if y = xm
 m! (m − n )
 (m − n )! .x n<m

where m is a positive integer, y(n) =  m! n=m
 0 n>m


(−1) n!.a n
n
1
(ii)  if y = , then y (n) =
(ax + b)
n +1
ax + b

(−1) (n − 1)!.a n
n −1

(iii)  if y = log (ax + b), then y = (n)

(ax + b)
n

(iv)  if y = eax, then y(n) = an eax


 np 
(v)  if y = sin(ax + b), then y(n) = an sin  ax + b + 
 2 
 np 
(vi)  if y = cos(ax + b), then y(n) = an cos  ax + b + 
 2 
6. Slope of a tangent
dy
y = f(x) be a continuous function. If P(x1, y1) is a point on the curve, then at P represents the slope of the
dx
tangent to this curve at P.
 dy 
7. Equation of the tangent at (x1, y1) to the curve y = f(x) is y - y1 =   (x - x1).
 dx  x , y
1 1

8. Equation of the normal at (x1, y1) is

1
y - y1 = − ( x − x1 )
 dy 
 dx 
(x , y )
1 1
2.60  Differential Calculus

2
 dy 
y 1+  
 dx 
9. Length of the tangent at (x, y) =
dy
dx

2
 dy 
10. Length of the normal at (x, y) = y 1 +  
 dx 

y
11. Length of the sub tangent at (x, y) = dy
dx

dy
12. Length of the subnormal at (x, y) = y
dx
m1 − m 2
13. Angle q between two curves at a point of intersection of the curves, is given by tan q = where, m1
1 + m1m 2
and m2 denote the slopes of the tangents to the two curves at their point of intersection.

14. Rolle’s theorem  If f(x) is continuous in [a, b], differentiable in (a, b) and f(a) = f(b), then there exist atleast one
point c in (a, b) such that f ’(c) = 0
15. Lagrange’s theorem or Lagrange’s Mean Value theorem  If f(x) is continuous in [a, b] differentiable in (a, b),
f(b) − f(a)
then there exist a point c ∈ (a, b) such that f ’(c) =
b−a
dx
16. Percentage Error  Let dx be an error in the independent variable x. Then is called relative error in x and
x
dx
× 100 is called the percentage error in x.
x

17. L’ Hospital’s Rule


(i) Let the functions f(x) and g(x), in some interval containing a point a satisfy the conditions of Cauchy’s theorem
f(x) f(x) f ' (x )
and suppose f(a) = 0 = g(a). Then, if lim exists, then lim = lim
x →a g(x) x → a g(x) x →a g ' (x )

(ii) Let the functions f(x) and g(x) be differentiable for all x ≠ a (i.e., in a neighbourhood of the point a, exclud-
f(x)
ing a) and g’(x) does not vanish in this neighbourhood. Let lim f(x) = ∞, lim g(x) = ∞ and let lim
x →a x →a x → a g(x)

f(x) f ' (x )
exist. Then, lim = lim
x → a g(x) x →a g ' (x )

18. Increasing and Decreasing functions A function f(x) is said to be strictly increasing in [a, b] if f ’(x) > 0 for all
x ∈ [a, b] and f(x) is strictly decreasing in [a, b] if f ’(x) < 0 for all x ∈ [a, b]. A function f(x) is said to be monotonic
in an interval [a, b] if it is either strictly increasing or strictly decreasing in [a, b]
Differential Calculus  2.61

19. First derivative test for maximum and minimum

Sign of the derivative f ‘(x) when passing


through the critical point x0 Nature of the critical point
x < x0 x = x0 x > x0
+ f ’(x0) = 0 – f(x) is a maximum at x0.
or f ’(x) does not exist at x0­
– f ’(x0) = 0 + f(x) is a minimum at x0.
or f ’(x) does not exist at x0.
+ f ’(x0) = 0 + f(x) is neither a maximum nor a
or f ‘(x) does not exist at x0. minimum at x0.
– f ’(x0) = 0 – f(x) is neither a maximum nor a
or f ’(x) does not exist at x0. minimum at x0.

20. Second derivative test for maximum and minimum Let f(x) is twice differentiable in an interval [a, b] and
x0 ∈ [a, b], f ’(x0) = 0 and f ”(x0) < 0, then x0 is a point of maximum. If f ’(x0) = 0 and f ”(x0) > 0, then x0 is point
of minimum.
21. Convexity and Concavity of a Curve
An arc of a curve y = f(x) is said to concave upwards (convex) of at each of its points the arc lies above the tangent
at the point and f ”(x) > 0.
   An arc of a curve y = f(x) is said to be concave downwards if at each of its points, the arc lies below the tangent
at the point and f ”(x) < 0.
   A point on the curve f(x) is said to be point of inflexion if the curve changes from convex to concave or vice
versa at that point.
2.62  Differential Calculus

C o nce p t co n n ec to rs

sin x
Connector 1: lim
x →∞ x
sin x 1
Solution: We have ≤ , since |sin x| ≤ 1.
x x

1 sin x
tends to zero as x tends to infinity. Therefore, lim = 0.
x x →∞ x
1
Connector 2: Show that the function f(x) = sin does not have a limit as x tends to zero.
x
1
Solution: As x approaches zero, sin takes values between -1 and +1 and therefore does not approach a definite
x
1
value as x approaches zero. We conclude that sin does not have a limit as x tends to zero.
x
2x 3 + 3x 2 − 7
Connector 3: lim
x →∞ 5x 3 − x 2 + 8

Solution: Dividing numerator and denominator by x3 (i.e., by the highest power of x),
3 7
2+
− 3
x x
Limit = xlim
→∞ 1 8
5− + 3
x x
2+0−0 1 1 2
= (since and 3 tend to zero as x tends to infinity) = .
5−0+0 x x 5

x + 2 − 2x
Connector 4: lim
x →2 x 2 − 2x

x + 2 − 2x (x + 2) − 2x −1
Solution: = 2 =
x 2 − 2x (x − 2x)( x + 2 + 2x ) x ( x + 2 + 2x )
−1
Note that the above expression tends to as x tends to 2
8
−1
Note that the limit is obtained by directly substituting x = 2 in
x ( x + 2 + 2x )
−1
Hence, the limit = .
8

Connector 5: lim
x →∞
( x 2 + 3x + 9 − x 2 + 9 )
(x 2 + 3x + 9) − (x 2 + 9) 3x
Solution: x 2 + 3x + 9 − x 2 + 9 = 2 2
=
x + 3x + 9 + x + 9 x + 3x + 9 + x 2 + 9
2

3
= , (on dividing numerator and denominator by x.)
3 9 9
1+ + 2 + 1+ 2
x x x
Differential Calculus  2.63

3 9 9
As x tends to infinity, both 1 + + and 1+ tend to 1.
x x2 x2
3 3
Therefore, the limit is = .
(1 + 1) 2

sin x − sin b
Connector 6: lim
x →b x−b

x + b x − b  x − b
2cos  sin  sin 
sin x − sin b 2   2   x + b  2 
Solution: = = cos  
(x − b) (x − b)  2  x − b
 2 
x − b
As x → b,  → 0.
 2 

x − b
sin 
x + b  2 
Limit = limcos  × lim = cos b × 1 = cos b.
x →b  2  x −b → 0  x − b 
2
 2 

1 − tan x
Connector 7: limp
x→
4
1 − 2 sin x

1 − tan x (1 − tan x)(1 + 2 sin x) (1 − tan x)(1 + 2 sin x)


Solution: = =
1 − 2 sin x 1 − 2sin2 x cos 2x
2
(1 − tan x)(1 + 2 sin x)(1 + tan x) (1 + 2 sin x) (1 + tan2 x)
= =
(1 − tan2 x) (1 + tan x)
 1 
1 + 2 ×  (1 + 1)
2
1 − tan x (1 + 2 sin x)(1 + tan x)  2
lim = lim = =2
x → 1 − 2 sin x
π
x→
π (1 + tan x) (1 + 1)
4 4

|x|
Connector 8: Prove that lim does not exist.
x →0 x

 −x
,x < 0
| x |  x | x |  −1, x < 0
Solution: x =  or =
 +x , x > 0 x +1 , x > 0
 x

|x| |x|
Clearly, xlim = −1 and lim+ = +1.
→0 −
x x →0 x
|x|
Since the right and left limits are different, lim does not exist.
x →0 x

Evaluate lim (1 + x)(1 + x 2 )(1 + x 4 )(1 + x 8 )....(1 + x 2 ) , |x| < 1.
n
Connector 9:
n →∞

(1 + x)(1 − x)(1 + x 2 )
Solution: (1 + x)(1 + x 2 ) =
(1 − x)
(1 − x )(1 + x 2 )
2
1 − x4
= =
(1 − x) 1− x
2.64  Differential Calculus

1 − x8
(1 + x) (1 + x2) (1 + x4) =
1− x
………………………………….
………………………………….
2n + 1

(1 + x) (1 + x2) (1 + x4)….. (1 + x
2n
) = 1 1− −x x →
1
(1 − x)
as n → ∞, since |x| < 1.
2
Connector 10: Check for continuity and differentiability of the function f(x) = 1 − 1 − x at x = 0.
Solution: Note that f(0) = 0.
2
f(h) = 1 − 1 − h
It follows that as h → 0 through positive or negative values, f(h) tends to zero.
or f(x) is continuous at x = 0.
1
f(0 + h) − f(0) 1 − 1 − h2 − 0 1 − (1 − h 2 ) 2
Now, consider = =   (1)
h h h
1 1
(1 − h 2 ) 2
= 1 − h 2 + terms involving powers of h greater than or equal to 4.
2
(we use the binomial expansion since |h| < 1).
2
1 1 2
\ 1 − (1 − h ) 2
= h + terms involving powers of h greater than or equal to 4.
2

1 − 1 − h2 1
⇒ = + terms involving h
h 2
f(0 − h) − f(0) −1 f(0 + h) − f(0) 1
So, lim
(h > 0) = and lim (h > 0) =
−h h→0 2 h→0 h 2
We see that f ’(0 ) ≠ f ’(0 ) or f(x) is not differentiable at x = 0.
- +

Connector 11: Test for continuity and differentiability of the function f(x) = x + | x–1| + 2 | x–2 |.
 x + (1 − x) + 2(2 − x), x < 1

Solution: We have f(x) =  x + (x − 1) + (2 − x)2, 1 < x < 2
 x + x − 1 + 2(x − 2), x > 2

5 − 2x, x < 1
 f(1) = 3,
= 3, 1 < x < 2 and
4x − 5, x > 2 f(2) = 3

Since f(x) is a polynomial, f(x) is continuous in (-∞, 1), (1, 2) and (2, ∞) and also differentiable in these
intervals. We have therefore to test the continuity and differentiability of f(x) at the points 1 and 2 only.
f(1-) = 5–2 × 1 = 3;f(1+) = 3 and f(1) = 3 ⇒ f(x) is continuous at x = 1.
Again, f(2-) = 3; f(2+) = 4 × 2 - 5 = 3 and f(2) = 3 ⇒ f(x) is continuous at x = 2
To test the differentiability of f(x), we proceed as follows:
At x = 1
f(1 − h) − f(1) 5 − 2(1 − h) − 3
lim (h > 0) = lim = lim(−2) = −2
h→0 (−h) h → 0 (−h) h→0

f(1 + h) − f(1) 3−3


lim (h > 0) = lim =0
h→0 h h→0 h

⇒ Left derivative at x = 1 i.e., f ’(1-) = - 2 and right derivative at x = 1 i.e., f ’ (1+) = 0


Differential Calculus  2.65

⇒ f(x) is not differentiable at x = 1


At x = 2
f(2 − h) − f(2) 3−3
lim (h > 0) = lim
h → 0 (− h)
= 0 ⇒ f ‘ (2-) = 0
h→0 (−h)
lim f(2 + h) − f(2) (h > 0)
h→0 h
4(2 + h) − 5 − 3
= lim = lim 4 = 4 ⇒ f ’ (2+) = 4
h→0 h h→0

⇒ f(x) is not differentiable at x = 2.


Connector 12: Find a function f(x): R → R which is differentiable exactly at one point.
Solution: The required function f(x) can be defined as follows:
Let f(x) = x2 when x is rational and f(x) = 0 when x is irrational.
When x is rational
f(x) − f(0)
lim = lim x = 0
x →0 x−0 x →0

When x is irrational,
f(x) − f(0) 0
lim = lim =0
x →0 x−0 x →0 x
Hence, f(x) is differentiable at x = 0
Let us consider a point say x0 ≠ 0
When x approaches x0 through rational values,
f(x) − f(x 0 ) x 2 − x 02
lim = lim = 2x0
x → x0 x − x0 x → x0 x − x
0
When x approaches x0 through irrational values,
f(x) − f(x 0 ) 0−0
lim = lim =0
x → x0 x − x0 x → x0 x − x0
Therefore, f(x) is not differentiable at any point x0 ≠ 0
Connector 13: Find the domain and range of f(x) = cot-1 x. Also find its derivative.
Solution: From the graph of f(x) = cot-1x, we see that the domain of f(x) is (-∞, ∞) and its range is (0, p).
(- ∞, ∞) → (0, p) : y = f(x) = cot -1 x is bijective. Therefore, the y
inverse of f(x) exists.
We have therefore, x = cot y π
dx
= - cosec2y = - (1 + cot2y) = - (1 + x2) π
dy 2
dy −1 x
or = o
dx (1 + x 2 )

Connector 14: Find the derivatives of (i) y = cosec-1 x, (ii) y = sec-1x


Solution: (i) y = cosec-1 x.
 p p
Domain of the function is (-∞, -1] ∪ [1, ∞) and its range is  − , 
 2 2
1
y = cosec-1 x = sin −1  
x
dy 1  −1  −1
= × 2 =
dx  x
2  x x2 − 1
1
1−  
x
2.66  Differential Calculus

(ii) y = sec-1x.
Domain of the function is (-∞, -1] ∪[1, ∞)
Range is [0, p]
1
y = sec-1x = cos −1  
x
dy −1  −1  1
= × 2 =
dx 
2 x  x x2 − 1
1
1−  
x

Connector 15: If f(x) =


x2 x x2 + 1
2
+
2
+ log (x + )
x 2 + 1 , show that x f ’(x) + log f ‘(x) = 2 f(x).

Solution: f(x) =
x2 x x2 + 1 1
2
+
2
+ log x + x 2 + 1
2
( )

2 2 2
2 f(x) = x + x x + 1 + log x + x + 1  ( )  (1)

Differentiating both sides of the above relation with respect to x,


1 1  1 
2 f ’(x) = 2x + x × × 2x + x 2 + 1 + 1 + × 2x 
2 2 2
2 x +1 x + x +1 2 x +1 

x2 1
= 2x + + x2 + 1 +
x2 + 1 x2 + 1

x2 + x2 + 1 + 1
= 2x + = 2x + 2 x 2 + 1
x2 + 1

or f ’(x) = x + x 2 + 1   (2)
\ (1) ⇒ 2f(x) = xf ’(x) + log f ’(x)
x.... ∞ dy y2
Connector 16: If y = x x , show that = .
dx x(1 − y log x)
Solution: We have y = xy
Taking logarithms both sides,
log y = y log x
Differentiating with respect to x,
log y
log x =
y
1
y. − log y
\ 1= y 2 dy
x y dx
dy
\y2 = x (1- log y)
dx
dy y2
\ =
dx x (1 − y log x )
Differential Calculus  2.67

Connector 17: If Sn stands for the sum of the first n terms of the G.P. a + ar + ar2 + …… show that
dSn
(r - 1) − (n − 1)Sn + nSn −1 = 0 .
dr

a(r n − 1)
Solution: We have Sn =
(r − 1)

or (r - 1)Sn = a(rn - 1)
Differentiating both sides with respect to r,
dSn
(r - 1) + Sn = na r n −1 = n(Sn − Sn −1 )
dr
Result follows.
2 2
 dx   dy 
Connector 18: If x = f(t) cos t - f ’(t) sin t and y = f(t) sin t + f ’(t) cos t, obtain the value of   +  
 dt   dt 
dx
Solution: = f '(t) cos t − f(t) sin t − f '(t) cos t – f ’’(t) sin t
dt
= -{ f(t) + f ”(t)} sin t.
dy
= f '(t) sin t + f(t) cos t − f '(t) sin t + f "(t) cos t = (f(t) + f ”(t)) cos t
dt
2 2
 dx   dy  2
 dt  +  dt  =  f(t) + f "(t) (sin t + cos t) = [f(t) + f ”(t)] .
2 2 2

dy
Connector 19: If sin (xy) + cos (xy) = tan (x + y), find .
dx
Solution: Differentiating the given relation with respect to x,
 dy   dy   dy 
cos (xy)  x + y  − sin(xy)  x + y  = sec2(x + y) 1 + 
 dx   dx   dx 

 dy 
⇒   [x cos (xy) - x sin (xy) - sec2 (x + y)]
 dx 
= sec2 (x + y) - y cos (xy) + y sin (xy)
2
dy sec (x + y) + y(sin(xy) − cos(xy))
⇒ = .
dx x(cos(xy) − sin(xy)) − sec2 (x + y)

1 +  y(sin(xy) − cos(xy)) cos2 (x + y)


=
x(cos(xy) − sin(xy))cos2 (x + y) − 1

Connector 20:. Differentiate x sin-1 x with respect to tan-1 x.


Solution: Let u = x sin-1x and v = tan-1x
du
We require
dv
1
du x + sin −1 x
du dx 1 − x 2
=
dv
=
dv  1 
dx  
1 + x2 

 x 
= (1 + x2)  + sin −1 x  .
 1− x 2

2.68  Differential Calculus

Connector 21: Let f(x) be a quadratic expression that is greater than zero for all real x. If g(x) = f(x) + f ’(x) + f ”(x) prove
that g(x) > 0 for all real x.
Solution: Let f(x) = ax2 + bx + c
Given that f(x) > 0 for all real x. This means that a > 0 and b2 - 4ac < 0   (1)
Now, g(x) = (ax + bx + c) + (2ax + b) + 2a
2

= ax2 + (b + 2a)x + (2a + b + c)


Discriminant of the quadratic function g(x) is given by
(b + 2a)2 - 4a(2a + b + c) = b2 - 4ac - 4a2 < 0 by (1)
Coefficient of x2 in g(x) = a > 0. by (1).
We infer that g(x) > 0 for all real x.
d  3 d2 y 
Connector 22: If y2 = P(x) where, P(x) is a polynomial of degree 3, show that 2 y = P(x) P'''(x) .
dx  dx 2 
Solution: Differentiating both sides with respect to x,
2yy’ = P ’(x)   (1)
Differentiating (1) with respect to x,
2y '2 + 2yy '' = P''(x)   (2)
Differentiating once more with respect to x,
4y’ y’’ + 2y y’’’ + 2y’ y’’ = P’’’ (x)
(ie) 6y’ y’’ + 2y y’’’ p’’’(x)   (3)
d 3
 y y '' 2
dx 
= 2 [3y2 y’ y’’ + y3 y’’’] = y2 [6y’ y” + 2y y’”]
= P(x) P”’ (x) using (3)
−1 d2 y dy
Connector 23: If y = x e x
, prove that x 3 −x + y = 0.
dx 2 dx
−1
Solution: y = x e x

−1
dy −1 −1 1 −1 e x
= e x + xe x × 2 = e x +
dx x x

dy −1 −1 −1
⇒ x = xe x + e x = y + e x   (1)
dx
Differentiating the above relation with respect to x,
−1
d 2 y dy dy e x
x + = + 2
dx 2 dx dx x

3 d2 y −1 dy
or x 2
=e x =x − y , from (1).
dx dx
Result follows.
Connector 24: Find the nth derivative of y = cos 3x cos 5x sin 6x.
1 1
Solution: y = (2cos 3x cos 5x) sin 6x = (cos 8x + cos 2x) sin 6x
2 2
1
= (2 cos 8x sin 6x + 2 sin 6x cos 2x)
4
Differential Calculus  2.69

1
= [(sin 14x - sin 2x ) + (sin 8x + sin 4x)]
4
1
= [sin 14x + sin 8x + sin 4x - sin 2x]
4
dn y 1  np  n  np 
= 14 n sin 14x + + 8 sin  8x +
dx n 4  2   2 

 np  n  np  
+4 n sin  4x +  − 2 sin  2x +
 2   2 


Connector 25: Find the equations of the tangent and normal at the point t on the curve whose equation is represented in
the parametric form x = at2, y = 2at.
Solution: x = at2, y = 2 at
On eliminating t, the equation of the curve is y2 = 4ax which is a parabola.
Point t means the point whose coordinates are (at2, 2at).
dy
dy dt = 2a = 1
=
dx dx 2at t
dt

dy 1
Slope of the tangent at t = at t =
dx t
1 x
Equation of the tangent at t on the curve is y - 2at = (x - at2) ⇒ y = + at
t t
Slope of the normal at t = -t
Equation of the normal at t is y - 2at = -t (x - at2)
⇒ y + xt = 2at + at3
Connector 26: Find the equations of the tangent and normal at the point q on the curves
(i)  x = a cos q, y = b sin q
(ii)  x = a sec q, y = b tan q
Solution: (i) On eliminating q from the two relations, the equation of the curve is
x2 y 2
+ =1
a 2 b2
dy

dy
= dq = bcos q
dx dx −a sin q
dq
Equation of the tangent at q (i.e., at the point whose coordinates are (a cosq, b sinq)), is
bcos q
y - b sinq = (x - a cosq) or ay sin q - ab sin2q = -bx cosq + abcos2q
−a sin q
⇒ bx cos q + ay sin q = ab (sin2q + cos2q)
x cos q y sin q
⇒ + =1
a b
−1 a sin q
Slope of the normal at q is = =
dy bcos q
dx
2.70  Differential Calculus

Equation of the normal at q is


a sin q
y - b sinq = (x - a cosq) or yb cos q - b2 sin q cosq
bcos q
= ax sin q - a2 sin q cos q
⇒ ax sinq - by cos q = (a2 - b2) sin q cos q
ax by
⇒ − = a 2 − b2
cos q sin q
x2 y 2
(ii) on eliminating q from the relations, the equation of the curve is − =1
a 2 b2
dy 2

dy
= dq = bsec q = bsec q
dx dx a sec q tan q a tan q
dq
 quation of the tangent at q (i.e., at the point whose coordinates are
E
(a sec q, b tan q) is
bsec q
y - b tan q = (x - a sec q) or ay tan q - ab tan2 q
a tan q
= x b secq - ab sec2q
xsec q y tan q
⇒ bx sec q - ay tan q = ab (sec2q - tan2q) = ab ⇒ − =1
a b
−1 −a tan q
Slope of the normal at ‘q’ is = =
dy bsec q
dx
Equation of the normal at q is
−a tan q
y - b tan q = (x - a sec q) or by sec q - b2 sec q tan q
bsec q
= - ax tan q + a2 sec q tan q
⇒ ax tan q + by sec q = (a2 + b2) sec q tanq
ax by
⇒ + = (a2 + b2)
sec q tan q
Connector 27: Find the equations of the tangent and normal for the curve y2(a + x) = x2(3a - x) at the points where, x = a.
x 2 (3a − x)
Solution: y 2 =   (1)
(a + x)
a 2 × 2a
When x = a, y2 = = a 2 ⇒ y ± a.
2a
The points at which equations of the tangent and normal are required are (a, a) and (a, -a).
Differentiating (1) with respect to x,
dy (a + x)[6ax − 3x 2 ] − (3ax 2 − x 3 )
2y =
dx (a + x)2
Point (a, a)
(2a)(3a 2 ) − 2a 3 1
Slope of the tangent = = .
4a 2 × (2a) 2
Slope of the normal = -2.
1
Equation of the tangent at (a, a) is (y - a) = (x - a)
2
⇒ x - 2y + a = 0
Differential Calculus  2.71

Equation of the normal at (a, a) is (y - a) = -2(x - a)


⇒ 2x + y - 3a = 0
Point (a, – a)
−1
Slope of the tangent = and slope of the normal = 2.
2
−1
Equation of the tangent at (a, -a) is (y + a) = (x − a)
2
⇒ x + 2y + a = 0
Equation of the normal at (a, -a) is (y + a) = 2(x - a)
⇒ 2x - y - 3a = 0
Connector 28: Find the equations of the tangents drawn to the curve y2 - 2x3 - 4y + 8 = 0 from the point (1, 2).
Solution: y2 - 2x3 - 4y + 8 = 0   (1)
Differentiating (1) with respect to x
dy dy dy 3x 2
2y − 6x 2 − 4 =0⇒ =
dx dx dx (y − 2)
Let the coordinates of the point of contact of a tangent from (1, 2) to (1) be (h, k).
3h 2
Then, slope of the tangent to the curve at (h, k) is .
(k − 2)
Equation of the tangent at (h, k) and passing through (1, 2) is
3h 2
y-2= (x − 1)   (2)
k−2
(h, k) lies on (1) and (2).
⇒ k2 - 2h3 - 4k + 8 = 0 ⇒ (k - 2)2 = 2h3 - 4   (3)
3h 2
and (k - 2) = (h − 1)   (4)
(k − 2)
From (3) and (4),
3h3 - 3h2 = 2h3 - 4 or h3 - 3h2 + 4 = 0
⇒ (h + 1) (h - 2)2 = 0 giving h = -1, 2.
Putting h = -1 in (3) we find that k is complex. Hence, h = -1 is not admissible.
Putting h = 2 in (3),
(k - 2)2 = 12 ⇒ k = 2 ± 2 3.

(
The points of contact are 2, 2 + 2 3 and 2, 2 − 2 3 . ) ( )
The equation of the tangents are 3y = 6x + 2 3 − 6 and 3 y = − 6x + 6 + 2 3

Connector 29: The tangent at P(x1, y1) on the curve xmyn = am + n meets the x and y axes in M and N. Find MP : PN.
Solution: We have m log x + n log y = (m + n) log a
 my 1 
On differentiating with respect to x, the slope of the tangent at P(x1, y1) is obtained as  −
 nx1 
The equation of the tangent at P on the curve is
 my 1 
y - y1 =  − (x − x1 )
 nx1 
or my1 x + nx1 y = (m + n) x1 y1
2.72  Differential Calculus

On putting y = 0 and x = 0 in the above equation we readily Y


obtain the coordinates of M and N as
N
 m + n    m + n  m+n P(x, y)
   x1 , 0 and 0, y y1
 m     n  1  m
L
m+n O X
x1 − x1 m+n
x1
M
MP ML m n
PN = LO = x1
=
m
m

Connector 30: For the curve xy = c2, prove the following:


(i) The intercept between the axes on the tangent at any point on the curve is bisected at the point of
contact.
(ii) The tangent at any point makes with the coordinates axes, a triangle of constant area.

c2
Solution: (i) xy = c2 ⇒ y =
x
dy −c2
=
dx x 2
− c2 − y1
\ Slope of the tangent at (x1, y1) = =
x12 x1
− y1
The equation of the tangent at (x1, y1) on the curve is y - y1 = (x − x1 )
x1

or yx1 + y1x = 2x1 y1 = 2c2


 2c2   2c2 
The points of intersection of the tangent with the axes are  ,0  and  0,
 y1   x1 
 2c2 2c2 
The midpoint of this intercept is at  , or at (x1, y1).
 2y 1 2x1 
(ii) Area of the triangle formed by the portion of the tangent between the axes and the coordinate axes
equals
1  2c2   2c2 
= 2c2 = a constant
2  x1   y 1 

Connector 31: Tangents are drawn from the origin to the curve y = sin x. Prove that their points of contact lie on the curve
x2y2 = x2 - y2.
Solution: Let the point of contact be (x1, y1). Then, y1 = sin x1
Slope of the tangent to the curve y = sin x at x = x1 is cos x1. Since the tangent passes through the origin,
its equation y = x cos x1.
We have y1 = x1 cos x1 and y1 = sin x1
2
 y1  2 2 2 2
y1 +  x  = 1 ⇒ x1 − y 1 = x1 y 1
2
1

⇒ (x1, y1) lies on the curve x2 - y2 = x2 y2.


Connector 32: Find the condition for the curves Ax2 + By2 = 1 and A1x2 + B1y2 = 1 to intersect orthogonally.
 Ax   A1 x 
Solution: The slopes of the tangents to the two curves at a point of intersection are  − and  − B y  where,
 By  1
(x, y) is a point of intersection.
Differential Calculus  2.73

 Ax   A x 
Since the curves intersect orthogonally,  −   − 1  = −1
 By   B1 y 
⇒ AA1 x2 + BB1y2 = 0
For a point of intersection (x, y),
Ax2 + By2 - 1 = 0 and
A1 x + B1 y - 1 = 0.
2 2

x2 y2 1
From the above relations we obtain = =
(B1 − B) (A − A1 ) (AB1 − A1B)

Substituting for x2 and y2 in the condition for orthogonality,


AA1 (B1 − B) BB1 (A − A1 )
+ = 0.
(AB1 − A1B) (AB1 − A1B)
On simplification, we finally get
1 1 1 1
− = − as the condition.
A B A1 B1

Connector 33: If p and q are respectively the lengths of the perpendiculars from the origin on the tangent and normal to
the curve x 2 3 + y 2 3 = a 2 3 at a point, then show that
4p2 + q2 = a2.

Solution: Given the curve x 2 3 + y 2 3 = a 2 3   (1)


13
−y
Differentiating (1), we have y’ = 13
x
13
− y1
\ Equation of the tangent to (1) at (x1, y1) is y - y1 = 13 ( x − x1 )
x 1
⇒ xy =0   (2)
13 13 13 13
1 + yx 1 −x 1 y1 − x y
1 1

Perpendicular distance (p) of origin from (2) is


13 13
x1 y 1 + x1 y 1
p = = (ax1 y 1 )
13
[using (1)]   (3)
23 23
x1 + y1
Equation of the normal at (x1, y1) is
13
x1
y - y1 = 13 ( x − x1 )
y1

⇒ xx1 − yy 1 − x1 + y 1 = 0   (4)
13 13 43 43

Perpendicular distance (q) of origin from (4) is
43 43
x1 − y1
q=
23 23
= x1( 23
− y1
23
)a 13
[using (1)]   (5)
x1 + y1
Consider 4p2 + q2 - a2

( )
2
= 4(ax1 y 1 )2 3 + a 2 3 x12 3 − y 12 3 − a 2 [using (3) and (5)]

= 0. [using (1)]
OR
The parametric representation of a point on the curve is x = a cos3q,
y = a sin3q, 0 ≤ q < 2p
2.74  Differential Calculus

dy
Slope of the tangent at any point ‘q’ is = dq = − sin q
dx cos q
dq
Equations of the tangent and normal at q on the curve are given by
sin q
y - a sin3q = -
cos q
(x − a cos3 q)
and
cos q
y - a sin3q =
sin q
(x − a cos3 q)
i.e., x sinq + y cosq = a cosq sinq
and x cosq - y sinq = a cos2q
We have p2 = a2sin2q cos22q and q2 = a2cos22q
⇒ 4p2 + q2 = a2(sin22q + cos22q) = a2
Connector 34: If a tangent to the curve y2 = 4a(x + a) meets a tangent to the curve
y2 = 4a’ (x + a’) at a point on the line x + a + a’ = 0 then prove that these tangents intersect at a right angle.
Solution:
Let P (x1, y1) be a point on y2 = 4a(x + a)   (1)
2a
Slope of the tangent at P to (1) is m1 =
y1
Similarly, slope of the tangent at Q(x2, y2) to y2 = 4a’(x + a’)   (2)
2a '
is m2 =
y2
These two tangents intersect at right angles if m1m2 = -1
(i.e.,) if y1y2 = - 4aa’   (3)
Equation of the tangent to (1) at P is
y - y1 = m1 (x - x1)   (4)
Equation of the tangent to (2) at Q is
y - y2 = m2 (x - x2)   (5)
(4) and (5) meet at a point on x + a + a’ = 0
⇒ y1 + m1 (-a - a’ - x1) = y2 + m2 (-a - a’ - x2)
y1 − y 2 2aa '(y 2 − y 1 )
⇒ = ⇒ y1y2 = -4aa’
2 y1y 2
⇒ tangents intersect at right angles.
2at 2 2at 3 1
Connector 35: Obtain the equations of the tangent and normal to the curve x = 2
,y = at t = .
1+ t 1 + t2 2
2 2
dx (1 + t )4at − (2at )2t 4at
Solution: = =
dt (1 + t 2 )2 (1 + t 2 )2
2 2 3
dy (1 + t )(6at ) − (2at )2t 6at 2 + 2at 4
= =
dt (1 + t 2 )2 (1 + t 2 )2
 dy 
dy  dt  6at 2 + 2at 4 3t + t 3
= = =
dx  dx  4at 2
 dt 
Differential Calculus  2.75

3 1
1 dy = 2 + 8 = 13
At t = , dx
2 2 16
1
2a ×
4 = 2a × 4 = 2a
x coordinate of the point = 1 4 5 5
1+
4
1
2a ×
8 = 2a × 4 = a
y coordinate of the point = 1 8 5 5
1+
4
2a a
The point is  , 
 5 5
1
Equation of the tangent at the point t = is
2
a 13  2a 
y − = x −  ⇒ 65 x - 80 y = 10 a
5 16  5
⇒ 13x - 16 y = 2a.
1
Equation of the normal at the point t = is
2
a 16  2a  5y − a −16(5x − 2a)
y - = − x −  ⇒ =
5 13  5 5 13 × 5
⇒ 16x + 13y - 9a = 0.
3 3
x 2
y 2
Connector 36: It is given that x cos a + y sin a = p touches the curve   +  = 1. Show that a 3 cos 3 a +
a b
b3 sin3a = p3.
4 4
Solution: Any point on the curve can be represented by x = a cos 3 q , y = bsin 3 q where, q is a parameter.
(0 ≤ q ≤ 2p)
Slope of the tangent at any point “q’’ is
1
b × 4 sin 3 q cos q
2
3 b cos 3 q
= 1
= − 2
−a × 4 cos 3 q sin q a sin 3 q
3
Equation of the tangent at ‘q’ is
2
b cos 3 q
(x − a cos q)
4 4
y - b sin 3 q = − 2
3

a sin 3
q
2 2
cos 3 q sin 3 q
⇒ x +y =1   (1)
a b
This is identical with the line x cos a + y sin a = p   (2)
2 2
cos q 3
sin q 3
1
⇒ = =
a cos a b sin a p
2 a cos a 2 b sin a
cos 3 q = , sin 3 q =
p p
Cubing the relations above and then, adding,
a 3 cos3 a b3 sin3 a
1 = 3
+ ⇒ a3 cos3 a + b3 sin3 a = p3.
p p3
2.76  Differential Calculus

Connector 37: If loge4 = 1.3868, find an approximate value of loge4.01 using differentials.
Solution: Let y = logex
For a small change dx in x an approximation to the change in y is given by
1 0.01
dy =   dx = = 0.0025
x 4
Therefore, loge4.01 is approximately equal to
1.3868 + 0.0025 = 1.3893
Connector 38: Use differentials to estimate the amount of paint needed to apply a coat of paint 0.05 cm thick to a hemi-
spherical dome with diameter 50 metres.
Solution: If S denotes the surface area of the hemisphere of radius r metres, S = 2pr2
Taking differentials,
dS = 4pr dr
0.05
Given r = 50, dr= = 0.0005
100
dS = 4p × 50 × 0.0005 = (200 × 0.0005)p = (0.1)p
\ 0.1 p cubic metres of paint is required.
Connector 39: Car A is travelling west at 50 km/h and car B is travelling north at 60km/h. Both are headed for the inter-
section of the two roads. At what rate are the cars approaching each other when car A is 0.3 km and car B
is 0.4 km from the intersection?
Solution: Let C be the intersection of the two roads. At a given time t, let x be the distance from car A to C, let y be
the distance from car B to C, and let z be the distance between the cars, where x, y and z are measured in
kilometers.
dx dy C x A
We are given that = −50 and = −60 (since x and y are decreasing with
dt dt
dx dy y z
time, both and are negative).
dt dt
From DABC, z2 = x2 + y2 B
Differentiating each side with respect to t,
dz dx dy dz 1  dx dy 
2z = 2x + 2y ⇒ = x +y 
dt dt dt dt z  dt dt 
When x = 0.3 and y = 0.4, z = 0.5 (using z2 = x2 + y2)
dz 1
⇒ = [0.3(-50) + 0.4(-60)] = -78
dt 0.5
⇒ The cars are approaching each other at a rate of 78 km/h.
Connector 40: A man walks along a straight path at a speed of 4 units/sec. A search light is located on the ground 20 units
from the path and is kept focused on the man. At what rate is the search light rotating when the man is
15 units from the point on the path closest to the search light?
Solution: Let x be the distance from the point on the path closest to the search light, to the man. Let q be the angle
between the beam of the search light and the perpendicular to the path.
= 4 and we want dq when x = 15.
dx
We are given that
dt dt
x
From the figure, = tan q ⇒ x = 20 tan q
20
Differential Calculus  2.77

Differentiating both sides with respect to t, Man


dx dq O x M
= 20sec2 q
dt dt 90°
dq 1 dx 1 2 dx 20
= (cos2 q) = cos q (since = 4)
dt 20 dt 5 dt
θ
4
When x = 15, the length of the beam = SM = 25 and so, cos q = . S Search light
5
2
dq 1  4  16
⇒ =   =
dt 55 125
16
The search light is rotating at a rate of radians per second.
125
Connector 41: Find the intervals of monotonicity of the function f(x) = x2 e-x
Solution: f ’(x) = -x2 e-x + 2x e-x = e-x (2x - x2)
Note that e-x > 0 for all x and (2x - x2) is positive when x lies between 0 and 2 and (2x - x2) is negative
when x lies beyond 0 and 2.
Therefore, f(x) is monotonic increasing in (0, 2) and monotonic decreasing in (-∞, 0) and (2, ∞)
Connector 42: Verify Rolle’s Theorem for the following functions:
(i) f(x) = (x - a)m (x - b)n, m, n being positive integers and x ∈ [a, b]
sin x
(ii) f(x) = , x ∈ [0, p]
ex
Solution: (i) f(x) is continuous in [a, b] and differentiable in (a, b)
Also f(a) = 0 = f(b)
f ’ (x) = (x - a)m n(x - b)n - 1 + (x - b)n m (x - a)m - 1
= (x - a)m - 1 (x - b)n - 1 [n (x - a) + m(x - b)]
f ’(x) = 0 for x = a, b and n (x - a) + m(x - b) = 0
mb + na
⇒ x(m + n) = (mb + na) ⇒ x = ∈ (a, b)
m+n
Verified.
(ii) f(x) is continuous in [0, p] and differentiable in (0, p)
f(0) = 0 = f(p)
e x cos x − (sin x)e x
f ’(x) = = 0,
(e x )2
Then, ex(cos x - sin x) = 0
Since ex ≠ 0, cos x = sin x
p
⇒ x = ∈ (0, p)
4
Verified.
1
Connector 43: Find c in the Mean Value Theorem where, f(x) = x3 - 3x2 + 2x and a = 0, b = .
2
Solution: Since f(x) is a polynomial, it is continuous and differentiable everywhere. Applying mean value theorem,
 1
there exists a point c ∈  0,  such that f ’ (c) =
f(b) − f(a)
 2 (b − a)
2.78  Differential Calculus

  1 3 1
2

   − 3   + 1 − 0
  2   2   3
3c2 - 6c + 2 =  =
1 4
 2 

12c2 - 24c + 8 = 3 ⇒ 12c2 - 24c + 5 = 0


6 ± 21
giving c =
6

6 + 21  1
Note that c = does not belong to  0, 2  .
6

6 − 21  1
However, c = which belongs to  0,  .
6  2

p
Connector 44: Use mean value theorem to show that tan -1 x + cot -1 x = .
2
Solution: Let f(x) = tan-1 x + cot-1 x
1 1
f ‘(x) = − =0
1 + x2 1 + x2

We note that f ’(x) = 0 for all x


⇒ f(x) is a constant for all x.
p p p
f(1) = tan-1 1 + cot-1 1 = + =
4 4 2

p p
or f(x) = or tan-1 x + cot-1 x =
2 2
1 
Connector 45: Show that the equation 4x3 + x2 - 2x - 1 = 0 has only one real root in  , 1 .
3 
Solution: Let f(x) = 4x + x - 2x - 1
3 2

⇒ f ’ (x) = 12x2 + 2x - 2 = 2(2x + 1) (3x - 1)


From the sign scheme of f ’(x) shown below we observe that f(x) is monotonic increasing in
 1 1   1 1
 −∞, −  ∪  , ∞  and monotonic decreasing in  − 2 , 3  +ve +ve
 2 3  − ve
1 1

 1 1 2 3
Also, f  −  < 0 and f   < 0
 2 3
⇒ The curve as represented in the adjoining figure cuts
the x-axis only once and now f(1) = +ve. y = f(x)
0
1
f   and f(1) are of opposite signs −1 1
3
2
3
1
⇒ curve cuts the x-axis between and 1
3
1 
⇒ f(x) =0 has exactly one root in  ,1
3 
Differential Calculus  2.79

(
Connector 46: Show that 1 + x log x + x 2 + 1 ≥ 1 + x 2 for all x ≥ 0.)
Solution: Consider the function

2
( 2
f (x) = 1 + x log x + x + 1 − 1 + x  )  (1)

( )
1  x  x
f ’(x) = x × × 1 +  − + log x + x2 + 1
( x + x +1 2
) x +1
2
1 + x2

= 2
x +1
x

1+ x
x
2
( )
+ log x + x 2 + 1 = log x + ( x2 + 1 )
1
f ”(x) = 2
x +1
We observe that f ’(0) = 0 and f ”(x) > 0 for all x.
⇒ f ’(x) ≥ 0 for x ≥ 0 ⇒ f(x) is increasing for x ≥ 0
f(0) = 1 - 1 = 0
Since f(0) = 0, f(x) ≥ 0 for all x ≥ 0 which establishes the inequality.
Connector 47: Find the points on the curve y2 - x2 = 4 that are closest to the point (2, 0).
Solution: Let (x, y) be the point on the curve y2 - x2 =4 that is closest to the point (2, 0).
Then, D2 = [distance between (x, y) and (2, 0)]2
= (x - 2)2 + y2 = (x - 2)2 + 4 + x2 (since y2 - x2 = 4)
= 2x2 - 4x + 8
If D is minimum, D2 also minimum
d(D2 ) d 2 (D2 )
= 4x - 4 ⇒ =4>0
dx dx 2
d(D2 )
= 0 ⇒ x = 1.
dx
d 2 (D2 )
Since > 0, x = 1 corresponds to the minimum of D.
dx 2
When x = 1, y2 = 5, y = ± 5.
The two points closest to (2, 0) are 1, 5 and 1, − 5 . ( ) ( )
Connector 48: Find the coordinates of the point on the straight line y = 3x - 3 which is closest to the parabola
y = x2 + 7x + 2.
Solution: Let P be the point on the line closest to the parabola and let the shortest distance line through P meet the
parabola in Q. Then, PQ is the shortest distance between the line and the parabola. PQ must be perpen-
dicular to the given line and must be normal to the parabola at Q.
Let the equation of the line PQ be
−1
y= x + k. (since PQ is perpendicular to the given line)
3
Let the coordinates of Q be (x1, y1)
 dy 
Slope of the tangent to the parabola at x = x1 is given by   = 2x1 + 7.
dx x = x 1
2.80  Differential Calculus

−1
⇒ Slope of the normal at x = x1 is
(2x1 + 7)
−1 −1
= ⇒ x1 = −2
(2x1 + 7) 3

⇒ y1 = x12 + 7x1 + 2 = 4 - 14 + 2 = -8.


⇒ Q is (-2, -8)
−1
⇒ Equation of PQ is y + 8 = (x + 2)
3
⇒ 3y + 24 = -x -2 ⇒ 3y + x + 26 = 0.
Solutionving the two equations 3y + x + 26 = 0 and 3x - y - 3 = 0,
 −17 −81 
we get P as  ,
 10 10 
OR
Let Q be (x1, y1)
Distance from Q to the line 3x - y - 3 = 0 is
3x1 − y 1 − 3 3x1 − x12 − 7x1 − 2 − 3 x12 + 4x1 + 5 (x1 + 2)2 + 1
= = =
10 10 10 10

Distance is minimum when x1 = -2.


Further proceed as above.
Connector 49: Find the maximum value of xy when a2x4 + b2y4 = c6, x > 0, y > 0.
Solution: When xy is a maximum, x4y4 is maximum.
We shall find the maximum value of x4y4.
 c6 − a 2 x 4  1 6 4 2 8
Let u = x4y4 = x4  b2  = b2 (c x − a x )

du 1
= 2 (4c6x3 - 8a2x7)
dx b

d2 u 1 x2
2
= 2 (12c6 x 2 − 56a 2 x 6 ) = 2 (12c6 − 56a 2 x 4 )
dx b b
du c6
= 0 ⇒ x 4 = 2 (since x ≠ 0)
dx 2a
1
d 2 u  c6  2
 1  6 2  c 
6

For this value of x , 4
=  2  2  12c − 56a  2   < 0
dx 2  2a  b   2a  

c6
\ x 4 = corresponds to maximum.
2a 2

c6 c6
We have b2y4 = c6 - a2x4 = ⇒ y4 = 2
2 2b
The maximum value of x4y4 is
c3 c3
⇒ Maximum value of xy is 1
=
(4a 2 b2 ) 4 2ab
Differential Calculus  2.81

OR
We know that if x1 + x2 +…..+xn = a constant where xi > 0, I = 1, 2, 3, …..n. then the product x1x2,…..xn is
maximum when x1 = x2 = ……..xn.
In our problem, a2x4 + b2y4 = c6 and a2x4 and b2y4 are both positive.
Therefore, their product is (a2x4) (b2y4) is maximum
c6 4 c6 4 c6
when a2x4 = b2y4 = or when x = 2 , y = 2
2 2a 2b
c12
Maximum value of x4y4 is .
4a 2 b2
Connector 50: Towns A and B are situated on the same side of a straight road at distances a and b respectively, from the
road. Perpendiculars drawn from A and B meet the road at the points C and D respectively. The distance
between C and D is c. A hospital is to be built at a point P on the road between C and D such that distance
APB is minimum. Find the position of P.
Solution: Let P be at a distance x from C
Distance APB = AP + PB
⇒ D = a 2 + x 2 + b2 + (c − x)2
dD 1 1 A
= × 2x + × 2(c − x) × (−1)
dx 2
2 a +x 2
2 b + (c − x)2
2
B
a
x (c − x) b
= −
2 2 2 2
a +x b + (c − x)
C P D
dD x c−x
For D to be maximum or minimum =0
dx
x (c − x)
⇒ =
2 2
a +x b + (c − x)2
2

x2[b2 + (c - x)2] = (c - x)2 [a2 + x2]


x2[b2 + c2 + x2 - 2cx] = (a2 + x2)[c2 + x2 - 2cx]
x2(b2 + c2) + x4 - 2cx3 = a2c2 + (a2 + c2)x2 - 2cxa2 - 2cx3 + x4
x2(b2 - a2) + 2ca2x - a2c2 = 0
b2x2 - [a2x2 - 2ca2x + a2c2] = 0
b2x2 - a2{x2 - 2cx + c2} = 0
b2x2 - a2 (c - x)2 = 0
ac ac
[bx + a(c - x)] [bx - a (c - x)] = 0 ⇒ x = or x =
(a − b) (a + b)
ac ac − bc
When x = ,y = c−x = c− =
(a − b) a − b (a − b)
ac
If x > 0, y < 0 or if x < 0, y > 0 which means that x = is not acceptable.
(a − b)
ac bc
\ x = ,y = c−x =
(a + b) a+b
2.82  Differential Calculus

(a + b)  1 1 
3
d2 D a2 b2
= + =  + 
dx 2 (a + b) + c2  a b 
3 3 2

(a 2
+ x2

)
 b2 + (c − x )2  2
2


at the above point > 0
This value of x corresponds to D minimum

2 a 2 c2 b2 c 2 a 2 (a + b)2 + a 2 c2 b2 (a + b)2 + b2 c2
D= a + 2
+ b2 + = +
(a + b) (a + b)2 (a + b) (a + b)

a b
= (a + b)2 + c2 + (a + b)2 + c2
(a + b) (a + b)

= (a + b)2 + c2   (1)
Since D is defined in 0 ≤ x ≤ c, we have to see whether the values of D at the two ends corresponding to
ac
x = 0 and x = c are less than that obtained for x = .
(a + b)

When x = 0, AP = a, PB = b2 + c 2

Distance APB = D = a + b2 + c 2
Now,

(a + )
2
b2 + c 2 = a 2 + b2 + c2 + 2a b2 + c2

( )
2
> a2 + b2 + c2 + 2ab > (a + b)2 + c2

When x = c, AP = a 2 + c2 and PB = b

D = b + a 2 + c2 > a 2 + b2 + c2 + 2ab
Thus, for x = 0 and x = c, the values of D are greater than that given by (1). Therefore, the point P has to
ac
be at a distance from C and the minimum value of APB is given by (1).
(a + b)

Connector 51: Find all values of the parameter a for which the point of minimum of
x2 + x + 2
f(x) = 1 + a2x - x3 satisfies the inequality < 0.
x 2 + 5x + 6

x2 + x + 2
Solution: Since x2 + x + 2 > 0 for all real x, < 0 when x lies
x 2 + 5x + 6
between -3 and -2   (1)
f(x) = 1 + a2x - x3
f ’(x) = a2 - 3x2, f ”(x) = -6x.
|a|
f ’(x) = 0 ⇒ x = ±
3

−|a|
For f(x) to be minimum, f ”(x) > 0 i.e., when x =
3
|a|
Now, − must lie between -3 and -2.
3
Differential Calculus  2.83

|a|
If a > 0, it is clear that if a lies between 2 3 and 3 3 , − will lie between -3 and -2. If a < 0 a must lie
3
between −3 3 and −2 3 .

(
Hence a∈ −3 3 , − 2 3 ∪ 2 3 , 3 3 ) ( )
Connector 52: The parabola y = x2 + ax + b cuts the straight line y = 2x - 3 at a point with abscissa 1. For what values of
a and b is the distance between the vertex of the parabola and the point the least? Find that distance.
Solution: When x = 1, y = -1 (since the point lies on the line y = 2x - 3)
Since this point whose coordinates are (1,-1) has to lie on the parabola,
-1 = 1 + a + b
⇒ a + b = -2   (1)
The equation of the parabola may be written as
2
 a a2
y - b =  x +  −
 2 4 (3, 3 )
2
  a a2  a2 
⇒  x +  = y + − b = y −  b −  (0, 1) (2, 0)
 2 4  4
(1, −1)
 a a2 
Vertex of the parabola is at  − , b −
 2 4 

If D denotes the distance between the vertex of the parabola and the point,
2 2 2
 a  a2  a2  a2 
D2 = 1 +  +  −1 − b +  = 1 + + a + 1 + b − 
 2  4 4  4

2
a2  a2 
= 1+ + a + 1 − 2 − a − 
4  4

a 4 a 3 7a 2
= + + + 3a + 2, on simplification.
16 2 4

d (D )
2
4a 3 3a 2 14a
= + + +3   (1)
da 16 2 4


( ) = 12a
d 2 D2 2

+
6a 14
+   (2)
2
da 16 2 4
dD2
= 0 ⇒ 4a3 + 24a2 + 56a + 48 = 0 ⇒ a3 + 6a2 + 14a + 12 = 0
da
a = - 2 is a root.
On factoring, the other values of a satisfy the equation a2 + 4a + 6 = 0, the roots of which are complex.

Therefore, a = -2 is the only critical point and for this value of a,


( ) > 0.
d 2 D2
da 2
Or, a = -2 corresponds to D minimum. Since a + b = - 2, when a = -2, b = 0.
The values of a and b are thus -2 and 0 respectively.
Minimum D2 = 0 (ie) minimum distance = 0
2.84  Differential Calculus

c
Connector 53: The intensity of illumination at a point x units of length from a light of candle power c is . The two lights
x2
whose candle powers are k and 8 k are 60 units of length apart. Find the point between them at which the
intensity of illumination is a minimum. (Assume the intensity at any point is the sum of the intensities due
to the two sources.)
Solution: If I denotes the intensity of illumination due to the two sources,
k 8k
I= +
x 2 (60 − x)2

dI −2k 16k
= 3 +
dx x (60 − x)3

d2 I +6k 48k
= 4 +
dx 2 x (60 − x)4

dI d2 I
dx = 0 ⇒ x = 20 and when x = 20, 2 > 0.
dx

Therefore, the intensity of the illumination is minimum at the point 20 units of length from the light source
with candle power k.
Connector 54: Find the area of the largest rectangle that can be inscribed in a semicircle of radius r.
Solution: Let us take the semicircle to be the upper half of the circle x2 + y2 = r2 with centre at the origin. The rectangle
which is inscribed is shown in the figure.
Let (x, y) be the vertex that lies in the first quadrant.
Then the rectangle has side of length 2x and y,
so its area is A = 2xy.
To eliminate y we use the fact that (x, y) lie on the circle x2 + y2 = r2 and (x, y)
2x
so y = r 2 − x 2 y
x
−r O r
Thus, A = 2x r − x 2 2

The domain of this function is 0 ≤ x ≤ r.


Its derivative is
dA 2x 2 2(r 2 − 2x 2 )
= 2 r2 − x2 − =
dx 2
r −x 2
r2 − x2

dA r
= 0 ⇒ 2x 2 = r 2 or x =
dx 2
This value of x gives a maximum value of A sin i.e., A(0) = 0 and
A(r) = 0. Therefore, the area of the largest inscribed rectangle is r r sin θ
θ
r   r  2 r2
A  = 2 r − = r2 O
2  2  2

OR
A is expressed in terms of q. X = rcosq; y = r sin q
A = (2r cos q) sin q = r 2 sin 2q and the maximum value of sin 2q = 1 and therefore, the maxi-
mum value of A is r 2 . Note that we did not use calculus at all for solving this problem in this
method.
Differential Calculus  2.85

Connector 55: f(x) is a continuous function for all real x. The value of the function at the mid-point of any interval
(a, b) is the average of the values of the function at the end points. If f ’(0) exists and = -3 and f(0) = 5,
find f(4).

Solution: f
a + b  f(a) + f(b)
 2  = 2
 a  f(a) + f (0) f (a ) + 5 a 
Taking b = 0, f   = = ⇒ f(a) = 2 f   − 5   (1)
2 2 2 2
f (2x ) + f (2h )
f (x + h) − f (x ) lim − f(x)
f ’(x) = lim =
h→0 2
h→0 h h
2f(x) − 5 1
+ 2f(h) − 5 − f(x)
= lim 2 2 using (1)
h→0 h
f (h ) − 5 f(h) − f(0)
= lim = lim = f ’(0) = -3 (given)
h→0 h h→0 h
\ f ’(x) = -3x + c
But f(0) = 5 ⇒ c = 5
\ f(x) = -3x + 5
\ f(4) = -12 + 5 = -7
x−5
Connector 56: (a) Find the derivative of inverse of the function f(x) = at x = 5.
x+3
(b) Given that f(0) = 1, f(1) = 0
f ’(0) = 2 and f ’(1) = 1. Find g’(1), where g(x) is the inverse of f(x).
Solution: (a) Consider g(x) is the inverse of f(x).
\ g (f ( x )) = x
Differentiate with respect to x, we get
\ g ' (f ( x )).f ' ( x ) = 1
1
(
i.e., g ' f ( x ) = ) f ' (x )

\ g’ (x0) = 1 , where x0 = f(x) ⇒ x = f −1 ( x 0 )


(
f ' f (x 0 ) −1
)
x−5
Now f(x) =
x+3
1
\ g '(5) =
( (5))
f' f −1

x−5
Let x = f--1(5) ⇒ 5= f(x) =
x+3
\ 5x + 15 = x - 5 ⇒ x = –5

\ g’ (5) = 1
f '(−5)
2.86  Differential Calculus

x−5 8 8
f(x) = x + 3 = 1 − x + 3 , f ’(x) =
( x + 3)
2

1 1
\ g’(5) = =
f '(−5) 2
1
(b) g’(x0) =
( (x ))
f' f −1
0

\ g’(1) = 1
(
f ' f −1 (1) )
Let a = f-1 (1) ⇒ 1= f(a), given that f(0) = 1 ⇒ a = 0
1 1
\ g’(1) = =
f '(0) 2
Differential Calculus  2.87

topic grip

Subjective Questions
1. Evaluate the following limits (without applying L’ Hospitals Rule):
3− x − 3+ x
(i) lim
x →0 x
 x 5 − 3125 
(ii) lim 
x →5  x − 5 

1+ x −1
(iii) lim
x →0 x
sin5x
(iv) lim
x →0 tan7x

1 − cos7x
(v) lim
x →0 1 − cos9x
x tan x
(vi) lim
x →0 1 − cos x
 tan7x − 3x 
(vii) lim  
x → 0  7x − sin 2 x 

 sin2x + sin6x 
(viii) lim  
x → 0  sin5x − sin3x 

 tan −1 2x 
(ix) lim 
x → 0  sin3x  
1 1 1 1
(x) lim + 2 + 3 + ... + n 
n  3 3 3 3 
sin(x − 1)
(xi) lim
x →1 x2 + x − 2

(xii) lim 2x
x →∞
( x2 + 1 − x )
x − sin x
(xiii) lim
x →∞ x + cos2 x
sin x − x
(xiv) lim
x →0 x3

2− 2+x
(xv) lim
x→2 2 3 − ( 4 − x ) 1 3 
1

 
2.88  Differential Calculus

2. Check the continuity of the following functions:


 sin2 ax
 , when x ≠ 0
(i)  f(x) =  x 2 at x = 0
 a 2 when x = 0

2 − x, x<2
(ii)  f(x) =  at x = 2
2+x x≥2

 5x − 4 0 < x ≤ 1
(iii)  f(x) =  2 at x = 1
4x − 3x 1 < x < 2
 3  e1/ x − e −1/ x  x≠0
x  ,
(iv)  f (x) =   e1/ x + e −1/ x at x = 0
0 , x=0

 cos 3x − 1 x≠0
 ,
(v)  If f (x) =  5x 2 + 1 − 1 , find the value of l so that f(x) is continuous at x = 0.
λ , x = 0

3. Find the derivatives of the following functions:
sec x − 1
(i) y =
sec x + 1
1
(ii) y =
log (cos x )
(iii) y = log (log(log(x5)))
(iv) y = sin (tan–1x)
 1 
(v) y = sec–1  2
 2x − 1 

 1+ x + 1− x 
(vi) y = sin–1  
 2 

 a cos x − bsin x 
(vii) y = tan–1 
 bcos x + a sin x 

x 2  1 − x 2 + x2 
(viii) y = tan −1 2
+ log  
(1 − x )  1 + x 2 + x2 
 a + a2 − x2  2 2
(ix) y = a log   − a − x where, a is a constant
 x 
x
(x) y = (1 + log x)x

−1
 1 + x2 − 1 
(xi) y = tan  
 x 
 1 + x2 − 1 − x2 
(xii) y = tan −1  
 1 + x2 + 1 − x2 
Differential Calculus  2.89

−1  2x  −1  3x − x 
3
−1
(xiii) y = tan x + tan   + tan  1 − 3x 2 
 1 − x2 

−1  3sin x − 3cos x 
(xiv) y = tan 
 3sin x + 3cos x 
 2x + 3 
(xv) f(x) = sin(log x) and y = f 
 3 − 2x 
4. Find the derivatives of the following functions:
(i)  y = x
(x )x

x ....... ∞
(ii)   y = x x

(iii)  y = tan x + tan x + tan x + ....


2
dy y log x
(iv)  If xy = ex - y, show that =
dx x2
dy sin (a + y )
2

(v)  If x sin(a + y) = sin y, show that =


dx sina
5. Evaluate the following limits:
px
tan
 x 2a
(i) lim  2 − 
x →a  a
1 1 
(ii) lim − x
x →0  x 
e −1 

( )
1/x
(iii) lim e3x + 2x
x →0

1/x 2
 tan x 
(iv) lim 
x →0  x 
1
x 4 sin + 2x 2
(v) lim x
x →−∞ 3+ | x |3
 x +1 p 
(vi) lim x  tan −1 −
x →∞  x + 2 4 
2x
x −2
(vii) lim  
x→∞  x + 3 

 p  x 
1

(viii) lim  tan  + x 


x→ 0 
  4  
1
 sin x  (x − a )
(ix) lim 
x → a sina 
 
2− cos x − 1
(x) lim
x→ p  p
2
x x − 
 2
2.90  Differential Calculus

2 2 d2 y k
6. (i)  If x 1 − y + y 1 − x = k , show that 2
=− .
dx (1 − x 2 )3/2
d2 y
(ii)  If x = a(t – sin t), y = a(1 – cos t), find .
dx 2
d2 y dy
(iii)  If y = sin (m sin-1 x), prove that (1 − x 2 ) − x + m2 y = 0.
dx 2 dx
n
y x d2 y dy
(iv)  If cos −1   = log   , show that x 2 2 + x + n2 y = 0 .
b n dx dx
d2 x
2
d y dy 2
(v)  If y = f(x) and f is inversible, establish the result =− .
dx 2  dx 
3

 dy 

d2 y
(vi)  Use the result in (v) above to obtain for the functions given below:
dx 2
(a)  y = sin x -1

(b)  sin y = x sin(a + y)


n
7. Evaluate ∑r x
r =1
r −1
using Calculus.

8. Prove the inequalities


x2
(i)  x - log(1 + x) > ,x>0
2(1 + x)2
x2 x2 x 4
(ii)  1 - < cos x < 1 − + , for 0 < x < p
2 2 24 2
9. For the curve x = a cos3 q, y = a sin3 q, the tangent at ‘q’ meets the coordinate axes in M and N. Find MN. Also find
(i)  the length of tangent
(ii)  the length of normal
(iii)  the length of sub tangent and
(iv)  the length of sub normal at the point “q”.
10. Let A(p2, -p), B(q2, q), C(r2, -r) be the vertices of a triangle ABC. A parallelogram AFDE is drawn with D, E, F on
the line segments BC, CA, AB respectively. Using calculus, show that the maximum area of such a parallelogram is
1
(p + q)(q + r)(p − r).
4
1 dn 2
11. Pn(x) represents the nth degree polynomial in x defined as Pn (x) = (x − 1)n . Find P0(x), P1(x), P2(x) and
2n n! d x n

P3(x) and verify that (1 - x2) Pn’’(x) - 2x P’n(x) + n(n + 1) Pn(x) = 0 where n = 0, 1, 2, 3.

a cot −1 (x − 3) , 0≤x<3

bx , x =3

12. Find values of a, b, c so that f(x) =  −1  1  3 < x < 4 is continuous in the interval [0, 6].
c tan  x − 3  ,
 −1  
cos (4 − x) + aπ , 4 ≤ x ≤ 5
Differential Calculus  2.91

13. Let y = f(x) be a real valued function having first order derivatives and satisfying the relation f(x + 2) + f(x + 6) =
f (x + 4).
(i) Show that f(x) is periodic with period 12.
dy dy
(ii) Evaluate − .
dx x =13
dx x = 25

14. (i)  Show that the tangent to the curve x5 - x3 + 2x + y - 8 = 0 at (0, 8) meets the curve again at two points.
(ii) Show that the tangents at these two points are parallel.
15. The function f(x) is such that 2f(xy) = (f(x))y + (f(y))x for all real x, y and f(1) = e
(i) Determine the function f(x).
f(x) + f(− x)
(ii) Find the maximum value of the function g(x) =
2

Straight Objective Type Questions


Directions: This section contains multiple choice questions. Each question has 4 choices (a), (b), (c) and (d), out of
which ONLY ONE is correct.

3x + | x |
16. lim =
x →∞ 7x − 5 | x |

3 3 3
(a) (b) (c) (d) 2
2 7 5

xsin x
17. The function f(x) = is
(x 2
+2 )
(a) continuous for all x (b) discontinuous for all x
(c) constant function (d) discontinuous only at x = ±2

 x +1  x −1 dy
18. If y = sec −1   + sin-1  x + 1  , x > 1, then is
 x −1 dx
(a) 0 (b) 1 (c) –1 (d) 2
19. The function f(x) = xex (1 – x)
is strictly increasing for all x belonging to
1   1
(a)  , 1 (b)  −1, − 
2   2
 1  1 
(c)  −∞, −  ∪ (1, ∞) (d)  − , 1
 2  2 

20. The maximum and minimum values of f(x) = 3 sin2 x + 4 cos2 x is


(a) {-4, -3} (b) {7, 3} (c) {4, -3} (d) {4, 3}

 tan3q − sin3q 
21. lim   is
q→ 0  q3
3 1 27 −1
(a) (b) (c) (d)
2 2 2 2
2.92  Differential Calculus

1 + cos x − 3 − cos x
22. If f(x) = is continuous everywhere, then f(0) equals
x2
2 − 2
(a) (b) 1 (c) (d) 2 2
4 4
dy
23. If (tan x ) = (tan y ) , then
y x
=
dx
log tan y log (tan y ) − 1
(a) (b)
log tan x 2log tan x
2y log tan y − 2 y cosec 2 x
(c) log (tan y ) − (d)
sin2x log ( tan x ) - 2 x cosec 2 y

x3
24. The tangent at the point (5, 5) on the curve y 2 = meets the curve again at the point Q. The coordinates
10 − x
of Q are
 −1   1
(a) 1,  (b) 1,  (c) (2, -2) (d) (2, -1)
 3   3

25. The point on the curve y = x2 – 2x + 3, which is closest to the straight line y = 2x – 2 is
(a) (3, 6) (b) (2, 3) (c) (-2, 11) (d) (1, 2)

e x − e − x + 2sin x + x 3 − 4x
26. lim is
x →0 10x 3
1 1 1 1
(a) (b) (c) (d)
5 20 10 15
4
1x x x
x
   + 16 + 2 
4
27. If f ( x ) =   , then lim f (x) is
 3  x 0
 
 
(a) 2 (b) 8 (c) 4 (d) 16

dy
28. If y = cos x cos 2x cos 3x, then is
dx
(a) y (tan x + tan 3x) (b) –y (tan x + 2 tan 2x + 3 tan 3x)
(c) y (tan x + 2 tan 2x + 3 tan 3x) (d) 2y (sin x + sin 2x + sin 3x)
29. AB is a diameter of a circle of radius r, C is any point on the circumference of the circle. Then,
(a) the area of the triangle ABC is maximum when it is isosceles
(b) the area of the triangle ABC is minimum when it is isosceles
(c) the perimeter of the triangle ABC is minimum when it is isosceles
(d) None of the above.
 1 + x2   1 + x2 
30. For the curve defined as y = cosec −1   + sec −1  the set of points at which the tangent is parallel to the
x-axis is  2x   1 − x 2 

(a) f (b) R (c) N (d) Z


Differential Calculus  2.93

Assertion–Reason Type Questions


Directions: Each question contains Statement-1 and Statement-2 and has the following choices (a), (b), (c) and (d), out
of which ONLY ONE is correct.
(a) Statement-1 is True, Statement-2 is True; Statement-2 is a correct explanation for Statement-1
(b) Statement-1 is True, Statement-2 is True; Statement-2 is NOT a correct explanation for Statement-1
(c) Statement-1 is True, Statement-2 is False
(d) Statement-1 is False, Statement-2 is True

31. Statement 1
 sin x − e x + 1  sin x ex − 1
lim   exists and lim = 1, lim =1
x →0
 x  x →0 x x →0 x

and
Statement 2
If lim(f(x) − g(x))exists, then, both lim f(x) and limg(x) exist.
x →a x →a x →a

32. Statement 1
f(x) = [x]2 - 5[x] + 3, 1 < x < 4, where [ ] represents the greatest integer function is not continuous at x = 2 and 3 but
is bounded.
and
Statement 2
A continuous function in a closed interval I is bounded.
33. Statement 1
cos x
lim =0
x →∞ x
and
Statement 2
tan x
lim =1
x →0 x
34. Statement 1
Let f(x) = (x + 3) (x - 1) (x - 4) (x - 7)
Roots of f ’(x) = 0 lie in (-3, 1), (1, 4) and (4, 7)
and
Statement 2
Let f(x) be continuous in [a, b] and differentiable in (a, b) and f(a) = f(b). Then, there exists a point c, a < c < b such
that f ’(c) = 0.
35. Statement 1
The line ax + by + c = 0 can be a tangent to the curve y = 2x3 only if a and b are of opposite signs.
and
Statement 2
The function f(x) = 2x3 is a monotonic increasing function for all x.
2.94  Differential Calculus

36. Statement 1
( x − 2)2 , 0≤x<3
Consider f(x) =  then f ’(2) = 0.
 x − 2 , 3≤x≤6

and
Statement 2
f(x) is continuous in [a, b] and differentiable in (a, b) and f(a) = f(b) then, there exists a point c in (a, b) such that
f ’(c) = 0.
37. Statement 1
( (
The function f(x) = cos log e x + x 2 + 1 )) is an even function.
and
Statement 2
1
x+ x2 + 1 = for all x.
2
x +1 − x
38. Statement 1
x = 2 is a repeated root of the equation x3 - 3x2 + 4 = 0.
and
Statement 2
If f(x) is such that f ’(a)= 0 and f ”(a)≠ 0, then x = a is a repeated root of f(x) = 0.
39. Statement 1
The function f(x) defined by
 4 − 5x , 1 ≤ x < 3
2

f(x) =  attains its minimum at the point x = 3.


3x − 50 , 3 ≤ x ≤ 5

and
Statement 2
A continuous function which is not differentiable at a point x0 will have an extremum at that point.
40. Statement 1
e −4 x
Let f(x) = where, [ ] denotes the greatest integer function and let x = x0 where x0 is not an integer. Then,
2 + [x]
4e −4 x0
f ’(x0) = −
2 + [x 0 ]

and
Statement 2
f(x) = [x] is not continuous for any integral value of x.
Differential Calculus  2.95

Linked Comprehension Type Questions


Directions: This section contains 1 paragraph. Based upon the paragraph, 6 multiple choice questions have to be
answered. Each question has 4 choices (a), (b), (c) and (d), out of which ONLY ONE is correct.

y = f(x)

Q
P

A ψ ψ + ∆ψ
O T1 T2
x

Let y = f(x) be the equation of a curve where, f(x) is a function of x which is twice differentiable in an interval I.
Let P, Q be two neighbouring points on the curve with coordinates, (x, y) and (x + Dx, y + Dy) respectively, (x, x + Dx
∈ I). (Refer Figure) Let the tangents to the curve at P and Q make angles y and y + Dy with the x-axis. Let A be a fixed
Dy
point on the curve from which arcs are measured and arc AP = s, arc AQ = s + Ds. Then y is a function of s and is the
Ds
average bending or average curvature of the arc PQ of the curve.
Dy Dy dy dy
lim = lim = is defined as the curvature of the curve at P. represents the instantaneous rate of change
Dx → 0Ds Ds → 0 Ds ds ds
dy
of the inclination y of the curve with respect to the arc length. may be positive or negative or zero.
ds
The numerical value of the reciprocal of the curvature is called the “radius of curvature” of the curve at P and it is de-
noted by r.

1 ds
r= = .
dy dy
3
ds   dy  2  2

1 +   
  dx  
The radius of curvature of the curve y = f(x) at a point P(x1, y1) on it is given by r =
d2 y
dx 2
at (x1 , y1 )

41. The radius of curvature at the point (1, 10) on the curve y = 3x + 7x is
2

3 3
14 2 170 2
(a) (b)
6 6
3
170 2
170
(c) (d)
3 6
42. The radius of curvature at any point ‘t’ on the curve x = 2t, y = t2 - 1 is
3 3
(1 + t 2 ) 2
(1 + t 2 ) 2
(a) (b)
2t 2
3 3
(c) (1 + t 2 ) 2 (d) 2 (1 + t 2 ) 2
2.96  Differential Calculus

p
43. The radius of curvature at the point q = on the curve
2
x = a(q - sin q) y = a(1 - cos q), (a > 0) is
(a) 2 2a 2 ( )
(b) 2 2 a

(c) 4a (d) 2a

a  a 2 (a − x)
44. The radius of curvature at the point  , a  on the curve y 2 = is
2  x
3 3
5 2a 5 2a
(a) (b)
8 2
3
5 2a 5a
(c) (d)
4 4

x y
45. If r is the radius of curvature at any point (x, y) on the curve + = 1, then r is
a b
2 3 1 3
(a) (ax + by) 2 (b) (ax + by) 2
ab ab
1 3 3
(c) (ax + by) 2 (d) ab(ax + by) 2
2ab
46. If r1 and r2 are the radii of curvature at q and q + p on the curve x = a(q - sin q), y = a(1 - cos q), r12 + r22 is
(a) a2 (b) 2a2 (c) 8a2 (d) 16a2

Multiple Correct Objective Type Questions


Directions: Each question in this section has four suggested answers out of which ONE OR MORE answers will be correct.
47. Given f(x2 + 1) = 2x4 - 3x2 + 1. Then the tangent to the curve y = f(x)
(a) at x = 0 passes through (1, -1)
(b) at x = 1 passes through (3, 3)
(c) at x = 2 passes through (0, -2)
(d) at x = 3 passes through (2, 1)
 p2 
48. Given f(x) = tan  − x 2  and A = R - [0, 1]
 16 
(a) Range of f(x) is A (b) Range of f(x) is A’
(c) Maximum value of f(x) = 1 (d) Minimum value of (f(x))-1 = 1

 1 1  1
49. If f  x +  = x2 + 2 = g  x −  , then
 x  x  x
(a) f(1) = 2 = g(1) (b) f ’(1) = 2 = g’(1)
(c) (f o g)’(1) = 4 = (gof ’) (-1) (d) (g o f)’(1) = -4 = (f ’ o g’)(-1)
Differential Calculus  2.97

Matrix-Match Type Question


Directions: Match the elements of Column I to elements of Column II. There can be single or multiple matches.
50.
Column I Column II
(a) Coordinates of point of contact of a vertical tangent to 9x2 + 16y2 - 54x - (p) (2, 3)
128y + 193 = 0 is
(b) The function f(x) = 2x3 - 9x2 - 24x + 30 is decreasing in the open interval (q) (3, 9)
4
(c) The tangent at the point whose eccentric angle is q = cot-1 3 on the ellipse (r) (-1, 4)
4
9x2 + y2 + 18x = 216 and the tangent at the point q = cot −1 on the curve
3
x = 4secq - 5, y = 4tanq + 1 intersect at the point whose coordinates are
sin x ; 2 < x < 3

(d) f(x) =  1 (s) (7, 4)
 x ; 3<x<9

f(x) is decreasing in the interval


2.98  Differential Calculus

I IT Assig n m e n t E x e rcise

Straight Objective Type Questions

Directions: This section contains multiple choice questions. Each question has 4 choices (a), (b), (c) and (d), out of
which ONLY ONE is correct.

x 8 − 6561
51. lim is
x → 3 x 4 − 81

(a) 162 (b) 3 (c) 9 (d) 81


2
x −1
52. lim is
x →1 | x − 1|
(a) 1 (b) 2 (c) –2 (d) Does not exist
 1+ x − 1− x 
53. lim   is
x →0
 x 
(a) 0 (b) –1 (c) 2 (d) 1
 1− cos kθ 
54. lim  is
θ→0 1− cos Rθ 

k2 R2
(a) k2 R2 (b) (c) (d) 1
R2 k2

a x + log(1 + x) − sin x − cos2x


55. The value of lim where, a > 0, is
x →0 ex + 1
1 −1
(a) (b) 0 (c) (d) 2
2 2
dy
56. If e x + y = x y , then is
dx
log x log x − 1 log x − 2
(a) (b) (c) (d) 1
(log x − 1) (log x + 1) (log x − 1)
2 2 2

p  dy 2
57. If x = a cos q, y = a sin q, where 0 < q < , then 1+  is
3 3
2  dx 
(a) sec q (b) sec2 q (c) a sec q (d) tan q

 13 1 
x −a 3 dy
58. If y = tan −1   , x > 0, then is
 1 + x 13 a 13  dx

1 3 1 1
(a) (b) (c) (d)
x
2
3
(1 + x )
2
3
x
2
3
(1 + x )
2
3
3x
2
3
(1 + x ) 2
3
3x
1
3
(1 + x )
2
3
Differential Calculus  2.99

59. The speed v of a particle moving along a straight line at a distance x from the origin is given by 4 + 5v2 = x2. The ac-
celeration of the particle is
5 x
(a) 25x (b) (c) 5x2 (d)
x 5
3
  dy  2  2
1 +   
  dx  
60. If x = a cos q, y = a sin q, then
3 3
is equal to
d2 y
dx 2
(a) sec q (b) 3a sin q cos q (c) 3a tan q (d) a sin q cos q

d2 y
61. If y2 = 4ax, then is
dx 2
a2 a3 −4a 2 3a 2
(a) (b) (c) (d)
y2 y2 y3 2y 2

62. If f(x) = 3 sin x – 4 cos x – kx + l, such that f(x) is a decreasing function for all real values of x, then
(a) k ≥ 5 (b) k ≤ 2 (c) k is always negative (d)1 ≤ k ≤ 3
x2 x 4 x3 x5
63. If f(x) = 1 + + + ... ∞ and g(x) = x + + + ...∞ and f (x) = f(x) + g(x), then f(x) is
2! 4! 3! 5!
(a) an increasing function for all positive real values of x and decreasing for negative real values of x.
(b) an increasing function for all real values of x.
(c) a decreasing function for all real values of x.
(d) a decreasing function for all positive real values of x and increasing for negative real values of x.
64. If the function f(x) = 2x3 – 9ax2 + 12a2 x + 1, where a > 0, attains its maximum and minimum at x = p and x = q
respectively such that p2 = q, then the value of ‘a’ is
1 1
(a) 2 (b) (c) (d) 4
4 8
px + q
65. If the function y = has an extremum at P (2, -1), then
( x − 4 ) ( x − 1)
(a) p = 0, q = 1 (b) p = 1, q = 0 (c) p = 1, q = 1 (d) p = 2, q = -1
66. For the parabola y2 = 4ax, the ratio of sub tangent to the ordinate is equal to
(a) 1 : 1 (b) x : y (c) 2x : y (d) x 2 : y
67. If the function f(x) = px2 + qx2 + rx + s on [0, 1], satisfies the mean value theorem, then the value of c in the interval
(0, 1) is
1 1 2 2
(a) (b) (c) (d)
2 3 3 3 3
68. Which of the following functions is differentiable at x = 0?
(a) sin |x| + |x| (b) sin |x| - |x| (c) cos |x| + |x| (d) cos |x| - |x|

69. If the pressure P and volume V of a gas are connected by the relation PV1/4 = C where, C is a constant, then the
1
percentage increase in the pressure corresponding to a decrease of % in the volume is
2
1 1 1
(a) % (b) % (c) % (d) 1%
4 8 2
2.100  Differential Calculus

1
70. The value of lim 1
is
x →0
5 + 3x
(a) 5 (b) 3 (c) 0 (d) does not exist

e5x − e −3x
71. The value of lim is
x →0 sin x + tan x
(a) 16 (b) 4 (c) 8 (d) 2
1
 1x + 2 x + .......... + n x  x
72. lim   is
x →0  n
(a) n ! (b) (n – 1) ! (c) (n !)1 / n (d) n

73. lim (cos x) cos x


is
p
x→
2
1 2
(a) 1 (b) 0 (c) (d)
e e

 g(x)f(a) − g(a) f(x) 


74. If f(a) =2, f ’(a) = 1, g(a) = -1, g’(a) =2, then the value of lim   is
x →a
 x−a 
1 −1
(a) –5 (b) (c) 5 (d)
5 5
 e tan x − e x 
 
75. lim   is
x →0  tan x − x 
 
1
(a) 1 (b) e (c) (d) e - 1
e
76. lim
x →∞
( x 2 + ax + b − x is )
a −a
(a) a (b) – a (c) (d)
2 2
 1 1 1 
77. lim + + ... +  is
n →∞ 1⋅3 3⋅5 (2n −1)(2n +1) 
1
(a) 0 (b) 1 (c) (d) 2
2
3x + 1 , x ≤ 1
78. Let f(x) =  2
. The value of ‘a’ for which f(x) is continuous is
2 − ax x > 1
(a) -2 (b) 6 (c) 1 (d) 2

 sin(4k − 1)x
 , x≤0
79. The value of ‘k’ so that f(x) =  3x is continuous everywhere in its domain of definition is
 tan(4k + 1)x , 0 < x < p
 5x 2
1 1
(a) 1 (b) (c) − (d) 0
4 4
Differential Calculus  2.101

3sin x − 2x  −p p 
80. The value of f(0) so that f(x) = is continuous at each point of  ,  is
tan x + 4x 2 2
1 3
(a) 0 (b) (c) (d) 1
5 4
1

(27 − 2x )
3
−3
81. The function f (x) = 1
, x ≠ 0 is continuous at x = 0. The value of f (0) should then be defined as
9 − 3 (243 + 5x )
5

2
(a) (b) 6 (c) 2 (d) 4
3
1
82. The number of points at which the function f(x) = ceases to be continuous is
sin | x |
(a) 0 (b) 1 (c) finitely many (d) infinitely many
83. The coordinates of a point on the curve x3 + y3 = 6xy at which the tangent is parallel to the x-axis are

(a) (2 4
3
, −2
5
3
) (b) 2 3 , 2( 4 5
3
) (c) (2 5
3
,2
4
3
) (d) None of these

3 3
x 2
y 2
84. It is given that the line x cos q + y sin q = p touches the curve   +  = 1. Then
6 4
(a) (6 cosq)3 + (4 sin q)3 = p3 (b) (6 cos q)3 – (4 sin q)3 = p3
3 3 3
(c) (6cos q) 2 + (4sin q) 2 = p 2 (d) 36 cos2 q + 16 sin2 q = p2
2 2 2
85. If l and m are the lengths of perpendiculars from the origin to the tangent and normal to the curve x 3
+y 3
=5 3

respectively, 4l2 + m2 is
2
(a) 625 (b) 125 (c) 25 (d) 25 3

p
86. The normal at the point P corresponding to q = on the curve x = 3 cos q - cos3 q, y = 3 sin q - sin3 q meets the
3
x-axis in G. If N is the foot of the perpendicular from P to the x-axis, NG2 equals
3 3 9 9
(a) (b) (c) (d)
8 16 32 64
87. Given that x2 – 5x + 4 + 0, the minimum value of f(x) = 2x3 – 25x2 + 100x + 14 is
(a) -861 (b) 142 (c) 91 (d) 109
x
88. The coordinates of the point on the curve y = where the tangent has the greatest slope are
1 + x2
 − 3
(a) (1, 2) (b)  − 3,
 4 
 3
(c)  3, (d) (0, 0)
 4 

89. The sum of the squares of the roots of the quadratic equation x2 + (3 + a) x + 2 (a3 – 2a + 5) = 0 is maximum for ‘a’ is
7 −7
(a) (b) –1 (c) 1 (d)
6 6
2.102  Differential Calculus

90. The curve y = ax3 + bx2 + cx + 5 touches x-axis at A(-2, 0). The curve intersects the y-axis at a point B where its slope
equals 3. The value of ‘a’ is
−1 1
(a) –2 (b) 2 (c) (d)
2 2
91. If f(x) = 2 cos x – x + 5, x ∈ [-2p, 2p], the interval in which f is monotonic increasing is
 −5p 7 p   −5p − p   7 p 11p 
(a)  , (b)  , ∪ ,
 6 6   6 6   6 6 
 −5p   − p 7 p   p 7p 
(c)  −2p, ∪ , (d)  − , 
 6   6 6   6 6 

92. The x coordinate of the point on the curve xy = (2 + x)2, the normal at which cuts off numerically equal intercepts on
the coordinate axes is
1 1
(a) (b) (c) 2 (d) 2
2 2

( )
1
93. If 0 < x < y, lim y n + x n n
is
n →∞

(a) e (b) x (c) y (d) nxn–1

5sin [cos x ]
94. If [ ] denotes the greatest integer function, lim is
x→
p
2
[cos x ] + 2
(a) 0 (b) 1 (c) ∞ (d) Does not exist
 2x − 1 
95. lim  =
x →0 
1+ x −1
log e 2
(a) 2 (b) log e 2 (c) (d) 2 log e 2
2
96. The height of the cylinder of maximum volume that can be inscribed in a sphere of radius 6 cm is
(a) 4 3 (b) 3 3 (c) 4 (d) 9

m
97. If lx 2 + ≥ n for all positive values of x (l > 0, m > 0), then
x
(a) 27lm2 ≤ 4n3 (b) 27lm2 ≥ 4n3 (c) 4lm2 ≥ 27n3 (d) 4lm2 ≤ 27n3
px 2 + qx + r
98. Let f(x) = . Given that lim f(x) = 3, lim f(x) = 2 and lim f(x) = 4 , lim f(x) is
x2 + x + 1 x →0 x →−1 x →1 x →2

29 7
(a) (b) (c) 29 (d) 7
7 29
sin x
99. lim x + is
x →0 sin x
x+
x + ...
1
(a) 0 (b) 1 (c) -1 (d)
2
  x +1  p
100. lim x sin −1  − is
x →∞
  2x + 1  6 
1 2 3
(a) (b) (c) (d) 2 3
2 3 3 2
Differential Calculus  2.103

d2 y dy
101. If y = sin (8 sin-1x) then (1 - x2) 2 -x equals
dx dx
y y
(a) -64y (b) (c) 64y (d) -
64 64
102. The number of points at which the circle x2 + y2 = 8x and the curve (2 - x)y2 = x3 intersect at 45° is
(a) 0 (b) 1 (c) 2 (d) 3

 3x , 0 ≤ x ≤ 1,
103. Consider f(x) = |x2 - 3|, 0 ≤ x ≤ 6 and g(x) =  . Then Rolle’s theorem can be applied in the respective

 4 − x, 1 < x ≤ 3
intervals
(a) to both f(x) and g(x) (b) only to f(x)
(c) only to g(x) (d) neither to f(x) nor to g(x)
p cos x + sin x
104. The function f(x) = is monotonically increasing for all x ∈ R if p belongs to
p sin x + cos x
(a) (-1, 1) (b) (-∞, -1) (c) (1, ∞) (d) R - (-1, 1)

105. The approximate value of 3 1000025 is


(a) 105 (b) 100.0008 (c) 100.008 (d) 100.00008
106. The respective values of p, q, r for which f(x) = p log |x| + qx + rx + x has extremum at x = ± 1, 2, are
3 2

1 1 1 1
(a) −2, − ,1 (b) −2, , −1 (c) −2, − , −1 (d) 2, ,1
3 3 3 3
n
 1
107. lim  cot x −  is
x →0  x
(a) n (b) -n (c) 0 (d) 1
sin x
1
108. lim   is
x →0  x 

(a) 1 (b) -1 (c) e (d) Does not exist

 x  dy  1 
109. If y = tan −1   , | x | ≤ 1, then dx at  2  is
 1 + 1 − x2 
1 3 2
(a) (b) 3 (c) (d)
3 2 3
110. For a sphere of radius ‘r’, the ratio of percentage rate of change in the volume to the percentage rate of change in the
radius is
(a) 1 : 3 (b) 3 : 1 (c) 1 : 1 (d) None of these
111. Let f(x) = [x2 - x + 1] where [ ] denotes the greatest integer function. Then, in (0, 2), f(x) is discontinuous at
the point
1+ 5 1− 5
(a) (b) (c) 1 (d) Both (a) and (c)
2 2
dy
112. If f ’(x) = cos (x2-1) and y = f(x2 + x + 1) then at x = 0 is
dx
(a) 0 (b) 1 (c) -1 (d) None of these
2.104  Differential Calculus

1  p
113. The function f(x) = x(1 - x cot x) - in  0,  has
x  2
(a) only one minimum (b) only one maximum
(c) no extrema (d) one maximum and one minimum
n(n + 3)
2x + 3x 2 + .... + (n + 1)x n −
114. lim 2 is
x →1 x −1
n(n + 1)(n + 2) n(n + 1)(2n + 1)
(a) (b) 0 (c) (d) 1
3 6
115. Let f(x) = max (1 - x, x2 -1). Then f is
(a) not continuous at x = 1, -2 (b) continuous and differentiable everywhere
(c) not differentiable at x = -2, 1 (d) continuous but not differentiable at x = 1, -1
f ii (0) f iii (0) f iv (0) f n +1 (0)
116. If f(x) = (1 - x)-1, |x| < 1 then the value of + i + ii + ... + n −1 is
f(0) f (0) f (0) f (0)
n(n + 1)(n + 2) n(n + 1)(2n + 1) n(n + 1)(n + 2)
(a) (b) (c) (d) None of the above
3 6 6

 x + 3, x ≤ 3
117. If f(x) =  2 then for f (f(x)) = g(x) we have
 x − 3, x > 3
(a) lim g(x) exists but lim g(x) doesn’t exist (b) lim g(x) exists but lim g(x) doesn’t exist
x →0 x →3 x →3 x →0

(c) lim g(x) and lim g(x) exist (d) neither lim g(x ) nor lim g(x) exist
x →0 x →3 x →0 x →3

118. The number of points at which the tangent is parallel to x - axis for y = x (x -4) e in [0,4] is 2x

(a) 0 (b) 1 (c) 2 (d) 3


 p
119. If f(x) = 8 + sin 3x cos x and g(x) = 8 + cos 3x sin x for x ∈  0,  then
 2
 p p p
(a) f(x) > g(x) in  0,  (b) f(x) < g(x) in  , 
 2 4 2
 p p  p
(c) f(x) = g(x) in  ,  (d) f(x) < g(x) in  0, 
 12 6   4
120. The number of critical points of f(x) = min (sin x, cos x) in (0, 2p) is
(a) 4 (b) 1 (c) 2 (d) 3
4x 3 + 8x 2 + 5x + 2
121. lim is
x →∞ 2x 3 − 1
(a) 4 (b) 2 (c) -1 (d) -2
xsin2x
122. lim equals
x →0 sin x 2
1
(a) 1 (b) 2 (c) (d) 0
2
123. lim−  x  + 1 , where [ ] denotes the greatest integer function, is equal to
x→4

(a) 3 (b) 4 (c) - 4 (d) 2


Differential Calculus  2.105

3x 2 − 4x + 1
124. lim is
x →1 x2 − 1
(a) 0 (b) 1 (c) –1 (d) 2
eax − e bx
125. lim is
x →0 x
b
(a) a + b (b) a − (c) a – b (d) (a – b)2
2

| x |
 , x≠0
126. The function f(x) =  x is
0 , x = 0
(a) continuous everywhere (b) continuous nowhere
(c) continuous everywhere except at x = 0 (d) discontinuous at all integer points.

 x  x
log 1 +  − log 1 − 
 2  2
127. If f(x) = for x ≠ 0 and f(0) = a and f(x) is continuous at x = 0, then a is
x
1
(a) 0 (b) 1 (c) –1 (d)
2
dy
128. If y = u4 where u = cos x, is
dx
(a) 4u3 (b) –4 cos3 x sin x (c) 4 sin3 x cos x (d) u

 1 − t2   1 + t2  dy
129. If x = cos −1  2 
; y = sec −1  , where 0 < t < 1, then is
1 + t   1 − t 2  dx
1
(a) 1 (b) 0 (c) –1 (d)
2
 1 + sin x  dy
130. If y = tan −1  , is
 cos x  dx
1 1
(a) 1 (b) (c) (d) 0
2 3

du
131. If u = 3x12 and v = x6, then is
dv
(a) 6x6 (b) 36x11 (c) 6x5 (d) 3x6

7 dy 1
132. If x2 + xy + y2 = , then at x = 1 and y = is
4 dx 2

3 −5 21 −21
(a) (b) (c) (d)
4 4 8 8
2 dy
133. If y = esin x. e x , then is
dx
(a) y cos x (b) y2 (x + sin x) (c) y2 (x2 + sin2 x) (d) y (2x + cos x)

134. If f(x) = (1 + x)(1 + x2)(1 + x4)(1 + x8), then f ’(1) is


(a) 16 (b) 40 (c) 80 (d) 120
2.106  Differential Calculus

−1 − cos −1
x d2 y
135. If y = esin x
and z = e , then is
dz 2
p
(a) 1 (b) (c) e (d) 0
2
136. If g(x) = f(x) – (f(x))2 + (f(x))3 for every real number x, then which of the following is true?
(a) g(x) is increasing whenever f(x) is increasing (b) g(x) is decreasing whenever f(x) is decreasing
(c) both (a) and (b) (d) neither (a) nor (b)
137. For the function f(x) = 2 – x4, at the point x = 0,
(a) f(x) is a minimum (b) f(x) is maximum
(c) f(x) is neither a maximum, nor a minimum (d) f(x) has a point of inflexion
138. The maximum and minimum slope of the curve given by the equations x = 2t (t2 + 3) – 3t2 and y = 2t (t2 + 3) + 3t2 are
 1  1  1  1
(a) 2,  (b) 2,  (c) 3,  (d) 3, 
 2   3   2   3
139. If A (x1, y1) and B (x2, y2) are two points on the parabola y = 8ax2 and C (x3, y3) is a point on the arc AB such that the
tangent at C to the parabola is parallel to the chord AB, then
(a) x1, x3, x2 are in AP (b) y1, y2, y3 are in AP (c) y1, y2, y3 are in GP (d) x1, x2, x3 are in AP
cos3x − cos7x
140. lim is
x →0 x2
(a) 40 (b) 20 (c) 21 (d) 10

x 2 +1 − 3 x 3 +1
141. lim is
x →∞
x 4 +1 + 5 x 5 +1
1
(a) 1 (b) –1 (c) 0 (d)
2
142. The point(s) at which the function
x3 , x ≤ −1

2 x + 1 , −1 < x ≤ 0
f(x) =  x fails to be continuous is (are)
e , 0 < x ≤1
x2 , 1< x

(a) x = 1 (b) x = 0, -1 (c) x = -1 (d) x = -1, 0, 1
143. The tangent at a point P on the curve x4 y5 = a9 meets the x and y axes in A and B respectively. Then, AP : PB is
4 5 9 1
(a) (b) (c) (d)
5 4 1 9

2 + 4 − x2
144. The tangent at a point P on the curve y = log − 4 − x 2 meets the y-axis at T. Then PT2 is
2
2− 4−x
(a) 4 (b) 16 (c) 8 (d) 2
145. The x coordinate of a point on the curve 9y2 = x3, the normal at which cuts off equal intercepts on the axes is
(a) 2 (b) 4 (c) 8 (d) 6
|x|
146. The function f(x) = is differentiable on
1 + x2
(a) [0, ∞) (b) (-∞, 0] (c) (-∞, ∞) (d) (-∞, ∞) - {0}.
Differential Calculus  2.107

147. The set of x coordinates of the points on y = x3 - 9x2 + 24x + 13 where the tangents make an acute angle with the posi-
tive x-axis is
(a) f (b) (2, 4) (c) (-∞, 2) ∪ (4, ∞) (d) R
148. The function f(x) = (x4 - 42x2 - 80x + 32)3 is
(a) monotonically increasing in (-4, -1) ∪ (5, ∞) (b) monotonically increasing in (-∞, -4) ∪ (-1, 5)
(c) monotonically decreasing in (-4, 5) (d) monotonically increasing in (-1, 5)
x
 x −5
149. lim   is
x→∞  x + 3 

(a) 8 (b) -8 (c) e-8 (d) e8


p
x x − sin x
150. lim 2 is
x →1 p
cos x
2
2 p p
(a) -2p (b) − (c) (d) -
p 2 2

Assertion–Reason Type Questions

Directions: Each question contains Statement-1 and Statement-2 and has the following choices (a), (b), (c) and (d), out
of which ONLY ONE is correct.
(a) Statement-1 is True, Statement-2 is True; Statement-2 is a correct explanation for Statement-1
(b) Statement-1 is True, Statement-2 is True; Statement-2 is NOT a correct explanation for Statement-1
(c) Statement-1 is True, Statement-2 is False
(d) Statement-1 is False, Statement-2 is True
151. Statement 1
 x + 4
2
0 < x <1
Consider f(x) =  , then f(3) is positive.
2x + 3 1≤ x < 4
and
Statement 2
If f(x) is differentiable in an interval I and is increasing in I, then, f(x) > 0 in I.
152. Statement 1
f(x) = sin(x - [x]) + ex-[x] where, [ ] denotes the greatest integer function is periodic with period 1.
and
Statement 2
g(x) = x - [x] is a periodic function with period 1.
153. Statement 1
Maximum value of f(x) = x3 - 5x2 + 9x + 1 in [0 2] occurs at x = 2.
and
Statement 2
Given that f(x) is differentiable in an interval (a, b) and if there exists a point x0 where, a < x0 < b such that f ’(x0) = 0
and f ”(x0) < 0, then, f(x) is a maximum at x = x0.
2.108  Differential Calculus

Linked Comprehension Type Questions

Directions: This section contains 1 paragraph. Based upon the paragraph, 3 multiple choice questions have to be
answered. Each question has 4 choices (a), (b), (c) and (d), out of which ONLY ONE is correct.
Let C be a point on the normal at P such that CP = r
The circle whose centre is C and radius equals the radius of curvature of the curve at P is called the ‘circle of curvature’
at P. C is called the centre of curvature of the curve at P.
The coordinates of the centre of curvature at P are ( X ,Y ) , X and Y are given by the formulas

( )
3
 y '  1 + y ' 2  1 + y '2
y
(
X = x1 −  1 + y '
"
2
) , Y = y 1 +   and r 2
=
y "2
 at (x1 , y1 )  y " at (x , y )
1 1

dy d2 y
where y ' ≡ and y " ≡ 2 .
dx dx
The equation of the circle of curvature at P is ( x − X ) + ( y − Y ) = r2
2 2

(
or |z - z0| = r where, z = x + i y, z0 = X + i Y i = −1 . )
1 1
154. The coordinates of the centre of curvature at the point  ,  on y = x2 are
2 4

 1 5 1 9  1 9  1 5
(a)  − ,  (b)  ,  (c)  − ,  (d)  − , 
 2 4 2 4  2 4  2 2

155. The equation of the circle of curvature at the point (0, 1) on y = ex is given by
(a) x2 + y2 - 4x - 6y + 5 = 0 (b) x2 + y2 +4x - 6y + 5 = 0
(c) x + y +4x + 6y - 5 = 0
2 2
(d) x2 + y2 +4x + 6y - 8 = 0

x2
156. Equation of the circle of curvature on the curve y = mx + at the origin is
a
(a) x2 + y2 = a2(1+ m2)2 (b) x2 + y2 = a(1+ m2) (y - mx)
(c) x2 + y2 = a(1+ m2) (y - mx) + (1 + m2)2 (d) none of these.

Multiple Correct Objective Type Questions


Directions: Each question in this section has four suggested answers out of which ONE OR MORE answers will be correct.
157. Given that f(x) is a linear function. Then the curves
(a) y = f(x) and y = f-1(x) are orthogonal (b) y = f(x) and y = f-1(-x) are orthogonal
(c) y = f(-x) and y = f-1(x) are orthogonal (d) y = f(-x) and y = f-1(-x) are orthogonal
158. If f(x) = ax2 + bx + c is continuous in [a, b] and differentiable in (a, b) such that f(a) = f(b) + (a - b) f ’(g), then
(a) a, g, b are in AP (b) a, g, b are in HP
(c) a + b + g = 0 (d) (a + b + g)3 = 27g3
Differential Calculus  2.109

159. If f(x) = max{x2 - 4, |x - 2|, |x - 4|} then


−1 ± 33
(a) f(x) is continuous for all x ∈ R (b) f(x) is differentiable except at x =
2
(c) f(x) has a critical point at x = 2 (d) f(x) has no maximum

Matrix-Match Type Question

Directions: Match the elements of Column I to elements of Column II. There can be single or multiple matches.
160.
Column I Column II
f(x) Property
 2x 
(a) sin-1  2 
(p) decreases in (0, ∞)
1 + x 

(b) 6x
4
3
1
− 3x 3 (
(q) increases in 1 , ∞
2 )
9
(c) 2x2 - log|x|; x ≠ 0 (r) has a minimum = -
8
log(x + e)
(d) log x (s) has absolute maximum at x = 1
2.110  Differential Calculus

Ad dit io n a l P r a ct ic e E x e rcis e

Subjective Questions
pxe x + 2q log(1 + x) + 3rxe − x
161. Determine p, q, r such that lim =3
x →0 x 2 sin x

x
1 − sin
162. Evaluate lim 2
x→p  x  x x
 cos 2   cos 4 − sin 4 

163. Prove that


(i) lim x n e − x = 0
x →∞

(ii) lim e − mx cosax = 0 (m > 0)


x →∞

(iii) lim e − mx sinax = 0 (m > 0)


x →∞

 q  q  q
164. If f(n, q) = 1 − tan2  1 − tan2 2  1 − tan2 3  ...n factors
 2   2   2 
find lim f(n, q) .
n →∞

3x cos x + x n e x
165. Check the continuity of the function f(x) = lim in [0, ∞)
n →∞ (2 + x n )

166. Let f: R → R such that for all x1, x2 ∈ R,| f(x2) – f(x1) | ≤ | x2 - x1 |3. Prove that f(x) is a constant function.

dn n
167. If In =
dx n
( )
x log x , show that In = nIn −1 + (n − 1)! . Hence deduce that

 1 1 1
In =  log x + 1 + + + ... +  n!
 2 3 n

a sin x + b cos x
168. Prove that f(x) = is monotonic decreasing in R if (ad - bc) < 0.
csin x + d cos x

1
169. Use the function f(x) = x x , x > 0 to determine the order of relationship between ep and pe.

170. Show that -4x3 + 18x2 - 24x + p = 0 has a unique root in (1, 2) if 8 < p < 10.
171. f(x) and g(x) are two differentiable functions, for x in [a,b] such that f(a) = 3; g(a) = -3; f(b) = 30; g(b) = 6. Prove that
there exists a point c satisfying a < c < b at which the ratio of the derivatives of f and g is 3:1.

x = t 5 − 5t 3 − 20t + 7 
172. A function is defined parametrically as follows: |t|< 2
y = 4t 3 − 3t 2 − 18t + 3 
Find the maximum and minimum values of y and find the values of t where they are attained.

173. Normal at P on y = x2 meets the curve again at Q. Find the coordinates of P such that PQ is minimum.
Differential Calculus  2.111

2
174. Show that the cone of the greatest volume which can be inscribed in a given sphere has an altitude equal to of the
3
diameter of the sphere.
175. A sector with a central angle a (radians) is cut out of a given circular lamina, to be made into a cone. Find the value
of the angle a, which will yield the greatest possible volume of the cone.
sin(a + 3h) − 3sin(a + 2h) + 3sin(a + h) − sina
176. Evaluate lim .
h→0 h3
d 2 u a 2 b2
177. If u2 = a2 cos2 x + b2 sin2 x, prove that u + = 3 .
dx 2 u
178. Find the nth derivatives of
(i) eax cos (bx + c) (ii) eax sin (bx + c)
179. Show that by changing the independent variable x to z using the substitution x2 = z the relation
d2 y dy 2
2 d y dy
x2 2
+ x + 4(x 4
− l 2
)y = 0 reduces to z 2
+ z + (z 2 − l 2 )y = 0
dx dx dz dz

180. Sum the series ∑r
r =1
2
x r − 2 , | x |< 1 using Calculus.
1
181. (i) Find Sr, the sum of the infinite geometric series whose first term is r and common ratio is
r +1
 2 n−1  
3

 ∑ (S −1)  
r
  
(ii) Evaluate lim  r=1 2

n →∞ 2 n−1
 
 ∑ (Sr −1)  
2

 r=1 
182. The function f(x) is defined as f(x) = x + |2 - x|, -2 < x < 3 where x denotes the least integer greater than or
equal to x.
(i) Split the function f(x) into integral intervals
(ii) Discuss the continuity of f(x)
(iii) Draw the graph of f(x).
183. A function y = f(x) is defined as x = 3t - |t|, y = e4t for all t.
(i) Express y in terms of x.
(ii) Discuss the continuity of f(x).
2x − 1, − 4 ≤ x ≤ 0
184. Let f(x) = 
 x −3, 0< x < 4
(i) Determine the functions g1(x) = f(|x|) and g2(x) = |f(x)|.
(ii) Discuss the continuity and differentiability of g(x) = g1(x) + g2(x).
p
185. Let f(x) = 24x5 - 70x3 + 45x2 +
4
(i) Find the intervals where, f(x) is increasing or decreasing.
(ii) Find the values of p for which all the roots of the equation f(x) = 0 are real and distinct.
186. Consider the functions f(x) = x2 + x + 1 and g(x) = x2 - x - 1.
(i) Determine f(g(x)) and g(f(x)).
(ii) Find the values of x for which the rate of change in f(g(x)) equals that in g(f(x)).
2.112  Differential Calculus

187. A vessel in the form of an inverted cone of height 10 feet and semi vertical angle 30° is filled with a solution. This
solution is drained through an orifice at its bottom into a cylindrical beaker of radius 6 feet in such a way that the
height of solution in the conical vessel decreases at a uniform rate of 2 inches/min. Find the rate at which the height
of the solution increases in the beaker when the height of the solution column in the conical vessel is 6 ft.
[Hint: Flow out of conical vessel is equal to flow into the beaker]
188. A balloon is in the form of a right circular cone surmounted by a hemisphere having the radius equal to half the height
of the cone. Air leaks through a small hole; but the balloon keeps its shape. What is the rate of change of volume with
respect to the total height (H) of the balloon when H = 18 cm.
189. Side of a regular hexagon increases at the rate of 3cm/hour. At the instant when the side is 120 3 cm, find the rate of
increase in the
(i) area of the hexagon.
(ii) radius of the inscribed circle of hexagon.
(iii) radius of the circumscribed circle of the hexagon.
(iv) And also find the ratio of the rate of increase in the areas of the inscribed circle, the hexagon and the circumscribed
circle.
190. Let the real valued function f be such that f(xy) = f(x) + f(y) for all positive x and y.
(i) If the function is continuous at x = 1, show that it is continuous for all positive x.
(ii) If the function is differentiable at x = 1, show that it is differentiable for all positive x.

Straight Objective Type Questions


Directions: This section contains multiple choice questions. Each question has 4 choices (a), (b), (c) and (d), out of
which ONLY ONE is correct.

d  1 − x 2  d  −1  2x  
191. cos −1  + tan   , 0 < x < 1 equals
dx  1 + x 2  dx   1 − x 2  
2 3 4 5
(a) (b) (c) (d)
1 + x2 1 + x2 1 + x2 1 + x2

dy
192. Let y = sin x + sin x + sin x + ...... , then is
dx
2y cos x 2y − 1 cos x
(a) (b) (c) (d)
cos x 2y − 1 2y + 1 2y + 1

dy
193. If x = sin 2t; y = log t, then is
dx
2t cos t 2cos t sec2t
(a) (b) (c) (d)
1+ t (t + 1)
2
cos t 2t

du
194. If u = 2ex and v = log x, then is
dv
2e x
(a) 2ex (b) (c) 2ex log x (d) 2x . ex
log x
Differential Calculus  2.113

p dy
195. If y = sin-1 (cos x) + cos-1 (sin x), 0 < x < , then is
2 dx
(a) –2 (b) 2 (c) 1 (d) 0

 2x + 3  dy
196. If f(x) = sin (log x) and y = f  , then at x = 1 is equal to
 3 − 2x  dx
12 5
(a) cos (log 5) (b) sin (log 5) (c) cos (log 5) (d) sin (log 5)
5 12
197. A function f: R → R is such that f(x + y) = f(x) . f(y) for all x, y in R and f(x) ≠ 0 for any x in R. If f(x) is differentiable
and f ’(0) = 2, then
(a) f ’(x) = 2f(x) (b) f(x) = 2f ’(x) (c) f(x) = f1(x) (d) f ’(x) = -f(x)

cos3x
198. lim is
x→
p cos7x
2
3
(a) 3 (b) 7 (c) 21 (d)
7
6a x 2 + ax + a 2 − x 2 + a 2
199. lim 1
is
x →∞
 1
a  6x + 2a −  − xe x
 ax 
(a) 6a - 1 (b) 6a + 1 (c) 1 (d) 0

x13x − x
200. lim is
x →0
1+ sin2 x − 1− sin2 x
1 1
(a) log 8 (b) log 9 (c) log 7 (d) log 13
2 2
 x 3 − ( k + 4) x + 2.k x≠3
 ,
201. If the function f(x)  x −3 is continuous at x = 3, then k is
8 , x =3

(a) 3 (b) -3 (c) -15 (d) 15

f (x) − 3
202. If f(9) = 9 and f ’(9) = 4 , then lim is
x →9
x −3
(a) 4 (b) 3 (c) 2 (d) 1
203. A kite flying at a height of h metres has x metres of string paid out at time t seconds. If the kite moves horizontally
with a speed of v metres/sec and assuming the string to be taut, the rate at which the string is paid out is given by
vx vh v 2 v 2
(a) (b) (c) x − h2 (d) x + h2
h x x x
204. A submarine telegraph consists of a core of copper wires with a covering made of non-conducting material.
If x denotes the ratio of the radius of the core to the thickness of the covering, it is known that the speed of signaling
1
varies as x 2 log   . The greatest speed is attained when x is
x
1 1
(a) e (b) e (c) (d)
e e
2.114  Differential Calculus

 1 − tan3 x p p
 , ≤x<
 3 2 (cos x − sin x) 6 4
 p p
205. If f(x) =  p, x= is continuous at x = then the values of p and q are respectively,
 4 4
 q(1 − 2 sin x) p p
 , <x<
 cos2x 4 3

(a) 1, 2 (b) 2, 3 (c) 1, 1 (d) 2, 2

206. The function f(x) = min (1 - x, 2), x ∈(-∞, ∞) is


(a) continuous everywhere but not differentiable at x = -1
(b) continuous at all points except at x = -1
(c) continuous nowhere
(d) continuous and differentiable everywhere

x − p + x 2 − p2
207. lim is
x→p
x−p
1
(a) 0 (b) 1 (c) 2p (d)
2p
  1+ tan x  
 log  
  2  1  is
208. lim −
x→ 
π 4x − π  π 
4 sin x − 
  4 
1
(a) 4 (b) (c) 0 (d) does not exist
4
log(tan x cot a)
209. lim is
x→a log(cos a sec x)
(a) cosec a (b) cosec2 a (c) sec a (d) sec2 a

xf(1) − f(x)
210. If f(1) = 10, and lim exists and is equal to 5 then f ’ (1) is
x →1 x −1
(a) 5 (b) -5 (c) 15 (d) -15

 p
cos x x < 2 p
211. Let f(x) =  be continuous for all x. Then the value of is
px + q p ≤ x q
 2
2 p 2 p
(a) (b) (c) - (d) -
p 2 p 2

sin3x − 3sin x
212. lim is
x →0 cos x − cos2 x
3
(a) 0 (b) 1 (c) (d) does not exist
2
Differential Calculus  2.115

213. The set of all points where the function f(x) = sec-1 (cosec x) is not differentiable is

(a) f (b) {x = np/n ∈ Z}


 nπ   (2n + 1)π 
(c)  x = /n ∈ Z  (d)  x = /n ∈ Z 
 2   2 
214. The function f(x) = 1 +|tan x| is
(a) continuous everywhere
(b) discontinuous when x = np, n∈Z.
p
(c) not differentiable when x = (2n + 1) ,n∈Z
2
p np
(d) discontinuous at x = (2n + 1) , n ∈ Z and not differentiable at x = , n∈Z
2 2
dy
215. If cos y = x cos (a - y) then (1 + x 2 − 2x cos a). is
dx
(a) sin a (b) -sin a (c) cos a (d) -cos a
2
d2 y  dy 
216. If x = sin q and y = cos q then 2y 2 + 4   is
3
dx  dx 
(a) 6cos2 q (7 sin2 q - cos2 q) (b) cos2 q (13 sin2 q - cos2 q)
(c) 3cos2 q (cos2 q -13 sin2 q) (d) 3cos2 q (17 sin2 q + cos2 q)
dy
217. If y = cos x + cos x + cos x + .. then is
dx
sin x sin x sin x
(a) (b) (c) (d) (1 + 2y)sinx
2y − 1 1 − 2y 1 + 2y

218. A point P (other than origin) on y = 4x3 - 2x5 such that the tangent at P passes through the origin is
(a) (1, 3) (b) (1, 2) (c) (-1, 2) (d) (2, -32)
x2
219. The angle between the two tangents to the curve y = at the points x = 2 and x = -2 is
4
p p 3p
(a) (b) p (c) (d)
2 4 4
220. If the only point of inflection of the function f(x) = (x - a)m (x - b)n, m, n ∈ N and m ≠ n is at x = a, then
(a) m, n are even (b) m is odd, n is even (c) m is even, n is odd (d) m, n are odd

221. For the curve y = 3x4 - 8x3 - 6x2 + 25x + 5, the interval in which the ordinate increases at a faster rate compared to
the abscissa
(a) (0, 2) (b) (-1, 1) ∪ (2, ∞) (c) (-2, 1) (d) (-∞, ∞)

p
222. The function f(x) = p(6 cosx - 3 cos2x - 2 cos3x) -12 sin x - 6 sin 2x has a minimum at x = . Then p equals
6

(a) 2 3 + 4 (b) 0 (c) 2 (d) 3 + 1

223. Let f(x) = a0 + a1 x2 + a2 x4 + …+an x2n with a1, a2, ….an < 0. Then f(x) has
(a) only one maxima (b) only one minima (c) no extrema (d) None of the above
2.116  Differential Calculus

n
224. If the function f(x) = ∑ (x − r )
r =1
2 2
has a minimum at x = 11 then n equals

(a) 5 (b) 4 (c) 6 (d) 7


1
 sin x  x
225. lim   is
x →0  x 

(a) 1 (b) -1 (c) 0 (d) e


2
x + x 4 + x 9 + ... + x n − n
226. lim is
x →1 x −1
n(n + 1)(n + 2) n(n + 1)(2n − 1)
(a) (b)
6 6
n(n + 1)(2n + 1) n(n − 1)(2n + 1)
(c) (d)
6 6
1
 cos x x−α π
227. lim  where, α ≠ (2n +1) , n∈Z equals
x →α cos α  2
(a) -tan a (b) cot a (c) e-tan a (d) log tan a

x
228. lim is
x →∞
4x + 3x + 2x + x
1
(a) 2 (b) 0 (c) (d) 1
2
n
k2
229. If Sn = ∑
k =1 1 + n
3
then lim Sn equals
n→∞

1 1
(a) 2 (b) − (c) (d) -2
3 3
 sin ([x]+ x )
 x≠0
230. If f(x) =  [x]+ x , where [ ] denotes the greatest integer function, then
1 , x=0

(a) lim f(x) = sin1 (b) lim f(x) = 1
x →0 x →0

(c) lim f(x)doesnot exist (d) lim f(x) exists but f(x) is not continuous at x = 0
x →0 x →0

231. If x2 + 4x + 3 + |y| = 3y then y as a function of x is


(a) discontinuous at x = -1, -3
(b) continuous at x = -1 only
(c) differentiable everywhere
(d) differentiable everywhere except at x = -3, -1

(1 + 2x) − 3 1 + 2x
232. The value of f(0), so that its function f(x ) = is continuous, is
x
1 1
(a) (b) 3 (c) − (d) 0
3 3
Differential Calculus  2.117

233. If f(x) = |x| sin x, then f is


(a) differentiable everywhere (b) not differentiable at x = n p, n∈Z
(c) not differentiable at x = 0 (d) not continuous at x = 0


p
p
(1+ | cos x |) ,
|cos x|

0<x<
 2
 p
234. If f(x) =  q , x=
 2
 cot l (x − p2 ) / cot m(x − p2 ) p
e , <x<p
 2
is a continuous function on (0, p) then the values of p and q are respectively
m ml m m
(a) ,e (b) e l , (c) ml . eml (d) eml , ml
l l
ex − e− x
235. The set of all points of discontinuity of the inverse of f(x) = is
ex + e− x
(a) f (b) (-∞, -1] (c) [1, ∞) (d) R - (-1, 1)

 x 3 , x ≤ x0
236. Let f(x) =  2 . If the two roots of px2 + qx + r = 0 are reciprocal to one another, then, the values of
 px + qx + r , x > x 0

p, q, r for which f(x) is continuous and differentiable at x = x0 are respectively.


2x 30 x 20 (3 + x 20 ) 2x 30 x 20 (3 + x 20 )
(a) p = r = ,q = (b) p = r = ,q =
x 20 − 1 1 − x 20 1 − x 20 1 − x 20
2x 30 x 20 (3 + x 20 ) 2x 30 x 30 (3 + x 20 )
(c) p = r = ,q = (d) p = r = ,q =
x 20 − 1 x 20 − 1 1 − x 20 1 − x 20

g(x)f(p) − g(p)f(x)
237. If f(p) =2, f ’(p) = 6 and lim =0 then g’(p) : g(p) is
x→p x−p
(a) 3 : 1 (b) 1 : 3 (c) 1 : 12 (d) 12 : 1
238. Let f(x) be a continuous function defined for x > 0 satisfying f(xy) = f(x) + f(y). If f ’(1) = 1 then f ’(10) is
(a) 10 (b) 0.1 (c) -10 (d) 100

239. If f(x) = x + 2 3x − 9 + x − 2 3x − 9 then


(a) f ’(x) = 0 for all x ∈ (6, ∞) (b) f is differentiable for all x ∈ (3, ∞)
(c) f is differentiable in [3, ∞) - {6} (d) f is differentiable for all x ∈ (-2, ∞) - {3, 6}
p+q p−q
240. Let f(x) be a continuous function such that f  = . Then
 2  2
 3
1
 (p + q) n 2
+n+  
2
lim f   equals
n →∞  3

 2n 2 + 2n + 2 
 
p−q p+q
(a) (b)
2 2
(c) p + q (c) p - q
2.118  Differential Calculus

241. Let f(x) = x4 - 4x3 + 2x2 - 3x + 5. Then


(a) f(x) = 0 has 2 real roots (b) If f(x) = 0 has minimum at x = a
(c) f(x) = 0 has a root in (1, 2) (d) f(x) = 0 has a root in (3, 4)
242. In (0, 1), f(x) = [3x2 + 1], where [x] stands for the greatest integer not exceeding x is
(a) continuous (b) continuous except at one point
(c) continuous except at two points (d) continuous except at three points
 p  p 3
243. If f(x) = sin2x + sin2  x +  - sinx sin  x +  and g   = 8 then g o f (x) is
 3  3 4
1 3
(a) 8 (b) (c) Cannot be determined (d) 8 sin
8 4
244. If g(x) = 8 + x - x3 + x5 and f be any function such that g o f is defined then g o f is
(a) increasing whenever f(x) is increasing (b) decreasing whenever f(x) is increasing
(c) increasing whenever f(x) is decreasing (d) independent of f(x)

245. The number of tangents to the curve y = sin(x - y), -2p ≤ x ≤ 2p, that are parallel to the line x - 2y = 0, is
(a) 0 (b) 1 (c) 2 (d) 3
246. If the tangents at the two points x = p and x = q to the curve y = (x – p) (x – q) (x – r) are parallel to one
another then
(a) p, q, r are in GP (b) q, p, r are in AP (c) r, q, p are in AP (d) p, r, q are in AP
247. If ax2 + bx + c, (a > 0) is positive for all real x. Then the function
g(x) = 2ax3 + 3(b – 2a)x2 + 6(2a – b +c)x + 30 is
(a) monotonic increasing for all real x (b) positive for all real x
(c) monotonic decreasing for all real x (d) negative for all real x
248. A ladder of length 30 m bears against a vertical wall, with its foot on the horizontal floor in a vertical plane perpen-
dicular to the wall. At the instant when the ratio of the rate at which the top is sliding downwards to the rate at which
the foot is sliding away from the wall is 4 : 3, the distance of the foot of the ladder from the wall is
(a) 24 m (b) 18 m (c) 6 m (d) 12 m
249. Let g(x) = f(x) - [f(x)]2 + [f(x)]3 where f(x) = x3 + 5x2 + 9x + 5 then
(a) g(x) is decreasing for all x ∈R- {0} (b) g(x) is decreasing for all x ∈R
(c) g(x) is increasing for all x ∈R- {0} (d) g(x) is increasing for all x ∈R
p
x−
250. The function f(x) = 2 in (0, p) has
 p
1 −  x −  cot x
 2
 p p 
(a) one maximum in  0,  and one minimum in  , p 
 2 2 
 p p 
(b) one minimum in  0,  and one maximum in  , p 
 2 2 
 p
(c) both maximum and minimum in  0, 
 2
p 
(d) both maximum and minimum in  , p 
2 
Differential Calculus  2.119

 p p
251. For the function f(x) = 3 cos x – 2 cos3 x in  − , 
 2 2
(a) only the greatest value exists (b) only the least value exists
(c) both the greatest and the least values exist (d) neither the greatest nor the least values exist
14 − 10x + x 2 − x 3 , x ≤ 1
252. Let f(x) =  2
.
3x + log10 (p − 4), x > 1
Then f(x) attains the absolute minimum value at x = 1 if p takes values in the interval
(a) (4, 14) (
(b) − 14 , − 2 ∪  4, 14 )
(c)  − 14, 14  ) (
(d)  − 14, −2 ∪ 2, 14 

 x x
253. The set of values of p for which the function f(x) = (p2 − 5p + 6)  cos 4 − sin 4  + (p − 3)x + k has no critical
 4 4
point, is
(a) (0, 4) (b) (-∞, 0) (c) (0, ∞) (d) (0, 3) ∪ (3, 4)
3x + 2
254. Given the function f(x) = , the point of discontinuity of the composite function f 2n (x) where,
4x − 3
fn(x) = f (f (f (…(f(x) (n times) is
3 4 2 −3
(a) (b) (c) − (d)
4 3 3 2
d2 y
255. If x2 + xy + 3y2 = 1, then (x + 6y)3 is
dx 2
(a) 0 (b) -12 (c) 22 (d) -22

 p
1 − cos 4  x − 
 2
256. lim is
p p
x→
2 x−
2
(a) 1 (b) -1 (c) 2 (d) does not exist
n 1 + k (k + 1)2 (k + 2)
257. ∑ t k is equal to, where tk = cot −1
k =1 2(k + 1)
(a) tan (n + 1)(n + 2) - tan 2
-1 -1
(b) tan-1(n + 1) - tan-12
(c) tan-12 (d) n - tan-12
1 + 3.4 1 + 8.9 1 + 15.16
258. lim Tn , where Tn = cot −1 + cot −1 + cot −1 + … n terms is
n→∞ 4 6 8
p
(a) p - cot-12 (b) tan-12 (c) cot-12 (d)
4
S2n
259. Let Sn be the sum to n terms of series 1 + 22 + 3 + 42 + 5 + 62 +…; then lim is
n →∞ S2n +1
(a) 1 (b) 2 (c) 0 (d) ∞

260. The minimum value of f(x) = 4x3 – 9x2 – 30x + 12 where 2 x2 – x – 6 ≤ 0 is


−227
(a) -64 (b) (c) -52 (d) None of the above
4
2.120  Differential Calculus

a a sin2q
261. The points on the curve x = a cos q − cos2q; y = a sin q − which are farthest from the point (2a, 0) is/are
2 2
a   −3a   a −3a 
(a)  , 0  (b)  , 0 (c)  , (d) both (a) and (b)
2   2   2 2 

x y
262. Let the line + = 1 be a tangent to the curve x2 + y2 = c2 then a2, 2c2, b2 are
a b
(a) in AP (b) in GP (c) in HP (d) not in any progression
n
263. The largest term of the sequence given by an = , n ∈N is
n 4 + 1875
1 1 5 1
(a) (b) (c) (d)
200 500 727 20

264. The function f(x) satisfies the following conditions:


 x+y 
(i) f(x) + f(y) = f  for all real x, y such that xy < 1.
 1 − xy 
(ii) f(0) = 0
f(x)
(iii) lim = 1 , then which of the following is true?
x →0 x

(a) f(x) is odd function (b) f(x) ≠ 0 any x ∈ R


(c) f(x) has no critical point (d) (a) and (c)
265. The area of the largest rectangle that can be inscribed in the region bounded by the two curves 6x = 24 - y2 and 3x =
y2 - 24. Assume that the sides of the rectangle are parallel to the coordinate axes is
(a) 32 (b) 22 2 (c) 12 2 (d) 32 2
266. The possible range of values of p if the function f(x) = (p - 4)x + (p - 2)x + (p - 3) x + 2 has a minimum at some x ∈
3 2

(-∞, 0) and maximum at some x ∈ (0, ∞), is


(a) (-1, 4) (b) (2, 4) (c) (0, 4) (d) (3, 4)
x+y x−y
267. A function f : R → R satisfies the equation f(x) + f(y) = 2f  f ∀ x, y ∈ R and f(0) ≠ 0. Then which of
the following is/are true?  2   2 
(a) f(x) is even.
(b) If f ”(0) = -1, f(x) satisfies the equation f ’’(x) + f (x) = 0.
(c) f ”(x) - f(x) = 0
(d) both (a) and (b)

Assertion–Reason Type Questions


Directions: Each question contains Statement-1 and Statement-2 and has the following choices (a), (b), (c) and (d), out
of which ONLY ONE is correct.
(a) Statement-1 is True, Statement-2 is True; Statement-2 is a correct explanation for Statement-1
(b) Statement-1 is True, Statement-2 is True; Statement-2 is NOT a correct explanation for Statement-1
(c) Statement-1 is True, Statement-2 is False
(d) Statement-1 is False, Statement-2 is True
Differential Calculus  2.121

268. Statement 1
 1  1
Let f(x) = [x] +  x +  +  x +  , x ∈ R where, [ ] denotes the greatest integer function. Then, the number of points
 4  2
of discontinuity of f(x) in [-1, 1] is 7.

and

Statement 2
[x + k] = [x] + k where, k is an integer.

269. Statement 1
1 − cos ( x − 2) 1
lim =
x →2 ( x − 2) 2

and
Statement 2
sin q
lim =1
q→ 0 q

270. Statement 1
Let g(x) = sinx and f(x) = ex
f(g(x)) is periodic, since sinx is periodic.
and

Statement 2
Let g(x) and f(x) be two functions of x where, g(x) is periodic. Then, f(g(x)) is also periodic.

271. Let f(x) = 2x3 - 5x2 + 9x - 6


Statement 1
f(x) = 0 has no negative roots.

and
Statement 2
f(k) < 0 for k ∈ (-∞, 0)

272. Statement 1
 2x 
f(x) = sin-1  , x ∈ R is not differentiable at x = ±1.
 1 + x 2 
and

Statement 2
 2tan −1 x , |x| <1
 2x  
sin-1   = 1
1 + x 2  2tan −1   , | x |>1
 x
2.122  Differential Calculus

x2 x3 x 4 x5
273. Let F(x) = x − + − +
2 3 4 5
Statement 1
F(x) has exactly one zero at x = 0.
and
Statement 2
F’(x) is minimum at x = -1.
4
274. Let f(x) = e2x ( x − 1) 5

Statement 1
3
f(x) has a maximum at x = .
5
and
Statement 2
f(x) is not differentiable at x = 1.
275. Statement 1
Roots of the equation 8x4 + 12x3 - 30x2 + 17x - 3 = 0 are of the form a, a, a, b where a, b are real.
and
Statement 2
If g(x) is a polynomial in x such that g(a) = g’(a) = g”(a) = 0 and g’’’(a) ≠ 0, then g(x) = 0 has a root a repeated
3 times.
276. Let f(x) = x2 - x sinx - cosx.
Statement 1
f(x) = 0 has only two real solutions.
and
Statement 2
f(0) = -1
 1 1 1 
277. Let f(x) = x  + + + ...∞  , x > 0 be continuous at x = 0
 x (1 + x ) (1 + x ) (1 + 2 x ) (1 + 2 x ) (1 + 3x ) 
Statement 1
Equation of the normal to the curve y = f(x) at x = 2 is 2x + 9y - 10 = 0.
and
Statement 2
f(x) is differentiable at x = 0.

Linked Comprehension Type Questions


Directions: This section contains 3 paragraphs. Based upon the paragraph, 4 multiple choice questions have to be
answered. Each question has 4 choices (a), (b), (c) and (d), out of which ONLY ONE is correct.
Passage I
Economics is an analytical study concerned with the relations that exist or can be assumed to exist between quantities
which are numerically measurable like forces, interest rates, incomes, cost of production, amount of goods sold and so on.
Differential Calculus  2.123

Mathematical methods are thus possible in the study of economics and economic relations are expressible by means of
mathematical functions.
We consider below some of the functions commonly employed in economic analysis.
Demand functions and curves
Let p denote in definite units, the market price of a consumers’ item say X, and let x denote, in definite units, the
amount of X demanded by the market. Then x is a single valued function of p which can be written in symbolic form
x = f(p) called the demand function of X.
Note that the variables x and p take only positive values. It is assumed that the larger is the price the smaller is the
demand for the good concerned. This assumption implies that x decreases as p increases which means that the demand
function is a monotonic decreasing function.
Elasticity of demand
Price elasticity of demand is defined as the absolute value of the ratio, of the relative change in the demand to the relative
change in the price.
Suppose that the demand changes from x to (x + Dx), when the price changes from p to (p + Dp), then, elasticity of
demand is defined as follows:

 Dx 
 x 
 p   Dx 
Elasticity of demand = =   
 Dp   x   Dp 
 p 

 p   Dx 
In fact,     gives us the average elasticity of demand over the price range (p, p + Dp). But, we are interested in
 x   Dp 
the point elasticity i.e., elasticity of demand at a particular price level, say p.
  p   Dx    p   dx 
Elasticity of demand at price p denoted by ed = lim       =     ,
Dp → 0  x   Dp   x   dp 
 

dx dx
where, denotes the derivative of the given demand function x = f(p). Since x is a monotonic decreasing function,
dp dp
is negative.
 p   dx 
Therefore, ed = −     .
 x   dp 
If ed > 1, we say that demand is elastic.
If ed < 1, we say that demand is inelastic and if ed = 1, we say that the demand is unitary.
If the demand function is given by x = 16 + 6p - p2 where x is the demand for commodity at price p.

278. Elasticity of demand at p = 4 is


1 1 1
(a) (b) (c) (d) 5
2 3 4
279. Price p of the commodity for which the demand is unitary is a root of the equation
(a) 2p2 - 13p + 16 = 0 (b) 3p2 - 12p - 16 = 0
(c) 3p - 2p - 1 = 0
2
(d) p2 + 12p - 1 = 0
280. Elasticity of demand at p = 2 for the demand function x = p e-p
(a) 5 (b) 4 (c) 1 (d) 2
The supply of certain goods in the market is given by x s = a p − b where p is the price, a and b are positive constants
and p > b.
2.124  Differential Calculus

281. An expression for es i.e., elasticity of supply as a function of price is


3
p 2(p − b)
(a) (b) 2p p − b (c) p(p − b) 2 (d)
2(p − b) p

Passage II
Revenue Functions
The demand x for a commodity X is represented by x = f(p) where p is the price per unit. It can also be represented by the
inverse function p = f -1 (x). When the demand is x and the price per unit is p, the product R = xp is called the revenue
obtainable from this demand and price.
R represents the total money revenue of the producers supplying the demand.
R = xp or R = x f -1(x)
i.e., R can be expressed as a function of the price or as a function of the demand. The latter is the more convenient
expression. As in the case of cost function, we can define the average revenue and marginal revenue.

282. If the demand function is p = 9 − x , the value of x for which total revenue is maximum, is
(a) 6 (b) 5 (c) 4 (d) 11
A firm has the following total cost and demand functions as follows:
x3
Cost function C = − 7x 2 + 111x + 50 and the demand function is x = 100 - p.
3

283. Profit function P is given by P is


x3 x3
(a) − 6x 2 + 11x + 50 (b) − + 6x 2 − 11x − 50
3 3
x4
(c) − 7x 3 + 11x 2 + 50x (d) 100 x - x2
3
284. The profit maximizing level of output is given by
(a) 11 (b) 3 (c) 1 (d) None of these
285. The maximum profit is given by
2996 334 505
(a) 555 (b) (c) (d)
3 3 3
Passage III
A manufacturer of an electronic gadget produces x gadget per week at a total cost of (x2 + 78x + 2500). The demand function
600 − p
for his product is x = where the price is Rs p/- per set.
8
286. The revenue function is given by
(a) x2 - 600 x (b) 600x - x2 (c) 600x - 8x2 (d) 600 - 8x2
287. Profit function is given by
(a) -9x2 + 522x - 2500 (b) 9x2 - 522x + 2500
(c) x2 - 79x + 2508 (d) None of the above
288. Number of gadgets to be produced by the manufacturer so as to maximize the profit is
(a) 22 (c) 9 (c) 78 (d) 29
289. The price of a gadget that the manufacturer quotes which yields the maximum profit is
(a) Rs 736 (b) Rs 368 (c) Rs 348 (d) Rs 232
Differential Calculus  2.125

Multiple Correct Objective Type Questions


Directions: Each question in this section has four suggested answers out of which ONE OR MORE answers will be
correct.

290. lim 2
x →∞
( )
25x 2 + x − 5x is equal to
x
2x − log e (1 + x)2 sin
e− x − 1 + x 2(1 − cos x 2 ) 5
(a) lim (b) lim (c) lim (d) lim
x →0 5x 2 x →0 x2 x →0 5x 4 x →0 x
x2 − 4
291. Let f(x) = , x ∈ R . Then
x2 + 4
(a) minimum value of f(x) is 1 (b) y = 1 is an asymptote of the curve y = f(x)
(c) f(x) is bounded (d) f(x) is unbounded
292. Let the function f be defined by
 p + qx + x 2 , x < 2
f(x) =  2
. Then,
2px + 3qx , x ≥ 2
4
(a) f(x) is continuous in R if 3p + 10q = 4 (b) f(x) is differentiable in R if p = q =
13
(c) f(x) is continuous in R if p = -2, q = 1 (d) f(x) is differentiable in R if 2p + 11q = 4
293. Which of the following functions are monotomic increasing for all x ∈ R?
x+3 3
+ 9x 2 + 42x − 5
(a) 2 (b) 2x3 + 3x2 - 36x +7 (c) 2x3- 3x2 + 6x - 1 (d) e2x
x + 5x + 9

294. Let f(x) = 2x − 9 + 2x + 2x + 9


Which of the following are true?
9 −9
(a) f(x) is not differentiable at x = (b) f(x) is not differentiable at x =
2 2
−9 9
(c) f(x) is not differentiable at x = 0 (d) f(x) is differentiable at x = ,0,
2 2
 e[ x ] + x − 4
 , x≠0
295. Let f(x) =  x [ x ] + x
 3 x=0

Where [ ] denotes the greatest integer function. Then,
(a) f(x) is discontinuous at x = 0 (b) f(x) is continuous at x = 0
(c) f(x) is left continuous at x = 0 (d) f(x) is right continuous at x = 0
296. Which of the following functions do not satisfy the conditions of mean value theorem?
p
(a) e −2x cos x in [0, ]
2
p
(b) sin [x] in [-1, 1], where [ ] denotes the greatest integer function
2
 5
(c) (x + 2) (2x - 5)4 in 2, 
 2
1  −p p 
(d) sin in  , 
x  2 2
2.126  Differential Calculus

297. Let f(x) = max (x, x2, x3) in -2 ≤ x ≤ 2. Then


(a) f(x) is continuous in -2 ≤ x ≤ 2 (b) f(x) is not differentiable at x = 1
 3  35  3  −35
(c) f(-1) + f   = (d) f ’(-1) f ’   =
2 8 2 4

Matrix-Match Type Questions


Directions: Match the elements of Column I to elements of Column II. There can be single or multiple matches.
298.
Column I Column II
(a) e - e is increasing at
x -x
(p) 0
−1
(b) e x
is increasing at (q) 1
4
(c) x + is decreasing at (r) -1
x
(d) x3 - 5x2 + 11x - 9 is increasing at (s) e
299.
Column I Column II
3
(x + 27)log(x − 2)
(a) lim = (p) 12
x →3 x2 − 9
 x 
 
 e x − 1   x +1 − e x 
(b) lim  = (q) 8
x →0  x 
x(a + cos x) − bsin x
(c) If lim = 1 then a and b arerespectively (r) 9
x →0 x3
(d) If f(x) is a thrice differentiable function such that (s) e-1
f(4x) − 3f(3x) + 3f(2x) − f(x)
lim = 12, then f ’”(0)
x →0 x3
is equal to
300.
Column I Column II
 1
(a) 3x - log (1 + 3x + 2x2) > 0 for (p)  0, 
 2
5x 2
(b) Let g(x) = log (1 + 3x + 2x2) - 3x + . Then g”(x) is increasing on (q) (0, 1)
2
5
(c) 3x − x 2 < log(1 + 3x + 2x 2 ) < 3x in (r) (0, 2)
2
  2 2  ax
 x − a 2  e , x ≤ 0
(d) Let f(x) =  where, a > 0. the interval in which f ’(x) is increasing (s) (0, 3)
 x 3 − 2x − 2 , x > 0
 a a2
Differential Calculus  2.127

Solutions
a ns w e r K eys

Topic Grip 1 sec2 x


(iii) (iii)
1. (i)
−1
(x log x ) (log (log x )) 5 (2y − 1)
3 2
1 5. (i) e p
(ii) 3125 (iv)
(1 + x )
3
1 2 2 1
(iii) (ii)
2 2
( )
−1
(v) −2 1 − x 2 2 (iii) e5
5
(iv) 1
7 (iv) e 3
−1
( )
−1
49 (vi) 1 − x2 2
(v) -1
(v) 2
81 −1
(vii) -1 (vi)
(vi) 2 2
4 (viii)
(
2 3x 2 − 1 ) (vii) e-10
(vii)
7 ( 1+ x 4
) (viii)
(ix)
e2
ecot a
(viii) 4
− a2 − x2 2log 2
2 (ix) (x)
(ix) x p
3
1  x x −1 −1
6. (ii)
(x)
2
(x) y 
 (1 + log x ) 4a sin 4 t ( 2)
1
(xi) x  x
3 + 
 log(1+ log x)x (1+ log x) 
 (vi) (a)
(1 − x )
3
 2 2

(xii) 1
1
(xiii) 1 (xi) 2sin3 (a + y)cos(a + y)
1 (
2 1 + x2 ) (b)
sin2 a
(xiv) −
6 1 − (n + 1)x + nx n n +1

( )
−1
2 (xii) x 1 − x 4 2
7.
−3 × 2 3 (1 − x)2
(xv)
4 6 9. MN = a
(xiii)
2. (i) continuous at x = 0 (1 + x ) 2 (i) |a| sin2 q
(ii) not continuous at x = 2 (ii) |a sin2 q tan q|
(iii) continuous at x = 1 (xiv) 1
(iii) |a sin2 q cos q|
(iv) continuous at x = 0 12   2x + 3  
(xv) cos log  
a sin 4 q
−9
(v) (9 − 4x 2 )   3 − 2x   (iv)
cos q
5
2sec x tan x 4. (i) y {x x −1
+ (log x ) x (1 + log x )
x
} 11. P0(x) = 1, P1(x) = x,
3. (i)
(1 + sec x ) 1
2

y2 P2(x) = (3x 2 − 1) ,
(ii) 2
(ii)
tan x x (1 − y log x )
1 3
(log cos x )
2
P3(x) = (5x − 3x)
2
2.128  Differential Calculus

−2 −p 120. (a) 121. (b) 122. (b) 1+ x


12. a = c = ,b= 123. (b) 124. (b) 125. (c) 180.
3 9 x(1 − x)3
126. (c) 127. (b) 128. (b)
13. (ii)  0 181. (i) (1 + r)
129. (a) 130. (b) 131. (a)
15. (i)  ex
132. (b) 133. (d) 134. (d) 9
(ii) Maximum value does not (ii) 
exist 135. (d) 136. (c) 137. (b) 8
16. (d) 17. (a) 18. (a) 138. (d) 139. (a) 140. (b)
1 − x − 2 < x ≤ − 1
19. (d) 20. (d) 21. (c) 141. (c) 142. (a) 143. (b) 
22. (c) 23. (d) 24. (d) 144. (a) 145. (b) 146. (d) 2 − x − 1 < x ≤ 0
25. (b) 26. (c) 27. (d) 147. (c) 148. (a) 149. (c) 182. (i)  f(x) = 3 − x 0 < x ≤ 1
28. (b) 29. (a) 30. (a) 150. (b) 151. (c) 152. (a) 4 − x 1 < x ≤ 2
153. (b) 154. (a) 155. (b) 
31. (c) 32. (d) 33. (b) 1 + x 2 < x < 3
34. (a) 35. (a) 36. (b) 156. (b) 157. (b), (c)
(ii)   f(x) is not continuous at
37. (a) 38. (c) 39. (c) 158. (a), (d)
159. (a), (d) x = ±1, 0, 2 in (-2, 3)
40. (b) 41. (b) 42. (d)
43. (b) 44. (c) 45. (a) 160. (a) → (s) e x x < 0
(b) → (r) 
46. (d) 47. (a), (c) 183. (i)  y = 1 x = 0
48. (b), (c), (d) (c) → (q) e2x x > 0
49. (b), (d) (d) → (p) 
(ii)  continuous everywhere
50. (a) → (r), (s)
(b) → (r) − x − 3 − 4 ≤ x < 0
(c) → (q) 
Additional Practice Exercise 184. (i)  f(|x|)= −1 x=0
(d) → (p) x − 3 0 < x ≤ 4

9 9
161. p = , q = -9, r =
2 2 −2x + 1 − 4 ≤ x < 0

IIT Assignment Exercise 1 1 x=0
162.
2 |f(x)|= − x + 3 0 < x ≤ 3
51. (a) 52. (d) 53. (d)
164. q cot q 0 x= 3
54. (b) 55. (b) 56. (c) 
57. (a) 58. (c) 59. (d) 165. not continuous at x = 1  x − 3 3 < x ≤ 4
60. (b) 61. (c) 62. (a) 169. ep > pe
63. (b) 64. (a) 65. (b) 172. Maximum = 14, −3x − 2 − 4 ≤ x < 0
66. (c) 67. (a) 68. (b) 0 x =0
Minimum = -17.25 
69. (b) 70. (d) 71. (b) (ii)  g(x)= 
0 0<x≤3
72. (c) 73. (a) 74. (c)  1 1   −1 1 
173.  , or , 2x − 6 3 < x ≤ 4
75. (a) 76. (c) 77. (c)  2 2   2 2 
78. (a) 79. (a) 80. (b) g(x) is differentiable at all
81. (c) 82. (d) 83. (b) 2
84. (a) 85. (c) 86. (d) 175. 2p points in [-4, 4] except at
3
87. (c) 88. (d) 89. (a) x = 0 and 3
176. -cos a
90. (c) 91. (b) 92. (d) 185. (i)  f(x) is increasing in
( )
n
93. (c) 94. (d) 95. (d) 178. (i) a 2 + b2
2
eax x
96. (a) 97. (b) 98. (a)  −3   1 
 −∞, 2  ,  0, 2  (1, ∞)
99. (a) 100. (a) 101. (a)  b
cos  bx + c + n tan −1 
102. (b) 103. (d) 104. (a)  a  3 
105. (b) 106. (a) 107. (c) f(x) is decreasing in  − , 0 
 2 
(a )
n
2
108. 109. 110. (ii) + b2 eax x
2
(a) (a) (b) 1 
111. (d) 112. (b) 113. (c) and  , 1
 b 2 
114. (a) 115. (c) 116. (a) sin  bx + c + n tan −1 
117. (c) 118. (b) 119. (a)  a (ii)  p ∈ (-13, 0)
Differential Calculus  2.129

186. (i)  f(g(x)) = x4 - 2x3 + x + 1 221. (b) 222. (c) 223. (a) 290. (a), (c), (d)
g(f(x)) = x + 2x + 2x + x - 1
4 3 2 224. (a) 225. (a) 226. (c) 291. (b), (c)
−1 227. (c) 228. (c) 229. (c) 292. (a), (b), (c)
(ii)  x = 0, 230. (c) 231. (d) 232. (a) 293. (c), (d)
3
233. (a) 234. (a) 235. (d) 294. (a), (b), (c)
187. 4 inches/min 295. (a)
236. (a) 237. (a) 238. (b)
188. 48p 239. (c) 240. (a) 241. (a) 296. (b), (d)
189 (i)  3240 cm2/hr 242. (c) 243. (a) 244. (a) 297. (a), (b), (c)

( )
(ii)  540 3 p 245.
248.
(d)
(a)
246.
249.
(d)
(d)
247.
250.
(a)
(b)
298. (a) → (p), (q), (r), (s)
(b) → (q), (r), (s)
(iii)  (720 3 ) p 251. (a) 252. (d) 253. (d)
(c) → (q), (r)
254. (a) 255. (d) 256. (d)
(iv)  (3 3 ) p : 18 : ( 4 3 ) p 257. (a) 258. (c) 259. (a) (d) → (p), (q), (r), (s)
260. (c) 261. (d) 262. (c) 299. (a) → (r)
191. (c) 192. (b) 193. (d) 263. (b) 264. (d) 265. (d)
194. (d) 195. (a) 196. (c) (b) → (s)
266. (d) 267. (d) 268. (b)
197. (a) 198. (d) 199. (c) (c) → (q), (r)
269. (d) 270. (c) 271. (a)
200. (d) 201. (d) 202. (a) (d) → (p)
272. (a) 273. (c) 274. (b)
203. (c) 204. (d) 205. (a)
275. (a) 276. (a) 277. (d) 300. (a) → (p), (q), (r), (s)
206. (a) 207. (c) 208. (d)
209. (b) 210. (a) 211. (c) 278. (b) 279. (b) 280. (c) (b) → (p), (q), (r), (s)
212. (a) 213. (d) 214. (d) 281. (a) 282. (a) 283. (b)
284. (a) 285. (c) 286. (c) (c) → (p), (q), (r), (s)
215. (b) 216. (a) 217. (b)
218. (b) 219. (a) 220. (b) 287. (a) 288. (d) 289. (b) (d) → (p), (q), (r), (s)
2.130  Differential Calculus

H INT S AND EXPLANAT I ON S

Topic Grip
= lim
(
x 1 − cos2 x )
x →0 sin x cos x (1 − cos x )
3− x − 3+ x 3− x + 3+ x  1 + cos x  1
1. (i) lim = lim   =2×1=2
X →0
x 3− x + 3+ x x → 0  cos x   sin x 
 x 
(3 − x ) − (3 + x )
= lim
x →0
x ( 3− x + 3+ x ) (vii) lim 
 tan7x − 3x 

x → 0  7x − sin 2 x 
−2 −1
= lim =  tan7x 3 
x →0
3− x + 3+ x 3  7x − 7 
 x 5 − 55  = lim
(ii) lim  = 5(5)4 = 55 x →0 1  sin x 
x →5  x − 5  1−  .sin x
 7  x 
= 3125 3
1−
1+ x −1 1+ x +1 7 3 4
(iii) lim × = =1− =
x →0 x 1+ x +1 1 7 7
1− ×1× 0
7
(1 + x ) − 1 x 1
= lim = lim =
x →0
x ( 1+ x +1 ) x →0
x ( 1+ x +1) 2 (viii) lim 
 sin2x + sin6x 
x → 0  sin5x − sin3x 

 sin5x   2 sin 4 x cos 2 x 
5x  = lim
sin5x  5x  x →0 
 2 cos 4 x sin x 
(iv) lim = lim
x → 0 tan7x x →0  tan7x  sin 4x 1 1
7x 
 7x  = lim .4. cos2x.
x →0 4x  sin x  cos 4x
5  x 
=
7 =1×4×1×1×1=4
7x (ix) Put 2x = tanq. x → 0 ⇒ q → 0
2sin2
1 − cos7x 2  
(v) lim = lim
x → 0 1 − cos9x x →0 2 9x −1  
2sin tan 2x q
2 \ lim = lim  
x → 0 sin3x q→ 0
 sin  3 tan q  
2   2  

 7x 
sin
 2  × 49 x 2
lim   3  
x →0 7x  4   2 tan q  
  q 2
 2  49 = lim  . . 
= 2
= q→ 0
 sin  3 tan q  tan q 3 
 9x  81   2  
sin
 2  81
lim  × x2
x →0 9x  4 2
  =
 2  3
(x) Sum of n terms of a GP
x tan x xsin x
(vi) lim = lim 1 1
x →0 1 − cos x x → 0 cos x (1 − cos x ) Sn = 1 − n 
2 3 
x sin2 x 1 1 1
= lim \ S∞ = lim 1 − n  =
x → 0 sin x cos x (1 − cos x )
n →∞ 2  3  2
Differential Calculus  2.131

sin(x − 1) sin x − x 1 sin q − q 4  sin q 


3
(xi) limit = lim lim = lim − lim
x →1 (x − 1) (x + 2)
x →0 x3 9 q→ 0 q3 27 q→ 0  q 
sin(x − 1) 1 sin x − x
= lim × lim If we denote lim = k, we have
x →1 (x − 1) x →1 (x + 2) x3 x →0

k 4 1
sin(x − 1) 1 1 1 k = − × 13 , giving k = − .
= lim × = 1× = 9 27 6
(x −1) → 0 (x − 1) 3 3 3
2− 2+x
(xv)
(xii) 2x ( 2
x +1 − x = ) 2x(x 2 + 1 − x 2 )
2
x +1 + x
=
2x
x + x +12
2
1
3
− (4 − x )
1
3

(2 − )
2 + x 2 3 + (4 − x ) 3 + (2 (4 − x )) 3 
2 2 1
2
= , on dividing  
=
1
1+ 1+ 2 − (4 − x )
x2
(2 − )
2 + x 2 3 + (4 − x ) 3 + (2 (4 − x )) 3 
2 2 1
numerator and denominator by x
 
2 =
⇒ as x → ∞ ( x − 2)
1+1
 4 − (2 + x ) 2 3 + (4 − x ) 3 + (2 (4 − x )) 3 
2 2 1
Limit = 1
 
sin x =
x − sin x
1−
x ( x − 2)(2 + 2+x )
(xiii) =
x + cos2 x cos2 x
− 2 + (4 − x ) 3 + (2 (4 − x )) 3 
2 2 1
1+ 3
x  
=
Now, lim
sin x
= 0 and since | cos x | ≤ 1, (
2+ 2+x )
x →∞ x
2

cos2 x −3 × 2 3

lim =0. Limit of the above as x → 2 equals


x →∞ x 4

1− 0 2. (i) f(0) = a2
\ Required limit = =1
1+ 0 sin2 ax
lim f(x) = lim
(xiv) Method 1:
x →0 x →0 x2
x3 x5 sin2 ax 2
We have, sin x = x − + − ..... = lim .a
x →0 a 2 x2
3! 5!
2
x3 x5  sinax  2 2 2
− + − .... = lim 
x → 0  ax 
 .a =1 × a = a
sin x − x 3! 5! 1 x2
\ = = − + + .... f(x) is continuous at x = 0
x3 x 3
6 5!
(ii) f(2) = 2 + 2 = 4
1
→ − as x → 0.
6 lim f ( x ) = lim f (2 − h ) = lim2 − (2 + h ) = 0 ≠ f(2)
x → 2− h→0 h→0

1 \ f(x) is not continuous at x = 2


Limit = − .
6
(iii) f(1) = 5 - 4 = 1
Method 2: lim f ( x ) = lim f (1 + h )
x →1+ h→0
Replacing x by 3q ⇒ as x → 0, 3q → 0
= lim.4 (1 + h ) − 3 (1 + h )
2
sin x − x sin3q − 3q 3sin q − 4sin3 q − 3q h→0
= =
x3 (3q)3 (3q)3 = 1
2.132  Differential Calculus

lim f ( x ) = lim f (1 − h ) 1

x →1 h→0 (ii) y= = (log(cos x))−1
log(cos x)
= lim5 (1 − h ) − 4 = 1
h→0 dy 1
=−(log(cos x))−2 . .(−sin x)
\ lim f (1) = f (1) dx cos x
x →1

\ f(x) is continuous at x = 1 sin x tan x


= =
cos x (log cos x ) (log cos x )
2 2
 1 x − e −1 x 
− 3 e
(iv) f(0 ) = lim− x  1 
x →0  e x + e −1 x  dy 1 1 1
(iii) = . . 5 .5x 4
dx log log x log x x
5 5
( ) ( )
 2 x − 1  0 × −1
3 e
= lim− x  2 = =0 1
 e x + 1  1 =
(x log x ) log (log (x ))
x →0
5

 e 1 x − e −1 x  (iv) y = sin (tan-1x)


+ 3
f(0 ) = lim+ x  1 
x →0  e x + e −1 x  putting tan-1x = t

\ y = sin t
 1 − e −2 x  1
3
= lim+ x   =0× =0 dy dt 1
x →0  1 + e −2 x  1 = cos t, =
dt dx 1 + x 2
We see that f(0-) = f(0+) = f(0) = 0. dy dy dt cos t
\ = . =
f(x) is continuous at x = 0. dx dt dx 1 + x 2
(v) For continuity of f(x) at x = 0, 1 1
= =
lim f(x) = f(0) = l
x →0
2
1+ x 1+ x ( 2
) (1 + x ) 3
2 2

cos3x − 1 0 
lim  0 form   1 
(v) y = sec-1  2
 2x − 1 
x →0 2
5x + 1 − 1

−3sin3x x = cosq
lim , by L’ Hospital’s rule
x →0 1  1 
× 10x y = sec-1 
2 5x 2 + 1  2cos2 q − 1 

 3  sin3x  1 
lim  − 5x 2 + 1  × lim = sec-1 

x →0  5  x →0 x  cos2q 
3  sin3x  3 9 = sec-1 sec2q
= − × lim  × 3 = − ×1× 3 = −
5 x → 0  3x  5 5 = 2q = 2 cos-1x
9 dy −2
⇒l= − \ =
5 dx 1 − x2

2  1+ x + 1− x 
3. (i) y = 1 – (vi) y = sin-1  
sec x + 1  2 

dy +2(sec x tan x) putting x = cosq


=
dx (sec x +1) q q

2 cos + 2 sin
2sec x tan x  2 2
y = sin-1  
= 2
(sec x + 1)  
2
 
Differential Calculus  2.133


 1
= sin-1 
q 1
cos +
q
sin  +
(1 + )( ) (
2x + x 2 2x − 2 − 1 − 2x + x 2 2x + 2 )( )
 2 2 2 2
(1 + x ) 2 2
− 2x 2
p q
= sin −1 sin  + 
4 2 =
(
2 3x 2 − 1 )
1 + x4
p q
= +
4 2 (ix)
dy
dx
d
dx   (
d
= a  log a + a 2 − x 2  − a (log x)
dx
)
p 1
y= + cos −1 x
4 2  −
d
dx
( a2 − x2 )
dy −1 a  1 
\ = = × 0 + × −2x 
dx 2 1 − x 2
(a + a2 − x2 ) 2
 2 a −x
2

(vii) Putting a 1
− − × (−2x)
a = r sinq x 2 a2 − x2

y = r cosq
−ax x a
= + −
 r (sin q cos x − cos q sin x ) 
\ y = tan-1   (
a + a2 − x2 a2 − x2 a2 − x2 x )
 r (cos q cos x + sin q sin x ) 

 sin (q − x )  =
(
−ax + x a + a 2 − x 2 ) −
a
= tan
( a − x )(a + )
-1
 cos q − x  x
( )
2 2 2 2
 a −x

= tan-1 tan (q -x) = q - x


=
x
− =
2
a x −a a+ a −x
2 2
( )
a
\ y = tan-1 − x
b
(a + a2 − x2 ) x
(
x a + a2 − x2 )
dy −(a 2 − x 2 ) − a a 2 − x 2
\ = −1 =
dx
x 2 
(
x a + a2 − x2 )
(viii) y = tan
( )=−
-1
 1 − x2  − a2 − x2 a + a2 − x2
  a2 − x2
=
+ log(1 − x 2 + x 2 ) − log(1 + x 2 + x 2 ) (
x a + a2 − x2 ) x

OR

dy
=
1
.
(1 − x ) 2
2 − x 2 ( −2x )
Let x = a sin q
dx
1+
2x 2 (1 − x ) 2 2
 a + a cos q 
(1 − x ) 2 2 y = a log 
 a sin q 
− a cos q

+
1
1 − x 2 + x2
(
x − 2 + 2x )
 1 + cos q 
= a log 
 sin q 
− a cos q

q 
+
1+ x 2 + x
1
2
. ( 2 + 2x )


= a log 
2cos2
2  − a cos q

q q
 2cos sin 
=
(1 − x ) 2 (1 + x )
2 2 2  2 2
 q
(1 − x ) + 2x (1 − x )
2 2 2 2 2
= a log  cot  − a cos q
 2
2.134  Differential Calculus

Here, y is a function of q where q is a function (xi) We use a proper trigonometric substitution for
of x. x so that y becomes a simple function. This will
Therefore, reduce the algebraic work considerably.
dy Let x = tan q
dy dy dq
= × = dq   (1) 1 + x 2 − 1 sec q − 1 1 − cos q q
dx dq dx dx = = = tan
dq x tan q sin q 2
dy 1  q 1
=a× ×  − cosec2  × + a sin q  q q 1
dq q  2 2 Therefore, y = tan −1  tan  = = tan −1` x
cot  2 2 2
2
q dy 1
sin =
−a 2 + a sin q dx 2(1 + x 2 )
= ×
2 q q (xii) Let x2 = cos 2q
2sin cos
2 2
1 + x2 − 1 − x2 1 + cos2q − 1 − cos2q
a
2
−a(1 − sin q) −a cos q 2 =
= − + a sin q = = 2
1+ x + 1− x 2
1 + cos2q + 1 − cos2q
sin q sin q sin q
dx 2(cos q − sin q) 1 − tan q p 
= a cos q = = = tan  − q 
dq 2(cos q + sin q) 1 + tan q 4 

dy −a cos 2 q  p  p
⇒ = y = tan −1  tan  − q   = − q
dx (sin q) × a cos q   4  4
p 1
a cos q − a 2 − x 2 = − cos −1 x 2
= − = 4 2
a sin q x
dy 1 −1 x
(x) Taking logarithms, =0− × × 2x =
dx 2 1− x 4
1 − x4
log y = xx log (1 + logx)
Differentiating both sides with respect to x, (xiii) Let x = tan q
1 dy 1 1 2x 2tan q
= xx × × = = tan 2q
y dx (1 + log x)  x  1 − x 2 1 − tan2 q

d x 3x − x 3 3tan q − tan2 q
+ [ log(1 + log x)] × (x )   (1) = = tan 3q
dx 1 − 3x 2 1 − 3tan2 q

Let u = xx Therefore,
Taking logarithms, y = tan-1 (tan q) + tan-1 (tan 2 q) + tan-1 (tan 3q)
log u = x logx = q + 2q + 3q = 6q = 6 tan-1x
Differentiating both sides with respect to x, dy 6
⇒ =
1 du 1 dx (1 + x 2 )
= x × + (log x) × 1
u dx x
 3sin x − 3cos x 
du (xiv) y = tan −1 
⇒ = u (1 + logx) = xx (1 + logx)  3sin x + 3cos x 
dx
Substituting in (1)
 tan x − 1   p 
1 dy x x −1 = tan −1   = tan −1  − tan  − x  
= + [log (1 + logx)] xx (1 + logx) 1 + tan x   4 
y dx (1 + log x)

dy   p  p dy
from which is obtained. = tan −1  tan  x − = x− ⇒ =1
dx   4   4 dx
Differential Calculus  2.135

 (iv) Taking logarithms,


 2x + 3  
(xv) y = sin  log  
  3 − 2x   y log x = x - y
⇒ y(1 + log x) = x
dy   2x + 3   (3 − 2x )
= cos log   × x
dx   3 − 2x   (2x + 3) ⇒y=
1 + log x

(3 − 2x )2 − (2x + 3)(−2) dy (1 + log x ) −1 log x log x


× = = =
(3 − 2x )
2
dx (1 + log x )2 (1 + log x )2  x  2
 y 
   2x + 3   12
= cos log    ×
   3 − 2x   9 − 4x
2
( ) =
y 2 log x
x2
(x )x
sin y
4. (i) y = x (v) x =
sin (a + y )
log y = xx (logx)
Differentiating dx sin (a + y ) cos y − sin y cos (a + y )
=
dy sin2 (a + y )
1 dy d
= x x (log x) (x x )
y dx dx sina
=
dy sin (a + y )
2

\
dx
(
= y x x −1 + (log x ) x x (1 + log x ) )
dy sin (a + y )
2

( ) x x −1 + log x x x 1 + log x ⇒ =
= x
xx
( ( ) ( )) dx sina

(ii) Given function can be written as tan


px
 x 2a
y=x y
5. (i) Let L = lim  2 −  (1∞)
x →a  a
⇒ log y = y log x
 px   x
1 dy 1 dy log L = lim  tan  log  2 −  (∞ × 0)
= y. + log x x →a  2a   a
y dx x dx

1  dy y  x
log  2 − 
 y − log x  dx = x  a 0 
= lim  form 
x →a px 0 
dy y2 cot
\ = 2a
dx x (1 − y log x )
1  −1 
×
 x   a 
(iii) y = tan x + y  2 − a 
= lim ,
2 πx  π
x →a 
\ y2 = tan x + y
 −cosec 2a  ×  2a 
dy dy
2y = sec2 x +
dx dx by L’ Hospital’s rule
dy px
(2y - 1) = sec2 x sin2
dx 2 2a = 2 ⇒ L = e 2 p
= lim
dy sec2 x p x →a  x p
\ =  2 − a 
dx 2y − 1
2.136  Differential Calculus

1 1  (v) Let x = - t. Then as x → -∞ , t → ∞.


(ii) lim  − x   ((∞ - ∞) form)
x →0  x e −1  −1 
t 4 sin   + 2t 2
 t 
ex − 1 − x 0  Limit = lim
= lim   0 form 
t →∞ 3 + t3
x → 0 x(e x − 1)
1
2t 2 − t 4 sin
ex − 1 = lim t
= lim , by L’Hospital’s rule t →∞ (3 + t 3 )
x → 0 xe x + (e x − 1)

 2t 2   sin 1   3 
ex t t
= lim   t →∞  1
− lim ×
= lim x , by L’Hospital’s rule t →∞ (3 + t 3 )   3 + t 
3
x → 0 xe + 2e x   
t
1  sin 1   
= t 1 
2 = 0 - lim  × 
t →∞  1   3 3 + 1 
 t  t 
( )
1/x
(iii) Let lim e + 2x 3x
= L  (1 form )

x →0
= -1 × 1 = -1
1 0  ⇒ Limit = -1.
log L = lim log(e3x + 2x)   0 form 
x →0 x
  x +1  p
(vi) x  tan −1  −
1
× (3e3x + 2)   x + 2  4 
3x
(e + 2x)
= lim , 
x →0 1  x +1  −1 
= x  tan −1   − tan 1
  x + 2 
5
= =5
1   x + 1 
 −1  x + 2 − 1  
⇒ L = e5 = x  tan  
 x + 1 
 1+ 
1   x + 2  
 tan x  x2  tan x 
(iv) lim  (1∞ form, since lim  =1)  −1 
x →0  x  x →0  x  = x tan-1  
 2x + 3 
1
 tan x  x2  1 
Let lim  =L − tan −1 
x →0  x   2x + 3   x 
= ×
 1   2x + 3 
 tan x   2x + 3 
log 
 x 
log L = lim 1 −1
x →0 x2 Limit of the above as x → ∞ is = -1 × =
2 2
2x
x  xsec2 x − tan x   2
= lim 2x 1−
  x −2  
x → 0 tan x
 2x 3  (vii) lim   = lim  x 
x →∞  x + 3  x →∞ 3
1 + 
 x
x  x − tan x x tan2 x 
= lim  3
+   2 −x 2 
−4
x → 0 tan x
 2x 2x 3 
 1 −  
  x  
 x − tan x 1  = lim
= 1.  lim 3
+  x →∞
 3  x 3 
6

 x → 0 2x 2
 1 +  
1 − sec2 x 1 −1 1 1   x  
= lim + = + =
x →0 6x 2 2 6 2 3 e −4 1
= =
\ L = e 1/3
e6 e10
Differential Calculus  2.137

  p 
1
x log sin x − log sina
log L = lim
(viii)  tan  + x   x →a x−a
  4 
= limcot x = cota
 1 + tan x  x (1 + tan x ) x
1 1
x →a
=   = ⇒ L = e cota
 1 − tan x  (1 − tan x ) x
1

p
(x) Putting x - = t, we have,
Now, 2
tan x
p
lim (1 + tan x ) = lim (1 + tan x ) tan x 
1 1 x
x as x → ,t→0
x →0 x →0   2

p 
− cos  + t 
= e1 = e 2 2 
−1
⇒ L = lim
Proceeding in a similar manner,
t→0 p 
 2 + t  t

lim (1 − tan x ) x = e −1
1

x →0
 2sin t −1  sin t 1
= lim   .
e t→0
 sin t  t  p + t 
Required limit = = e2  2 
e −1
1
 sin x  (x − a ) 2 2log 2
(ix) lim  = (log 2) × 1 × =
 p p
x →a
 sina 
a x −1
1 (since lim = log a)
 sin x − sina  x −a x→0 x
= lim 1 + 
x →a
 sina 
6. (i) x = cos q; y = cos f
1
 x+a x −a  x −a cos q sin f + sin q cos f = k
 2cos 2 sin 2 
= lim 1 +  sin (q + f) = k
x →a
 sina 
  q + f = sin-1 k
x+a x −a cos-1x + cos-1y = sin-1k
2cos sin
Putting y = 2 2 , −1 1dy
sina − =0
1− x 2
1 − y dx 2

as x → a, y → 0
 y  dy 1 − y2
 x − a  =− — (1)
Hence, limit = lim (1 + y ) y 
1
dx 1 − x2 

y →0  
y Differentiating (1) with respect to x,
Since lim = cota,
y →a (x − a ) d2 y
=
dx 2
Required limit = ecota
 2 1 dy 1(−2x) 
 1− x × × −2y − 1 − y2 × 
Aliter: 2 1− y 2 dx 2 1 − x2
1 − 
 sin x  x −a  2
(1 − x ) 
If L = lim    
x → a  sina 
 
2.138  Differential Calculus

 − y 1 − x 2 dy x 1 − y 2  y
 +  (iv) cos-1 = n  log x − log n 
b
 1 − y 2 dx 1 − x2 
= −  −1 y1 n
 (1 − x 2 )  =
y2 b x
  1−
  b2

 1 − y2  y2
y + x

= − 1 − x2

 (
x 1 − y 2 + y 1 − x2 ) x y1 = bn 1 −
b2

(1 − x 2 ) =− (1 − x 2 )
32
  ⇒ x2 y 12 = b2n2 - n2y2
 
x2 2y1 y2 + y12 2x = -n22y y1
k x2y2 + xy1 + n2y = 0
= − 2 32
(1 − x ) (v) We have
dy dx dy 1
(ii) = a sin t, = a (1 - cos t) =
dt dt dx dx
dy
 dy  t t (x is a function of y)
2sin cos
dy  dt  sin t 2 2 = cot t
= = = Differentiating the above with respect to x,
dx dx(dt )
(1 − cos t)
2sin2
t
2
2
   
d 2 y d  1  d  1   dy 
= = × 
d2 y d  t d  t  dt dx 2 dx  dx  dy  dx   dx 
=  cot  =  cot  ×  dy   dy 
dx 2 dx  2  dt  2  dx
d2 x
 t 1 1 dy 2
= −  cosec2  × × = −
 
2 2 dx 3
dt  dx 
 dy 
−1 1 −1
= × =
t a(1 − cos t) t (vi) (a) y = sin-1x ⇒ x = siny
2sin2 4a sin 4
2 2 dx d2 x
= cos y, = − sin y
(iii) y = sin (m sin-1 x) dy dy 2
dy m Therefore,
= cos(msin −1 x) ×
dx 1 − x2 d2 y (− sin y) sin y x
2
=− 3
= =
dx cos y cos y (1 − x 2 ) 3 2
3
dy
⇒ 1 − x2 = m cos (m sin-1 x)
dx sin y
(b) We have x =
Differentiating with respect to x, sin(a + y)
d2 y 1 dy
1 − x2 + × −2x dx sin(a + y)cos y − sin y cos(a + y)
2
dx 2 1 − x 2 dx =
dy sin2 (a + y)
m ×  − sin(msin −1 x) × m −m2 y sina
= = =
1− x 2
1− x 2 sin2 (a + y)
2
d y dy d2 x 2sina
⇒ (1 − x 2 ) − x + m2 y = 0 =− 3 × cos(a + y)
dx 2 dx dy 2 sin (a + y)
Differential Calculus  2.139

d 2 y 2sina cos(a + y) sin6 (a + y) (1 + x)2 − 1 x (x 2 + 2x)x


giving = × =  = > 0 for x > 0
dx 2 sin3 (a + y) sin3 a (1 + x)3 (1 + x)3
2sin3 (a + y)cos (a + y) f (0) = 0
=
sin2 a Result follows.
7. Consider the series x2
(ii) Let g(x) = 1 - - cos x
S = x + x2 + x3 + … + xn   (1) 2
(1) is a G.P with first term x, common ratio x and g(0) = 0
number of terms n g ‘(x) = -x + sin x ≤ 0 for all x.
n
x(1 − x ) and g(0) = 0
⇒ S =
(1 − x) x2
⇒ 1 - < cos x
Differentiating both sides of (1) with respect 2
to x, Consider the function
dS n
x2 x 4
= 1 + 2x + 3x 2 + ... + nx n −1 = ∑ rx r −1 h (x) = cos x - 1 + −
dx r =1 2 24
Hence, h (0) = 0
x3
n
d  x(1 − x n )  d  x − x n +1  h ‘(x) = - sinx + x -
∑ rx
r =1
r −1
=  = 
dx  (1 − x)  dx  1 − x 
 6
h ‘(0) = 0 and
(1 − x)[1 − (n + 1)x n ] − (x − x n +1 )(−1)
= x2
(1 − x)2 h ‘’(x) = - cosx + 1 - < 0, from (2)
2
(1 − x) − (n + 1)x n (1 − x) + x − x n +1 ⇒ h ‘(x) is a decreasing function.
= p
(1 − x)2 Since h(0) = 0, h(x) < 0 for 0 < x <
n n +1 n +1 2
1 − (n + 1)x + (n + 1)x −x
=
(1 − x)2 dy
dy a × 3sin2 q cos q sin q
1 − (n + 1)x + nx n n +1 9. = dq = =−
= dx dx a × 3cos2 q(− sin q) cos q
(1 − x)2 dq

Equation of the tangent at ‘q’ (i.e., at the point whose
n
coordinate are (acos3q, a sin3q) is
Note that ∑ rx r −1
is an arithmetico-geometric
r =1 sin q
series. Algebraic method is available for sum- y - a sin3q = − (x - a cos3 q)
cos q
ming up the above series (see unit: Sequences and y cos q - a sin3 q cos q = -x sin q + a sin q cos3q
Series)
⇒ x sin q + y cos q
x2 = (a sinq cos q) (cos2q + sin2q)
8. (i) Let f(x) = x - log (1 + x) -
2(1 + x)2 = a sin q cos q   (1)
1 1  (1 + x) × 2x − x × 2(1 + x) 
2 2 To find the points of intersection of the tangent line
f ‘(x) = 1 − −   (1) with the coordinate axes,
1 + x 2  (1 + x)4 
put y = 0
2
x x(1 + x) − x Giving the coordinates of M as (a cosq, 0);
= −
(1 + x) (1 + x)3 Put x = 0 giving the coordinates of N as
x x (0, a sin q)
= −
(1 + x) (1 + x)3 MN2 = a2 cos2 q + a2 sin2 q = a2 ⇒ MN = |a|
2.140  Differential Calculus

(We have the result: Tangent at any point on the curve d2 D c


x = a cos3 q, y = a sin3 q is such that the portion of the Since 2
< 0, x = corresponds to maximum D.
dx 2
tangent intercepted between the axes is a constant.)
1 c
‘Length of tangent’ at the point q is Maximum D = bcsin A, on substituting x = in D.
4 2
y a sin3 q cos q
= 1 + y '2 = 1 + tan2 q 1
y' − sin q = Area of DABC
at q 2
= |a sin q cos q × sec q| = |a sin2 q| = |a| sin2 q.
2 1
= [p2(q + r) + q2 (-r + p) + r2(-p - q)]
4
‘Length of normal’ at the point q is
1
= (p + q) (q + r) (p - r)
= y 1 + y '2 = a sin3 q × sec q = a sin2 q tan q 4
at q

Length of sub tangent at ‘q’ is 1


11. P0(x) = 0
×1 = 1
2 0!
y a sin3 q cos q
= = = a sin2 q cos q 1 d 2
y ' at q − sin q P1(x) = (x − 1) = x
2 × 1 dx
Length of subnormal corresponding to q is 1 d2
P2(x) = (x 2 − 1)2 
22 × 2 dx 2 
4
− sin q a sin q
= |y y’|at q = a sin3 q × =
cos q cos q 1 d2
=  x 4 − 2x 2 + 1
8 dx 2 
10. A
M 1 1
= 12 x 2 − 4  = (3x 2 − 1)
F 90° 8 2
E
1 d3
B P3(x) = (x 2 − 1)3
C 23 × 6 dx 3
D
1 d3
Let the sides BC, CA, AB of the triangle ABC be de- =  x 6 − 3x 4 + 3x 2 − 1
48 dx 3 
noted by a, b, c respectively.
   Let AF = x = DE and AE = FD = y. Triangles 1 1
= 120x 3 − 72x  = [5x3 - 3x]
CDE and CAB are similar. We therefore have 48  2
CE DE b−y x Verification
= ⇒ = ⇒ bc - cy = bx.
CA AB b c P0’(x) = P0’’(x) = 0
   If D represents the area of the parallelogram AFDE, n = 0 → (1 - x2) P0’’(x) - 2x P0’(x) + 0 = 0 - 0 = 0
D = AF × EM where EM is the perpendicular from n = 1 → (1 - x2) P1’’(x) - 2x P1’(x) +2P1(x)
E to AB
= (1 - x2) × 0 - 2x × 1 + 2x = 0
x(bc − bx)
= x × y sin A = sin A n = 2 → (1 - x2) P2’’(x) - 2x P2’(x) + 6P2(x)
c
1
dD  sin A  = (1 - x2) (3) - 2x (3x) + 6 (3x2 - 1) = 0
= [bc − 2bx] 2
dx  c  n = 3 → (1 - x ) P3’’(x) - 2x P3’(x) + 12P3(x)
2

d 2 D  sin A  1 1
= (−2b) < 0 = (1 - x2) × (30x) - 2x × (15x2 - 3)
dx 2  c  2 2
12
dD c + (5x3 - 3x)
=0⇒x= . 2
dx 2 = 15x - 15x3 - 15x3 + 3x + 30x3 - 18x = 0
Differential Calculus  2.141

12. f(x) is continuous in [0, 6] ⇒ In particular, it is con- 14. (i) Given the curve
tinuous at x = 3, 4 (points of subdivision) x5 - x3 + 2x + y - 8 = 0   (1)
At x = 3 we have, We have y’ = - 5x + 3x - 2
4 2

ap
L.H.L. = lim− f(x) = lima cot −1 (x − 3) = At (0, 8), y’ = -2.
x →3 x →3 2
Equation of the tangent at (0, 8) is
 1  cp
R.H.L. = lim+ f(x) = limc tan −1  = y - 8 = -2 (x - 0)
x →3 x →3  x − 3  2
or 2x + y - 8 = 0   (2)
f (3) = 3b
ap cp To find the point of intersection of the tangent
As f is continuous at x = 3 ⇒ = = 3b (2) with the curve (1)
2 2
6b Using (2) in (1) we have x5 - x3 = 0
⇒ a = c =   (1)
p ⇒ x3 (x2 - 1) = 0 ⇒ x = 0, ±1
At x = 4 When x = 0, y = 8, x = 1, y = 6; x = -1, y = 10
1 cp −1    The tangent to (1) at (0, 8) meets the curve
L.H.L = lim− f(x) = limc tan =
x→4 x→4 x−3 4 again at two points (1, 6) and (-1, 10).
R.H.L. = lim+ f(x) = limcos −1 (4 − x) + ap (ii) Slope of the tangent to (1) at (1, 6) = - 4.
x→4 x→4
Also Slope of the tangent to (1) at (-1, 10) = -4
p
=
+ ap = f(4) ⇒ The tangents to (1) at (1, 6) and (-1, 10) are
2
parallel.
f is continuous at x = 4
cp p p 15. (i) Given 2f(xy)
⇒ = + ap = + cp   (2) using (1)
4 2 2 = (f(x))y + (f(y))x for all real x, y   (1)
−2 −p and f(1) = e   (2)
⇒ c= =a \b= (using (1))
3 9 Let y = 1 in (1) we have
−2 −p 2f(x) = f(x) + ex. ⇒ f (x) = ex
⇒a=c= & b=
3 9 ex + e− x ex − e− x
(ii) g(x) = ⇒ g '(x) =
2 2
13. (i) Given f(x + 2)+ f(x + 6) = f(x + 4)   (1)
For extreme values, g’(x) = 0
Replace x by (x + 2) we have
⇒ ex - e-x = 0 ⇒ x = 0
f(x + 4) + f(x + 8) = f(x + 6)   (2)
Now,
(1) + (2) gives, f(x + 2) + f (x + 8) = 0
ex + e− x
⇒ f(x + 2) = - f (x + 8)   (3) g "(x) x = 0 = =1> 0
2 x =0
Replace x by (x + 6) in (3) we have
⇒ x = 0 gives a minimum point and minimum value
f(x + 8) = -f(x + 14)   (4)
of g(x) = 1
From (3) and (4) we have f(x + 2) = f(x + 14) Maximum value of g(x) does not exist.
⇒ f(x) = f(x + 12) ∀ x ∈ R
16. As x → ∞; x= x
⇒ f is periodic with period 12
3x + x 4x
(ii) f ’(x) is periodic with period 12 lim = lim = 2.
x →∞ 7x − 5 x x →∞ 2x

⇒ f ’(x) = f ’(x + 12) ∀ x ∈ R


17. sin x is continuous.
Put x = 13, f ’(13) = f ’(25)
   x, x + 2 being polynomial functions are also con-
2

dy dy tinuous, also x2 + 2 ≠ 0
⇒ − =0
dx x =13
dx x = 25    Hence the function is continuous for all x.
2.142  Differential Calculus

 x −1  x −1 p 2(cos x − 1) 1


18. y = cos −1  + sin −1  = = xlim .
 x + 1   x + 1  2 →0 x2 1 + cos x + 3 − cos x
dy
= 0. x
dx sin2
2 . 1
= − lim 2
19. f ’ (x) = ex (1 – x) + x ex (1 – x) . (1 – 2x)
x →0 x 1 + cos x + 3 − cos x
= ex (1 – x) {1 + x – 2x2} 4
\ f ’ (x) > 0 when 1 + x – 2x2 > 0 
 sin q 
= 1
Qlim q
i.e., 2x2 – x – 1 < 0  
q→0

i.e., (2x + 1) (x – 1) < 0 1 2


= - (1) × =−
 1 2+ 2 4
i.e.,  x +  ( x − 1) < 0
 2 23. y log tan x = x log tan y
 1  1 dy
i.e., x lies in the interval  − , 1 ⇒ y sec2 x + log tan x
 2  tan x dx
f(x) is increasing in this interval. 1 dy
= x sec2 y + log tan y.
20. f(x) = 3 sin2 x + 4 cos2 x tan y dx
= 3 + cos2 x dy log tan y − 2y cosec2x
= .
and 0 ≤ cos x ≤ 1
2
dx log tan x − 2x cosec2y
\ max. 4, min. 3
24. y2 (10 – x) = x3
7 1 7 1
or max f(x) = + = 4 , min f ( x ) = − = 3 Differentiating w.r.t. x
2 2 2 2
-y2 + (10 – x) 2yy1 = 3x2
sin3q [1 − cos3q] At (5, 5), -25 + 5 × 2 × 5y1 = 75
21. lim 3
q→ 0 q cos3q 100
y' = =2
3q 50
sin3q ⋅ 2sin2
2 Equation of the tangent at (5, 5) is
⇒ lim
q→ 0 q3 cos3q y – 5 = 2 (x – 5)
3q 3 3q 2x – y = 5
sin ⋅ sin
sin3q 2 2 . 2 ⋅3 Substituting y = 2x – 5 in the equation of the curve
⇒ lim 2 ⋅ ⋅3⋅
q→ 0 3q 3q 3q 2
(2x – 5)2 (10 – x) = x3
2 2
⇒ x3 – 12x2 + 45x – 50 = 0
3 3 27
= 2⋅3⋅ ⋅ = . (x – 5) is a factor
2 2 2
⇒ dividing x2 – 7x+ 10 = 0
1 + cos x − 3 − cos x ⇒ giving x = 5, 2
22. Given f(x) =
x2 \ Coordinates of Q are (2, -1).
Since f(x) is continuous everywhere, we must have
25. The shortest distance line must be normal to the curve
1 + cos x − 3 − cos x
f(0) = lim as well as perpendicular to the line y = 2x – 2
x →0 x2 y ' = 2x − 2 from the equation of the curve.
(1 + cos x) − (3 − cos x) 1
= lim Slope of normal =
x →0 x2 2 − 2x
1
× 1 −1
1 + cos x + 3 − cos x = ⇒ 2 = −2 + 2x
2 − 2x 2
Differential Calculus  2.143

x=2 29. C
y=4–4+3=3 90°
Point is (2, 3).
A B
26. Applying L’ Hospital’s rule three times
e x + e − x − 2cos x + 6
Limit = lim
x →0 60
Let AC = x, BC = y
1+1− 2 + 6 If A is the area of triangle ABC,
=
60 1
A = xy
1 2
= .
10 We have x2 + y2 = 4r2
1
27. Let lim f(x) = L A = x 4r 2 − x 2
x →0 2
Taking logarithms, 1 2 2 2 x4
A2 = x (4r - x ) = x2r2 -
log L = lim log f(x) 4 4
x →0
dA 2
  1 x  = 0 ⇒ 2r2x - x3 = 0 ⇒ x2 = 2 r2
x x dx
   + 16 + 2 
1  4   ⇒ y2 = 2 r2
lim log f ( x ) = 4 lim. .log  
x →0 x →0 x 3
  d2 A2
  and = 2r2 - 3x2 < 0 when x2 = 2r2
dx 2
0  \ A is maximum when DABC is isosceles.
 0 form 
 1 + x2  −1  1 + x 
2
30. Given y = cosec −1  + sec
   2x   1 − x 2 
  Let x = tan t then y = 4t
3
= 4 lim   ×
x →0  x 
1 dy dy / dt 4 4
   + 16 x + 2 x  ⇒ = = =
4 dx dx / dt sec t 1 + x 2
2

dy
 1  1 x 1   ≠ 0 for any real x

x x
   . log   + 16 log16 + 2 log 2  dx
 3   4  4   ⇒ At no point, the tangent is parallel to x-axis.

4  1  31. Statement 2 is false
= .  log   + log16 + log 2
3  4  For example, consider the problem lim (sec x − tan x )
p
x→
2
4
=
3
[ − log 4 + 4 log 2 + log 2] = lim
1 − sin x
= lim
− cos x
=0
cos x x → − sin x
p p
x→
2 2
= 4 log 2 = log 16.
However, lim sec x and lim tanx do not exist.
p p
Therefore, L = 16 x→
2
x→
2

28. log y = log cos x + log cos 2x + log cos 3x Consider Statement 1,
cos x − e x
dy 1 (− sin x ) −2 sin2x −3sin3x  lim = lim =0
⇒y  + +  x →0 x →0 1
dx  cos x cos2x cos3x  ⇒ True
= -y (tan x + 2 tan 2x + 3 tan 3x). Choice (c)
2.144  Differential Calculus

32. Statement 2 is true ⇒ a and b have to be opposite signs.


1 − 5 + 3 , 1< x < 2 ⇒ Statement 1 follows from Statement 2
−3 , x=2 Choice (a)

f(x) = 4 − 10 + 3 , 2<x<3 36. Statement (2) is true, (1) is true.
−3 , x=3
 (Statement of Rolle’s theorem)
9 − 15 + 3 , 3< x<4 Consider Statement 1:
 −1 , 1 < x < 2 f(x) is continuous in [0, 6]
=
−3 , 2 ≤ x ≤ 4 But, f(x) is not differentiable at x = 3
f(x) is continuous at x = 3 only Also, f(0) = f(6) = 4
⇒ Statement 1 is false Rolle’s theorem cannot be applied.
Choice (d) However, f ’(2) = 0

33. Statement 2 is true ⇒ Choice (b)


Consider Statement 1 37. Statement 2 is true
Since |cosx| ≤ 1 Using Statement 2,
cos x 1   1 
≤ f(x) = cos  log e  
x |x|   x2 + 1 − x  

As x → ∞,
1
|x|
→0 ( ( x + 1 − x))
> cos − log 2

cos x = cos (log ( x + 1 − x ))


2
⇒ lim =0
x →∞ x
= f(-x)
Statement 1 is true
⇒ Statement 1 is true
Choice (b)
Choice (a)
34. Statement 2 is true
(It is Rolle’s theorem) 38. Statement 2 is false
The correct statement is
f(x) being a polynomial satisfies the conditions of
Rolle’s theorem and f(-3) = f(1) = f(4) = f(7) = 0 If f(x) is such that f(a) = 0 and f ’(a) = 0, then x = a is
a repeated root of f(x) = 0
⇒ Using Statement 2, we infer that
x3 - 3x2 + 4 = 0
f '(x) vanishes in (-3, 1), (1, 4) and (4, 7)
⇒ Statement 1 is true Putting x = 2 ⇒ the equation is satisfied.

⇒ Choice (a) Dividing (x3 - 3x2 + 4) by (x - 2), we get


(x2 - x - 2)
35. f ’(x) = 2 × 3x2 = 6x2
Clearly, x = 2 is satisfying x2 - x - 2 = 0
> 0 for all x
⇒ Statement 1 is true
⇒ Statement 2 is true
Choice (c)
⇒ Slope of the curve y = 2x3 at any point must be
positive. 39. Statement 2 is false
−a Differentiability is a necessary condition for the func-
Slope of ax + by + c = 0 is tion to have an extremum at that point.
b
a Consider Statement 1
Hence, must be negative
b f(x) is continuous at x = 3
Differential Calculus  2.145

−10x 1≤ x < 3
f ’(x) = 
 3 3<x≤5
(1 + cot q 2 )
3

(1 + y ' )
3 2 2
2 2

f ’(x) does not exist at x = 3 r= = = 4a sin q .


y"   2
However, f ’(x) changes sign from negative to positive −1
 
as x crosses 3.  4a sin 4 q 
2
⇒ f(x) is minimum at x = 3
p p
⇒ Statement 2 is true r at q = , is given by 4a × sin
2 4
Choice (c)

40. Statement 2 is true


=
4a
2
( )
= 2 2 a

Let x be a non integer


a 2 (a − x)
44. y 2 =
(2 + [x])(−4e ) − e (0)−4 x −4 x
x
Then, f ’(x) =
(2 + [x])
2 Differentiating with respect to x,
 x(−1) − (a − x)  −a 3
d 2y y’ = a 2  = 2   (1)
Since [x]= 0  x2  x
dx
−4e −4 x0 Differentiating (1) with respect to x,
⇒ f ’(x0) =
2a 3
2 + [x 0 ]
{
2 y y " + y '2 =} x3
  (2)
⇒ Statement 1 is true
Choice (b) a  −a 3 × 4
y’ at  ,a  is = 2 = −2
2  a × 2a
41. y = 3x2 + 7x
y’ = 6x + 7, y” = 6. Substituting for y’ in (2),
(y’)at (1, 10) = 13 2a 3 × 8
2[a y” + 4] = = 16
a3
(y”)at (1, 10) = 6.
4
(1 + 132 )
3
2
170 2
3
2a y” = 8, y” =
r= = . a
6 6 3
 a  (1 + 4 )
3
2
a ×5 2
42. x = 2t, y = t2 – 1 r at  ,a  = = .
2  4 4
2t dt 1  a 
y’ = = t ⇒ y” = 1 × =
2 dx 2
45. x = a cos4 q; y = b sin4q
(1 + t )
3
2 2

( )
3
r at t = = 2 1 + t2 2
. dy 64 sin3 q cos q b
1 y’ = = 4 = - tan2 q
dx a cos3 q (− sin q) a
 2 
b dq
43. x = a(q – sin q), y = a(1 - cos q) y” = - 2tan q sec2 q
a dx
a sin q 2 sin q cos q −2b sin q 1 1
y' = = 2 2 = cot q =
a(1 − cos q) 2sin 2 q 2 a cos q cos q 4a cos q (− sin q)
2 3

2
b 1
(
y " = − cosec q 21 dq
× ×
2 2 dx ) =
2a cos6 q
2

3
−1 1 −1  b2  2 2a 2 cos6 q
= × = ρ = 1 + 2 tan 4 θ  ×
2 q a(1 − cos q) 4a sin 4 q
2sin  a  b
2 2
2.146  Differential Calculus

3
 2 4 2 4 2 6
 2 2a cos q p2
= a cos q + b sin q x 48. Given f(x) = tan
16
− x2
 a 2 cos 4 q  b
3 p2
2 i.e., − x2 ≥ 0
= (ax + by) 2 16
ab
p2
46. From problem no. 44, we have \ x2 ≤
16
r at q is given by 4a sin q and r at q + p is given by
2  −p p 
\ x ∈  , 
p+q  4 4
4a sin  = 4a cos q
 2  2
f(-x) = f(x) ⇒ f(x) is an even function
i.e., r1 = 4a sin q , r2 = 4a cos q  p p
2 2 f −  = f   = 0
r12 + r22 = 16a2  4 4

f(0) = 1
47. Given f(x2 + 1) = 2x4 - 3x2 + 1
Take y = x2 + 1 ⇒ x2 = y - 1 Range of f(x) is [0, 1]
\ f(y) = 2(y - 1)2 - 3(y - 1) + 1 = 2y2 - 7y + 6 ⇒ Range of f(x) is A’
f(x) = 2x2 - 7x + 6 Minimum value of (f(x))-1
f ’(x) = 4x - 7 1
= Minimum value of
f(x)
f ’(0) = -7
f (0) = 6 = Maximum value of f(x) = 1

\ Slope of the line joining (0, 6) and (1, -1)  1 1  1


2

−1 − 6 49. Given f  x +  = x 2 + 2 =  x +  − 2
= = −7 = f ’(0)  x x  x
1− 0
⇒ (a) is true \ f(x) = x2 - 2
2
f(1) = 1 and f ’(1) = -3  1 1  1
g  x −  = x2 + 2 =  x −  + 2
\ slope of line joining (3, 3) and (1, 1)  x  x  x
3 −1 \ g(x) = x2 + 2
= = 1 ≠ f ’(1)
3 −1 f(1) = -1, g(1) = 3 ⇒ f(1) ≠ g(1)
\ (b) is not true
f ’(x) = 2x, g’(x) = 2x
(c) f(2) = 0
f ’(1) = 2 = g’(1)
f '(2) = 8 - 7 = 1
(fog) (x) = (x2 + 2)2 - 2 = x4 + 4x2 + 2
Slope of the line joining (2, 0) and (0, -2)
(gof) (x) = (x2 - 2)2 + 2 = x4 - 4x2 + 6
−2 − 0
= = 1 = f ’(2) (fog)’(x) = 4x3 + 8x
0−2
\ (c) is true (gof)’(x) = 4x3 - 8x
f(3) = 3 (fog)’(1) = 12
f '(3) = 12 - 7 = 5 (gof)’ (1) = -4
Slope of the line joining (2, 1) and (3, 3) is f '(g’(x) = 2(2x) = 4x
3 −1 (f ’og’) (1) = 4
= 2 ≠ f ’(3)
3−2 (gof)’ (-1) = -4 + 8 = 4
\ (d) is not true. f 'og’(-1) = -4 = (gof)’ (1)
Differential Calculus  2.147

50. (a) 9x2 + 16y2 – 54x – 128y + 193 = 0 3


Now at 4sinq - 3cosq = 0 ⇒ tanq =
( x − 3) + ( y − 4 )
2 2
4
⇒  = 1 is an ellipse with major
16 9 5
and secq =
and minor axes parallel to x and y axes. Slope of 4
vertical tangents are not defined. So the eccentric x = 0, y = 4 \ point is (0, 4)
angle of the point are 0 and p.
dy 4sec2 q 1 5
Parametric-form of representation of any point = = =
dx 4sec q tan q sin q 3
on the given ellipse is
5
x = 3 + 4cosq and y = 4 + 3sinq \ y - 4 =
3
(x − 0) ⇒ 3y - 12 = 5x
At q = 0, x = 7 and y = 4
\ 5x - 3y = -12   (2)
q = p, x = -1 and y = 4
(1) × (3) ⇒ 12x + 3y = 63
\ Points are (7, 4) and (-1, 4)
____________________________
OR
−9 ( x − 3) 17x = 51
Slope of tangent y’ =
16 ( y − 4 ) ⇒ x = 3 and y = 9
\ point of intersection (3, 9)
For vertical tangents, y - 4 = 0 \ y = 4
(c) → (q)
\ (x - 3)2 = 16 ⇒ x = 3 ± 4 = 7 or -1
\ points are (7, 4) and (-1, 4) cos x ; 2 < x < 3

(d) f ’(x) =  −1
(a) → r, s ;3< x <9
 x 2
(b) y = 2x3 - 9x2 - 24x + 30
For x ∈ (3, 9), f ’(x) is negative and f(x) is
y’ = 6x2 - 18x - 24 = 6(x2 - 3x - 4)
decreasing
= 6(x + 1)(x - 4)
Now 1.57 < 2 < x < 3 < 3.14
dy
For x ∈ (-1, 4), <0 p
dx ⇒ <x<p
2
\ the function is decreasing in (-1, 4)
p 
(c) In (2, 3) ⊂  , p  , f ’(x) is negative
2 
5 \ f(x) is decreasing in (2, 3) also
3
θ (d) → (p)
4
(x +1)
2
y2
9x + y
2 2
+ 18x = 216 ⇒ + =1 IIT Assignment Exercise
25 225
\ x = 5cosq - 1 y = 15sinq x 8 − 6561 x −3 8 × 37
51. lim × 4 = = 162
( 3) x−3 x − 81 4 × 33
x→3
q = cot −1 4
 xn − an n −1 
4  x → a x − a = na 
lim
⇒ sinq = 3 , cos q =
5 5
\ point is (3, 9) x2 − 1
dy 15cos q 4 52. lim+ = lim x +1 = 2
= = − 3. = -4 x →1 x −1 x →1

dx −5sin q 3
\ Equation of the tangent y - 9 = -4(x - 3) x2 − 1 x2 − 1
lim− = lim− = - 2.
y - 9 = -4x + 12 ⇒ 4x + y = 21  (1)
x →1 x −1 x →1 (1 − x)
x = 4secq - 5, y = 4tanq + 1 \ Limit does not exist.
2.148  Differential Calculus

 1+ x − 1− x  59. 4 + 5v2 = x2
53. Limit = lim   Differentiating with respect to t
x →0
 x 
dv 2xdx dv x
1 1 10v = ⇒ =
+ dt dt dt 5
= lim 2 1 + x 2 1− x ,
dy 3a sin2 q cos q
x →0 1 60. = = – tan q
by L’ Hospital’s rule = 1. dx −3 a cos 2 q sin q

kq  kq 
2
d2 y − sec2 q sec 4 q
sin 2
sin = =
2 ⇒ lim  2  dx 2 −3a cos 2 q sin q 3 a sin q
54. Limit = lim 
q→ 0 R q q → 0 R q
sin2  sin  3
2  2    dy  2  2

1 +   
k2 R2 k2   dx   3 a sec3 q sin q
⇒ ÷ = =
4 4 R2 d2 y sec 4 q
a x + log(1 + x) − sin x − cos2x dx 2
55. lim = 3a sinq cosq.
x→0 ex + 1
1+ 0 − 0 −1 dy
= =0 61. 2y = 4a
1+1 dx
56. x + y = y log x ⇒  x = y [log x – 1] d 2 y  dy 
2

y +  =0
x dx 2  dx 
=y

( x − 1)
log 2
d2 y  4a  1 −4a 2
1 = − .
 2y  y = .
log x − 1 − x dx 2 y3
⇒ x = dy
(log x − 1)
2
dx
62. f ’ (x) = 3 cos x + 4 sin x – k
dy log x − 2 f ’ (x) < 0 for all x
⇒ =
dx (log x − 1)2 \ 3 cos x + 4 sin x – k ≤ 0
This is possible when k ≥ the maximum value of
dx 3 cos x + 4 sin x
57. = −3a cos2 q sin q
dq i.e., k ≥ 5.
dy
= 3a sin2 q cos q 63. f (x) = ex
dq
f’ (x) = ex > 0
dy
= − tan q for all real values of x
dx
2 \ f (x) is always increasing.
 dy 
1 +   = sec q .
 dx  64. f ’ (x) = 6x2 – 18ax + 12a2
1 1
when f ’ (x) = 0, x2 – 3ax + 2a2 = 0
58. y = tan −1 x 3 − tan −1 a 3 i.e., (x – 2a) (x – a) = 0
dy 1 1 −2 i.e., x = 2a, a
∴ = ⋅ x 3
2
dx 1 + x 3 3 f ” (x) = 12x – 18a
f ” (x) > 0 at x = 2a \ q = 2a
1
= .
3x
2
3
(1 + x ) 2
3
f ” (a) < 0, at x = a
Now, p = q a = 2a
2 2
\ p = a
\ a = 2.
Differential Calculus  2.149

65. P (2, –1) is a point on the curve


cos x = cos x is differentiable at x = 0
2p + q
−1 = x is not differentiable at x = 0
(2 − 4 ) (2 − 1)
2p + q = 2  — (1) ∴ cos x ± x is not differentiable at x = 0

dy ( x − 4 ) ( x − 1) (p) − (px + q ) (2x − 5) \ sin |x| -|x| is differentiable at x = 0


=
dx ( x − 4 ) ( x − 1)
2 2 OR

− sin x − x, x < 0
dy (a) f(x) = 
= 0 at x = 2 ⇒ \ -2p + 2p + q = 0 sin x + x , x ≥ 0
dx
\ q = 0 and from (1), p = 1. − sin x + x, x < 0
(b) f(x) = 
dy sin x − x , x ≥ 0
66. y2 = 4ax ⇒ 2y = 4a
dx cos x − x, x < 0
(c) f(x) = 
\
dy 2a
= cos x + x , x ≥ 0
dx y
cos x + x, x < 0
Ratio of subtangent to ordinate (d) f(x) = 
cos x − x , x ≥ 0
y y
= = All the functions in (a), (b), (c), (d) are continuous
dy 2a
.y .y at x = 0.
dx y
   Let us check the differentiability of these func-
y 2 4ax tions at x = 0
= = = 2x: y
2ay 2ay
( )
In the case of (a), f '(0− ) = −2,f ' 0+ = 2
f (1) − f (0) In the case of (b), f '(0 ) = − f ' (0 ) = 0
− +
67. f ’ (c) =
1− 0
f ’ (x) = 2px + 2qx + r In the case of (c), f '(0 ) = −1, f ' (0 ) = 1
− +

2pc + 2qc + r = p + q + r In the case of (d), f '(0 ) = 1, f ' (0 ) = −1


− +

p+q 1
c= = . Only, the function in (b) is differentiable at x = 0
2 (p + q ) 2
1
(For a quadratic, c is mid-point of the interval) 69. Percentage decrease in volume = %
2
68. x = x, x > 0 and = –x, x < 0 dV 1
× 100 = −
∴ sin x + x V 2
1
= sin x + x, x 0 and = – sin x – x, x < 0
PV 4 = C
R H Derivative = 2 at x = 0
1
L H Derivative = -2 at x = 0 log P + log V = log C
4
∴ sin x + x is not differentiable at x = 0
dP 1 dV
+ =0
sin x − x = sin x − x, x ≥ 0 P 4 V
= -sin x + x, x < 0 dP −1  dV 
⇒ × 100 =  × 100 
R H Derivative = 0 at x = 0 P 4  V 
L H Derivative = 0 at x = 0 −1 −1 1
= × = %
∴ sin x − x is differentiable at x = 0 4 2 8
2.150  Differential Calculus

1 p
73. (cos x )
cos x
70. LH limit = Lim− f(x) = Lim− 1
(= L say) as x → is of the form 00
x →0 x →0 2
5+3 x
Let limit = L
1
As x → 0 , → - ∞.-
L = lim (cos x )
cos x
x
x→p
1 1 2

so that 3 x → 0 and 5 + 3 x → 5 log L = lim cos x log (cos x )


p
x→
1 2
\ LH limit =
5 log (cos x )  ∞ 
1 = lim
RH limit = Lim+ f(x) = Lim+ 1x
(=R say) x→
p sec x  ∞ 
x→0 x→0 5+3 2

1 − tan x
As x → 0+, → ∞ = lim =0
x x→
p sec x tan x
2
1 1
1
so that 3 x → ∞ and 5 + 3 x → ∞ \ R = =0 L=1

As L ≠ R. we say that the Lim f(x) does not exist. g1 ( x ) f (a ) − g (a ) f 1 ( x )
x→0
74. Limit = lim
x →a 1
 e5x − 1   e −3x − 1  By applying L’ Hospital’s rule,
5x
e −e −3x  x  −  x 
71. lim = lim = g1 (a) f (a) – g (a) × f1 (a)
x → 0 tan x + sin x x→0 tan x sin x
+ = 2 × 2 + -1 × 1 = 5.
x x
[\ by dividing numerator and denominator by x] e tan x − e x e x e tan x − x − 1
75. Limit = lim = lim
=
( )
log e5 − log e −3 ( )= 4, x →0 tan x − x x → 0 tan x − x
1+1  e tan x − x − 1  x
= lim  .e
x → 0  tan x − x  
eax − 1
Q lim
x→0 x
( )
= log ea = a,
e tan x − x − 1
= lim e x lim
tan x sin x x →0 x →0 tan x − x
lim = 1 = lim
x→0 x x→0 x
= e0 × 1 = 1.
1
1x + 2 x + ... + n x 
( )
x

72. y =  x 2 + ax + b + x
76. lim ( x + ax + b − x ) ×

 n  2

⇒ log y =
1 1x + 2 x + ... + n x 
log 
x →∞
( x + ax + b + x )
2


x  n  x 2 + ax + b − x 2
= lim
lim log y = lim
x →0 x →0
1
x
1 + 2 + ... + n   0
log 
n
x x

  form 
 0 
x x →∞
( x 2 + ax + b + x )
 
 b
 n  x a + 
= lim  x
x → 0 1 + 2 x + ... + n x 
 x
  = lim
x →∞  a b 
1 x x  1 + + 2 + 1
× 1 log1 + 2 x log 2 + ... + n x log n   x x 
n
1 a+0 a
( )
. log (n!) = log (n!) n  ⇒ lim y = (n!) n .
1 1
⇒ = .
n   x → 0 1+ 0 + 0 +1 2
Differential Calculus  2.151

 1  80. Since f(x) is continuous at x = 0 we must have


1 1
77. nlim  + + ... +  = lim S 3sin x − 2x
→∞ 1 ⋅ 3
 3⋅5 (2n − 1) (2n + 1)  n→∞ n f(0) = lim f(x) = lim
x→0 x → 0 tan x + 4x

1 1 1 1  3sin x
Tn = = − −2
(2n − 1) (2n + 1) 2  2n − 1 2n + 1 

= lim x (dividing numerator and denomi-
x → 0 tan x
Put n = 1, 2, 3, … +4
x
1  1 1 1 1
T1 = 1 −  ; T2 =  −  nator by x)
2  3  2 3 5
3−2 1
1 1 1 1  1 1  = =
1+ 4 5
T3 =
2  5 − 7  ; … Tn = 2  2n − 1 − 2n + 1 
1
1  1   2x  3

Adding we get Sn = 3 1 −  −3
2 1 − 2n + 1   27 
  81. f (x) = 1
1  5x  5
lim Sn = 9 − 9 1 +
n →∞ 2  243 
78. As polynomial functions are continuous everywhere –2
+ terms containing x and higher power of x
 3x + 1 , x ≤ 1  1  81
f(x) =  is continuous everywhere except  3  –1
2
2 − ax , x > 1 243
+ terms containing x and higher power of x
possibly at x = 1 ⇒ f(0) = lim f(x) = 2.
For f(x) to be continuous at x= 1 also we must have x →0

f(1) = lim+ f(x) = lim− f(x) 82. The function sin|x| is defined for all x.
x →1 x →1
1
⇒ lim 2 − ax 2 = lim 3x + 1 ⇒ 2 - a = 4 For f(x) = to be defined we must have sin
x →1 x →1 sin | x |
|x| ≠ 0
⇒ a = -2.
⇒ x ≠ np, n ∈Z.
 sin(4k − 1)x ⇒ x = np, n∈Z are the points of discontinuity of f(x)
 , x≤0
3x which are infinite in number.
79. f(x) = 
 tan(4k + 1)x , 0 < x < p
 5x 2 83. 3x2 + 3y2 y1 = 6 [xy1 + y]
 p x2 + y2 y1 = 2xy1 + 2y
f(x) is continuous everywhere in  −∞ ,  except
 2 y1 (y2 – 2x) = 2y – x2
possibly at x = 0
2y − x 2
For f(x) to be continuous at x = 0, we must have y' =
y 2 − 2x
f(0) = lim− f(x) = lim+ f(x)
x→0 x→0 y = 0 ⇒ 2y = x2
1

sin(4k − 1)x Substituting in the equation of the curve,


lim f(x) = lim
x → 0− x→0 3x  x2 
3
 x2  x6
x 3 +   = 6x   ⇒ x 3 + = 3x 3
sin(4k − 1)x (4k − 1)x 4k − 1 2 2 8
= lim × =
x→0 (4k − 1)x 3x 3
x6
 sin q  = 2x 3
= 1 8
Qlim q
 q→0
 4
x 3 = 16, x = 2 3

tan(4k + 1)x (4k + 1)x 4k + 1 8


Also lim+ f(x) = lim × = x2 2 3 5
x→0 x→0 (4k + 1)x 5x 5 y= = =2 3
2 2

( ).
4k − 1 4k + 1 4 5
Equating, = ⇒ k =1 Point is 2 3 ,2 3
3 5
2.152  Differential Calculus

84. Parametric form of the equation of the curve is sin3 q p


4 4 Slope of normal = and corresponding to q = ,
x = 6 (cos a ) 3 , y = 4 (sin a ) 3 3
cos q 3
2 3 38
dy −2  cos a  3 Slope of normal = =3 3
= 8 1
dx 3  sin a 
p 3 1 11
Equation of the tangent at “α” on the curve is When q = , x = − =
3 2 8 8,

(x − 6(cos a) )
2
4 −2  cos a  3 4 3 3 3 3 1 1 9 3
y − 4 (sin a ) 3
= 3
y= − =3 3 −  =
3  sin a  2 8 2 8 8

Simplification leads to ⇒ 8x = 11 – 3 = 8, x = 1
2 2  11 
2x (cos a ) 3 + 3y (sin a ) 3 = 12   (1) G is (1, 0) and N is  , 0 
8 
But the equation of the tangent is given as 2
 11  9
x cos q + y sin q = p   (2) NG2 =  − 1 = .
8  64

We obtain from (1) and (2)
cos q sin q p 87. x2 – 5x + 4 ≤ 0 ⇒ 1 ≤ x ≤ 4
2
= 2
= f(x) = 2x3 – 25x2 + 100x + 14
2 (cos a ) 3 3 (sin a ) 3 12
f1 (x) = 6x2 – 50x + 100
⇒ (6 cos q)3 + (4 sin q)3 = p3. f11 (x) = 12x – 50
85. The parametric equation of the curve is x = 5 cos3 q, 10
f1 (x) = 0 gives x = 5,
y = 5 sin3 q 3
dy 5 × 3sin2 q cos q − sin q f(x) is a minimum at x = 5 and maximum at
= = 10
dx −5 × 3cos2 q sin q cos q x=
3
Equation of the tangent at q is
Since 1 ≤ x ≤ 4, minimum value of f(x)
− sin q
y − 5sin3 q =
cos q
x − 5cos3 q( ) i.e., either f(1) or f(4). But f(1) = 91 < f(4) = 142
y cos q - 5 sin3 q cos q = -x sin q + 5 sin q cos3 q x 1 + x 2 − 2x 2 1 − x2
88. y = ⇒ y' = =
x sin q + y cos q = 5 sin q cos q   (1)
( ) ( )
2 2 2
1+ x 1 + x2 1 + x2
Equation of the normal at q is
We have to find the maximum value of y1
cos q
3
y − 5sin q =
sin q
(
x − 5cos3 q ) 1 − x2
Let S =
y sin q - x cos q = -5 cos4 q + 5 sin4 q (1 + x )
2 2

= -5 (cos2 q - sin2 q)
1 + x2 > 1
or x cos q - y sin q = 5 (cos2 q - sin2 q)   (2)
\ S is maximum when 1 - x2 is maximum i.e.,
4l2 + m2 = 4 (5 sin q cos q)2 at x = 0
+ 25 (cos2 q - sin2 q)2 = 25 {(cos2 q - sin2 q)2
89. If a and b are the roots of the quadratic equation
+ 4 sin2 q cos2 q} = 25.
S = a2 + b2 = [- (3 + a)]2 – 4 (a3 – 2a + 5)
dy 3cos q − 3sin2 q cos q = - 4a3 + 14a + a2 – 11
86. =
dx −3sin q + 3cos2 q sin q dS
= −12a 2 + 14 + 2a
da
=
(
3cos q 1 − sin2 q ) =
cos3 q
d2S
3sin q (cos 2
q − 1) − sin3 q = −24a + 2
da 2
Differential Calculus  2.153

dS 1
= 0gives − 12a 2 + 14 + 2a = 0or 1   x n  n
da (
93. y n + x )
n n
= y 1 +    as
6a2 – a – 7 = 0   y  
n
(6a – 7) (a + 1) = 0 x  1
< 1 and lim 1 +  = e
2
y n →∞  n
7 7 d S
a= or − 1 for a = , 2 < 0 .
6 6 da x 1
n

 y  × n
 
n
y
 x 
90. We have –8a + 4b – 2c + 5 = 0    x  n  
= y  1 +    
y1 = 3ax2 + 2bx + c    y   
When x = 0, y1 = 3 ⇒ c = 3  

Also 12a – 4b + c = 0 (as it touches x-axis)  x  1 1 x


n

→ y × eo as n → ∞    × < , since <1


i.e., 12a − 4b + 3 = 0 or we have  y  n n y

−8a + 4b = 1  x n 1  
Hence, as n → ∞ , lim    ×  = 0  = y.
12a − 4b = −3   y  n  
4a = −2 5sin[cos x] 5sin0
94. lim− = =0
−1 x→
p [cos x] + 2 2
a= . 2
2 [Since x is in the first quadrant where, 0<cosx<1,
91. f1 (x) = –2 sin x – 1 [cosx] = 0]
5sin[cos x] 5sin(−1)
1 lim+ = = −5sin1
f ‘(x) > 0 ⇒ sinx < − p [cos x] + 2 −1 + 2
2 x→
2
Since x is in the second quadrant, cos x is negative,
 −5p − p   7 p 11p 
⇒ x ∈  , ∪ , [cos p] = -1
 6 6   6 6 
lim− (f(x)) ≠ lim+ (f(x))
or f(x) is monotonic increasing in x→
p
x→
p
2 2

 −5p − p   7 p 11p  \ Limit does not exist.


 6 , 6  ∪  6 , 6  .
2x − 1 0 
95. Limit = lim  0 form 
x →0
1+ x −1
92. y = (
2 + x)
2

2 x . log 2 log 2
x = lim = = 2 log e 2
x →0 1 1
2x (2 + x ) − (2 + x )
2 2
x −4 2 1+ x 2
y' = =
x2 x2
96.
− x12
Slope of normal at a point ( x1 , y 1 ) is O
(x 1
2
−4 ) 6

Now, a line has numerically equal intercepts on the


axes iff its slope = ±1
Let h be the height and r be the radius of the cylinder
x12
\ = ±1 h
2

x12 − 4 we have   + r 2 = 36
2
⇒ x12 = 2
h2
r 2 = 36 −
⇒ x1 = ± 2 4
2.154  Differential Calculus

 h2   h3  sin x
Volume v = pr 2 h = ph 36 −  = p 36h −  99. Let y = x + . Then,
4 4 sin x
  x+
x + ...
dv 144
= 0 gives h = = 4 3 and this corresponds sin x
dh 3 y = x+
y
d2 v
to <0. ⇒ y2 - yx - sinx = 0
dh 2
x ± x 2 + 4sin x
m 2 ⇒ y = ∴ lim y = 0
97. We have lx + − n ≥ 0 for x > 0 or lx 3 + m − nx ≥ 0 2 x →0
x
Let f ( x ) = lx 3 − nx + m   x +1  p
100. L = xlim x sin −1  − [ 0 × ∞ form]
 2x + 1  6 
f ' ( x ) = 3lx − n 
2 →∞


1
n Let x = . As x → ∞, y → 0 and we have
f '' ( x ) = 6lx, ,f ' ( x ) = 0 gives x = ± But x > 0 y
3l
n  1+ y  p
for x = , f '' > 0 sin −1  −
3l  2 + y  6 0 
L = lim  0 form 
n y →0 y
which means that f is minimum at x = and
3l
 1 (2 + y) − (1 + y) 
since f ( x ) ≥ 0 for x > 0  × 
 1+ y 
2 (2 + y)2 
n
min (x 3 − nx + m) ≥ 0for x=  1 −  2 + y  
3l = lim  
y →0  1 
⇒ 27lm 2 ≥ 4n3  
 
98. Given lim f(x) = 3, lim f(x) = 2 and  
x →0 x →−1
 
lim f(x) = 4 where
x →1  (using L’ Hospital’s rule)
2
px + qx + r 1 1 2 1 1
f(x) = . = × = × =
2
x + x +1 1 4 3 4 2 3
1−
4
We have
px 2 + qx + r 101. Given y = sin (8 sin–1x), differentiating w.r.t. x, we have
3 = lim =r   (1)
x →0 2
x + x +1 dy 8
= cos (8 sin–1 x) ×
dx 1 − x2
px 2 + qx + r p − q + r
2 = lim 2 =
x →−1 x + x + 1 1  dy 
2

⇒ (1 - x2)   = 64 × cos2 (8 sin-1x)


⇒ p − q = −1   (2) [using (1)]  dx 

px 2 + qx + r p + q + r = (1 – y2) 64
4 = lim =
x →1 x 2 + x + 1 3 Differentiating again w.r.t x,
⇒ p + q = 9   (3) [using (2)] dy d 2 y  dy  dy
2

(1 - x2) 2 + (-2x)   = -2y × 64


Solving (2) and (3) we get p = 4, q = 5 dx dx 2  dx  dx
2
4x + 5x + 3 29 d2 y dy
\ lim f(x) = lim 2
= (1 - x2) −x = - 64y.
x →2 x →2 x + x +1 7 dx 2
dx
Differential Calculus  2.155

102. Given x2 + y2 = 8x  (1) and (2 - x)y2 = x3   (3) p cos x + sin x


We can find that they intersect at the points 104. Given f(x) = we have
p sin x + cos x
(
5 5 )
(0, 0), 8 , 16 and 8 , −16 .
5 5 ( ) 1 − p2
f '(x) =
4−x (p sin x + cos x)2
For curve (1) the slope is m1 = and for curve
y
Now, f(x) is monotonically increasing
(3 − x) x ⇒ f ’(x) > 0 ⇒ 1 - p2 > 0 ⇒ -1 < p < 1.
(2) it is m 2 = 3
(2 − x) 2
105. Consider the function y = 3 x
At (0, 0) m1 = ∞ , m2 = 0
1 −2 3
⇒ (1) and (2) do not intersect at 45°. ⇒ dy = x dx
3
 8 16  1
At  ,  When x = 106, dx = 25 then dy = × 25
5 5  3 × 104
3 m1 − m 2
3
1000025 ≈ 100.0008
m1 = , m2 = 7 and tan q = =1
4 1 + m1 m 2
 plog x + qx 3 + rx 2 + x, x > 0
106. We have f(x) = 
⇒ angle of intersection is 45° 3 2
plog(− x) + qx + rx + x, x, < 0
 8 −16 
At  ,
 5 5  p 2
 x + 3qx + 2rx + 1, x>0
⇒ f(x) = 
−3  p + 3qx 2 + 2rx + 1,
m1 = , m2 = 7, tan q ≠ 1 x<0
4  x
\ No. of points = 1 f ’(x) = 0 at x = 1, 2, -1

103. Consider f(x) = |x2 – 3| in 0, 6  ⇒ p + 3q + 2r + 1 = 0


p
3 − x2 , + 12q + 4r + 1 = 0
0≤x≤ 3 2
=  f(0)
2
 x − 3, 3<x≤ 6 - p + 3q - 2r + 1 = 0
Solving we get p = -2, q = -1/3, r = 1
= lim− f (f(x)) = lim+ f (f(x)) we say that
x→o x→o
n
Since f is modulus function,  1
107. L = lim  cot x − 
x →0  x
it is continuous at all points but f is not differentiable
n
at x = 3   1 
=  lim  cot x −  
x →0  x 
So Rolle’s theorem is not applicable to f(x) in [0, 6] 

3 ,
x
0 ≤ x ≤1  1
Now  Now, lim  cot x −  (∞ - ∞ form)
x →0  x
4 − x, 1 < x ≤ 3
x cos x − sin x  0 
log 3 × 3x 0 < x <1 log 3 × 3x 0 < x <1 = lim  0 
g '(x) =   x →0 x
−1 0<x<3 −1 1< x ≤1
g’(1- ) = 3 log 3 ≠ g’(1+ )
= lim ( − xsin x ) , by L’ Hospital’s rule
x →0

\ g(x) is not differentiable at x = 1. =0


Rolle’s theorem cannot be applied to g(x) Hence, required limit = 0
2.156  Differential Calculus

1
sin x \ lim f(x) = 0, lim+ f(x) = 1
108. L = lim   (∞0 form) x →1− x →1
x →0  x 
lim − f(x) = 1, lim + f(x) = 2
5 +1 5 +1
x→ x→
log L = lim − log x 2 2

x → 0 cosec x 5 +1
\ f is discontinuous at x = 1 and x =
2
1
= lim x (using L ‘Hospitals rule) 112. y = f(x2 + x + 1)
x →0 cosec x cot x
dy 1 2
⇒ = f (x + x + 1) (2x + 1)
sin2 x 0  dx
= lim  0 form 
x → 0 x cos x
= cos[(x2 + x + 1)2 - 1] × (2 x + 1)
2sin x cos x dy
= lim = 0 ⇒ L = e0 = 1. at x = 0 is given by cos(o) × 1 = 1
x →0 cos x − xsin x dx

109. Given f(x) = tan–1


x
2
, |x| ≤ 1. 113. Given f(x) = x (1 - x cot x) – 1
x
in 0, p ( 2 ) , we have
1+ 1− x
1
q x f ’(x) = 1 + x2 cosec2 x - 2x cot x +
Put x = sinq then = tan so that x2
1+ 1− x 2 2.
1
= (x cosec x - cos x)2 + sin2 x +
q 1 x2
y= = sin −1 x
2 2 > 0 ∀ x ∈ 0, p ( 2 )
\
dy 1
=
1
dx 2 1 − x 2
⇒ f(x) has no extrema in 0, p ( 2 )
dy 1 1 n(n + 3)
at = . 2x + 3x 2 + ... + (n + 1)x n −
dx 2 3 114. Let L = lim 2
x →1 x −1
4 3 0 
110. v = pr is the volume of a sphere of radius r.  0 form 
3
⇒ dv = 4pr2 dr. 2 + 3(2x) + ....n(n + 1)x n −1
= lim
\ Percentage rate of change in volume x →1 1
dv = 2 + 3.2 + ……(n + 1)n
= × 100 n n n
v = ∑ k (k + 1) = ∑ k + ∑ k 2

4 pr 2 dr  dr  k =1 1 1
= × 100 = 3  × 100  n (n + 1)(2n + 1) n (n + 1)
4 3  r 
pr = +
3 6 2
= 3 times percentage rate of change in r n (n + 1)(n + 2)
=
111. f(x) = [x – x + 1] in (0, 2)
2 3
115. y
 f(x)
0 , 0 < x <1 (−2, 3)

 5 +1
Then f(x) = 1 , 1 ≤ x <
 2 x
 5 +1 y = x2–1 o
1
2 , ≤x<2
2 −1
Differential Calculus  2.157

We can rewrite f(x) as f II (0) f III (0) f IV (0) f n +1 (0)


\ + I + II + ... + n −1
 x − 1 , x < −2
2
f(0) f (0) f (0) f (0)

 3 , x = −2 = 1.2 + 2.3 + …+n (n + 1)

f(x) = 1 − x , − 2 < x < 1
 0
n n(n + 1)(n + 2)

,x =1 = ∑ k(k + 1) =
k =1 3
2
 x −1, x > 1
From the graph, it is clear that f(x) is continuous  x + 3, x ≤3
117. Given f(x) =  2 we have
everywhere but not differentiable at  x − 3, x>3
x = -2, 1
 f(x) + 3, if f(x) ≤ 3
OR g(x) =  2
[f(x)] − 3, if f(x) > 3
Continuity:
f(x) is continuous in everywhere except perhaps at Now, f(x) ≤ 3, ⇒ x + 3 ≤ 3 ⇒ x ≤ 0
x = -2, 1 (i.e.,) when x ≤ 0,
At x = -2, lim− f(x) = lim x − 1 = 3 = f(−2)
x → −2 x → −2
( 2
) f(x) = x + 3 and f(f(x)) = (x + 3) + 3 = x + 6

lim f(x) = lim (1 − x ) = 3. When 3 < f (x) ≤ 6 we have 3 < x + 3 < 6


x → − 2+ x → −2
⇒ 0 < x ≤ 3
As lim− f(x) = lim+ f(x) = f(−2), f(x)is continuous (i.e.,) when 0 < x ≤ 3, f(x) = x + 3 and lies in (3, 6]
x → −2 x → −2

at x = -2. ⇒ f (f(x)) = f (x + 3) = (x + 3)2 - 3 = x2 + 6x + 6


At x = +1, lim− f(x) = 0 = f(1) = lim+ f(x) When f(x) > 6, we have x > 3. i.e., for x > 3, f(x) =
x → +1 x → +1
x2 - 3 and f(x) > 6
f(x) is continuous x =1 also.
⇒ f (f(x)) = f(x2 - 3) = (x2 - 3)2 - 3 = x2 - 6x2 + 6
f(x) is continuous everywhere
 x +6 , x ≤0
Differentiability:  2
 2x , x < −2 \ f f(x) =  x + 6x + 6, 0 < x ≤ 3
  x 4 − 6x 2 + 6, x > 3
Now f (x) = −1 , − 2 < x < +1
I

 2x , x > 1
 Clearly, g(0-) = ff(0-) = 6
At x = -2, f (-2 ) = - 4 ≠ f (-2 )(=-1)
I - I +
g(0+) = x2 + x + 6 at x = 0
⇒ f does not exist at x = -2
I
=6
⇒ f(x) is not differentiable at x = -2
At x = +1, fI (1-) = -1 ≠ fI(1+) (= 2)
⇒ f(x) is not differentiable at x = 1 1 sin 2x
⇒ f(x) is differentiable everywhere except at
x = -2, 1. O π π
4 2
116. Given f(x)= (1 – x) -1, |x| < 1.
we have on successively differentiating with respect
to x ⇒ g(x) is continuous at x = 0

f (x) = (1 - x)-1 ; f(0) = 1 Again,


fI (x) = (1 - x)-2 ; fI(0) = 1 g(3-) = 33
fII(x) = 1.2. (1 - x)-3 ; fII(0) = 2! g(3+) = (x4 - 6x2 + 6) at x = 3
fIII(x) = 1.2.3.(1 - x)-4 ; fIII(0) = 3! = 33
f (x) = k! (1 - x)
k -k - 1
; f (0) = k!
k
⇒ g(x) is continuous at x = 3
2.158  Differential Calculus

118. f(x) = x(x - 4) e2x in [0, 4] 4x 3 + 8x 2 + 5x + 2


f(0) = f(4) = 0 121. lim
x→∞ 2x 3 − 1
f(x) is continuous in [0, 4] and f ’(x) exists in (0, 4) 8 5 2
4+ + +
\ f ’(c) = 0 for some c in (0, 4) x x2 x3 4
= lim = = 2.
⇒ [c(c - 4)2 + (2c - 4)] e = 0
2c x→∞ 1 2
2− 3
⇒ 2c2 - 6c - 4 = 0 x

3 ± 17 sin2x x2
⇒ c = 122. lim ×2×
2 x→0 2x sin x 2

3 − 17 sin2x x2
But ∉0,4  = 2 × lim . lim = 2.
2 x→0 2x x2 → 0 sin x 2
\ The number of points where f ’(c) = 0 is 1  sin q 
Qlim = 1 .
⇒ Number of points where the tangent is parallel to  q→0 q 
x-axis = 1
123. lim− [ x ] + 1 = 3 + 1⇒ 4.
x→4
 p
119. f(x) – g(x) = sin 2x > 0 in  0, 
 2 (3x − 1)( x − 1)
124. limit = lim
 p x →1 ( x + 1)( x − 1)
\ f(x) > g(x) in  0, 
 2 (3x − 1) 2
= lim = =1
x →1 ( x + 1) 2
120.

a ⋅ eax − b ⋅ e bx
y = sin x 125. Limit = lim = a e0 − be0
x →0 1
y = cos x
π = (a – b) by L’ Hospital’s rule.
5
4
s 126. We can rewrite the given function as
0 π π 3π/2 2π
4 −1, x < 0

f(x) =  0, x = 0
 1, x > 0
From the graph we see that 
⇒ f(x) is continuous everywhere except at x = 0.
 p
 sin x, 0 < x ≤ 4 127. As f(x) is continuous at x = 0, a = f(0) = lim f(x)
 x →0
 p 5p
f(x) = cos x, < x ≤  x  x
 4 4 log 1 +  − log 1 − 
 2  2
5 p
sin x, < x ≤ 2p = lim
 4
x →0 x
 x  x
log 1 +  log 1 − 
 cos x , 0<x≤p  2  2
 4 = lim −
 p < x < 5p x x x
f ’(x) = − sin x , →0
×2 − × −2
4 4 2
2 2

 cos x , 5 p < x < 2p
4 1 1
= + =1
Number of critical points = 1 + 1 + 1 + 1 = 4 2 2

p 5p 3p dy
(at , p, , ) 128. = −4cos3 xsin x.
4 4 2 dx
Differential Calculus  2.159

−1  1 − t  −1  1 + t 
2 2 sin −1 x
= 135. y = e
129 x = cos  sec
 1 + t 2   1 − t 2 
−1 sin −1 x − p −1 −p
z = e − cos x
=e 2
= esin x
e 2

dy
= y \ =1 p dy p
dx y=z e 2
∴ =e 2
dz
x x x x d2 y
sin2
+ cos2 + 2sin cos =0
−1 2 2 2 2 dz 2
130. y = tan
2 x 2 x
cos − sin 136. g’ (x) = f ’(x) – 2f ’ (x) f(x) + 3f ’ (x) (f(x))2
2 2
x x = f ’ (x) [3 (f(x))2 – 2f(x) + 1]
sin + cos  2 1
= tan −1 2 2 = 3f ’ (x)  y 2 − y + 
x x  3 3
cos − sin
2 2  1  2
2

where, y = f(x) = 3f ' ( x )   y −  + 


p x
= tan −1 tan  +    3  9 
4 2
g’ (x) > 0 whenever f ’ (x) > 0 and g’ (x) < 0 whenever
p x dy 1 f ’ (x) < 0.
= + \ =
4 2 dx 2
137. f1 (x) = –4x3
131. u = 3x ; v = x12 6
f11 (x) = -12x2
du dv f1 (x) = 0 gives x = 0
= 36x11 ; = 6x 5
dx dx Now f1 (0-) > 0, f1 (0+) < 0.
11
du 36x Hence, f(x) is maximum at x = 0.
= = 6x 6 .
dv 6x 5
138. x = 2t3 – 3t2 + 6t
dy dy y = 2t3 + 3t2 + 6t
132. 2x + x + y + 2y =0
dx dx dy t 2 + t + 1
= =m
dy dx t 2 − t + 1
2x + y = − ( x + 2y )
dx \ t2 + t + 1 = mt2 – mt + m
dy  2x + y  i.e., t2 (1 – m) + t (1 + m) + 1 – m = 0
= −  Since t is real, (1 + m)2 – 4 (1 – m)2 ≥ 0
dx  x + 2y 
−5 i.e., –3m2 + 10m – 3 ≥ 0
dy 1
at x = 1, y = is = 2 = −5 .
dx 2 2 4 i.e., 3m2 – 10m + 3 ≤ 0
 1
dy i.e., (m – 3)  m −  ≤ 0
= esin x + x (cos x + 2x )  3
2
133.
dx
1 
= y (cos x + 2x). \ m lies in the interval  , 3
3 
134. f(x) = (1 + x) (1 + x2) (1 + x4) (1+ x8) \ Maximum slope is 3.
log f(x) = log (1 + x) +log (1 + x2) + log (1 + x4) 1
Minimum slope is .
+ log (1+ x8) 3
1 1 2x 4x 3 8x 7 139. By Mean Value Theorem,

f(x)
f '(x) = + + +
1 + x 1 + x2 1 + x 4 1 + x8 f ( x 2 ) − f ( x1 )
f ’ (x3) =
x 2 − x1
when x = 1, f(x) = 16
1 2 4 8 8ax 22 − 8ax12
∴ f ' (1) = 16  + + +  = 120. 16ax3 =
2 2 2 2 x 2 − x1
2.160  Differential Calculus

x 22 − x12 As lim f(x) ≠ lim f(x), x = 1 is a point of discon-


2x3 = = x1 + x 2 −
x →1 +
x →1
x 2 − x1 tinuity.
x1, x3, x2 are in AP
143. x4 y5 = a9
cos3x − cos7x 4 log x + 5 log y = 9 log a
140. lim
x →0 x2 4 5 −4y
+ y' = 0 ⇒ y' =
2sin5x sin2x x y 5x
⇒ lim
x x
Tangent at ( x1 , y 1 ) on the curve is
x →0

sin5x sin2x
⇒ lim 2 ⋅ ⋅ (5) (2) ⋅ ⇒ 20.
−4y 1
x →0 5x 2x y − y1 = ( x − x1 )
5x1
x2 + 1 − 3 x3 + 1
141. lim
x →∞ 4
x 4 + 1 + 5 x5 + 1 Y
B
 1 1 
x  1+ 2 − 3 1+ 3 
x x  P(x1, y1)

= lim =0
x →∞  1 5 1  X
x  1+ 4 + 1+ 5 
4 O L A
 x x 

 x3 , x ≤ −1 or 5x1 y − 5x1 y 1 = −4xy 1 + 4x1 y 1



2x + 1, − 1 < x ≤ 0 4xy 1 + 5x1 y = 9x1 y 1
142. Given f(x) =  x
 e , 0 < x ≤1
x y
 x 2 , 1< x or + =1
 9 9y 1
x
As polynomial functions and exponential functions 4 1 5
are continuous everywhere, f(x) is continuous at all x 9x1
except possibly at x = 0, ± 1. − x1
AP AL OA − OL 5
= = = 4 =
At x = -1 PB LO OL x1 4
lim f(x) = lim x 3 = − 1 = f(−1)
x →−1− x →−1  9x   9y 1 
A and B are the points  1 ,0  and  0, 5 
lim f(x) = lim 2x + 1 = − 1  4 
x →−1+ x →−1

As f(-1) = lim+ f(x) = lim− f(x), x = -1 is a point 2 + 4 − x2


x →−1 x →−1
144. y = log − 4 − x2
of continuity. 2 − 4 − x2
At x = 0 Put x = 2 sin q
lim− f(x) = lim 2x + 1 = 1 = f(0) q
x →0 x →0
y reduces to 2log cot − 2cos q
2
lim f(x) = lime x = 1
x → 0+ x →0
 q 
−2cosec2
As f(0) = lim− f(x) = lim+ f(x), x = 0 is a point of dy  2 + 2sin q dq
x →0 x →0 = 
continuity. dx  q  dx
2cot
 2 
At x = 1
lim f(x) = lim e x = e = f(1) − cos q − 4 − x 2
x →1− x →1 = =
sin q x
lim f(x) = lim x 2 =1
x →1+ x →1 Let P be (x1, y1)
Differential Calculus  2.161

Equation of the tangent at P on the curve is ⇒ f(x) is differentiable at all points except perhaps
2 at x = 0
− 4−x
y − y1 = 1
( x − x1 ) At x = 0
x1
h
To get T we put x = 0, we obtain −0
f(h) − f(0) + 2
f '(0+) = lim+ = lim h
1 =1
y = y 1 + 4 − x12 h→0 h−0 h→0 h

(
Hence, T is 0, y 1 + 4 − x12 and P is (x1, y1) ) f(h) − f(0)
−h
+ 2
−0
f ’(0 ) = lim−
-
= lim h
1 =−1
PT2 = 4. h→0 h−0 h→0 h
As f ’(0+) ≠ f ’(0-), f(x) is not differentiable at x = 0.
x2
145. 9y = x ,18yy ' = 3x ⇒ y ' =
2 3 2

6y 147. Given y = x3 - 9x2 + 24x + 13


 3
 We have
x 2
Let the required point be ( x1 , y 1 ) =  x1 , 1  y’ = 3x2 - 18x + 24
 3 
  Tangents make an acute angle with the positive x-axis
⇒ y’ > 0 at these points
Equation of the normal at ( x1 , y 1 ) is
⇒ 3(x2 - 6x + 8) > 0
−6y
y − y 1 = 2 1 ( x − x1 ) ⇒ (x – 2) (x – 4) > 0 ⇒ x < 2 or x > 4
x1
3
148. –ve –4 + –1 –ve 5 +
x12
3
x 6 2
2 Given f(x) = (x - 42x - 80x +32)3 we have
4 2
y- = - 32 ( x − x1 ) = - 1 (x − x1 )
1

3 x1 f ’(x) = 3(x4 - 42x2 - 80x +32)2 (4x3 - 84x – 80)


x12
1 = 12(x4 - 42x2 - 80x +32)2 (x + 1) (x + 4) (x – 5)
3y x 12 - x12 = − 6x + 6x1
⇒ f(x) is monotonically increasing or monotonically
1 decreasing accordingly as f ’ ≷ 0
6x + 3y x = x12 + 6x1
2
1
i.e., accordingly as (x + 1) (x + 4) (x – 5) ≷ 0
x 2 + 6x1 ⇒ function is monotonically increasing in
x- intercept = 1
6 (-4, -1) ∪ (5, ∞) and monotonically decreasing
x12 + 6x1 in (-∞, -4) ∪ (-1, 5), as per the sign scheme,
y - intercept = 1 given above
3x12
x−5
1
149. xlim =1
\ 6 = 3x or x1 = 4 2
1
→∞ x+3
x
|x|  x −5
146. f(x) = can be rewritten as Therefore, lim   is of the form 1

x →∞  x + 3 
1 + x2
 −x  x −5
x

 , x<0 Let L = lim 


2

1 + x x→∞  x + 3 
f(x) =  0 , x = 0
 x  x −5
 log L = lim x log 
 1 + x2
, x>0 x→∞  x + 3 
Q x2 ≥ 0 for all x, we have 1 + x2 > 0 all x.  x-5
log 
x −x  x + 3   0 form 
⇒  and are differentiable at all points = lim  0 
1 + x2 1 + x2 x→∞ 1
in their respective domains. x
2.162  Differential Calculus

1 1 1 2 5
− Y= + = .
= lim x − 5 x +3 4 2 4
x→∞ 1  −1 5 
− 2 Centre of curvature is at  , 
x  2 4
−8x 2 155. y = ex
= lim = -8
x →∞ ( x − 5)( x + 3)
y’ = y ” = ex
⇒ L = e-8 At (0, 1), y ’ = y ” = 1.
3
p (1 + 1) 2
x x − sin x r at (0, 1) is = = 2 2.
2  0 form  1
150. L = lim  0 
x →1 p
cos x 1 (1 + 1)
2 X = 0 − (1 + 1) = −2 ; Y = 1 + = 3.
1 1
p p
x x (1 + log x) −
cos x Centre of curvature is at (-2, 3).
= lim 2 2
x →1 p p Equation of the circle of curvature at (0, 1) is
− sin x
2 2
( )
2
(x + 2)2 + (y - 3)2 = 2 2
−2
= (using L ‘Hospital’s rule)
p ⇒ (x + 2)2 + (y - 3)2 = 8.
151. Statement 2 is not always true. x2 + y2 + 4x - 6y + 5 = 0
The only conclusion is that f ’(x) ≥ 0 in I
x2
Consider Statement 1 156. y = mx +
a
f(3) is clearly > 0
Differentiating with respect to x,
Statement 1 is true
2x
Choice (c) y' = m + y ’ at origin = m
a
152. Statement 2 is true 2
Using Statement 2 we can see that Statement 1 is also y" =
a
true.
(1 + y ' ) (1 + m )
3 3
2 2 2 2
Choice (a) a
( )
3
r= = = 1 + m2 2
.
153. Statement 2 is true y" 2 2
 a 
Consider Statement 1:
f ’(x) = 3x2 - 10x + 9 > 0 for all x, as discriminant of If ( X , Y ) are the coordinates of the centre of curvature
the quadratic is < 0
at the origin,
f(x) is monotonic increasing and f(0) = 1
ma
Hence, max f(x) occurs at x = 2 X =0− (1 + m 2 ) ;
2
However, this does not follow from Statement 2
(1 + m 2 )a
Choice (b) Y =0+
2
154. y = x2 Equation of the circle of curvature at the origin is
1 1
y ’ = 2x, y ” = 2 ⇒ y ’ at  ,  is = 1 am 
2
a(1 + m 2 ) 
2
2 4  2 
 x + (1 + m ) +
  y − 
 2   2 
1  y'  1 1 −1
X=
2  y "
(
−  1 + y '2  ) = − ×2= .
2 2 2 a2
 at  1 , 1  = (1 + m 2 )3
2 4 4
Differential Calculus  2.163

⇒ x2 + y2 + am(1 + m2)x - a(1 + m2)y


−1 − 33
a2 a 2 m2 a 2 (1 + m 2 )2 ⇒ x=
= (1 + m 2 )3 - (1 + m 2 )2 − 2
4 4 4
−1 + 33
a2 x= ⇒ y = f(x) graph
{
= (1 + m 2 )2 1 + m 2 − m2 − 1 = 0
4
} 2

⇒ x2 + y2 = a(y - mx) (1 + m2)

157. Let f(x) = ax + b


x−b
\ f-1(x) =
a
Slope of the curve y = ax + b at any point equals a.
− 1 − 33 −3 33 − 1
1
Slope of the curve y = f-1(x) at any point equals 2 2
a
−1 ⇒ f(x)
Slope of the curve y = f-1(-x) at any point equals
a
f(-x) = -ax + b
x−b
f-1(x) =
a
(c) is true

158. f(a) = f(b) + (a – b) f ’(g)


f(b) − f(a) − 1 − 33 33 − 1
⇒ f ’(g) =
(b − a) 2 2

2ag + b =
( )
a b2 − a 2 + b (b − a )
(b − a )
⇒ from graph of y = f(x), it is clear that f(x) is con-
i.e., 2ag + b = a(b + a) + b
−1 + 33
b+a tinuous and not differentiable at x =
\ g = 2
2 and maximum does not exist and at x = 1 it is
i.e., a, g, b are in A.P. continuous and differentiable and f ’(1) ≠ 0
\ (a) is true − p − 2tan −1 x , x < −1
a + b + g = 3g 160. (a) sin −1 2x  −1
=  2tan x , −1 ≤ x ≤ 1
1 + x2 
\ (a + b + g)3 = 27g3 −1
 p − 2tan x , x >1
\ (d) is true It decreases steadily in (-∞, -1), increases in (-1, 1)
159. x2 – 4 = x – 2 x > 2 x2 – 4 = 2 – x x<2 and then decreases.
⇒ x - x - 2 = 0
2
x +x-6=0
2    The function is not differentiable but is continu-
(x - 2) (x + 1) = 0 (x + 3) (x - 2) = 0
ous at x = 1.

x = 2, x = -1 ⇒ x = 2 x = -3
   x = ±1 are critical points.
2.1 p 2 (−1) − p
x2 - 4 = x - 4 (x ≥ 4) x2 - 4 = 4 - x x<4 sin-1 = , sin −1 = ,
1+1 2
2 1+1 2
⇒ x = 0, 1 not possible x +x-8=0
2

p −p
−1 ± 1 + 32 −1 ± 33 and is the maximum value of sin-1k, is the
x= = 2 2
2 2 minimum.
2.164  Differential Calculus

p 1 1
\ the function has an absolute maximum at log x. − log(x + e).
2 x +e x
x =1 in any interval containing 1 (d) f ’(x) =
(log x)2
1 −2 8x −1
(b) f ’(x) = 8x 3 − x 3 = 2 ; x ≠ 0 x log x − (x + e)log(x + e)
x 3 =
x(x + e)(log x)2
1
f ’(x) = 0 ⇒ x = f ’(x) = 0 ⇒ xlogx = (x + e)log(x + e) which is
8
not possible since xlogx < (x + e)log(x + e) for all
1−  1+  x; logx being an increasing function
f ’   < 0 and f '   > 0
8  8  Also f ’(x) < 0 ∪ x > 0
1 \ f(x) is monotonically decreasing in (0, ∞)
⇒ x = is a local minimum and the minimum
8 (d) → (p)
1 9
value is f   = −
8 8
1 1
Also f ’(x) < 0 ⇒ x < and f ’(x) > 0 ⇒ x > Additional Practice Exercise
8 8
(b) → (r)
161. We use the expansions of ex, e-x, log (1 + x) and sin x
(c) f(x) = 2x2 - log|x|; x ≠ 0 in ascending powers of x.
Case (i) x < 0
As x → 0 in a neighbourhood of 0, | x | < 1.
1
f ’(x) = 4x + < 0 ⇒ f is monotonically decreas- p x e x + 2q log(1 + x) + 3r x e − x
x Let X =
ing in (-∞, 0) x 2 sin x
Also f ’(x) ≠ 0 for any x < 0 Then numerator of X
Case (ii) x > 0  x x2 x3 
1 4x 2 −1 = px 1 + + + + .... 
f ’(x) = 4x - =  1! 2! 3! 
x x
1 1  x2 x3 x 4 
f ’(x) = 0 ⇒ x2 = ⇒ x = ± . But x > 0 +2q  x − + − + .... 
4 2  2 3 4 

( 2 ) < 0 & f '( 12 ) > 0


f' 1
− +
 x x2 x3 
+3r x 1 − + − + .... 
 1! 2! 3! 
1
⇒ f(x) has a local minimum at x = , and the = (p + 2q + 3r) x + (p – q – 3r) x2
2
1  p 2q 3r  3
minimum value is + log2 +  + + x
2  2 3 2 
( 2 2 )
f ’(x) < 0 ⇒ x ∈ − 1 , 1 . But x > 0 terms involving x 4 and

+

  (1)
higher powers of x
f ’(x) > 0 ⇒ x ∈ ( 1 , ∞ )
 


2
⇒ f is decreasing in (-∞, 0) ∪ (0, 1 ) and is increas-
 x3 x5 
2 Denominator of X = x 2  x − + − .... 
 3! 5! 
ing in ( 1 , ∞ )
2 x5 x7
= x3 − + − ....   (2)
(c) → (q) 3! 5!
Differential Calculus  2.165

p + 2q + 3r p − q − 3r p 2q 3r h
+ + + + + sin
x2 x 2 3 2 4. 1 1
= 2 ×
terms involving x and
  h  h 2
sin
  4  2
higher po
 owers of of x 
  h 
X=
terms involving x 4 and
x2    
 2 
1− + 
3! 
higher powers of x 
 1 1
→ 2 . 1. 1. =
 2 2
 p + 2q + 3r = 0 Aliter:

Since lim X = 3 , we must have  p − q − 3r = 0
x →0
 p 2q 3r
 + + =3
cos
x
4
p
− sin =
4
( 4(
2 sin p − p
4 ))
2 3 2
x p
9 9 y = −y
Solving for p, q, r, we get p = , q = -9 and r = . 2 2
2 2 x x
y → 0 as x → p; sin = cos y, cos = sin y
2 2
162. Let x = p + h
x x y
As x → p, h → 0 cos − sin = 2 sin
4 4 2
x
1 − sin 1 − sin x
2 2
∴ lim
 x  x x
 cos 2  cos 4 − sin 4 
 
x→p
cos x
2(cos x − sin x
4 4 )
1 − cos y
p h = lim
1 − sin  +  y →0 y
sin y. 2 .sin
2 2 2
=
p+h  p+h  p + h  2 y
cos  cos  4  − sin  4   2sin 1
 2    = lim 2 =
y →0 y y 2
2sin2 2 .cos
2 2
 h
1 − cos 2 
= 163. (i) Method 1
 h  p h  p h  lim x n e − x (∞ × 0 form)
− sin  cos  +  − sin  +  

x →∞

2   4 4   4 4 
xn  ∞ 
= lim  ∞ form 
 h x →∞ e x
1 − cos 2 
= nx n −1 ∞ 
 h  1  h h 1  h h  = lim , by L’Hospital’s rule  ∞ form 
 − sin 2    cos 4 − sin 4  −  cos 4 + sin 4   x →∞ e x
 2 2 
n(n − 1) x n − 2
= lim
 h h ex x →∞
2 1 − cos  2 × 2sin2
 2 4 …………………………
= =
h h h h …………………………
2sin sin 2sin sin
2 4 2 4 Since n > 0, after some stage, the index of x in
h the numerator of the above becomes negative.
2 sin Consequently, limit reduces to the form
= 4
h a constant
sin lim where k > 0 = 0
2 x→∞ x k ex
2.166  Differential Calculus

(ii) | e − mx cosax | ≤ e − mx for any x =


1
(2sin q 2 cos q 2 )
since | cos ax | ≤ 1 for any x 2 n −1
(
sin q
2 n −1 )
As x → ∞
sin q
lim | e − mx cosax | ≤ lim e − mx = 0 since m > 0 =
x →∞ x →∞  q 
2n −1 sin  n −1 
2 
It immediately follows that lim e − mx cosax = 0
x →∞
Substituting in (1)
(iii) Proof is exactly on the same lines as (ii)

  
q
  q 
f(n, q) = 2n −1 sin q ( 2 n −1 ) × tan1 q × 1
q
 cos q   cos 2   cos 22  cos2  n 
2 
164. f(n, q) =  × × × .......
q  q   q 
 cos2   cos2 2   cos2 3 
( )  ×
 2  2   2   
q  sin q n −1 1
× 2
 q  =
 cos 2n −1  tan q   q   q
cos2  n 
..... ×    2n −1   2 
q   
 cos2 n 
( )
 2 
 sin q n −1
 q  2
cos q lim f(n, q) =  × lim
=
cos q cos q cos q .....cos q


× n →∞
 tan q  n →∞
 q
2 n −1 ( )
2 22 23 2n −1 
 
1  1 
  (1)
( 2)
 × lim  
cos2 q n
n →∞ q
 cos2 n 
 2 
cos q cos q 2 cos q 3 ......cos q n −1
2 2 2 2  q 
×
 sin q
 2 n −1 ( ) 
=  lim  ×1
=
1
( 2 2 )
× 2sin q n −1 cos
q
 
 tan q  nq−1 → 0  q n −1 ( )
(2 )
n −1 2
 2 
2sin q n−2

q
× cos ( q
2 )
× cos ( q
2 )
= × 1 = q cot q
× ......cos q tan q
n−2
2 n−3

× ( sin q
2 )
1 165. When x < 1, xn → 0 as n → ∞
= cos q ×
2 sin (
2 )
n−2 n−2
q 2 3x cos x
n −1 f(x) =
2
× cos ( q
2 )
 × .....cos q 3cos 1+ e
2 n−3 When x = 1, f(x) = =
3
× ( sin q
2 )
1
= cos q When x > 1,
2 sin ( )
n−3 n−3
2 q 2
2 n −1  3x cos x x 
 n
+e 
.......cos
q f(x) = lim  x 
 2 n →∞   2  
= ………   x n + 1 
 
=
1
(
× sin q ) cos ( q 2 ) ×
( ) 2n − 3  3cos x x 
n−3
2 n −1
sin q n −1  n −1 + e 
2
= lim  x  = ex
n →∞   2  
q
……. cos   x n + 1 
2  
Differential Calculus  2.167

3 1 I 1
 2 x cos x, x <1 I = n−3 +
(n − 2)! n − 2 (n − 3)! (n − 2)

 e
Therefore, f(x) =  cos1 + , x = 1 I 2 I1 1
 x 3 = +
2! 1! 2
e , x >1
 Addition gives
In 1 1 1
Clearly, f(1-) ≠ f(1+ ) = I1 + + + ... +
n! 2 3 n
⇒ f(x) is not continuous at x = 1, it is continuous at d 1 1 1
all other points of (0, ∞) = (x log x) + + + ... +
dx 2 3 n
166. We show that the derivative of f(x) is zero everywhere 1 1 1
in R from which it follows that f(x) is a constant in R. = log x + 1 + + + ... +
2 3 n
For any two points x1, x2 ∈ R, we are given
 1 1 1
|f(x2) - f(x1)| ≤ |x2 - x1|3 In = n!  log x + 1 + + + ... + 
 2 3 n
f(x 2 ) − f(x1 )
Now, f ’(x1) = lim (csin x + d cos x)(a cos x − bsin x)
x 2 → x1 (x 2 − x1 )
− (a sin x + bcos x )(ecos x − dsin x )
f(x 2 ) − f(x1 ) 168. f '(x) =
|f ’(x1)| = lim (csin x + d cos x )
2
x 2 → x1 (x 2 − x1 )

f(x 2 ) − f(x1 )
(ad − bc)
=
= lim ≤| x 2 − x1 |2 ,
(csin x + d cos x )
2
x 2 → x1 (x 2 − x1 )
from the given condition → 0 as x2 → x1 f '(x) < 0 if (ad - bc) < 0
It follows that f ’(x1) = 0 1

Since x1 is an arbitrary point in R, 169. f(x) = x x


f ’(x) = 0 in R. 1
log f(x) = log x
x
dn n Differentiating both sides of the above with respect
167. If In = [x logx]
dx n to x,
d n −1  d n  1 1 − log x
= n −1 
(x log x) f '(x) =
dx  dx  f(x) x2
1
d n −1
=
dx n −1
{
x n −1 + nx n −1 log x } ⇒ f ’(x) =
x x (1 − log x)
  (1)
x2
n −1
d


= (n - 1) ! + n

= (n - 1) ! + nIn - 1
dx n −1
(xn - 1 logx)
f ”(x) =
(1 − log x) d 1 x
x2 dx
x ( )
 2  −1  
In = nIn - 1 + (n - 1)!   (1)
 x  x  − (1 − log x)2x 
1
Divide both sides of (1) by n! +x x  
 x4 
1 I 1
I n = n −1 +   (2)  
n! (n − 1)! n
Replace n by (n - 1) in (2)
1 I 1
=
(1 − log x)
x 2
×
d 1x
dx ( )1  2x log x − 3x 
x +x x
 x4


I n −1 = n − 2 +
(n − 1)! (n − 2)! (n − 1) f ’(x) = 0 ⇒ log x = 1, x = e.
Change n to (n - 2) in (2) When x = e, f ”(x) < 0
2.168  Differential Calculus

1 1 This means that y is maximum at t = -1 and minimum


Therefore, the maximum value of x x
is e e .
3
1 1 1 1 at t = .
x x
≤e e
⇒ p p
≤e e 2
1 1
Maximum value of
⇒ e e
≥ p p
y = 4(-1) – 3 × 1 + 18 + 3 = 14.
( ) > (p )
1 pe 1 pe
We have e = e p e p Minimum value of
3 2
3 3 3
i.e., ep > pe. y = 4   − 3   − 18 × + 3 = –17.25
2 2 2
170. Consider f(x) = – 4x3 + 18x2 – 24x + p, x ∈[1,2]
so that f ’ (x) = - 12 (x - 1) (x - 2) > 0 ∀ x ∈(1,2) 173. A point P on y = x2 may be taken as (t, t2). Slope of the
tangent at P = 2 × t = 2t.
⇒ f(x) is monotonic increasing in (1, 2)
−1
Also f(1) = -10 + p, f(2) = - 8 + p ⇒ Slope of the normal at P =
2t
Thus f(x) = 0 has exactly one root in [1, 2] if
f(1) < 0 and f(2) > 0 t 2 − t12 −1
If the normal meets the again at ‘t1’, =
i.e., 8 < p < 10 t − t1 2t

171. Let f(x) = f(x) – 3g(x) −1


t + t1 = where, t1 is the value of x correspond-
Both f(x) and g(x) are differentiable in [a, b] 2t
f(a) = f(a) -3g(a) = 3 - 3 (-3) = 12 ing to Q.
f(b) = f(b) - 3g(b) = 30 - 3(6) = 12 −1  −1 1  
2

⇒ t1 = − t ⇒ Q is  − t,  + t  
\ f(a) = f(b) 2t  2t  2t  
\ By Rolle’s theorem, there exists a point c in 2
2
  −1     
2
[a, b] at which f’(c) = 0 1
PQ =  t −  − t   +  t 2 −  + t  
2
\ f ’(c) - 3g’(c) = 0   2t     2t  
f '(c) 3
=  1
2
1  1
2
g '(c) 1 =  2t +  + 2

 2t  4t  2t + 2t 

dx 1 3
172. = 5t4 – 15t2 – 20 = 5(t4 – 3t2 – 4)
dt = + + 3 + 4t 2
16t 4 4t 2
dy
s = 12t2 - 6t - 18 = 6(2t2 - t - 3)
dt As PQ is +ve, if PQ is minimum, PQ2 will be
minimum
 dy 
dy  dt  6(2t 2 − t − 3) ⇒
d d2
[PQ2 ] = 0, 2 [PQ2 ]> 0
= = dt dt
dx  dx  5(t 4 − 3t 2 − 4)
 dt  d 1 3
[(PQ)]2 = − 5 − 3 + 8t
2
d y 6 dt 4t 2t
= ×
dx 2  5   1  2 1   1  1 
2

=  2t −   4 + 2 + 4  =  2t −   2 + 2 
4 2
(t − 3t − 4)(4t − 1) − (2t − t − 3)(4t − 6t) 2 3
1  t   t 4t   t   2t 
×
(t 4 − 3t 2 − 4)2  dx d2 5 9
 dt  2
[(PQ)2 ] = 6 + 4 + 8
dt 4t 2t
dy 3
= 0 ⇒ t = −1, . d 1  1 
2
dx 2 [(PQ)2 ] = 0 ⇒ t = ± . , as  2 + 2  > 0 for
d2 y 3 d2 y dt 2 2t
When t = -1, < 0 and when t = , > 0.
dx 2
2 dx 2 all real t
Differential Calculus  2.169

1 d2 175.
For t = ± , [(PQ)2 ] is positive. α
2 dt 2
1
Therefore, PQ is minimum when t = ± .
2 B
A
 1 1   −1 1 
⇒ The coordinates of P are  , or ,
 2 2   2 2  Let r be the radius of the circle. Observe that the slant
height of the cone is r and the circumference of the
174. O base is ra.
If x is the radius of the base,
ra
C x=
R 2p
Let V represent the volume of the cone.
r
A B 1
D V = px 2 h where, h is the height of the cone.
3
We have r2 - x2 = h2.
Let R be the radius of the sphere.  r6 
1
Since a right circular cone has to be inscribed, the \ V 2 = p2 x 4 h 2 = 
9 9 × 64 p 4  (
4 p2 a 4 − a 6 )
section of the cone is as shown in the figure.  
Let r be the radius of the base and h be the height of If we denote (4p2 a4 - a6) by u, V is maximum when
the cone, If V represents the volume of the cone. u is maximum
1 du d2 u
V = pr 2 h   (1) = 16p2 a3 - 6a5; = 48p2a2 - 30 a4
3 da da 2
From DCAD, R2 = r2 + CD2
du 8 p2
= r2 + (h - R)2 = 0 ⇒ a2 =
da 3
⇒ r2 = R2 - (h - R)2 = 2hR - h2
For this value of a2, it can be seen that
Substituting in (1),
d2 u −256 4
1 1 = p is negative.
V = ph(2hR − h 2 ) = p(2Rh 2 − h 3 ) da 2 3
3 3
Therefore, the required value of the angle is a =
dV d2 V
For V to be maximum, = 0, <0 2
dh dh 2 2p radians.
3
dV 1
= p(4Rh − 3h 2 )
dh 3 2a + 3h 3h 2a + 3h h
2cos sin − 3.2cos .sin
d2 V 1 2 2 2 2
= p(4R − 6h) 176. It is lim
dh 2 3 h→0 h3
dV 4R 2a + 3h
= 0 ⇒ h = 0 or . 2cos
2  h h
dh 3 = lim sin3 2 − 3sin 2 
Clearly, h = 0 is a trivial solution
h→0 h3  
4R d 2 V h
For h = , <0 −4sin3
3 dh 2 2a + 3h 2
= lim2.cos .
4R
h→0 2 h3
or h = makes V maximum. (sin3A - 3sinA = -4sin3A)
3
3
2 1
h = diameter of sphere = -8. cosa .   = − cosa
3 2
2.170  Differential Calculus

177. Differentiating the given relation, we get = Real part of RHS of (1)
du
( ) ( )
n
2u = a2 (-2cosx sinx) + b2 (2 sin x cosx) = a 2 + b2 2
.eax .cos bx + c + n tan −1 b
dx a
du and
⇒ u = (b2 - a2) sinx cosx   (1)
dx D (e sin(bx + c))
n ax

( ) ( )
n
Differentiating the above relation with respect to x = a 2 + b2 2
.eax .sin bx + c + n tan −1 b
again, a
2 179. When the independent variable x is changed to z by
d 2 u  du  the substitution z = x2, y reduces to a function of z.
u +   = (b2 - a2) (cos2x - sin2x)
dx 2  dx  We have
= (a2 sin2x + b2 cos2x) - (a2 cos2x + b2 sin2x) dy dy dz dy dy
= × = × 2x = 2x
= (a2 sin2x + b2 cos2x) - u2 dx dz dx dz dz
2 d2 y d  dy 
d2 u  du 
(
u 2 + u 2 = b2 cos2 x + a 2 sin2 x −  
dx  dx 
) =  2x 
dx 2 dx  dz 

= (b2 cos2 x + a2sin2x) dy d  dy  dy d  dy  dz


=2 + 2x   = 2 + 2x ×   ×
dz dx  dz  dz dz  dz  dx
(b )
2 2
− a 2 sin2 x cos2 x
- using (1) dy d2 y dy d2 y
u2 =2 + 2x × 2 × 2x = 2 + 4x 2 2
dz dz dz dz
1 dy d2 y
= [(a2 cos2 x +b2 sin2 x) (b2cos2x + a2 sin2x) =2 + 4z 2
u2 dz dz

- (b2 - a2)2 sin2 x cos2 x] Substituting in
1 d2 y dy
= [a2b2 cos4x + a2b2 sin4 x x2 +x + 4 (x4 - l2) y = 0
u2 dx 2 dx
+ 2a2b2 sin2 x cos2x] it reduces to
2 2
1 2 2 ab  dy d2 y   dy 
= 2
a b (cos2x + sin2 x)2 = 2 z 2 + 4z 2  + x  2x  + 4 (z2 - l2 ) y = 0
u u  dz dz   dz 
d2 u a 2 b2 d2 y dy
\ + u = ⇒ 4z 2 + 4z + 4 (z2 - l2 ) y = 0
dx 2 u3 dz 2 dz
178. Consider d2 y dy
⇒ z2 + z + (z2 - l2 ) y = 0.
{ }
2
F(x) = eax cos (bx + c ) + isin (bx + c ) dz dz

180. Consider the infinite GP


= e(
a + ib) x
× eic
1 + x + x2 + x3 + … ∞, | x | < 1
nth derivative of F(x) 1
= F(n) (x) = (a + ib)ne(a+ib)x × eic Its sum is
1− x
= rn(cosnq + isinnq) e(a + ib)x × eic 1
⇒ = 1 + x + x 2 + x 3 + ...∞
where r = (a 2
+b 2
) 1− x
Differentiating both sides with respect to x,
b 1
q = tan-1 a = 1 + 2x + 3x 2 + ...∞
(1 − x)2
⇒ F(n)(x) = rneax . ei(bx + c) × einq Multiplying both sides of the above by x,
= rneax . ei(bx + c + nq)   (1) x
= x + 2x 2 + 3x 3 + ...∞
Hence, Dn(eaxcos(bx + c)) (1 − x)2
Differential Calculus  2.171

Differentiating the above with respect to x, For - 1 < x ≤ 0, x = 0, |2 - x| = 2 - x


d  x  2 2 2 2 3 ⇒ f(x) = 0 + 2 - x = 2 - x.
  = 1 + 2 x + 3 x + 4 x + ...∞
dx  (1 − x)2  For 0 < x ≤ 1, x = 1, |2 - x| = 2 - x

∞  ⇒ f(x) = 3 - x.
1 
= x  + 22 + 32 x + 42 x 2 + ... = x  ∑ r 2 x r − 2  For 1 < x ≤ 2, x = 2, |2 - x| = 2 - x
x   r =1 
⇒ f(x) = 4 - x.

1 d  x 
⇒ ∑ r 2 xr −2 =
r =1
 
x dx  (1 − x)2  For 2 < x < 3, x = 3, |2 - x| = x - 2
⇒ f(x) = 1 + x.
1  (1 − x)2 + x × 2(1 − x) 
=   1 − x, − 2 < x ≤ −1
x (1 − x)4   2 − x,
−1 < x ≤ 0

1 1 − x + 2x  1+ x \ f(x) =  3 − x, 0 < x ≤1
=  3 
=
x  (1 − x)  x(1 − x)3  4 − x, 1< x ≤ 2

 1 + x, 2<x<3
181. (i) Sr = Sum of infinite geometric series whose first
1 (ii) As polynomial functions are continuous every-
term is r and common ratio is
r +1 where, f(x) is continuous at all points of (-2, 3)
except possibly at x = -1, 0, 1, 2.
r r(1 + r)
= = ⇒ Sr = 1 + r At x = -1
1 r
1−
1+ r LHL = lim− f(x) = lim (1 − x) = 2 = f(−1)
x →−1 x →−1
2n −1 2n −1 (2n − 1)2n
(ii) ∑ (S
r =1
r − 1) = ∑r =r =1 2
= n(2n − 1) RHL = lim+ f(x) = lim (2 − x) = 3 ≠ f(−1)
x →−1 x →−1

As LHL ≠ RHL, f(x) is not continuous at x = - 1


2n −1 2n −1 (2n − 1)2n(4n − 1)
and ∑ (S
r =1
r − 1) = 2
∑r
r =1
2
=
6 At x = 0
LHL = lim− f(x) = lim (2 − x) = 2 = f(0)
n(2n − 1)(4n − 1) x →−0 x →−0
=
3 RHL = lim+ f(x) = lim (3 − x) = 3
x →−0 x →−0
  2n −1  
3

 ∑ (Sr − 1) 
 As LHL ≠ RHL, f is not continuous at x = 0
  r =1   n3 (2n − 1)3 × 9
lim  2  = lim 2 At x = 1
n →∞ n →∞ n (2n − 1)2 (4n − 1)2
  2n −1 2 
 ∑ r (S − 1)   LHL = lim− f(x) = lim (3 − x) = 2 = f(1)
  r =1   x →1 x →1

 1 RHL = lim+ f(x) = lim (4 − x) = 3


n (2n − 1)  2 − n  x →1 x →1

= 9 lim = 9 lim As LHL ≠ R.H.L, f is not continuous at x = 1


n →∞ (4n − 1)2 n →∞
 1
2

 4 − n  At x = 2
LHL = lim− f(x) = lim (4 − x) = 2 = f(2)
9×2 9 x →2 x →2
= =
16 8 RHL = lim+ f(x) = lim (1 + x) = 3
x →2 x →2

182. (i) f(x) = x + |2 - x|, -2 < x < 3. As LHL ≠ RHL, f is not continuous at x = 2
For - 2 < x ≤ - 1, x = -1, |2 - x| = 2 - x \ f is continuous everywhere in (-2, 3) except at
⇒ f(x) = -1 + 2 - x = 1 - x. x = ±1, 0, 2.
2.172  Differential Calculus

(iii) Graph of f(x) (ii) g(x) = f(|x|) + |f(x)|


−3x − 2, −4≤x<0
y  0,
 x=0
5 =
 0, 0<x≤3
4  2x − 6, 3< x≤4
y=1+x
3 Continuity of g(x) : Polynomials are continuous
y=1−x y=4−x
everywhere ⇒ g(x) is continuous everywhere
2 except perhaps at x = 0, 3.
y=2−x y=3−x
1 At x = 0
LHL = lim− g(x) = lim(−3x − 2) = −2
x1 x x →0 x →0
−4 −3 −2 −1 0 1 2 3 4 5
RHL = lim+ g(x) = 0
x →0

LHL ≠ RHL ⇒ g(x) is not continuous at


183. (i) Given x = 3t – |t|, y =e4t for all t x=0
For t < 0, x = 4t, y = ex. At x = 3
For t = 0, x = 0 , y = e = 1 0
LHL = lim− g(x) = 0
x →3
For t > 0, x = 2t, y = e2x
 e x RHL = lim+ g(x) = lim(2x − 6) = 0
, x<0 x →3 x →3
\ y = f(x) =  2x
e , x≥0 LHL = RHL = g(0) ⇒ g(x) is continuous at
x=3
(ii) Exponential functions are continuous every-
where. \ g(x) is continuous in [-4, 4] except at x = 0.
⇒ f(x) is continuous everywhere exceptpossibly at Differentiability:
x=0 −3, − 4 < x < 0

At x = 0 Now g’(x) =  0, 0 < x < 3
x  2 , 3<x<4
LHL = lim− f(x) = lim e = 1= f(0) 
x →0 x →0
⇒ g(x) is not differentiable at x = 0, since g is not
RHL = lim += f(x) = lim e2x = 1 continuous at 0
x →0 x →0

As LHL = RHL = f(0); x = 0 is a point of continuity of g’(3-) (=0) ≠ g’(3+) (= 2)


f(x) ⇒ f(x) is continuous everywhere. ⇒ g is not differentiable at x = 3
2x − 1, − 4 ≤ x ≤ 0 \ g (x) is differentiable at all points of [-4, 4] except
184. (i) Given f(x) =  x = 0, 3.
 x − 3, 0 < x ≤ 4
When - 4 ≤ x ≤ 4, 0 ≤ |x| ≤ 4 185. (i) + ve + ve + ve
− x − 3, − 4 ≤ x < 0 −3/2 − ve 0 1/2
− ve 1

\ f(|x|) =  −1 , x = 0
 x−3 , 0< x ≤ 4 p
 Consider f(x) = 24x5 - 70x3 + 45x2 +
4
−2x + 1, − 4 ≤ x < 0
 1 We have f ’(x) = 120x4 - 210x2 + 90x
, x=0
 = 30x (2x – 1) (x – 1) (2x + 3)
Also |f(x)| =  − x + 3, 0<x<3
 0, From the sign scheme of f ’(x) shown alongside, we
x=3
 can conclude that the shape of the curve y = f(x) will
 x − 3 ,3 < x ≤ 4 as depicted below.
Differential Calculus  2.173

187.
r

30 10 ft
−3 1 h
2 2
6

h1
 3  1
\ So in  −∞, −  ,  0, 2  and (1, ∞) f(x) is increas-
 2
Let r, h, v be the radius, height and volume of the solu-
 3  1  tion in the conical vessel at any time t, then
ing and in  − ,0  and  ,1 f(x) is decreasing.
 2  2  r
= tan30 (semi vertical angle be = 30°)
(ii) For 5 distinct real roots, the two maxima must h
be above the x-axis and the two minima below 1
⇒ r = h
the x-axis. 3
 3 1 1 2 ph 3
⇒ f  −  > 0, f(0) < 0, f   > 0, f(1) < 0 \ v = pr h =
 2 2 3 9
 3 dv ph 2 dh
f  −  > 0 ⇒ p > - 621 so that =
 2 dt 3 dt
f(0) < 0 ⇒ p < 0 Let h1, v1 be the height and volume of the solution
in the cylindrical beaker of radius 6 cm at time ‘t’.
1
f   > 0 ⇒ p > - 13 Now v1 = pr12h1 = 6ph1
2
dv 1 dh
f (1) < 0 ⇒ p < 4 ⇒ -13 < p < 0 so that = 6p 1
dt dt
\ p
 ∈ (-13, 0) if f(x) = 0 has 5 distinct real But flow out of conical vessel = flow into the beaker
roots.
dv dv 1 ph 2 dh dh
⇒ − = ⇒− = 6p 1
186. (i) Here f(x) = x + x + 1; g(x) = x - x - 1 so that
2 2
dt dt 3 dt dt
p(x) = f(g(x)) = x4 - 2x3 + x + 1 dh 2
When h = 6, = −2 inches/min = − ft/min
and q(x) = g(f(x)) = x4 + 2x3 + 2x2 + x - 1 dt 12
dh1
(ii) Given that dp = dq. \ = 4inches/min
dt
We have
\ The height of solution column increases at the
(4x3 - 6x2 + 1) dx = (4x3 + 6x2 + 4x + 1) dx rate of 4inches/min.
⇒ (12x2 + 4x) dx = 0 Aliter:
⇒ 4x (3x + 1) = 0 [Q dx is arbitrary] If y ft is the depth of water in the cylinder and x
ft that of water in the case, then
1
⇒ x = 0 or x = −
1   10   x  
2 2
3
( )
2
p 6 y = p  .10 −  x
1 3   3  
3  
hence when x = 0, − , the rate of change in f(g(x))
3 1
equals that in g(f(x))
y=
54
(103 − x 3 )
2.174  Differential Calculus

dy 1 dx (ii) For the inscribed circle of the hexagon


= − x2 a p
dt 18 dt Radius of the circle r = cot
2 6
1 2 2  dx −2  1 p
= .x .  =  so that dr = cot da
18 12  dt 12  2 6
\ when x = 6 When a = 120 3 c.m and da = 3 cm//h we have
dy 1 1
= × 62 = dr 3 3
dt 18 × 6 3 r = 180cm and = cm/h
dt 2
1 Now area of the inscribed circle is AI = pr2
Depth in cylinder increases at ft or 4 inches/
sec 3
so that dAI = 2prdr = 540 3p   (2)
(iii) For the circumscribed circle of the hexagon
188. Let r, h be the radius and volume of the cone. Then a p
radius of hemisphere is also r. Its radius R = cosec = a
2 6
Given that h = 2r
dR da
Volume of the balloon so that = 3; so, 3 cm/h
dt dt
= volume of cone Now the area of the circumscribed circle is
+ volume of hemisphere AC = p a2 ⇒ dAC = 2p ada = 720 3 p cm2/h
1 2 p
= pr 2 h + pr 3 = h 3 [\ h = 2r] (iv) Using (1), (2) and (3) we have
3 3 6
dAI : dAH : dAC = 3 3p :18 : 4 3p

r 190. (i) f satisfies f(xy) = f(x) + f(y), x, y > 0  (1)


r
Put x = y = 1 ⇒ f(1) = 2f(1) ⇒ f(1) = 0   (2)
H
h Let f be continuous at x= 1
⇒ f(1) = lim f(1 + h)   (3)
h→0

  h 
3h 3 Let x > 0; lim f(x + h) = lim f  x 1 +  
h→0   x 
But H = r + h = ⇒ dH = dh h→0
2 2
3   h 
p2  = lim  f(x) + f 1 +   using (1)
Now V = H  x 
6  3  
h→0

4 pH2 dV 4 pH2  h
so that dV = dH ⇒ = = f(x) + lim f 1 +  = f(x) + f(1)
h→0  x
27 dH 27
dV = f(x) [ using (2)]
When H = 18cm, = 48p
dH ⇒ f(x) is continuous for all positive x.
\ Rate of change in volume w.r.t the total height is
(ii) Given f is differentiable at x = 1,
48p cm3/cm = 48cm2
it is also continuous at x = 1
189. (i) The area of a regular hexagon of side ‘a’ is Now f(1) = f(1 × 1) = f(1) + f(1) using (1)
3 3 2
AH = a ⇒ f(1) = 0   (2)
2
The rate at which its area increases is given by Now
dAH = 3 3 a da f(x + h) − f(x)
f ’(x) = lim
da
h→0 h
When a = 120 3 cm , = 3cm/h   h 
dt f (x)1 +   − f(x)
da H   x 
\ = 3240cm2/h   (1) = lim
dt h→0 h
Differential Calculus  2.175

 h 195. y = sin–1 (cos x) + cos–1 (sinx)


f(x) + f 1 +  − f(x)
 x p  p 
= lim [using (1)] = sin-1 sin  − x  + cos −1 cos  − x 
h→0 h 2  2 
 h
f 1 +  − f(1) p 
 x =  − x  2 = p − 2x
= lim Q f(1) = 0 2 
h→0 x (h x )
1 f(1 + q) − f(1) dy
=lim ∴ =− 2 .
x q → 0 q dx
h
[take q = as h → 0, q → 0] cos log x
x 196. f(x) = sin log x ∴ f ' ( x ) =
1 x
= f '(1)
x  2x + 3 
cos log 
As f ’(1) exists, f ’(x) exists for all x > 0  2x + 3   3 − 2x  d  2x + 3 
f '  = × 
⇒ f(x) is differentiable for all x > 0  3 − 2x   2x + 3  dx  3 − 2x 
 3 − 2x 
Aliter:
From the definition of the function we see that   2x + 3    3 − 2x 
= cos log  ×
f(x) = log x, x > 0 clearly, log x is differentiable
  3 − 2x    2x + 3 
for all x > 0
 (3 − 2x ) 2 − (2x + 3)(−2) 
1 − x2   
191. u = cos-1 x = tan θ
(3 − 2x )
2
1 + x2  
= cos-1 cos 2q = 2 tan-1 x
2x  2x + 3 
12cos log 
v = tan −1 = tan −1 tan2q = 2tan −1 x  3 − 2x 
1 − x2 =
du 2 dv 2 dy 4 (2x + 3)(3 − 2x )
= 2
; = ; = .
dx 1 + x dx 1 + x dx 1 + x 2
2
At x = 1
192. Let sin x + y = y dy 12
= cos ( log 5)
sin x + y = y2 dx 5
dy dy
2y = + cos x 197. f (x + y) = f (x) . f (y)
dx dx
dy Put y = 0, f(x) = f(x) f(0) ⇒ f(0) = 1

dx
[2y − 1] = cos x f(x + h) − f(x) f(x) − f(h) − f(x)
dy cos x f ’(x) = lim = lim
= .
h→0 h h→0 h
dx 2y − 1 f(h) −1 f(h) − f(0)
dx f(x) = lim = f(x)lim
193. = 2cos2t h→0 h h→ 0 h
dt = f(x)f '(0) = 2f(x)
dy 1
= or, from the definition of the function, f(x) = ekx since
dt t
dy 1 sec2t f '(0) = 2, k = 2 ⇒ f '(x) = 2f(x)
= = .
dx 2t cos2t 2t cos3x  0 
198. = lim form 
x
194. u = 2e v = log x x→
p cos7x  0 
2

dv dv 1 By L’Hospital’s rule
= 2e x =
dx dx x 3sin3x 3
du = lim =
\ = 2x e x . x→
p 7sin7x 7
dv 2
2.176  Differential Calculus

6a x 2 + ax + a 2 − x 2 + a 2 y 2 = x 2 − h 2 or y = x 2 − h 2
199. lim 1
x→∞
 1 dy xdx
a  6x + 2a −  − xe x =
 ax  dt x − h dt
2 2

a a2 a2 dx v x 2 − h 2
6a 1 + + 2 − 1+ 2 = .
x x x dt x
= lim 1
x→∞
 2a 1 
a 6 + − 2  − e x 204. Let s represent the speed
 x ax 
1
(Dividing numerator and denominator by x), s = kx 2 log   = − kx 2 log x
x
6a − 1
= = 1. ds
6a − 1 = − k {x + 2x log x}
dx
200. Limit d2s
= − k [1 + 2 + 2log x ]
( 1 + sin x + )
x 2
(13 − 1) x dx 2
= lim × × 2
1 − sin2 x
x →0 x 2sin2 x
ds −1
= 0 gives x = 0 or log x =
= log 13. dx 2
x 3 − (k + 4)x + 2k (x = 0 is not admissible)
201. Given f(x) =
x−3 −1 d 2 s
For log x = , <0
f(x) is continuous at x = 3 2 dx 2
−1
x 3 − (k + 4)x + 2k 1
⇒ maximum speed is when x = e 2 =
⇒ lim = f(3) = 8 e
x →3 x−3
⇒ x - 3 is a factor of x3 - (k + 4) x + 2k
 1 − tan3 x p p
 , ≤ x<
⇒ 15 - k = 0 ⇒ k = 15.  3 2 (cos x − sin x ) 6 4
 p
202. f (9) = 9; f1 (9) = 4 205. f(x) =  p , x=
 4
f 1 (x )
f (x ) − 3 2 f (x ) 
(
 q 1 − 2 sin x
,
) p
<x<
p
lim = lim  cos2x 4 3
x →9
x −3 x →9 1
2 x p
is continuous at x =
4
f 1 (x ) 4×3
= lim . x = = 4. p
x →9
f (x ) 3 ⇒ f   = lim− f(x) = lim+ f(x)
 4  x→ p p
x→
4 4
203. K y Now we have
P
1 − tan3 x
h lim− f(x) = lim
x x→
p x→
p
4
3 2 (cos x − sin x)
4

M (1 − tan x) (1 + tan x + tan2 x)


1
= lim
P is the position of the kite at time t 3 2 x→
p cos x (1 − tan x)
4
dy
Let KP = y given =v 1 3
dt = =1
dx 3 2 1
To find we have 2
dt
Differential Calculus  2.177

q (1 − 2 sin x)  p sec2 x
lim+ f(x) = lim sin  x −  × +
p x→
p cos2x  4  (1 + tan x)
x→ 4
4
 p  1 + tan x 
cos  x −  log   − 4
1 − 2 sin x 1 q  4   2 −4
= q lim = q lim = = lim = which
p 2
1 − 2sin x x → 1 + 2 sin x
p 2 x→
p  p   p  0
x→ 4 4 sin x − + (4x − p)cos  x − 
4 4  4   4
q does not exist
\ p = = 1 ⇒ p = 1, q = 2.
2
log(tan x cot a) 0 
209. L = lim  0 form 
206. The graph of the function f(x) = min (1 – x, 2), x →a log(cos a sec x)
x ∈ R is given below.
1
sec2 x
y = 1–x = lim tan x
x →a 1
sec x tan x
(−1, 2) y=2 sec x
2 (using L ‘Hospitals rule) = cosec2a.

1 x f(1) − f(x)
210. Given 5 = lim
x →1 x −1
x
−3 −2 −1 0 1 2 3 x f(1) − f(1) + f(1) − f(x)
= lim
(1, 0) x −1
x →1

(adding and subtracting f(1)


f(1)(x − 1) −  f(x) − f(1)
So f(x) is continuous for all x ∈R but not differentiable = lim
x →1 x −1
at x = -1.
f(x) − f(1)
2 2
= f(1) − lim
x − p+ x − p 0  x −1
x →1
207. L = lim  0 form 
x→p
x−p = f(1) - f ’(1) = 10 - f ’(1) [Q f(1) = 10]
⇒ f ’(1) = 5.
1 x
+
2 x x 2 − p2 211. Given a continuous function
= lim , by L’ Hospital’s rule
x→p 1  p
 cos x x<
2 x−p 2
f(x) = 
px + q p
x≥
 x−p 2x  2p  2
= lim  + = = 2p
x→p
 x x+p 2p p
f(x) is continuous, in particular, at x =
2
  1 + tan x   ⇒ lim− f(x) = lim+ f(x)
 log  2  1  x→
p
2
x→
p
2
208. L = limp  − 
x→  4x − p  p p p p 2
4
 sin  x −   ⇒ cos =p + q ⇒ =−
  4  2 2 q p
 p (1 + tan x)
sin  x −  log − (4x − p) sin3x − 3sin x 0 
 4  2 0  212. L = lim
= lim  0 form   0 form  ,
x→
p  p
x →0 cos x − cos2 x
4 (4x − p) sin  x − 
 4 Using L Hospital’s rule
2.178  Differential Calculus

3cos3x − 3cos x 0   1 + tan x, tan x ≥ 0


L = lim  0 form  Now f(x) =  except when
x →0 − sin x + 2sin x cos x  1 − tan x, tan x < 0
−9sin3x + 3sin x (2n + 1)π
= lim x= ,n ∈Z
x →0 − cos x + 2cos2x 2
(using L Hospital’s rule) = 0 sec2 x , x > 0 (2n + 1) p
f ’(x) =  except at x =
213. Given f(x) = sec–1 (cosecx).
2
− sec x, x < 0 2
Differentiating w.r.t. x, we have ⇒ f(x) is not differentiable at x = np, n ∈ Z
1 1 ⇒ f(x) is continuous at all points except
f ’(x) = (-cosec x cot x)
| cosecx | cosec2 x − 1
(2n + 1)p
x = , n ∈ Z and is differentiable at all
− cosecx cot x 2
= points except at x =, np . ,n ∈ Z
| cosecx cot x |
If cot x = 0, f ’(x) is not defined. This means that f(x) 215. Given cos y = x. cos (a – y) we have
(2n + 1) p  xsin a 
y = cot-1 
is not differentiable at x = , n ∈Z
2  1 − x cos a 
Differentiating w.r.t. x
OR
dy −1
f(x) = cos-1(sinx) = cos-1 = 2
dx  xsin a 
1+ 
  p   1 − x cos a 
cos  2 − x  
  (1 − x cos a)sin a + xsin a.(cos a)

 p − x, 0 < x < p (1 − x cos a)2


 2 2 − sin a
 p p 3 p =
= x − , ≤ x< 1 − (2cos a)x + x 2
2 2 2

5 p 3 p
 2 − x, 2 ≤ x < 2 p
216. Given x = sin q, y = cos3q we have
Clearly, f(x) is continuous in [0, 2p] but not differen- dx dy dy −3
= cos q and = 3cos2q (-sin q) so that =
3p dq dq dx 2
tiable at x = p ,
2 2 sin 2q.
Since f(x) is periodic f(x) is not differentiable at d2 y d  dy  dq 3cos2q
\ =   . = − cos q
(2n + 1) p dx 2 dq  dx  dx
x= , n ∈ Z.
2 2
d2 y  dy 
214. Let f(x) = 1 + |u(x)| where, u (x) = tanx. \ 2y 2 + 4   = 6 cos2q (7 sin2q - cos2q).
dx  dx 
u(x) is defined at all points except when cos x = 0 (i.e.)
(2n + 1)p 217. Given y = cos x + cos x + .... we have
when x = , n ∈ Z.
2
y= cos x + y ⇒ y 2 − y = cos x
⇒ |u(x)| is defined at all x except when
Differentiating with respect to x,
(2n + 1)p
x= , n ∈ Z. sin x
2 2yy1 - y1 = - sinx ⇒ y1 =
1 − 2y
⇒ 1 + |tanx| is not continuous at
(2n + 1)p 218. Given y = 4x3 – 2x5  (1)
x= , n∈Z.
2 We have y’ = 12x2 – 10x4
Differential Calculus  2.179

Equation of the tangent at (x1, y1) is p p


f(x) has a minimum at x = ⇒ f '  = 0
y – y1 = (12x12 - 10x14) (x – x1) 6 6
This tangent passes through (0, 0) ⇒ p = 2.
⇒ -y1 = -x1 (12x1 – 10x1 ) 2 4
p
We also note that f "   when p = 2 is > 0
⇒ 8x1 (1 - x1 ) = 0
3 2 6
⇒ x1­= 0, ± 1 ⇒ y1 = 0, 2 or - 2 223. Given f(x) = a0 + a1x2 + a2x4 + ….+anx2n. we have
The points are (0, 0), (1, 2), (-1, -2) f ’(x) = 2a1x + 4a2x3 + …..+ 2n anx2n - 1
\ We have points other than origin = 2x(a1 + 2a2 x2 + …..+ nanx2n – 2)

x2 x f ’(x) = 0 ⇒ x = 0 [Q a1 + 2a2x2 + …
219. Given y = we have y’ =
4 2 + nanx2n – 2 < 0 ∀ x. as ai’s are negative]
At x = 2, m1 = 1 and at x = -2, m2 = -1
Now f "(x) x = 0 = 2a1 + 12a 2 x 2 + .... + 2n(2n − 1)a n x 2n − 2
m1m2 = -1 x =0

= 2a1 < 0
⇒ Two tangents are perpendicular to each other.
⇒ x = 0 corresponds to a point of maximum
220. f(x) = (x – a)m (x – b)n, m, n∈N, m ≠ n and a < b n

( )
2
f ’(x) = (x - a)m - 1 (x - b)n - 1 × [(m + n)x - (na + mb)] 224. Given f(x) = ∑ x − r 2 we have
r =1
f ”(x) = (x - a) m-1
(x - b) (m + n)
n-1
n
+ [(m + n)x - (na + mb)] f ’(x) = ∑ 2(x − r
r =1
2
)
× (x -a) (y - b)
m -2 n -2
n
× [(m + n - 2)x - (na + mb)] f '(x) x =11
= 0 ⇒ ∑ (11 − r 2 ) = 0
r =1
Clearly, f ”(a) = 0
n(n + 1) (2n + 1)
If (m - 1) is even and (n - 1) is odd, ⇒ 11n − =0
6
f ’(a-) = (+)(-)(-) = +ve
−13
f ’(a+) = (+)(-)(-) = + ve ⇒ n = 0, 5 or
2
f ’(x) does not change sign as x crosses a. But n cannot be negative nor zero⇒ n = 5
1
221. +ve +ve  sin x  x
225. L = lim 
x →0  x 
 (1 form)

−1 1 2
 sin x 
log 
Given y = 3x4 - 8x3 - 6x2 + 25x + 5  x   0 
⇒ log L = lim  form 
⇒ dy = (12x3 -24x2 - 12x + 25)dx x →0 x  0 
dy > dx ⇒ 12x2 - 24x2 - 12x + 25 > 1 1
cot x −
⇒ x - 2x – x + 2 > 0
3 2
= lim x (∞ - ∞ form)
x →0 1
⇒ (x + 1) (x – 1) (x – 2) > 0
(using L Hospital’s rule)
The sign scheme for the above expression is given
 x cos x − sin x   0 
below. = lim   form 
x →0  xsin x  
 0 
⇒ x ∈ (2, ∞) ∪ (-1, 1)
 − xsin x + cos x − cos x   0 
222. Given f(x) = p(6 cos x – 3 cos 2x – 2 cos 3x) - 12 sin = lim    0 form 
x →0  sin x + x cos x
x - 6 sin 2x
We have − sin x − x cos x 0
= lim = =0
f '(x) = p(- 6 sin x + 6 sin 2x + 6 sin 3x) - 12 cos cos x + cos x − xsin x 2
x →0

x - 12 cos 2x ⇒ L = e = 1
0
2.180  Differential Calculus

2 Aliter:
x + x 4 + x 9 + ... + x n − n
226. L = lim
x →1 x −1 as x → ∞ x , x … are negligible in comparison

= lim
(x − 1) + (x 4 − 1) + ... + x n − 1 ( 2

) with x
x →1 x −1 x 1
So lim = lim =
x →∞
4x 2
 xn − an 
= 1 + 4 + 9 + …+ n2 ∴ lim = na n −1 
 x →a x −a  n
k2 1 n 2
n (n + 1) (2n + 1)
229. Sn = ∑1+ n
k =1
3
= ∑k
1 + n 3 k =1
= 12+22+32…n2 =
6
1 n(n + 1) (2n + 1) n(2n + 1)
= 3
=
1 1+ n 6 6(n2 − n + 1)
 cos x  x −a
227. Let y = lim  
x → a  cos a 
.
1
2+
1 n
Then we have log y lim Sn = lim
n →∞ 6 n →∞ 1 1
1   cos x    0  1− +
= lim n n2
x→a x − a
 log  cos a    0 form 
 
1 2 1
= × =
 − tan x  6 1 3
= lim  = -tan a
x→a  1 
 sin ([x] + x )
⇒ L = e-tan a  , x≠0
230. f(x) =  [x] + x , ([x] denotes the greatest
1 , x = 0

 1 
228. Consider 2x + x = x  2 +  integer less than or equal to x)
 x
f(x) is defined as
 1   sin(x − 1)
= x 2 +   for − 1 ≤ x < .0
 x
 x −1
f(x) = 1 x=0
1  sin x
⇒ 3x + 2x + x = 3x + x. 2 +
x  0 < x <1
 x
1 1 sin(x − 1) sin(−1)
= x. 3 + 2+ Now  lim− f(x) = lim = = sin1 and
x x x→0 x→0 x −1 −1
f(10 + h) − f(10)
lim sin x
h→0 h lim f (x) = lim =1
x → 0+ x→0 x
  h 
f 10 1 +   − f [10 × 1] As lim f (x) ≠ lim+ f(x), lim f(x) does not exist.
  10  x → 0− x→0 x→0
= lim
h→0 h
231. Given x2 + 4x + 3 + |y| = 3y
x Let y > 0 then x2 + 4x + 3 + y = 3y
\ lim
x →∞
4x + 3x + 2x + x ⇒ x2 + 4x + 3 = 2y > 0
⇒ (x + 1) (x + 3) > 0
1 1
= lim = ⇒ x > -1 or x < -3
x →∞ 2
1 1 1 x 2 + 4x + 3
4+ 3+ 2+
x x x ⇒ y = if x < -3 or x > -1.
2
Differential Calculus  2.181

Let y < 0.  1  1 


Then x2 + 4x + 3 - y = 3y     −  
1 1 2  2
⇒ (x + 1) (x + 3) < 0 = lim 
x→0 x 
1 + (2x) +
2 2!
( 
)
2x)2 + .... −

⇒ (x.<-1 and . x > -3) or (x > -1 and x < -3) 
  

⇒ x < -1 and x. > -3
 1  1   −2  
x 2 + 4x + 3   3   3   3   
when -3 < x < - 1, y =
4 1 + (2x) + (2x)2 + ...  provided
 1! 2! 
Also when y = 0, x = -1. or -3
 
 
 x 2 + 4x + 3
 , x < −3 1
 2 |x| <
0 , x = −3 2
 x + 4x + 3
2
[Q (1 + x)n =
\ y =  , − 3 < x < −1
 4 nx n(n − 1) 2 n(n − 1)(n − 2) 3
0 x = −1 1+ + x + x + .... provided
1! 2! 3!
 x 2 + 4x + 3
 , x > −1 |x| < 1, n any rational number (Binomial Theorem)].
 2
(i) y as a function of x is defined for all real x. 1   1 1  (2x)2  1 2  
= lim 2x − + − + + ...
x→0 x   2 3  2!  4 9  
(ii) y is continuous everywhere except possibly at
x = -3, -1
 1 (−1)  1
Since x2 + 4x + 3 = 0 when x = -1 or -3 = lim .2 × + 2x × + .... =
x→0
 6 36  3
we get f(x) is continuous at x = -1, -3.
Hence f is continuous everywhere. 233.
 x + 2 , x < −3
x + 2 1

(iii) we have y’ = f ’(x) =  , −3 < x < −1
π
 2 −
π π π
 x + 2 , x > −1 2 −1 2
−1
we have f ’(-3-) = -1 and f ’ (-3+) = .
2 f(x) = |x| sin x. We can rewrite f(x) as
As f ’ (-3-) ≠ f ’ (-3+) we say that at x = -3, y is not − xsin x, x < 0
differentiable. 
f(x) =  0 , x=0
1  xsin x , x > 0
f ’ (-1-) (= + ) ≠ . f ’ (-1+) (=1) ⇒ at x = -1, y 
2
As polynomials and sine functions are differentiable
is not differentiable. f(x) is differentiable at all points except perhaps at
⇒ y is differentiable at all points except at x = x = 0. At x = 0, we have
-1, -3. −(sin x + x cos x) , x < 0
f '(x) = 
1 + 2x − 3 1 + 2x  (sin x + x cos x) , x > 0
232. Given f(x) =
x \ f ’(0-) = 0
f(x) is continuous everywhere except perhaps at f ’(0+) = 0
x = 0, we must have
As f ’(0-) = f ’(0+) we have x = 0 as a point of
1 + 2x − 3 1 + 2x differentiability
f(0) = lim
x→0 x ⇒ f(x) is differentiable everywhere.
2.182  Differential Calculus

 ex − e− x 1 + y 2x
 p 235. Let y = f(x) = ⇒ =e
p x
e +e −x
1− y
(1+ | cos x |)|cos x| , 0 < x <
 2 (using componendo and devidendo rule)
 p
234. f(x) =  q ,x = 1 1 + y 
 2 ⇒ x = log 

  p 
cot l  x −  
2 
2  1 − y 

   p 
cot m  x −  
 2  p 1 1 + x 
 e , <x<p \ The inverse of f(x) is g(x) = log 
 2 2  1 − x 
is continuous on (0, p) 1 + x  1+ x
The function log  is defined only if >0
p  1 − x  1− x
⇒ f(x) is continuous at x =
2 (i.e.,) if (1 + x > 0 and 1 - x > 0) or
p (1 + x < 0 and 1 - x < 0)
⇒ f   = lim− f(x)
 2  x→ p (i.e.,) if (x > -1 and x < 1) or
2
(x < - 1 and x > 1)
p
(i.e.,) if -1 < x < 1 or 1<x and x <-1
We have q = lim (1+ | cos x |)
|cos x|


x→
p
2
There is no x satisfying the two conditions x < -1
and x > 1.
p
= lim (1 + cos x) cos x Therefore, g(x) is defined only in (-1, 1)
p
x→
2 \ set of points of discontinuity of g(x) = R-(-1, 1)
[Q In the first quadrant cosx > 0]
 x 3 , x ≤ x0
p 236. f (x) =  2
  1

= lim  (1 + y)  y px + qx + r , x > x 0


y →0
  is continuous at x = x 0
[Taking y = cosx we have as ⇒ Lim− f(x) = Lim+ f(x) = f(x 0 )
x → x0 x → x0
p
x→ , y → 0] = ep.
2 Now, f(x 0 ) = x 30
  p 
 cot l  x −  
LHL = Lim− f(x) = lim x 3 = x 30
 2  x → x0 x → x0
  p
 cot m x −
 2  
Also, q = lim e 
RH Lim+ f(x) = lim px 2 + qx + r = px 20 + qx 0 + r
p x → x0 x → x0
x→
2

  p  so that we have x 30 = px 20 + qx 0 + r   (1)


 cot l  x −  
 2 

 cot m
 p
x − f(x) is differentiable at x = x 0
 2  
⇒ ep = lim e 

x→
p
2
( )
⇒ f 1 x 0+ = f 1 x 0− ( )
 3x
2
 p  p  p x < x0
cot l  x −  × l  x −  × m  x −  We have f ’(x) = 
 2  2  2 2px + q x > x 0
⇒ p = lim
x→
p  p  p  p
2 l x− cot m  x −  × m  x −  f '(x 0 − ) = lim 3x 2 = 3x 02
 2   2  2 x → x0

+
m  f '(x 0 ) = lim 2px + q = 2px 0 + q
= Q lim q cot q = 1 x → x0
l  q→ 0 
⇒ 3x0 = 2px0 + q 
2
 (2)
m
m Roots of px2 + qx + r = 0 are reciprocals of
\ p = and q = e l
l each other
Differential Calculus  2.183

r
⇒ = 1 ⇒ r = p  (3) 2 3 , x ∈[3,6)
p 
=  2. 3x − 9
Using (3) in (1) and (2) we have  , x ∈ [6, ∞)
 3
2px0 + q = 3x 20   (4)
2 3 , x ∈[3,6]
px 20 + qx 0 + p = x 30   (5) =

2 x − 3 , x ∈ (6, ∞)
2x 30
(4) × x0  (5) gives p = 1
x 20 − 1 f '(6−) = 0, f '(6+) =
3
Substituting in (4) we have q =
(
x 20 3 + x 20 ) As f ’(6-) ≠ f ’(6+), f ’(6) does not exist
2
1− x 0
 3
1
 (p + q)n + n +
2
g(x)f(p) − g(p) f(x) 
237. Given 0 = lim 240. Lim f  2
3
x→p x−p n →∞
 2n 2 + 2n + 2 
 
g(x)f(p) − f(p)g(p) + f(p)g(p) − g(p)f(x)
= lim
x→p x−p  3
1
 (p + q)n 2 + n + 
2
f(p) (g(x) − g(p)) − g(p) (f(x) −f(p)) = f Lim 3
= lim  n →∞ 2n 2 + 2n + 2 
x→p x−p  
(adding and subtracting f(p) g(p)) [Q f is continuous]
 g(x) − g(p)   f(x) − f(p)   1 1 
= f(p) lim   − g(p)lim   (p + q) + + 3 
x→p  x−p  x→p  x − p   n 
= f Lim 2n 2
= f(p) g’(p) - g(p) f ’(p) = 2g’(p) - 6g(p) n →∞ 2 2 
 2+ + 3 
⇒ g’(p) : g(p) = 3 : 1.  n 
n2
3
238. We note that f(x) has to be of the form k log x (dividing numerator and denominator by n 2 )
k
f ’(x) = . p+q p−q  p+q p−q
x = f = Q f  2  = 2 
 2  2
f ’(1) = 1 ⇒ k = 1
1
Therefore, f ’(x) = 241. f(x) = x4 – 4x3 + 2x2 – 3x + 5
x
f ’(x) = 4x3 - 12x2 + 4x - 3
f ’(10) = 0.1
f(1) = 1 - 4 + 2 - 3 + 5 > 0
239. Domain of
f(2) = 16 - 32 + 8 - 6 + 5 < 0
f(x) = x + 2 3x − 9 + x − 2 3x − 9 is [3, ∞). f(3) = 8 - 108 + 18 - 9 + 5 < 0

Put t = 3x − 9 . Then t ∈ [0, ∞) and f(4) = 44 - 44 + 2.16 - 3.4 + 5 > 0 ⇒ one root is in (1, 2)
one root is in (3, 4)
t2 t2
g(t) = f(x (t)) = + 3 + 2t + + 3 − 2t
3 3
| t + 3 |+ | t − 3 | y = f(x)
=
3
2α3
2 3 0 ≤ t < 3 0 1
 4
=  2t
 t ≥ 3.
 3
2.184  Differential Calculus

f ’(x) = 4x3 - 12x2 + 4x - 3


 1 3 
if x < 0 ⇒ f ’(x) < 0 - (sinx)  sin x + cos x 
 2 2 
x = 1 ⇒ f ’(x) = 4 - 12 + 4 - 3 = -7 < 0
1 2 3 1
x = 2 f ’(x) = 32 - 48 + 8 - 3 = -11 < 0 = sin2x + sin x + cos2 x − sin2 x
4 4 2
x = 3 f ’(x) = 108 - 108 + 12 - 3 > 0
3
x = 4 f ’(x) = 256 - 192 + 16 - 3 > 0 =
4
(
sin2 x + cos2 x )
if x > 4 ⇒ 4x3 > 2x2 , 4x > 3 ⇒ f ’(x) > 0
3
\ f(x) is decreasing in (-∞, 2 + h) = = a constont
4
and f(x) is increasing in (2 + h, ∞)
3
⇒ f(x) has min at x = a, where [a] = 2. ⇒ g o f(x) = g   = 8 for all x
4
⇒ only 2 real roots are possible
244. Let h(x) = [g o f ](x)
1
242. [3x2+1] = 1 0<x< = 8 + f(x) - (f(x))3 + (f(x))5
3
Then h’(x) = f ’(x) [1 - 3(f(x))2 + 5 (f(x))4]
1 2
=2 ≤ x<  11 
2
3 3 3
= 5f ’(x)   ( f(x)) −  +
2

2   10  100 
=3 ≤ x <1 ⇒ h’(x) and f ’(x) have the same sign ⇒ h(x) and
3
f(x) behave alike.
\ The function is continuous in (0, 1) except at two
1 2 245. Given y = sin(x – y)
points , .
3 3 We have y’ = (cos (x - y)) (1 - y’)
1
 p  p Tangent is parallel to x - 2y = 0 ⇒ y’ =
243. Given f(x) = sin2x + sin2  x +  − sin xsin  x +  , 2
 3  3
1 1
we have on differentiating with respect to x ⇒ = cos (x - y) ⇒ cos(x - y) = 1
2 2
 p  p ⇒ sin(x - y) = 0 and (x - y) = 0, ± 2p,
f1(x) = 2sin x cos x + 2sin  x +  cos  x +  -
 3   3 ⇒ y = 0 ⇒ x = 0, ± 2p (Q -2p ≤ x ≤ 2p)
 p  p \ At (0, 0), (± 2p, 0) the tangents are parallel to
sin x. cos  x +  - cos x sin  x +  x - 2y = 0
 3  3
\ 3 tangents
 2p   p
= sin 2x + sin  2x +  - sin  2x + 
 3   3 246. Given y = (x – p) (x – q) (x – r)
 p p  p We have
= 2 sin  2x +  cos   − sin  2x +  = 0 y’ = (x – p) (x – q) + (x – q) (x – r) + (x – p) (x – r)
 3 3  3
⇒ f(x) = k (k being a constant) At x = p; y’ = (p – q) (p – r)
3 3 3 At x = q; y’ = (q – p) (q – r)
But f(0) = 0 + − 0 = ⇒ f(x) = ∀x
4 4 4 Tangents at p and q are parallel
3 ⇒ (p – q) (p – r)
\ (gof)(x) = g ( f(x)) = g   = 8.
4 = (q – p) (q – r) (i.e.,) (p - q) (p + q - 2r) = 0
OR ⇒ As p ≠ q, 2r = p + q ⇒ p, r, q are in AP
2
1 3 
f(x) = sin2x +  sin x + cos x  247. ax2 + bx + c > 0
2 2 
where, a > 0 ⇒ b2 - 4ac < 0   (1)
Differential Calculus  2.185

Consider, p
x−
g(x) = 2ax3 + 3(b – 2a)x2 + 6(2a – b + c) x + 30 2
Given f(x) =
We have, g’(x) = 6[ax2 + (b – 2a) x + (2a – b + c)]  p
1 −  x −  cot x
 2
Consider the discriminant
2
= (b – 2a)2 - 4a(2a - b + c)]  p
sin2 x −  x − 
 2
= b2 – 4ac – 4a2 < 0 [using (1)] We have f '(x) = 2
  p 
⇒ g’(x) > 0 (g’(x) is a quadratic expression with coef- sin2 x 1 −  x −  cot x 
ficient of x2 positive and negative discriminant)   2 
⇒ g(x) is monotonic increasing ∀ x ∈ R The extrema, if any are given by f ’(x) = 0
2
 p
248. Let OA be the vertical wall i.e., by sin2 x =  x − 
A  2
B be the horizontal floor
30 p
AB be the ladder y or sin x = ±x -
2
Let OA = ym and OB = xm We observe that the two graphs intersect at two
O x B
Given AB = 30 m points x1 and x2 in (0, p) that are on either side
of x = p/2 equidistant from x = p/ 2. Let x1 < x2
Now x2 + y2 = 900   (1) without loss of generality. Then f ’(x 1- ) < 0 and
dx dy f ’(x1+) > 0
⇒ x +y =0   (2)
dt dt ⇒ x1 is a minimum point.
dy dx −4 Also f ’(x2-) > 0 and f ’(x2+) < 0
Given : = we have
dt dt 3 ⇒ x2 is a maximum point.

 dy 251. Given, f(x) = 3 cos x – 2 cos3 x


 = −4k (A slips down) We have,
⇒  dt
 dx = 3k  1  1 
 dt f ’(x) = (6 sin x)  cos x −   cos x + 
 2 2
Using in (2) we have 4y = 3x
1
Using in (1) we have x = 24 m f ’(x) = 0 ⇒ sin x = 0 or cos x = ±
2
249. g’(x) = [1 – 2f(x) + 3(f(x))2]f ’(x)  p p
But, cos x > 0 in  − , 
f ’(x) = 3x + 10x + 9
2  2 2
= [‘+’ ve]f ’(x) f ’(x) > 0 ∀ x ∈ R 1 p
⇒ sin x = 0 or cos x = ⇒ x = 0, ±
⇒ g’(x) > 0 ∀ x ∈ R 2 4
⇒ g(x) is always increasing Now, f ’’(x) = 3 sin x (-2 sin 2x) + 3 cos 2x cos x
At x = 0 f ’’(x) > 0 ⇒ x = 0 is a minimum point and
250. y minimum value = 1
π p p
π At x = ± f ’’(x) < 0 ⇒ x = ± are maximum points
y = −x + 2 y = x– 2 4 4
and maximum value = 2
 p +
 p− 
0 x1 x2 x f  −  = 0 and f   = 0
y = sin x  2  2 
π
x= 2 ⇒ only greatest value exists.
2.186  Differential Calculus

252. For x ≤ 1, f ’(x) = –10 + 2x – 3x2 ⇒ f2n(x) = f (f (f (…f(x), (2n times) is continuous
 1  29 
2
3
= −3   x −  +  < 0 everywhere except at x = .
4
  3 9 
⇒ f is decreasing for x ≤ 1 255. Differentiating the given relation with respect to x,
For x > 1, f ’(x) = 3 > 0 ⇒ f is increasing for x > 1 dy dy
2x + x + y + 6y =0
Function has least value at x = 1 dx dx
If f ’(1+) ≥ f(1) dy
⇒ (x + 6y) = - (2x + y)   (1)
If 3+ log10 (p - 4) ≥ 4
2 dx
If p2 - 4 ≥ 10 Differentiating (1) with respect to x,
If p2 ≥ 14 d2 y  dy  dy  dy 
(x + 6y) + 1 + 6  = − 2 + 
For log10 (p2 - 4) to be defined p2 > 4 dx 2  dx  dx  dx 

⇒ 4 < p ≤ 14
2
d2 y  dy 
2
dy
⇒ (x + 6y) = −6   − 2 − 2
⇒ − 14 ≤ p < −2 or 2 < p ≤ 14 dx 2  dx  dx
2
253. Given  2x + y   2x + y 
= −6   − 2 + 2 ,
 x x
f(x) = (p2 − 5p + 6)  cos 4 − sin 4  + (p − 3)x + k  x + 6y   x + 6y 
 4 4
dy
substituting for from (1)
x dx
= (p2 − 5p + 6)cos + (p − 3)x + k
2
−6(2x + y)2 − 2(x + 6y)2 + 2(x + 6y) (2x + y)
We have, =
(x + 6y)2
 p − 2  x 
f '(x) = (p − 3) 1 − sin    −22(x 2 + xy + 3y 2 ) −22
 2  2  = = ,
(x + 6y)2 (x + 6y)2
Function does not have critical points
⇒ f ’(x) ≠ 0 for any real x, since f ’ exists everywhere since x2 + xy+ 3y2 = 1(given)
 p−2 x
256. Let left hand limit (LHL) = L
⇒ (p − 3) 1 − sin  ≠ 0 for any real x.
 2 2 
 p
p−2 x 1 − cos 4  x − 
⇒ p ≠ 3 and 1 = sin has no solution in R.  2
2 2 = lim
p− p
2
x→
2 x−
⇒ p ≠ 3 and >1 2
p−2
 p
⇒ p ≠ 3 and 0 < p < 4 2.sin2 2  x − 
 2
= lim
254. − ve +ve −ve p− p
x→
2 x−
−3 −1 2
| sin2(−h) |
3x + 2 = 2 .lim
Given f(x) = h→ 0 −h
4x − 3
3 p
f(x) is continuous everywhere except at x = , where where, h = −x
4 2
it is not defined.
3 sin 2h  sin q 
If x ≠ , u(x) = f(f(x))= x and u(x) is continuous = − 2 .lim = 1
4 h→ 0 h Qqlim
→0 q 
3
everywhere except at x =
4 Right hand limit (RHL) = R
Differential Calculus  2.187

 p 3m 2 + 2m(m + 1)(2m + 1)
1 − cos 4  x −  = + (2m + 1)
 2 3
= lim
p+ p 3m 2 + 6m + 3 + 2m (m + 1) (2m + 1)
x→
2 x− =
2 3
| sin2h | 2
3(m + 1) + 2m(m + 1)(2m + 1)
= 2 .lim =  — (2)
h→ 0 h 3
sin 2h  sin q  n
= 2 .lim = 2 2 Qlim = 1 If n is even, replace m by in (1)
h→ 0 h  q → 0 q  2
n −1
 p If n is odd, replace m by in (2)
1 − cos 4  x −  2
p  2
As L ≠ R at x = , we say that lim  3n2 + 2n(n + 1)(n + 2)
2 p p
x→
2 x−  if n is even,
2 ∴ Sn =  12
does not exist. 2
 3(n + 1) + 2(n − 1)n(n + 1) if n is odd.
257. For k = 1, 2, …n we have  12
t1 = tan-1 2.3 - tan-1 1.2 S2n
lim
t2 = tan-1 3.4 - tan-1 2.3 n →∞ S2n +1

………………………….. 3n2 + 2n(n + 1) (2n + 1)
tn = tan (n + 1) (n + 2) - tan n (n + 1)
-1 -1 = lim
n →∞ 3(n + 1)2 + 2n(n + 1)(2n + 1)

n

∑t k = tan-1 (n + 1) (n + 2) - tan-1 2. 3  1  1
+ 2 1 +   2 + 
k =1 n  n  n 4
= lim 2
= =1
n
n →∞
 1  1  1  1 4
Tn = lim ∑ t k 3 1 +  + 2 1 +   2 + 
258. nlim
→∞ n→∞
 n n  n n
k =1

= lim  tan −1 (n + 1)(n + 2) − tan −1 2 260. f ’(x) = 12x2 – 18x – 30


n→∞
= 6 (2x2 - 3x - 5) = 6 (2x - 5) (x + 1)
p 5
= − tan −1 2 = cot −1 2 f ’(x) = 0 at x = -1 and x =
2 2
5
259. (i) The given series can be rewritten as f(x) is increasing in (-∞, -1) and decreasing in ( , ∞).
2
(1 + 3 + 5 + …) + (22 + 42 + …) Since 2x2 - x - 6 ≤ 0, we have to examine the values of
S2m = [1 + 3 + …+(2m - 1)]  3 
+ [22 + 42 + …+ (2m)2] f(x) in the interval  − , 2 only.
 2 
m  3
= [1 + 2m -1] + 22 [12 + 22 + …+ m2] f  −  > 0 and f(2) = 32 - 36 - 60 + 12 = -52
2  2
m m(m + 1)(2m + 1)
= × 2m + 22 × 261. (i) Given the curve,
2 6
a
3m 2 + 2m(m + 1)(2m + 1) x = a cos q − cos2q
=  — (1) 2
3 a
S2m+1 = [1 + 3 + …+(2m + 1)] y = a sin q − sin2q
2
+ [22 + 42 + …+ (2m)2] Distance ‘l’ between a point on the given curve and
= S2m + (2m + 1) (2a, 0) is given by l2
2.188  Differential Calculus

 cos2q  
2
sin2q  
2 x
2 2  263. Consider f(x) = , x>0
l = a  cos q − − 2 +
  sin q − 
  2 2   x 4 + 1875
3(25 + x 2 ) (25 − x 2 )
d 2 5  Now f ’(x) =
(l ) = 2a 2  sin q − 4sin q cos q (x 4 + 1875)2
dq  2 
Q x > 0, f ’(x) < 0 if 5 - x < 0 (i.e.,) if x > 5.
d 2 5
(l ) = 0 ⇒ sin q = 0 (or) cos q = And f ’(x) > 0 if 5 - x > 0 (i.e.,) if x < 5
dq 8
5 ⇒ f(x) is decreasing if x > 5 and increasing
⇒ cos q = ± 1 or cos q = if 0< x < 5.
8
Now \ Maximum occurs at x = 5.
d2 2 5  \ Largest term is for n = 5 and is equal to
(l ) = 2a2  cos q − 4cos2 q
dq 2
2  5 1
a5 = =
625 + 1875 500
5 
= 2a2  cos q − 8cos2 q + 4 
2   x+y 
264. Given f(x) + f(y) = f  for x, y∈ R
When cos q = ±1,
d2 2
(l ) < 0  1 − xy 
dq2 such that xy < 1
⇒ This corresponds to maximum point Put y = - x
5 d2 2 f(x) + f(-x) = f(0) = 0
When cos q = , (l ) > 0
8 dq2
⇒ f(x) is an odd function
which corresponds to minimum value
 a   −3a  Now,
\ The two points are P  ,0  ;Q  ,0
 2   2  f '(x) = lim
f(x + h) − f(x)
h→0 h
262. Given x2 + y2 = c2   (1)
f(x + h) + f(− x)
dy − x = lim
Slope of the tangent to (1) is = h→0 h
dx y
 [ Q f is an odd function]
x y 1  h
Given that + = 1  (2) is a tangent to (1) for 
a b = lim f  
h → 0 h  1 + x(x + h) 

dy b
which =− . [using functional relation]
dx a
 h 
x b x y f
We have = ⇒ = 1  1 + hx + x 2  h 1
y a b a = lim × =
h→0 h  h  1 + hx + x 2
1 + x2
y  2 
x x +y 1 + hx + x 
a b 1 ab
⇒ = = a b = = 2 ⇒ f ’(x) ≠ 0 for any real x
b a b +a b +a a + b2
a b a b a b ⇒ f(x) has no maximum or minimum.
ab2 a2 b
⇒ x = 2 2
and y = 2 265. Let the rectangle be bounded by x = k, x = l
a +b a + b2
Then its vertices are given by
2 2
 ab2 
 a 2 + b2  +
 a2b 
 a 2 + b2  = c
2
( ) (
A k, 24 − 6k , B l, 24 + 3l , )

1 1 1 C ( l, − 24 + 3l ) , D ( k, − 24 − 6k )
⇒ + 2 = 2 ⇒ a2, 2c2, b2 are in HP
a 2
b c Now sides AD and BC are parallel to y axis
Differential Calculus  2.189

⇒ y coor of A and B are equal ⇒ k = - l x+y x−y


267. Given f(x) + f(y) = 2f  f  (1) and
\ Area of the rectangle is A’ = 6k 24 − 6k  2   2 
f(0) ≠ 0.
y
Put x = y = 0 in (1)
2 f(0) = 2[f (0)]2
⇒ f(0) [f(0) - 1] = 0
B A ⇒ f(0) = 1 [Qf(0) ≠ 0]
x Put y = -x in (1)
O

C D f(x) + f(-x) = 2 f(0) f(x)


2
y = 6(4 − x) 2
y = 3x + 24 ⇒ f(x) = f(-x) [Q f(0) = 1]
⇒ f(x) is an even function
Differentiating (1) w.r.t. x,
For extremum f ' (x) + f ' (y) y '

dA' 8   x + y   1 + y1   x − y 
=0⇒k = = 2 f 1   f  
dk 3   2  2   2 
8 d 2 A'
When k = , <0  x + y  1  x − y   1 − y1  
3 dk 2 +f  f 
 2   2   2  
⇒ This corresponds to a maximum
8 ⇒ f ' (x)
\ Maximum area A’ = 6 × 24 − 16 = 32 2
3 x+y x−y x+y 1x−y
= f' f +f f
 2   2   2   2 
266. Given f(x) = (p – 4)x3 + (p – 2)x2 + (p – 3) x + 2
  (2)
We have
f ’(x) = 3 (p - 4) x2 + 2 (p - 2) x + p - 3  (1) [Q x and y are independent y = 0] 1

f ’(x) must vanish at two real and distinct points a, b differentiating (2) w.r.t x,
such that a < 0 and b > 0   x + y  1 + y '   x − y 
f ” (x) =  f ''   f  
⇒ (p - 3) (p - 4) < 0   2  2   2 
⇒ p ∈ (3, 4) x+y x−y
+f '  f'
 2   2 
(1 − y ')
The graph of f ’(x) must look as below.
 x + y   x − y  1 − y ' 
+f  f"
y  2   2   2 

y = f ’(x)  x + y  1 + y '   x − y ' 


+f ' 
 2   2   2  

x 1 x+y x−y x+y x−y


α 0 β = f ''  f + f ' f'
2  2   2   2   2 

1 x+y x−y
+ f f ''   (3)
For the curve to look like this, we must have 2  2   2 
f ’(0) > 0 (ie) p - 3 > 0 (i.e.,) p > 3 from (1) [Q y ’ = 0 ]
\ Possible values of p ∈ (3, 4) Put x = y in (3)
2.190  Differential Calculus

1 1 f(1-) = 0 + 1 + 1 = 2
f ”(x) = f ” (x) f (0)+ f ’(x) f ’ (0) + f(x) f ’’ (0)
2 2 f(1+) = 1 + 1 + 1 = 3
1 1 f(x) is discontinuous at x = 1
= f11(x) + [0]2 + f(x) (-1)4 f(0) = 1
2 2 Statement 1 is true
[f(x) is even ⇒ f (x) is odd ⇒ f (x) = even and given
1 11 Statement 2 is true
f11(0) = -1] ⇒ f11 (x) = -f(x). However, result in Statement 2 is not used to prove 1.
268. (b) 269. (d) Statement 2 is true, being a standard
−1 −1 1 3
Consider the points −1, , ,0, , , 1 x −2
2 4 2 4 2sin2 
1 − cos(x − 2)  2 
f(-1-) = (-2) + (-1) + (-1) = -4 lim = lim   (1)
x →2 (x − 2) x →2 (x − 2)
f(-1 ) = (-1) + (-1) + (-1) = -3
+

f(x) is discontinuous at x = -1 x −2


− 2 sin 
1 − cos(x − 2)  2 
 −1−  lim − = lim−
f
 2 
= (-1) + (-1) + (-1) = -3 x →2 (x − 2) x →2 ( x − 2)
 −1+  x −2
sin 
f = (-1) + (-1) + 0 = -2  2 
 2  ( )
= − 2 lim−
x →2  x − 2 
1  2  × 2
f(x) is discontinuous at x = −
2


 −1− 
f
 4 
= (−1) + (−1) + 0 = −2 = ( 2 ) × 12 = −21
 −1+  1 − cos(x − 2)
f = (−1) + 0 + 0 = −1 = lim +
 4  x →2 (x − 2)
−1 x −2
f(x) is discontinuous at x = 2 sin 
4  2  1
= lim =
( )
f 0− = (−1) + 0 + 0 = −1 x →2 + x −2
 2  × 2 2

f (0 ) = 0 + 0 + 0 = 0
+

1 − cos(x − 2)
f(x) is discontinuous at x = 0 ⇒ lim does not exist
x →2 (x − 2)
 −1− 
f =0+0+0=0
 2  Statement 1 is false

 −1+  270. Statement 2 will be true only when both f(x) and g(x)
f = 0 + 0 +1 = 1 are continuous in R
 2 
Since both ex and sin x are continuous in R, esinx is
1 periodic
f(x) is discontinuous at x =
2 Statement 1 is true.
 3− 
f   = 0 + 0 +1 = 1 271. Statement 2 is true
4
and using Statement 2, we infer that
 3+ 
f   = 0 +1+1 = 2 f(x) = 0 has no negative root.
4
3 272. Consider Statement 2
f(x) is discontinuous at x = Since the range of the function
4
Differential Calculus  2.191

 p p Also, f ’(x) changes sign from positive to negative as


sin-1 x is  − ,  3
 2 2 x crosses
5
When x < 1 3
⇒ f(x) is a maximum at x =
5
 2  ⇒ Statement 1 is true
 
 2x  −1  x 
sin −1  = sin 275. Statement 1 is true
 1 + x 2    1 2 
1 +    Let
 x  f(x) = 8x4 + 12x3 - 30x2 + 17x - 3
1 f ’(x) = 32x3 + 36x2 - 60x + 17
= 2tan −1  
x f ’’(x) = 96x2 + 72x - 60
Statement 2 is true = 12{8x2 + 6x - 5}
2 −2 f ’’’(x) = 12{16x - 6}
f ’(1-) = ; f '(1+ ) =
1 + x2 (1 + x 2 ) f ’’(x) = 0 ⇒ 8x2 + 6x - 5 = 0
⇒ f ’(x) does not exist at x = 1 −6 ± 36 + 160 −6 ± 14
x= =
similarly, we can sow that f ’(x) 16 16
does not exist at x = -1 −20 8
= ,
Statement 1 is true 16 16
−5 1
273. Statement 1 = ,
4 2
F(0) = 0; if F(a) is zero for a ≠ 0 then there is at least
1
one point between 0 and a Substituting x =
2
When 1 - x + x2 - x3 + x4 = 0, which is not true.
32 36 60
\ Statement 1 is true ⇒ f ’(x) + − + 17
8 4 2
Statement 2
= 4 + 9 - 30 + 17 = 0
F’(x) = 1 - x + x2 - x3 + x4 Again,
F’(-1) = 5 ≠ 0
 1  8 12 30 17
\ F’(x) is not minimum at x = -1 f   = + − + −3
 2  16 8 4 2
Statement 2 is false
1 3 15 17
Choice (c) = + − + −3
2 2 2 2
274. f(x) is continuous for x ∈ R =0
4 −
1 4
1
f ’(X) = e2x × (x − 1) 5 + (x − 1) 5 × 2e2x f ’’’   ≠ 0
5 2
e2x Using statement 2, we conclude that the roots of the
= 1 [ 4 + 2 × 5(x − 1)] equation
5(x − 1) 5
8x4 + 12x3 - 30x2 + 17x - 3 = 0
e2x (10x − 6) are of the form a, a, a, b
= 1
5(x − 1) 5 276. Statement 2 is true
Clearly f(x) is not differentiable at x = 1 Consider Statement 1
Statement 2 is true f ’(x) = 2x - x cos x - sin x + sin x
3 = x(2 - cos x)
x= and x = 1 are critical points of f(x)
5 f ’(x) > 0 for x > 0
2.192  Differential Calculus

and f ’(x) < 0 for x < 0 −2


f ’(x) =
x = 0 is a critical point of f(x) (1 + x)2
f(x) is monotomic decreasing in (-∞, 0) and mono- −2
tomic increasing in (0, ∞) f ’(2) =
9
since f(0) = -1, the graph of y = f(x) will be as shown 9
below Slope of the normal at x = 2 equals
2
2
y When x = 2, y =
3
Equation of the normal at x = 2 is
2 9
y− = (x − 2)
x 3 2
o
3y − 2 9x − 18
=
−1
3 2
6y - 4 = 27x - 54
⇒ f(x) = 0 has only two real solution 27x - 6y - 50 = 0

278. x = 16 + 6p – p2
 1 1 
277. x  + + .....n terms  dx
 x(1 + x) (1 + x)(1 + 2x)  = 6 − 2p
dp
 1 x x
= + + +  p 
2  (
1 + x (1 + x)(1 + 2x) (1 + 2x)(1 + 3x) ed = −  6 − 2p)
 16 + 6p − p 
x 
..... + 
[1 + (n − 1)x][1 + nx]
(e ) d
p= 4
=−
4
24
1
× −2 = .
3
1  1 1   1 1 
= + − + − .
1 + x  1 + x 1 + 2x   1 + 2x 1 + 3x 
 279. Demand is unitary ⇒ ed= 1
⇒ 2p2 - 6p = 16 + 6p - p2
 1 1 
+ ..... +  −
 1 + (n − 1)x (1 + nx)  ⇒ 3p2 - 12 p - 16 = 0.

2 1 280. x = p e–p
= − dx
1 + x 1 + nx = − pe − p + e − p = e − p (1 − p).
dp
 2 1 
f(x) = lim  −
n →∞ 1 + x
 1 + nx  p dx p −p
ed = = e (1 − p) = |(1 - p)|
2 x dp pe − p
=
1+ x
ed at p = 2 is = |(1 - 2)| = 1.
since f(x) is given to be continues at x = 0,
 2 281. xs = a p − b
 ' x ≠0
f(x) = 1 + x
 2' dx s a
x =0 =
dp 2 p − b
2
⇒ f(x) = , x >0 Elasticity of supply
1+ x
It is clear that f(x) is differentiable for all x ≥ 0 p d xs p a p
= = × =
Statement 2 is true x s dp a p−b 2 p−b 2(p − b)
Differential Calculus  2.193

282. The revenue function R is given by 286. Revenue function is given by


R = px = x 9 − x R = xp
dR d2 R = x (600 - 8x) = 600x - 8x2
R is maximum when = 0 and < 0.
dx dx 2
287. Profit function P
dR x = Revenue - cost
= × (−1) + 9 − x
dx 2 9−x = (600x - 8x2) - (x2+ 78x + 2500)
2(9 − x) − x 18 − 3x
= = =-9x2 + 522x - 2500
2 9−x 2 9−x
dP d2 P
dR 288. P is maximum when = 0 and <0
=0⇒x=6 dx dx 2
dx
dP
d2 R
(2 )
9 − x (−3) + (18 − 3x) ×
2
2 9−x

dx
= -18x + 522
=
dx 2 4(9 − x) d2 P
= -18 < 0
d2 R dx 2
When x = 6, < 0.
dx 2 dP
= 0 ⇒ -18x + 522 = 0 ⇒ x = 29
Therefore, the value of x for which the revenue is dx
maximum is 6.
600 − p
283. Profit function P 289. We have x =
8
= Total revenue - Total cost
⇒ p = 600 - 8x = 600 - 8 × 29 = 600 - 232
 x3 
= xp -  − 7x 2 + 111x + 50  = Rs 368
3 
 x3  290. (a), (c), (d)
= x (100 - x) -  − 7x 2 + 111x + 50 
3  lim 2( 25x 2 + x − 5x)
x →∞

x3 2x 2
=- + 6x 2 − 11x − 50 = lim = lim
3 x →∞ 2
( 25x + x + 5x x →∞
1
25 + +5
dP dP2 x
284. For P to be maximum, = o, 2 < 0 1
dx dx =
dP 5
= -x2 + 12x - 11
dx 2 2
2−
1 + x = lim (1 + x) = 1
2
2
dP (a) lim = lim
= -2x + 12 x →0 10x x →0 10 5
dx 2
dP −e− x + 1 e− x 1
= 0 ⇒ x2 - 12x + 11 = 0 (b) lim = lim = lim =
dx x →0 2x x →0 2 2
⇒ (x - 11) (x - 1) = 0 ⇒ x = 1, 11 2 × (sin x 2 ) × 2x 1  sin x 2 
d2 P (c) lim = lim = lim ×  2 
When x = 11, <0 x →0 20x 3 x →0 5  x 
dx 2
Profit maximum level of output is = 11 1
=
5
 −x3  x
285. Maximum profit =  + 6x 2 − 11x − 50 sin
 3  x =11 (d) lim = lim 5 =1
334 x →0 x →0 x 5
= 5×  
3 5
2.194  Differential Calculus

291. y f(x) is increasing if f ’(x) ≥ 0


−(x 2 + 6x + 6)
>0
y=1 (x 2 + 5x + 9)2
⇒ x lies between (-3 ± 3)
x
o ⇒ f(x) is not monotonic increasing for all x ∈R
(b) f(x) = 2x3 + 3x2 - 36x + 7
f ’(x) = 6x2 + 6x - 36 = 6(x2 + x - 6)
= 6(x +3) (x -2)
2
x −4 ⇒ f(x) is not monotonic increasing for all x ∈ R
f(x) =
x2 + 4 (c) f(x) = 2x3 - 3x2 + 6x - 1
8 f ’(x) = 6x2 - 6x + 6
=1- 2
(x + 4) = 6(x2 - x + 1)
As x → ∞, f(x) → 1 > 0 for all x ∈ R
when x = 0, f(x) = 1 - 2= -1 (c) is true
Curve y = f(x) is symmetrical about; y axis (d) Since ef(x) is monotonic increasing if f(x) is mono-
tonic increasing,
Choice (b) and (c) are true and hence (d) is false
We consider
292. f(x) is continuous in R provided it is continuous at f(x) = 2x3 + 9x2 + 42x - 5
x=2
f ’(x) = 6x2 + 18x + 42
⇒ p +2q + 4 = 4p + 12q
= 6(x2 + 3x + 7)
⇒ 3p + 10q = 4
> 0 for all x ∈ R
f(x) is differentiable in R, if f(x) is continuous and
differentiable at x = 2 ⇒ (d) is true

f ’(2-) = q + 4  −9
f ’(2+) = 2p + 12q −2x − 9 − 2x − 2x + 9, −a < x < 2

f ’(2-) = f ’(2+) gives q + 4 = 2p + 12 q  −9
<x<0
 2x + 9 − 2x − 2x + 9, 2
⇒ 2p + 11q = 4 294. f(x) = 
 2x + 9 + 2x − 2x + 9, 9
Hence, f(x) is differentiable at x = 2, if 0 < x, <
 2
3p + 10q = 4 and 2p + 11q =4  2x + 9 + 2x + 2x − 9, 9
 <x<a
4 4  2
 p= , q= satisfy both the above relations
1`3 13  −9
⇒ (b) is true  −6x, −a < x <
2

p = -2, q = 1 satisfy 3p + 10q = 4 only  − 9
<x<0
 2
⇒ (c) is true = −2x + 18,
2x + 18, 9
(d) is false 0<x<
 2
6x 9
293. (a)  <x<a
 2
x+3
f(x) =  −9   −9 
x 2 + 5x + 9 f   = 27 = f  +  ⇒ f(x) is continuous at
 2   2 
(x 2 + 5x + 9) − (x + 3) (2x + 5) −9
f ’(x) = x=
(x 2 + 5x + 9) 2
Differential Calculus  2.195

f(0-) = 18 = f(0+) ⇒ f(x) is continuous at x = 0 297. Since both x and x3 are < 0 in (–2, 0)
We have
9− 9+
f  = 27 = f  ⇒ f(x) is continuous at
 2   2  x2 −2 ≤ x < 0

9 f(x) =  x 0 < x <1
x= x3 1< x ≤ 2
2 
⇒ f(x) is continuous for all x Clearly, f(x) is continuous in[-2,2]
Since polynomial functions are differentiable, f ’(1-) = 1, f ’(1+) = 3
 −9 ⇒ f(x) is not differentiable at x = 1
−6 −a < x < 2 3
 3  3  35
 −9 f(-1) + f   = (−1)2 +   =
<x<0 2 2 8
−2 2
f ’(x) 
2 9 Since f is differentiable in [-2,0] and (1,2],
0<x<
 2 2
6 9 3 3
<x<a f ’(-1) - f ’   = 2(−1) − 3 ×  
 2 2
 2
27
−9 9 = -2 -
⇒ f(x) is not differentiable at x = , 0, 4
2 2
−35
e −1− x − 4 =
295. lim− f(x) = lim− x =0 4
x →0 x →0 −1 − x
298. (a) f ’(x) = ex + e-x ³ 2 for all values of x
ex − 4 \ f(x) is an increasing function
lim+ f(x) = lim− x = − 3 and f(0) = 3
x →0 x →0 x −1
e x
Only (a) holds. (b) f ’(x) = 2 is positive except at x = 0
x
296. (a) f(x) = e–2x cosx is continuous and differentiable 4
(c) f ’(x) = 1 - is negative at x = ±1
x2
in 0, p 
 2 (d) f(x) = 3x2 - 10x + 11 > 0 for all values of x
 
⇒ conditions of mean value theorem are satisfied 299. (a) By L’ Hospital Rule
p 1
(b) f(x) = sin
2
[x] 2−3
(
lim x 3 + 27
x − 2
)
+ log ( x − 2).3x 2
54
= =9
f(-1-) = 0, f(-1+) = -1 2x 6
 x 
f(0-) = -1, f(0+) = 0 
 e x − 1   x +1 − e x 

(b). L = lim 
f(1-) = 0, f(1+) = 1 x →0  x 
f(x) is not continuous in[-1, 1]  
 1 
(c) f(x) = (x + 2) (2x - 5) 4
 ex − 1 

 e x −1 

1−
= lim   
x 
Being a polynomial function, x →0  x 
5
f(x) is continuous and differentiable in [2, ]
2 ex − 1
Put = t . As x → 0, t → 1.
⇒ Mean value theorem conditions are satisfied x
1 1
(d) sin is not continuous at x = 0 \ L = lim t 1− t [1∞ form]
x t →1
2.196  Differential Calculus

Taking logarithm, 300. (a) f(x) = 3x – log(1 + 3x + 2x2)


1 0  = 3x - log(1 + x) (1 + 2x)
Log L = lim log t  0 form  1 2
t →1 1− t f '(x) = 3 - − > 0 if x > 0, f ’(0) = 0
1 + x 1 + 2x
1
\ 3x - log(1 + 3x + 2x2) > 0 for x > 0 in all this
= lim t (by L. Hospital’s rule) = -1⇒ L = e-1. intervals
t →1 −1
1 2
x2 x 4 x6 (b) g’(x) = + − 3 + 5x
(c) We know that cos x = 1 − + − + ... 1 + x 1 + 2x
2! 4! 6!
−1 4
g”(x) = − +5
x3 x5 (1 + x ) (1 + 2x )
2 2
sin x = x − + − ...
3! 5!
2 16
 x2 x 4   x3  g”’(x) = + > 0 for x > 0
ax + x 1 − + − ...  − b  x − + ..  (1 + x ) (1 + 2x )
3 3

 2! 4!   3! 
∴ L = lim \ g”(x) is increasing in all the intervals
x→o x3
5
1 b 1 (c) f(x) = log(1 + 3x + 2x2) - 3x + x 2
= Lim (a − b + 1) × + − 2
x →0 x 2  3! 2! 
1 2
+ terms containing x f ’(x) = + − 3 + 5x
1 + x 1 + 2x
As L = 1, we must have x 4x
b 1 = 5x - − ≥ 0 for x ≥ 0
a - b + 1 = 0 and − = 1 1 + x 1 + 2x
3! 2! f(0) = 0
⇒ b = 9 and a = 8.
\ f(x) > 0 for x > 0
− f(x) + 3f(2x) − 3f(3x) + f(4x)  0 
(d) L = lim 3  0 form  g(x) = 3x - log (1 + 3x + 2x2) > 0 for x > 0
x →0 x
5x 2
− f '(x) + 6f '(2x) − 9f '(3x) + 4f '(4x) \ 3x − < log (1 + 3x + 2x 2 ) < 3x for all
= lim 2
x →0 3x 2 x>0
− f "(x) + 12f "(2x) − 27f "(3x) + 16f "(4x) Choice (p), (q), (r), (s)
= lim
x →0 6x  ax  2 2
0  e  ax + 2x − a  , x < 0
 0 form  

(d) f ’(x) =  −2 , x=0
a
− f "'(x) + 24f "'(2x) − 81f '"(3x) + 64f "'(4x) 
= lim  3x 2 − 2 , x>0
6 a
x →0

0 
 form 
0  
(
 eax a 2 x 2 , x<0 )
f ”(x) = not defined , x = 0
6.f '"(0) 
= = f "'(0) .  6x , x>0
6
But L = 12 ⇒ f ’”(0) = 12. \ f ”(x) > 0 for all the given interval
chapter INTEGRAL
CALCULUS

3
nnn  Cha p t e r O u t l i n e
Preview
sTUDY MATERIAL Solutions of First Order First Degree Differential
Equations
Introduction
• Concept Strands (60-92)
Definite Integral as The Limit of a Sum
Concept connectors
Anti-Derivatives
• Concept Strands (1-11) • 56 Connectors
Indefinite Integrals of Rational Functions topic grip
• Concept Strands (12-20) • Subjective Questions (15)
dx dx a cos x + b sin x • Straight Objective Type Questions (15)
Integrals of the form ∫ ,∫ ,∫ dx
a + b cos x a + b sin x c cos x + d sin x • Assertion–Reason Type Questions (10)
• Linked Comprehension Type Questions (6)
• Concept Strands (21-24) • Multiple Correct Objective Type Questions (3)
Integration By Parts Method • Matrix-Match Type Question (1)
• Concept Strands (25-30)
iit assignment exercise
Integrals of the form ∫ ax 2 + bx + c dx • Straight Objective Type Questions (100)
• Assertion–Reason Type Questions (3)
• Concept Strands (31-34)
• Linked Comprehension Type Questions (3)
Evaluation of Definite Integrals • Multiple Correct Objective Type Questions (3)
• Concept Strands (35-37) • Matrix-Match Type Question (1)
Properties of Definite Integrals
Additional Practice Exercise
Improper Integrals • Subjective Questions (30)
• Concept Strands (38-54) • Straight Objective Type Questions (77)
Differential Equations • Assertion–Reason Type Questions (10)
Formation of a Differential Equation • Linked Comprehension Type Questions (12)
• Concept Strands (55-59) • Multiple Correct Objective Type Questions (8)
• Matrix-Match Type Questions (3)
3.2  Integral Calculus

Introduction

In the chapter on differential calculus, we used the tangent ds


and velocity problems to introduce the concept of the de- time t. i.e., if we are given , we wish to know s. Or, if we
dt
rivative which is the central idea contained in differential dy
calculus. This chapter starts with the area and displacement are given the slope at any point x on a curve, we wish to
dx
problems and uses them to formulate the idea of a definite know the function y.
integral, which is the basic concept of integral calculus. Recall that, given the graph of velocity versus time, i.e.,
Computations of areas bounded by curves, arc lengths, vol- the graph of v = f(t) between times t = t1 and t = t2: Displace-
umes, work done, moment of inertia, reduce to the evalu- ment of the particle in the time interval (t1, t2) is given by
ation of definite integrals. The fundamental theorem of in- the area bounded by the graph v = f(t), the t-axis and the
tegral calculus links the integral and the derivative and we ordinates at t1 and t2. We have introduced this idea of cal-
will see in this chapter that it greatly simplifies the solution culating displacement of the particle as the area under the
of many problems involving definite integrals. curve and further as ‘definite integral between limits t = t1
In the second part of this chapter, we deal with dif- and t = t2, in Chapter 1. Now, in this unit let us understand
ferential equations and their solutions. Many physical laws the mathematical concepts behind this concept.
and relations appear mathematically in the form of differ- This means that if we consider the problem:
ential equations. Also, certain geometrical results may be Given the velocity function v(t), find the displacement
expressed as differential equations. s(t)”. This is equivalent to the problem of finding the area
The material in this chapter is presented in the follow- bounded by the curve v(t), the t-axis and the ordinates at
ing order: t = t1 and t = t2.
(i) Areas of plane figures, distance covered by a par- In other words, our problem at hand is:
ticle moving along a straight line in a given interval Given a function f(x), we want to find the area bound-
of time are looked upon as limits of sum of areas of ed by the graph of the curve y = f(x), the x-axis and the
rectangles. In other words, areas, distance covered ordinates at x = a and x = b.
by a particle in a given interval of time are obtained Suppose f(x) is a constant function, say, f(x) = V0 (>0)
through a limit process.
(ii) Fundamental theorem of integral calculus linking
the integral and the derivative. y
(iii) Concept of anti-derivative or indefinite integral of a
P Q
function. f(x) = V0
(iv) Various methods of finding anti-derivatives of
functions.
(v) Definite integrals and their properties. A B
(vi) Application of definite integrals in the computation x
x=a x=b
of areas of plane figures.
(vii) Ordinary differential equations-Formulation-
General solution-Particular solution.
(viii) Solutions of first order and first degree equations.
(ix) Linear differential equations of first order-Initial Fig. 3.1
value problems.
(x) Application of differential equations in solving a few
physical problems. Required area = Area of the rectangle ABQP = V0(b - a)
The above problem may be interpreted as that of find-
ing the displacement of a particle moving along a straight
Area and displacement problems line with uniform velocity V0 in the time interval (a, b).
Suppose we know the velocity of a particle moving along a Suppose f(x) is a linear function, say, f(x) = 3x + 2. Let
straight line at time t, we might wish to know its position at us take a = 1, b = 5.
Integral Calculus  3.3

y y
Q y = 3x + 2 y = x2
Q

P3
P
P2
P1
P (4)
A B (3)
x (2)
O x=1 x=5 (1)
x
O A A1 A2 A3 B
Fig. 3.2
Fig. 3.4
In this case,
Required area = Area of the trapezium ABQP =
1 Through these points A1, A2 and A3, draw verticals
(AP + BQ) × AB
2 A1P1, A2P2, A3P3, to meet the curve at P1, P2 and P3. Area of
1
= ( 5 + 17) × (5 - 1) = 44 the subregion APP1A1 is approximately equal to the area of
2 the rectangle (1). Similarly, for the other three subregions.
The above problem may be interpreted as that of find- Let the sum of the areas of the rectangles (1), (2), (3) and
ing the displacement of a particle moving along a straight (4) be denoted by S4.
line with a uniform acceleration (i.e., with a constant ac- Since AA1 = A1A2 = A2A3 = A3B = 1, S4 = AP + A1P1 +
celeration). A2P2 + A3P3 = 12 + 22 + 32 + 42 = 30
Suppose f(x) = x2 and a = 1 and b = 5 (refer Fig. 3.3) Let AA1 = A1A2 = A2A3 = A3B be denoted by h (here,
h = 1).
y
Then, AP = f(1), A1P1 = f(1 + h), A2P2 = f(1 + 2h) and
Q y = x2 A3P3 = f(1 + 3h)
Thus,
S4 = h[f(1) + f(1 + h) + f(1 + 2h) + f(1 + 3h)]
3
P = h ∑ f (1 + rh) — (1), where f(x) = x2
r =0

Although S4 is not equal to S, S4 gives an approxima-


A B
x tion to S.
O x=1 x=5

y
Fig. 3.3 Q 2
y=x
P7
Required area = Area of the region ABQPA P6
The above problem may be interpreted as that of find- P5
ing the displacement of a particle moving along a straight P4
P3
line given that the velocity v at time x is given by v = x2 in
P2
the time interval (1, 5). P1
We note that since PQ is not a straight line, the prob- P
lem of finding the area ABQP has to be approached in a (1)(2) (3) (4) (5) (6) (7) (8)
way different from that for the two previous examples. x
Let the area of the region ABQP be denoted by S. Let
O A A1 A2 A3 A4 A5 A6 A7 B
us split the region S into subregions as shown in Fig. 3.4.
Fig. 3.5
We divide AB into 4 equal parts at A1, A2 and A3.
3.4  Integral Calculus

By dividing AB into 8 equal parts, and repeating the y


above process, if S8 denotes the sum of the areas of the
8 rectangles thus formed, then. y = x2
S8 = h[f(1) + f(1 + h) + f(1 + 2h) + ……..+ f(1 + 7h)] Q
where h = 0.5
7
= h ∑ f (1 + rh) —(2)
r =0 
Actual computation of S8 gives its value as 35.5 . It can
be observed from Fig. 3.5 that S8 is closer to S than S4.
P 40
By dividing AB into 16 equal parts and repeating the
above process, if S16 denotes the sum of the areas of the 16 divisions
rectangles thus formed (refer Fig. 3.6), then, x
O A B
S16 = h[f(1) + f(1 + h) + f(1 + 2h) +…..+ f(1 + 15h)]
15
Fig. 3.7
= h ∑ f (1 + rh) — (3), (here, h = 0.25)
r =0

It can be observed from Fig. 3.7, that S40 is closer to S


than S16.
y If we continue the above process and compute S50, S100
y = x2 etc., we will be getting better and better approximations to
Q S. It is interesting to note that we had the same experience
in the tangent problem. As the number of divisions increas-
es, the computations of the areas of the rectangles becomes
time consuming, but the accuracy is very high. So, as in the
case of the tangent problem, the area problem also leads us
to a limiting process. This is equivalent to the statement “as
P the number of divisions n of the region is increased indefi-
nitely, the sum of the areas of the rectangles approach the
x area S.” This can be symbolically written as
O A B
n −1

Fig. 3.6 S = lim h ∑ f (1 + rh)


n →∞
r =0

(Note that as n → ∞, h → 0)
Actual computation of S16 gives its value as 38.375. In general, the area of the region bounded by the curve
It can be observed from Fig. 3.6 that S16 is closer to S y = f(x), the x - axis and the ordinates at x = a and x = b is
than S8. equal to
By dividing AB into 40 equal parts and repeating the n −1
b−a
above process, if S40 denotes the sum of the areas of the lim h ∑ f (a + rh) where h =
n →∞ n
40 rectangles thus formed, then, r =0

(we assume that f(x) ≥ 0 in (a, b))


S40 = h[f(1) + f(1 + h) + f(1 + 2h) +…. + f(1 + 39h)] From the above discussions, we see that the solution
39 of the area problem or the displacement problem lies in the
= h ∑ f (1 + 39h) , where h = 0.1 evaluation of the limit of a sum (the sum representing the
r =0
sum of the areas of the rectangles when we divide the re-
Actual computation of S40 gives its value as 40.14 gion into n sub-regions).
Integral Calculus  3.5

Definite integral as the limit of a sum

Definition we could use any letter in place of x without changing


the value of the definite integral.
b b b

In other words, ∫ f (x)dx = ∫ f (t)dt = ∫ f (u)du .


a a a

x0 = a x1 x2 x3 xn−1 xn = b (iii) It can be proved that if f is continuous, the limit in


the above definition always exists.
Fig. 3.8 (iv) We note that the definite integral is the limit of a
Riemann sum. Other Riemann sums, which tend
(b − a) n
to the same definite integral are ∑ f(a + rh)
n r =1
Let y = f(x) be a continuous function of x defined in the
interval [a, b]. (a, b finite). We divide the interval [a, b] into (b − a ) n
(b − a)
and ∑ f (x * ) , where xr* represents the mid-
n r =1 r
n equal sub intervals each of length h = at x0 = a,
n point of the rth sub-interval (xr-1, xr)
x1, x2,….., xn-1, xn = b. Then, xi = x0 + ih, i = 0, 1, 2,.., n. We b
(b − a) n −1
compute the sum Rn where We have, therefore, ∫ f (x)dx = lim
n →∞
∑ f (a + rh)
n r =0
a
Rn = h[f(x0) + f(x1) + f(x2) + … + f(xn-1)]
(b − a) n (b − a) n
(b − a) n −1 = lim ∑ f (a + rh) = lim ∑ f (x r * )
= ∑ f (a + rh)
n r =0
n →∞ n r =1 n →∞ n r =1
(v) The limit in the definition of definite integral exists
(b − a) n −1 even if f(x) is piecewise continuous in [a, b].
Then, lim R n = lim
n →∞ n →∞
∑ f (a + rh) is called the
n r =0 b

“definite integral” of f(x) over [a, b] and it is denoted by


(vi) If f(x) ≥ 0 in [a, b], the definite integral ∫ f (x)dx
a
b
represents the area under the curve y = f(x), the
∫ f (x)dx .
a
x - axis and the ordinates at x = a and x = b.
(vii) Suppose f(x) ≥ 0 in [a, c] and f(x) ≤ 0 in [c, b] where
b
(b − a) n −1 c lies between a and b (i.e., a < c < b) (refer Fig. 3.9)
We write
n →∞ n →∞
∑ f (a + rh) .
∫ f (x)dx = lim R n = lim
n r =0
a
y
Rn is called a Riemann sum. In particular, when a = 0,
1 1 n −1  r 
b = 1, ∫ 0
f (x)dx = lim
n →∞
∑f
n r =0  n 

Remarks
A1
c x=b
(i) The symbol ∫ was introduced by Leibniz and is called x
O x= a A2
an integral sign. It is an elongated S and was chosen
b
y = f(x)
because an integral is a limit of sums. In ∫ f (x)dx,
a
f(x) is called the integrand. a and b are called the Fig. 3.9
lower and upper limits of integration. The procedure
of calculating a definite integral is called integration.
b The Riemann sum in this case is the sum of the
(ii) The definite integral ∫ f (x)dx is a number (positive, areas of rectangles that lie above the x-axis and the
a negative of the sum of the areas of rectangles that lie
negative or zero). It does not depend on x. In fact, below the x-axis. [This is because a Riemann sum is
3.6  Integral Calculus

the sum of the values of f(x) at the different points y


of sub division multiplied by the length h of a sub
b y = f2(x)
interval] Hence, the definite integral ∫ f (x)dx gives
a y = f1(x)
us area of region A1 - area of region A2.
If we require the combined areas of regions A1 and
A2, it is given by
Area of region A1 + Area of region x
O x=α x =λ
c b

A2 = ∫ f (x)dx − ∫ f (x)dx
a c Fig. 3.11
(viii) Suppose we are interested in the area of the region as
given in Fig. 3.10 To sum up, to find the area of a plane region, or to find
the displacement of a particle moving along a straight
y line in a given linear interval, a definite integral has to
be evaluated. We have so far not given any method to
evaluate a definite integral. We now take up this task.
y=d The following theorem, called the fundamental
theorem of integral calculus enables us to compute a
definite integral.

y=c
y = f(x) Fundamental theorem of integral calculus

x Part I
O
If f(x) is continuous in [a, b], then the function defined by
Fig. 3.10 x

g(x) = ∫ f (t)dt , a ≤ x ≤ b is continuous and differentiable in


a
Let the inverse of the function f(x) be denoted by g(y)
[a, b] and g’(x) = f(x).
i.e., x = f-1(y) = g(y) (say). Then, area of the shaded
d

region is given by the definite integral ∫ g(y)dy . Part II


c b

(ix) Area of the region bounded by the curves y = f1(x) If f(x) is continuous in [a, b], then, ∫ f (x)dx = F(b) - F(a),
β a

and y = f2(x) is given by ∫ [f1 (x) − f 2 (x)]dx (refer Fig. where, F(x) is an anti derivative of f(x), i.e., F’(x) = f(x).
α From the above, it is clear that evaluation of a definite
3.11) where f1(x) > f2(x) in (a, b) integral depends on finding an antiderivtaive of a function.

Anti-derivatives

Definition Taking the cue from the formula for the derivative
x n +1
of xn (n being a rational number), if we take F(x) = ,n
If F(x) is a function of x such that F’(x) = f(x) then F(x) is (n + 1)
called an anti-derivative of f(x). being a rational number (n ≠ -1), derivative of F(x) is given
For f(x) = 4x3, since the derivative of x4 is 4x3, we easily 1
by F ' (x) = × (n + 1)x n = x n .
see that an anti-derivative of 4x3 is x4. n +1
Integral Calculus  3.7

Thus, an anti-derivative of xn (where, n is a rational


It is of interest to note that the operator symbols ∫ and
x n +1
number ≠ -1) is .
(n + 1)
d are inverse of each other. Here, ∫ denotes finding
antiderivative and d denotes taking differential.
Observe that
1 1
x n +1 (ii) ∫ f (x)dx may also be represented by {f (x)} or
Derivative of + 4 is xn; D D
(n + 1) operation means indefinite integration.
x n +1 9 (iii) If f1(x) and f2(x) are two functions of x, and k1 and k2
Derivative of − is xn;
(n + 1) 7 represent constants,

∫ 1 1
x n +1 [k f (x) ± k 2 f 2 (x)]dx = k 1 ∫ f1 (x)dx ± k 2 ∫ f 2 (x)dx .
Or, in general, + C where, C is any constant
(n + 1) The above result can be extended to more than two
can be an antiderivative of xn. functions.
We therefore have: (iv) Going back to f(x) = 4x3, we see that the general
x n +1
an anti-derivative of xn = + C, where C is any antiderivative of f(x) is x4 + C, where C is an arbitrary
(n + 1) constant. By assigning specific values to the constant
constant (called an arbitrary constant) C, we obtain a family of functions whose graphs are
The above result is symbolically written as shown in Fig. 3.12.
x n +1 Note that all these curves have the same slope 4x3 at
∫ x dx = (n + 1) + C , n rational ≠ -1, where C is an
n
any point x.
arbitrary constant (v) Since the derivative of x is 1, ∫ 1dx = x + C
Result
If F(x) is an antiderivative of f(x) in an interval I, then the y y = x4 + 2
most general antiderivative of f(x) in I is F(x) + C where C y = x4 + 1
is an arbitrary constant. y = x4
We write this result in symbolic form as y = x4 − 2
∫ f (x)dx = F(x) + C y = x4 − 3
The process of getting an antiderivative F(x) of f(x)
(where f(x) is given) is called “indefinite integration”. x
O
∫ f (x)dx is called an indefinite integral of f(x) (or an anti-
derivative of f(x))

Observations Fig. 3.12


(i) Recall that the differential of a function, say y = g(x)
is given by dy = g’(x) dx, since g(x) is a function of x
whose derivative is g’(x), an antiderivative of g’(x) is g(x) Given below are a few standard indefinite integrals:
or, we have x n +1
∫ x dx =
n
(i) + C , n rational ≠ -1.

g ' (x)dx = g(x) + C n +1
When n = 0, ∫ dx = x + C
This means that when we write ∫ f (x)dx we are
asking for a function F(x) whose differential is f(x)dx 1
[d F(x) = F'(x) dx = f(x) dx] (ii) ∫ x dx = log x + C
For example,
∫ e dx = e
x x
(iii) +C

cos xdx = sin x + C , since d(sin x) = cos x dx.

Again, ∫ sec xdx = tan x + C , since d(tan


2
x) = ax
∫ a dx =
x
(iv) + C (a > 0)
sec2 x dx log a
3.8  Integral Calculus

1 (ax + b)n +1
(v) ∫ sin x dx = − cos x + C (i) ∫ (ax + b) dx =
n
+ C (n ≠ -1 and n is
a (n + 1)
(vi) ∫ cos x dx = sin x + C rational)
1 1
∫ sec x dx = tan x + C
2
(vii) (ii) ∫ dx = log(ax + b) + C
(ax + b) a
(viii) ∫ cos ecx cot xdx = − cos ecx + C e ax
∫ e dx =
ax
(iii) +C
(ix) ∫ sec x tan xdx = sec x + C a
1
(x) ∫ cos ec xdx = − cot xdx
2 (iv) ∫ sin(ax + b)dx = − a cos(ax + b) + C
1 1
(xi) ∫ 1− x 2
dx = sin −1 x + C (v) ∫ cos(ax + b)dx = a sin(ax + b) + C
1 1
∫ sec (ax + b)dx = a tan(ax + b) + C
2
(vi)
(xii) ∫1+ x 2
dx = tan −1 x + C
−1
∫ cos ec (ax + b)dx =
2
(vii) cot(ax + b) + C
Evaluation of indefinite integrals by “substitution a
methods” −1
(viii) ∫ cos ec(ax + b)cot(ax + b)dx = cos ec(ax + b) + C
The method consists in a change of the variable of inte- a
gration “x” to another variable say “u” by a suitable sub- 1
(ix) ∫ sec(ax + b)tan(ax + b)dx = sec(ax + b) + C
stitution. This will reduce the given indefinite integral to a a
standard form. We will illustrate the method by consider-
ing two examples. Results
Suppose we have to evaluate ∫ cos(4x + 5)dx
We set u = 4x + 5 (i) ∫ tan xdx = log sec x + C
1
Taking differentials, du = 4dx or dx = du (ii) ∫ cot xdx = log sin x + C
4
On substitution, the given indefinite integral reduces to (iii) ∫ sec xdx = log(sec x + tan x) + C or
1  1 1 π x
∫ (cos u)  4 du  = 4 ∫ cos udu = 4 sin u + C log tan  +  + C
4 2
1
= sin(4x + 5) + C (iv) ∫ cos ecxdx = log(cos ecx − cot x) + C or
4
As another example, consider the indefinite integral x
log tan +C
dx 2
∫ 3
.
sin x
2 2
(4 − x ) (i) ∫ tan xdx = ∫ cos x dx
Setting x = 2 sin q, we get dx = 2 cos qdq setting cos x = t , -sin xdx = dt
On substitution, the given indefinite integral reduces to
−dt 1 
2 cos θdθ 1 ∫ =∫ t
= − log t + C = log   + C = log sec x + C
t 
∫ 8 cos 3 θ = 4 ∫ sec θdθ
2

cos x dt
1
= tan θ + C =
x
+C
(ii) ∫ cot xdx = ∫ sin x dx = ∫ t
where we have put sin
4 4 4 − x2 x=t

= log t + C = log sin x + C
From the above two examples, we are in a position to
list a few indefinite integrals which may be used as standard (sec x)(sec x + tan x)
(iii) ∫ sec xdx = ∫ dx
results. (sec x + tan x)
Integral Calculus  3.9

Let sec x + tan x = t ⇒ (sec x) (tan x + sec x)dx = dt dt


Substituting, ∫ =∫ t
= log t + C = log(cosec x - cot x) + C
dt

∫ = ∫ = log t + C = log(sec x + tan x) + C
t Now, cosec x - cot x =
1 − cos x
sin x
1 + sin x
Now, sec x + tan x = x
cos x 2 sin 2
2 x
 x x
2 = = tan
x x x x 2
 sin 2 + cos 2  sin + cos 2 sin cos
= = 2 2 2 2
2 x 2 x x x
cos − sin cos − sin  x
2 2 2 2 Hence, ∫ cos ecxdx = log  tan  + C
 2
x
1 + tan
= 2 = tan  π + x  Remarks
x  4 2 
1 − tan (i) If the integrand is of the form (a2 + x2)p, the substitutions
2
x = a tan q or x = a cot q may reduce the indefinite
π x integral to a standard form.
Hence, ∫ sec xdx = log tan  +  + C
4 2 (ii) If the integrand is of the form(a2 - x2)p, substitutions x
(cos ecx)(cos ecx − cot x) = a sin q or x = a cos q may be tried.
(iv) ∫ cos ecxdx = ∫ dx (iii) If the integrand is of the form (x2 - a2)p, substitutions
(cos ecx − cot x)
x = a sec q or a cosec q may be tried.
Let cosec x - cot x = t We work out a few examples to illustrate the substitu-
(cosec x) (cosec x - cot x)dx = dt tion methods.

C o nce p t S t r a n ds
Concept Strand 1 Solution
x2 1
Evaluate ∫ (8 + x 3 4
)
dx . Let log x = t ⇒
x
dx = dt

dt t −4 −1
Solution ∫ =∫ t 5
=
−4
+C =
4(log x)4
+C

Let 8 + x3 = t
1 Concept Strand 3
3x2dx = dt ⇒ x2dx = dt
3
1 Evaluate ∫ (x − 2) x 2 − 4x + 5dx .
dt
Given integral = ∫ 3 4
t
Solution
1 −1 −1
= ∫ t −4 dt = t −3 = +C Let x2 - 4x + 5 = t ⇒ (2x - 4)dx = dt
3 9 9(8 + x 3 )3
3
dt 1 1 3 2 t2
Concept Strand 2

∫ =∫ t × = ∫ tdt = × t 2 × = + C
2 2 2 3 3
1 1 3
Evaluate ∫ x(log x) dx . 5 = (x 2 − 4x + 5) 2 + C
3
3.10  Integral Calculus

Concept Strand 4 Solution


1
e x (1 + x) (i) We have sin mx cos nx = [sin(m + n)x + sin(m - n)x]
Evaluate ∫ dx . 2
cos 2 (xe x ) ∫sin mx cos nxdx
1 1
= ∫ sin(m + n)xdx + ∫ sin(m − n)xdx
Solution 2 2
Let xex = t ⇒ (1 + x)ex dx = dt 1 cos(m + n)x 1 cos(m − n)x
=− − +C
dt 2 (m + n) 2 (m − n)
∫ = ∫ cos 2 t = ∫ sec tdt = tan t + C = tan(xex) + C
2
1
(ii) sin mx sin nx = [cos(m - n)x - cos(m + n)x]
2
Concept Strand 5 ∫sin mx sin nxdx
1 1
= ∫ cos(m − n)xdx − ∫ cos(m + n)xdx
Evaluate ∫ sin 2 xdx and ∫ cos 2 xdx . 2 2
1 sin(m − n)x 1 sin(m + n)x
= − +C
Solution 2 (m − n) 2 (m + n)
1
1 1 sin 2x  (iii) cos mx cos nx = [cos(m + n)x + cos(m - n)x]
2
∫ sin xdx = ∫ (1 − cos2x)dx =  x −
2
+C
2 2 2  ∫cos mx cos nxdx
1 1
1 1 sin 2x  = ∫ cos(m + n)xdx + ∫ cos(m − n)xdx
∫ cos xdx = ∫ (1 + cos2x)dx =  x +
2
+C 2 2
2 2 2 
1 sin(m + n)x 1 sin(m − n)x
= + +C
Concept Strand 6 2 (m + n) 2 (m − n)

Evaluate ∫ sin 3 xdx and ∫ cos 3 xdx . Concept Strand 8

Evaluate ∫ sin 2 x cos 9 xdx .


Solution
We have sin 3x = 3sin x - 4sin3 x Solution
3 1 Let sin x = t ⇒ cos x dx = dt
⇒ sin3 x = sin x − sin 3x
4 4
∫ sin x cos xdx = ∫ t (1 − t ) dt
2 9 2 2 4

3 1
∫ sin xdx = 4 ∫ sin xdx − 4 ∫ sin 3xdx
3
= ∫ t 2 (1 − 4t 2 + 6t 4 − 4t 6 + t 8 )dt

3 1  − cos3x  −3cos x cos3x = ∫ (t 2 − 4t 4 + 6t 6 − 4t 8 + t10 )dt


= (− cos x) −   +C = + +C
4 4 3  4 12 t 3 4t 5 6t 7 4t 9 t 11
= − + − + + C , where t = sin x
Again, cos 3x = 4cos3 x - 3cos x 3 5 7 9 11
1 3
⇒ cos3 x = cos3x + cos x Concept Strand 9
4 4
1 3 Evaluate ∫ sec 9 x tan xdx .
∫ cos xdx = 4 ∫ cos3xdx + 4 ∫ cos xdx
3

1 sin 3x 3
= + sin x + C Solution
4 3 4
∫ = ∫ sec
8
x sec x tan xdx
Concept Strand 7 Let sec x = t ⇒ sec x tan x dx = dt
Evaluate ∫sin mx cos nxdx, ∫sin mx sin nxd and t9 (sec x)9
Given integral = ∫ t 8 dt = + C = +C
∫cos mx cos nxdx, where m ≠ n. 9 9
Integral Calculus  3.11

Concept Strand 10 Concept Strand 11


(1 + cos x) x3
Evaluate ∫ (x + sin x)7 dx . Evaluate ∫ (x 2
− 1)5
dx .

Solution
Solution Let x2 - 1 = t ⇒ 2x dx = dt
Let x + sin x = t 1
x 2 × dt
⇒ (1 + cos x) dx = dt 2 1 t +1
∫ = ∫ t 5 = 2 ∫ t 5 dt
dt
Substituting ∫ = ∫ 7 = ∫ t −7 dt 1 −4 −5 1  t −3 t −4 
t
2∫
= (t + t )dt =  + 
2  −3 −4 
t −6 −1
= +C = +C −1  1 1 
−6 6 ( x + sin x )
6
=  + +C
2  3(x 2 − 1)3 4(x 2 − 1)4 

Indefinite integrals of rational functions

dx We sketch the proof of (iii)


Integrals of the form ∫ Ax 2
+ Bx + C dx dx
∫a 2
+ x2 ∫
=
 x2 
We list below three standard results, which are used for the a 2 1 + 2 
evaluation of integrals of the above type.  a 

dx 1 x −a x
Set = t ⇒ dx = a dt
(i) ∫x 2
−a 2
= log 
2a  x + a 
+C a

dx 1 a + x  1 adt 1 dt
(ii) ∫a = log  +C ∫ = a ∫ (1 + t =
a ∫ 1 + t2
 a − x 
2 2
2
−x 2
2a )

dx 1 x 1 1 x
(iii) ∫a 2
+x 2
= tan −1   + C
a a 
= tan −1 t + C = tan −1   + C
a a a 

C o nce p t S t r a n ds
Concept Strand 12 Solution
Evaluate the following integrals: (i) x2 + 4x + 10 = (x + 2)2 + 6
dx dx
dx ∫ x 2 + 4x + 10 = ∫
(i) ∫x ( )
2
2
+ 4x + 10 (x + 2)2 + 6

dx setting t = x + 2, dt = dx,
(ii) ∫ 2x 2
− 5x + 1 dt 1  t 
∫ =∫ 2 = tan −1 
 6 
+C
( )
2
t + 6 6
dx
(iii) ∫ 4 − 5x − 9x 2
1 x +2
= tan −1  +C
6  6 

3.12  Integral Calculus

 2 5x 1   5 4
(ii) 2x2 - 5x + 1 = 2  x − + (iii) 4 - 5x - 9x2 = −9  x 2 + x − 
 2 2   9 9
  25 4 
2
5
  5  25 1    5  17 
2 2
= −9   x +  − − 
= 2  x −  − +  = 2  x −  − 
 4  16 2    4  16    18  324 9 
   5  169    13   5  
2 2 2

= −9   x +  −  = 9 − x+  
dx   18  324    18   18  
∫= ∫  2 2

which is of the form
dx
 5  
17 
2  x −  − 
 4   4  
∫ = ∫  13 2 5  
2
which is of the form
  
  9   −  x +  
  18   18  
dt 5
dt 5

∫t 2
−a 2
where, t = x −
4 ∫ a 2 − t 2 where t = x + 18
  13 5 
 5 17    18 + x + 18  
1 1 x− 4 − 4  Hence, ∫ = ×
1 1
log  +C
Hence, ∫ = × log  +C 9  2 × 13    13 5 
2  2 17  5 17

 x − +

  18    18 − x − 18  
 4  4 4  
 
 
1  x +1 
1  4x − 5 − 17  = log  +C
= log  +C 13 4
17  4x − 5 + 17   −x
9 

(ax + b)dx
Integrals of the form ∫ px 2
+ qx + r

C o nce p t S t r a n d
Concept Strand 13 1 16 
 3 (6x + 5) + 3  dx
 
(2x + 7)dx =∫
Evaluate ∫ (3x2
+ 5x − 1)
(3x 2 + 5x − 1)
1 (6x + 5)dx 16 dx
= ∫ 2
+ ∫ 2
Solution 3 (3x + 5x − 1) 3 (3x + 5x − 1)
1 16
d = I1 + I 2 — (1)
We write 2x + 7 = A (3x2 + 5x - 1) + B, where A and B 3 3 
dx dt
are constants. I1: By setting 3x2 + 5x - 1 = t, I1 reduces to ∫ t
We get 2x + 7 = A(6x + 5) + B which is log t
Therefore, I1 = log(3x2 + 5x - 1)
6A = 2, 5A + B = 7
 5 1
1 16 I2 : 3x2 + 5x - 1 = 3  x 2 + − 
giving A = ,B =  3 3
3 3
\ Given integral  5  25 1 
2
 5  37 
2

= 3  x +  − −  = 3  x +  − 
  6  36 3    6  36 

Integral Calculus  3.13

dx 1 6x + 5 − 37 
I2 = ∫ = log  +C
 2  2
 5   37   37  6x + 5 + 37 
3  x +  −  
 6  6   Substituting for I1 and I2 in (1), we obtain the indefi-
 
nite integral.
 5 37 
1 1 x + − 
= × log  6 6 +C
3  2 37   5 37 
 6   x + 6 + 6 
 

Observation A(2px + q) Bdx


∫ px2
+ qx + r
dx + ∫ 2
px + qx + r
= I1 + I2
(ax + b)
In order to evaluate integrals of the form ∫ (px 2
+ qx + r)
dx ,
It is clear that I1 = A log(px2 + qx + r) and I2 will reduce
d to one of the forms
we express ax + b as A (px2 + qx + r) + B
dx dx dx dx
so that the integral becomes ∫ (x + λ) 2
−µ 2
or ∫ (x + λ) 2
+µ 2
or ∫µ 2
− (x + λ)2

Integrals, which can be evaluated by


decomposition using partial fractions

C o nce p t S t r a n ds
Concept Strand 14 Concept Strand 15
3x 2 + 4x − 1 (8x 2 − 9x + 4)dx
Evaluate ∫ (x + 1)(2x + 3)(x − 4) dx . Evaluate ∫ (2x + 1)2 (x − 3) .
Solution Solution
(8x 2 − 9x + 4)
3x 2 + 4x − 1 We resolve into partial fractions as
We resolve into partial fractions (2x + 1)2 (x − 3)
(x + 1)(2x + 3)(x − 4)
2 3 1
− +
2 1 63 2x + 1 (2x + 1) (x − 3)
2

= 5 − 11 + 55 dx dx dx
(x + 1) (2x + 3) (x − 4) ∫ =2 ∫ 2x + 1 − 3 ∫ (2x + 1) + ∫ x − 32

Given integral log(2x + 1) 3


=2 + + log(x − 3) + C
2 1 63 2 2(2x + 1)
= log(x + 1) − log(2x + 3) + log(x − 4) + C 3
5 22 55 = log[(2x + 1)(x − 3)] + +C
2(2x + 1)
3.14  Integral Calculus

Concept Strand 16 11
Equating the constant term, 4A - 2C = 1 ⇒ C =
2
(3x + 1)dx 4
Evaluate ∫ (x − 2)(x 2
+ 4)
dx .  13   11 11 
 8   8 x + 4  dx
Solution ∫ = ∫ x − 2 dx + ∫ x 2 + 4
3x 2 + 1 A Bx + C 13 11 xdx 11 dx
log(x − 2) + ∫ 2
8 x + 4 4 ∫ x2 + 4
Let = + = +
(x − 2)(x 2 + 4) x − 2 x 2 + 4 8
A(x2 + 4) + (Bx + C)(x - 2) = 3x2 + 1 13 11
= log(x − 2) + log(x 2 + 4) +
13 8 8×2
Putting x = 2 ⇒ 8A = 13, A = 11 1 x
8 × tan −1   + C
4 2 2
Equating the coefficient of x2 on both sides, A + B = 3 ⇒ 13 11 11 x
= log(x − 2) + log(x 2 + 4) + tan −1   + C
11 8 16 8 2 
B=3-A =
8

dx We outline the proof of (iii)


Integrals of the form ∫ Ax 2 + Bx + C
set x = a sec q ⇒ dx = a sec q tan q dq
dx a sec θ tan θdθ
We list below the following standard results, which can be ∫ x −a2 2
=∫
a 2 (sec 2 θ − 1)
used for the evaluation of integrals of the above form
a sec θ tan θ dθ
(i) ∫
dx x
= sin −1   + C ∫ a tan θ
= ∫ sec θ dθ
2
a −x 2 a 
= log(sec q + tan q ) + C
(ii) ∫
dx
(
= log x + a 2 + x 2 + C )  
( )
2
a2 + x2 x x
= log  +   − 1  + C = log x + x 2 − a 2 + C ,
 a a  
(iii) ∫
dx
x2 − a2
(
= log x + x − a 2 2
)+C 
where, C is the constant of integration.

C o nc e p t S t r a n ds
Concept Strand 17 1 dx
∫= 2
∫ 2 2
dx  5   31 
Evaluate ∫ 2x 2 + 5x + 7
.  x + +
4   4 
 2 2
1  5  5   31  
Solution = log  x + +  x +  +  +C
2  4  4   4  

 5x 7 
2x2 + 5x + 7 = 2  x 2 + +
 2 2 
Concept Strand 18
  5  25 7    5  31 
2 2
dx
= 2  x +  − +  = 2  x +  + 

 4  16 2  
 4  16 
Evaluate: ∫ 1 − x − x2
.
Integral Calculus  3.15

Solution  1
 x+ 
1 - x - x = -(x + x - 1)
2 2 dx 2 +C
∫= ∫ 2
= sin −1 
 5 
 1 5 5 
2
1
2
5  1  
= −  x +  −  = −  x +  − x +  2 
4  2
  2  4  4  2
 2x + 1 
= sin −1  +C
 5 

(ax + b)dx
Integrals of the form ∫ px 2 + qx + r
.

We illustrate the method by working out two examples.

C o nce p t S t r a n ds
Concept Strand 19 
( )
2 
= log  x + 3 + (x + 3)2 − 2 2 
(3x + 4)dx  
Evaluate ∫ .
x 2 + 6x + 1 (
= log x + 3 + x 2 + 6x + 1 )
Solution Substituting for I1 and I2 in (1) we get the answer.

d 2
Let 3x + 4 = A (x + 6x + 1) + B
dx Concept Strand 20
= A(2x + 6) + B, we have, 2A
(4x − 5)dx
= 3, 6A + B = 4 Evaluate ∫ 1 − 4x − 5x 2
.
3
⇒ A = , B = -5
2 Solution
3 (2x + 6)dx dx d
∫=2 ∫ 2
x + 6x + 1
− 5∫
2
x + 6x + 1
Let 4x - 5 = A (1 - 4x - 5x2) + B = A(-4 - 10x) + B
dx
3 −2 −33
= I1 − 5I 2 + C  — (1) -10A = 4, -4A + B = -5 ⇒ A = ,B=
2 5 5
I1: −2 (−4 − 10x)dx 33 dx
Set x2 + 6x + 1 = t ⇒ (2x + 6)dx = dt ∫= 5 ∫ 1 − 4x − 5x 2

5 ∫ 1 − 4x − 5x 2
dt −2 33
I1 = ∫ 2
= 2 t = 2 x + 6x + 1 = I − I +C  — (1)
t 5 1 5 2
I2: I1:
( )
2
x + 6x + 1 = (x + 3) - 9 + 1 = (x + 3) − 2 2
2 2 2 Set 1 - 4x - 5x2 = t
⇒ (-4 - 10x)dx = dt
dx
I2 = ∫ dt
( ) I1 = ∫
2
(x + 3)2 − 2 2 = 2 t = 2 1 − 4x − 5x 2
t
3.16  Integral Calculus

I2 :  2
1- 4x - 5x2 x+
1 dx  1 5

−1
I2 = = sin  
 4 1   2
2
4 1  5 2
5 3
9  2  
= −5  x 2 + x −  = −5   x +  − −  − x+   5 
 5 5   5  25 5  25  5
  2
2
9  1  5x + 2 
= sin −1 
= −5   x +  − 
5  3 
  5  25 

 9  2   Substituting for I1 and I2 in (1) we get the answer.


2

= 5 −  x +  
 25  5  

dx dx acosx + bsinx
Integrals of the form ∫ a + bcosx , ∫ a + bsinx , ∫ ccosx + dsinx
dx

We illustrate the methods by working out a few examples.

C o nce p t S t r a n ds
Concept Strand 21 Concept Strand 22
dx dx
Evaluate ∫ 3 + 5cos x . Evaluate ∫ 5 − 4 sin x .
Solution Solution
x x 1
Put tan = t ⇒ sec 2 × dx = dt x x
2 2 2 Putting tan = t and writing sin x in terms of tan ,
2 2
2dt 2dt
⇒ dx = = 2dt
x (1 + t 2 ) 2dt
sec 2 given integral = ∫ 1 + t 2 dt =
2 4 × 2t ∫ 5 + 5t 2 − 8t
1 − t2 5−
Also, cos x = 1 + t2
1 + t2
  4  16 
2
Substituting,  8 
5 + 5t 2 − 8t = 5  t 2 − t + 1 = 5   t −  − + 1
 5    5  25 
2dt
(1 + t 2 ) 2dt   4  2 9 
∫ = ∫ 5(1 − t 2 ) = ∫ 8 − 2t 2 = 5 t −  + 
3+   5  25 
(1 + t 2 ) dt
dt 1 2 +t 
∫ = 2∫  4 2 9 
=∫ = log  +C  
5 t −  + 
4 − t2 2 × 2  2 − t    5  25 

 x t − 4 
1  2 + tan 2  =
2
×
1
tan −1  5 +C
= log  +C
4 x 5 3  3 
 5 
 2 − tan   5 
 2
Integral Calculus  3.17

2  5t − 4  1 dt
= tan −1 
3  3 
+C ∫= 2 ∫  1 1
2

 t + 2  + 4
 x 
5tan − 4
2 −1  2   1
= tan  +C
 + 
3 3 t
1



 = tan  2  + C
−1

1 1
2×  
2  2 
Concept Strand 23
 x 
= tan-1 (2t + 1) + C = tan −1  2 tan + 1  + C
dx  2 
Evaluate ∫ .
2 sin x − cos x + 3
Concept Strand 24
Solution
x 2 cos x + 3sin x
Putting tan
2
= t , and writing sin x and cos x in Evaluate ∫ 3cos x + 5sin x dx .
x
terms of tan ,
2 Solution
2dt Let 2cos x + 3 sin x
1 + t2
∫ = ∫  2 × 2t 1 − t 2  d
= A (3cos x + 5sin x) + B(3cos x + 5sin x)
 1 + t 2 − 1 + t 2 + 3  dx
= A(-3 sin x + 5 cos x) + B(3 cos x + 5 sin x)
2dt 2dt
=∫ 2 2
=∫ 2 Equating coefficients of sinx and cosx, -3A + 5B = 3,
4t − 1 + t + 3 + 3t 4t + 4t + 2 5A + 3B = 2
dt 1 21
=∫ Solving, we get A = ,B =
2t 2 + 2t + 1 34 34
1 (−3sin x + 5cos x)dx 21
 1   1  2 1 
2t2 + 2t + 1 = 2 t 2 + t +  = 2   t +  + 
∫ = 34 ∫ (3cos x + 5sin x) + 34 ∫ dx
 2  2  4
  1 21x
= log(3cos x + 5sin x) + +C
Substituting, 34 34

Integration by parts method

Integration by parts method may be used in cases when which is called the rule for integration by parts.
the integrand in an indefinite integral is the product of two The above rule may be stated as:
functions. Integral of (u × differential of v) = u × v -
Let u and v represent two functions of x. We have integral of (v × differential of u)
d du dv OR
(uv) = v +u
dx dx dx If f(x) and g(x) are two functions of x,
or d(uv) = vdu + udv
∫
f (x)g(x)dx = f(x) × integral of g(x) -
⇒ ∫ d(uv) = ∫ vdu + ∫ udv
⇒ uv = ∫ vdu + ∫ udv ∫ int egral of g(x) × [differential of f(x)]
In the product f(x) g(x) (whose integral is sought), f(x)
⇒ ∫ udv = uv − ∫ vdu may be called ‘first function’ and g(x) may be called ‘second
3.18  Integral Calculus

function’. Then, the rule for integration by parts may be ex- On the other hand, had we chosen the first function as
pressed as cos x and the second function as x, application of the rule
for integration by parts leads us to
∫ (1st function × 2nd function) dx = 1st function ×
x2 x2
integral of 2nd function ∫ x cos xdx = (cos x) 2
− ∫ × (− sin x)dx
2
- ∫ integral of 2nd function × (differential of 1st
x 2 cos x 1 2
2∫
function) = + x sin x dx
2
As an illustration, let us consider the problem of evalu- Observe that the application of the rule has only made
ation of ∫ x cos xdx . If we choose 1st function as x and 2nd the problem more difficult in the sense that we encounter
∫x
2
function as cos x, application of the rule for integration by sin xdx on the right side.
parts gives us It is therefore very essential that, in the application of
integration by parts method, which one is to be chosen as
∫ x cos xdx = x × integral of cos x - ∫ integral of cos x 1st function and which one is to be chosen as 2nd function
× differential of x matters much in the reduction. If one of the functions in
= x sin x - ∫ sin x × 1dx = x sin x - ∫ sin xdx the product is a polynomial in x, choosing the polynomial
as the first function usually clicks. Also, the integral of the
= x sin x + cos x + C 2nd function must be known.

C o nce p t S t r a n ds
Concept Strand 25  sin 2x   sin 2x 
(ii) ∫ x cos2xdx = x ×  2   −∫
 2 
× 1dx
Evaluate ∫ xe x dx .
(by choosing f(x) as x and g(x) as cos2x)

Solution x sin 2x cos2x


= + +C
2 4
Choose f(x) as x, g(x) as ex
Applying the integration by parts rule, Substituting for ∫ x cos2xdx in (1),

∫ xe dx = xe − ∫ e dx = xe
x x x x
- ex + C
x 2 1  x sin 2x cos2x 
∫ x sin x dx =
2
− + +C
4 2  2 4 

Concept Strand 26
Evaluate the following integrals: Concept Strand 27
∫ x sin xdx
2
(i)
Evaluate ∫ x 2 e 7 x dx
(ii) ∫ x cos2xdx
Solution
Solution Choosing x2 as the 1st function and e7x as the 2nd
x(1 − cos2x) function,
∫ x sin xdx = ∫
2
(i) dx
2 e7 x e7 x
∫x e −∫
2 7x
dx = x 2 × × 2xdx
1 1 x2 1 7 7

=
2 ∫ xdx − ∫ x cos2xdx =
2 4 2∫
− x cos2xdx
x 2 e7 x 2
= − ∫ xe 7 x dx — (1)
— (1) 7 7 

Integral Calculus  3.19

Concept Strand 28
∫ xe
7x
For dx , we choose x as the 1st function and e7x
as the 2nd function, and applying the rule, Evaluate ∫ log xdx .
7x 7x
e e
∫ xe 7 ∫ 7
7x
dx = x × − × 1dx Solution


xe
=

e 7x 7x
∫ log xdx = ∫ (log x) × 1dx (choose the 1st function as
7 49 log x and the second function as 1)
Substituting in (1), 1
x 2 e7 x 2 7 x 2 7x
∫ log xdx = (log x)x − ∫ x × x dx = x logx - x + C
∫x e
2 7x
dx = − xe + e +C
7 49 343
Concept Strand 29
Remark
Evaluate ∫ x 2 log xdx .
If the integrand is of the form xnh(x), where n is a positive
integer, denoting xn by u and denoting h(x) by v, a gen-
eralized form of the rule for integration by parts may be
Solution
written as (Choose the first function as (log x) and the second func-
tion as x2)
∫ uvdx = uv -1
- u1v-2 + u2v-3 - …….,
x3 x3 1
∫ x log xdx = (log x) − ∫ × dx
2
where a positive suffix k represents differentiation of the 3 3 x
function k times with respect to x and a negative suffix
represents integration of the function k times with respect x 3 log x 1 2 x 3 log x x 3
= − ∫ x dx = − +C
to x. 3 3 3 9
Since u is a polynomial in x, the right side will be a
finite expression (For example, if u is a polynomial of de- Concept Strand 30
gree n, un+1 = un+2 = …..= 0 and therefore, the right side will
have (n + 1) terms. It is assumed that v-1, v-2, v-3, ……can Evaluate ∫ tan −1 xdx .
be evaluated.)
As an illustration, consider ∫ x 4 e 5 x dx . We choose x4 Solution
as u and e5x as v
∫ tan xdx = ∫ (tan −1 x) × 1dx
−1
Applying the generalized rule,
 e5x  3 e
5x
 1

4 5x 4
x e dx = x ×  5  − (4x )  25  + = (tan −1 x)x − ∫ x × dx
(1 + x 2 )
 e5x   e5x   e5x  1
(12x 2 )  − (24x) + 24  3125  + C = x tan-1 x - log(1 + x2) + C
 125   625 
2

Results Method I

e ax  sin bx  sin bx
C = ∫ e ax cos bxdx = e ax × 
 b  ∫ b
− × ae ax dx
∫e
ax
(i) cos bxdx = [a cos bx + b sin bx] + C
(a + b 2 )
2

e ax 1 ax a
= e sin bx − S
∫ e sin bxdx =
ax
(ii) [a sin bx − b cos bx] + C b b
(a 2 + b 2 )
(in the above, a and b are constants and C is the inte- ⇒ bC + aS = eax sin bx  — (1)
gral constant) Again,
We outline the proofs of the above.
Let C = ∫ e ax cos bxdx and S = ∫ e ax sin bxdx S = ∫ e ax sin bxdx
3.20  Integral Calculus

 − cos bx   − cos bx  ⇒ C = Real part of ∫ e


(a +ib)x
dx
= e ax ×   −∫ ax
 × ae dx
 b   b
∫e
(a +ib)x
S = Imaginary part of dx
e ax cos bx a
=− + C
b b e (a +ib)x e ax
⇒ aC - bS = eax cos bx — (2) Now, ∫ e (a +ib)x dx = = 2
(a + ib) (a + b 2 )
{
(a − ib)e ibx }
Solving for C and S, we obtain
e ax e ax
C= (a cos bx + b sin bx) =
(a 2 + b 2 )
{(a − ib)(cos bx + i sin bx)}
(a + b 2 )
2

e ax It is very easy to infer that


and S = (a sin bx − b cos bx)
(a 2 + b 2 ) e ax
C= (acos bx + b sin bx) and
Method 2 (a + b 2 )
2

e ax
C + iS = ∫ e ax (cos bx + i sin bx)dx (where i = −1) S= (asin bx - b cos bx)
(a 2 + b 2 )
= ∫ e ax × e ibx dx = ∫ e (a +ib)x dx (Arbitrary constant C is to be added finally)

Integrals of the form ∫ ax 2 + bx +c dx

For the evaluation of integrals of the form above, we list the 1


relevant formulas to be used: = x2 − a2 × x − ∫ x × × 2xdx
2 x − a2
2

x a2 − x2 a2 x x2
(i) ∫ a 2 − x 2 dx =
2
+ sin −1   + C
2 a  = x x2 − a2 − ∫ dx
x2 − a2
x x2 − a2 (x 2 − a 2 ) + a 2
(ii) ∫ x 2 − a 2 dx =
2
− = x x2 − a2 − ∫ dx
x2 − a2
( )
2
a
log x + x 2 − a 2 + C dx
2 = x x 2 − a 2 − ∫ x 2 − a 2 dx − a 2 ∫
x2 − a2
2 2
x x +a
(iii) ∫ x 2 + a 2 dx =
a
+

(
= x x 2 − a 2 − I − a 2 log x + x 2 − a 2 )
( )
2

( )
a
log x + x 2 + a 2 + C ⇒ 2I = x x 2 − a 2 − a 2 log x + x 2 − a 2
2
We sketch the proof of (ii)
− log ( x + −a )+C
x x2 − a2 a 2
or I= x2 2
Let I = ∫ x − a dx = ∫ x − a × 1dx
2 2 2 2
2 2

C o nce p t S t r a n ds
Concept Strand 31 Solution
1
Evaluate: ∫ 2x 2 + x − 4dx . 2x2 + x - 4 = 2(x2 +
2
x - 2)
Integral Calculus  3.21

   Substituting in (1),
1  33 
2 2
1 1
= 2  x +  − − 2 = 2  x +  −  we obtain the answer.
  4  16    4  16 
2
 1  33 Concept Strand 33
∫ = 2 ∫  x + 4  − 16 dx
dx
 1  1 33 33
Evaluate ∫ (2 + 3x) 1+ x
.
= 2 x +  x + 2 − −
 4  4  16 32
 Solution
1  33 
2
1 
log  x + +  x +  −  + C Put 1 + x = t2 ⇒ dx = 2t dt
 4  4  16 
 
2tdt dt 2 dt
Concept Strand 32
∫ = ∫ [2 + 3(t
2
− 1)]t
= 2∫ 2
3t − 1
= ∫
3 1
t2 −
3
Evaluate: ∫ (4x + 7) 1 − 2x − 3x 2 dx .
 1 
t−
2 3  3
Solution = × log  +C
1 
3 2 t+
We write  
3
d  3(1 + x) − 1 
4x + 7 = A (1 - 2x - 3x2) + B 1
dx = log  +C
3  3(1 + x) + 1 
= A(-2 - 6x) + B
-6A = 4, -2A + B = 7
−2 17 Concept Strand 34
giving A = ,B=
3 3
dx
−2
∫ = 3 ∫ (−2 − 6x) 1 − 2x − 3x dx +
2 Evaluate ∫ (2 + 5x) 1 + x 2 + 5x
.

17
3∫
1 − 2x − 3x 2 dx Solution
3
1
=
(
−2 1 − 2x − 3x
2
) 2
17
+ I  — (1)
Put 2 + 5x =
t
3 3 3 −1
 2  ⇒ 5dx = 2 dt
t
I: 1 - 2x - 3x2 1 
  t − 2 
1  1 1
2
 2 1
= −3  x 2 + x −  = −3   x +  − −  x=
 3 3   3  9 3  5
1 − 2t
 1 4
2
4  1 
2
=
= −3   x +  −  = 3  −  x +   5t
  3  9   9  3   Substituting,
2 dt
4  1 −
I = 3∫ − x +  dx 5t 2
9  3 ∫ =∫ 2
  1  1 − 2t  5(1 − 2t)
 t  1 +  5t  + 5t
2
1 4  1
  x +  − x +  
x+
1 
 3 9  3 4  3  + C dt
= 3 + sin −1   − × (5t)
 2 18 2
  =∫ 5t
  3 
  25t + (1 − 2t)2 + 25t(1 − 2t)
2
3.22  Integral Calculus

= −∫
dt
2
= −∫
dt
=
−1
 (2t − 1)

sin −1 
( 21 )  + C ,
1 + 21t − 21t  25  1   2
5
21  −  t −   21  
 84  2  
 1 1
 t −  where, t =
−1 2 + 5x
= sin −1  2 
21  5 
 84 

Evaluation of definite integrals

We have already defined a definite integral as the limit Since evaluation of definite integrals hinges on antide-
of a sum. The fundamental theorem of integral calculus rivatives we first took up the problem of indefinite integra-
provides us a method to evaluate a definite integral. tion (or problem of finding anti derivatives of functions).
The fundamental theorem is reproduced here. Having discussed the various methods for finding antide-
If f(x) is continuous in [a, b], then, there exists a dif- rivatives, we now switch back to the main task, namely, the
b
evaluation of definite integrals. It is clear that the definite
ferentiable function F(x) such that ∫ f (x)dx = F(b) - F(a), b

where F(x) is an anti-derivative of f(x)


a
integral ∫ f (x)dx can be evaluated once we know an anti
a
i.e., F’(x) = f(x) derivative of f(x).
or F(x) = ∫ f (x)dx (indefinite integral of f(x)) We illustrate by a few examples.

C o nce p t S t r a n ds
Concept Strand 35  34 2 × 33 32 
= − + + 5 × 3 + C −
3 4 3 2 
Evaluate ∫ (x 3 − 2x 2 + x + 5)dx . 1 2 1  50
1  4 − 3 + 2 + 5 + C  = 3

Solution [Observe that C does not enter into our final com-
putation of the definite integral. Therefore, C is omitted
x 4 2x 3 x 2 in F(x) (or in the anti-derivative) when we are evaluating
∫ (x − 2x + x + 5)dx =
3 2
− + + 5x + C definite integrals].
4 3 2
x 4 2x 3 x 2 Concept Strand 36
Therefore, F(x) = − + + 5x + C
4 3 2
2
⇒ Definite integral Evaluate ∫ e 4 x dx .
= F(3) - F(1) 0

3 Solution
 x 4 2x 3 x 2 
= − + + 5x + C  2
 e4x 
2
e8 − 1
4 3 2  x =1 ∫0 e dx =  4  = 4
4x

0
Integral Calculus  3.23

Concept Strand 37 π
2 π
π
2
Therefore, ∫ cot xdx =  log sin x  2π
Evaluate: ∫ cot x dx
π 4
4
π
4  π  π
= log  sin  − log  sin 
 2  4
Solution
 1  1
= 0 − log  = log 2
∫ cot xdx = log sin x  2  2

Properties of definite integrals

b b b 2a a

(i) ∫ f (x)dx = ∫ f (t)dt = ∫ f (y)dy


a a a
(xi) ∫ f (x)dx = 2 ∫ f (x)dx, if f(2a - x) = f(x)
0 0

Since the definite integral is the limit of a sum, it does and = 0, if f(2a-x) = -f(x)
not involve the variable of integration. b b
b a (xii) ∫ f (x)dx = ∫ f (a + b − x)dx
(ii) ∫ f (x)dx = − ∫ f (x)dx
a b
a a

b b

∫ f (x)dx ≤ ∫ f (x) dx
b
(xiii)
(iii) ∫ kdx = k(b - a), where k is any constant.
a
a a

b b b
(xiv) Area bounded by the curve y = f(x), x-axis and the
∫ k 1f1 (x) ± k 2 f2 (x) dx = k 1 ∫ f1 (x)dx ± k 2 ∫ f2 (x)dx
b
(iv)
a a a
ordinates at x = a and x = b is given by ∫ f (x) dx
a
(where, k1 and k2 are constants and f1(x) and f2(x) are (If f(x) ≥ 0 in (a, b), area under the curve y = f(x),
continuous in [a, b]) x - axis and the ordinates at x = a and x = b is given
(v) If c lies between a and b, b
b c b by ∫ f (x)dx )
∫ f (x)dx = ∫ f (x)dx + ∫ f (x)dx
a a c
a

b We outline the proofs of (x) and (xi)


(vi) If f(x) ≥ 0 in a ≤ x ≤ b, ∫ f (x)dx ≥ 0 a
a
(x) Consider ∫ f (a − x)dx
b b
0
(vii) If f(x) ≥ g(x) in a ≤ x ≤ b, ∫ f (x)dx ≥ ∫ g(x)dx Put a - x = t ⇒ -dx = dt
a a
(viii) If m and M are the minimum and maximum values when x = 0, t = a and when x = a, t = 0
a 0 0
b

of f(x) in [a, b], m(b - a) ≤ ∫ f (x)dx ≤ M(b - a) ∫ f(a − x)dx = ∫ f(t)(−dt) = − ∫ f(t)dt
0 t =a a
a
a a
a a
= ∫ f(t)dt = ∫ f(x)dx
(xi) If f(x) is an even function, ∫ f (x)dx = 2 ∫ f (x)dx
−a 0
and
0 0

2a a 2a
a

if f(x) is an odd function, ∫ f (x)dx = 0 (xi) ∫ f (x)dx = ∫ f (x)dx + ∫ f (x)dx — (1)


−a
0 0 a 
a a 2a

(x) ∫ f (x)dx = ∫ f (a − x)dx Now, consider ∫ f (x)dx


a
0 0
3.24  Integral Calculus

Put x = 2a - t ⇒ dx = -dt π /2
(v)
∫ sin x cos x dx =
m n
When x = a, t = a and when x = 2a, t = 0
2a a 0

∫ f(x)dx = ∫ f(2a − t)(−dt) [(m − 1)(m − 3)(m − 5)...3.1][(n − 1)(n − 3)...3.1] × π ,


a 0
a a (m + n)(m + n − 2)(m + n − 4)...4.2 2
= ∫ f(2a − t)dt = ∫ f(2a − x)dx if both m and n are even

0 0
a a (m − 1)(m − 3)(m − 5)...2 or 1
Suppose f(2a - x) = f(x), then ∫ f (2a − x)dx = ∫ f (x)dx , (n − 1)(n − 3)...2 or 1
0 0
= , otherwise
which means that, in this case, (1) reduces to (m + n)(m + n − 2)(m + n − 4)...2 or 1

2a a a a
As examples to illustrate the (iv) and (v), consider the
∫ f (x)dx = ∫ f (x)dx + ∫ f (x)dx = 2 ∫ f (x)dx definite integrals.
0 0 0 0
π/2
On the other hand, suppose f(2a - x) = -f(x),
∫ sin
8
a a
(a) x dx
∫ f (2a − x)dx = − ∫ f (x)dx , which means that, in this
0
π/2
0 0
∫ cos
7
case, (1) reduces to (b) x dx
2a a a 0

∫ f (x)dx = ∫ f (x)dx − ∫ f (x)dx = 0


π/2

∫ sin
10
(c) x cos 6 x dx
0 0 0
0
π/2

Results ∫ sin
5
(d) x cos 8 x dx
0

If m and n are positive integers, π/2

∫ sin
3
(e) x cos 9 x dx

0 , m≠n 0
(i) ∫ sin mx sin nxdx = 
π , m=n 7.5.3.1 π 105 π
π/2

∫ sin
8
0
(a) x dx = × = ;

 0. m≠n 0
8.6.4.2 2 768
(ii) ∫ cos mx cos nxdx =  π/2
π , m=n 6.4.2 16
∫ cos
7
0
(b) x dx = = ;

0
7.5.3.1 35
(iii) ∫ sin mx cos nxdx = 0 π/2
(9.7.5.3.1)(5.3.1)  π 
∫ sin
10
0 (c) x cos 6 x dx = ×
π /2 π /2
0
16.14.12.10.8.6.4.2  2 
(iv)

0
sin m x dx = ∫0
cos m x dx
=
45
π
(m − 1)(m − 3)(m − 5)...3.1 π 65536
= × , π/2
m(m − 2)(m − 4)...4.2 2 (4.2)(7.5.3.1) 8
∫ sin
5
(d) x cos 8 x dx = ;=
if m is even and 0
13.11.9.7.5.3.1 1287
π/2
(m − 1)(m − 3)(m − 5)...4.2 2.(8.6.4.2) 1
∫ sin
3
= , if m is odd (e) x cos 9 x dx = =
m(m − 2)(m − 4)...3.1 0
12.10.8.6.4.2 60

Improper Integrals

In defining a definite integral, we considered a function f b

defined on a finite interval [a, b]. If f(x) is continuous in


integral ∫ f (x)dx exists. We now extend the concept of a
a
[a, b] (or f(x) is piecewise continuous in [a, b]), the definite definite integral to the case where the interval of integra-
Integral Calculus  3.25

tion is infinite and also to the case where f(x) has an infinite b b
discontinuity in [a, b]. In either case, the integral is called (ii) If ∫ f (x)dx exists for every r ≤ b, then ∫ f(x)dx
an “improper integral”. r −∞

= lim ∫ f (x) dx , provided the limit on the right side


Infinite intervals r→− ∞
r
exists (limit is a finite number).
Consider the infinite region S bounded by the curve y =
If both the limits in (i) and (ii) exist, we say that
3
, above the x axis and to the right of the line x = 1. ∞ b
x2 the improper integrals ∫ f (x)dx and
a
∫ f (x)dx are
−∞
convergent. If these limits do not exist, we say that
y both the improper integrals are divergent.
For example,
y = 3/x2

1
∫ x dx does not exist.
2

∞ r
1 1
We have, ∫2 x dx = lim
r →∞ ∫ x
2
dx

= lim(log r − log 2).


r →∞

x Since, lim log r is infinite, the improper integral


O x=1 x=r r →∞

Fig. 3.13 1
∫ x dx
2
does not exist.
1

∫ xe
2x
The area of the part of S that lies to the left of the line Again, consider the integral dx .
−∞
x = r (shaded portion in Fig. 3.13) is given by
r x e2 x e2 x
We have ∫ x e 2 x dx =
r
3  3 3 −
A(r) = ∫ 2 dx =  -  = 3 - 2 4
1 x  x 1 r
1 1

∫ xe dx = lim ∫ x e2x dx
2x
Note that whatever be the value of r (i.e., no matter how r →−∞
−∞ r
large r is chosen) A(r) < 3. As r → ∞, A(r) → 3. Therefore,   e2 e2  
the area of the infinite region S is equal to 3 and we express = lim   −  − re2r   — (1)
r →−∞  2 4 
 

3
r
3
this result as ∫ x dx = lim ∫ x
2 r →∞ 2
dx = 3.
r  ∞ 
1 1 Now, lim re2r = lim  = ∞ form 
−2r
Now, we are in a position to define the integral of a
r →−∞ r →−∞ e
function f (not necessarily a positive function) over (a, ∞)  1 
or over (-∞, b) = lim   , by L Hospitals rule = 0.
r →−∞  −2e −2 r

1
Definition Therefore, e2 e2

−∞
x e2x dx = − −0
2 4
r ∞ r
e2
= .
(i) If ∫ f (x)dx exists for every r ≥ a, then
a
∫ f (x)dx = lim ∫ f (x)dx
a
r→ ∞
a
4
∞ r 1

∫ f (x)dx = lim ∫ f (x)dx , provided the limit on the right side ∫ xe


2x
⇒ The improper integral dx exists (or the
r→ ∞
a a −∞

exists (limit is a finite number). integral is convergent).


3.26  Integral Calculus

∞ a b(x)

(iii) If both ∫ f (x)dx and ∫ f (x)dx exist, Differentiation rule for ∫ f(t)dt , where a(x) and
a −∞ a(x)
∞ ∞ a ∞
b(x) are functions of x
we define ∫ f (x)dx as
−∞
∫ f (x)dx =
−∞
∫ f (x)dx + ∫ f (x)dx
−∞ a
Let ∫ f (t)dt = F(t)
Discontinuous integrals (i.e., F’(t) = f(t))
2
1 1 b(x )
Consider the definite integral ∫ dx . The integrand is
−2
x x
Then ∫
a(x )
f (t)dt = F(b(x)) − F(a(x))
discontinuous at x = 0 (The integrand has infinite discon- b(x )
d d
tinuity at x = 0).
We now define an improper integral of this type. ∫
dx a(x )
f (t)dt =
dx
{F(b(x)) − F(a(x))}
(i) If f(x) is continuous in [a, b) and is discontinuous at b, d d
then = F '(b(x)) ×
(b(x)) − F '(a(x)) × (a(x))
b r dx dx
db(x) da(x)
∫ f (x)dx = lim− ∫ f (x)dx , provided the limit on the left
r →b
= f (b(x)) ×
dx
− f (a(x)) ×
dx
a a
side exists (limit is a finite number). As an illustration,
(ii) If f(x) is continuous in (a, b] and is discontinuous at a, x4
then d
dx x∫2
(3t 2 + 1)dt = [3(x4)2 + 1] × 4x3 - [3(x2)2 + 1] × 2x
b b

∫ f (x)dx = lim+ ∫ f (x)dx , provided the limit on the


r →a = 4x3 (3x8 + 1) - 2x(3x4 + 1)
a r
right side exists (limit is a finite number).
Observations
b

The improper integral ∫ f (x)dx


a
is said to be conver-

gent if the corresponding limits exist. d


b(x )
db(x)
(iii) If f(x) has a discontinuity at c, where a < c < b, and both (i) If a(x) = a constant k,
dx ∫
k
f (t)dt = f (b(x)) ×
dx
c b

∫ f (x)dx and ∫ f (x)dx are convergent, then we define


a c
(ii) If a(x) = k (constant) and b(x) = x, then,
x
b c b d
dx ∫k
f (t)dt = f (x)
∫ f (x)dx = ∫ f (x)dx + ∫ f (x)dx
a a c

We now work out a few examples in the:


(i) evaluation of definite integrals using their properties.
(ii) computation of areas of plane regions using definite integrals.

C o nce p t S t r a n ds
Concept Strand 38 Solution
π
2
5
Let I represent the required definite integral.
(sin x) 2
Evaluate ∫ 5 5
dx .
 π  2
5

0
(sin x) + (cos x)
2 2 π
2 sin  2 − x  
 
I= ∫ 5 5
dx
0
 π  2   π  2
sin  2 − x   +  cos  2 − x  
   
Integral Calculus  3.27
π
sin x
a a ⇒ 2I = π ∫ dx
(since ∫ f (x)dx = ∫ f (a − x)dx ) 0
1 + sin x
π π
0 0
 1   1 − sin x 
π 5 = π ∫ 1 −  dx = π ∫ 1 −   dx
 1 + sin x cos 2 x 
2
(cos x) 2 0 0
=∫ 5 5
dx π
0
(cos x) + (sin x) 2 2 = π ∫ [1 − sec2 x + sec x tan x]dx
0
π 5 5 π
= π [ x − tan x + sec x ]0
π
2 2
(sin x) + (cos x) 2 2
π
2I = ∫ dx = ∫ dx =
5 5
2 = p[(p - 1) - 1] = p(p - 2)

0
(sin x) 2 + (cos x) 2 0

1
π ⇒ I = p(p - 2)
⇒ I = 2
4
Concept Strand 42
Concept Strand 39
4
x
2

Evaluate ∫ x(2 − x) dx . 10
Evaluate ∫
1 5−x +x
dx .
0

Solution
Solution
We shall first establish the result
2 2 2
b b
I = ∫ x(2 − x) dx = ∫ (2 − x)x dx = ∫ (2x − x )dx
10 10 10 11

0 0 0
∫ f (x)dx = ∫ f (a + b − x)dx
a a

 2x x  11
2 2 2 2 12
2
12 12 12 11 Put a + b - x = t ⇒ -dx = dt
= −  = − = = x = a ⇒ t = b and x = b ⇒ t = a
 11 12  0 11 12 11x12 66 a a b

R.H.S = − ∫ f (t)dt = − ∫ f (x)dx = ∫ f (x)dx = L.H.S


Concept Strand 40 b b a

π Now,
2
(sin x − cos x)dx 4
x 5−x
4
Evaluate ∫ 1 + sin x cos x
. I=∫ dx = ∫ dx ,
0
 1 5−x + x 1 x + 5−x
using the above result
Solution 4 4
5 + 5−x 3
π π Hence, 2I = ∫ dx = ∫ dx = 3 ⇒ I =
2
(sin x − cos x)dx (cos x − sin x)dx 2 1 5−x + x 1
2
I=∫ =∫ = -I
1 + sin x cos x 1 + cos x sin x
0 0
Concept Strand 43
⇒ I = 0
π
6
1
Concept Strand 41 Evaluate: ∫1+
π cot x
dx.

π 3
x tan x
Evaluate: ∫ sec x + tan x dx .
0 Solution
Denoting the required integral as I,
Solution
π π
π π
(π − x)tan( π − x)dx ( π − x)tan x 6
1 6
1
I=∫ =∫ dx I=∫ dx = ∫
0
sec( π − x) + tan( π − x) 0
sec x + tan x π π π  π 1 + tan x
π 3 1 + cot  + − x  3
tan x 3 6 
= π∫ dx − I
sec x + tan x
0
3.28  Integral Calculus

π π Concept Strand 46
6 6
sin x cos x
⇒ 2I = ∫ dx + ∫ dx 2x, 0 < x ≤ 1
π sin x + cos x π cos x + sin x 
3 3  πx
If f(x) is given by, f (x) = sin , 1 < x < 3 , evaluate
 22
π
6
 π π  −π π 2x − 19, 3 ≤ x ≤ 4
= ∫ dx =  −  = ⇒ I=−
 6 3  6 12 4

∫ f (x)dx .
π
3
0

Concept Strand 44 Solution


1
x sin −1 x We have,
Evaluate ∫
−1 1− x 2
dx . 4 1 3
πx
4

∫ f (x)dx = ∫ 2xdx + ∫ sin 2


dx + ∫ (2x 2 − 19)dx
Solution 0 0 1 3

3 4
 −2 πx   2x 3 
1
x sin x −1 1 −1
x sin x = (x 2 )10 +  cos  +  − 19x 
∫ 1− x 2
dx = 2 ∫
1 − x2
dx ,  π 2 1  3 3
−1 0

 2 × 64  20
x sin −1 x =1+0 +  − 76  − (18 − 57) =
since f (x) = = f(-x)  3  3
1 − x2
Put x = sin q ⇒ dx = cos q dq
π Concept Strand 47
x = 0 ⇒ q = 0 and x = 1 ⇒ q =
2 Find the area bounded by the curve y = x2 + 4 , x-axis and
π
2 the ordinates at x = 1 and x = 5.
θ sin θ cos θdθ
Required integral = 2 ∫
0
cos θ
π
y y=x +4
2
2 π
= 2 ∫ θ sin θdθ = 2  −θ cos θ + sin θ = 2 2
0
0

Concept Strand 45
50

∫e
x −[x ]
Evaluate dx where [ ] represents the greatest
x
0 A’ O x=1 x=5
integer function.

Solution
The function g(x) = x - [x] is periodic with period 1.
In (0, 1), ex -[x] = ex In (1, 2), ex-[x] = ex - 1 and so on. Solution
i.e., f(x) = ex - [x] is periodic with period 1
5 5
or f(x + 1) = ex + 1 - [x + 1] = ex + 1 - [x] - 1 = ex - [x] = f(x)  x3 
Required area = ∫ (x + 4)dx =  + 4x 
2
⇒ f(x) periodic with period 1 1 3  1
50 1
 125  1  124 172
∫e dx = 50 ∫ e dx = 50(e − 1)
x −[x ] x
Therefore, = + 20  −  + 4  = + 16 =
0 0
 3  3  3 3
Integral Calculus  3.29

Concept Strand 48 The points of intersection of the curves are at (-1, 2)


and (1, 2)
x2 y2
Find the area of the ellipse + =1 .
a 2 b2 y
2
y=x +1
Solution A

M L
y B
x
−1 O 1
B
y = 3 − x2

x
A’ O A

B’ By symmetry of both curves,


Required area = 2 × Area of the region ABL
1

= 2 ∫ (y 1 − y 2 )dx
0

where y1 = 3 - x2 , y2 = x2 + 1
Required area = area of the region A BA’ B’ 1
= 4 times the area of the region OAB Area = 2 ∫ [(3 − x 2 ) − (x 2 + 1)]dx
y2 x2
= 4 ∫ ydx where y is given by 2 = 1 − 2 0

b a 
1
x3 
1
8
4b
a
= 2 ∫ (2 − 2x 2 )dx = 4  x −  =
a ∫0
= a 2 − x 2 dx 0  3  0
3

Putting x = a sin q and noting that the limits of inte-


Concept Strand 50
π
gration for q are from 0 to Find the area bounded by the curve y =(x + 2) (1 - x) and
2
π π
the ordinates at x = 0 and x = 3.
4b 2 2 2

a ∫0 ∫0 (1 + cos2θ)dθ
2
Required area = a cos θd θ = 2ab
Solution
π
 sin2θ 2 y
= 2ab  θ + = πab
 2  0

Concept Strand 49
−2 1 3
Find the area bounded by the curves y = x2 + 1 and x
y = 3 - x2. O

y = (x + 2) (1 − x)
Solution
To find the points of intersection of the curves, we solve
y = x2 + 1 and y = 3 - x2
3 - x2 = x2 + 1 The shaded region in the figure represents the required
⇒ 2x2 = 2 ⇒ x = ±1 area. Since the portion of the curve between 1 and 3 is
When x = ±1, y = 2 below the x - axis,
3.30  Integral Calculus

1 3 Solution
Required area = ∫ (x + 2)(1 − x)dx − ∫ (x + 2)(1 − x)dx
0 1 r
4
1 3 n
4r n
n2
= ∫ (2 − x − x 2 )dx − ∫ (2 − x − x 2 )dx ∑ 2
r =1 n + 4r
2
=∑
r =1 4r 2
0 1 1+ 2
1 3 n
 x x  
2 3
x2 x3 
=  2x − −  −  2x − −   2   2r 
 2 3 0  2 3 1 2r
n  n   n 
2 n
=∑ = ∑ n
 1 1   9   1 1  2
n 2
= 2 − −  − 6 − − 9 − 2 − −  r =1  2r  r =1  2r 
 2 3   2   2 3  1+   1+  
n n
7 9 59
= +3+ = 2
3 2 6 x
= Riemann sum corresponding to ∫1+ x
0
2
dx
Concept Strand 51 n
4r
lim ∑
 1 1 1 1  n →∞ n 2
+ 4r 2
Evaluate lim  + + + .... +  . r =1
n →∞ n + 1 n +2 n +3 2n 
 2 2
x 1  1
=∫ dx =  log(1 + x 2 ) = log 5
Solution 0 1 + x 2
 2 0 2

1 1 1 1 1
+ + + ... + = + Concept Strand 53
n +1 n + 2 n + 3 n+n  1
n 1 + 
 n If f(x) is a continuous function of x such that
x x
1 1 1
+ + ... + ∫ f (t)dt = xe + ∫ e f (t)dt for all x, find an explicit for-
2x −t

 2  3  n
n 1 +  n 1 +  n 1 +  0 0
 n  n  n mula for f(x).
1 n
1
= ∑ r
n r =1 Solution
1+
n
Differentiating the given relation,
Therefore, f(x) = 2xe2x + e2x + e-xf(x)
 1 1 1  1 n 1 (1 + 2x)e 2 x
lim  + + .... +  = lim ∑ ⇒ (1 - e-x)f(x) = (1 + 2x)e2x ⇒ f (x) =
n →∞  n + 1 n +2 2n  n→∞ n r =1  r  (1 − e − x )
1 + n 

1
Concept Strand 54
1
= lim [Riemann sum corresponding to ∫ dx ] Find a function “f ” and number ‘a’ such that
n →∞
0
1+ x x
f (t)
1
1 6+∫ 2
dt = 2 x .
= ∫ dx , by the fundamental theorem of integral a t
0
1 + x
calculus Solution
1
=  log(1 + x) 0 = log 2 Differentiating the given relation,
f (x) 1 1
0+ = 2x =
Concept Strand 52 x2 2 x x
3
n
4r f(x) = x 2
Evaluate lim ∑ 2 .
n →∞
r =1 n + 4r 2 substituting for f in the given relation,
Integral Calculus  3.31

3
x
t2 ⇒ 6 + 2 x − 2 a = 2 x
6 + ∫ 2 dt = 2 x
a t ⇒ 2 a = 6
x
x
−  1
1
⇒ 6 + ∫ t dt = 2 x ⇒ 6 + 2t 2  = 2 x
2
⇒ a =3
a  a a=9

Differential Equations

Definition (xi) Laws governing many physical phenomena, when


formulated mathematically lead us to differential
A relation, which contains, besides the dependent and equations. As an illustration, consider a particle
independent variables, derivatives of different orders of moving along a straight line such that the velocity
the dependent variable with respect to the independent at any time t is always (3t2 + 5t - 1). If x represents
variable or variables is called a differential equation. The the displacement of the particle in time t, we may
relation may contain constants also. If there is only one write the law governing the motion of the above
independent variable, the corresponding equation is called particle as
an ordinary differential equation. dx
In what follows, differential equations mean ordinary = 3t2 + 5t - 1,
dt
differential equations. which is a differential equation where t is the
Examples independent variable and x is the dependent variable.
d2 x
dy (xii) Again, = a constant, represents the differential
(i) = x3 + 3x - 2 dt 2
dx equation of the motion of a particle along a straight
dy line with constant acceleration.
(ii) = 4y - 7 + e-y
dx (xiii) Newton’s law of cooling states that the difference “x”
dy between the temperature of a body and that of the
(iii) + 7y = 2x2 + cos3 x surrounding air decreases at a rate proportional to
dx
this difference. When formulated mathematically,
d2 y dy this law may be expressed as the differential equation
(iv) 2 −4 + 5y = 7x + e 4 x
dx 2
dx dx
= -kx, k > 0
 dy 
3
dy dt
(v)   + 3 − 2xy 3 = 4 d2 x
 dx  dx (xiv) The differential equation + w2x = 0 characterizes
dt 2
d2 y or represents a simple harmonic motion.
(vi) + k 2 y = 24x (xv) Let us consider a geometrical problem. The differ-
dx 2
dy
3 4 ential equation = k (a constant) represents the
 d2 y   dy  d3 y dx
(vii)  2  + 3   − y 2 3 = 2 + 7x
 dx   dx  dx differential equation of a family of curves with a con-
3
stant slope k at all points (x, y) on any curve. It is
 d 4 y  dy clear that it is a family of parallel lines.
(viii) x  4  − + 2y = 1
 dx  dx
(ix) xdy + ydx = (x2 + y2)dy Order of a differential equation
dy The order of a differential equation is defined as the order
(x) (x 2 + y 2 ) + (1 − xy) = 2e 4 x
dx of the highest derivative present in the equation.
3.32  Integral Calculus

In the examples above, (i), (ii), (iii), (v), (ix), (x), (xi), y = Ce5x where “C” is an arbitrary constant represents solu-
(xiii) and (xv) are first order equations; (iv), (vi), (xii) and dy
(xiv) are second order equations; (vii) is a third order equa- tions of the equation = 5y .
dx
tion and (viii) is a fourth order equation. The “general solution” (or primitive) of a first order dif-
ferential equation is a relation between x and y involving
Degree of a differential equation one arbitrary constant such that the differential equation is
satisfied by this relation.
The degree of a differential equation is defined as the great- OR
est power of the highest order derivative, when the equation The general solution of a first order differential equa-
has been made rational and integral as far as the derivatives tion is a one parameter family of curves where the param-
are concerned. eter is the arbitrary constant. Coming back to our example,
In the examples above, (i), (ii), (iii), (iv), (vi), (vii), (ix), we say that y = Ce5x where C is an arbitrary constant repre-
(x), (xi), (xii), (xiii), (xiv) and (xv) are of first degree while sents the general solution (or primitive) of the differential
(v) and (viii) are of third degree. dy
Consider the differential equation equation = 5y .
5
dx
 d2 y 2 dy By assigning particular values to the arbitrary constant
 dx 2 + 3x  = y dx − 1 C, we generate what are called particular solutions of the
dy
equation = 5y . y = e5x, y = -7e5x, y = 23e5x….. are all
It is a second order equation. In order to determine its dx
degree, we free the equation from the fractional power, by dy
squaring both sides. We get particular solutions of = 5y , since these solutions are
dx
 d2 y
5
  dy 
2
obtained by assigning values 1, -7, 23 respectively to C in
 dx 2 + 3x  =  y dx − 1  y = Ce5x.
These particular solutions are graphically represented
The degree of the equation is 5. in Fig. 3.14.
Again, the differential equation To put it in another way, y = Ce5x where, C is an
3 arbitrary constant represents a family of exponential curves.
  dy  2  2 d2 y We may represent this family of curves either geometrically
1 +    = k 2 is of order 2 and degree 2. (Fig. 3.14) or by y = Ce5x where, C is an arbitrary constant
  dx   dx
dy
or by the differential equation = 5y .
dx
Solution of a differential equation
A function y = f(x) or F(x, y) = 0 is called a solution of a y
given differential equation if it is defined and differentiable y = 23e5x
(as many times as the order of the given differential equa-
tion) throughout the interval where the equation is valid,
and is such that the equation becomes an identity when
y = 8e5x y = e5x
dy d 2 y
y, , ,.... in the differential equation are replaced by
dx dx 2
f(x), f ’(x), f ’’(x),… respectively(In the case of F(x, y) = 0,
dy d 2 y O
x
one has to get the derivatives , ,.... by successive dif-
dx dx 2
ferentiations of F(x, y) = 0 and solving for the derivatives).
For example, y = −7e5x

x 4 3x 2
y= + − 2x + 4 is a solution of the example (i)
4 2
dy y = −10e−5x
Again, y = e5x is a solution of the equation = 5y
dx Fig. 3.14
. Note that y = 3e5x, y = -7e5x, y = 23e5x, …or in general,
Integral Calculus  3.33

In the case of a second order differential equation, the dy d2 y


general solution is a relation between x and y involving two On substituting for and in
dx dx 2
arbitrary constants. For example, y = Aex + Be3x where A
d2 y dy
and B are arbitrary constants is the general solution of the −4 + 3y = 0 ,
dx 2
dx
d2 y dy
second order equation −4 + 3y = 0 . we easily see that the equation is satisfied. By assigning par-
dx 2
dx ticular values to A and B, we can generate particular solu-
dy tions.
For, = Aex + 3Be3x and Therefore, the general solution of a second order dif-
dx
ferential equation is a two parameter family of curves.
d2 y In general, the general solution of an nth order differ-
= Aex + 9Be3x
dx 2 ential equation is an n parameter family of curves.

Formation of a differential equation

(i) As we have already seen, a physical phenomenon d2 y


involving rate of change of one quantity with respect and = 2B — (3)
dx 2
to another quantity, when formulated mathematically
gives rise to a differential equation. Similar is the case dy d2 y 1 d2 y
From (2) and (3) A = − x 2 and B =
with a geometrical result. The differential equation of dx dx 2 dx 2
a family of curves having the property that subnormal Substituting for A and B, the result of eliminating A
at any point (x, y) on a curve of the family is a constant and B in (1) is the relation
dy
k is given by y =k. d2 y dy
dx x2 − 2x + 2y = 0  — (4)
(ii) Consider the relation dx 2
dx
y = Cx3 —(1) (4) is a second order differential equation. Clearly, (1)
is the general solution of (4). Also, (1) represents a
where C is an arbitrary constant.
family of parabolas.
For eliminating C in (1), we need one more relation.
This can be obtained by differentiating (1) with respect
to x. We have y
dy
= 3Cx 2  —(2)
dx
dy 3y
Dividing, =  —(3)
dx x x
O
which is a first order differential equation.
We may say that (3) is the result of eliminating the
arbitrary constant C from (1). Clearly, (1) is the general
solution of (3). One parameter
(iii) Let us now consider the relation family of curves

y = Ax + Bx2 — (1)  dy 
⇒ f(x, y, C) = 0 ⇒ g  x, y,=0
where A and B are arbitrary constants.  dx 
For eliminating A and B in (1) we need two more Equation First order
relations. These can be obtained by differentiating (1) representing the differential
family (where C is equation
two times successively with respect to x. an arbitrary representing the
We have, constant) family
dy
= A + 2Bx — (2) Fig. 3.15
dx
3.34  Integral Calculus

y    Therefore, the 2-parameter family of parabolas (1)


can be represented by the second order differential
equation (4).
   We may sum up our findings diagrammatically:
x Refer Fig. 3.15 and Fig. 3.16.
O
   Note that f(x, y, C) = 0 is the general solution of
Two parameter family of curves  dy 
the differential equation of g  x, y,  = 0 ; and
  dx 
dy d2y 
⇒ F(x, y, A, B) = 0 ⇒ G  x, y, , 2  = 0 F(x, y, A, B) = 0 is the general solution of the differential
 dx dx 

Equation representing Second order differential  dy d 2 y 
equation G  x, y, , 2  = 0.
the family (where A and equation representing  dx dx 
B are arbitrary constants) the family
Fig 3.16

C o nce p t S t r a n ds
Concept Strand 55 Solution
Find the differential equation whose general solution is y = The general equation of a circle with centre at the origin is
e-4x(Ax + B) where A and B are arbitrary constants. of the form x2 + y2 = a2 where a is a parameter. To find the
differential equation of the family of such circles we have
to eliminate a in x2 + y2 = a2 . We differentiate the equation
Solution with respect to x .
The differential equation is obtained by eliminating the ar-
dy
bitrary constants A and B from the given relation. We get 2x + 2y =0
We have dx
ye4x = Ax + B — (1) dy
We note that x + y = 0 represents the differential
Differentiating (1) with respect to x, dx
dy equation of the family of circles with their centers at the
e4x + 4e 4 x y = A  — (2) origin.
dx
Differentiating (2) with respect to x,
 4 x d2 y 4 x dy   4x 4 x dy  Concept Strand 57
 e dx 2 + 4e dx  + 4 4e y + e dx  = 0
  Form the differential equation of the family of straight
2
d y dy lines which are at a distance of 1 unit from the origin.
⇒ +8 + 16y = 0  —(3)
dx 2
dx
(3) is the differential equation satisfied by (1) or, (3) is Solution
the differential equation whose general solution is (1).
The general equation of a straight line, which is at a dis-
tance of 1 unit from the origin may be written as
Concept Strand 56
x cos q + y sin q = 1
Form the differential equation of the family of circles with
where, q is an arbitrary constant (or parameter)
their centers at the origin.
Integral Calculus  3.35

To find the differential equation of the family of such 3


dy dy  dy 
lines, we have to eliminate q. Now, differentiating the ⇒ The differential equation is y = x + −
equation with respect to dx dx  dx 

 dy  dy
x, cos θ +   sin θ = 0 ⇒ = − cot θ Concept Strand 59
 dx  dx
Form the differential equation of the family of parabolas
From the general equation of the line, we get x cot q
whose axes are parallel to the y-axis.
+ y = cosec q
or (x cot q + y)2 = cosec2 q = 1 + cot2 q
Solution
1 dy 1
Replacing cot q by n + P x n −1 = Q , we get the The general equation of the family of parabolas whose axes
y dx y
are parallel to the y-axis is (x - a)2 = k(y - b)
differential equation of the family as where a, b, k are variable parameters (i.e., by assign-
 dy 
2
 dy 
2 ing values to a, b, k we get members of the family of
 y − x dx  = 1 +  dx  parabolas).
We have to eliminate a, b, k in the equation to get the
which is a first order second degree equation. differential equation of the family
Differentiating successively three times with respect
Concept Strand 58 to x,
dy
Eliminate c from the relation y = cx + c - c3 2(x − a) = b
(or form the differential equation whose general solu- dx
tion is y = cx + c - c3 where c is an arbitrary constant). d2 y
2=b
dx 2
Solution d3 y
⇒ =0
Differentiating the given relation with respect to x, dx 3
du which represents the differential equation of the fam-
=c
dx ily of parabolas.

Our next task is, given a differential equation, how to obtain its general solution. We restrict our study to first order and
first degree equations only.

Solutions of first order first degree differential equations

A first order first degree differential equation can always be Type 1: Separable equations
written in the form
Here, the given differential equation can be expressed in
dy the form
= F(x, y)
dx f(x)dx = g(y)dy
or F1(x, y)dy = F2(x, y)dx Direct integration of the above relation with respect to
or P(x, y) dx + Q(x, y) dy = 0 the variable on each side gives the general solution.
The general solution is given by
Where, P and Q denote functions of x and y.
∫ f (x)dx = ∫ g(y)dy + C
where, C is an arbitrary constant.
3.36  Integral Calculus

C o nce p t S t r a n ds
Concept Strand 60 ydy 1 + x2 
⇒ =  dx
dy (1 + y 2 )  x
Find the general solution of = 3x 2 y 3
dx Integrating both sides, the general solution is given by
1 x2
Solution log e (1 + y 2 ) = log e x + +C
2 2
dy where C is an arbitrary constant.
We rewrite the equation as = 3x2dx
y3
dy −1
On integration, ∫y 3
= ∫ 3x 2 dx + C ⇒
2y 2
= x3 + C Concept Strand 62

where, C is an arbitrary constant, represents the general dy


(1 - logy) = tan x sec2 x
solution. dx

Solution
Concept Strand 61
The given equation may be rewritten as
dy (1 - log y)dy = tan x sec2 x dx
Find the general solution of xy = 1 + x2 + y2 + x2y2. Integrating both sides,
dx
∫ (1 − log y)dy = ∫ tan x sec
2
xdx
Solution
tan 2 x
The given equation may be rewritten as ⇒ y - [y logy - y] = +C
2
dy
xy = (1 + x2) (1 + y2) tan2 x
dx ⇒ or y(2 − log y) = +C
2

Type 2: Homogeneous equations  y2 


x 2 1 + 3 2 
In this case, the given differential equation can be expressed  x  y
= = x × a function of
in the form  7y  x
x 4 + 
f(x, y) dy = g(x, y) dx  x 
where f(x, y) and g(x, y) are homogeneous functions in x or
and y of the same degree. A little elaboration of homoge-  x2 
neous functions is not out of place here. y 2 1 + 2 
 y  x
f(x, y) is said to be a homogeneous function in x and y = = y × a function of
 x  y
of degree n (where n is a rational number positive or nega- y 7 + 
 y
y
tive) if f(x, y) can be expressed as xn × a function of   . ⇒ f(x, y) is a homogenous function of x and y of degree 1.
x
As a second example, consider the function
x 7 
7

OR yn × a function of   . 2 
 y 2 
y x 1+ 4 
7 7
 x 
x 2 + 4y 2  
For example, consider the function f (x, y) = =
x 2 + 3y 2 ( 3
2x + y 3
)  y 
x 3 2 +   
3

f (x, y) = x 
(4x + 7y)  
Integral Calculus  3.37

1
y y
= x 2 × a function of f(x, y) in the form xn × a function of or yn × a function
x x
x
of .
⇒ f(x, y) is a homogenous function in x and y of degree y
1  y y
y3 
. Again, f (x, y) = x 4  tan −1 + e x − 3  is a homog-
2  x x 
[Observe that the numerator is a homogenous func- enous function in x and y of degree 4.
7 4
 x x x2 
tion in x and y of degree while the denominator is a ho- f (x, y) = y 3  cos + tan 2 + 2  is a homogenous
2  y y y 
mogenous function in x and y of degree 3. f(x, y) is homog- 4
function in x and y of degree .
7 3
enous in x and y of degree − 3].
2 Now, suppose the given differential equation f(x, y)dy =
g(x, y)dx is such that f(x, y) and g(x, y) are homogenous func-
x 3 + 2y 2
The function f (x, y) = is not a homogenous tions in x and y of the same degree n. In this case, we change
2x − y the dependent variable y to v by the substitution y = vx.
function in x and y. This is because the numerator is not Then, the given equation reduces to a separable equa-
a homogenous function and therefore, we cannot express tion. (Type 1)

C o nce p t S t r a n ds
Concept Strand 63 where, C is an arbitrary constant (we may also write
x
dy2 the general solution as xe y = C where C is an arbi-
Find the general solution of the equation x = y 2 + xy .
dx
trary constant).
Solution
Since, both x2 and (x2 + xy) are homogenous functions in x
Concept Strand 64
and y of degree 2, we substitute 3
dy  y  y
y = vx Find the general solution of the equation = + .
dx  x  x
dy dv
⇒ =v+x
dx dx Solution
On substitution y = vx, the given differential equation The given equation is a homogeneous equation.
 dv  Putting y = vx, in the equation, we get
reduces to x  v + x  = x 2 (v 2 + v)
2

 dx  dv dv
v+x = v3 + v ⇒ x = v3
dv dx dx
⇒ x = v2, which is a separable equation
dx which is a separable equation.
dv dx dv dx
we have = ⇒ 3 =
v2 x v x
−1
−1 Integration gives, = log x + log c
= log e x + log C
Integration gives 2v 2
v
−x 2
−x x 1 ⇒ log Cx = 2
⇒ = log Cx ⇒ = log 2y
y y Cx
− x2
x x
1 y 1 y ⇒ Cx = e 2 y
2

⇒ = e ⇒ xe =
Cx C where C is an arbitrary constant is the general solution.
3.38  Integral Calculus

Concept Strand 65 dv
⇒ x = (2v + 1)2
dy dx
Find the general solution of the equation x2 dx = dv dx
⇒ =
x2 + 5xy + 4y2. (2v + 1)2
x
Integrating both sides,
Solution
−1
Both x2 and (x2 + 5xy + 4y2) are homogeneous in x and y = log x + log C = log Cx
2(2v + 1)
of degree 2.
Setting y = vx, the given equation reduces to −x
⇒ log Cx = or the general solution of the
 dv  2(2y + x)
x 2  v + x  = x2 + 5x2v + 4v2x2 = x2(1 + 5v + 4v2)
 dx  −x
given equation is log Cx = where, C is an
dv 2(2y + x)
⇒ v + x = 1 + 5v + 4v2
dx arbitrary constant.

Type 3: Linear Equations


⇒ d  ye ∫  = Qe ∫ Pdx dx
Pdx

 
A first order differential equation is said to be linear if it is
dy Integrating both sides of the above with respect to x,
linear in y and . That is, the differential equation is of
dx
ye ∫ = ∫ Qe ∫
Pdx Pdx
dy dx + C — (3)
the form + Py = Q  — (1) 
dx where, C is an arbitrary constant
where P and Q are functions of x only. Thus, the general solution of the linear equation is
For example, the equations given by (3).
dy dy Procedure for obtaining general solution of the linear
+ 4xy = x3, - x3y = x cos x are linear differential
dx dx dy
equations of first order. To solve a linear equation of first equation + Py = Q
dx
order represented by (1) we proceed as follows:
Step 1: Find ∫ Pdx
Multiplying both sides of (1) by e ∫
Pdx

we get
Step 2: Find e ∫
Pdx

dy
e∫ + Pye ∫ = Qe ∫
Pdx Pdx Pdx
—(2)
Step 3: Find ∫ Qe ∫
Pdx
dx  dx
d  ∫ Pdx  Then, the general solution of the equation above is
But, the left hand side of the above is ye
dx  
ye ∫ = C + ∫ Qe ∫ dx
Pdx Pdx

 d  ∫ Pdx  ∫ Pdx
since dx  ye 
=e
dy
dx
+ ye ∫ ×
Pdx d
dx
( )
∫ Pdx 

where C is an arbitrary constant.
 
dy  ∫ Pdx  ∫ Pdx dx , constant
 = e∫ 
∫ Pdx ∫ Qe
Pdx
+ ye P [In the evaluation of and
 dx   
of integration need not be written, as it is already appearing
Therefore, (2) may be written as in the general solution].
d  ∫ Pdx 
= Qe ∫ We shall illustrate the solution procedure by working
Pdx
ye
dx   out three examples.
Integral Calculus  3.39

C o nce p t S t r a n ds
Concept Strand 66 Solution
dy 3y dy tan x
Find the general solution of the equation + = x4 . The equation may be rewritten as + (sec 2 x)y =
dx x dx cos 2 x
Solution tan x
P = sec2 x, Q =
cos 2 x
3
∫ Pdx = ∫ sec
2
P= , Q = x 4; xdx = tan x
x
tan x
e∫ = e tan x ; ∫ Qe ∫
Pdx Pdx
dx = ∫
× e tan x dx
∫ Pdx = 3 log x = log x 3
cos2 x
Putting tan x = t, sec2 x dx = dt
e∫
Pdx
= x3;
R.H.S = ∫ te t dt =tet - et
∫ Pdx x8
∫ Qe dx = ∫ x 4 × x 3 dx =
8 General solution is yetanx = C + etanx (tan x - 1), where
c is an arbitrary constant.
x8
General solution of the equation is yx 3 = C + Remarks
8
where, C is an arbitrary constant. (i) Note that when we multiplied the liner equation
dy
+ Py = Q by e ∫ , the equation reduced to
Pdx

dx
Concept Strand 67
d  ye ∫  = Qe ∫ Pdx dx
Pdx

 
Find the general solution of the equation
dy 2xy 4x 2 ⇒ d  ye ∫  = d  Qe ∫ Pdx dx 
Pdx

∫
+ 2 = 2 .
  
dx x + 1 x + 1
We say that the equation becomes exact on
Solution multiplication of the given linear equation by e ∫ .
Pdx

e∫
Pdx
2x 4x 2 is called an integrating factor of the equation.
P= ,Q =
x2 + 1 x2 + 1 dy
(ii) General solution of the equation + Py = Q is
2xdx dx
= log(x 2 + 1); e ∫ = (x 2 + 1)
Pdx
∫ Pdx = ∫ x 2
+1
given by

ye ∫ = C + ∫ Qe ∫
Pdx Pdx
General solution is dx

4x 2 x3 or y = Ce ∫ +  e ∫  ∫ Qe ∫ dx
− Pdx − Pdx Pdx
y(x 2 + 1) = C + ∫ (x 2
+ 1)dx = C + 4 —(1)
(x 2 + 1) 3  

where, C is an arbitrary constant. ∫ − Pdx

y = Ce is the general solution of the equation


dy
+ Py = 0
Concept Strand 68 dx
d  − ∫ Pdx 
+ P  Ce ∫ 
− Pdx
Find the general solution of the equation for, Ce
dx    
dy
cos 2 x + y = tan x . − ∫ Pdx
(−P) + CPe ∫ = 0
− Pdx
dx = Ce
3.40  Integral Calculus

Concept Strand 69
y = e ∫ ∫ Pdx dx is a particular solution of the
Pdx
∫ Qe
dy Find the general solution of the equation
given equation + Py = Q
dx
dy dy
d  − ∫ Pdx ∫ Pdx  − ∫ Pdx ∫ Pdx 1 + 2xy = y3 .
for,
dx 
e ∫ Qe dx  + Pe ∫ Qe dx dx dx

= e ∫ × Qe ∫ + ∫ Qe ∫ × Solution
− Pdx Pdx Pdx

e ∫ (−P) + Pe ∫ ∫ Qe ∫ dx = Q
− Pdx − Pdx Pdx
dx
The equation may be rewritten as + 2xy = y 3
dy
We may therefore say that the general solution of the
linear equation P1 = 2y , Q1 = y3

dy , e∫
P1 dy
∫ P dy = y
2
2
+ Py = 0 is given by 1
= ey
dx
∫ P1dy dy = y 3 e y 2 dy
y = [General solution of the corresponding ∫Q e 1 ∫
dy
homogeneous equation + Py = Q ] + [a particular Putting y2 = t,
dx
solution of the given equation] 1 
R.H.S = ∫  dt  × te t
dx 2 
(iii) The equation + P x = Q1
dy 1 1 1 1

2
te t dt = (t − 1)e t = (y 2 − 1)e y
=
where P1 and Q1 are functions of y only may be called 2 2 2
dx general solution is
linear equation in x and (y is the independent
dy 2 1 2

variable and x is the dependent variable) xe y = C + (y 2 − 1)e y


2
The general solution of the above equation is given by
2 1
⇒ x = Ce − y + (y 2 − 1)
xe ∫ = C + ∫ Q1 e ∫
P1 dy P1 dy
dy 2

where, C1 is an arbitrary constant. where, C is an arbitrary constant.

Equations reducible to linear form 1 dy 1 du


⇒ =
dy y n dx (1 − n) dx
Consider the differential equation + Py = Qy n — (1)
dx  Substituting in (2), the given equation reduces to
where P and Q are functions of x. It is clear that the equa- 1 du
tion is not linear. On dividing both sides by yn, we obtain + Pu = Q
(1 − n) dx
1 dy 1 du
+ P x n −1 = Q — (2) ⇒ + (1 − n)Pu = Q(1 − n)  — (3)
y n
dx y dx

We change the dependent variable y to u by the sub- du
(3) is linear in u and and therefore, we can get the
stitution dx
y1-n = u solution of (3) easily. (1) is called Bernoulli’s equation .
dy du We illustrate the above procedure by working out three
⇒ (1 − n)y − n = examples.
dx dx
Integral Calculus  3.41

C o nce p t S t r a n ds
Concept Strand 70 1 dy du
Set y =u⇒ =
dy 2 y dx dx
Find the general solution of the equation + xy = x 3 y 3 .
dx Substituting,
du 2 du
Solution − ux = xe x , which is linear in u and
dx dx
On dividing by y3, −x 2
P = -x ⇒ ∫ Pdx =
1 dy 1 2
+ x = x3
y 3
dx y 2 − x2 + x2 + x2
∫ Pdx
∫ Qe dx = ∫ xe × e dx = ∫ xe
x2
Change the dependent variable y to u by the substitution
2 2
dx = +e 2

1 −x 2
x 2

=u General solution is ue 2
=C+e 2
y2
x2
−2 dy du 1 dy −1 du ⇒ u = Ce 2 + e x
2

⇒ 3 = ⇒ 3 =
y dx dx y dx 2 dx x2
2
Substituting in the given equation, we get ⇒ y = Ce 2
+ e x , where C is an arbitrary constant.
−1 du
+ xu = x 3
2 dx Concept Strand 72
du du
⇒ - 2xu = - 2x3, which is linear in u and dy
dx dx Find the general solution of the equation 6cos2 x -
dx
y sin 2x + 2y4 sin3 x = 0.
P = -2x ⇒ e ∫ = e − x
Pdx 2

∫ Pdx dx = −2x 3 e − x 2 dx
∫ Qe ∫ Solution
Set x2 = t ⇒ 2x dx = dt
The equation can be rewritten as
R.H.S = ∫ −te − t dt = (1 + t)e-t = (1 + x 2 )e − x
2

1 dy 1 sin 2x −2 sin 3 x
General solution is ue − x2 2
= C + (1 + x )e − x2 − =
y 4 dx y 3 6 cos 2 x 6 cos 2 x
x2 2
⇒ u = Ce + 1 + x 1 dy 1 tan x − sin 3 x
1 ⇒ − × =
y 4 dx y 3 3 3cos 2 x
2
⇒ 2 = Ce x + 1 + x 2
y
1 −3 dy du
where C is an arbitrary constant. Set =u⇒ 4 =
y3 y dx dx
Substituting,
Concept Strand 71
−1 du tan x − sin 3 x
−u =
Find the general solution of the equation 3 dx 3 3cos 2 x

dy 2
− 2xy = 2xe x y . du sin 3 x
dx ⇒ + u tan x =
dx cos 2 x
Solution sin 3 x
P = tan x, Q =
cos 2 x
Dividing both sides by 2 y ,
1 dy 2 ∫ Pdx = log sec x
− x y = xe x
2 y dx e∫
Pdx
= sec x
3.42  Integral Calculus

sin3 x General solution is


∫ Pdx
∫ Qe dx = ∫
cos2 x
× sec xdx
tan 2 x
u sec x = C + − log sec x
= ∫ tan3 xdx 2

= ∫ tan xsec2 xdx − ∫ tan xdx sec x tan 2 x


⇒  = C + − log sec x , where C is an arbi-
y3 2
tan 2 x
= − log sec x trary constant.
2

Type 4: Solutions of equations of the form


dy
(a1x + b1y + c1) = (a2x + b2y + c2)
dx
(where a1, b1, c1, a2, b2, c2 are constants)

C o nce p t S t r a n ds
Concept Strand 73

(1 + v ) dv + dX = 0
Find the general solution of the equation (x + y + 5)
dy (v 2
)
+1 X
dx 1
= (y - x + 1). Integrating, tan-1 v +
log(v2 + 1) + log X = C
2
y +3 1  (y + 3)2 
Solution ⇒ tan −1   + log  + 1 + log(x + 2) = C
x +2 2  (x + 2)
2

We find the point of intersection of the lines x + y + 5 = 0
y +3 1
and y - x + 1 = 0 ⇒ tan −1  + log{(y + 3)2 + (x + 2)2 } = C
We get x = -2, y = -3  x + 2  2
Let x = X - 2, y = Y - 3 where, C is an arbitrary constant.
In other words, we are changing both independent
and dependent variables x and y to X and Y by the substi-
Concept Strand 74
tutions x = X - 2, y = Y - 3
dy dY dy x + 2y + 1
We have , = Find the general solution of the equation = .
dx dX dx 2x + 4y + 3
Substituting in the given equation, it reduces to
(X + Y)
dy
= (Y - X)
Solution
dx In the above case, the system of equations x + 2y + 1 = 0
The above equation is a homogeneous equation in X
and Y. 2x + 4y + 3 = 0 is inconsistent
We set Y = vX We therefore put u = x + 2y + 1
Substituting, du dy dy 1  du 
⇒ =1+2 ⇒ = − 1
 dv  dx dx dx 2  dx 
(1 + v)  v + X = (v − 1)
 dX  Substituting

1  du  u
dv − 1 =
v + v2 + (1 + v) × = ( v − 1) 
2  dx  (2u + 1)
dx
dv du 2u
⇒ (v2 + 1) + (1 + v) =0 −1 =
dX dx (2u + 1)
Integral Calculus  3.43

du 2u 4u + 1 Solution
⇒ = +1 =
dx (2u + 1) 2u + 1 du dy
Set u = x + y ⇒ =1+
(2u + 1)du dx dx
= dx
(4u + 1) Substituting,

Integrating,  du 
⇒ u 2  − 1 = 5
1 1  dx 
 2 ( 4u + 1) + 2  du du
 
∫ (4u + 1)
= ∫ dx + C ⇒ u 2
dx
= 5 + u2

1 1 u 2 du
⇒ u + log(4u + 1) = x + C ⇒ = dx
2 8 (5 + u 2 )
1  5 
⇒ u + log(4u + 1)=2x + 2C ⇒ 1 −  du = dx
4  5 + u2
⇒ log[4(x + 2y + 1) + 1] = 8x - 4u + 2C = 4x - 8y + C 5  u 
or general solution is Integrating, u − tan −1  = x +C
log(4x + 8y + 5) = (4x - 8y + C’), where C’ is an arbi- 5  5 
trary constant . x+y
⇒ (x + y) − 5 tan −1  = x +C
 5 
Concept Strand 75 x+y
⇒ y = 5 tan −1  +C
dy  5 
Obtain the general solution of the equation (x + y)2 = 5.
dx where, C is an arbitrary constant gives the general solution.

Exact equations 1
(xdy − ydx) = dx + dy
Consider the differential equation ydx + xdy = 0. It can be (x 2
+ y2 )
easily seen that the equation can be rewritten as d(xy) = 0, y y
and we obtain the general solution as xy = C where C is an x2d   d 
x x
arbitrary constant. The above differential equation is called ⇒ = d(x + y) ⇒ = d(x + y)
an exact equation. x2 + y2 y2
1+
In general, the differential equation x2
y
P(x, y)dx + Q(x, y)dy = 0 ⇒ d tan −1   = d(x + y)
x
is said to be an exact equation if we can rewrite the equation
as d{F(x, y)} = 0, which yields the general solution F(x, y) = C which is an exact equation. We immediately obtain the gen-
A few examples of exact equations are listed below. y
eral solution as tan −1 = x + y + C.
x
 x2 y2  In the above example, the given equation, on dividing
(i) x dx + ydy = 0 ⇒ d  +  = 0
2 2 both sides by (x2 + y2) reduced the given equation to an
exact equation. Therefore, in some cases, we may be able
1 x to reduce a given differential equation to an exact equation
(ii) ( ydx − xdy ) = 0 ⇒ d  y  = 0 by multiplying or dividing the equation by a suitable func-
y2  
tion of x and y (or by a function of x or by a function of y).
1 y
2 (
(iii) xdy − ydx ) = 0 ⇒ d   = 0 1
The function 2 2 above is an integrating factor of the
x x (x + y )
Again, consider the equation equation xdy - ydx = (x2 + y2)(dx + dy)
xdy - ydx = (x2 + y2)(dx + dy) We illustrate the above procedure by working out a few
On dividing both sides by (x2 + y2) we get examples.
3.44  Integral Calculus

C o nce p t S t r a n ds
Concept Strand 76 Solution
⇒ (sin y - x)dy = ydx
Obtain the general solution of the equation
⇒ sin y dy - [xdy + ydx] = 0
dy y d{cos y + xy} = 0
= .
dx sin y − x The general solution is
cos y + xy = C, where C is an arbitrary constant

Alternative method ∫ P1dy = y


dy 1 sin y
∫ P dy = log y ⇒ e
1

The equation can be rewritten as + x=


dx y y ∫ P1dy dy = sin y × ydy = -cos y
dx
∫Q e 1 ∫ y
which is linear in x and
dy general solution is
1 sin y
P1 = ,Q = xy = - cos y + C
y y
⇒ xy + cos y = C

C o nce p t S t r a n ds
Concept Strand 77 Concept Strand 78
2y Obtain the general solution of the equation
Find the general solution of the equation dx +
x (sin xcos y + e2x)dx + (cos x sin y + tan y)dy = 0.
(2logx - y)dy = 0.

Solution Solution
2y ⇒ (sin x cos y dx + cos x sin y dy) + e2xdx + tan y
⇒ dx + (2logx)dy - ydy = 0
x dy = 0
 dx 
⇒ 2  y + (log x)dy  − ydy = 0  e2 x 
 x  ⇒ d(sin x sin y) + d   + d(log sec y) = 0
 2
 y2 
⇒ 2d(ylogx) - d 
2  = 0 The general solution is given by

y2 e2 x
which gives the general solution as 2y logx - =C sin x sin y + + log sec y = C
2 2

Initial value problems dy


(x + 1) = 2y , y(0) = 1
A first order equation together with the condition that y = dx
y0 when x = x0 (written as y(x0) = y0) is known as an initial is an initial value problem.
value problem. For example, To solve such problems, we first obtain the general
solution of the differential equation and find that particu-
Integral Calculus  3.45

lar value of the arbitrary constant in the general solution Or, the general solution is
which satisfies the condition y(x0) = y0. This means that the
solution of an initial value problem is a particular solution Cy = (x + 1)2
of the given differential equation.
In the above example, the equation may rewritten as The condition y =1 when x = 0 (called initial condi-
dy 2dx tion) gives C = 1
=
y (x + 1) The solution of the given initial value problem is there-
On integration, log y + log C = 2log (x + 1) fore given by y = (x + 1)2

C o nce p t S t r a n ds
Concept Strand 79 ⇒ solution of the initial value problem is
y2 = 4sec 2x
dy 2
Solve the initial value problem: y = xe y , y(1) = 0.
dx
Concept Strand 81
Solution
dy 3y 1 1
Solve the initial value problem: + = 2 , y(1) = .
The equation is rewritten as dx x x 2
− y2
ye dy = xdx
Solution
Integration gives
2 We note that the given equation is linear
−e − y x2
= + arbitrary constant 3 1
2 2 P= ,Q= 2
x x
or, the general solution of the equation is
x2
e∫ = x 3 ; ∫ Qe ∫
2
Pdx Pdx
x2 + e − y = C dx = ∫ xdx =
2
using the condition y(1) = 0 [i.e., y = 0 when x = 1]
we get 1 + 1 = C ⇒ C = 2 General solution is
x2
2
The solution of the initial value problem is x 2 + e − y = 2 yx3 = C +
2
1
Using the condition y(1) =
Concept Strand 80 2
dy 1 1
Solve the initial value problem: = ytan 2x , y(0) = 2. =C+ ⇒C=0
dx 2 2
Therefore, the solution of the initial value problem is
Solution
x2
The equation can be rewritten as yx 3 =
2
dy ⇒ 2xy = 1 [as x cannot be zero]
= tan 2x dx
y

log sec 2x
Integration given log y + log C = or the general Concept Strand 82
2
solution is
Cy2 = sec 2x, where C is an arbitrary constant dy
Solve the initial value problem: + y cot x = 4x cosec x,
Using the condition y(0) = 2 dx
1  π  π2
4C = 1, C = y  = .
4 2  2
3.46  Integral Calculus

Solution For obtaining the general solution of the equation we


proceed as follows. We note that the equation is homog-
The equation is linear enous.
P = cot x, Q = 4xcosec x dy dv
Putting y = vx, =v+x
dx dx
e∫ = sin x, ∫ Qe ∫
Pdx Pdx
= ∫ 4xdx = 2x2
dv 2
General solution is ⇒ v + x = v − cos v
dx
ysin x = C + 2x2 dv
 π  π2 ⇒ x = − cos 2 v
Using the condition y   = dx
2  2 dx
⇒ -sec2 v dv =
π2 2 × π2 x
×1 = C + ⇒C=0 Integration gives log Cx = -tan v
2 4 y
or, the general solution is log Cx + tan   = 0
Solution of the initial value problem is ysin x = 2x2 x
 π
Since the curve passes through 1, 
Concept Strand 83  4
1
dy ⇒ log C + 1 = 0 ⇒ C =
y satisfies the differential equation = e −2 y and y = 0 e
dx We obtain the particular solution as
when x = 5. Find the value of x when y = 3.
x y
log   + tan   = 0
e x
Solution which is the equation of the curve.
e2 y
The general solution of the equation is =x+C
2
−9 Concept Strand 85
Since y = 0 when x = 5, C =
2 Newton’s law of cooling states that the difference ‘x’ be-
The particular solution satisfying the condition y(5) = 0 is tween the temperature of a body and that of the surround-
therefore, ing air decreases at a rate proportional to this difference. If
e2 y 9 x = 80°, when t = 0; x = 40° when t = 30, find the value of
=x− t when x = 10°.
2 2

When y = 3, x =
9 e6
+ =
e6 + 9 ( )
2 2 2 Solution
Mathematical formulation of the law gives
Concept Strand 84 dx
= − kx , k > 0
 π dt
A curve passes through the point 1,  and its slope at dx
 4 = − kdt
x
dy y y
any point (x, y) on it is given by = − cos 2 . Find its The general solution gives
dx x x log x + log C = -kt
equation. ⇒ Cx = e-kt, where C is an arbitrary constant.
1
When x = 80°, t = 0 ⇒ 80C = 1 ⇒ C =
Solution 80
The general solution of the above equation represents the When x = 40°, t = 30 ⇒ 40C = e-30k
family of curves satisfying the given differential equation. 40 1
⇒ = e −30 k ⇒ e-30k =
Our problem is to find that member of the family which 80 2
 π 1
passes through the point 1,  . ⇒ k = log 2
 4 30
Integral Calculus  3.47

We want to find the value of t when x = 10° 1


Substituting in the general solution, − kt = log   = -3log 2
8
10
= e − kt
80 3
t= log 2 = 3 × 30, (on substituting for k) = 90
1 k
e − kt =
8

Orthogonal trajectories If (x, y) is a point of intersection of a member of (1)


and a member of (2), then, the product of the slopes of the
Let F(x, y, C) = 0  — (1) tangents at (x, y) to these curves equals “-1”.
represent a one parameter family of curves where C is This means that the differential equation of the orthog-
the parameter. Consider another one parameter family of dy −1
curves onal family can be obtained by replacing by in
dx  dy 
G(x, y, D) = 0 — (2)  dx 
where D is the parameter. (3). In other words, the differential equation of the orthog-
Suppose every member of (2) intersects every member onal family is
of (1) orthogonally (i.e. the two families of curves intersect
at right angles). We say that (1) and (2) are orthogonal tra-  
jectories.  −1 
f  x, y, =0  — (4)
If (1) is given, our problem is to find (2). dy 
 
Let the differential equation of the family (1) be  dx 
 dy  Solving (4), (2) is obtained.
f  x, y,  = 0 — (3)
 dx  We illustrate the above procedure by working out two

(3) is obtained by eliminating C in (1)). examples.

C o nce p t S t r a n ds
Concept Strand 86 ⇒ The differential equation of the given family is
Find the orthogonal trajectories of the family of parabolas dy
2x =y — (3)
y2 = kx. dx
y2 = kx — (1) The differential equation of the orthogonal family is
dy −1
obtained by replacing by in (3). This gives
Solution dx dy
dx
Differentiating (1) with respect to x,
dy
dy 2x + y =0 — (4)
2y =k —(2) dx
dx
y2
y Solving (4), the orthogonal trajectories x 2 + =D
Dividing (1) by (2), =x 2
dy
2 or 2x2 + y2 = l where l is a parameter.
dx
3.48  Integral Calculus

Concept Strand 87  dy x   dy y 
Find the orthogonal trajectories of the family of curves
 dx − y   dx + x  = 0
y2 = kx3.
dy x dy − y
⇒ = + and =
dx y dx x
Solution
dy x
y2 = kx3  — (1) The general solution of =+ is x2 - y2 = C
dx y
Differentiating (1) with respect to x,
where, C is an arbitrary constant.
dy dy − y
2y = 3x 2 k — (2) The general solution of = is xy = C where,
dx  dx x
Dividing, (1) by (2), we get C is an arbitrary constant
y x Therefore, the solution of the given equation is
= (x2 - y2 - C) (xy - C) = 0
dy 3
2 Observe that we use the same C for the arbitrary
dx
constant. This is because, the differential equation is of first
dy
⇒ 3y = 2x  — (3) order and therefore, it represents a one parameter family
dx of curves. The one parameter family is either x2 - y2 = C
(3) represents the differential equation of the given or xy = C.
dy −1
family (1). Replacing by in (3), the differen-
dx dy
Concept Strand 89
dx
tial equation of the orthogonal family is given by Solve the equation
2
dy  dy  dy
3y + 2x = 0  — (4)  dx  + 2pycot x = y , where p = dx .
2
dx
Solving (4)
3y 2 Solution
+ x2 = D — (5)
2  dy
Solving for ,
where, D is an arbitrary constant represents the or- dx
thogonal family. 2
dy −2y cot x ± 4y cot x + 4y
2 2

=
We wind up this unit by solving a few equations, dx 2
which are of first order but not first degree and also = y(cosec x - cot x) or -y(cosec x + cot x)
a few equations which are of second order and first dy
degree. Considering the equation = y(cosec x - cot x), the
dx
general solution is easily obtained as
Concept Strand 88 x
log y = 2 log sec + C or y(1 + cos x) = C
2
dy 1 x y dy
Solve the equation − = − . Again, considering the equation, = -y(cosec x +
dx dy y x dx
dx cot x), the general solution is obtained as y( 1 - cos x) = C
The solution of the given equation may be represented as
Solution [y(1 + cos x) - C] [y(1 - cos x) - C] = 0, where C is an
The equation can be rewritten as arbitrary constant.
2
 dy   x y  dy
 dx  −  y − x  dx − 1 = 0 Concept Strand 90

The above equation is of first order but second degree. d3 y


Solve the equation: = e4x
By factorizing, we get dx 3
Integral Calculus  3.49

Solution C−x
y + C1 = ∫ dx
2
d y e 4x 1 + Cx
On integrating both sides with respect to x, = + C1
dx 2 4  −1  1 1 
= ∫  + C +  dx
 C  C  1 + Cx 
dy e 4 x
Again, integrating, = + C1 x + C 2 −x  1  log(1 + Cx)
dx 16 = + C + 
On integrating the above, C  C C

e4x x2 −x  1 
y= + C1 + C2 x + C3 ⇒ y + C 1 = + 1 + 2  log(1 + Cx)
64 2 C  C
where, C1, C2, C3 are arbitrary constants represents the where, C and C1 are arbitrary constants in the general solu-
general solution of the given equation. tion of the given second order equation.

Concept Strand 91 Concept Strand 92


2 2
d 2 y  dy  d 2 y  dy 
Solve the equation (1 + x 2 ) + + 1 = 0. Solve the equation y + + 1 = 0.
dx 2  dx  dx 2  dx 

Solution Solution
dy dy
Let =p Let =p
dx dx
d 2 y dp dp dy dp
d 2 y dp ⇒ = = × =p
Then, = dx 2
dx dy dx dy
dx 2 dx
Substituting in the given equation, substituting, in the given equation,
dp
dp yp + p2 + 1 = 0  — (2)
(1 + x 2 ) + (p 2 + 1) = 0 — (1) dy
dx 
(2) is a first order equation where y is the independent
which is a first order equation in which x is the indepen-
variable and p is the dependent variable.
dent variable and p is the dependent variable.
pdp dy
dp dx (2) ⇒ + =0
Rewriting (1), + =0 p2 + 1 y
(p + 1) (1 + x 2 )
2

Integration gives
Integration gives
tan-1p + tan-1 x = C1 1
log(p 2 + 1) + log y = log C 1
 p+x  (p + x) 2
⇒ tan −1  = C1 ⇒ = tan C 1 = C
 1 − px  1 − px  2 
 dy 
log  y 1 +    = log C 1
dy   dx  
⇒ p(1 - Cx) = C - x ⇒ (1 + Cx) = (C − x)  — (2)  
dx
2
(2) is a first order equation  dy 
⇒ y 1 +   = C 1
dy C − x  dx 
we have =
dx 1 + Cx
  dy  2 
C−x Squaring , y 2 1 +    = C , where we have used C
dy = dx  dx  
1 + Cx  
Integrating, to represent C12 since they are arbitrary constants only
3.50  Integral Calculus

 dy 
2
− ydy
y2   = C − y2 ⇒ = dx
 dx  c − y2

dy C − y2 C − y2 Integration gives
=± = ±
dx y2 y c − y 2 = x + C1

Taking + sign,
⇒ (x + C1)2 + y2 - C = 0
ydy
⇒ = dx In both cases, we get the general solution of the given
C − y2 equation as the two-parameter family of curves
Integration gives − C − y 2 = x + C1 (x + C1)2 + y2 - C = 0
It may be noted that the general solution in this case is
⇒ (x + C1)2 + y2 - C = 0
Taking “-” sign, the family of circles whose centres are on the x-axis.

Summary
Indefinite Integrals 1
If F(x) is such that F (x) = f(x), then F(x) is called an
1
(xi) ∫ 1− x 2
dx = sin −1 x + C

antiderivative of f(x) and we write ∫ f (x)dx = F(x) + C , 1


(xii) ∫1+ x 2
dx = tan −1 x + C
where, C is an arbitrary constant. ∫ f (x)dx is called an
indefinite integral of f(x). II (i) ∫ tan xdx = log sec x + C
Results
(ii) ∫ cot xdx = log sin x + C
n +1
x (iii) ∫ sec xdx = log(sec x + tan x) + C or
I (i) ∫ x n dx = + C , n rational ≠ -1.
n +1
π x
and when n = 0, ∫ dx = x + C log tan  +  + C
4 2
1
(ii) ∫ x dx = log x + C (iv) ∫ cosec xdx = log(cosec x − cot x) + C or
x
∫ e dx = e
x x
(iii) +C log tan +C
2
ax
∫ a dx =
x
(iv) +C (a > 0) dx 1 x −a
log a III (i) ∫x 2
−a 2
= log 
2a  x + a 
+C

(v) ∫ sin x dx = − cos x + C dx 1 a + x 


(ii) ∫a 2
= log 
− x 2 2a  a − x 
+C
(vi) ∫ cos x dx = sin x + C
dx 1 x
(vii) ∫ sec x dx = tan x + C
2 (iii) ∫a 2
+x 2
= tan −1   + C
a a 
(viii) ∫ cosec x cot xdx = −cosec x + C dx x
IV (i) ∫ a −x2
= sin −1   + C
a 
2
(ix) ∫ sec x tan xdx = sec x + C
(x) ∫ cosec xdx = − cot xdx
2 (ii) ∫ a +x
dx
2 2
( )
= log x + a 2 + x 2 + C
Integral Calculus  3.51

(iii) ∫
dx
x2 − a2
(
= log x + x 2 − a 2 + C ) (vi) ∫
dx
,∫
dx
a + b sin x a + b cos x
and

dx
V (i) ∫ a 2 − x 2 dx =
x a2 − x2 a2 x
+ sin −1   + C ∫ a sin x + b cos x + c can be reduced by using
2 2 a 
2t 1 − t2 2dt
x x2 − a2 sinx = 2
, cos x = 2
and dx =
1+ t 1+ t 1 + t2
(ii) ∫ x 2 − a 2 dx =
2

x
( )
a2 where t = tan
log x + x 2 − a 2 + C 2
2 a sin x + b cos x
(vii) ∫ dx is reduced by expressing
x x2 + a2 c sin x + d cos x
(iii) ∫ x 2 + a 2 dx =
a
+
d
(D r )
( )
a2 Nr = A. Dr + B .
log x + x 2 + a 2 + C dx
2 and the integral = Ax + B log Dr (where, Nr
VI Integration of special types of functions means numerator and Dr means denominator)
dx 1 (viii) For m, n positive integers, ∫ sin m x .cos n x dx .
(i) ∫ can be reduced to ∫ 2 dx
2
ax + bx + c x − a2 If m or n be odd, take cosx = t or sinx = t and if
1 both are even, use sin2x = 2 sinx cosx, 2sin2x =
or ∫ 2 dx by completing square of 1 − cos2x and 2 cos2 x = 1 + cos2x
x + a2
dx dx
(ix) ∫ ,∫

 b b2 c b2 
ax2 + bx2 + c as a  x 2 + x + 2 + − 2  (
(ax + b ) px + q ax + bx + c px + q
2
)
 a 4a a 4a 
can be simplified by setting px + q = t2
  b  4ac − b2 
2

= a  x +  +  dx
(x) ∫ is reduced by using x −
  2a  4a 2 
( x − λ ) ax 2 + bx + c
px + q 1
(ii) ∫ dx can be reduced by taking Nr
2 l=
ax + bx + c t
(where Nr means numerator). dx
dt (xi) ∫ ,∫ ( x − α )(β − x ) dx and
px + q = l (2ax + b) + M and using ∫ = log t
t
( x − α )(β − x )
and (1) x −α
(iii) ∫
dx
can be reduced to ∫
1 dt ∫ β−x
dx can be evaluated by using
2
ax + bx + c t2 ± a2 x = a cos2 q + b sin2 q
dt e ax
or ∫ forms by completing the squares. (xii) ∫ e ax .sin bx dx = 2
a + b2
(a sin bx − b cos bx )
a − t2
2

px + q +C
(iv) ∫ dx can be reduced by taking and
ax 2 + bx + c
e ax
dt (xiii) ∫ e ax .cos bx dx = 2 (a cos bx + bsin bx)
Nr = a. (2ax + b) + B and by using ∫ =2 t a + b2
t +C
and (3)
Integration by partial fractions
(v) ∫ ax 2 + bx + c dx can be reduced to ax + b
Example: ∫ dx is done by using
( x − α )( x − β)
∫ x 2 ± a 2 dx or ∫ a 2 − x 2 dx form by
ax + b p q
completing the squares. = + where
( x − α )( x − β) x −α x −β
3.52  Integral Calculus

aα + b aβ + b a a
P=
α−β
and q =
β−α
(ix) If f(x) is an even function, ∫ f (x)dx = 2 ∫ f (x)dx
−a 0
a
ax + b A B C
= + + and if f(x) is an odd function, ∫ f (x)dx = 0
( x − α ) ( x − β) x − α (x − α) x − β
2 2
−a
 a a

where, C =
aβ + b (x) ∫ f (x)dx = ∫ f (a − x)dx
(β − α )
2 0 0
2a a

aα + b
(xi) ∫ f (x)dx = 2 ∫ f (x)dx, if f(2a - x) = f(x) and = 0, if
B = and A can be obtained by equating 0 0
α−β f(2a-x) = -f(x)
like coefficient. b b

Definite Integrals
(xii) ∫ f (x)dx = ∫ f (a + b − x)dx
a a
Let f(x) be a continuous function and a and b b b
b

are finite (a < b). Then, ∫ f (x)dx = F(b) − F(a)


(xiii) ∫ f (x)dx ≤ ∫ f (x) dx
a a
a
(xiv) Area bounded by the curve y = f(x), x-axis and the
where, F(x) is an antiderivative of f(x). b

ordinates at x = a and x = b is given by ∫ f (x) dx


a
Properties of definite Integrals (xv) If m and n are positive integers,
b b b 2π
0 , m≠n
(i) ∫ f (x)dx = ∫ f (t)dt = ∫ f (y)dy
a a a
(a) ∫ sin mx sin nxdx = π ,
0
m=n
b a 2π
 0, m ≠ n
(ii) ∫ f (x)dx = − ∫ f (x)dx
a b
(b) ∫ cosmx cosnxdx = π ,
0
m=n
b 2π

(iii) ∫ kdx = k(b - a), where k is any constant.


a
(c) ∫ sin mx cos nxdx = 0
0

b b b π /2 π /2
(d)
∫ sin m x dx = ∫ cos
m
x dx =
(iv) ∫ k 1f1 (x) ± k 2 f2 (x) dx = k 1 ∫ f1 (x)dx ± k 2 ∫ f2 (x)dx
a a a 0 0

(where k1 and k2 are constants and f1(x) and f2(x) are (m − 1)(m − 3)(m − 5)...3.1 π
× , if m is even
continuous in [a, b]) m(m − 2)(m − 4)...4.2 2
(v) If c lies between a and b, (m − 1)(m − 3)(m − 5)...4.2
= , if m is odd
b c b m(m − 2)(m − 4)...3.1
∫ f (x)dx = ∫ f (x)dx + ∫ f (x)dx π /2
(e)
∫ sin x cos x dx =
a a c m n

b 0
(vi) If f(x) ≥ 0 in a ≤ x ≤ b, ∫ f (x)dx ≥ 0 [(m − 1)(m − 3)(m − 5)...3.1]
[(n − 1)(n − 3)...3.1]
a
π
b b
× ,
(vii) If f(x) ≥ g(x) in a ≤ x ≤ b, ∫ f (x)dx ≥ ∫ g(x)dx (m + n)(m + n − 2)(m + n − 4)...4.2 2
a a
if both m and n are even
(viii) If m and M are the minimum and maximum values
of f(x) in [a, b], (m − 1)(m − 3)(m − 5)...2 or 1
b
(n − 1)(n − 3)...2 or 1
m(b - a) ≤ ∫ f (x)dx ≤ M(b - a) otherwise
a (m + n)(m + n − 2)(m + n − 4)...2 or 1

Integral Calculus  3.53

Definite integral as the limit of a sum (ii) Homogeneous Equation

a = x0 < x1 < x2 < ----- < xm = b and x1 = a + h, x2 = a + 2h, The differential equation is said to be of homogeneous
functions in x and y of the same degree. The equation can
b−a
---- xn = a + nh = b ⇒ h = be reduced to the same degree. The equation can be re-
n duced to the separable form by changing the dependent
b
( b − a ) n−1 f (x)or = lim ( b − a ) n f (x) variable y to v using the substitution y = vx.
∫ f ( x ) dx = lim
a
n →∞ n i =0
∑ n →∞

n i =1
(iii) Linear equation
Differential equations A first order linear equation is of the form.
• A relation, which contains the dependent variable y and dy
+ Py = Q
independent variable x, derivatives of different orders dx
of y with respect to x is called an ordinary differential
equation. The relation may contain constants also. Where P and Q are functions of x only. The general
• The order of a differential equation is defined as the order solution of the equation is given by ye ∫
pdx
= ∫ Qe ∫
pdx
+C
of the highest derivative present in the equation.
• The degree of a differential equation is defined as the dy
greatest power of the highest order derivative, when The equation + Py = Q yn
the equation has been made so that the derivatives are dx
present in the equation are in integral powers. Where n is a rational number can be reduced to linear
• A function y = f(x) or F(x, y) = 0 is called a solution form by the substitution
of a given differential equation if it is defined and 1
differentiable (as many times as the order of the given Y=
differential equation) throughout the interval where the y n −1
equation is valid, and is such that the equation becomes
an identity when y and its derivatives obtained from f(x) (iv) The equation
or F(x,y) = 0 are substituted in the equation.
(a1 x +b1 y + c1)dx + (a2 x + b2 y + c2) dy = 0 where a1 b2 - a2
b1 ≠ 0 can be reduced to
First order first degree equations: the homogeneous form by the substitution.
• The general form of an equation which is of first order x = X + h, y = Y + k
and first degree is
M(x, y) dx + N(x, y) dy = 0. Where a1 h + b1 k + c1 = a2 h + b2 k + c2 = 0
• The general solution of a first order differential equation
is a one parameter family of curves where the parameter
(v) If the equation
is the arbitrary constant. By assigning particular values
to the arbitrary constant, particular solutions of the
equation are obtained. M(x, y) dx + N (x, y) dy = 0

can be rewritten in the form


Solutions of first order first degree differential
d{F(x, y)} = 0
equations
by a proper rearrangement of the terms, the equation is
(i) Separabl equations said to be exact.
The differential equation can be reduced to the form
F(x) dx + g(y) dy = 0 Initial value problems
The general solution is
A first order first degree equation with the condition that
∫ f (x)dx + ∫ g(y)dy = C y = y0 when x = x0 is known as an initial value problem.
3.54  Integral Calculus

Orthogonal trajectories  
If F(x, y, C) = 0 and G (x, y, D) = 0 where C and D are  −1 
is f  x, y,  = 0, on solving the differential equation
arbitrary constant are two one parameter family of curves dy 

such that every member of G (x, y, D) = 0 intersects every  dx 
member of F(x, y, C) = 0 orthogonally, we say that F(x, y,
 
C) = 0 and G (x, y, D) = 0 are orthogonal trajectories.
 −1 
dy  f  x, y,  =0
If f (x, y, = 0 is the differential equation repre- dy 
dx  
 dx 
senting the one parameter family of curves F (x, y, C) =
0, the differential equation of the orthogonal trajectory we get G (x, y, D) = 0.
Integral Calculus  3.55

C o nce p t C o n n ec to rs

Evaluate the following:


n −1
1 π
Connector 1: Prove that lim ∑ = .
n →∞
r =0 n −r2 2 2

n −1
1 1 n −1 1
Solution: lim ∑ = lim ∑
n →∞ 2 2 n →∞ n
r =0 n −r r =0 r2
1−
n2
1
1 n −1 1 1 π
( )
1
= lim
n →∞ n
∑ 2
=∫
2
dx = sin −1 x
0
=
2
r =0
r  0 1− x
1−  
n

n −1
n2 1
Connector 2: Prove that lim ∑ 3
= .
(n ) 2
n →∞
r =0 2 2 2
+r

n −1
n2
Solution: lim ∑ 3

(n )
n →∞
r =0 2
+ r2 2

π
1 4
1 n −1 1 1 1
= lim ∑ =∫ dx = ∫ sec 2 θdθ
n →∞ n 3 3 3
r =0
  r 2  2 0
(1 + x ) 2 2 0
(1 + tan 2 θ) 2

1 +   
  n  
by putting x = tanq
π π
4
sec 2 θdθ 4 π
1
=∫ 3
= ∫ cos θd θ = 
 sin θ  04 =

0 sec θ 0 2

2
1 dx 7
17 ∫1 1 + x 4 24
Connector 3: Show that ≤ ≤ .

1 1
Solution: Minimum value of in (1, 2) =
1 + x4 17
2
dx 1 1
Therefore, ∫1+ x
1
4

17
(2 − 1), i.e, ≥
17
1 1
Also, 4
≤ 4 in (1, 2)
1+ x x
2 2
1 1 7
Therefore, ∫1+ x
1
4
dx ≤ ∫
1 x 4
dx =
24
Result follows.
3.56  Integral Calculus

Connector 4: Solve:
dx
(i) ∫ sin 2
x + 2 cos 2 x

sec 2 xdx
Solution: ∫ = ∫ 2 + tan 2
x
, on dividing numerator and denominator by cos2 x

dt
=∫ where t = tan x
2 + t2
1  t  1  tan x 
= tan −1   + C = tan −1  +C
2  2 2  2 

tan x
(ii) ∫ sin x cos x dx
1
2
tan xdx
Solution: = ∫ cos x , on dividing numerator and denominator by cos2 x
tan x
t
=∫ dt where t = tanx = 2 t = 2 tan x + c
t
 1 − tan x 
(iii) ∫   dx
 1 + tan x 

cos x − sin x
Solution: =∫ dx
cos x + sin x
Set t = cos x + sin x
dt = -sin x + cos x
dt
∫ =∫ t
= log t + C = log(cos x + sin x) + C

(iv) ∫ tan 4 x dx

Solution: = ∫ tan 2 x(sec 2 x − 1)dx = ∫ tan 2 x sec 2 xdx − ∫ tan 2 xdx

tan 3 x
= ∫ tan 2 xd(tan x) − ∫ (sec 2 x − 1)dx = − tan x + x + C
3
dx
(v) ∫ sin 2
x + sin 2x

sec 2 xdx dt
=∫
tan 2 x + 2 tan x ∫ t 2 + 2t
Solution: = where, t = tan x

dt 1 1 
=∫ = ∫ −  dt
t(t + 2)  2t 2(t + 2) 
1 1  tan x 
=  log t − log(t + 2) = log  +C
2 2  (2 + tan x) 
Integral Calculus  3.57

cos 2 x
(vi) ∫ sin 6 x dx
 cos2 x   1 
Solution: = ∫  2   2  cosec2 xdx
 sin x   sin x 
Put t = cot x
dt = -cosec2 x dx
t3 t5
∫ = − ∫ t (1 + t )dt = −
2 2
− +C
3 5
cot 3 x cot 5 x
=− − +C
3 5
dx
(vii) ∫ (sin x)(3 + 2 cos x)
sin xdx sin xdx
Solution: =∫ 2
=∫ 2
(sin x)(3 + 2 cos x) (1 − cos x)(3 + 2 cos x)
Put t = cos x
⇒ dt = -sin x dx
−dt dt
∫ = ∫ (1 − t 2
)(3 + 2t)
= −∫
(1 + t)(1 − t)(3 + 2t)
1 dt 1 dt 4 dt 1 1 2
=− ∫ − ∫ + ∫
2 1 + t 10 1 − t 5 3 + 2t
= − log(1 + t) + log(1 − t) + log(3 + 2t) + C
2 10 5
1 1 2
= − log(1 + cos x) + log(1 − cos x) + log(3 + 2 cos x) + C
2 10 5
2 cos xdx
(viii) ∫ 3 + 4 sin 2x
[(cos x + sin x) + (cos x − sin x)]dx (cos x + sin x)dx
Solution: =∫ =∫
(3 + 4 sin 2x) −4(cos x − sin x)2 + 7
(cos x − sin x)dx
+∫
4(cos x + sin x)2 − 1
= I1 + I2
I1:

Put t = cosx - sinx
dt = -(sinx + cosx)dx
 7 
−dt 1 dt 1 1 t − 2 
Substituting , I1 = ∫
7 − 4t 2 4 ∫ 2 7 4
= = × log  
t − 7  7 
4
2  t + 
4 2
I2:
Put t = cosx + sinx
dt = (cosx - sinx)dx
3.58  Integral Calculus

 1
 −4
t
dt 1 dt 1 1
Substituting, I 2 = ∫ 2
4t − 1 4 ∫ 2 1 4
= = × log 
1 1
t − 2  t + 
4 4  4
Therefore,
 7  1
 cos x − sin x −  cos x + sin x − 
2 cos x 1 2  + 1 log  4 +C
∫ 3 + 4 sin 2x dx = 4 7 log    
 cos x − sin x + 7 4  cos x + sin x + 1 
 2   4

Connector 5: Solve:
x3
(i) ∫ 1− x
dx

Solution: Set 1 - x = t2
dx = -2t dt
(1 − t 2 )3 (−2tdt)
∫ =∫ t
= −2 ∫ (1 − t 2 )3 dt

 3t 5 t 7 
= −2 ∫ (1 − 3t 2 + 3t 4 − t 6 )dt = 2  −t + t 3 − + +C
 5 7

 3
3 5
1 7

= 2  − 1 − x + (1 − x) 2 − (1 − x) 2 + (1 − x) 2  + C
 5 7 

1 + x2
(ii) ∫ 1 − x2
dx

Solution: Set x = sin q


dx = cos q dq
(1 + sin 2 θ)cos θdθ
Substituting , ∫ = ∫ = ∫ (1 + sin 2 θ)dθ
cos θ
1 − cos2 θ 3 1 sin 2 θ 3 −1 1
= ∫1 + dθ = θ − = sin x − × 2x 1 − x 2
2 2 2 2 2 4

3sin −1 x x 1 − x 2
= − +C
2 2

a−x
(iii) ∫ x dx
a+x

x(a − x) xdx − x 2 dx
Solution: =∫ dx = a ∫ +∫
a2 − x2 a2 − x2 a2 − x2
(a 2 − x 2 ) − a 2 dx
= −a a 2 − x 2 + ∫ dx = −a a 2 − x 2 + ∫ a 2 − x 2 dx − a 2 ∫
2 2
a −x a − x2
2
Integral Calculus  3.59

 x a2 − x2 a2  x  x
= −a a 2 − x 2 +  + sin −1    − a 2 sin −1   + C
 2 2  a   a 

x a2 − x2 a2 x
= −a a 2 − x 2 + − sin −1   + C
2 2 a 
We can also use the substitution x = a cos 2q and then reduce the integral to standard form.

(x 3 + x 2 + x + 1)dx
(iv) ∫ x 2 + 2x + 3

(x 2 + 1)(x + 1)dx [(x 2 + 2x + 3) − (2x + 2)][x + 1]


Solution: =∫ =∫ dx
x 2 + 2x + 3 x 2 + 2x + 3
(x + 1)2 dx
= ∫ x 2 + 2x + 3(x + 1)dx − 2 ∫
(x 2 + 2x + 3)
x 2 + 2x + 3 − 2
= ∫ x 2 + 2x + 3(x + 1)dx −2 ∫ dx
( x 2 + 2x + 3 )
dx
= ∫ x 2 + 2x + 3(x + 1)dx − 2 ∫ x 2 + 2x + 3dx + 4 ∫
2
x + 2x + 3
3
2 (x 2 + 2x + 3)
( 2 ) dx + 4 ∫ dx
2 2
= − 2 ∫ (x + 1)2 +
3 2
( 2)
2
(x + 1)2 +

3
 (x + 1) x 2 + 2x + 3 
=
(x 2 + 2x + 3) 2
3
− 2
 2
(
+ log x + 1 + x 2 + 2x + 3 )
(
+ 4 log x + 1 + x 2 + 2x + 3 + C )
3

=
(x 2 + 2x + 3) 2
3
(
− (x + 1) x 2 + 2x + 3 + 2 log x + 1 + x 2 + 2x + 3 + C )
dx
(v) ∫ (2x + 1) x 2 + 5x + 3

1
−1
1 −1 1−t
Solution: Put 2x + 1 = ⇒ 2dx = 2 dt and x = t =
t t 2 2t
2
2  1 − t  5(1 − t)
x + 5x + 3 =  + +3
 2t  2t

(1 − t)2 + 10t(1 − t) + 12t 2 3t 2 + 8t + 1


= =
4t 2 4t 2
3.60  Integral Calculus

 −1 
 2  dt dt
2t
∫ =∫  1  2
(3t + 8t + 1)
= −∫
2
3t + 8t + 1
 t 
4t 2

 2

  4  1 16    4   13  
2 2

We have 3t + 8t + 1 = 3   t +  + −  = 3   t +  − 
2
 
  3  3 9    3   3  

−1 dt
Therefore, ∫= 3
∫ 2 2
 4   13 
 t + 3  −  3 
 

 2 2 
−1  4   4   13  
= log   t +  +  t +  −  +C
3  3   3   3  
 
1
where, t =
(2x + 1)

dx
(vi) ∫ e2 x − 1

dx e − x dx
Solution: =∫ =∫
e 2 x (1 − e −2 x ) 1 − (e − x )2
t = e-x , dt = -e-x dx
−dt
∫ = ∫ 1 − t 2 = - sin-1 t = - sin-1(e-x) + C
2x 2 + 5x + 15
(vii) ∫ (x + 2)(x 2 + 9) dx
2x 2 + 5x + 15 A Bx + C
Solution: Let = + 2
(x + 2)(x + 9) (x + 2) (x + 9)
2

A(x2 + 9) + (Bx + C) (x + 2) = 2x2 + 5x + 15


x = -2 ⇒ 13A = 8 - 10 + 15 = 13
A=1
A+B=2⇒B=1
9A + 2C = 15 ⇒ C = 3
 1 x +3 
∫ = ∫  x + 2 + x 2  dx
+9
1 x
= log(x + 2) + log(x 2 + 9) + tan −1   + C
2 3
Integral Calculus  3.61

dx
(viii) ∫x 4
+ 1 + 7x 2

1 2 1   1 
dx  1 + 2  − 1 − 2   dx
dx
x2 1 1 
x2 x x 
Solution: =∫ = ∫ = ∫
1 2 1 2  2 1 
x2 + 7 + 2 x2 + 7 + 2  x + 2 + 7 
x x x

 1   1 
1 1 + 2  dx
1 1 − 2  dx
x x
2∫
= − ∫
1
2
2  1
2

 x − x  + 9  x + x  + 5

 1  1
x− x+
1 −1  x  1 −1  x +C
= tan   − tan  
2 ×3  3  2 5  5 
   

Connector 6: Solve:

sin −1 x
(i) ∫ x 2 dx
Solution: Let x = sin q
dx = cos q dq
θ cos θdθ
∫ =∫ sin 2 θ
= ∫ θ cos ecθ cot θ = ∫ θd[ − cos ecθ]

= −θ cos ecθ + ∫ cos ecθdθ = −θ cos ecθ + log(cos ec θ− cot θ) + C

sin −1 x 1 
=− + log  − x 2 − 1  + C
x x 

(ii) ∫ x 2 sin 2 x dx

x2
Solution: = ∫ 2 (1 − cos2x ) dx
x3 1 2
6 2∫
= − x cos2x dx

x 3 1  2  + sin 2x   − cos2x   − sin 2x  


= − x   − 2x   + 2  8   + C
6 2   2   4 
x3 x2 x cos2x sin 2x
= − sin 2x − + +C
6 4 4 8

x3  1 x2  x cos2x
= + −  sin 2x − +C
6 8 4 4
3.62  Integral Calculus

(iii) ∫ x 2 sin 4x Cos3x dx

Solution: ∫x
2
sin 4x cos3x dx
x2
= ∫ 2 (sin 7x + sin x ) dx
x 2  − cos 7x   − sin 7x   cos 7x 
=  − cos x  − ( x )  − sin x  + (1)  + cos x  + C
2  7   49   343 

(iv) ∫ sin(log x)dx

1
Solution: = ∫ sin(log x)d(x) = x sin(log x) − ∫ x × cos(log x) × dx
x
= x sin(log x) − ∫ cos(log x)dx
 x × − sin(log x) 
= x sin(log x) −  x cos(log x) − ∫ dx 
 x 
= x sin(log x) − x cos(log x) − ∫ sin(log x)dx
x
⇒ ∫ sin(log x)dx = 2 {sin(log x) − cos(log x)} + C
(v) ∫ cos xdx
Solution: Put x = t2
dx = 2t dt
∫ = ∫ (cos t) × 2tdt = 2 ∫ td(sin t) = 2{t sin t − ∫ sin tdt}
= 2{t sin t + cos t} + C
= 2{ x sin x + cos x } + C

∫e
3x
(vi) sin 2 x cos 3 xdx

 1 − cos2x   cos3x + 3cos x 


Solution: sin 2 x cos 3 x =   
 2  4 
1
= {cos 3x + 3cos x - cos 2x cos 3x - 3 cosx cos 2x}
8
1 1 3 
= cos3x + 3cos x − (cos5x + cos x) − (cos3x + cos x)
8 2 2 
1 1 1 
= cos x − cos3x − cos5x 
8 2 2 

∫e
3x
sin 2 x cos 3 xdx
1 3x 1 1
=
8 ∫ e cos xdx − ∫ e 3 x cos3xdx − ∫ e 3 x cos5xdx
16 16

1 e 3x
 
1 e 3x
 1  e3x 
=  (3cos x + sin x) −  (3cos3x + 3sin 3x) −  (3cos5x + 5sin 5x) + C
8  10  16  18  16  34 
Integral Calculus  3.63

x tan −1 x
(vii) ∫ 3
dx
(1 + x 2 ) 2
Solution: Put x = tan q
dx = sec2 q dq
θ tan θ sec 2 θdθ
∫ ∫ sec 3 θ = ∫ θ sin θdθ = - q cos q + sin q + C
=

x tan −1 x (x − tan −1 x)
= − +C = +C
1 + x2 1 + x2 1 + x2

∫ (log x )
2
(viii) dx

1
Solution: = ∫ (log x)2 d(x) = x(log x)2 − ∫ x × (2 log x) × dx
x
= x(log x)2 − 2 ∫ log xdx = x(log x)2 − 2 ∫ log xd(x) = x(log x)2 − 2  x log x − ∫ dx 
= x(log x)2 - 2x log x + 2x + C

Connector 7: Evaluate ∫
1
t log t dt .

2 3  2 3 2 3 1
Solution: ∫ t log t dt = ∫ log td  t 2  = t 2 log t − ∫ t 2 × dt
3  3 3 t
2 3 2 1 2 3 2 2 3
= t 2 log t − ∫ t 2 dt = t 2 log t − × t 2
3 3 3 3 3
Therefore, the definite integral
4
2 3 4 3 2 4 4 32 28
=  t 2 log t − t 2  = × 8 × log 4 − × 8 + = log 2 −
3 9 1 3 9 9 3 9


1
Connector 8: Evaluate ∫ 4
dx .
2
(x + 3) 3

−4 ∞
+1 ∞    
1 (x + 3) 3 −3  −3  = 0 −  −3  = 3
Solution: ∫ 4
dx =
 −4 
= 1 ∫2 
= 1 
 13  1
(x + 3) 3  3 + 1  (x + 3)  (x + 3) 3  2 5 53
3

sec x cos x sec 2 x + cot x cos ecx π


2
Connector 9: 2
If f (x) = cos x cos x 2
cos ec x 2
find ∫ f (x)dx .
2 2 0
1 cos x cos x

sec x cos x − sec x cos 2 x (sec 2 x + cot x cos ecx − sec x cos 2 x)
Solution: f (x) = cos 2 x cos 2 x − cos 4 x cos ec 2 x − cos 4 x
1 0 0
C2 - (cos2 x)C1 and C3 - (cos2 x)C1
3.64  Integral Calculus

sec x 0 sec 2 x + cot x cos ecx − sec x cos 2 x


= cos 2 x cos 2 x sin 2 x cos ec 2 x − cos 4 x
1 0 0
= (-cos2 x sin2 x) [sec2 x + cot x cosec x - sec x cos2 x]
= -[sin2 x + cos3 x - cos3 x sin2 x] = -[sin2 x + cos5 x]
π π
2 2

∫ f (x)dx = − ∫ (sin
2
x + cos 5 x)dx
0 0

 1 π 4.2  π 8 
= − × +  =−  + 
 2 2 5.3.1   4 15 

Connector 10: Evaluate the following definite integrals:


a
dx
(i) ∫x+
0 a2 − x2
Solution: Put x = a sin q
dx = a cos q dq
when x = 0, q = 0
π
when x = a, q =
2
π π
2 2
a cos θdθ cos θdθ
Given definite integral = ∫ =∫
0
a sin θ + a cos θ 0 sin θ + cos θ

π π π  π
cos  − θ  dθ
cos θdθ 2 2
2  2
sin θdθ
Let I = ∫ =∫ =∫
sin θ + cos θ 0 π  π  cos θ + sin θ
0
sin  − θ  + cos  − θ  0
2  2 
π
2 π
π π
⇒ 2I = ∫ 1dθ = θ 02 = ⇒ I=
0
2 4
π
4
(ii) ∫ sin 3 2x cos 3 2xdx
0

Solution: Set 2x = t
2dx = dt
π π
x = 0 → t = 0 and x = →t=
4 2
Given intergral
π
1 2 3
= ∫ sin t cos t dt
3

20

1 2 ×2 1
= × =
2 6 × 4 × 2 24
Integral Calculus  3.65

π
x sin 3 xdx
(iii) ∫0 1 + cos 2 x
π
x sin 3 xdx
Solution: Let ∫ 1 + cos
0
2
x
=I

π π
( π − x)sin 3 x sin 3 x
Then, I = ∫ dx = π ∫0 1 + cos 2 x dx − I
0 1 + cos 2 x
π
sin 3 xdx
⇒ 2I = π ∫ 2
 — (1)
0 1 + cos x

Put cos x = t ⇒ -sin x dx = dt


sin 3 x − sin 2 xdt
∫ 1 + cos2 x ∫ 1 + t 2
dx =

(t 2 − 1)  2 
= ∫ dt = ∫ 1 − 2 dt = t - 2tan-1 t
2
t +1  t + 1 
= cos x - 2tan-1 (cos x)
Substituting in (1)
π  2 π   π 
2I = π cos x − 2 tan −1 (cos x) = π   −1 + − 1− 
0
 4   2  
 π  2 π( π − 2)
= 2 π  − 1 =
2  2
π( π − 2)
⇒ I =
2
100 π

(iv) ∫
0
1 − cos2xdx

Solution: 1 − cos2x = 2 sin x


|sin x| is a periodic function with period p
100 π π

∫ 0
1 − cos2xdx = 100 ∫ 2 | sin x |dx
0

= 100 2 ∫ sin xdx = 100 2( − cos x)0π


0

= 100 2 × 2 = 200 2
5

(v) ∫ { x − 2 +  x } dx where [ ] represents the greatest integer function


−1

5
2 5

Solution: ∫ { x − 2 +  x } dx =  ∫ (2 − x)dx + ∫ (x − 2)dx  +
−1  −1 2 
0 1 2 3 4 5

 ∫ (−1)dx + ∫ 0dx + ∫ 1dx + ∫ 2dx + ∫ 3dx + ∫ 4dx 
 −1 0 1 2 3 4 
3.66  Integral Calculus

2 5
 x2   x2 
=  2x −  +  − 2x  + (− x)0−1 + 1 + 2 + 3 + 4
 2  −1  2 2
5 25
=2+ + − 10 − 2 + 4 − 1 + 10 = 18
2 2
4

(vi) ∫ 3 2 x +1
dx
0

Solution: Put 2x + 1 = t2
2dx = 2t dt
x = 0 ⇒ t = 1
x = 4 → t = 3
3

Given definite integral = ∫ 3 t × tdt


1
3
3
 te e t log 3 
t log 3
= ∫ te t log 3
dt =  − 2 
1  log 3 (log 3) 1
 3e 3 log 3 e 3 log 3   e log 3 e log 3   81 27   3 3 
= −  −  log 3 − (log 3)2 = − − −
 log 3 (log 3)2   log 3 (log 3)2   log 3 (log 3)2 
78 24
= −
log 3 (log 3)2
1

(vii) ∫ cot −1 (1 + x 2 − x)dx


0

 1   1  −1  x + (1 − x) 
Solution: cot-1(1 + x2 - x) = tan −1   = tan −1   = tan  
1 + x − x 
2
1 − x(1 − x)  1 − x(1 − x) 
= tan-1 x + tan-1(1 - x)
1 1 1 1

∫ cot (1 + x − x)dx = ∫ tan xdx + ∫ tan (1 − x)dx = 2 ∫ tan xdx ,


−1 2 −1 −1 −1

0 0 0 0

a a

since ∫ f (x)dx = ∫ f (a − x)dx


0 0

1
 1  π 1 
= 2  x tan −1 x − log(1 + x 2 ) = 2  − log 2 
 2 0 4 2 
π
x 2 cos x
(viii) ∫ (1 + sin x) dx
0
2

π π π π
x 2 cos x 2  −1   − x 2  −1
Solution: ∫0 (1 + sin x)2 dx = ∫0  1 + sin x  1 + sin x  ∫0 1 + sin x × 2xdx
x d = −
0

π
x
= −π2 + 2 ∫ dx  — (1)
0
1 + sin x
Integral Calculus  3.67

π π π π
x ( π − x) ( π − x)dx dx
I=∫ dx = ∫ dx = ∫ = π∫ −I
0
1 + sin x 0
1 + sin( π − x) 0
1 + sin x 0
1 + sin x
π π
1 − sin x
2I = π ∫ 2
dx = π ∫ (sec 2 x − sec x tan x)dx = π[tan x − sec x]0π
0 cos x 0

= p[1 - (-1)] = 2p
Substituting in (1),
Given definite integral = -p2 + 2p
π
x
(ix) ∫ 1 − cos α sin x dx
0

π π π
x ( π − x)dx dx
Solution: I=∫ dx = ∫ = π∫ −I
0
1 − cos α sin x 0
1 − cos α sin( π − x) 0
1 − cos α sin x
π
π 2
dx dx
2I = π ∫ = 2 π∫ , since f(2a - x) = f(x)  — (1)
0
1 − cos α sin x 0
1 − cos α sin x
dx
∫ 1 − cos α sin x
2dt
1 + t2 dt dt
=∫ = 2∫ 2
= 2∫
 2t  (1 + t − 2t cos α) (t − cos α)2 + sin 2 α
1 − (cos α)  2 
1 + t 
 x 
tan − cos α
1 −1  t − cos α  2 −1  2 
=2 tan   = tan  
sin α  sin α  sin α sin α
 
 
Substituting in (1),
π
  x  2
 tan − cos α 
2π −1  2  2 π  −1  1 − cos α  
I= tan    = tan   − tan −1 ( − cot α )
sin α  sin α sin α  sin α  
 
    0

2 π  −1  α π  2π α π  π( π − α)
= tan  tan  + − α  =  + − α =
sin α  2 2  sin α  2 2  sin α
π
2
dx
(x) ∫a
0
2
cos 2 x + b 2 sin 2 x
π π π
2 2 2 2
dx sec xdx 1 sec 2 x
Solution: ∫0 a 2 cos 2 x + b2 sin 2 x ∫0 a 2 + b2 tan 2 x b2 ∫0  a 2
= = dx
2
 b  + tan x
π
1 1  −1  b tan x   2 1 π π
= 2 × tan    = × =
b a  a   0 ab 2 2ab
 b 
3.68  Integral Calculus

3
2
Connector 11: Evaluate ∫ | x sin πx | dx .
−1


Solution: Note that sin q is positive in 0 < q < p and is negative in p < q ≤
2
Hence, we may write the definite integral as
3
1 2
∫ x sin πxdx + ∫[− x sin πx]dx
−1 1

3 3
1 2 1
 − x cos πx sin πx   − x cos πx sin πx  2
= 2 ∫ x sin πxdx − ∫ x sin πxdx = 2  + − +
0 1  π π2  0  π π2 1
2  −1 1  3 1
= − − = +
π  π2 π  π π2

π
2
Connector 12: Evaluate ∫ log tan xdx .
0

π π π
2 2 2
π 
Solution: I = ∫ log tan xdx = ∫ log tan  − x  dx = ∫ log cot xdx
0 0
2  0

π π
2 2
2I = ∫ log(tan x cot x)dx = ∫ log1dx = 0
0 0

⇒ I = 0
π
2
Connector 13: Evaluate ∫ log sin θdθ .
0

π π π
2 2 2
π 
Solution: Let I = ∫ log sin θdθ = ∫ log sin  − θ  dθ = ∫ log cos θdθ
0 0
2  0

π π π π
2 2 2 2
1  1
2I = ∫ log(sin θ cos θ)dθ = ∫ log  sin 2 θ  dθ = ∫ log   dθ + ∫ log sin 2 θdθ
0 0
2  0
2  0

π 1
= log   + I1  — (1)
2 2 
π
2
I1 = ∫ log sin 2 θdθ
0

Put 2q = t
2dq = dt
π
when q = 0, t = 0 and when θ = ,t=p
2
Integral Calculus  3.69

π
π 2
1 1
I1 = ∫ log sin tdt = × 2 ∫ log sin tdt (since f(2a - q) = f(q)) = I
0
2 2 0

Substituting in (1),
π 1 π
I= log   = − log 2
2 2  2

1
log(1 + x)
Connector 14: Evaluate ∫
0 1 + x2
dx .

Solution: Put x = tan q


⇒ dx = sec2 q dq
x = 0, ⇒ q = 0
π
x = 1 ⇒ θ =
4
π
4
Definite integral = ∫ log(1 + tan θ)dθ
0

π
4
Let I = ∫ log(1 + tan θ)dθ
0

π π π
4
  π   4
 1 − tan θ  4
 2 
= ∫ log 1 +  tan  − θ    dθ = ∫ log 1 +  d θ = ∫ log   dθ
0   4   0  1 + tan θ  0
 1 + tan θ 

π π
4 4
π π
= (log 2)∫ dθ - I ⇒ 2I = (log 2)∫ dθ = log 2 ⇒ I = log 2
0 0
4 8

2 +h
1
Connector 15: Find lim
h →0 h ∫
2
1 + t 3 dt .

Solution: Let F(x) = ∫ 1 + t 3 dt ; F’(x) = 1 + x3


2

F(2 + h) − F(2)
lim is required and it is = F’(2) = 1 + x 3 at x = 2 or it is equal to 3
h →0 h

8
Connector 16: Find the area bounded by the x-axis, part of the curve y = 1 + and the ordinates at x = 2 and x = 4. If
x2
the ordinate at x = a divides the area into two equal parts, find a.
8
Solution: The curve y = 1 + is symmetrical about the y - axis. As x → ∞, y → 1. Therefore, y = 1 is an asymptote
x2
of the curve. Again, as x → 0, y → ∞. This means that the y axis is an asymptote of the curve. A rough
sketch of the curve is shown below.
3.70  Integral Calculus

8
y = 1+
x2

y=1

x
O x=2 a x=4

4 4
 8   8
Required area = ∫ 1 + 2  dx =  x − x  = 4
2
 x 2

a
 8  1
Given that ∫ 1 + 2  dx = 2 × 4 = 2
2
 x
a
 8
⇒  x −  = 2 ⇒ a2 - 8 = 0 ⇒ a = 2 2
 x 2

Connector 17: Find the area of the region in the first quadrant bounded by the y-axis and the curves y = sin x and
y = cos x.
Solution: y
y = cos x
1

π y = sin x
2
O π π
4

π
The two curves intersect at x = . We want the area of the shaded portion.
4
π
4 π
Required area = ∫ (cos x − sin x)dx = (sin x + cos x )04 = 2 − 1
0

Connector 18: Find the area bounded by the curve y = 3x(x - 2) 2 and the y
x-axis.
Solution: The curve intersects the x - axis at x = 0 and x = 2
y = 3x(x2 - 4x + 4) = 3[x3 - 4x2 + 4x] O
x
dy 2
= 3(3x2 - 8x + 4] = 3(3x - 2) (x - 2)
dx
Integral Calculus  3.71

dy 2 
is < 0 in  , 2  . The slope of the curve is as shown in diagram.
dx 3 
2 2
Required area = ∫ 3x(x − 2)2 dx = 3 ∫ (x 3 − 4x 2 + 4x)dx
0 0

2
x 4
4x  3
 32 
= 3 − + 2x 2  = 3  4 − + 8 = 4
 4 3 0  3 

Connector 19 Find the area bounded by one arch of the cycloid x = a(q - sin q), y = a(1 - cos q).

2π 2π 2π
 dx 
Solution: ∫θ=0 ydx = ∫0 a(1 − cos θ) ×  dθ  dθ = ∫0 a (1 − cos θ) dθ
2 2
Required area =

2π π
θ θ
= 4a 2 ∫ sin 4 dθ = 4a 2 × 2 ∫ sin 4   dθ , since y
0
2 0
2 
f(2a - q) = f(q) One arch of the
cycloid
θ
Put = t ⇒ dq = 2 dt
2
O
π
Limits for t become 0 and θ=0 θ = 2π
2
π
2

∫ sin
2 4
Required area = 16a θdθ
0

π π
2
(1 − cos2 θ)2 2
= 16a 2 ∫ dθ = 4a 2 ∫ (1 + cos 2 2 θ − 2 cos2 θ)dθ
0
4 0

  1 + cos 4 θ 
2

= 4a 2 ∫ 1 +   − 2 cos2 θ dθ
0
 2  

π π 3π
= 4a 2  +  = 4a 2 × = 3pa2
2 4  4

Connector 20: Find the area common to the parabola y = 5x2 and y = 2x2 + 9.
y
Solution: Solving the two equations, we get x = ± 3
2
By symmetry, area of the shaded portion y = 2x + 9

3
2
= 2 ∫ (2x 2 + 9) − 5x 2 ]dx y = 5x
0
3

= 2 ∫ (9 − 3x 2 )dx = 2[9x − x 3 ]0 3
0
x
= 12 3 O 3
3.72  Integral Calculus

Connector 21: Find the area bounded by the line y = x - 1 and the parabola y2 = 2x + 6.
Solution: y=x−1
Q P(5, 4)
(0, 6)

IV
I
(− 1, 0)
M A(1, 0)
O(0, 0) X
(− 3, 0) N
III II (5, 0)
B(0, 1)

(−1, −2) R
(0, − 6 )

We want the area of the region PARQP


Solving the two equations y = x - 1 and y2 = 2x + 6,
(x - 1)2 = 2x + 6
x2 - 4x - 5 = 0
(x - 5) (x + 1) = 0, x = -1, 5
The required area consists of 4 regions as shown in the figure:
5
Area of I = ∫ −3
2x + 6 dx - area of D ANP
5
2 1 1
(2x + 6 ) 2
3
= − ×4×4
3 2  −3 2

=
1 32
3
4 ( )
−8=
40
3
1 1
Area of II = ×1 ×1 =
2 2
1 3
Area of III (trapezium) =
2
(1 + 2 ) ×1 = 2
−1
2 1 8
(2x + 6 ) 2 .  =
−1 3
Area of IV = ∫ −3
2x + 6 dx =
3 2  −3 3
40 1 3 8
\ Total area = + + + = 18 units
3 2 2 3
Aliter:
4

Required area = ∫ (x
−2
1 − x 2 ) dy, based on y-axis

4
 y2 − 6 
= ∫  − ( y + 1)  dy
−2
2 
4
1
∫ (y )
2
= − 2y − 8 dy = 18 units
2 −2
Integral Calculus  3.73

Connector 22: Find the general solution of the differential equation


(y + y2 cos x)dx - (x - y3)dy = 0
Solution: The given equation is rewritten as (ydx - xdy) + y2cos x dx + y3 dy = 0
x
⇒ y2d   + y2 cos x dx + y3 dy = 0
y
x
⇒ d   + cos x dx + ydy = 0
y
x  y2 
⇒ d   + d(sin x) + d 
y 2  = 0

Integrating, the solution is


x y2
+ sin x + =C
y 2

Connector 23: Obtain the general solution of the differential equation


(x x 2 + y 2 − y)dx + (y x 2 + y 2 − x)dy = 0

Solution: The equation is rewritten as


x 2 + y 2 (x dx+ y dy) - (y dx + x dy) = 0

 x2 + y2  1 2
⇒ x2 + y2 d  2 2 2
 − d(xy) = 0 ⇒ 2 x + y d(x + y ) − d(xy) = 0
 2
3 3
1 2 2
Integrating, × (x + y 2 ) 2 − xy = C ⇒ (x 2 + y 2 ) 2 − 3xy = C
2 3
y y
Connector 24: Obtain the general solution of the differential equation (2x − y)e x dx + (y + xe x )dy = 0
Solution: The equation is homogeneous
Put y = vx
dy dv
=v+x
dx dx
Substituting,
 dv 
(2x - vx)ev + (vx + xev)  v + x  = 0
 dx 
 dv 
⇒ (2 - v)ev + (v + ev)  v + x  = 0
 dx 
dv
⇒ (2 - v)ev + v(v + ev) + (v + ev)x =0
dx
dv dx (v + e v )dv
⇒ (2ev + v2) + (v + ev)x =0 ⇒ + =0
dx x (v 2 + 2e v )
Integrating,
1
log x + log(v 2 + 2e v ) = log C
2
3.74  Integral Calculus

⇒ log[x2 (v2 + 2ev)] = log C


y
⇒ log(y 2 + 2x 2 e x ) = log C
y
⇒ y 2 + 2x 2 e x = C

dy
Connector 25: Obtain the general solution of the differential equation 3x(1 - x2) y2 dx + (2x2 -1)y3 = ax3.

Solution: The equation is rewritten as


dy (2x 2 − 1) 3 ax 3
3y 2 + y =
dx x(1 − x )
2
x(1 − x 2 )
dy dY
Put y3 = Y ⇒ 3y 2 =
dx dx
Substituting,
dY (2x 2 − 1) ax 3
+ Y = — (1)
dx x(1 − x 2 ) x(1 − x 2 ) 
−1 1
2x 2 − 1 −1 2 2
P = = + +
x(1 − x 2 ) x 1 + x 1 − x
1  
1 1  1 

2 2
Pdx = − log x − log(1 + x) − log(1 − x) = − log x(1 − x ) = log
2 2  1 

 x(1 − x ) 2 
2

1
e∫ =
Pdx
1
x(1 − x 2 ) 2
General solution is
y3 ax 3 1
1
=C+∫ × dx
2 2 x(1 − x 2 ) 1
2 2
x(1 − x ) x(1 − x )
−1
xdx a(1 − x 2 ) 2 −1
= C + a∫ = C + ×
2 2
3
−1 2
(1 − x )
2
−1
= C + a(1 − x 2 ) 2 ⇒ y 3 = ax + Cx 1 − x 2
Integral Calculus  3.75

To p ic G r i p

Subjective Questions

1. Obtain the anti derivatives of the following.


x2
(i)
(a + bx )
2

(x − 1)
(ii)
(1 + x) x 3 + x 2 + x
x2 + 1
(iii)
x4 +1
1
(iv)
(5 − 4x − x )
5
2 2

(v) tan x
1
(vi) 1
− (1 + x )
1
(1 + x) 2 3

x2
(vii)
(x sin x + cos x)2

10 sin 2 11x
2. (i) Show that ∑ sin (2r + 1) x =
r =0 sin x
.

sin 2 11x
(ii) Hence evaluate ∫ dx .
sin x

 1  2 − 4x
3. f : R-{0, 1} → R and f(x) + f  = . Then
 1 − x  x − x 2
(i) find f(x)
(ii) find g(x) where g(x) = ∫ f ( x ) dx; g (2 ) = 2
  
4. Let a i + b j + ck be the position vector of a point A.
a2 + x2 ab ac
2 2
(i) Find f(x) = ab b +x bc
ac bc c + x2
2

1
12
(ii) Find the locus of A if ∫ f ( x ) dx =
0
35

5. (i) Write the equation of the system of circles having the line of centres as y-axis and radical axis as the x-axis.
(ii) Find the differential equation of above system.
3.76  Integral Calculus

b
dx π
6. Show that ∫ x ( x − a )( b − x ) =
a ab
(a, b > 0)

1
2 2 n +1 (n!)2
7. If n is a positive integer or zero, show that ∫ (1 − x ) dx =
2 n
.
−1
(2n + 1)!

8. Evaluate: ∫ x sin(( πx  x  ) dx
−2
where, [ ] represents the greatest integer function.

π
2
sin nx π
9. If n = 2 m + 1, m = 0, 1, 2 show that ∫
0
sin x
dx =
2
.

x
x
sin t
10. Evaluate lim
x →5 (x − 5) ∫
5
t
dt .

x
[x]([x] − 1)
11. Prove that ∫ [t]dt = + [x](x − [x]) where, [ ] represents the greatest integer function.
0
2

10
 7x − 8 
12. Let D be an interval that does not include x = 7. If f 
 x − 7 
= x for all x ∈ D, evaluate ∫ f (f (f (x)))dx
8

13. (i)  Sketch the region bounded by 0 ≤ y ≤ x2 +1, 0 ≤ y ≤ x + 1, 0 ≤ x ≤ 2


(ii) Find its area.
(iii) Find the area of the region bounded byx2 = y-1, x-y+1 = 0 and x = 2 in first quadrant.

1 + x2
14. (i) Find ∫ x
dx
dy
(ii) Obtain the general solution of the differential equations 1 + x 2 + y 2 + x 2 y 2 + xy =0
dx
tan −1 x tan −1 x
15. (i)  Find ∫ 1 + x 2 e dx
(ii) Solve the equation: (1 + x2) dy + (y - tan-1x) dx = 0

Straight Objective Type Questions

Directions: This section contains multiple choice questions. Each question has 4 choices (a), (b), (c) and (d), out of
which ONLY ONE is correct.

16. If x = sec y, then ∫ y dx =

(a) y + log x + x 2 − 1 + C (b) xy − log x + x 2 − 1 + C


1
(c) 2y + log x + x 2 − 1 + C (d) xy + 2 log x + x 2 − 1 + C
2
Integral Calculus  3.77

π
dx
17. ∫ 5 + 4 cos x is
0

π π π π
(a) (b) (c) (d)
2 4 3 6

18. The area bounded by the curves y = x , x = 2y + 3 in the first quadrant and x-axis is

(a) 2 3 (b) 18 (c) 9 (d) 18 3


dy
19. x = y + x 2 + y 2 is the differential equation of the family of curves represented by
dx
(a) y + x 2 + y 2 = Cx 2 (b) x + x 2 + y 2 = Cy 2 (c) x + x 2 + y 2 = Cy (d) y 2 + x 2 + y 2 = Cx
π
2
20. Let f: R → R and g: R → R be continuous functions. Then the value of the integral ∫ f (x) + f (− x) g(x) − g(− x)dx =
−π
2

π  π π  π
(a) 1 (b) 0 (c) f   + f  −  (d) g   + g  − 
2   2  2   2 

dy
21. General solution of the equation = sin ( x + y ) + cos ( x + y ) is
dx
 x +y x+y
(a) log 1 + tan = x + C (b) 1 + tan = Ce x
 2  2
 x +y
(c) log 1 − tan = x + C (d) Both (a) and (b)
 2 

22. A curve passes through the point (5, 3) and at any point (x, y) on the curve, the product of its slope and the ordinate
is equal to its abscissa. The equation of the curve is
(a) x2 + y2 = 34 (b) x2 – y2 = 16 (c) 5x2 - 3y2 = 98 (d) 3x - 5y = 0
y
1 d2 y
23. The variables x and y are related by the equation x = ∫
0 1 + 9v 2
dv , then
dx 2
(a) varies directly as y (b) varies inversely as y (c) is a constant (d) is independent of y

 1 x+ 
1
x+ e x
24. ∫  e x − 2  dx is
 x 
 
x+
1 1 x+
1
1 x+
x+ e x
x + C −e x
(a) e x
+ C (b) + C (c) - e (d) +C
x x
dx
25. ∫ 3x 2
+ 2x + 7
is

1  3x + 1  1  3x + 1 
(a) tan −1  + C (b) tan −1  +C
5  5  2 5  5 
1  3x + 1  1  x +1
(c) tan −1  + C (d) tan −1  +C
2 5  2 5  2 5  2 5 
3.78  Integral Calculus

1 + x2
26. ∫ (1 − x )(1 + x + x ) dx = A log(1 +x) + Blog(1 - x ).
2 2
3
Then A + 3B is

(a) 0 (b) 1 (c) -1 (d) 2

ex
27. ∫ 3e x − e2x − 2
dx =

(a) sin-1 ( )
ex + 1 + C (b) 2sin −1 ( )
ex − 1 + C (c) 2sin −1 ( )
ex + 1 + C (d) sin-1 ( )
ex − 1 + C
3
1 x +3
28. Coefficient of in ∫  dx is
x +2  x + 2 
(a) 1 (b) -1 (c) 3 (d) -3
2
3x − x + 7
29. If F(x) = ∫ ( x − 1) 7
dx and 4F(0) + 5 = 0. Then, coefficient of x2 in 4(x - 1)6 F(x) is

(a) -2 (b) 2 (c) 3 (d) -3

ncos n −1 x
30. ∫ (1 + sin x ) n
dx equals

cos n x sin n x − cos n x


(a) +C (b) +C (c) +C (d) (secx + tanx)-n + C
(1 + sin x ) (1 + sin x ) (1 + sin x )
n n n

Assertion–Reason Type Questions


Directions: Each question contains Statement-1 and Statement-2 and has the following choices (a), (b), (c) and (d), out
of which ONLY ONE is correct.
(a) Statement-1 is True, Statement-2 is True; Statement-2 is a correct explanation for Statement-1
(b) Statement-1 is True, Statement-2 is True; Statement-2 is NOT a correct explanation for Statement-1
(c) Statement-1 is True, Statement-2 is False
(d) Statement-1 is False, Statement-2 is True
31. Statement 1
dx
∫ 2 x (1 + x ) = tan x + C.
−1

and
Statement 2
dx
∫ ax + b cx + d can be evaluated by using the substitution cx + d = t .

32. Statement 1

∫ sin
2
Let m, n be positive integers mx cos nxdx = 0 .
0

and
Statement 2

∫ sin mx cos nxdx = 0 , where m and n are positive integers.


0
Integral Calculus  3.79

33. Statement 1
T 3+3T
3
Let f(x) be a periodic function with period T and I = ∫ f (x)dx. Then ∫ f (2x)dx is Ι.
0 3
2
and
Statement 2
a +nT T

If f(x) is periodic with period T then ∫ a


f (x)dx = n ∫ f (x)dx, n ∈ Z .
0

34. Statement 1
8
The area bounded by the parabola y2 = x, straight line y = 2 and y axis is .
3
and
Statement 2
If the equation of a curve remains unchanged on interchanging x and y, then the curve is symmetric about the
line y = x.
35. Statement 1
The differential equation of all parabolas with their axes along the x-axis is 2, is of order 2.
and
Statement 2
If the general equation of a family of curves contains 2 arbitrary constants then the differential equation of that family
of curves is of order 2.
36. Statement 1
f(x) is a polynomial of degree 3 such that f(0) = 2, f(1) = 1 and 0 is a critical point of f(x). However, f(x), is neither a
maximum nor a minimum at x = 0.
f (x)dx 1
( )
3/2
Then ∫ = 2 log x + x 2 + 5 - x 2 + 5 + 5 x 2 + 5 + c
2
x +5 3
and
Statement 2
f is twice differentiable and c is a root of the equation f ”(x) = 0. Then f(x) has a local maximum at x = c if f ''(c) < 0
and f(x) has a local minimum at x = c if f ” (c) > 0.
37. Statement 1 π

For any integer n the integral ∫ e cos x cos 3 (2n+1) x dx has the value 0.
2

0
and
Statement 2
a a

∫ f (x) dx = ∫ f (a − x)dx
0 0

38. Statement 1
π
sin (2n + 1) x
For all positive integral values of n > 1, ∫
0
sin x
dx = nπ

and
Statement 2
C +D  D−C 
cos C - cos D = 2 sin   sin 
 2   2 
3.80  Integral Calculus

39. Statement 1
x

f(x) = ∫ e t t 2 + ab − t (a + b ) dt where a < b is increasing in (a, b).


0

and
Statement 2
If f(x) is an increasing function in an interval I, then f ’(x) ≥ 0 in I.
40. Statement 1
x2
The area bounded by the curve y = f(x) = x4 - 2x3 + + 5, the x-axis and the ordinates corresponding to the
2
73
minimum of the function f(x) is .
15
and
Statement 2
Let f(x) be differentiable and f ’(a) = 0 Then, f(x) is a minimum at x = a if f ’(x) changes sign from negative to positive
as x crosses the point a.

Linked Comprehension Type Questuion

Directions: This section contains a paragraph. Based upon the paragraph, multiple choice questions have to be answered.
Each question has 4 choices (a), (b), (c) and (d), out of which ONLY ONE is correct.

Passage I
Beta and Gamma functions
1

∫ x (1 − x )
m −1 n −1 1 ∞
dx
= ∫ (1 − x ) x n −1 dx = β (n, m ) , m, n>0 and Γ (n) = ∫ e − x x n −1 dx where, n > 0.
m −1
We define b(m, n) =
0
0 0
Some results:
Γm, Γn,
(i) G (n) = (n-1) G (n-1) (ii) β(m, n)
Γm + n)
(iii) If n is a positive integer, Γn = (n − 1)! (iv) Γ (1) = 1

1 x n −1
(v) Γ   = π (vi) β (m, n ) , = ∫ dx
2  (1 + x )
m +n
0


x n −1 π
(vii) ∫0 1 + x dx = sin nπ , 0 < n < 1

∫x e
3 −x
41. dx is
0

(a) 2 (b) 6 (c) 24 (d) 12


1

∫ x (1 − x ) dx
3
42. 4

1 1 1 1
(a) (b) (c) (d)
140 35 40 280
Integral Calculus  3.81

∫e
− cx
43. x n −1 dx
0
n Γn Γn
(a) c Γ n (b) c n −1 Γ n (c) (d)
c n
c n −1

x4
44. ∫ (1 + x )
0
6
dx is

(a) b (4, 6) (b) b(5, 1) (c) b(3, 3) (d) b (2, 4)


π
8 1
2

∫ (sin x ) (sec x )
3 2
45. dx is
0
11 1
ΓΓ
11 1 6 4
(a) Γ . Γ (b)
6 4 25
Γ
12
11 1 11 1
Γ .Γ 2Γ Γ
(c) 6 4 (d) 6 4
25 25
2Γ Γ
12 12

dx
46. ∫1+ x
0
4
=

π π π
(a) (b) (c) (d) 2π
2 2 2 2

Multiple Correct Objective Type Questions

Directions: Each question in this section has four suggested answers out of which ONE OR MORE answers will be correct.

47. ∫ tan 2x tan 3x tan 5xdx = log sec


a
2x.sec b 3x.sec c 5x + K. Then
(a) a, b, c are in H.P. (b) a = b = c
(c) ab + bc + ca = 0 (d) a-3 + b-3 + c-3 = 3a-1b-1c-1
f (x ) g(x )

48. Let x > 0. Given f(x) = x and g(x) = [f(x)] and ∫0


g(t)dt = ∫
0
f (t)dt , ( ⋅ denotes the greatest integer function and

{}⋅ denotes the fractional part function), then

1 1
(a) {x} = (b) [x] = 1 (c) {x} = (d) [x] = 0
2 4

1 −x y  y −1
49. Let y2 =
1+x
and ∫ x dx = g  y +1  + 2f(y) + C

where, C is an arbitrary constant. Then,


(a) g(x) = log|x| (b) g(x) = tan-1x (c) f(x) = log|x| (d) f(x) = tan-1x
3.82  Integral Calculus

Matrix-Match Type Question

Directions: Match the elements of Column I to elements of Column II. There can be single or multiple matches.

50. Given f(x) = e2x and g(x) = sin-1x


Column I Column II
π
e f   − f (0)
2 
∫ (f o f )(x)f
−1 −1
(a) (x)dx (p)
2
1

( )
1
1
(b) ∫ (f o g )(x)g '(x)dx (q) f − 3 − f ( −1)
4  2 
0
π
2
f (1) + f (0)
∫ f (x)g
−1
(c) (x)dx (r)
8
0
π
−1 2f   + f (0)
2 
(d) ∫ xf (x)dx = (s)
5
−3
2
Integral Calculus  3.83

IIT Assi g n m e n t E x e rc i se

Straight Objective Type Questions

Directions: This section contains multiple choice questions. Each question has 4 choices (a), (b), (c) and (d), out of
which ONLY ONE is correct.

x + 31
51. ∫x 2
+ 2x − 35
dx is

(x − 5) + C (x − 5) + C
2 2 2 3
 x −5 x +7
(a) log  +C (b) log  +C (c) log (d) log
 x + 7   x − 5  (x + 7 )
3
(x + 7 )
2

sin x dx
52. ∫ 1 + sin x is

(a) x – tan x – sec x + C (b) x + tan x – sec x + C (c) x + tan x + sec x + C (d) x – tan x + sec x + C

a−x
53. ∫ a+x
dx is

x x
(a) sin −1 + a 2 − x 2 + C (b) a sin −1 + a2 − x2 + C
a a
x x
(c) sin −1 − a 2 − x 2 + C (d) cos −1 + a2 − x2 + C
a a
x ex
54. ∫ (1 + x ) 2
dx is

ex ex
(c) e x (1 + x ) + C (d) e x (1 + x ) + C
2
(a) +C (b) +C
(1 + x )
2
(1 + x )

∫ tan
−1
55. x dx is

(a) x tan-1 x – log 1 + x 2 + C (b) (sec-1 x)2 + C


(c) x tan-1 x – log (1 + x2) + C (d) x tan-1 x + log (1 + x2) + C

∫e
−x
56. sin x dx is
1 1 x
(a) − e − x (sin x + cos x ) + C (b) e (sin x + cos x ) + C
2 2
1 −x
(c) e − x (sin x − cos x ) + C (d) e (sin x − cos x ) + C
2

g ( x ) − g (− x ) 3

57. If f(x) =
2
, defined over [-3, 3] and g(x) = 2x2 – 8x + 1, then ∫ f ( x ) dx is
−3

(a) –8 (b) 8 (c) 16 (d) 0


3.84  Integral Calculus

π
2
f (sin x )
58. ∫ f (sin x ) + f (cos x ) dx is
0

π π π
(a) (b) (c) (d) 0
2 3 4
π
2
dx
59. ∫ 1 + cot x is
0

π π
(a) (b) (c) 0 (d) does not exist
4 2

4e x + 6e − x
60. If ∫ 9e x − 4e − x dx = Ax + Blog(9e2x - 4) + C, then (A, B) is
 3 35   3 35   3   3 35 
(a)  , −  (b)  − , −  (c)  − , 0  (d)  − ,
 2 36   2 36   2   2 36 

∫ (x )( )
1
61. For any natural number t, 3t
+ x 2t + x t 2x 2t + 3x t + 6 t
dx is
t +1
1 1
(a)
6 (t + 1)
(
2x 3t + 3x 2 t + 6x t ) t
+C (b)
6t
(2x2t + 3xt + 6)t + C

1 1
( ) ( )
2
(c) x 3t + x 2 t + x t + C (d) 2x 2 t + 3x t + 3 + C
t +1 3t

62. Let g(x) be a function satisfying g1(x) = g(x) with g(0) = 1 and f(x) be a function that satisfies f(x) + g(x) = x2. Then
1

the value of the integral ∫ f ( x ) g ( x ) dx is


0

e−7 e−3 e2 3 e2 3
(a) (b) (c) e − − (d) e + +
4 2 2 2 2 2

63. The area enclosed by the curve x = 2 + 4 cos q and y = 3 + 3 sin q is


(a) 6p (b) 12p (c) 3p (d) 4p
64. The area of the smaller region bounded by the curve 4x + 9y = 36 and the line 2x + 3y = 6 is
2 2

3 3 1 5
(a)
2
( π − 2) (b)
2
( π − 1) (c)
2
( π − 4) (d) ( π − 2 )
2
3
  dy  2  2 d2 y
65. The order and degree of the differential equation 1 +    = a is
  dx   dx 2
(a) 1, 2 (b) 2, 2 (c) 3, 2 (d) 2, 3
66. The differential equation of the family of circles of radius ‘r’ and centre at (h, k) where, h and k are arbitrary
constants is
d2 y d2 y dy
(a) = 0 (b) +r +y =0
dx 2 dx 2 dx
3 2
  dy  2   2 
2
 dy  d2 y
2 d y
(c) 1 +    = r  2  (d) 1+   = r 2
  dx    dx   dx  dx
Integral Calculus  3.85

dy 2y
67. The differential equation representing a curve passing through (1, 2) is = . Then the equation of the curve is
dx x
(a) y = 2x2 (b) y = x2 + 1 (c) y = 2x (d) x = 2y - 3

dy  dy 
68. The solution of the differential equation y − x = a  y 2 +  is
dx  dx 
(a) y = C (a + x) (1 – ay) (b) y = C (a + x) (1 + a2 y)
(c) y = C (a + x) + log (1 + a2 y) (d) y = ca(x + y)

 dy 
69. The solution of sin −1   = x + y is
 dx 
(a) tan (x + y) – sec (x + y) = x + C (b) tan (x + y) + sec (x + y) = x + C
(c) tan (x + y) – sec2 (x + y) = x + C (d) tan (x + y) + sec2(x - y) = cx
sin x dx
70. ∫ sin x − cos x is

x x 1
(a) + sin x − cos x +C (b) + log (sin x − cos x ) +C
2 2 2
x x 1
(c) + log (sin x + cos x ) +C (d) + log (sin x + cos x ) +C
2 2 2

(1 + x )dx 2

71. ∫ 1 + x4
is

 x2 − 1  1  x2 − 1 
(a) tan −1   + C (b) tan −1  +C
 2x  2  2x 

1  x +1 1  x2 − 1 
(c) tan −1  + C (d) tan −1  +C
2  2  2  2x

x 3 dx
72. ∫ x 8 + 4x 4 + 13 is
1  x4 + 2  1  x4 + 2 
(a) tan −1  + C (b) tan −1  +C
6  3  12  3 
1  x4 + 2   x4 + 2 
(c) tan −1  + C (d) tan −1  +C
2  3   3 

 x + 1 − x2 
∫e
−1
73. sin x
  dx is
 1 − x2 
−1 −1

sin −1 x sin −1 x e sin x


xe sin x
(a) e +x +C (b) x e +C (c) +C (d) +C
1 − x2 1 − x2
log x dx
74. ∫ (1 + log x ) 2
is

x 1 x
(a) +C (b) x (1 + log x) + C (c) +C (d) +C
1 + log x 1 + log x (1 + log x )
2
3.86  Integral Calculus

2x + 3
75. ∫ x2 + x + 1
dx is

 1   1 
(a) 1 + x + x 2 + 2 log  x + + 1 + x + x 2  +C (b) 2 1 + x + x 2 +2 log  x + + 1 + x + x 2  + C
 2   2 

(c) 2 1 + x + x 2 − 2 log ( 1+ x + x )+C


2
(d) 2 1 + x + x 2 + log 1 + x + x 2 + C

π
2
76. ∫ sin
3
θ cos 4 θ dθ is
0

7π π 2 1
(a) (b) (c) (d)
2 2 35 35
1

77. ∫ × ( x + 3) dx is
−1

3 1
(a) 0 (b) (c) 3 (d)
2 3
1
2− x 
78. ∫  1 + x  dx is
0

(a) 3 log 2 (b) 2 log 2 - 1 (c) 3 log 2 + 1 (d) 3 log 2 - 1


e2 2
dx ex
79. If I1 = ∫ and I 2 = ∫ dx , then
e
log x 1
x
(a) I1 = 2 I 2 (b) I 2 = 2 I1 (c) I1 + I 2 = 0 (d) I1 − I 2 = 0

80. The total area bounded by the curve y = x (x – 1) (x – 2) and the x-axis is
1 1
(a) (b) 1 (c) (d) 2
4 2

x2 y2 x y
81. Area lying in the first quadrant between the ellipse + 2 = 1 and the straight line + = 1 is
a 2
b a b
π 1 ab ab ab
(a)
4
ab + ab
2
(b)
2
( π − 2) (c)
4
( π + 1) (d)
4
( π − 2)

82. Area of the region {(x, y): (x – 1)2 < y < |x – 1|} is
1 2 4 5
(a) (b) (c) (d)
3 3 3 3
83. The slope of a curve at any point is the reciprocal of twice the ordinate at the point and it passes through the point
(4, 3). The equation of the curve is
(a) y2 = x + 5 (b) y2 = 2x + 1 (c) x2 = y + 5 (d) 2x = 5 + y
dy x −2 x 2 (2x − 1)
84. An integrating factor of the equation − y= , is
dx x ( x − 1) x −1
x +1 x −1 x2 x2
(a) (b) (c) (d)
x2 x2 x +1 x −1
Integral Calculus  3.87

85. The general solution of the equation (x3 y2 + xy) dx = dy is


2 2
x x
1 1 − 1 − 1
(a) = x 2 + 2x + Ce − x (b) = − x 2 + 2 + Ce 2 (c) = 2 + x + Ce 2 (d) = 2 − x 2 + Ce − x
y y y y

 e −2 x y  dx
86. General solution of  −  = 1 is
 x x  dy

(a) y = 2 x e 2 x
+C (b) y = x + e 2 x
+C (c) y ⋅ e 2 x
=2 x +C (d) ye x
=2 x +C

x dy − y dx
87. General solution of the equation x dx + y dy + = 0 is
x2 + y2
y  x 
(a) x 2 + y 2 = tan −1   + C (b) x 2 − y 2 = tan −1  2  + C
x  x + y2
x  C2 − x 2 − y 2 
(c) x 2 + y 2 + tan −1   = C (d) y = x tan  
y  2 

dy
88. General solution of the equation − 2y tan x = y 2 tan 3 x is
dx
(a) 4 sec2 x + y tan4 x = 4Cy (b) sec2 x + tan2 xy = C
(c) 4 sec2 x = tan x + C2 y (d) tan2x + 4sec2y = C
e x (1 + x)
89. ∫ cos 2 (xe x )dx =
1
(a) tan ex + x + C (b) +C (c) sec (exx) + C (d) tan(exx) + C
cos(xe x )
1

90. ∫ [x + 1]dx , where [ ] represents the greatest integer function equals


−1

(a) 0 (b) -1 (c) 1 (d) 2


π

91. ∫ sin x + 2x sin 2 x(x 2 + 1) + x 2 sin x(1 + 2x 2 + x 4 ) dx is


3

−π

π
(a) 1 (b) -1 (c) (d) 0
4
30 π

92. ∫
−30 π
| cos x | dx =

(a) 0 (b) -120 (c) 120 (d) 120p


9
93. If the area of the region bounded by curve y = x - x2 and the line y = mx (m > 1) equals ,m
2
(a) 4 (b) - 4 (c) 2 (d) - 2
2
d y
94. The general solution of the equation = e −3 x is
dx 2
(a) y = 9e-3x + C1x + C2 (b) y = -3e-3x +C1x + C2
(c) 9y = e-3x +C1x + C2 (d) y = 3e-3x +C1x + C2
3.88  Integral Calculus

e2
log e x

95. ∫
1 x
dx is
e

5
(a) (b) 3 (c) 0 (d) 5
2
1000

96. ∫ 0
e {x } dx is equal to {x} = fractional part of x

(a) 1000e (b) 999(e - 1) (c) 1000(e - 1) (d) 1000(e + 1)


97. Area bounded by y = |x - 2|, y = 0 and |x| = 4 is
(a) 10 (b) 15 (c) 20 (d) 12
2
 dy  dy
98. If the curves given by the solutions of x   + (y − x) − y = 0 are passing through (2, 3) then another point of
 dx  dx
intersection is
(a) (-3, 2) (b) (3, -2) (c) (3, 2) (d) (-3, -2)
x +1
99. ∫ x(1 + xe x 2
)
dx is

xe x 1 xe x 1
(a) log x
− + C (b) log + +C
1 + xe 1 + xe x 1 + xe x
1 + xe x

1 + xe x 1 1 + xe x 1
(c) log x
+ + C (d) log − +C
xe 1 + xe x xe x
1 + xe x

e
 tan −1 x log x 
100. ∫  x + 1 + x 2  dx is
1

1
(a) -tan-1e + C (b) log(tan e) + C (c) tan −1   + C (d) tan-1 e + C
e
2
1− x
101. ∫1+ x
0
3
dx is

1 1 1
(a) log 3 (b) log 3 (c) log 3 (d) log 2
2 3 3

∫ (1 + x ) (1 + x )(1 + x ) (1 + x )... (1 + x )dx = ∑  1r  . Then the value of k is


1 k
2 4 8 2 n −1
102.
0 r =1

(a) n2 + 1 (b) n2 (c) 2n (d) 2n-1 + 1


3a
f (x)
103. If f(3a - x) = g(x), g(3a - x) = h(x) and h(3a - x) = f(x). Then ∫ f (x) + g(x) + h(x) dx
0
is

3a
(a) a (b) 2a (c) 3a (d)
2
x2
 t −1
104. The minimum value of ∫  t + 1  dx
0
is

(a) log e (b) log 4 (c) 1 - log e (d) 1 - log 4


Integral Calculus  3.89

(sin x ) dx is
2
1 −1

105. ∫
−1 1 + πsin x
π2 − 8 π2 + 8 π2 − 8 π2 + 8
(a) (b) (c) (d)
4 4 2 2

(tan x )
2
1 −1

106. ∫
−1 1 + e tan x
dx

(a) depends only on e (b) depends only on p


(c) depends on both e and p (d) does not depend on e and p

107. The value of ∫ f (x)dx


0
where, f(x) = max{1 + sinx, 1 - cosx}

(
(a) 2 π + 2 ) (
(b) 2 π − 2 ) (c) 2 (π + 2 ) (d) 2 (π − 2 )

3
 πx 
108. ∫ sin  2
1
 dx equals

π π 4 2
(a) (b) (c) (d)
4 2 π π

(1 − x ) sin −1 (1 − x )
3
1

109. ∫0 x (2 − x )
dx is

(a)
(
4 π2 + 8 ) (b)
(
4 π2 − 8 ) (c)
(
3 π2 + 8 ) (d)
(
3 π2 − 8 )
3 3 4 4
6

∫ (sin )
2
−1
110. {x} d{x} , where {x} is fractional part function is
0

(a) 3(p + 8) 2
(b) 3(p - 8)
2
(c)
(
3 π2 − 8 ) (d)
(
3 π2 + 8 )
2 2
111. The area bounded by 2y = (1 + y )x and its vertical asymptotes is
2 2

π
(a) (b) p (c) 2p (d) 4p
2
112. Area bounded by y2(x - 3) = 4(4 - x)5, the ordinates x = 3, x = 4 and above the x-axis is
5π 5π 3π π
(a) (b) (c) (d)
6 8 8 2
113. Area bounded by x-axis and the curve f(x) = e[x] . e|x| . e{x} between the lines x = -1 and x = 2, where [ ] represents
greatest integer function and { } represent fractional part function, is
e +1 e2 + 1 e3 + 1 e4 + 1
(a) (b) (c) (d)
2 2 2 2
114. A curve y = f(x) passes through (3, 0) and slope of the normal of y = f(x) at any point is 2(2 - y). Then area bounded
by that curve, x axis and one of its tangent at (0, 3) is
1 9
(a) 1 (b) 9 (c) (d)
2 2
3.90  Integral Calculus

115. Area bounded by the pair of straight line y2 = 2 and the curve y2 = 3(1 - x2) is
2  −1 2 2  −1 2
(a)  2 2 + 3tan  (b)  2 2 + 3sin 
3 3 3 3

2  −1 2 2  −1 2
(c)  2 + 3 sin  (d)  2 + 3sin 
3 3 3 3

dy
116. If = 3x 2 y 2 + 3x 2 + y 2 + 1; y(0) = 0. Then y equals
dx

(a)
( )
tan x + tan x 3
(b)
tan x − tan x 3
(c) tan-1(x + x3) (d) tan-1x + tan-1x3
1 − tan x.tan ( x ) 3
1 + tan x.tan x 3

 dy  π
117. log sec x.  = sin x + 2 log sin x . Then y   is
 dx  2 
1
(a) 0 (b) 1 (c) e (d)
e
x

3 x
 π
118. If ∫ ( )
1 − f '(t)2 dt = ∫  ∫ f (z)dz  t  dt, [ . ] represents greatest integer function and f(0) = 1, then f   is
2 
−x 2 0 
(a) 1 (b) -1 (c) 2 (d) 0
dy x
1
dx y
119. Solution for 1 0 1 = 0 is
y dy
1
x dx
(a) x2 + y2 = cx (b) x2 + y2 = cy (c) x2 - y2 = cx (d) x2 - y2 = cy
x

120. If x(fx) + ∫ f (z)dz = xe x , f(1) = e, Then f(2) is


0

3e 3e 2 4e 2 4e
(a) (b) (c) (d)
4 4 3 3
1 − cot x
121. ∫ 1 + cot x dx is
(a) log |sin x + cos x| + C (b) log |sin x – cos x| + C
(c) –log |sin x + cos x| + C (d) –log (1+ cot x) + C

122. ∫ 1 + sin 2x dx is
(a) cos x + sin x + C (b) sin x – cos x + C (c) –sin x – cos x + C (d) sin x cos x + C

∫x
3
123. log 2x dx is
x4 x4 x4
(a) x 3 log 2x − + C (b) log x − +C
16 4 16
x4 x4 x4 x4
(c) log 2x + + C (d) log 2x − +C
4 16 4 16
Integral Calculus  3.91

e x dx
124. ∫ (2 + e ) (e x x
+1 )
is

 ex + 1  1  ex + 1   ex + 2 
(a) (e x
)(
+ 2 ex + 1 + C ) (b) log  x
 e + 2 
+C (c)
2
log  x
 e + 2 
+C (d) log  x
 e + 1 
+C

125. If f(x) = A . 2x + B such that f ’ (1) = 2 and ∫ f ( x ) dx = 7, then B is


0
2
 1  7 7 7
1 − ( log 2 e )  1 + ( log 2 e ) 
2 2
(a) 1 −  (b) (c) (d) 1 − log 2 e 
 log e 2  3  3  3
b

126. If f(a + b – x) = f(x), then ∫ x f ( x ) dx is


a

b b b b
a−b a+b
f ( x ) dx f ( x ) dx (a + b ) ∫ f ( x ) dx (d) (a − b ) ∫ f ( x ) dx
2 ∫a 2 ∫a
(a) (b) (c)
a a


x 2 dx π

dx
127. If ∫ = , then the value of ∫ (x is
0 (x 2
+a 2
) (x 2
+b 2
) (x 2
+c 2
) 2 (a + b ) ( b + c ) (c + a ) 0
2
)(
+ 4 x2 + 9 )
π π π π
(a) (b) (c) (d)
60 16 12 5

( ) dx is
2
π 3
3
128. ∫ 0
x cos 2 x 2

π π π π
(a) (b) (c) (d)
3 4 6 12
4 4 2

129. If ∫ f ( x ) dx = 4
−1
and ∫ 7 − f ( x )  dx = 7 , then the value of
2
∫ f ( x ) dx is
−1

(a) 5 (b) 4 (c) 3 (d) –3

130. The area of the region bounded by the parabola y = x2 and the lines y = |x| is
1 2 1 5
(a) (b) (c) (d)
6 3 3 6

131. The differential equation of the family of curves y = Ae3x + Be5x, where A, B are arbitrary constants is
d2 y dy d2 y dy
(a) +8 + 15y = 0 (b) 2 −8 + 5y = 0
dx 2
dx dx 2
dx
d2 y dy d2 y dy
(c) +5 + 6y = 0 (d) − 8 + 15y = 0
dx 2
dx dx 2
dx
−1
dy 1 e cos x
132. An integrating factor of + y= is
dx 1 − x2 1 − x2

(a) e 1− x 2
(b) e − cos
−1
x
(c) e
( )
sin −1 x 2
(d) e
− 1 − x2
3.92  Integral Calculus

 y y  y y  dy
133. Solution of the equation  x cos + y sin  y −  y sin − x cos  x =0
 x x  x x  dx
y y y
(a) xy cos = x +y +C (b) x cos =C (c) xy cos =C (d) xy cos(xy) = C
x x x

dy
134. The solution of = xy + x + y + 1 is
dx
x2 x2
(a) y = + x + C (b) log y = + 2x + C
2 2
x2
(c) 2 log y = x2 + x + C (d) log ( y + 1) = +x +C
2
135. The differential equation whose general solution is y2 + x = 1 + my where m is an arbitrary constant, is
dy d2 y dy d2 y dy dy
(a) y = x + 2 (b) + x + y =0 (c) x2 + x + y =0 (d) y = (x - y2 - 1)
dx dx 2
dx dx 2
dx dx
sin x dx
136. ∫ sin3x
is

1  3 − tan x  1  3 + tan x 
(a) log   +C (b) log   +C
2 3  3 + tan x  2 3  3 − tan x 
1  3 + tan x  1  3 + tan x 
(c) sin −1   +C (d) cos −1   +C
2 3  3 − tan x  2 3  3 − tan x 

 2 + sin2x 
137. ∫e
x
 1 + cos2x  dx is
(a) ex (tan x + sec x) + C (b) ex (tan x – sec x) + C (c) ex tan x + C (d) ex sec x + C
1.5

∫  x  dx , where [ ] denotes the greatest integer function, equals


2
138.
1

(a) 2 + 2 (b) 2 − 2 (c) 2.25 (d) 1.25


1 1

139 If l 1 = ∫ x m (1 − x ) dx and l 2 = ∫ x n (1 − x ) dx , where m and n are > 0, then


n m

0 0

m n
(a) l 1 = l 2 (b) l 1 = l (c) l 1 = l (d) l 1 = (m + n ) l 2
n 2 m 2
π
2

∫ log (tan x ) dx is
n
140.
0

π π π
(a) (b) (c) 0 (d) −
2 3 2
π
2
 1 
141. ∫  1 + tan
0
8  dx is
x
π π π
(a) log 2 (b) p (c) (d)
2 4 2
Integral Calculus  3.93

π
2
2 sin x + 3cos x
142. ∫
0
sin x + cos x
dx is equal to

π 5π
(a) (b)
4 2

(c) (d) 5p
4

143. The ratio is which the area enclosed by the curves y2 = 12x and x2 = 12y is divided by the line x = 3 is
15 15
(a) (b)
64 49
10 16
(c) (d)
49 49
144. Area enclosed by the curve x = –2 + 5 cos q, y = 1 + 4sin q is
(a) 20p (b) 80p (c) 10p (d) 40p
π
4

∫ tan
n
145. If In = θ dq, then Ι 8 + I6 is
0

1 1 1
(a) (b) 0 (c) (d)
14 7 2
π
2

∫x
n
146. If Ι n = sin x dx , n > 1, then the value of I8 + 56I6 is
0

π7 π6
(a) (b)
16 32
π5
(c) (d) p8
8
x2
t 2 − 5t + 4

147. The number of points at which ∫0 2 + e t dt has extremum is
(a) 1 (b) 2 (c) 5 (d) 0
148. The differential equation representing the family of curves y = ae + be + ce , where a, b, c are arbitrary parameters is
x 2x -3x

d2 y dy d3 y dy
(a) –3 + y = 0 (b) +7 – 6y = 0
dx 2
dx dx 3
dx
d3 y dy d2 y dy
(c) –7 + 6y = 0 (d) -7 + 6y = 0
dx 3
dx dx 2
dx
dx 2x
149. An integrating factor of the equation + = 10y 2 , is
dy y
2 2
(a) y2 (b) x2 (c) e y (d) e x
x2 + y2
150. The slope of a curve at any point (x, y) on it is and it passes through the point (2, 1). The equation of the
2xy
curve is
(a) 2 (x2 – y2) = 3y (b) 2 (y2 – x2) = 3y (c) 2 (x2 – y2) = 3x (d) 2 (y2 – x2) = 3x
3.94  Integral Calculus

Assertion–Reason Type Questions

Directions: Each question contains Statement-1 and Statement-2 and has the following choices (a), (b), (c) and (d), out
of which ONLY ONE is correct.
(a) Statement-1 is True, Statement-2 is True; Statement-2 is a correct explanation for Statement-1
(b) Statement-1 is True, Statement-2 is True; Statement-2 is NOT a correct explanation for Statement-1
(c) Statement-1 is True, Statement-2 is False
(d) Statement-1 is False, Statement-2 is True
151. Statement 1
e x ( x + 4 ) dx ex
∫ = +C
(x + 5) x +5
2

and
Statement 2
dx
∫ x + 5 = log e ( x + 5) + C
152. Statement 1
1
14.12.10.8.6.4.2
∫ sin
15
xdx =
−1
15.13.11.9.7.5.3

and
Statement 2
 (n − 1). (n − 3 ) ......2
π  if n is odd
2
 n (n − 2 )..............3
∫0
n
sin x dx = 
 (n − 1)(n − 3 )......1 π
 n(n − 2)...........2 × 2 if n is even

153. Statement 1
dy y y
General solution of the equation x = y − x tan   is xsin   = C .
dx x x

and
Statement 2
dy f (x, y)
If = , where f(x, y) and g(x, y) are homogeneous functions in x and y of the same degree, then the differential
dx g(x, y)
equation can be converted into a separable form by the substitution y = vx.
Integral Calculus  3.95

Linked Comprehension Type Questions

Directions: This section contains a paragraph. Based upon the paragraph, 3 multiple choice questions have to be
answered. Each question has 4 choices (a), (b), (c) and (d), out of which ONLY ONE is correct.
A and B are points on the curve y = f (x) (see figure). Let the x co ordinates of A y
and B be a and b. If L denotes the length of the arc AB of the curve, L is given by B y = f (x)

b 2
 dy 
L=∫ 1 +   dx A
a
 dx 
If the equation of the curve is given in the parametric form x = f (t), y = g
(t), and if x = a correspond to t = t1 and let x = b correspond to t = t2.
t2
2 2
L = ∫ f '(t) + g '(t) dt O M N x
t1

Also, the area of the surface generated when the arc AB revolves about the x axis is given by surface area
b 2
 dy 
S = 2 π ∫ y 1 +   dx
a
 dx 

154. The length of the arc of the parabola y2 = 4 x between x = 0 and x = 1 is

(a) 2 + log ( )
2 + 1 (b) 2 + log ( 2 +1 )
(c) 2 + 1 + log 2 (d) log ( 2 +1 )
4 23
155. The shape of a cable from an antenna tower is given by the equation y = x from x = 0 to x = 20. The total length
3
of the cable is
164 171 643 364
(a) units (b) units (c) units (d) units
3 3 3 3

156. The length of the loop of the curve 3 y2 = x (x - 1)2 is


4 3 8
(a) 8 3 (b) (c) (d)
3 4 3

Multiple Correct Objective Type Questions

Directions: Each question in this section has four suggested answers out of which ONE OR MORE answers will be correct.
157. y = y(x) satisfies the differential equation (x + 1)dy + [3 - y - (x + 1)2]dx = 0 subject to the condition y(0) = 0. Then,
(a) The solution curve is y = x2 - 2x
(b) The solution curve is y = x3 - 4x2 + x
(c) Area under the solution curve between x = -1 and x = 3 is 4
1
4
(d) If y = y(x) is the solution curve, ∫ xy(x)dx = -
−1
3
3.96  Integral Calculus

158. Let f(x) be continuous in R


2a a

Then, ∫ f (x)dx = 2 ∫ f (x)dx is true if


0 0

(a) f(x) is odd (b) f(x) is symmetric about x = a


(c) f(x) is periodic with period a (d) f(2a - x) = f(x)
159. Given f(a - x) = f(x) and f(2a - x) = -f(x). Also given that g(a - x) = -g(x) and g(2a - x) = g(x). Then
a a a a

(a) ∫ f (x)g(x)dx = ∫ f (x)dx ∫ g(x)dx


0 0 0
(b) ∫ f (x)g(x)dx = 0
0

2a 2a a

(c) ∫ f (x)g(x)dx = 0
a
(d) ∫ f (x)g(x)dx = 2 ∫ f (x)g(x)dx
a 0

Matrix-Match Type Question

Directions: Match the elements of Column I to elements of Column II. There can be single or multiple matches.

160. . represents the modulus function


[.] represents the greatest integer function and
{}. represents the fractional part function
Column I Column II
3
3e 4 − 7
∫ xe
|x| + |x −2|
(a) dx (p)
4
0
3
53e 2
∫ e [x]dx
x
(b) (q)
2
0
4

∫x
2
(c) − x − 6 e 2 dx (r) 2e3 - e2 - e
−3
2
{} (
5e 2 e 2 + 1 )
∫ xe .e   .e dx
|x| x  x
(d) (s)
4
−2
Integral Calculus  3.97

Ad d i t i o n a l P r a ct i c e E x e rci se

Subjective Questions
π
π
161. Show that ∫ cos2x log sin x dx = − .
0
2
x
dt π
162. Solve the equation: ∫
log 2 e −1t
=
6
.

4
 x2 + x + 1  7
163. Without evaluating the integral I = ∫  x
−3
2
− x + 1  dx show that 3 < I < 21.

164. If In = ∫ x n tan −1 x dx n being a positive integer, show that


0

π 1
(n + 1) In + (n - 1) In - 2 = − hence evaluate I6.
2 n
1

∫[p + (1 − p)x]
15
165. Evaluate dx where p is a parameter independent of x. Use this result to prove that
0
1
1
∫x (1 − x)15−m dx =
m
.
0 16 × 15 C m

166. Find the maximum value of f(x) if fI(x) = g(x), gI(x) = - f(x), f(0) = 3 and g(0) = 4.
167. Find the area of the loop of the curve y2 (2a - x) = x (x - a)2.

168. Find the area lying above the x axis and included between the curves x2 + y2 – 2 ax = 0 and y2 = ax.
169. Compute the total area of the loop of the curve a2 y2 = x2 (a2 – x2).
π
170. Find the area bounded by the curve y = sin-1 x and the lines x = 0, y = ± .
2
171. Find the area of the region bounded by the parabola y2 – x – 9 y + 21 = 0, the tangent at the vertex and the latus rectum.

172. Find the area bounded by the x and y axes and the curve x + y = 1 in I quadrant.

173. Find the area of the figure bounded by the curves y = log x and y = (log x)2

174. Solve the equations:
x dy
 y 
(i) 2
= 2 22
− 1  dx
x +y x +y 

 3x 4 
(
(ii) x 3 sin 3 y − y 2 cos x dx + 
 4
) cos y sin 2 y − 2 y sin x  dy = 0

dy
(iii) (sin y) = (cos x)(2 cos y − sin 2 x)
dx
3.98  Integral Calculus

( )
(iv) 2 x 2 − 3 y 2 − 7 x dx − (3 x 2 + 2 y 2 − 8)y dy = 0

dy y x
(v) + = .
dx (1 − x 2 ) 2 ( )
3 2
1 − x2

dy
175. Solve the initial value problem = 1 − x(y − x) − x 3 (y − x)2 , y(1) = 2 .
dx
y dy y2
176. Solve: − =
x dx x x
sin 4   + cos 4  
y y

177. If In ∫ x n e - x cos x dx
0

J n = ∫ x n e - x sin x dx
0

Show that for n > 1,


n
(i) In = (I n −1 − J n −1 )
2
n
(ii) Jn = (I n −1 + J n −1 )
2
1
(iii) In - n In-1 + n (n − 1) I n −2 = 0
2
π

 sin 3 θ 
4
178. (i) Find the value of ∫  sin θ + dθ .
0 cos 2 θ 
π
k 4
dx
(ii) Find the value of k if ∫
0 x + x+k
= ∫ sec 2 x sin xdx .
0

179. f is a continuous for all x and f2 (x) = ∫ f (t ).g (t ) dt then


0

(i) find the relation between f(x) and g(x)


sin x
(ii) if g(x) = find f(x)
2 + cos x
1

( )
180. If g(x) = 3x + ∫ xz 2 + x 2 z g ( z ) dz and g(0) = 0
0

(i) find g(x)


(ii) find the roots of g(x) = 0
π π
2
sin (2n + 1) x 2
sin (2n − 1) x π
181. (i) Show that ∫
0
sin x
dx = ∫
0
sin x
dx =
2
π
2
sin 2 nx π
(ii) Vn = ∫ dx then show that Vn +1 − Vn =
0 sin x
2
2
(iii) Find Vn
Integral Calculus  3.99

x
log t

182. For x > 0, f(x) = ∫ 1 + t dt
1

(h (x ))
2
1
(i) Find g(x), h(x) if g(x) = = f(x) + f   .
2 x
(ii) Find ∫ g(x)dx .
(iii) Write the domains of h(x) and g(x).
183. (i) F  ind all the values of a (>0) so that the area of the bounded region enclosed between the parabolas y = x - ax2
and ay = x2 is maximum.
(ii) Also find the maximum area.
x
(iii) Show further that y = bisects the maximum area.
2
184. (i) Find the transformed equations of the lines 2x + 2y - 2 = 0, 3x + y - 5 = 0 when the origin is shifted to (2, -1),
without charging the direction of axes.
dy 2x + 2y − 2
(ii) Solve =
dx 3x + y − 5
185. (i) z is a point in the Argand plane then show that locus of z given by z 2 + z 2 − 2zz + 8z + 8z = 0 represents a pa-
rabola.
(ii) Sketch the area of the region bounded by 5 ≤ | z | ≤ 2 3 and z 2 + z 2 − 2zz + 8z + 8z ≥ 0

sin 2 x cos 2 x

∫ ∫
−1
186. If f(x) = sin t dt + cos −1 t dt then find
0 0

(i) f(x)
(ii) f ’(x)

187. If A = ∫ e ax cos bxdx , B = ∫ e ax sin bxdx then


(i) show that the geometric mean of (A2 + B2), (a2 + b2) is eax
b B 

(ii) tan −1   + tan −1   = bx
a  A
π

(
188. If I(x) = ∫ log 1 − 2x cos θ + x 2 dθ then )
0

(i) Show that I(X) = I(-X)


(ii) Find I(X) + I(-X).
π
4
189. If Ι n = ∫ tan n xdx n being a positive integer then show that
0

1
(i) In + In-2 =
n −1
1 1
(ii) < Ιn <
2 (n + 1) 2 (n − 1)
(iii) Find I6
b
dx
190. Evaluate ∫ ( x − a )( b − x ) ( b > a )
a
3.100  Integral Calculus

Straight Objective Type Questions

Directions: This section contains multiple choice questions. Each question has 4 choices (a), (b), (c) and (d), out of
which ONLY ONE is correct.
π
3

∫ (p sin )
3
191. If p, q, r are constants, then the value of x + q cos 2 x + r dx depends on the value of
−π
3

(a) q and r only (b) p and q only (c) p only (d) all of p, q and r
π
2
192. ∫ sin 2x log tan x dx is
0

π π π
(a) (b) (c) (d) 0
4 2 3

193. The area of the region in the first quadrant enclosed by the x-axis, the line x = 3 y and the circle x2 + y2 = 4 is
π π π 2π
(a) (b) (c) (d)
4 3 2 3
194. The equation of the curve, which passes through origin and having slope of the tangent at any point (x, y) on it equal
x 4 + 2xy − 1
to is
1 + x2
(a) y = (x – 2 tan-1 x) (1 + x2) (b) y = (x + 2 tan-1 x) (1 + x2)
(c) y = (1 + x ) tan x
2 2 -1
(d) y = (1 + x2) tan-1x

dx
195. ∫ ( x − α ) (β − x ) , b > a, is
x −α x −β x −β x −α
(a) sin −1 +C (b) sin −1 +C (c) 2 sin −1 +C (d) 2 sin −1 +C
β−α β−α β−α β−α

1
196. ∫ (x + 5) x+4
dx is

(a) tan −1 x + 4 + C (b) 2 tan −1 x + 4 + C


(c) − tan −1 x + 4 + C (d) −2 tan −1 x + 4 + C

∫ x cos(x )sin (x )dx is


2 3 2
197.

sin 4 (x 2 ) sin 2 (x 2 )
(a) + C (b) +C
4 4
cos 2 (x 2 ) sin 4 (x 2 )
(c) + C (d) +C
4 8

∫[e
x
198. sec x + e x . log(sec x + tan x)]dx is
(a) -ex log(sec x + tan x) + C (b) ex log(sec x - tan x) + C
(c) ex sec x + C (d) -ex log(sec x - tan x) + C
Integral Calculus  3.101

1
199. ∫ sin x cos 2
x
dx is

x x
(a) log tan − sec x + C (b) sec x − log tan +C
2 2
x x
(c) log tan + sec x + C (d) log | sec x | + tan +C
2 2

x tan −1 x
200. ∫ 1 + x2
dx is

(a) tan-1 x. (
1 + x 2 + log x − x 2 − 1 + C ) (b)) -tan-1 x. (
1 + x 2 + log x − x 2 − 1 x + C )
(c) tan-1 x. 1 + x2 − log ( x + x2 + 1)+ C (d) tan-1 x. (
1 + x 2 + log x + x 2 + 1 + C )
201. Let f(x) is a 5th degree polynomial with f(0) = 2, f ’(0) = 2 and f ’’(x) = 80x3 + 72x2 + 6. Then
(a) f(x) = 4x5 + 3x4 + 3x2 - 2x + 2 (b) f(x) = 4x5 + 6x4 - 3x2 - 2x + 2
(c) f(x) = 4x + 3x + 3x + 2x - 2
5 4 2
(d) f(x) = 4x5 + 6x4 + 3x2 + 2x + 2
π
2
sin2 x
202. ∫0 sin x + cos x dx is
(a) 2 log ( 2 +1 ) (b) 2 log ( )
2 −1 (c)
1
2
log ( )
2 −1 (d)
1
2
log ( )
2 +1


t+
2

203. ∫
t +2π
sin −1 (cos x) + cos −1 (sin x) dx is

π2 π2 π2 π2
(a) (b) (c) (d)
2 4 8 16
π
4

∫ tan
4
204. xd(x − [x]) , where [ ] denotes greatest integer function, equals
0

π 2 π 2 2 π π 3
(a) − (b) + (c) − (d) +
4 3 4 3 3 4 4 2
x 1
 −1 
205. If ∫ f (t)dt = x + ∫ tf (t)dt then the quadratic equation whose roots are f   and f(2) is
0 x
2 

(a) 3x2 + 7x - 2 = 0 (b) 3x2 - 7x - 2 = 0 (c) 3x2 - 7x + 2 = 0 (d) 3x2 + 7x + 2 = 0


206. The area of the region bounded by y = 4x3, x-axis and the line x = 2 is
(a) 4 (b) 16 (c) 12 (d) 8

207. The differential equation representing the one parameter family of curves 1 + x 2 + 1 + y 2 = k x 1 + y 2 − y 1 + x 2 , ( )
is
dy 1 + x 2 dy 1 + y 2 dy dy 1 + y2
(a) = (b) = (c) = (1 + x 2 )(1 + y 2 ) (d) =
dx 1 + y 2 dx 1 + x 2 dx dx 1 + x2
3.102  Integral Calculus

208. The general solution of the equation (3x - 4y - 3)dy = (3x - 4y - 2)dx is
(a) (x - y)2 + C = log(3x - 4y + 1) (b) x - y + C = log(3x - 4y + 4)

(c) x - y + C = log(3x - 4y - 3) (d) x - y + C = log(3x - 4y + 1)

cos8x − cos7x
209. ∫ 1 + 2cos5x
dx is

sin 3x sin 2x sin 3x sin 2x sin 2x sin 3x cos3x cos2x


(a) − +C (b) + +C (c) − +C (d) − +C
3 2 3 2 2 3 3 2

( x +2 )
10

210. ∫
1
x − 1 + x − 2 x − 1 dx is

110 −98
(a) 110 (b) (c) 18 (d)
3 3
f(x)g ' (x) − f ' (x)g(x)
211. ∫ f(x).g(x)
[log g(x) − log f(x)]dx is

2 2 2
 g(x)   f (x)  1 1 f (x) 
(a) 2  log +C (b) 2  log  +C (c) [log f (x).g(x)]2 + C (d)  − log  +C
 f (x)   g(x)  2 2 g(x) 

212. ∫ 2x sin(x 2 ) − sin x  dx



x2 x x x2

(a) ∫ sin tdt (b) ∫ sin tdt (c) ∫ sin tdt (d) ∫ cos tdt
x x2 x x

sin −1 x − cos −1 x
213. ∫ sin −1
x + cos −1 x
dx is

2 2
(a) (2x − 1)sin −1 x + x 1 − x  − x + C (b) (2x − 1)sin −1 x − x 1 − x  − x + C
π π
2 2
(c) (2x − 1)sin −1 x + x 1 − x  + x + C (d) (2x − 1)sin −1 x − x 1 − x  + x + C
π π

x 2 n + x −2 n (
x.f (x). log x + 1 + x 2 )dx is
214. If f (x) = lim 2 n
n →∞ x − x −2 n
, (x > 1) then ∫ 1 + x2

(a) ( )
1 + x 2 log x + 1 + x 2 + x + C (b) ( )
1 + x 2 log x + 1 + x 2 − x + C

( )
(c) log x + 1 + x 2 + x 1 + x 2 + C ( )
(d) log x + 1 + x 2 − x − 1 + x 2 + C

1 1
215. P(x) is a non-zero polynomial such that P(0) = 0 and P(x3) = x4P(x), P’(1) = 7 and ∫ P(x)dx = 1.5 then ∫ P(x)P ' (x)dx is
0 0

(a) 6 (b) 8 (c) 7.5 (d) 8.5


2
1 1 k
216. If for a non-zero x, af (x) + bf   = − 5 a ≠ b; and ∫ f (x)dx = 2 then k is
x x 1 a − b2
7b 7b 7b 7b
(a) a log 2 − − 5a
(b) a log 2 + + 5a (c) a log 2 + − 5a
(d) a log 2 − + 5a
2 2 2 2
Integral Calculus  3.103


217. The area formed by the set of points {(x, y); |x| ≥ y ≥ x2} is equal to
1 1 1
(a) (b) (c) (d) 1
2 3 4
218. If [x] is the greatest integer less than or equal to x, then the area bounded by y = x -[x] and y = - x - [x], and the x-
axis between x = - 2 to x = 2
3 1
(a) 1 (b) 2 (c) (d)
2 2
dy π π 3 3
219. The solution of the equation cos 2 x − y tan 2x = cos 4 x given | x |< and y   =
dx 4 6 8
(a) 4y(1 - tan2 x) = 3 sin 2x (b) y(1 - tan2 x) = sin 2x
(c) 2y(1 - tan2 x) = sin 2x (d) y(1 - tan2 x) = 2sin 2x
y y
220. General solution of the equation x cos (ydx + xdy) = y sin (xdy − ydx)
x x
x
(a) xy = k sec xy (b) xy = k sec
y
y y
(c) xy = k sec (d) xy = k cos ec
x x
2 n
 t 
221. ∫ 1 + dt =
0
 n + 1 
1
(a) e - 1 (b) e2 - 1 (c) e −1 (d) −1
e2
π ∞
222. Let I(n) = ∫ sin nx dx (n an integer ≥ 0). Then, ∑ I (5 ) is
n=0
n

5 3 5
(a) (b) (c) 1 (d)
2 2 4

223. Let y = F(x) represent the solution of the differential equation


y(1 + xy) dx - x dy = 0, y(1) = 2. Then,
(a) F(x) is an odd function
(b) No part of the curve lies between x = − 2 and x = + 2
(c) x = ± 2 are asymptotes of the curve
(d) choices (a) and (c)
tan x cot x
4du du
224. ∫
1 1 + u2
+ ∫ u (1 + u )
1
2

e e
2
(a) 0 (b) (c) 2 tan x (d) 1
e
225. Let F(x, y) = 0 represent the solution of the differential equation
dy y ( x log y − y )
= , y (e) = 1. Then, the value of y when x = 1 is
dx x ( y log x − x )
1
(a) e (b) (c) 1 - e (d) (e - 1)
e
3.104  Integral Calculus

226. The differential equation of the family of curves represented by


y3 = cx + c3 + c2 - 1, where c is an arbitrary constant is of
(a) order 1, degree 1 (b) order 2, degree 3 (c) order 1, degree 3 (d) order 3, degree 2

x log x
227. ∫ (1 + x ) dx is
0
2

1
(a) 0 (b) 1 (c) (d) does not exist
2
1

228. If f(x + y) = ex f(y) + ey f(x) for all x, y ∈ R and f ’(0) = 1, then ∫ x 2 f(x)dx is
0
2
e −1
(a) log (1 + e) (b) (c) (6 - 2e) (d) (e - 1)
e2 + 1
1 1
dy
229. If = x + ∫ y ( x ) dx , y(0) = 1, then, ∫ xy(x)dx is
dx 0 0

13 19 7 101
(a) (b) (c) (d)
36 48 36 72
x
1
230. If g(x) =
x ∫ 3t − 2g ' (t ) dt, then g’(2) is
2
1 3 3
(a) (b) (c) 1 (d)
2 4 2
π
4
x2
231. ∫ ( xsin x + cos x )
0
2
dx is

4 −π 4+π π2 π2
(a)  (b) (c) (d)
 4 + π  4−π 4 8
πx
232. The area of the region bounded by the curves y = x2 - 1, y = cos between the lines x = 0 and x = 2 is
2
2 4 2 4 4
(a) +2 (b) + (c) +2 (d) +1
π π 3 π π
233. Let F(x, y) = 0 represent the solution of the differential equation
dy
= x + y + y2 + xy + xy2 + 1, y(0) = 0. Then, the solution is given by
dx
2  2y + 1  x2 π 1  2y + 1  π
(a) tan-1   = +x+ (b) tan-1   = x2 + 2x +
3  3  2 3 3 3  3  6 3
2  (2y + 1)  π 1  y +1 π
(c) tan-1   =x+ (d) tan −1   = x +
3  3  3 3 3  3 6 3
λ 5−λ

234. Let f(x) = 2x3 - 15x2 + 24x and H(l) = ∫ f (t ) dt + ∫ f (t ) dt where, 0 < l < 5 the interval in which H(l) increasing, is
0 0

5   5
(a) (0, 5) (b)  , 5  (c)  0,  (d) (1, 5)
2   2
Integral Calculus  3.105

lim
1 n −1
 k
235. n → ∞
n
∑ log 1 − n  equals
k =0
1
(a) e (b) -1 (c) (d) 0
e
x

∫ sin u du
0
236. lim equals
x →0 sin x 2
1
(a) (b) 1 (c) 2 (d) 3
2
237. Area bounded by the curve x2 = 4y and the line x = 4y - 2 is
3 9 9 9
(a) (b) (c) (d)
4 2 4 8
238. Area of the region bounded by the x - axis and the curves defined by
π π π 3π
y = tanx, − ≤ x ≤ , p = cot x, ≤x≤ , is
3 3 6 2
1 1
(a) log 3 (b) log 2 (c) log 2 (d) 2 log 2
2 2
e2
f (x ) 1
239. Suppose f(x) is twice differentiable in (1, e2) and f ”(x) exists in (1, e2). Also, f ’(e2) = f(e2) = f(1) = 2 and ∫
1 x2
dx =
4
.
e2

Then, the value of ∫ f " ( x ) log x dx is


1

23 2 5 1 23 2 5 2 1
(a) − (b) + (c) + (d) − +
4 e2 4 e2 4 e2 2 e2 e

dx
240. ∫ (1 + x )(1 + x )
0
9 2
is

π π π π
(a) (b) (c) (d)
16 8 4 2
e
1 1
241. Let f defined for all x ≠ 0 such that 2f(x) + f(-x) =
x
sin (x - ). Then,
x ∫ f ( x ) dx
1
is
e

(a) 2e + 1 (b) e + 1 (c) 2e - 1 (d) 0



x 2 dx
242. ∫ (x
0
2
)(
+ p2 x 2 + q 2 x 2 + r 2 )( )
where p, q, r are distinct, equals

π π
(a) (b)
2 (q − r ) x − q ( 2 2
)(x 2
+r 2
) 2 (q + r )(r + p )( p + q )
π 2 π pqr
(c) (d)
(q 2
+r 2
)(r 2
+p 2
)(p 2
+q ) (p + q )(q + r )(r + p )
2 e4

243. If ∫ e dt = λ , then, ∫
t2
log t dt equals
1 e

(a) e2 - l - 2 (b) e4 - e2 + 1 - l (c) 2e4 + e - l (d) (2e4 - e - l)


3.106  Integral Calculus

π
4

∫ tan
n
244. Let In = x dx where n is a positive integer > 1. Then, lim (2n+ 1) (In + In-2) equals
n →∞
0
1 π
(a) 1 (b) 2 (c) (d)
2 2
λ

245. The value of ∫  x  F ' ( x ) dx where l > 1 and [ ] denotes the greatest integer function, is equal to
1

 λ  −1  λ 
(a) [l] F (l) - ∑ F (i )
i =1
(b) [l] F (l) - ∑ F (i )
i =1

 λ  −1  λ 
(c) ∑ F (i ) +  λ  F ( λ )
i =1
(d) ∑ F (i )
i =1

246. For 0 ≤ x ≤ p, the area bounded by the line y = x and the curve y = x + sin x is
(a) 1 (b) 2 (c) p (d) 2p
247. The area bounded by the curves x + 2|y| = 1 and x = 0 is
1 1 1
(a) (b) (c) (d) 1
4 3 2
248. The solution of the initial value problem (x2 + y2) dy = xy dx, y(1) = 1 is y = y(x).
If y (x0) = e, then x0 equals
e
(a) 2e + 1 (b) (c) e2 - 1 (d) e 3
3
( x + 1) + y − 3 .
2

249. A curve passes through (2, 0) and the slope of the tangent at P(x, y) on the curve is given by Then,
( x + 1)
the area bounded by the curve and the x-axis in the 4th quadrant is
2 4 5
(a) (b) (c) 1 (d)
3 3 3
250. Area of a loop of the curve represented by x = a(1 - t), y = at (1 - t2), -1 ≤ t ≤ 1 and a > 0, is
8a 2 16a 2 4a 2 8a
(a) (b) (c) (d)
15 15 15 15

 y  y π
251. If the differential equation y  cos  (x dy - y dx) + x  sin x  (x dy + ydx) = 0 with y (1) = 2 has the solution in
 x
π y
the form = sin , then k equals
k xy x
1
(a) 2 (b) 1 (c) 3 (d)
2
1
252. The area enclosed between the curves y = log   , y = loge (x + e) and the x-axis, is
y
1
(a) e - 2
e
(b) + 1 (c) e + 1 (d) 2
e
π
2  1 + sin 3x 
253. ∫  1 + 2 sin x  dx is
0

π π 1
(a) (b) (c) 1 (d)
2 8 2
Integral Calculus  3.107

2x − 1, − 3 ≤ x < 1 4x + 7, − 5 ≤ x < 0 2


254. Let f(x) =  2
 3x − 2, x ≥ 1
and g(x) =  2
 5x − x + 7, x ≥ 0
The value of ∫ (g o f )( x ) dx
−2
equals

101 1991 1991


(a) 0 (b) (c) (d)
12 6 12
2 2 1
255. Let f(x) be differentiable in R and f(x) = 2 + x2 ∫ f (t ) dt + ∫ t f (t ) dt . Then ∫ f (x ) dx is
0 0 0

6 3 14 −6
(a) (b) (c) (d)
19 19 19 19
π
256. The function y = f(x) satisfying the differential equation (sin2x)y’ - (y - 1) = 0 such that f   = 2 is
2 
(a) continuous everywhere (b) continuous everywhere except at x = np, n an integer
π
(c) discontinuous at x = (2n + 1) , n an integer (d) continuous nowhere
2

1  1 + f (x)  f (1 + x )
257. If ∫x log ex
e log 2
e x
e dx = log   + C then lim
 2 + f (x )  x → 0 x
equals

1
(a) 0 (b) 1 (c) e (d)
e
π
2
258. The value of ∫ sin x 2 − 2x + 1 dx equals
0

(a) cos 1 - sin 1 (b) 2 - cos 1 - sin 1 (c) 2(1 - cos 1) (d) 1 - sin 1
1
x
259. The smallest interval in which the value of 26 ∫ 5
lies, is
0 x + 25
 1  1 
(a) 0,  (b)  ,26  (c) [0, 26] (d) (0, 1)
 26   26 

260. The area bounded by the curves y = 3x2 + 2, y = mx, (m being negative), x = 0 and x = 1 is bisected by the x –axis.
The value of m is
(a) 6 (b) - 2 (c) -6 (d) -3
 x2 8 
261. Area enclosed by the curve f(x) = min  , 2 , the x-axis and the ordinates x = ±3, equals
 4 x + 4 
4 3 4 3 4 3 4 3
(a) + 8 tan −1 − 2 π (b) − 8 tan −1 − 2 π (c) + 8 tan −1 - p (d) + 8 tan −1 + 2p
3 2 3 2 3 2 3 2
262. Equation of the curve y = f(x), satisfying xy5 = y4 (suffixes denoting differentiation) and which is symmetric with
respect to y-axis, is
(a) y = C1x5 + C2 (b) y = C1x2 + C2 (c) y = C1x4 + C2 (d) y = C1x4 + C2x2 + C3

dx
263. ∫ =
(x − 1) − x 2 + 3x − 2

x −2 x −2 1 −x x −1
(a) −2 + C (b) −2 + C (c) 2 + C (d) +C
1−x x −1 x −2 x −2
3.108  Integral Calculus

264. The area bounded by the curve y = x + sinx and its inverse function between x = 0 and x = 4p is
(a) 4 (b) 8 (c) 16 (d) 20

 x x sin t 
x → 5 (x − 5) ∫ t
265. lim  dt  is
 5 
sin 5 cos5
(a) 0 (b) (c) (d) sin 5
5 5
dx
266. ∫ (x + 2) x +3
is

 x + 3 +1
(a) log ( x + 3 − 1) +C (b) log  +C
 x + 3 −1
 x + 3 −1
(c) log ( x + 3 + 1) +C (d) log  +C
 x + 3 +1

dy π
267. If + 2y tan x = sin x and y   = 0, then the maximum value of y is equal to
dx 3
1 1 1
(a) (b) (c) (d) 2
8 4 2

Assertion–Reason Type Questions

Directions: Each question contains Statement-1 and Statement-2 and has the following choices (a), (b), (c) and (d), out
of which ONLY ONE is correct.
(a) Statement-1 is True, Statement-2 is True; Statement-2 is a correct explanation for Statement-1
(b) Statement-1 is True, Statement-2 is True; Statement-2 is NOT a correct explanation for Statement-1
(c) Statement-1 is True, Statement-2 is False
(d) Statement-1 is False, Statement-2 is True
268. Statement 1
π

∫ cos xdx = 0
0

and
Statement 2
b

Let f(x) be continuous in [a, b] and f(a) ≠ 0, f(b) ≠ 0. If f(x) changes sign in [a, b], then ∫ f (x)dx = 0
a

269. Statement 1
The differential equation (1 + y2)dx = (tan-1y - x)dy is a linear differential equation where x is considered as the de-
pendent variable and y as independent variable.
and
Statement 2
dy
a0(x) + a1(x)y = a2(x) is a linear differential equation.
dx
Integral Calculus  3.109

270. Let f(x) be a function of x defined for all x ∈R and f(x) satisfies the condition f(x) = -f(1 - x) for all x.
Statement 1
3/2
∫ f(x)dx = 0
-1/2

and
Statement 2
1
curve y = f(x) is symmetric about the point ( , 0).
2

n
271. Let m, n be positive integers where m ≠ .
2
Statement 1
p
∫ sin
2
mx cos nx dx = 0
0

and
Statement 2

∫ sin mx cos nx dx = 0 for all m, n positive integers


0

272. Statement 1
Differential equation of the family of parabolas with their axis along the x-axis is of order 2.
and
Statement 2
The differential equation of a two parameter family of curves is of order 2.

273. Statement 1
p /2
sin x
∫x
0
7
+1
dx < 1

and
Statement 2
p /2
∫ sin x dx = 1
0

274. Statement 1
d2 y
If 2
= 3x 2 with y(0) = 1 and y’(0) = 2, then, y(2) = 8
dx

and
Statement 2
d  dy  d 2 y
=
dx  dx  dx 2
3.110  Integral Calculus

275. Statement 1
3p
2
∫ [sin x]dx = -p
p
where, [ ] denotes the greatest integer function.
2

and
Statement 2
b c b
∫ f(x)dx = ∫ f(x)dx + ∫ f(x)dx where a < c < b
a a c

276. Statement 1
1

∫e
x2
dx lies between 1 and e
0

and
Statement 2
For x ∈ (0, 1), 0 < x2 < 1

277. Statement 1

dx x
To evaluate ∫ 5 − 2 cos x
0
we cannot make use of the substitution tan
2
= t.

and
Statement 2
x
tan has a discontinuity at x = p.
2

Linked Comprehension Type Questions

Directions: This section contains paragraphs. Based upon the paragraphs, multiple choice questions have to be answered.
Each question has 4 choices (a), (b), (c) and (d), out of which ONLY ONE is correct.
Passage I

Y = f (x)
y

y = f (x)

x
A O A

O X=a X=b x    


Integral Calculus  3.111

Let A be the area of the region bounded by the curve y = f (x), x axis and the ordinates at x = a and x = b. [the figure above]
b b

If the area A is revolved about the x axis, the volume v of the solid thus generated is given by V = π ∫ y 2 dx = π ∫ (f (x))2 dx .
a a

x
−r O r

For example, suppose the semi circular portion of the circle x2 + y2 = r2 above the x axis is revolved about the x axis, we
get a sphere. Volume of the sphere = volume of the solid generated by revolving the area of the portion of the circle x2 + y2
a

= r2 above the x axis about the x axis = π ∫ y 2 dx


−a

a a
4 πr 3
( ) (
= π ∫ r 2 − x 2 dx = 2 π ∫ r 2 − x 2 dx = ) 3
−a 0

278. The volume of the solid generated by revolving the area included between the parabola y2 = 8 x, the x axis and its latus
rectum about the x axis is
(a) 32 p (b) 16 p (c) 8 p (d) 24 p
279. The figure bounded by the area of the curve y = sin x and the x axis between x = 0 and x = p is revolved about the x
axis. The volume of the solid of revolution is
π2 π2
(a) 2 p2 (b) p2 (c) (d)
2 8
280. The area of the portion of the curve y = x2 between x = 0 and x = 2 is revolved about the x axis. The volume of the
solid thus generated is
32 π 16 π 8π 4π
(a) (b) (c) (d)
5 5 5 5

281. The volume of the solid generated by revolving one arch of the cycloid x = a (q - sin q), y = a(1 – cos q) about the x
axis is
3 π2 a 3 π2 a 3 3 π2 a 3
(a) (b) (c) (d) 5 p2 a3
16 8 16

282. The volume of the spherical cap of a sphere whose height is h and whose base radius is c is
πh πh πh πh
(a) (3c 2 + h 2 ) (b) (3c 2 + h 2 ) (c) (3c + h) (d) (3c 2 + h 2 )
2 4 2 6

Passage II
Iterated Integrals
Multiple integrals over the rectangular region R:{x1 ≤ x ≤ x2; y1 ≤ y ≤ y2}, where x1, x2, y1, y2 are constants may be evaluated
by two successive integrations
x2 y y x
2 2

∫∫ f ( x, y ) dx dy = ∫
2

R x1
∫ f (x, y)dy dx = ∫ ∫ f (x, y)dx dy
y x1
y 1
1
3.112  Integral Calculus

where integration is carried out from the inner to outer rectangle. We shall follow the convention that limits are to be as-
signed in the order in which dx dy is given in the integral.
When x1 and x2 are functions of y and y1 and y2 are constants, f(x, y) is first integrated w.r.t.x between x1 and x2 keeping
y as fixed and the resultant expression is integrated w.r.t y between the limts y1 and y2
y2
 x2 
∫∫R f (x, y)dx dy = ∫y  x∫ f (x, y)dx  dy
1 1 
Similar procedure is followed when y1 and y2 are functions of x and x1 and x2 are constants.
The same procedure is followed in the case of iterated integrals involving 3 variables.
1 2

∫ ∫ (x
2
283. + y 2 )dx dy is
0 0

2 10
(a) (b) 3 (c) (d) Zero
3 3
π
2 π
284. ∫ ∫ cos(x + y)dx dy is
0 π
2

1
(a) 0 (b) 2 (c) -2 (d)
2
π a(1+cos θ )

285. ∫
0

0
r dr dθ is

3 2
(a) 3pa2 (b) πa (c) 16 p a2 (d) 0
4

286. ∫∫
x + y ≤1
xy dx dy is

1 1
(a) (b) (c) 4 (d) 8
24 12

1 x

∫ ∫ (x )
2
287. + y 2 dy dx is
0 x

1 3 2 −3
(a) (b) (c) (d)
4 35 35 35

288. ∫∫ xy dx dy over the first quadrant of the circle x 2


+ y2 = a2 is

a4 a4 a2 a2
(a) (b) (c) (d)
8 4 8 4

3 2 1

289. ∫ ∫ ∫ ( x + y + z ) dx dy dz
0 0 0
is

(a) 6 (b) 3 (c) 9 (d) 18


Integral Calculus  3.113

Multiple Correct Objective Type Questions

Directions: Each question in this section has four suggested answers of which ONE OR MORE answers will be correct.

1 1− 2
290. If f ’(x) = and f(0) = then f(1) equals
2
x +1 + x 2
1 −1
(a) log 2 +1 (b) log 2 −1
2 2
(c) log 2 − 1 (d) -log 2 +1

291. If I = ∫ cos ec 2 x sec4x dx = K tan3x + L tan x + M cot x + C then


1
(a) K = (b) M = -1 (c) L = 2 (d) M = -2
3

292. ∫e
x
 log (sec x + tan x ) + sec x  dx equals
 
(a) ex log (sec x + tan x) + c (b) -ex log (sec x - tan x) +c
(c) eX log sec x + c (d) ex log tan x + c

x dx 2
293. If ∫ 1− x 3
=
3
fog(x) + c then

(a) f(x) = x (b) g(x) = cos-1 x (c) f(x) = sin-1x (d) g(x) = x x
x

294. If f(x) = ∫
1
2 − t 2 dt, then the real roots of the equation x2 - f ’(x) = 0 are

1
(a) 1 (b) 2 (c) -1 (d)
2
log 4
39
295. If the function f(x) = Ae + Be + Cx satisfies the condition f(0) = -1, f ’(log 2) = 31 and
2x x
∫ (f (x) − cx ) dx = 2 , then
0

(a) A = 5 (b) B = -6 (c) C = 3 (d) B = 6


1 1 2 2

296. If I1 = ∫ 2 x dx , I2 = ∫ 2 x dx , I3 = ∫ 2 x dx , and I4 = ∫ 2 x dx , then


3 4 3 4

0 0 1 1

(a) I2 > I1 (b) I3 > I4 (c) I4 > I3 (d) I1 > I2


π π
2
sin x dx 2
cos x dx
297. If A = ∫ sin x + cos x
0
and B = ∫ sin x + cos x , then
0

π
(a) A + B = 0 (b) A + B =
2
π
(c) A - B = p (d) A = B =
4
3.114  Integral Calculus

Matrix-Match Type Questions

Directions: Match the elements of Column I to elements of Column II. There can be single or multiple matches.
1 1 1
298. Given a = 1 + + 2 + 3 + ......... ∞ ,
α α α
b 1 1 1 1 1  π 3π 
= 1 + + 2 + 3 + ..... ∞ where and are roots of 6x2 - 5x + 1 = 0 and satisfy cosq = a-b, q ∈  , 
a β β β α β 2 2 
Column I Column II
b
πdx
(a) ∫ ( x + α )( x +β)
a
(p) p

b
x −α 3π
(b) ∫
a
β− x
dx (q) 1 +
4
β  x −α β −x   25 
(c) ∫ 
α
b−x
+  dx
x −a 
(r) plog  
 24 

β

∫( ) dx
2 π
(d) x −a + b − x (s)
2
α
299.
Column I Column II
2x
(a) If ∫ dx = k sin −1 2 x + C , then k equals (p) log2( )
1 − 4x
dx 1 − x 3 −1 1
(b) If ∫x 1− x 3
= k log
3
1 − x +1
+ C , then k equals (q)
2

dx 1
(c) If ∫ 1 + tan x = k (x + log(sinx + cosx)) + C, then k equals (r)
3

(2x 3
+ 3 dx )
 x +1   −1  x   1
(d) ∫ (x 2
−1 x + 4
= k log 
)(  x − 1 
2
+k
)  tan  2   then k equals (s)
log 2

300.
Column I Column II
2n

x sin x π2
(a) For n > 0, ∫ sin 2n
x + cos x 2n
dx = (p)
16
0

π
2
x sin x cos x π2
(b) ∫ sin 4
x + cos 4 x
dx = (q)
4

π
x sin x π2
(c) ∫ 1 + cos 2
x
dx = (r)
2
0

x

∫ (tan x )
2
−1

(d) lim 0
dx = (s) p2
x →∞ 2
x +1

Integral Calculus  3.115

So l u t i o n s
a nsw e r K e ys
Topic Grip 5. (i) x2 + y2 +2fy + c =0 IIT Assignment Exercise
3
(ii) x y2 + y − y 1 = 0 51. (d) 52. (d) 53. (b)
−x2 1
1. (i)
b(a + bx) −7 54. (b) 55. (a) 56. (a)
8. 57. (d) 58. (c) 59. (a)
2  a  2p
+ 60. (d) 61. (a) 62. (c)
b2  x − b log(a + bx) + C 10. sin 5
  63. (b) 64. (a) 65. (b)
12. 14 + 41 loge 3 66. (c) 67. (a) 68. (a)
 1 69. (a) 70. (b) 71. (d)
(ii) 2tan −1  x + 1 +  + C 23
 x 13. (ii) 72. (b) 73. (b) 74. (a)
6 75. (b) 76. (c) 77. (c)
 1
x− (iii) 1 78. (d) 79. (d) 80. (c)
1  x +C
(iii) tan −1   81. (d) 82. (a) 83. (a)
2  2  1 x2 + 1 − 1
  14. (i) 1 + x 2 + log 84. (b) 85. (b) 86. (c)
2 x2 + 1 + 1 87. (d) 88. (a) 89. (d)
1  x+2 (ii) 1+ x 2 + 90. (c) 91. (d) 92. (c)
(iv)  93. (a) 94. (c) 95. (a)
81  5 − 4x − x 2
1  1+ x 2 −1  96. (c) 97. (c) 98. (d)
log 
 
3
1 x+2 2  2  99. (b) 100. (d) 101. (c)
+  +C  1+ x +1 
3  5 − 4x − x 2   102. (c) 103. (a) 104. (d)
 1 + y2 = c 105. (a) 106. (b) 107. (a)
1 108. (c) 109. (d) 110. (c)
(tan )
−1
−1
(v) sin −1 (sin x − cos x) 15. (i) e tan x
x −1 + c 111. (c) 112. (b) 113. (d)
2
−1 114. (b) 115. (b) 116. (a)
1 (ii) ye tan x =
− log [sin x + cos x + 117. (c) 118. (d) 119. (c)
(tan−1 x −1)+ c
−1
2 e tan x
120. (b) 121. (c) 122. (b)
123. (d) 124. (b) 125. (b)
sin2x  + C 16. (b) 17. (c) 18. (c)
126. (b) 127. (a) 128. (a)
19. (a) 20. (b) 21. (d)
 t3 t2  129. (d) 130. (c) 131. (d)
(vi) 6  + + t + log(t − 1) + c 22. (b) 23. (a) 24. (a)
132. (b) 133. (c) 134. (d)
3 2  25. (c) 26. (a) 27. (b)
135. (d) 136. (b) 137. (c)
28. (d) 29. (d) 30. (c)
1 138. (b) 139. (a) 140. (c)
where t = (1 + x) 6 31. (a) 32. (d) 33. (d)
141. (c) 142. (c) 143. (b)
34. (b) 35. (a) 36. (c)
−x 144. (a) 145. (c) 146. (a)
(vii) 37. (a) 38. (d) 39. (d)
147. (c) 148. (c) 149. (a)
(xsin x + cos x)cos x 40. (a) 41. (b) 42. (d)
150. (c) 151. (a) 152. (d)
+ tan x + C 43. (c) 44. (b) 45. (c)
153. (a) 154. (b) 155. (d)
x +1 46. (a)
156. (b)
3. (i) f(x) = 47. (c), (d)
x −1 157. (a), (c), (d)
48. (a), (d)
158. (b), (c), (d)
(ii) g(x) = x + 2 log |x - 1| 49. (a), (d)
159. (b), (c)
(
4. (i) f(x) = x 4 x 2 + a 2 + b2 + c2 ) 50. (a) → (r)
(b) → (p)
160. (a) → (s)
(b) → (r)
(ii) sphere of radius 1 unit, centre (c) → (s)
(c) → (q)
at origin. (d) → (q)
(d) → (p)
3.116  Integral Calculus

Additional Practice Exercise 215. (b) 216. (c) 217. (b)


178. (i) 2 −1 218. (a) 219. (c) 220. (c)
162. log 4 9 221. (b) 222. (a) 223. (d)
(ii)
π 5 1  16 224. (d) 225. (b) 226. (c)
164.  − + log 2 227. (a) 228. (c) 229. (d)
 28 84 14  1
179. (i) f ( x ) = g ( x ) dx
2∫
230. (d) 231. (a) 232. (c)
166. 5 233. (a) 234. (c) 235. (b)
167.
a2
(4 − p)
(
(ii) f(x) = - log c. 2 + cos x ) 236. (a) 237. (d) 238. (c)
2 239. (a) 240. (c) 241. (d)
1
168.
(3p − 8)a 2 180. (i) g (x) =
119
(
240x 2 + 540x ) 242.
245.
(b) 243.
(b) 246.
(d)
(b)
244.
247.
(b)
(c)
12 −9 248. (d) 249. (b) 250. (a)
(ii) x = 0, x = 251. (a) 252. (d) 253. (c)
4a 2 4
169. 254. (d) 255. (a) 256. (b)
3 np
181. (iii) Vn = 257. (b) 258. (b) 259. (d)
170. 2 2 260. (c) 261. (a) 262. (b)
1 (log x )
2
263. (a) 264. (c) 265. (d)
171. 182. (i) g (x) =
6 ,
2 266. (d) 267. (a) 268. (c)
1 h (x) = log (x) 269. (a) 270. (a) 271. (b)
172.
6 272. (a) 273. (d) 274. (d)
x
(ii) (log x ) − x (log x − 1) + c
2
275. (d) 276. (a) 277. (a)
173. (3 - e) 2 278. (b) 279. (c) 280. (a)
y (iii) domain of h (x) : x ∈ (0, ∞) 281. (d) 282. (d) 283. (c)
174. (i) x + tan −1   = C Domain of g (x): (0, ∞).
x 284. (c) 285. (b) 286. (a)
183. (i) a = 1 287. (b) 288. (a) 289. (d)
x 4 sin3 y 1 290. (a), (b)
(ii) − y 2 sin x = C (ii)
4 24 291. (a), (b), (c)
(iii) cos y = −Ce −2 sin x 184. (i) 2X + 2Y = 0, 3X + Y =0 292. (a), (b)
(ii) (y - x + 3)4 = c (2x + y - 3) 293. (c), (d)
sin2 x 1 1 294. (a), (c)
+ − sin x + p
2 2 4 186. (i) f(x) =   295. (a), (c), (d)
4
(iv) x4 - y4 + 8y2 - 7x2 296. (c), (d)
(ii) f ’(x) = 0 297. (b), (d)
- 3x2y2 = C’
188. I(x) + I(-x) = I(x2) 298. (a) → (r)
−x
2 x 13 p (b) → (s)
(v) y = Ce 1− x
+ −1 189 I6 = −
1 − x2 15 4 (c) → (p)
x2 190. π (d) → (q)
175. e 2
= (y − x) 191. (a) 192. (d) 193. (b) 299. (a) → (s)
194. (a) 195. (d) 196. (b) (b) → (r)
 
2
3  x2 197. (d) 198. (d) 199. (c) (c) → (q)
 1 +  e − x 2 − 2
200. (c) 201. (d) 202. (d) (d) → (q)
  e 
203. (b) 204. (a) 205. (c) 300. (a) → (s)
x  4x  206. (b) 207. (b) 208. (d) (b) → (p)
176. 12   + sin   = 8x 2 + C (c) → (q)
y  y  209. (a) 210. (b) 211. (d)
212. (a) 213. (a) 214. (b) (d) → (q)
Integral Calculus  3.117

HINTS AND EXPLANATION S

Topic Grip dx
(iv) ∫ 5

x2  −1  (5 − 4x − x 2 ) 2
1. (i) ∫ dx = ∫ x 2 d 
(a + bx)2  b(a + bx)  5 - 4x - x2 = -(x2 + 4x - 5)
−x2 2xdx = -[(x + 2)2 - 9] = 9 - (x + 2)2
= +∫
b(a + bx) b(a + bx) dx
∫ =∫ 5
−x2 2 a + bx − a [9 − (x + 2)2 ]2
b(a + bx) b2 ∫ a + bx
= + dx
Put x + 2 = 3 sin q
−x 2
2  a  3cos qdq 1
∫ = ∫ 35 cos5 q = 81 ∫ sec qdq
4
= + x − log(a + bx) + C
b(a + bx) b2  b 
(ii) We write the integral as Let I4 = ∫ sec 4 qdq
(x 2 − 1) dx
I= ∫ (x + 1) 2

3 2
(
I4 = ∫ sec2 q 1 + tan2 q dq )
x +x +x
(x 2 − 1) dx = ∫ sec2 q dq + ∫ tan2 q d (tan q)
=∫ 2 ⋅
x + 2x + 1 x + x 2 + x
3
tan3 q
= tan q +
 1 3
1 − 2  dx
x 3
=∫ ⋅ x+2 1 x+2 
1 1 = +  
x+2+ x +1+ 5 − 4x − x 2 3  5 − 4x − x 2 
x x
1   
3
1  1 x+2 1 x+2
Putting x + 1 + = t 2 , 1 − 2  dx = 2tdt \ I =  + +C
x  x  81  5 − 4x − x 2 3  5 − 4x − x 2  
 
2tdt dt
⇒ I = ∫ (t 2
+ 1)t
= 2∫ 2
t +1
= 2 tan-1 t
(v) ∫ tan xdx


 1
= 2tan −1  x + 1 +  + C Let I = ∫ tan xdx
 x

1  1 =
 (
1  ∫ tan x + cot x dx 
 )
x2 + 1
(iii) ∫ 4 dx = ∫
1+ 2
x
1 + 2  dx
x 2 +
 ∫ (
tan x − cot x dx 
 )
x +1 1
dx = ∫ 1
2
∫( )
2
x + 2
x  x − x  + 2 Now, I = tan x + cot x dx

1 (sin x + cos x) 2d(sin x − cos x)


Putting x −
x
= t, = ∫ sin x cos x
dx = ∫ sin2x
dt 1  t  d(sin x − cos x)
∫ =∫ t 2
+2
=
2
tan −1 
 2 
+C = 2∫
1 − (sin x − cos x)2
 1 dt

x− = 2∫ where t = sin x - cos x
=
1
tan −1  x+C 1 − t2

2  2 
  = 2 sin −1 t = 2 sin −1 (sin x − cos x)
3.118  Integral Calculus

I2 = ∫( tan x − cot x dx )
 x  
= ∫ d
−1 
 cos x   (xsin x + cos x) 
sin x − cos x
= ∫ sin x cos x
dx
=
−x
+∫
1
(xsin x + cos x)cos x (xsin x + cos x)
 1 p  4a 2
= 4a 2  −  = (4 − p)
2 8  8  (cos x) + xsin x 
×   dx
(sin x + cos x)  cos2 x 
= − 2 ∫
(1 + sin2x) − 1 −x
(xsin x + cos x)cos x ∫
= + sec2 xdx
d(sin x + cos x)
= − 2 ∫
(sin x + cos x)2 − 1 −x
= + tan x + C
dt (xsin x + cos x)cos x
= − 2 ∫ where t = sin x+ cos x
2
t −1 2. (i) sinx+sin3x+sin5x+….sin21x
= − 2 log t + t − 1 ( 2
) =
1
2sin2 x + cos2x − cos 4x + cos 4x
2sin x 
(
= − 2 log sin x + cos x + sin2x ) − cos6x + …. + cos20x - cos22x]
1
⇒ I = (I1 + I2 ) 1 − cos22x 2sin2 11x sin2 11x
2 = = =
2sin x 2sin x sin x
1
= sin −1 (sin x − cos x) sin2 11x  cos3x cos5x
2 (ii) ∫ sin x dx = − cos x + 3 + 5 +

1
2
(
log sin x + cos x + sin2x + C ) cos21x 
...... + C
dx 21 
(vi) ∫ 1 1
(1 + x) 2 − (1 + x) 3  1  2 (1 − 2x ) 2 2
3. (i) f(x) + f  = = − —(1)
Put (1 + x) = t6 ( 6 is the L.C.M of 2 and 3)  1 − x  x (1 − x ) x 1− x
dx 6t 5 dt 1

I= ∫ 1 1
= ∫t
3
− t2 Replace x by
1− x
(1 + x) − (1 + x)
2 3

t3 1  
= 6 ∫ dt = 6 ∫ (t 2 + t + 1) + dt  1   1 
(t − 1) (t − 1) f + f
 1 − x  1 
1 − 
 t3 t2   1− x
= 6  + + t + log(t − 1) + C
 3 2  2
= 2 (1 − x ) −
1 1
1−
where t = (1 + x) 6 1− x
x 2
 1  1 − x  2 (1 − x )
(vii) ∫ dx f  + f  = 2 (1 − x ) −
(xsin x + cos x)2 1 − x   −x  −x
d  1   1 2 (1 − x )
(xsin x + cos x) = xcos x+ sin x - sin x f + f 1 −  = 2 (1 − x ) +
dx  1 − x   x  x

= x cos x  — (2)
x 2 cos xdx 1
∫ = ∫ (cos x)(xsin x + cos x)2 Put x = 1 − in (1)
x
Integral Calculus  3.119

 1 2 5. Let two circles in above system be x2 +y2 + 2f1 y + c1 =


f 1 −  + f ( x ) = − 2x 0 and x2 +y2 +2 f2 y + c2 = 0
 x 1
1− ⇒ radical axis is 2 (f1 - f2) y + c1 - c2 = 0 is x - axis
x
⇒ y = 0 ⇒ c1 = c2 = c.
 1 2x
f 1 −  + f ( x ) = − 2x  — (3) \ Required system is x2 + y2 +2fy + c =0
 x x −1
2x + 2yy1 + 2fy1 = 0
(1) + (3) - (2)
−x
⇒ 2 f(x) ⇒ f = − y Differentiate once more w.r.t.x.
y1
2 2 2x 2
= − + − 2x − 2 + 2x − + x  y − xy 
x 1− x x −1 x ⇒ 0 = −  1 2 2 − y 1 
2x + 2  y1 
=
x −1 ⇒ x y2 + y 13 − y 1 = 0
x +1 dy d2 y
⇒ f(x) = where, y 1 = , y2 = 2
x −1 dx dx
( x − 1) + 2 6. Put x = acos2 q + bsin2 q,
(ii) g(x) = ∫ f ( x ) dx = ∫ .dx
x −1 dx = 2(b - a) sinqcosq dq
= x + 2log| x − 1 | +C p
b 2
2(b − a)sin q cos qdq
g(2) = 2 ⇒ C = 0 ∫ = ∫ (a cos
a a
2
q + bsin2 q)(b − a)sin q cos q
\ g(x) = x + 2 log |x - 1|
p p
2
a +x2 2
ab ac sec2 qdq 2 2 sec2 qdq
= 2∫
b ∫a a
2
=
a a + btan q + tan2 q
4. (i) ab 2
b +x 2
bc
b
ac bc c + x2
2
p

2 1  −1  b tan q   2 2 p
(
a a +x 2 2
) ab 2
ac 2
= ×
b
 tan 
a   a  =
  0 ab
×
2
1
=
abc
a2 b (b 2
+ x2 b ) bc2
b
a2c b2 c (c 2
+ x2 c ) p
=
a2 + x2 b2 c2 ab
= a2 b2 + x 2 c2 1 1

7. ∫ (1 − x ) dx = 2∫ (1 − x ) dx , since f(x) is even


2 n 2 n
2 2 2 2
a b c +x
−1 0

= (a2 + b2 + c2 + x2) Put x = sin q, ⇒ dx = cos q dq


1 b2 c2 1 b2 c2 p
Limits for q are 0 and
1 b2 + x 2 c2 (
= x2 + ∑ a2 0 x2 ) 0 p
2
2 2 2 2 1 2
1 b c +x 0 0 x
∫ =2∫ cos
2n +1
qdq = 2I2n+1 (say)
f(x) = x x + a + b + c4
( 2 2 2 2
) −1 0
p p
1 2 2
1
 x7  where I2n +1 = ∫ cos2n +1 qdq = ∫ cos2n q.d(sin q)
5
(ii) ∫ f ( x ) dx =  7 + (a
2
+ b2 + c 2 ) x5  =
12
35 0 0
0 0 p


a 2 + b2 + c2 12 1 1
= − =
(
= cos2n q sin q )
2
0

5 35 7 5 p
2
⇒ a2 + b2 + c2 = 1 ⇒ locus of A is a sphere of radius
∫ (sin q)2ncos
2n −1
q(− sin q)dq
1 unit, centre at origin. 0
3.120  Integral Calculus

p p
2 2
sinnx
= 2n ∫ cos 2n −1 q sin2 qdq 9. Let ∫ sin x dx = In
0 0
p p
2
2
sinnx − sin(n − 2)x
= 2n ∫ cos 2n −1 q(1 − cos2 q)dq Then, In - In - 2 = ∫
0
sin x
dx
0
p p p
2 2 2
2cos(n − 1)xsin x
= (2n)∫ cos2n −1 q − (2n)I2n +1 = ∫ dx = 2 ∫ cos(n − 1)xdx
sin x
0 0 0

(2n + 1)I2n + 1 = (2n)I2n - 1 p


 sin(n − 1)x  2
Similarly, (2n - 1)I2n - 1 = (2n - 2)I2n - 3 = 2  = 0, since (n - 1) is even.
 (n − 1)  0
(2n - 3)I2n - 3 = (2n - 4)I2n - 5
⇒ In = In - 2 = In - 4 = In - 6 = …. = I1
……
p p p
2 2
2
sin x p
3I3 = 2 × I1 = 2 ∫ cos qdq = 2 =∫ dx = ∫ dx =
0 0
sin x 0
2
Multiplying the equations,
 x  sin t  
(2n)(2n − 2)(2n − 4)....2  x∫  dt 
I2n +1 =   t  
(2n + 1)(2n − 1)(2n − 3)..3 10. Limit = lim 5
x →5  x−5 
(1.2.3..n)2 × 22n 22n × (n!)2  
= =  
(2n + 1)! (2n + 1)!
 0 
22n +1 (n!)
2
1  = 0 form 
∫ (1 − x )
2 n
\ dx =
−1 (2n + 1)! x
sin t  sin x 
−1 0 2 ∫ t
dt + x 
 x 
∫ = ∫ x sin(−2px)dx − ∫ x sin pxdx + ∫ x sin pxdx = lim 5
8. ,
−2 −1 1
x →5 1
1 by L’ Hospital’s rule
since ∫ = 0 as [x] = 0 in this interval. = sin 5
0

−1 x [x] x
  − cos2px   − sin2px  
= − x   − (1) 
 4 p2   −2
11. ∫[t]dt = ∫ [t]dt + ∫ [t]dt
  2p  0 0 [x]

0 = I1 + I2
  − cos px   − sin px  
− x   − (1) 
 p2   −1
1 2 3 [x]
  p  I1 = ∫ [t]dt + ∫ [t]dt + ∫ [t]dt + ... +
2 0 1 2
∫ [t]dt
[x]−1
  − cos px   − sin px  
+ x   − (1) 
  p   p2  1 = 1 + 2 + 3 + …. + ([x] - 1)
[x]([x] − 1)
 (−1)(−1) (−2)(−1)   (−1)(−1)  =
= − −  + (−1) 2
 2p 2p   p 
x x
 −2 1  ∫ [t]dt = ∫ [x]dt = [x](t)
x
I2 =
+ + (−1) [x]

p p  [x] [x]

 1 1 1 2 1 −1 3 −7 = [x] (x - [x])
= − − − − − = − =
 2p p  p p p 2p p 2p Result follows
Integral Calculus  3.121

7x − 8 7t − 8 2
 x3 x2 
2
12. t = g(x) = ⇒x= = g(t) A2 = ∫  x 2 + 1 − ( x + 1) dx =  − 
x−7 t−7
1 3 2 1
7x − 8
⇒ f(x) = g(x) ((i.e) f(x) = )  8 4   1 1  16 − 12 1 5
x−7 =  −  −  − = + =
3 2 3 2 6 6 6
⇒ f(x) is its own inverse
5 1
fff(x) = f(ff(x)) = f(f(x)) = f(x) Required Area = + = 1 square unit
6 6
10 10 10
 41 
∫ fff(x)dx = ∫ f(x)dx = ∫  7 + x − 7 dx 1 + x2


8

= 14 + 41 loge 3
8 8
14. (i) ∫ x dx
Put t2 = 1 + x2
13. (i) Required region will come only in the first
quadrant x2 ≥ y -1 ; x - y + 1 ≥ 0 represents area 2t dt = 2x dx
containing (0, 0) x ( 1 + x ) dx = 2
t2
x2 ≥ y -1 represents area outside parabola I= ∫ x2 ∫ t2 − 1 dt
(ii) 1
A2 x−y+1=0 = ∫ dt +
x2=y–1

−1
dt ∫t 2
y
(1,2) E 1 t −1
= t + log
B 2 t +1
A A1
A4 1 x2 + 1 − 1
(0,1) = 1 + x2 + log
2 x2 + 1 + 1
x
2 O Cx=1 D
dy
x=0 A3 (ii) 1 + x 2 1 + y 2 + xy =0
x =2 dx

Required area = A3 + A4 1 + x2 y
⇒ dx + dy = 0
1 2
x 1 + y2
(
= ∫ x 2 + 1 dx + ) ∫ ( x + 1) dx Integrating
0 1
1 1 + x2 − 1
1 2 1 + x2 + log + 1 + y2 = c
 x3   x2  2 1 + x2 + 1
=  + x +  + x
3 0  2 1
1
15. (i) Let t = tan-1x ⇒ dt = dx
4  22  1  1 + x2
= +  + 2  −  + 1
3 2  2 
∫ t e dt = t.e
t t
− ∫ e t dt = e t (t − 1)
4 3 8 + 24 − 9 23
= +4− = = (tan )
−1
−1
= e tan x
x −1
3 2 6 6
(iii) Required Area = A1 +A2 dy 1 tan −1 x
(ii) + 2
y =
1 dx 1 + x 1 + x2
A1 = ∫  x + 1 − x 2 + 1  dx ( ) dy
0
+ P ( x ) y = Q ( x ) then
x x  2
1 1 3 1
dx
= −  = −
 2 3 0 2 3 Its solution is y e ∫
pdx
= ∫ Q.e ∫ dx + c
Pdx

1 1
= square units ∫ 1+ x2 dx −1
6 e = e tan x
3.122  Integral Calculus

tan −1 x tan−1 x dy dv
∫ 1 + x 2 e dx
−1
y.e tan x
= 19. y = vx ⇒ =v+x
dx dx

(tan ) dv 
−1 −1
y. e tan x
= e tan x −1
x −1 + c 
x  v + x  = vx + x 1 + v 2
 dx 

16. y = sec-1 x
dv dx
∫ y dx = ∫ sec ⇒ =
−1
\ x dx
1+ v 2 x
x
= xsec −1 x − ∫x x2 − 1
dx
log(v + 1 + v 2 ) = log x + log C
dx y y2
= xsec −1 x − ∫ 2
x −1
+ 1 + 2 = xC
x x
= xsec −1 x − log x + x 2 − 1 y + x 2 + y 2 = Cx 2

= xy − log x + x 2 − 1 .+ C 20. Let F(x) = [f(x) + f(- x)] [g(x) - g(-x)]


F(-x) = - F(x)
x \ I = 0
17. t = tan
2
21. Put z = x + y

2 dt
dz dy
\ I = ∫ ( )  = 1+
 4 1 − t2 dx dx
( )
0
1 + t 2 5 + 
 1 + t2  dz
⇒ − 1 = sinz + cosz
dx

2 dt ∞
dt
= ∫ 5 + 5t
0
2
+ 4 − 4t 2
= 2∫
0 t 2
+ 32
dz
= dx
1 + sinz + cosz

2  −1  t   2 p  p dz
=
3  tan  3   = 3  2 − 0 = 3 . = dx
 0   z z z
2cos2 + 2sin cos
2 2 2
18.
z
sec2  
C 2
y= x dz = dx
(9, 3)  z
2 1 + tan 
 2
1 z
O A B sec2
(3, 0) Integrating, ∫ 2 2 dz = ∫ dx
z
1 + tan
2

x = y2 as x > 0  z
log 1 + tan  = x + C1
 2
y2 - 2y - 3 = 0
y = 3 or y = -1  x + y
⇒ log 1 + tan = x + C1
 2 
⇒ x = 9
9
9
 2 23  1 x+y
⇒ 1 + tan = e x + C1
∫0 xdx − D ABC =  (x)  − × 6 × 3
3 0 2
2

2 x + y
= × 27 − 9 = 9 sq.units 1 + tan  = Ce x .
3  2 

Integral Calculus  3.123

dy 1 + x2 1 + x2
22. Given y
dx
=x 26. ∫ (1 − x )(1 + x + x ) (1 + x )(1 − x )
2 2
= 3

y dy = x dx
y2
x 2
=
1 − x3 + x3 + x2
=
(1 − x ) + x (1 + x )
3 2

2
=
2
+C  --------- (1) (1 + x )(1 − x 3 ) (1 + x )(1 − x ) 3


Since it passes through (5, 3) 1 x2
= +
9 25 1 + x 1 − x3
= +C
2 2 1 + x2
−16
\ ∫ (1 − x )(1 + x + x ) dx
2 2
⇒ C = = −8
2
1
\ (1) becomes = log(1 + x) − log 1 − x 3
3
( )
2 2
y x
= − 8 ⇒ x2 – y2 = 16. −1
2 2 \ A = 1 and B =
3
dx 1 \ A + 3B = 0
23. =
dy 1 + 9y 2 27. 3ex - e2x - 2 = (2 - ex) (ex -1)
dy ex e x dx
\ = 1 + 9y 2 \ ∫ dx = ∫ =I
dx 3e x − e2x − 2 2 − ex . ex − 1
d2 y 1 dy Put u2 = ex -1 ⇒ exdx = 2udx
= × 18y ×
dx 2
2 1 + 9y 2 dx 2udu 2du
\ I = ∫ =∫
d2 y 9y
( (
2 − u2 + 1 u 1 − u2 ))
= × 1 + 9y 2 = 9y
dx 2 1 + 9y 2
= 2sin-1(u) = 2sin-1 ( ex − 1 + C )
2
d y
\ ∝y  x + 3
3
 1 
3
dx 2
28. ∫  x + 2  dx = ∫ 1 + x + 2  dx
1
x+  1  x+ 1
24. t = e x
⇒ dt = 1 − 2  e x ⋅ dx  
 x  3 3 1
= ∫ 1 + + + 3 
dx
x+
1  x + 2 ( x + 2)2
( x + 2) 
⇒I=e x
+C
3 1
1 dx = x + 3log(x + 2) − − +C
3∫
25. I = x + 2 2 ( x + 2)2
2 7
x2 + x +
3 3 \ required coefficient is -3
1 dx 29. Take y = x - 1 ⇒ x = y + 1, dx = dy
= ∫ 2
3  1
2
2 5  3x 2 − x + 7
 x + 3  +  3 
 
\ F(x) = ∫ (x − 1) 7
dx

 1 3 ( y + 1) − ( y + 1) + 7
2
x+ 
1
= ×
1 
tan −1  3

= ∫ y7
dy
3 2 5  2 5 
  3y 2 + 5y + 9
3 3  = ∫ du
y7
1  3x + 1 
= tan −1  +C
∫ (3y )
−5
 2 5  = + 5y −6 + 9y −7 du
2 5
3.124  Integral Calculus

−3 5 9 = tan-1 t + C
= − −
4y 4 5y 5 6y 6 = tan-1 ( x) + C
=
(
− 3y 2 + 4y + 6 )+C Statement 1 is true and follows from Statement 2
4y 6 32. Statement 2 is true
Consider statement 1
− 3x 2 − 2x + 5
F(x) = +C 2p

4 ( x − 1) ∫ sin
6 2
mx cosnxdx
0
−5
⇒ F(0) = +C 1
2p
4 =
2 ∫0
(1− cos2mx ) cosnxdx
Given 4F(0) + 5 = 0 ⇒ C = 0
2p 2p
2x − 3x 2 − 5 1 1
2 ∫0
\ F(x) = = cosnxdx − ∫ cos2mx cosnxdx
20
4 ( x − 1)
6

= 0 - 0 if 2m ≠ n
i.e., 4(x - 1)6 F(x) = 2x - 3x2 - 5
2p
\ required coefficient of x2 is -3 1
∫ sin
2
If 2m = n, mx cosnxdx = - ×p≠0
0
2
ncos n −1 x
30. I = ∫ cos x [sec x + tan x ]
n n
dx ⇒ Statement 1 is not always true

33. Statement 2 is true


sec x dx
= n∫ ; Put u = sec x + tan x 3 + 3T

(sec x + tan x ) ∫ f(2x)dx put 2x = t


n

3
du = (sec x tanx + sec x) dx 2
dt = 2dx
= sec x (secx + tanx)dx 3 + 3T T
1
dx = sec x. (u). dx =
2 ∫ f(t)dt = 3 ∫ f(t)dt
du 3 0
\ secx dx =
u = 3I
 du  Statement 1 is false
 u  nu − n
= n∫ = 34. Statement 2 is true
un −n y = 2 meets the parabola y2 = x
−1 − cos n x \ 4 = x
= = +C
(sec x + tan x ) (1 + sin x )
n n
\ point of contact (4, 2)
2 2
31. Statement 2 is true \ Required area = ∫ xdy = ∫ y 2 dy
dx
∫ 2 x (1 + x )
0 0

2
 y3  8
=   =
1  3 0 3
Let x = t⇒ dx = dt
2 x \ Statement 1 is true but does not follow from
⇒ dx = 2tdt Statement 2
dx 2tdt
\ ∫ =∫ 35. Statement 2 is true
2 x (1 + x ) 2t t 2 + 1 ( ) General equation of the parabola is y2 = 4a(x - b),
dt where a and b are arbitrary constants
= ∫t 2
+1 a and b are arbitrary constant
Integral Calculus  3.125

dy f(x)dx
Differentiating 2y
dx
= 4a \ ∫ x2 + 5
= 2 log x + x 2 + 5

dy 1 2
( )
3
y = 2a - x +5 2
dx 3
2
d 2 y  dy  + 5 x2 + 5 + c
y 2 +  =0
dx  dx  \ Statement 1 is true
\ order = 2 Statement 2 does not say whether c is a criti-
Statement 1 is true and follows from statement 2 cal point of f(x). Therefore Statement 2 is false.
Choice (c)
36. Given f(x) is a polynomial of degree 3
p
\ f(x) = ax3 + bx2 + cx + d 37. I = ∫ ecos x cos3 (2n + 1) x dx
2

f(0) = d \ d = 2 0

f(1) = 1 \ a + b + c + d = 1 p p

a + b + c = 1 - 2 = -1
\ ∫ f (x ) dx = ∫ f (p − x ) dx
0 0

Since 0 is a critical point f ’(0) = 0


f(x) = ecos x cos3 (2n + 1) x
2

f ’(x) = 3ax2 + 2bx + c p
f ’(0) = c = 0 ∫ f (p − x ) dx
\ f ’(x) = 3ax + 2bx 2 0
p
f ”(x) = 6ax + 2b {− cos (2n + 1) x}
3
= ∫ ecos
2
x

Since f(x) does not have an extremum at 0
p p
0, b = 0 a + b + c = -1 ⇒ a = -1 = ∫ −ecos x cos3 (2n + 1) x = − ∫ f ( x )
2

\ f(x) = -x3 + 2 0 0

f (x ) 2−x 3
\ I = -I
∫ x2 + 5
= ∫ x2 + 5
dx
2I = 0, I = 0
dx x 3 dx \ Statement 1 is true
= 2 ∫ - ∫ dx
x2 + 5 x2 + 5 Statement 2 is also true and is the correct explana-
tion of Statement 1 choice (a).
x 3 dx
I1 = ∫ put t = x2 + 5 p
sin (2n + 1) x dx

2
x +5 38. Statement 1: let In =
0
sin x
t2 = x2 + 5
2t dt = 2x dx
p
sin (2n + 3) xdx
x xdx 2
In+1 = ∫
0
sin x
I1 = ∫ x2 + 5 p
sin (2n + 3) x − sin (2n + 1) x
In+1 - In = ∫ dx
(t 2
− 5 t dt ) 0
sin x
= ∫ t p
2sin x cos (2n + 2) x
= ∫ dx
∫ (t )
− 5 dt = ∫ t dt − 5 ∫ dt sin x
2 2
= 0

=
t3
- 5t + c = ∫ 2cos (2n + 2) xdx = 0
5 0

1 ⇒ ln+1 = ln
(x )
3
2
= +5 2
- 5 x2 + 5
3 ln+1 = ln =… = l1
3.126  Integral Calculus

p p 43. Let cx = t; cdx = dt.


sin3xdx
= (
∫0 sin x = ∫0 3 − 4sin x dx
2
) ∞
t
n −1
dt 1

G (n )
I = ∫ e− t   . .= n ∫e
−t
.t n −1dt =
p 0
c c c 0 cn
= ∫ (1 + 2cos2x ) dx = p ∞
x 5 −1
∫ (1 + x ) dx = b (5,1)
0
44. 5 +1
\ Statement 1 is false 0

Statement 2 is true
dt
Choice (d) 45. t = sin2 x ⇒ = sin x cos x dx; x = 0 ⇒ t = 0 ;
2
39. f ’(x) = ex [x2 + ab - x(a + b)] p
x= ⇒t=1
ex (x - a) (x - b) < 0 in (a, b) 2
p
⇒ Statement 1 is false. However, Statement 2 is true. 1 2 5 −3

Choice (d)
I=
2 ∫ (sin x )3 (cos x ) 2 (2 sin x cos x ) dx
0

11 1
x2 1  11  1 G G
40. f (x) = x4 - 2x3 + +5 1  − 1 −1
6 4
= ∫ t  6  (1 − t ) dt =
4

2
20 25
f ’(x) = 4x3 - 6x2 + x = 2x (2x2 - 3x + 1) 2G
12
= 2x (x - 1) (2x - 1)
dy 1 −3
1 46. Put x4 = y ⇒ dx = 3
= y 4 dy.
\ x = 0, 1, 4x 4
2 −
3 1
−1
∞ ∞ ∞
f ”(x) = 12x - 12x + 1 =
2 dx 1 y 4 dy 1 y4
∫0 1 + x 4 = 4 ∫0 1 + y = 4 ∫0 1 + y dy
f ”(0) = 1 > 0, f ”(1) = 1 > 0
1 1 p p p
f”   < 0 = . using (f ) = =
2 4 p
  1 2 2
sin   4.
4 2
\ f(x) is minimum at x = 0 and x = 1
1
 x2  47. tan5x = tan(3x + 2x)
Required area = ∫  x 4 − 2x 3 + + 5  dx tan3x + tan2x
0
2  =
1
1 − tan3x.tan2x
 x5 x 4 x3  146 73
=  − 2. + + 5x  = = tan5x - tan2x.tan3x.tan5x = tan3x + tan2x
5 4 6 0 30 15
\ tan2x tan3x .tan5x = tan5x - tan2x - tan3x
\ Statement 1 is true \ ∫ tan2x tan3x.tan5xdx
Statement 2 is true
= ∫ ( tan5x − tan2x − tan3x ) dx
We made use of Statement 2 to prove
Statement 1. Choice (a) 1
= log | sec5x |
5
∞ ∞
1 1
∫x e dx = ∫ e − x .x 4 −1 .dx = G(4) = 3! = 6 − log | sec2x |− log | sec3x |
3 −x
41.
0 0
2 3
1 1
1
G 5 G (4 )
= log sec 5
(5x ) + log sec− 2 (2x ) +
∫ x (1 − x ) dx = b (5,4 ) =
5 −1 4 −1
42.
0
G (9) log sec
−1
3
(3x ) + k

4!3! 2×3 1 1 1 1
= = = = log sec 5
(5x )sec− 2 (2x )sec− 3 (3x ) + k
8! 5 × 6 × 7 × 8 280
Integral Calculus  3.127

−1 −1 1  1 1 
⇒ a = , b= , c = = 2∫ 2 + 2  dy
2 3 5  y −1 y +1 
\ a, b, c are not in H.P. and a ≠ b ≠ c
1 1 1  y −1 
But + + = 0 ⇒ ab + bc + ca = 0 = log   + 2tan y + c
-1
a b c  y +1 
1 1 1 1 1 1 ⇒ g(x) = log|x|
and 3 + 3 + 3 = 3 . .
a b c a b c
f(x) = tan-1x
i.e., a-3 + b-3 + c-3 = 3a-1b-1c-1
1
48. f(x) = x and g(x) = [f(x)] ⇒ g(x) = [x] 50. f(x) = e2x ⇒ f-1(x) = log x
2
f (x) x
g(x) = sin-1x ⇒ g-1(x) = sinx
\ ∫ g(t)dt = ∫[t]dt
0 0 (fof-1)x = x
−1
1 2 [x] x
(fog)x = e2 sin x

= ∫ [t]dt + ∫ [t]dt + .... + ∫ [t]dt + ∫ [t]dt


0 1 [x] − 1 [x] 1
g’(x) =
= 0 + 1 + 2 +…..+[x] - 1 + [x]{x} 1 − x2
([ x ] −1)[ x ] +[x]{x} e
1
e

∫ (f o f )(x ) f (x ) dx = ∫ x 2 log xdx


−1 −1
= (a)
2 1 1

e
g(x) [x]
t  2 [x]
[x] 2 1
= ∫ x. log x dx
Again, ∫ f(t)dt = ∫ tdt =  2 
0 0
=
2 1
2
0
e
Equating, 1 x2 x2 
=  log x. − 
 [x] − 1  2 2 4 1
[x] 2

 2  [ x ] + [ x ]{x} = 2 1 2
   x (2log x −1)
e
=  
8 1
[x] +
-
2
[ x ]( x −[x]) = 0 =
1 2
e (2 − 1) − 1. − 1 =
e2 + 1
8 8
[x] = 0 or
f (1) + f (0)
1 1 =
- + x − [x] = 0 ⇒ {x} = 8
2 2 1 1 −1
e2 sin x

1 −x
(b) ∫ (f o g ) x. g ' (x ) dx = ∫ 1 − x2
dx
49. From the relation y = 2 0 0
1+ x p
2

∫e
2q
1− y 2
= dq (taking sin-1x = q)
⇒ x = 0
1 + y2
p
−4y  e2 q  2
dx = dy =  
(
1 + y2 )
2
 2 0
ep − 1
y  1+ y 2  −4y =
∫ x ∫  1 − y 2  × 1+ y 2
dx = y dy 2
( )
2

p
f   − f(0)
y 2 dy 2
= -4 ∫ =
(1 − y )(1 + y )
2 2
2
3.128  Integral Calculus

p p
3 dx
∫ x − 5 dx − 2∫ x + 7
2 2
\ I =
∫ f(x)g (x)dx = ∫ e .sin x dx
−1
2x
(c)
0 0
= 3 log (x – 5) – 2 log (x + 7) + C
p
2
= log (x – 5)3 – log (x + 7)2 + C
∫ e sin xdx
2x
Let I =
( x − 5)
3
0
= log +C.
(x + 7)
π 2
π 2
I = e2 x − cos x  2
− ∫ 2e2 x . − cos x dx
0
0
sin x dx
p
2
52. ∫ 1 + sin x
∫e
2x
= (0 + 1) + 2 cos x dx
0 (sin x + 1 − 1)  1 
p
= ∫ 1 + sin x
dx = ∫ 1 −
 1 + sin x 
dx
= 1 + 2 e2x .sin x  0 2 - 4I
dx 1 − sin x
p
2e + 1
= x− ∫ 1 + sin x = x− ∫ cos2 x
dx
5I = 2ep + 1 ⇒ I =
5
∫ (sec )
2
= x− x − sec x tan x dx
p = x – tan x + sec x + C.
2f   + f(0)
2
=
5 a−x a−x a−x
−1 −1
53. ∫ a+x
dx = ∫ a − x2
2
dx

(d) ∫ xf(x)dx = ∫ xe2x dx


a−x
−3
2
−3
2
= ∫ a2 − x2
dx
−1
 xe2x e2x 
=  − a dx 1 −2x dx
4  − 3 ∫ 2 ∫ a2 − x2
 2 = +
2 2
2
a −x
 e e   3e −3 e −3 
−2 −2
x 1 du
= − − − − −
a 2∫ u
4   4 4  = a sin −1 +
 2
3e −2 x
=- + e −3 where, u = a2 - x2 = a sin −1 + a 2 − x 2 + C.
4 a
4e −3 − 3e −2
= x dx
4 54. I = ∫ e x
( x + 1)
2


1
4 2 (( )
= f − 3 − f (−1) )  x + 1 − 1
= ∫ ex   dx
 ( x + 1) 
2

IIT Assignment Exercise


 1 1 
= ∫ ex  −  dx
x + 31 x + 31  x + 1 ( x + 1) 
2
51. =
x + 2x − 35 ( x + 7 ) ( x − 5)
2

= ∫ e x  f ( x ) + f 1 ( x )  dx
A B
= +
x+7 x−5 1
where, f(x) =
\ x + 31 = A (x – 5) + B (x + 7) x +1
Put x = -7, A = -2 ex
\ I = + C.
Put x = 5, B = 3 x +1
Integral Calculus  3.129

60. 4ex + 6e-x = A (9ex - 4e-x) + B(9ex + 4e-x)


∫ tan
−1
55. x dx
Comparing coefficients of ex and e-x,
1 2x dx 19 35
2 ∫ 1 + x2
= x tan −1 x − A=- ;B =
36 36
1
= x tan −1 x − log 1 + x 2 ( ) d
( )
35 ∫ dx
2 9e x − 4e − x
19
−1 2 I = ∫ − 1dx + +C
= x tan x − log 1 + x .+ C 36 36 9e x − 4e − x
19 35 9e2x − 4
56. I = ∫ e − x sin x dx = − x + log +C
36 36 ex
= e − x (− cos x ) − ∫ cos x e − x dx
 19 35  35
= x  − −  + | 9e2x − 4 | + C
 36 36  36
= −e − x cos x − e − x (sin x ) + ∫ sin x e − x dx 

= −e − x cos x − e − x sin x − I (2x 3t


+ 3x 2t + 6x t ) dt
61. I = ∫( x 3t + x 2t + x t ) x
\ 2I = −e − x (sin x + cos x )
∫ (x ) (2x )
1
3t −1
e− x = + x 2t −1 + x t −1 3t
+ 3x 2t + 6x t t
dt
I=− (sin x + cos x ) + C
2 1 1
= t ∫ y t dy
57. g(x) = 2x – 8x + 1 2
6
t +1
g(–x) = 2x2 + 8x + 1 1 y t
\ f(x) = –8x which is odd where, y = 2x3t + 3x2t + 6xt = . +C
6 t +1
\ I = 0
62. g(x) = ex
p
2
f (sin x ) dx f(x) = x2 – ex
58. I = ∫ f (sin x ) + f (cos x ) 1
0 \ ∫ f (x ) g (x ) dx
0
p
2
f (cos x ) dx
= ∫ f (sin x ) + f (cos x )
0
1

(
= ∫ e x x 2 − e x dx )
p
2
0
p
∫ dx =
1
\ 2I =
∫ (x )
2
0
2 = e x − e2x dx
0
p
⇒ I = . 1
4  e2x 
=  x 2 e x − ∫ e x 2x dx − 
 2 0
p 
p p sin  − x 
2
sin x dx 2
2  e2x 
1
59. I = ∫ sin x + cos x =∫
p  p  = x 2 e x − 2x e x − ∫ e x 2 dx  − 
2 0
0 0
sin  − x  + cos  − 1
2  2 
1
p  e2x 
2
cos x =  x 2 e x − 2x e x + 2e x − 
=∫ dx  2 0
0
sin x + cos x
p
2  e2   1
p =  e − 2e + 2e −  −  2 −
\ 2 I = ∫ 1 dx =  2  2 
0
2
p e2 3
I= = e− − .
4 2 2
3.130  Integral Calculus

x − 2  y − 3 dy
63. cos q =  ; sin q =  From (2), x − h = − ( y − k ) ⋅
 4   3  dx
  dy  
2

( x − 2) ( y − 3)
2 2
1 +   
\ + =1  dx   dy
42 32 
=
This is an ellipse. d2 y dx
2
Area = p × 4 × 3 = 12p. dx
Substituting the values of x – h and y – k in (1)
64. 2 2
  dy    dy 
2 2
  dy  
2

2 1 +      1 +   
  dx    dx    dx  
3 2
+ 2
= r2
 d2 y   d2 y 
 dx 2   dx 2 
3 2
  dy  
2
2 d y 
2
x2 y 2 Simplifying, we get 1 +    = r  2 
+ =1  dx  
9 4   dx 
x y
+ =1 dy 2y
3 2 67. =
dx x
p×3×2 1 dy dx
Area = − ×2×3 =2
4 2 y x
3p 6 3 dy dx
= − = (p − 2) .
2 2 2
Solution is ∫ y
= 2∫
x
65. Squaring the given equation, log y = 2 log x + log C

3 2 log y = log x2 C
 dy  
2
2 d y 
2
1 +    = a  2  y = x2 C
  dx    dx 
Since the curve passes through (1, 2)
⇒ Order = 2,
2 = 12 C ⇒ C = 2
Degree = 2.
\ The equation is y = 2x2.
66. The equation of the circle is
dy
(x – h)2 + (y – k)2 = r2  ---------- (1) 68. The given equation is y − ay 2 = (a + x )
dx
Differentiating (1), w.r.t x,
dx dy
dy ⇒ =
2 (x – h) + 2 (y – k) =0 a + x y − ay 2
dx
dx 1 a 
dy ⇒ = +  dy
⇒ x – h + (y – k) =0  ----------- (2) a + x  y 1 − ay 
dx
Differentiating again w.r.t x, dx 1 −a 
2
∫ a + x = ∫  y − 1 − ay  dy
d 2 y  dy 
1 + (y − k ) 2 +   = 0 ------------ (3) log (a + x) + log C = log y – log (1 – ay)
dx  dx 
 y
 log C (a + x) = log
 dy  
2
1 − ay
− 1 +   
  dx   y
From (3), y − k = C (a + x ) =
d2 y 1 − ay
dx 2 \ y = C (a + x) (1 – ay).
Integral Calculus  3.131

 dy  73. Put x = sin q ⇒ dx = cos q dq


69. sin −1   = x + y
 dx  x + 1 − x2 sin q + cos q
\ =
dy 1− x 2 cos q
⇒ = sin ( x + y )
dx (sin q + cos q)
Put x + y = z I = ∫ eq cos qdq
cos q
dy dz
1+ = = ∫ eq (sin q + cos q)dq
dx dx
−1
dy dz dz = eq sin q = xesin x
+C
= −1 ⇒ − 1 = sinz
dx dx dx
x
dz 74. Put =t
= dx 1 + log x
1 + sinz
Integrating, we get tan z – sec z = x + C 1
(1 + log x)1 − x. log xdx
⇒ tan (x + y) – sec (x + y) = x + C. dt = x dx =
2
(1 + log x) (1 + log x)2
1 1
70. sin x =(sin x − cos x ) + (cos x + sin x ) I = ∫ dt = t + C
2 2
1 1
(sin x − cos x ) + (cos x + sin x ) (2x + 1 + 2) dx
∴I = ∫ 2 2 dx
75. I = ∫ x2 + x + 1
sin x − cos x
 1 1 cos x + sin x  (2x + 1) dx dx
= ∫ +
 2 2 sin x − cos x 
dx = ∫ 2
x + x +1
+ 2∫
2
x + x +1
x 1 du dx
= + log (sin x − cos x ) + C = 2∫ + 2∫
2 2 2 u 1 3
x2 + x + +
4 4
1
1+ dx
x 2 1 = 2 u + 2∫
71. I = ∫ dx where, u = x − 2 2
1 x  1  3
x2 + 2  x + 2  +  2 
x  
 1
x−
1  u −1 1 −1  x  1 
∴I = tan 
  = tan   = 2 1 + x + x 2 + 2log  x + + 1 + x + x 2  + C
2 2 2  2   2 
 
2 ⋅ 3 ⋅1 2
1  x2 − 1  76. I = = .
= tan −1  +C. 7 ⋅ 5 ⋅ 3 ⋅ 1 35
2  2x 
77. |x| = x, x > 0
72. u = x 4 ⇒ du = 4x 3 dx = -x, x < 0
du 0 1
∴I = ∫ 4 (u 2
+ 4u + 13 )
\ I = ∫ − x (x + 3) dx + ∫ x (x + 3) dx
−1 0
−1 1
1 du
= ∫ ∫ (x ) ∫ (x )
2 2
= + 3x dx + + 3x dx
4 (u + 2)2 + 32 0 0
−1 1
1 1 −1  u + 2   x 3 3x 2   x3 3 2 
= tan  =  +  + + x 
43  3  3 2 0  3 2 0
1  x4 + 2  1 3 1 3
= tan −1  +C = − + + + = 3.
12  3  3 2 3 2
3.132  Integral Calculus

OR Area below the x-axis


1 1 2 2
 x4 2
∫ = ∫ x | x | dx + 3 ∫ | x | dx ∫1
3
y dx =  −x +x 
−1 −1 4 1
1

= 0 + 6 ∫ xdx , since x|x| is an odd function. 1 


= ( 4 − 8 + 4 ) −  − 1 + 1
0 4 
= 3
1 1
1 = − =
 3  4 4
78. I = ∫  − 1 dx
0
 1 + x  1 1 1
Total area = + = .
4 4 2
= 3log(1 + x) − x ]0
1

= 3 log 2 - 1 81. Y
B
e2
dx
79. I1 = ∫ log x b
e o a A X
dx
Put u = log x ⇒ du =
x
dx = x du = eu du Required area
u p 1
2
e du = ab − ab
\ I1 = ∫
1
u
4 2
ab
2
e x dx
= (p − 2) .
4
⇒ = ∫
1
x
= I2
82. y
\ I1 − I2 = 0. (0, 1) (2, 1)
y = 1- x
y = x− 1
80. 2 y = (x− 1)2
0 1
O x
(1, 0)
The curve meets the x-axis at x = 0, x = 1 and
x = 2 and is roughly shown in figure. Required area
\ Area above the x-axis 1 1 
= 2 ×  × 1 × 1 − × 1 × 1
1
2 3 
= ∫ y dx
0 1 1
= 2× = .
1 6 3
(
= ∫ x x 2 − 3x + 2 dx )
0 dy 1
83. =
1 dx 2y
∫ (x )
3
= − 3x 2 + 2x dx
0
2y dy = dx
1
y2 = x + C  — (1)
 x 4 3x 3 2x 2 
=  − +  Since (1) passes through (4, 3)
4 3 2 0
32 = 4 + C
1 1 ⇒ C = 5
= −1+1 = .
4 4 \ (1) becomes y2 = x + 5.
Integral Calculus  3.133

dy x−2 x 2 (2x − 1)  e −2 x y  dx
84. The equation is − y = 86. The given equation is  −  =1
dx x ( x − 1) x −1  x x  dy
x−2 dy e −2 x y
P= – i.e., = −
x ( x − 1) dx x x
dy 1 e −2 x
x−2 2 1  + y =
∫ Pdx = − ∫ x ( x − 1) dx = - ∫  x − x − 1  dx dx x x
1 e −2 x

x 2
x −1 P= ;Q=
= − log = log 2 x x
x −1 x 2 x
I.F = e
x −1
\ I.F = . e −2 x
Solution is y ⋅ e2 ∫
x
x2 = ⋅ e2 x dx
x
85. The given equation is
y e2 x = 2 x + C.
dy
− xy = x 3 y 2 (Bernoulli’s equation)
dx 87. We have d (x2 + y2) = 2x dx + 2y dy
Dividing by y2 = 2 [x dx + y dy]
1 dy 1   y  1 x dy − y dx
− x = x3 d  tan −1    = ×
y 2 dx y   x  y 2
x2
1+
−1 x2
Put z = x dy − y dx
y =
x2 + y 2
dz 1 dy dz \ The given equation is
= ⇒ + xz = x 3
dx y 2 dx dx
1   y 
P = x, Q = x3

2
( 
)
d x 2 + y 2 + d  tan −1    = 0
 x 
x2
1 2 y
I.F = e 2 Integrating,
2 x
(
x + y 2 + tan −1 = C1 )
Solution is z (I.F) = ∫ Q (I.F ) dx y
⇒ x 2 + y 2 + 2tan −1   = 2C1
x2 x2
x
z ⋅ e 2 = ∫ x 3 ⋅ e 2 dx y
⇒ x 2 + y 2 + 2tan −1   = C
x 2 x
= ∫ x 2 ⋅ e 2 ⋅ x dx y
2tan −1   = C − x 2 − y 2
x2 x
Put t = ; 2dt = 2x dx; dt = x dx
2 y  C2 − x2 − y 2 
= tan   .
= ∫ 2t ⋅ e t dt = 2  te t − e t  + C x  2

−1 x2  x2 x
2
x 2 2
 88. The given equation is
⇒ e = 2 e2 −e2 + C
y  2  1 dy 2
− tan x = tan3 x
2 2 2
y 2 dx y
−1 x2 x x
⇒ e = x 2 e 2 − 2e 2 + C 1
y Put z =
y
2
−x
1 dz −1 dy
⇒ = − x 2 + 2 + Ce 2 . =
y dx y 2 dx
3.134  Integral Calculus

dz 93. Y
⇒ − − 2z tan x = tan3 x
dx
dz
⇒ + 2z tan x = − tan3 x
dx X
O (0, 0)
(0, 0)
I.F = sec2 x
Solution is z (sec2 x) = ∫ − tan3 x ⋅ sec2 x dx

tan 4 x The straight line meets the curve at the points where
= − +C
4 x2 = x(1 - m) (i.e.,) x = 0 or 1 - m
1 2 tan 4 x When m > 1, 1 - m is - ve
\ sec x = − +C
y 4 0
9
∫ (x − x )
2
\ Area = − mx dx =
4 sec x + y tan x = 4Cy.
2 4
1− m
2
(1 - m) = - 27
3
89. ∫ sec (xe
2 x
) ⋅ e x (1 + x)dx
1 - m = - 3; m = 4
t = xex ⇒ dt = ex(1 + x)dx 1
 x2 x3  1 1 1
I = ∫ sec tdt = tan t + C
2
94. = 2  −  = 2  −  =
2 3 0 2 3 3
1 1
e −3x
90.
−1
∫ [x + 1]dx = ∫ {1 + [x]} dx −1
⇒ y =
9
+ C1 x + C 2

0 1 1
95. < x < 1 ⇒ log e x < 0
=2+ ∫ (−1)dx + ∫ 0dx
−1 0
e
1 e2
= 2 - [0 - (-1)] log x log x
= ∫− dx + ∫ dx
=2-1=1 1 x 1
x
e

91. f(x) = sin3 x + 2x sin2 (x(x2 + 1) 1 e2


 (log x)2   (log x)2 
+ x2 sin (x(1 + 2x2 + x4)) = −  + 
 2  1  2 1
f(-x) = -sin3 x - 2x sin2 (x(1 + x2)) e

- x sin (x(1 + 2x + x ))
2 2 4 −1 1 1 5
= [0 − 1] + [4] = + 2 =
a 2 2 2 2
⇒ f(-x) = -f(x) ⇒ ∫ f(x)dx = 0
−a
96. {x} = x - [x]
1000 1 2 1000
92. As |cos x| is even,
∫ e x −[x]dx = ∫ e x dx + ∫ e x −1dx + ........ ∫e
x − 999
dx
30 p 0 0 1 999
I = 2 ∫ | cos x | dx = (e - 1) + (e1 - 1) + ………(e’ - 1) = 1000(e - 1)
0

p 97.
= 2 × 30 × ∫ | cos x | dx , C(−4, 6)
0
x−y= 2

As cos x is periodic with period p, x+y=2


(4, 2)
 p/2 p
 E
= 60 ×  ∫ cos x dx + ∫p/2 − cos x dx 
0
D
B x=4
= 60 × ˘˘˘˘˘)0 − )p/2  A
p /2 p
= x = −4
 x=2
Integral Calculus  3.135

1 1 2
1− x
2
1 − x2
Required area = (6 × 6) + × 2 × 2
2 2
101. ∫0 1 + x 3 dx = ∫ (1 + x ) (1 + x )dx
0
3

= 18 + 2 = 20
2 2
1 x2
98. xy12 + (y - x)y1 - y = 0 = ∫0 1 + x dx − ∫0 1 + x 3 dx
xy1(y1 - 1) + y(y1 - 1) = 0
2
1 
 dy
⇒  x
  dy
+ y

− 1 = 0 = log(1 + x) - log 1 + x 3 
3 0
( )
 dx   dx 
2
dy −y dy
(1 + x )  1
3
⇒ = (or) =1 1
dx x dx = log = log 3
1 1
3 (
1 + x 3 
0
3 )
⇒ − ∫ dy = ∫ dx or y = x + c2
y x = log 3 3
⇒ -log y = log x + log c or y = x + c2

∫ (1 + x ) (1 + x ) (1 + x )... (1 + x )dx
1
⇒ c2 = 1 102. I = 2 4 2n − 1

⇒ xy = c1 y = x + 1  — (2) 0

∫ (1 − x ) (1 + x ) (1 + x )... (1 + x )dx
1
2 × 3 = c1 2 2n − 1

⇒ xy = 6 — (1)  = 0

from (1) & (2) x(x + 1) = 6 (1 − x )


n
x2 + x - 6 = 0 1
1 − x2
⇒ x = 2, x = -3
 = ∫0 (1 − x ) dx
⇒ y = 3, y = -2
∫ (1 + x + x )dx
1
n
2 −1
\ (-3, -2) is the other point.  = + x 3 + ... + x 2
0

99. Let t = 1+ xex 


1
x2 
n
x2 x3
dt  = x + + + ... + n 
dt = ex(x+ 1)dx ⇒ dx = x
 2 3 2 0
e (x + 1)
2n
1 1 1 1
I =∫
dt  1
= ∫
1 1
− − 2  dt   = 1 + + + ... + n =
2 3 2
∑  r 
(t − 1)t 2
t −1 t t  r =1

\ k = 2n
1
= log | t − 1 | − log | t | + + C 3a
t f(x)
1
103. Let I = ∫ f(x) + g(x) + h(x) dx
= log | xe x | − log | 1 + xe x | + +C 0

1 + xe x 3a
f(3a − x)
  = log
xe
+
1 x
+C
= ∫ f(3a − x) + g(3a − x) + h(3a − x) dx
0
1 + xe x 1 + xe x
3a
g(x)
100. Let t = tan x .logx-1 I= ∫ g(x) + h(x) + f(x) = I
0
 log x tan −1 x 
⇒ dt =  2
+  dx Similarly,
1 + x x 
3a
x=1⇒t=0 g(3a − x)
I= ∫ g(3a − x) + h(3a − x) + f(3a − x)
x = e ⇒ t = tan-1e 0

tan −1 e 3a
h(x)

−1
dt = [t]0tan e

0
= tan-1 e + C = ∫ h(x) + f(x) + g(x) dx
0
3.136  Integral Calculus

3a
 (sin x ) x dx 
(( ) −∫
f(x) + g(x) + h(x) 1 1 −1
\ I + I + I = ∫0 f(x) + g(x) + h(x) dx = 3a )
2
= 2  sin −1 x x 2
 0
0 1 − x2 
3I = 3a ⇒ I = a p
p2 2
x 2
= − 4 ∫ q sin qdq
 t − 1 2
104. Let F(x) = ∫0  t + 1  dt 0

p2 p p2
For minimum value F’(x) = 0 and F”(x) > 0 2I = − 4 q.− cos q + sin q 2 = −4
2 0 2
x2 − 1 2x 3 − 2x
Now, F’(x) = .2x = p2 − 8
x2 + 1 x2 + 1 ( ) \ I =
4
F’(x) = 0 ⇒ x = 0, ±1
(tan x ) −1 2
1

F”(x) =
(x 2
)(
+ 1 . 6x − 2 − 2x − 2x .2x2
) ( 3
) 106. Let I = ∫ 1 + e tan x
dx Take y = -x
(x + 1)
2 −1
2

dy = -dx
F”(0) = < 0 and F”(±1) > 0
(tan y ) .du−1 2
−1

F(x) has minimum value at x = ±1 = −∫


1 1 + e − tan y
(±1)2
 t − 1
∫ (tan x ) .dx
2
\ Required minimum =  t + 1  dt −1 −1

0 i.e., I = − ∫
1 1 1 + e − tan x
 2 
= ∫ 1 − dt Adding we get
0
 t + 1  1
 1 + e tan x 
( )
2
∫  1 + etan x
−1
I+I=  . tan x dx
=  t − 2log (t + 1) 0
1
−1

1
= 1 - 2log2 = 1 - log4
( )
2
= 2 ∫ tan −1 x dx

(sin x )
2 0
1 −1

105. Let I = ∫ 1+ p dx 1 1
(
2 tan −1 x ) .x.dx 
(
= 2  tan −1 x .x  − )
sin x 2
−1
 0 ∫ (1 + x ) 2

Take y = -x, dy = -dx 0

(sin y ) .dy
2 p
−1
−1
p2 4

\ I = - ∫ 2I = − 4 ∫ q tan qdq
1 1 + p− sin y 8 0

\ I is depend only in ‘p’ not on ‘e’


(sin x ) dx −1 2
1

I= ∫
−1 1 + p− sin x 107.
1 + sinx
1
 1 1 
( )
2
\ I + I = ∫  + − sin x 
sin −1 x dx 1 − cosx
−1
 1 + p sin x
1 + p 
1
 1 + psin x 
( )
2
= ∫  1 + p
−1
sin x 

sin −1 x dx
3π π 7π 2π
4 4
1

∫ (sin x ) dx
−1 2
2I = From the graph it is clear that
−1
3p 7p
2p 4 4 2p
1

= 2 ∫ sin x dx ( −1
)
2
∫ F(x)dx =
0

0
f(x)dx + ∫
3p
f(x)dx + ∫ f(x)dx
7p
0 4 4
Integral Calculus  3.137

3p 7p p
4 4 2
= ∫ (1 + sin x ) dx + ∫ (1 − cos x ) dx + ∫q
3
\ I = sin qdq
0 3p 0
4
2p p

∫ (1 + sin x ) dx
2
= q (- cos q)]0 +
p
∫ 3q
2
3 2
cos qdq
7p
4 0

p
3π 3π 2
= x]0 4
− [cos x]0 4
+
= 0 + 3q . sin q]0 −
p


4
2 2
∫ 6q sin qdq
0

x ∫ −[sin x] 3π
4
4
+ [ x − cos x]27 ππ
4
3p2  2
p p
2

4 =
4
+ 6q.cos q −
0
∫ 6cos qdq
0
3p  3p   7 p 3p 
= − cos − cos0 +  − 
4 
p
4  4 4 3p2  2 3p2
= + 0 − 6sin q = −6
 7p 3p   7p  4 0 4
− sin − sin  + 2 p − 4 
 4 4  
=
3p2 − 24
=3
p2 − 8 ( )
 2 p 4 4
−  cos2p − cos 
 4 
6 1

∫( )
sin −1 {x} d {x} = 6 ∫ sin −1 {x} d {x} ( )
2 2

1 1 1  1  110. I =
= 2p + +1+ + − 1 −  0 0
2 2 2  2
Since {x} is periodic with period 1
1
( )
1
= 2p +
2
= 2p + 2 2 = 2 p + 2 ( )
\ I = 6 ∫ sin −1 x dx, since {x} = x for x ∈ (0, 1)
0

( )
1
3
p
2
px
3
px  2 sin −1 x 
( )
2
108. ∫ | sin x | dx = ∫1 sin 2 dx + ∫2 sin 2 dx \ I = 6  sin −1
x x −∫ .x  dx
1
2  1 − x 2  0
2 3
 px   px  p
= ∫ sin   dx + ∫ − sin   dx  p2  2

 2   2  = 6   − 12 ∫ (q sin q) dq
4
1 2
0
2
 px 
2
 px 
3 3π2 π
cos     = − 12[θ(− cos θ) + sin θ]0 2
cos
= −
 2   − − 2  2
 
p
2

  p 
=
3p 2
− 12 = =
2
3p2 − 24 3 p − 8 ( )
 2 2 2 2 2
1
−2  p  2  3p  111.
=  cos p − cos  + cos − cos p
p  2 p 2 
4
= 0 2
p
a a

109. Using property ∫ f(a − x)dx = ∫ f(x)dx we have


0 0

x
(1 − x )(sin−1 (1 − x )) ( )
3 3
1 1
x sin −1 x 2y2 = (1 + y2)x ⇒ y2 =
I= ∫ 0 2x − x 2
dx = ∫
0 1 − x2
dx 2−x
x
Since > 0 ⇒ x ∈ [0, 2]
Take sinq = x, dx = cosq dq 2−x
3.138  Integral Calculus

Graph is symmetric about x axis and x = 2 its vertical 114.


asymptotes.
 
2 y > 0 y' > 0
y' = ⇒ and
(2 − x ) y y < 0 y ' < 0
2

2
x
\ Required area A = 2 ∫ dx   
0
2−x
Take x = 2sin2q ⇒ dx = 4sinqcosqdq
p
2
2sin2 q
\ A = 2 ∫ .4sin q cos qdq dx
2 − sin2 q Given = −2 (2 − y ) ⇒ dx = 2 ( y − 2) dy
0 dy
p
2
1 p \ x = (y - 2)2 + C, it passes through (3, 0)
= 8 ∫ sin qdq = 8. . = 2p
2

0
2 2 \ C = -1
5
\ Equation of curve is
4 4
2(4 - x) 2
(y - 2)2 = x + 1
112. Required area = ∫ ydx = ∫ 1
dx
(x - 3) 2
3 3
dy 1 1
Take x = 4sin q + 3cos q ⇒ dx = 2sinqcosqdq
2 2 Slope of tangent is = at (0, 3) it is
dx 2 ( y − 2) 2
4 - x = cos2q, x - 3 = sin2q
p 1
When x = 3 ⇒ q = 0 and x = 4 ⇒ q =
2
\ Equation of tangent is y - 3 = ( x − 0)
2
p
2
2.cos5 q i.e., 2y - 6 = x
\ Area = ∫ sin q
.2sin q cos qdq 3 3
0 From the graph, required area = ∫ x1dy − ∫ x 2 dy
p 0 0
2
5 3 1 p 5p
∫ cos qdq = 4.
6
=4 . . = units Where x1 corresponds the tangent 2y - 6 = x and x2
64 2 2 8
0 corresponds the parabola (y - 2)2 = (x + 1)

∫ (2y − 6) − (( y − 2) )
2 3
2
[ x ] |x| {x}
113. Required area = ∫ e .e .e dx
−1
\ Area =
0
− 1 dy

2 3 0

∫( )
|x|+ [ x ] + {x}
∫e ∫ ( y − 3)
2
= dx = 6y − y 2 − 9 dy = dy = 9
−1 0 3

∫e
|x|+ x
= dx, since [x] + {x} = x. 115.
−1

0 2
\   
(x + x )
∫e dx + ∫ e
−x+x
= dx
−1 0

0 2
\   
(x + x )
= ∫ 1dx + ∫ e dx
−1 0

2 x2 y 2
22n  y2 = 3(1 - x2) ⇒ + = 1 is an ellipse having foci
= x ]−1
0
+  1 3
2 0

e4 − 1 e4 + 1
(
0, ± 2 )
= 1 + =
2 2 y2 = 2 ⇒ the lines are the respective latus rectum
Integral Calculus  3.139

2 3
x  
3 x

\ Required area = 4 ∫ xdy ∫2  ∫0 f(z)dz  ( t ) dt = ∫  ∫ f(z)dz  2.dt
0  20 
2
1 x
= 4 ∫ 3 − y 2 dy = ∫ f(z)dz.2 (3 − 2)
0 3 0
2 x
4 y 3 −1  y  
= 2
 3 − y + sin   = 2 ∫ f(z)dz
3 2 2  3   0 0

x x
4  3 −1 2 
\ 2 ∫ 1 − (f ' (+1)) = 2 ∫ f(z)dz
2
=  2 + sin 
3  2 3  0 0

2  2 1 − (f '(x)) = f(x)
2
−1 \
= 2 2 + 3sin 
3  3 
2
 dy 
\ 1−   = y
dy  dx 
116. = 3x 2 y 2 + 3x 2 + y 2 + 1
dx
dy
dy ⇒ = ± 1 − y2
(
= 3x 2 + 1 y 2 + 1 ⇒ )( ) y2 + 1
(
= 3x 2 + 1 dx ) dx
dy
i.e., tan-1y = (x3 + x) + C, y(0) = 0 ⇒ C = 0 \ = ±dx
3 1 − y2
tan x + tan x
\ y = tan(x + x3) = sin-1y = ± x +C
1 − tan x.tan x 3
Given that f(0) = 1 ⇒ when x = 0, y = 1
 dy 
117. log  sec x.  = sin x + 2log sin x p
 dx  \ =C
2
dy (sin x + log sin2 x )
secx . =e p 
dx \ y = sin  ± x  = cos x
2
2 
= esinx . e log sin x
= sin2 x.esin x
p p
\ dy = sin2 x.esin x cos xdx \ f   = cos   = 0
2 2
y = ∫ u 2 .eu du
119. Expanding the determinant we get
= u .e - ∫ 2u.e du
2 u u
dy x2 + y 2
=
(
= u 2 eu − 2u.eu + 2eu ) dx 2xy

(
= eu u 2 − 2u + 2 ) Put y = vx ⇒
dy
=v+x
dv
dx dx
y = esinx(sin2x - 2sinx + 2)
du 1 + v 2
when \ x = −v
dx 2v
p sin p   p 2 p 
x= y = e 2   sin  − 2sin + 2 = e 1 + v 2 − 2v 2 1 − v 2
= =
2   2  2  2v 2v
x x dx 2v
\ = du
1 − (f '(t)) dt = 2 ∫ 1 − (f ' (t )) dt ( )
2 2
118.
−x
∫ 0
x 1 − v2
log x = -log (1 - v2) + logC
Since 1 − (f ' (t )) is even function for all t.
2
\ x(1 - v2) = C ⇒ x2 - y2 = Cx
3.140  Integral Calculus

x 124. Let ex = u
120. Given xf(x) + ∫ f(z)dz = xe x
ex dx = du
0
Differentiate with respect to x du  1 1 
I= ∫ (u + 2) (u + 1) = ∫  u + 1 − u + 2  du
x.f '(x) + f(x) + f(x) = (x − 1)e x
u +1  ex + 1 
i.e., xf '(x) + 2f(x) = (x − 1)e x = log + C = log  x + C.
u+2  e + 2 
2  x − 1 x
\ f '(x) + f(x) =  e
x  x  125. f(x) = A . 2x + B
f1 (x) = A . 2x log 2
dy 2y  x − 1  x
i.e., + = e
dx x  x  f1 (1) = A 2 log 2 = 2
2 1
\ A = = log 2 e
\ I .F = e ∫ x = x 2
dx
log 2
i.e., y.x2 = ∫ x ( x − 1) e x 3

∫ (A2 )
x
+ B dx = 7
x2y = ex(x2 - x + 1) + C 0

When x = 1 y = e ⇒ e = e + C ⇒ C = 0  A ⋅ 2x 
3

\ y =
x
( 2
e x − x +1 ) = f(x) 
 log 2
+ Bx  = 7
0
2
x 8A A
+ 3B − =7
2 (
4 − 2 + 1) 3e2 log 2 log 2
\ f(2) = e =
4 4
7A  A 
3B = 7 − = 7 1 − 
sin x − cos x log 2  log 2
121. I = ∫ dx
sin x + cos x
= 7 1 − (log 2 e ) 
2
u = sin x + cos x
 
du = -(sin x – cos x) dx 7
\ B = 1 − (log 2 e )  .
2

\ I = − ∫
du
= − log u 3 
u b
= -log |sin x + cos x| + C. 126. I = ∫ x f ( x ) dx
a
122. ∫ 1 + sin2x dx b

= ∫ sin2 x + cos2 x + 2sin x cos x dx


= ∫ (a + b − x ) f (a + b − x ) dx
a

∫ (sin x + cos x ) dx
2 b
=
= ∫ (a + b − x ) f (x ) dx
= ∫ (sin x + cos x ) dx a

b b
 p
= ∫ (sin x + cos x ) dx, x ∈ 0,  = ∫ (a + b) f (x ) dx − ∫ x f (x ) dx
 2 a a

= sin x – cos x + C. b

\ 2I = ∫ (a + b) f (x ) dx
∫x
3
123. log 2x dx a

b
x4 x4 2 a+b
f ( x ) dx .
2 ∫a
= log 2x − ∫ dx I=
4 4 2x
x4 1 x4
= log 2x − +C 127. Put a = 0, b = 2, c = 3
4 4 4
x4 x4 p p
= log 2x − +C. \I= = .
4 16 2 (2) (5) (3) 60
Integral Calculus  3.141

128. u = x
3
2 d2 y dy
⇒ 2
−5 = −6Ae3x
dx dx
3 12
\ du = x dx −1  d 2 y dy 
2 ⇒ Ae3x =  2 −5 
p 6  dx dx 
2
3 ∫0
\ I = cos2 u du
d2 y dy
(3) – 3 (2) ⇒ −3 = 10Be5x
p
dx 2 dx
2
2 4 1 p p
= × 2 ∫ cos2 u du = × × = . 1  d2 y dy 
3 0
3 2 2 3 Be5x =  2 −3 
10  dx dx 
4 (1) becomes
129. ∫ 7 − f (x )  dx = 7
2 −1 d 2 y 5 dy 1 d2 y 3 dy
y= 2
+ + 2

4 6 dx 6 dx 10 dx 10 dx
\ [7x ]2 − ∫ f ( x ) dx = 7
4
d2 y dy
2 ⇒ 2
−8 + 15y = 0.
dx dx
4 4

7 × 2 − ∫ f ( x ) dx = 7 \ ∫ f ( x ) dx = 7 −1
dy 1 ecos x
2 2
132. + y =
4 dx 1 − x2 1 − x2
Now, ∫ f (x ) dx = 4
−1 Here, P =
1

2 4
1 − x2
\ ∫ f (x ) dx + ∫ f (x ) dx = 4
−1 2
∫ Pdx = sin
−1
x
−1 −1
2 − cos
p x
2
\ I.F = e − sin x
.= e
\ ∫ f (x ) dx + 7 = 4
−1 OR
−1 p
2 \ I.F = e − cos x
, since e 2
is a constant.
∫ f (x ) dx = −3.
−1
 y y
dy  x cos x + y sin x  y
130.
133. The given equation is =
dx  y y
(1, 1) (−1, 1)  y sin x − x cos x  x
1
Put y = Vx
1
dV ( x cos V + Vxsin V ) Vx
V+x =
dx (Vxsin V − x cos V ) x
1 1  1 1 1
Area = 2  × 1 × 1 − × 1 = 2  −  = . 2
 2 3   2 3  3 dV V cos V + V sin V
\ x = -V
dx Vsin V − cos V
131. y = Ae3x + Be5x  — (1) dV 2V cos V
i.e., x =
dy dx Vsin V − cos V
= 3Ae3x + 5Be5x — (2)
dx  Vsin V − cos V dx
2 dV = 2
d y V cos V x
= 9Ae3x + 25Be5x — (3)
dx 2   1 dx
(3) – 5 (2)
 tan V − V  dV = 2 x
3.142  Integral Calculus

Integrating,-log cos V – log V = 2 log x + C1 2 1⋅5


138. I = ∫  x  dx + ∫  x  dx
2 2
log cos V + log V + 2 log x = -C1
1 2
⇒ log (V cos V . x2) = log C (say)
2 1⋅5
y
Vx2 cos V = C ⇒ xy cos = C.
x
= ∫ 1 dx + ∫ 2 dx
1 2

134.
dy
= x ( y + 1) + ( y + 1)
= 2 − 1 + 2 1.5 − 2 ( )
dx
= 2 −1+3−2 2 = 2 − 2 .
dy dy
i.e., = ( x + 1) ( y + 1) ⇒ = ( x + 1) dx 1
dx y +1 139. I1 = ∫ x m (1 − x ) dx
n

0
x2
Integrating, log ( y + 1) = + x + C.
∫ (1 − x ) 1 − (1 − x ) dx
m n
2 =

135. y2 + x = 1 + my  — (1) = ∫ x n (1 − x ) dx = I2 .
m

dy
Differentiating both sides w.r.t. x, where, y1 ≡ p
dx 2

∫ log (tan x ) dx
n
140. I =
1
⇒ 2yy1 + 1 = my1 ⇒ m = 2y + 0
y1 p
2
 p 
∫ log  tan
n
 1 =  2 − x   dx
\ (1) becomes, y2 + x = 1 +  2y +  y 0

 y1 
p
2
y
∫ log cot
n
y + x – 1 = 2y +
2 2 = x dx
y1 0

p
y 2
 1 
x – y2 – 1 =
y1
= ∫ log  tan
0
n  dx
x
\ y = y1 (x – y2 – 1). p
2
dy = − ∫ log tan n x dx = − I .
i.e., y = (x - y - 1) 2
dx 0

\ I = 0.
sin x dx dx
136. I = ∫ 3sin x − 4sin 3
x
= ∫ 3cos2 x − sin2 x p
2
cos8 x dx
= ∫
sec xdx 2
1  3 + tan x  141. I = ∫ sin 8
x + cos8 x
2
3 − tan x
= log  +C 0
2 3  3 − tan x  p
2
sin8 x dx
 2 sin2x 
Also, I = ∫ sin 8
x + cos8 x
∫e
x
137.  1 + cos2x + 1 + cos2x  dx 0

p
2
p p
 1  \ 2I = ∫ 1 dx = ⇒ I= .
= ∫ e x  2 + tan x  dx 2 4
 cos x  0


(
= ∫ e x tan x + sec2 x dx ) p
2
2sin x + 3cos x
= ∫ e  f(x) + f (x) dx
x 1 142. I = ∫
0
sin x + cos x
dx

where f ( x ) = tan x
p
2
2cos x + 3sin x
x
∴ I = e tan x + C
I= ∫
0
sin x + cos x
dx
Integral Calculus  3.143

p p p
2
5 (sin x + cos x ) 4 4
\ 2I = ∫ sin x + cos x
dx = ∫ tan q sec q dq −
6
( 2
) ∫ tan
6
q dq
0 0 0

p p
2 4
p
= 5 ∫ dx = 5 [ x ]0
p
∫ tan
6
2
=5 \ I8 + I6 = q sec2 q dq
0
2 0
1
p 1
 u7  1
\ I = 5 . = ∫ u 6 du =   = .
4 7
 0 7
0

143.
(12, 12) C p
2
y = 12 x
∫x
8
146. I8 = sin x dx
A 0

p
B p 2
=  x (− cos x ) +
8 2
∫ (cos x ) . 8x
7
dx
0
0
O x=3 x = 12
D p
2
= 8 ∫ x 7 cos x dx
Total area OACBO 0
12
 x  2
 p

= ∫  12 x −
12 
dx p 2
= 8   x 7 sin x  2 − ∫ (sin x ) 7x dx 
6

0
 0 
12  0

2 3 x3 
= 12 x 2 −  = 48 p
3 36  0   p 7  2
= 8     − 56 ∫ x 6 sin x dx
 x2  45
3
  2   0
\ Area OABO = ∫  12 x −  dx =
0
12  4 7
p
= 8   − 56I6
\ Ratio required 2
45 7 7
4 45 p p
= = \ I8 + 56I6 = 8   = .
48 − 45 192 − 45 2 16
4
45 15
= = . x2
t 2 − 5t + 4
147 49 147. f(x) = ∫0 2 + et dt
x+2
144. cos q = x 4 − 5x 2 + 4
5 ⇒ f ’(x) = 2
× 2x
y −1 2 + ex
sin q =
4 = 0, when x = 0, ± 1, ± 2.
( x + 2) ( y − 1) \ f(x) has extremum at these 5 points.
2 2

\ + = 1 \ a = 5, b = 4
52 42 148. y = aex + be2x + ce-3x
\ Area = p ab = 20p . y1 = aex + 2be2x – 3ce-3x
p y2 = aex + 4be2x + 9ce-3x
4
y3 = aex + 8 be2x - 27ce-3x
∫ tan
8
145. I8 = q dq
0
= 7 (aex +2 be2x - 3ce-3x)
p
4
- 6 be2x - 6 ce-3x -6 aex = 7 y1 - 6 y
= ∫ tan q sec q − 1 dq
6 2
( ) \ y3 - 7 y1 + 6y = 0
0
3.144  Integral Calculus

149. The given equation is 2x − 10y 3 ( ) dy +y =0 =


ex ex
∫ x + 5 ∫ x + 5 2 dx
dx −
dx ( )
dx
or y + 2x − 10y 3 = 0 1 ex
dy = ∫ x +5 d( )
e x
− ∫ x + 5 2 dx
dx 2 ( )
+ x = 10y 2
dy y ex ex ex
= +∫ −∫ dx on integrating by
x +5 ( x + 5) ( x + 5)
2 2
2
P=
y parts
1 ex
∫ Pdy = 2 ∫ y dy = 2log y = log y2 =
x +5
+C

I.F = e ∫
Pdy 2
= e log y Statement 1 is true
I.F = y2. 152. Statement 2 is true
2 2
dy x +y a
150. =
dx 2xy
But ∫ f(x)dx if
−a
f(x) is odd is zero

2 2 2
dV x + V x 1
V+x =
∫ sin
15
dx 2x Vx \ x dx = 0
−1

dV 1 + V 2 1 + V 2 − 2V 2 \ Statement 1 is false
x = −V =
dx 2V 2V
153. Statement 2 is true
2V dx dy y y
dV = = − tan  
1 − V2 x dx x x
2V dx
Integrating, ∫ 2
dV = − ∫ Put y = vx
V −1 x
dv
log (V2 – 1) = -log x + loc C v+x = v − tan v
dx
x (V2 – 1) = C dv
x = − tan v
 y2  dx
x  2 − 1 = C
x  dv dx
y2 – x2 = Cx  ----------- (1) ∫ tan v = −∫
x

Since (1) passes through (2, 1) log sin v = -logx + logC
1 – 4 = 2C y
−3 xsin   = C
C= x
2 Statement 1 is true and follows from statement 2
−3
y2 – x2 = x 154.
2 y2= 4x
y A
2 (x2 – y2) = 3x.

151. Statement 2 is true


Consider statement 1 x
O
e x ( x + 5 − 1)
x=1
e x (x + 4)
∫ dx = ∫ x + 5 2 dx
(x + 5)2 ( )
Integral Calculus  3.145

dy dy 2 Differentiating the equation ,


We have 2y =4⇒ =
dx dx y dy
6y = 2x(x - 1) + (x - 1)2 = (x - 1)[3x - 1]
dx
2
 dy  4 4 x +1 2
1+   = 1+ 2 = 1+ =  dy  (x − 1)2 (3x − 1)2
 dx  y 4x x 1+   = 1+
 dx  36y 2
1
x +1
Arc length OA = ∫ x
dx
= 1+
(x − 1)2 (3x − 1)2
= 1 +
(3x − 1)2
0
12x(x − 1)2 12x
Putting x = tan q ⇒ dx = 2tan q sec2 q dq
2

(3x − 1)2 + 12x (3x + 1)2


x +1 sec q = =
∫ ∫ tan q × 2tan q sec qdq
2
dx = 12x 12x
x
Length of the loop
= 2 ∫ sec3 qdq 1 1
(3x + 1) 1  1 
= 2∫ dx = ∫ 3 x+  dx
 sec q tan q 1  0 12x 3 
0 x
= 2 + log(sec q + tan q)
 2 2  1
1  2 23 
p =  3 × x + 2 x
when x = 0, q = 0 and when x = 1, q = 3 3 0
4
1 4
Therefore, arc length = [2 + 2] =
p 3 3
= [sec q tan q + log(sec q + tan q)]04
157. We have,


= 2 + log ( 2 +1 ) dy
= x + 1+
y −3
dx x+ 1
4 23 dy 4 3 1
155. y = x ⇒ = × x2 = 2 x Let y - 3 = y
3 dx 3 2
dy dy
 dy 
2 =
1 +   = 1 + 4x dx dx
 dx 
dy 1
− y = x +1
dx ( x +1)
20
20
2 1  3
Arc length = ∫
0
1 + 4xdx =  × (1 + 4x) 
3 4 0
2
Equation is linear, general solution is
1 23 728 364 y
= (81 − 1) = = units =x+c
6 6 3
(x + 1)
y - 3 = (x + 1)(x + c)
156. y
y(0) = 0 gives c = -3
y - 3 = (x + 1)(x - 3)
A = x2 - 2x - 3
x
O 1 ⇒ solution curve is y = x2 - 2x

solution
3y2 = x(x - 1)2 curve
The curve passes through the origin. It is symmetrical
0 2
about x axis It intersects the x-axis at x = 0 and x = 1
Length of the loop = 2 × length of the arc OA of the
curve above the x - axis
3.146  Integral Calculus

3 0 2a a 2a
Area =
∫= ∫ (x ) ∫ f(x)g(x)dx = ∫ f(x)g(x)dx + ∫ f(x)g(x)dx
2
− 2x dx + \
−1 −1 a 0 a
2 3

∫ (2x − x ) dx + ∫ ( x )
2a
2 2
− 2x dx
0 2
= ∫ f(x)g(x)dx
0

4 4 4 2a
= + + =4
3 3 3 = ∫ f(2a − x)g(2a − x)dx
0
1 1

∫ xy(x)dx = ∫ x (x )
2
− 2x dx 2a

−1 −1
= - ∫ f(x)g(x)dx
0
1

= -2 ∫ x dx 2 2a

−1
∫ f(x)g(x)dx
a
=0
1
4
= -4 ∫ x 2 dx = − Also
0
3 a a

158. (d) is true ∫ f(x)g(x)dx = ∫ f(x)d(x).


0 0
since it is a standard result a a

Let (b) be true


∫ g(x)dx ; since ∫ g(x)dx = 0
0 0
Then, f(2a - x) = f(x) as g(a - x) = -g(x)
2a a

⇒ ∫ f(x)dx = 2∫ f(x)dx 3 2 3

∫ xe dx = ∫ xe dx + ∫ xe
|x| + |x − 2| |x| + |x − 2| |x| + |x − 2|
0 0 160. (a) dx
Let (c) be true 0 0 2

f(x + a) = f(x) 2 3

2a = ∫ xe x + 2 − x dx + ∫ xe x + x − 2 dx
Put x = a + y in ∫ f(x)dx
a
0

2 3
2

2a a = ∫ xe2 dx + ∫ xe2x − 2 dx
∫ f(x)dx = ∫ F(y + a)dy
a 0
0 2

2 3
 x2   x.e2x − 2  3 2x − 2
e
= e2  2  +  2  − ∫ 2 dx
a

= ∫ f(x + a)dx  0  2 2
0
3e 4 2e2 1 4

a

= ∫ f(x)dx
= 2e2 +
2

2
− . e − e2
4
( )
0

⇒ (c) is true 3e 4 1 4 1 2
= e2 + − e + e
2 4 4
a a

159. ∫ f(x)g(x)dx = ∫ f (a − x ) g (a − x ) dx =
5e2 2
e +1( )
0 0 4
a 3
= - ∫ f(x)g(x)dx ∫ e [ x ]dx
x
(b)
0 0
a a 1 2 3
\ ∫ f(x)g(x)dx + ∫ f(x)g(x)dx = 0 = ∫ e x [ x ] dx + ∫ e x [ x ] dx + ∫ e x [ x ] dx
0 0 0 1 2

a 2 3

⇒ ∫ f(x)g(x)dx =0 = 0 + ∫ e x .dx + ∫ e x .2dx


0 1 2
Integral Calculus  3.147

Additional Practice Exercise


( ) ( )
2 3
= ex + 2 ex
1 2
p
= e2 - e + 2e3 - 2e2
161. I = ∫ cos2x log sin xdx
= 2e3 - e2 - e 0

4 p

∫ | x − x − 6 | e dx = e I
2 2 2
(c) 2

−3
= 2 ∫ cos2x log sin xdx . Since (f(2a - x) = f(x))
0
−2 3
p
∫| x − x − 6 |dx + ∫ | x 2 − x − 6 | dx
2
I= 2
−3 −2 = ∫ (log sin x)d(sin2x)
4 0

 + ∫ | x − x − 6 | dx
2
p
2
p
1
= [sin2x log sin x ] − ∫ sin2x ×
3
2 × cos xdx
Since x - x - 6 < 0 in (-2, 3)
2 0
0
sin x
−2 3

∫ (x )
p
\ I = 2
− x − 6 dx − ∫ (x 2 − x − 6)dx 2
−3 −2 = − lim(sin2x log sin x) − 2 ∫ cos2 xdx
x →0
4 0

 + ∫ (x − x − 6)dx
2
p
3
= − lim[(2cos x) × sin x × log sin x] −
x →0 2
−2 3
 x3 x2   x2 x3  log sin x p
=  − − 6x  + 6x + −  = − lim − — (1)
3 2  −3  2 3  −2 cosecx
x →0 2
log sin x  ∞
Now, lim  = ∞ 
4
 x3 x2  x → 0 cosecx
 + − − 6x 
3 2 3 cot x
= lim = lim(− sin x) = 0
17 125 17 53 x →0 − cosecx cot x x → 0
= + + =
6 6 6 2 −p
Hence, the value of the definite integral =
4 2
53 2
∫x
2
\ − x − 6 e2 dx = e
−3
2 x
dt
2
162. L.H.S = ∫
log 2 et − 1
∫ xe
|x| [x] {x}
(d) .e .e dx −t
x
−2 x
e 2 dt   −t 
2 2
= ∫ 2
=  −2sin −1  e 2  
   log 2
|x|+[x] + {x}  −2t  
∫ xe ∫ xe
|x|+ x log 2
= dx = dx 1 − e 
−2 −2  

0 2
 −x   log 1

∫ xe dx + ∫ xe
−x+x x+x
= dx = −2sin −1  e 2  + 2sin −1  e 2

−2 0    

0 2  −x  p
= −2sin −1  e 2  + 2  
∫ xdx + ∫ xe
2x
= dx 4
 
−2 0
Given :
0 2
 x2   e2x   −x  p p
=   +  (2x − 1) −2sin −1  e 2  + =
 2  −2  4 0   2 6

3e 4
1 3e − 7 4  − x  −p
= −2 + + = −2sin −1  e 2  =
4 4 4   3

3.148  Integral Calculus

 −x  p p 1 1 x n −1
4(n + 1) n(n + 1) (n + 1) ∫ (1 + x 2 )
sin −1  e 2  = = − + dx  — (2)
  6
−x 1
1 −x x n +1 p
⇒ e 2 = ⇒ = − log 2 From (1), ∫ 2
dx = − (n + 1)In
2 2 0 (1 + x )
4
x = 2log 2 = log 4 Replacing (n + 1) by (n - 1) in the above,
1
x + x +12
x n −1 p
163. Let f(x) =
x2 − x + 1 ∫ (1 + x ) dx =
0
2
4
− (n − 1)In − 2

f(x) is continuous in [-3, 4]. Substituting in (2),


Therefore, it attains a maximum and minimum in p 1 1
this interval. If M and m represent the maximum and In = − +
4(n + 1) n(n + 1) (n + 1)
minimum of f(x) in [-3, 4].
p 
We have  4 − (n − 1)In − 2 
 
m[4 - (-3)] < I < M [4 - (-3)] — (1) p 1 p
⇒ (n + 1)In = − + − (n − 1)In − 2
To find m and M 4 n 4
(x 2 − x + 1)(2x + 1) − (x 2 + x + 1)(2x − 1) p 1
f '(x) = ⇒ (n + 1)In + (n − 1)In − 2 = −
(x 2 − x + 1)2 2 n
(2 − 2x 2 ) p 1
= Putting n = 6, 7I6 + 5I4 = −
(x 2 − x + 1)2 2 6
f ''(x) = p 1
n = 4 → 5I4 + 3I2 = −
(x 2 − x + 1)2 (−4x) − (2 − 2x 2 ) × 2 4

(2)(x 2 − x + 1)(2x − 1) p 1
n = 2 → 3I2 + I0 = −
2 4 2 2
(x − x + 1)
1 1
f ’(x) = 0 ⇒ x = ±I ⇒ f ’’(1) < 0, f ’’(-1) > 0 Now, I0 = ∫ tan −1 xdx = ∫ tan −1 xd(x)
f(x) is maximum at x = +1 and minimum at 0 0

x = -1. 1
x
1+1+1 1−1+1 1 = (x tan −1 x)10 − ∫ (1 + x ) dx
2
Therefore, M = = 3,m = = 0
1 1+1+1 3 1
7 p 1  p 1
Substituting in (1) we obtain, < I < 21 = − log(1 + x 2 ) = − log 2
3 4  2 0 4 2
1 p 1 p 1
⇒ 3I2 = − − + log 2
164. In = ∫ x n tan −1 xdx 2 2 4 2
0
p 1 1
1
 x n +1  = − + log 2
= ∫ tan −1 xd  4 2 2
 n + 1  p 1 p 1 1
0 
⇒ 5I4 = − −  − + log 2 
 x tan
1 1 2 4  4 2 2 
n +1 −1
x x n +1
=  −∫ dx — (1)
(p + 1) 1
 (n + 1) 0 0 (n + 1)(1 + x2 )
 = − log 2
1
4 2
p 1 x n −1 (x 2 + 1 − 1)
4(n + 1) (n + 1) ∫0
= − dx p 1  (p + 1) 1 
1 + x2 ⇒ 7I6 = − −  − log 2
2 6  4 2 
1 1
p 1 1 x n −1dx p 1 1 1
= − ∫
4(n + 1) (n + 1) 0
x n −1dx + ∫
(n + 1) 0 1 + x 2 = − − + log 2
4 6 4 2
Integral Calculus  3.149

p 5 1 ⇒ f ' (x) = 25 − [f(x)]2


= − + log 2
4 12 2
f ’(x) = 0 when f(x) = ± 5
p 5 1
⇒ I6 = − + log 2 when f(x) = 5, f ”(x) = - f(x) = - 5 < 0
28 84 14
⇒ Maximum value of f(x) = 5.
 h 2 (c2 + h 2 ) h 3  ph 2
165. = p  − = (3c + h 2 ) 167. y
 2h 3 6

1 − p16 1 − p16
= (1 − p)16 =
16(1 − p) 16(1 − p) x
O
x=a x = 2a
a a

Now, using ∫ f(x)dx = ∫ f(a − x)dx ,


0 0

1 1

∫[p + (1 − p)x] dx = ∫ [p + (1 − p)(1 − x)]15 dx


15

0 0 x(x − a)2
y2 =
1
2a − x
= ∫ [px + (1 − x)]15 dx
The curve passes through the origin.
0

1 It is symmetrical about the x - axis. It intersects


 15 
= ∫  ∑ 15 C m (px)m (1 − x)15 − m  dx the x - axis at x = 0 and x = a. As x → 2a, y → ∞
0 m =0  ⇒ x = 2a is an asymptote of the curve. No part of the
15 1
curve lies to the right of x = 2a and no part of the curve
= ∑∫ 15
C m (px)m (1 − x)15 − m dx lies to the left of the y - axis.
m =0 0 a

Area of the loop of the curve = 2 ∫ ydx


15  1

= ∑ ( 15
C m )pm ∫ x m (1 − x)15 − m dx  0

m =0  0  a
x
(1 − p ) 16
= ∫ 2a − x
(x − a)dx  — (1)
This is already obtained as 0

16(1 − p) Put x = 2a sin2 q


2p dx = 2a × 2 sin q cos q dq
Hence, = p ∫ y 2 dx p
0 When x = 0, q = 0 and when x = a, q =
4
1
(1 + p + p2 + p3 + .... + p15 )
= x sin q
16 (x − a)dx =
Equating the coefficient of pm on both sides, 2a − x cos q
1
(2a sin2 q − a) × 4a sin q cos qdq
1
15
C m ∫ x m (1 − x)15 − m dx = = 4a2 (2 sin2 q - 1) sin2 q dq
16
0
(1 − cos2q)
1 = 4a 2 (cos2q) dq
1 2
∫x (1 − x)15 − m dx =
m
⇒ 15
0 16 × Cm = 2a2 [cos 2q - cos2 2q]dq
 1 + cos 4q 
166. We have f ’’(x) = g’(x) = -f(x) = 2a 2 cos2q −  dq
 2 
⇒ 2f ’’(x) f ’’(x) = -2f(x) f ’(x)
Substituting in (1)
⇒ [f (x)] = -[f(x)] + C, on integration
’ 2 2
p
When x = 0, [f ’(0)]2 = -[f(0)]2 + C 4
1 1 
Area = 4a 2 ∫  cos2q − − cos 4q  dq
⇒ C = [g(0)]2 + [f(0)]2 = 25  2 2 
0
3.150  Integral Calculus

p p p
 sin2q q sin 4q  4 2 2
= 4a  2
− − Required area = 2 ∫ xdy = 2 ∫ sin ydy
 2 2 8  0 0 0
2 2
 1 p  4a a p
= 4a 2  −  = (4 − p) = (4 − p)
= 2[− cos y]0 = 2
2
 2 8  8 2

168. Circle : (x - a)2 + y2 = a2 171. \ /


Parabola : y2 = ax $
They intersect at x = 0 and x = a
Required area = Area of quadrant of circle - Area I /
a
2 3 2
Area I = ∫ a x dx = a a 2 = a 2 [ 
0
3 3 2

a 2 p 2a 2 (3p − 8)a 2
\ Area = − =
4 3 12 The parabola is y2 - 9y = x - 21
2
169. y  9 81 3
⇒  y −  = x − 21 + =x−
 2 4 4

3 9
Vertex of the parabola is at  , 
x 4 2
O (a, 0) LL’ is the latus rectum.
(−a, 0)
3 9
We shift the origin to  , 
4 2
a2y2 = x2(a2 - x2) 3 9
x=X+ ,y = Y +
The curve is symmetrical about both x and y 4 2
axes. It passes through the origin. No part of the
Equation of the parabola in the new coordinate
curve lies beyond x = ±a. Curve intersects the
system is Y2 = X
x - axis at x = -a, 0, +a.
Since the area is invariant,
Total area of the curve
1
a a
x a2 − x2 4
= 4 ∫ ydx = 4 ∫ dx Required area = 2 ∫ XdX
a
0 0 0
a
4  −2 2  3 1
=  (a − x )  2 2
2  23  4 1 4
1
a 3 × 2 0 =2×  X  = × 3 =
3  0 3 6
42
4 4a 2
= × a3 =
3a 3 172. y
170. y B

π
y=
2
y = sin–1x
O x
x O A

−π
y=
2
Integral Calculus  3.151

x = cos 4 q y


 , 0 ≤ q < 2p gives the parametric form d 
y = sin 4 q  x
⇒ = −dx
representation of the curve. y2
1+ 2
p x
q corresponding to A is and corresponding to B is 0
2 y
dx Integrating tan −1   = − x + C
Required area = ∫ ydx = ∫ y .dq x
dq
0 y
= ∫ (sin 4 q) × 4cos3 q(− sin q)dq ⇒ x + tan −1   = C
x
p
2
p (ii) (x3 sin3 y - y2 cos x)dx +
2
4×4×2×2 1  3x 4 
= 4 ∫ sin5 q cos 3 qdq = = 2

0
8×6×4×2 6  4 cos y sin y − 2y sin x  dy = 0

173. y  3x 4 
⇒  x 3 sin3 ydx + cos y sin2 ydy 
 4 
y = (log x) 2 P(e,1) - (y2 cos x dx + 2y sin x dy) = 0

x  x4 
O (1, 0) ⇒ d  sin3 y  - d(y2 sin x) = 0
 4 
y = log x
Integrating, the general solution is
x 4 sin3 y
− y 2 sin x = C
Points of intersection of the curves are obtained from 4
log x = (log x)2
dy
log x = 0 or 1 ⇒ x = 1, e (iii) (sin y) = (cos x) [2 cos y - sin2 x]
dx
In the interval 1 < x < e
dy
log x > (log x)2 ⇒ sin y - (2cos x)cos y = -cos x sin2 x
e dx
Required area = ∫ [log x − (log x)2 ]dx dy dY
1 Let - cos y = Y⇒ sin y =
e
dx dx
= (x log x − x)1e − ∫ (log x)2 d[x] Substituting in the given equation,
1
dY
e
1 + (2cos x)Y = -cos x sin2 x
= 1 − [x(log x)2 ]1e + ∫ x × (2log x) × dx dx
1
x
which is linear where x is the independent vari-
= (1 − e) + 2(x log x − x)1e able and Y is the dependent variable
= (1 - e) + 2(1) = (3 - e) General solution is
xdy  y  Ye2 sin x = C − ∫ (cos xsin2 x)e2 sin x dx — (1)
174. (i) = 2 − 1 dx 
x2 + y 2  x + y 2

For evaluating the right side integral we proceed
xdy − ydx
⇒ = −dx as follows
x2 + y 2
Set sin x = t
2 y cos x dx = dt
x d 
x
⇒ = −dx
∫ (cos xsin x)e2 sin x dx = ∫ t 2 e2t dt
2
x2 + y 2
3.152  Integral Calculus

General solution is,


2 e   e2t   e2t 
2t

= t  2  − 2t  4  + 2  8  x x
1− x 2
 x  1− x 2
ye =C+ − 1 e
 1− x 2

 t2 t 1 
= e2t  − +  −x
 2 2 4 1− x 2
x
⇒ y = Ce + −1
Substituting in (1), general solution is 1 − x2
 sin2 x 1 1 175. Let u = y - x
(-cos y)e2sinx = C - (e2sinx)  − sin x + 
 2 2 4
du dy
= −1
sin2 x 1 1 dx dx
⇒ cos y = −Ce −2 sin x + − sin x +
2 2 4 dy
Substituting for in the given equation,
(iv) (2x - 3y - 7)xdx - (3x + 2y - 8)y dy = 0
2 2 2 2 dx
(2x3 dx - 7xdx - 2y3dy + 8ydy) du du
1+ = 1 - ux - u2 x3 ⇒ = -xu - x3u2
- (3y xdx + 3x ydy) = 0
2 2 dx dx
 x 4 7x 2 y 4  1 du 1
⇒ d  − − + 4y 2  −
2
+ x × = −x3
 2 2 2  u dx u
3xy(ydx + xdy) = 0 −1 du  −1 
⇒ + x   = x3  — (1)
u 2 dx u
x 7x 4
y  2 4
⇒ d  − − + 4y 2  − 3xyd(xy) = 0 1 dY −1 du
 2 2 2  Set Y = ⇒ = 2
u dx u dx
Integrating, the general solution is,
(1) reduces to the linear form
(x 4 − y 4 ) 7x 2 3x 2 y 2
+ 4y 2 − − =C dY
2 2 2 − xY = x 3
⇒ x4 - y4 + 8y2 - 7x2 - 3x2y2 = C’ dx
General solution is given by
dy y x
(v) + 3
= − x2 − x2
dx (1 − x 2 )2
(1 − x ) 2 2
Ye 2
= C + ∫x e 3 2
dx
1 x  −x 
2

P= 3
,Q = = C + ∫ x 2 d  −e 2 
(1 − x 2 )2
(1 − x ) 2 2
 
dx cos qdq − x2 − x2
∫ Pdx = ∫ 3
= ∫ cos3 q =C−xe 2 2
+ ∫ 2xe 2
dx
(1 − x 2 ) 2
− x2 − x2
where we have set x = sin q = C − x2e 2
− 2e 2

x
= ∫ sec2 qdq = tan q = 1 −x −x 2 2

1 − x2 ⇒ = C − x 2 e 2 − 2e 2
y−x
x
∫ Pdx x Since y(1) = 2
∫ Qe dx = ∫ (1 − x 2 )2 × e 1− x 2
dx
−1 −1
3
x 1 1 = C − e 2 − 2e 2 ⇒ C = 1 +
Put = t⇒ 3
dx = dt e
1 − x2 (1 − x ) 2 2
Solution of the initial value problem is
 
x 2 2
x
 x  3  x2
e 2 = (y − x)  1 + e − x 2 − 2
∫ = ∫ te dt = te - e =  1 − x 2 − 1 e 1− x 2
t
t t

  e 
Integral Calculus  3.153

p p
y dy y2
176. − =  — (1) 4 4
178. (i) ∫ sin q (1 + tan q) dq = ∫ sin q sec
2 2
x dx x x qdq
sin 4   + cos 4  
y y 0 0
p
4
x dt x  y dy 
Let = t . We have = 2 − = ∫ sec q tan qdq
y dx y  x dx  0

(1) becomes p
= [sec q]04 = 2 −1
dt x
=
1 x − (x + k )
k k
dx sin 4 t + cos 4 t dx
(ii) ∫ x+ x+k
=−
k ∫0 x + x + k
dx
⇒ [(sin2 t + cos2 t)2 - 2sin2 tcos2 t]dt = xdx 0

k

⇒ 1 −
sin2 2t 
2 
 dt = xdx = −
1
k ∫0
( x − x + k dx )

k
⇒ (3 + cos 4t)dt = 4xdx −1 2  23 3
=  x − ( x + k )2 
sin 4t C k 3 0
Integrating, 3t + = 2x 2 +
4 4
x  4x 
=
−1 2 
.
k 3
(
K k − 2K. 2k − 0 − k k 
 ) ( )
⇒ 12   + sin   = 8x 2 + C
y  y 
=
−1 2
. 2k k 1 − 2
k 3
( )

177. Let In = ∫ x n e − x cos x dx


0
=
4
3
k 2 − 1 = 2 − 1 (given) ( )
∞ 3 9
Jn = ∫ x n e − x sin x dx ⇒ k = ⇒k =
4 16
0
x

179. f2(x) = ∫ f (t ) g (t )dt



n − (1− i )
In + iJn = ∫ x e dx
0 0

Differentiate with respect to x


n (1 + i ) ∞ − x n −1 ix

xne ( ) 
− 1− i x
g (x )
2 ∫0
= −  + e .x e dx
− (1 − i )  0 2f(x) f ’(x) = f(x) g(x) ⇒ f ’(x) = or f(x) = 0
2
∞ 1 1
= 0 +
n
(1 + i ) ∫ e− x .x n −1eix dx f ’(x) = g ( x ) ⇒ f ( x ) = ∫ g ( x ) dx
2 2 2
0
sin x
Compare real and imaginary parts if g(x) =
2 + cos x
n
In = (I - J )  — (1) 1 sin x
2 n-1 n - 1 f(x) = ∫ dx
2 2 + cos x
n 1
Jn = (I + J )  — (2) = − log (2 + cos x ) + log c
2 n-1 n - 1 2
(1) + (2) ⇒ In + Jn = nIn-1 ⇒ f(x) = - log c. 2 + cos x ( )
In - nIn-1 = - Jn
\ In-1 - (n - 1) In-2 = - Jn-1 1 1

180. g’ (x) = 3+ ∫ z 2 g (z ) dz + 2x ∫ z g (z ) dz
nIn-1 - n(n -1) In-2 = - n Jn-1 0 0

= 2 In - nIn-1 from (1) \ g (x) will be in the form of ax2 +bx, as


\ 2In - 2n In-1 + n(n - 1) In-2 = 0 g(x) = 0
3.154  Integral Calculus

1 1 p
2ax+b = 3+ ∫ (az ) (
+ bz 3 dz + 2x ∫ az 3 + bz 2 dz ) sin2 (n + 1) x − sin2 nx
4 2
(ii) Vn +1 − Vn =
0 0 ∫
0 sin2 x
dx
1 1
 az 5 bz 4   az 4 bz 3  p
= 3+  +  + 2x  +  2
sin (2n + 1) x p
 5 4 0  4 3 0 = ∫ dx =
0
sin x 2
a b a b
= 3+  +  + 2x  +  p
5 4 4 3 V0 = 0, V1 =
2
a b 3a b p np
2a = 2  +  i.e., = ⇒ V2 = 2   …….. Vn =
4 3 4 3 2 2
⇒ 9a = 4b  — (1) 1

a b 180 + 12a + 15b  1  x log t


b = 3+ + ⇒ b = 182. f   = ∫ dt
5 4 60 x 11+ t
12a-45b+180= 0  — (2) 1 −1
Put t = ⇒ dt = 2 dy
4a - 15b + 60 = 0 y y
16b 1
− 15b + 60 = 0 , using (1) t= ⇒y=x
9 x
540 t=1⇒y=1
⇒ - 119b + 540 = 0 ⇒ b =
119 x x
1 − log y  −1  log y dy

4 540 240
a= × =
f =
x ∫1 1  y 2  dy = ∫1 1 + y . y
9 119 119 1+
y
1
\ g (x) =
119
(
240x 2 + 540x ) x
log t dt
= ∫ 1 + t. t
Roots of g (x) = 0 1

−9 x x
⇒ 4x2 + 9x = 0 ⇒ x = 0, x = 1 log t  1 log t
4 f (x) +f   = ∫ 1 +  dt = ∫ dt
 x  1 1 + t  t 1
t
p
x
sin (2n + 1) x − sin (2n − 1) x  (log t )2  (log x )
2 2

181. (i) ∫
0
sin x
dx = 
 2 
 =
2
1
p

(log x )
2
2
2cos2nx.sin x
= ∫
0
sin x
dx g (x) =
2

 1  h ( x )
2
p
 −2sin2nx  2 and f (x) + f   =
=   =0 x 2
 2n 0
p p
⇒ h (x) = log (x)
2
sin (2n + 1) x 2
sin (2n − 1) x 1
∫ (log x ) dx
2
⇒ ∫ sin x
dx = ∫ sin x
dx = A (ii) g (x) dx =
2 ∫
0 0

1 1
S n +1 = S n = S n −1 = − − − − (log x ) .x − ∫ 2log x xdx 
2
=
p 2  x 
2
p x
S3 = S1 = ∫ dx = =A = (log x ) − ∫ log x dx
2

0
2 2
Integral Calculus  3.155

x Area is max
(log x ) − x (log x − 1) + c
2
=
2 dA
⇒ =0
x 2 da
∫ g (x ) dx = 2 h (x ) − xh ( x ) + x + c  ( 2 2
)
2
1  2a 1 + a − a .2a.2 1 + a
2
( )  = 0

(iii) domain of h (x) : x ∈ (0, ∞) 6 ( ) 
4
 1 + a2 
Domain of g (x): (0, ∞).
( )
⇒ 2a 1 + a 2 1 + a 2 − 2a 2  = 0
183. 
[
\  ⇒ a = ± 1 ⇒ a = 1, as a > 0
< D
\
[
 d2 A
 is -ve for a = 1
da 2
1
(ii) max area = square units
§ § 24
3 ¨ D  D
¨
¨
©  D   D 
[


¨
©
x
(iii) y = x-x2, y = ⇒ The point of intersection is P
2
2
\ [D[

1 1
 2 , 4  besides the origin. Equation of OP is y
x
=
2

D[ [ \
The area enclosed between the parabola y = x - x2
ax2 -x = -y x
and the line y = is
2 2
 1 1 −y
 x − 2a  − 2 = a
1 1
4a 2
 x  x2 x3  2
= ∫  x − x 2 −  dx =  − 
 1 1
2
1  0
 2 4 3 0
 x − 2a  = − a  y − 
4a  1 1 1
= − = which is half the maximum
x 2 16 24 48
y= , y = x − ax 2 area.
a
x
x2 \ y = bisects the max area made by given
⇒ = x − ax 2 2
a
parabolas


(1 + a ) x
2
2
= x ⇒ x = 0, x =
a
184. (i) x = X + 2, y = Y - 1
a 1 + a2 ⇒ 2x + 2y - 2 = 0
a
1+ a 2 ⇒ 2X + 2Y = 0; 3x + y - 5 = 0
 x2 

2
Area A =  x − ax − dx ⇒ 3X + Y =0
0
a 
(ii) dx = dX, dy = dY
a
 x2  1  x 3  1+ a 2 dY 2X + 2Y dY dν
=  − a +   = Let Y = νX; =ν+X
dX 3X + Y dX dX
2  a  3 0
dν 2X + 2νX 2 + 2ν
ν+X = =
a2 a2 + 1 a3 dX 3X + νX 3+ν
= − .
( 2 2
) ( )
3
2 1+a 3a 1 + a 2 Xdν 2 − ν − ν 2
=
dX 3+ν
a2 dX 3+ν
= ⇒ − = .dν
( )
2
6 1 + a2 X (ν − 1)(ν + 2)
3.156  Integral Calculus

1 1
− dX 1  4 1 
⇒ =  − dν 2 2
X 3  ν − 1 ν + 2  ⇒ f(x) = ∫ sin −1 t dt + ∫ cos −1 t − dt
0 0
3 log X = log (ν + 2) − 4 log (ν − 1) + log c 1 1
2
p p  2 p
X3 = c (ν + 2) / (ν − 1)
4
= ∫ 2 .dt =  2 .x 
0 0
=
4
X (Y − X ) c (Y + 2X ) p p
3 4

= \ f(x) = if 0 ≤ x ≤
X 4
X 4 2

(y - x + 3) = c (2x + y - 3)
4 eax eax
187. A = cos bx. − ∫ − bsin bx. .dx
a a
185.
eax b  eax eax 
= cosbx . + sin bx. − ∫ bcos bx. .dx 
a a a a 
B
A 
ax
b2  e .cos bx b
A 1 + 2  = + 2 sin bxeax
 a  a a
O 5 X
7
( )
A a 2 + b2 = eax (a cos bx + bsin bx )
D eax
C A = cos (bx − f)
a 2 + b2
a
where cosf = and
a 2 + b2
(i) z2+ z 2 − 2z. z + 8z + 8z = 0 b
sin f =
a + b2
2
(z − z ) = −8 (z + z )
2

b
⇒ (2i y)2 = -8 (2x), where z = x + iy tan f =
a
⇒ -4y2 = -8 (2x) ⇒ y2 = 4x, which is a parabola.
eax eax
B = sin bx. − ∫ bcos bx. dx
(ii) 5 ≤ x2 + y 2 ≤ 2 3 a a

⇒ x2 +y2 ≥ 5, x2 +y2 ≤ 12 eax b  eax eax 


=sin bx. − cos bx. − ∫ − bsin bx. dx 
a a a a 
z2 + z 2 − 2zz + 8z + 8z ≥ 0
(2iy)2 + 8 (2x) ≥ 0  b2  eax b
B 1 + 2  = .sin bx − 2 eax cos bx
-y2 + 4x≥ 0 ⇒ y2 ≤ 4x.  a  a a
The shaded portion represents the required B(a2 + b2) = eax (a sin bx − bcos bx )
region.
eax
sin2 x cos2 x B = sin (bx − f)
a 2 + b2
186. f(x) = ∫ sin −1 t dt + ∫ cos −1 t dt
0 0
e2ax
f ’(x) = sin–1 (sin x).2sin x cos x + cos–1 (cos x).2cos x (i) A 2 + B2 =
a + b2
2
p
x – sin x = 0 if 0 ≤ x ≤
2 (
⇒ GM of A 2 + B2 and a 2 + b2 is eax) ( )
p B b
⇒ f(x) is constant if 0 ≤ x ≤ (ii) = tan (bx − f) , and = tan (f)
2 A a
p  B   b 
\ f(x) is constant ⇒ f(x) = f   ⇒ tan −1   + tan −1   = bx
4 A a 
Integral Calculus  3.157

p We have
(
188. (i) I(x) = ∫ log 1 − 2x cos q + x 2 dq )  p
0 In+2 < In < In-2 in  0, 
b b
 4
∵ ∫ f (x ) dx = ∫ f (a + b − x ) dx
a a
In+2 +In < 2In < In-2 + In
p 1 1
(
I (x)= ∫ log 1 − 2x cos (p − q) + x 2 dq )
i.e.,
n +1
< 2In <
n −1
0
1
( )
p

= ∫ log 1 − 2 (− x ) cos q + (− x ) dq = I (− x )
2 (iii) I6 = − I4
5
0

(ii) I(x) + I (-x) 1 1 


= − − I2 
p 5  3 
(
= ∫  log 1 − 2x cos q + x 2 + ) 1 1 1 
0
= − +  − I0 

(
log 1 + 2x cos q + x  dq 2
) 5 3 1 
p 1 1 p
= ∫ log  1 + x 2 ( ) ( − 4x 2 cos2 q  dq ) = − +1−
2

  5 3 4
0
p 13 p
= −
(
= ∫ log 1 − 2x 2 cos2q + x 4 dq ) 15 4
0

Let f(q) = 1-2x2 cos2q + x4 190. Put x = acos2 q + b sin2 q


dx = -2acos q sin q + 2bsin q cos q
f(p - q) = 1 − 2x 2 cos2q + x 4
2b b
= 2(b - a)sin q cos q
\ ∫ f ( x ) dx = 2 ∫ f ( x ) dx if f(2b-x) = f(x) (x - a)(b - x) = [a(cos2 q - 1)
0 0
p
+ bsin2 q][b(1 - sin2 q) - acos2 q]
2 = (b - a)2 sin2 qcos2 q
I(x) + I(-x)= 2 ∫ log 1 − 2x 2 cos2q + x 4 dq ( ) dx 2(b − a)sin q cos qdq
0 ∫ (x − a)(b − x) = ∫ (b − a)sin q cos q
Let a = 2q ⇒ da = 2dq
p
q= ⇒a=p = 2 ∫ dq = 2q
2
q=0⇒a=0 p
when x = a, q = 0, and when x = b, q =
p 2
I(x) + I(-x) = ∫ log 1 − 2x 2 cos a + x 4 da ( ) p
0 2
p
p Definite integral = 2 ∫ dq = 2 = p
(
= ∫ log 1 − 2x cos q + x dq 2 4
) 0
2
0
191. p sin3 x is odd while q cos2 x and ‘r’ are even.
I(x) + I(-x) = I(x2)
\ The value depends on q and r only.
p p
4  tan n −1 x  4 p
189. In = ∫ tan n−2
( 2
x sec x − 1 dx =  )
 - In-2
2

0  n − 1  0
192. I = ∫ sin2x log tan x dx
0
1 1
= − I n − 2 ⇒ In + In-2 = p
2
n −1 n −1 p  p 
 p
= ∫ sin2  2 − x  log tan  2 − x  dx
0
In  0  , tann+2 x < tann x < tann-2 x.
 4 p
2
1
In+2 + In = = ∫ sin2x log cot x dx
n +1 0
3.158  Integral Calculus

p
2
195. Put x = a cos2 q + b sin2 q
 1  \ x – a = (b - a) sin2 q
= ∫ sin2x log  dx
 tan x 
0 b - x = (b - a) cos2 q
p
2 dx = (b - a) 2 sin q cos q dq
= − ∫ sin2x log tan x dx = − I
(b − a ) 2sin q cos q dq
0
\ I = ∫ (b − a ) sin q cos q = 2q
\ I = 0.

193. x − a
= 2sin −1  +C
 b − a 

θ 196. Let x + 4 =t
2
1
⇒ dx = 2dt
x+4
1 dt
1
∫ (x + 5) x + 4 dx = 2∫ 1 + t2
tan q =
3 = 2tan −1 ( x + 4 +C)
p
q = radians 197. sin x2 = t ⇒ 2x cos x2dx = dt
6
1 t 3 dt 1 4 sin 4 (x 2 )
\ Area = r 2 q I =∫ = t = +C
2 2 8 8
1 p p 198. t = exlog (sec x + tan x)
= ×4× = .
2 6 3 
dt = e x log(sec x + tan x) +

dy x 4 + 2xy − 1
194. Given = e x sec x(sec x + tan x) 
dx 1 + x2  dx
sec x + tan x 
2xy x4 − 1
= 2
+ = [exsec x + ex log (sec x + tan x)]dx
1+ x 1 + x2
2xy I = ex log (sec x + tan x)
= + x2 − 1
1 + x2 = - ex log (sec x - tan x) + C
dy 2x cos2 x + sin2 x 1 sin x
⇒ − y = x2 − 1
dx 1 + x 2 199. ∫ sin x cos 2 x
dx = ∫
sin x
dx + ∫
cos 2 x
dx
−2x
P= ; Q = x2 – 1 sin x −dt
1 + x2 ∫ cos 2
x
dx = ∫ t2
where t = cos x
1 1
∫ Pdx = log 1 + x 2 ; I.F = 1 + x 2 I = log tan
x
+ sec x + C
2
Solution is y (IF) = ∫ Q (I.F ) dx
x
1 x2 − 1 200. = tan −1 x ∫ dx
y.
(1 + x ) 2
=C+ ∫1 + x 2
dx 1 + x2
1  x 
x2 + 1 − 2 −∫ 2
⋅ ∫ dx  ⋅ dx
1+ x  1+ x
∫ 
2
=C+ dx = C + x - 2 tan x.
-1
1 + x2
1
y(0) = 0 = tan −1 x ⋅ 1 + x 2 − ∫ 1 + x2
dx
0=C+0-0⇒C=0
Solution is y = (x - 2 tan-1x) (1+ x2) (
= tan −1 x ⋅ 1 + x 2 − log x + x 2 + 1 + C )
Integral Calculus  3.159

p
80x 4 72x 3
201. f '(x) = ∫ f ''(x)dx = + + 6x + C 1 4
4 3 = − ∫ (sec2 x − 1)dx
3 0
f ’(0) = c = 2 ⇒ f ’(x) = 20x4 + 24x3 + 6x + 2
1 p
1 p
f(x) = ∫ f '(x)dx =
20x 5 24x 4 6x 2
+ + + 2x + C = − [ tan x − x ]04 = − 1 +
5 4 2 3 3 4
f(0) = 2 ⇒ C = 2 p 2
= −
\ f(x) = 4x5 + 6x4 + 3x2 + 2x + 2 4 3
p p 1
2 205. Differentiating f(x) = 1 - xf(x), hence f(x) =
sin2 x 2
cos2 x 1+ x
202. I = ∫ dx = ∫ dx
0
sin x + cos x 0
sin x + cos x  −1  1
⇒ f   = 2 ; f(2) =
p p 2  3
2
1 1  x p  2
2I = ∫  p
dx =  log tan  +  
2 2 8 0
 1 2
x 2 −  2 +  x + = 0 ⇒ 3x2 - 7x + 2 = 0
0
2 sin  x +   3 3
 4
1  p p p 2
 4x 4 
2
=  log tan  +  − log tan  ∫0
3
206. 4x dx =   = 16 sq.units
2 4 8 8  4 0
p
cot
1 8 =
= log 2 log( 2 + 1) y
2 p
tan
8
p 1 O (2, 0) x
as cot = 2 + 1⇒ I = log( 2 + 1)
8 2
p
2 x=2
∫[sin
−1
203. (cos x) + cos −1 (sin x)]dx
0
207. Let x = tan a , y = tan β
p

p 2
p  1 1  tan a tan b 
= ∫  − x + − x  dx + = k − 
0
 2 2  cos a cos b  cos b cos a 
p
cos β + cos a = k[sin a - sin β]
 2x 2  2 p2 p2 p2
=  px −  = − = a − b
 2 0 2 4 4 cot  = k ⇒ a - β = 2 cot-1 k
 2 
 p tan-1 x - tan-1 y = 2cot-1 k
204. x ∈  0,  ⇒ {tan4 x} = tan4 x, where, { } denotes
 4 1 1 dy
− ⋅ =0
fractional part. 1 + x 2 1 + y 2 dx
 p
Also x - [x] = {x} in  0 ,  dy 1 + y 2
 4 =
dx 1 + x 2
p p p
4 4 4
dy 3x − 4y − 2
∫ {tan x}d{x} = ∫ tan 4 x ⋅ dx = ∫ tan2 x(sec2 x − 1)dx =
4
208.
0 0 0
dx 3x − 4y − 3
p p Put 3x - 4y = t
 tan3 x  4 4 dy dt
 − ∫ tan xdx
2
= 3−4 =
 3 0 0 dx dx

3.160  Integral Calculus

dy dt 2 10
4 =− +3 = ∫ 2dx + ∫ 2 x − 1dx = 2(2 − 1) +
dx dx
1 2
10
t − 2 dt dt t − 2  3
2
⇒ 4 
t − 3  = − dx + 3 ⇒ dx = 3 − 4
 t − 3   2(x − 1) 2
× 
   3 2
dt −t − 1
⇒ = 4  23 3
 4 110
dx t−3 =2+  9 − 12
 = 2 + × 26 =
3  3 3
3−t  4 
dt = dx ⇒ − 1 − dt = dx
1+ t  1+ t  211. Let t = log g(x) - log f(x)
- t + 4log(1 + t) = x  g '(x) f '(x) 
⇒ dt =  −  dx
-3x + 4y + 4 log(3x - 4y + 1) = x  g(x) f(x) 
⇒ 4log(3x - 4y + 1) = 4x - 4y + C t2 1 g(x)  + C
2

I = ∫ t ⋅ dt = + c =  log 
⇒ log(3x - 4y + 1) = x - y + C 2 2  f(x) 
2
cos8x − cos7x 2sin5x 1 f(x) 
209. × =  − log  +C
1 + 2cos5x 2sin5x 2 g(x) 
sin13x − sin3x − sin12x + sin2x
= d
x2
d 2
2(sin5x + sin10x)
dx ∫x
212. sin tdt = sin x 2 ⋅ (x ) − sin x
dx
15x 11x 15x 9x
2sin cos − 2sin cos
= 2 2 2 2 = 2x sin x 2 − sin x
15x 5x
2 ⋅ 2sin cos d
g(x)
2 2 ∫ f(t) dt = f (g (x )).g '(x) − f(h(x))h'(x)
dx h(x)
x
−2sin5xsin
= 2 = −2sin 5x sin x g(x)

5x 2 2 ⇒ ∫ f(t) dt = ∫  f(g(x) g '(x) − f (h ( x )) h' ( x ) dx


2cos h(x)
2
= cos 3x - cos 2x \ ∫ (2x ( ) )
sin x 2 − sin x dx ⇒ f(x) = sin x
I = ∫ (cos3x − cos2x)dx g(x) = x2
sin3x sin2x h(x) = x
= − +C x2
3 2 \ Required integral becomes ∫
x
sin t dt

( )
2
210. x +2 x −1 = 1+ x −1 +2 x −1 p
213. sin −1 x + cos −1 x =
2
= 1+ x −1
2 p 4
∫ p  2sin dx = ∫ sin −1 xdx − x + c
−1
x−
2  p
x −2 x −1 = 1− x −1
x = sin2 q ⇒ dx = sin 2q . dq
∫ sin xdx = ∫ q ⋅ sin2q ⋅ dq
−1
x +2 x −1 + x −2 x −1
= 1+ x −1 +1− x −1 − cos2q  − cos2q 
= q⋅ − ∫1 ⋅  dq
2  2 
1 ≤ x ≤ 2
−q ⋅ cos2q sin2q
= + +c
= 1+ x −1 −1+ x −1 x≥2 2 4
Integral Calculus  3.161

− sin −1 x[1 − 2x] x 1− x 1 1


= + +C 216. af(x) + bf   = − 5  — (1)
2 2 x x

\ I =
2
p
(
(2x − 1)sin −1 x + x 1 − x − x + C ) 1
af   + bf(x) = x − 5 
x
— (2)

214. n → ∞ (1) × a - (2) × b ⇒ (a2 - b2) f(x)


x-n → 0 if x > 1 a
= − 5a − bx + 5b
1 + x −4n x
f(x) = lim =1
n →∞ 1 − x −4n
1 a 
⇒ f(x) = − bx + 5(b − a)
a 2 − b2  x
I= ∫ 1+ x
x
2
(
log x + 1 + x 2 dx ) 

1  bx 2 
∫ f(x)dx = . a log x −
2 
+ 5(b − a)x 
(
= log x + 1 + x 2 )∫ x
1 + x2
dx
2
a −b  2 
2

 x + 1 + x2  ∫ f(x)dx
  1
1 + x2 x
−∫  ∫ dx  dx 1
 x + 1 + x2 1 + x2  =
  a − b2
2

   
 −b 
a log 2 − 2b + 10(b − a) −  2  − 5(b − a)
d x  
1 + x2 =
dx 1 + x2 1  7b 
= a log 2 + − 5a 
I = 1 + x log x + 1 + x 2
( 2
)−
a 2 − b2  2 

1 7b
⇒ k = a log 2 + − 5a
∫ 1 + x2
⋅ 1 + x 2 dx
2

= 1 + x 2 log x + 1 + x 2 − x + C( ) 217.
x= y
x = –y
215. Let P(x) = 0 be a polynomial of degree n (–1, 1) (1, 1)
P(x3) ⇒ polynomial of degree 3n
3n = n + 4 ⇒ n = 2 ⇒ P(x) = ax2 + bx + c
(0, 0)
P(0) = 0 ⇒ c = 0,
P’(x) = 2ax + b
P’(1) = 2a + b = 7  — (1) 1

ax bx 3 2 Required area = 2 ∫ (x − x 2 )dx


∫ P(x)dx = 3
+
2
0

1
1
a b  x2 x3  1 1 1
= 2  −  = 2  −  =
∫0 P(x)dx = 3 + 2 = 1.5 2 3 0 2 3 3
2a + 3b = 9  — (2) 218. y = x - [x] y = - x - [-x]
(1) & (2) ⇒ b = 1, a = 3 ⇒ P(1) = 4 (-2, -1) x - y = -2 x + y = -1
1
1
 P2 (x)  16 − 0 (- 1, 0) x - y = -1 x+y=1
∫0 P(x)P'(x)dx = 
 2 0
 =
2
=8
(1, 2) x - y = 1 x+y=2
3.162  Integral Calculus

2
2
t  
n +1
 2 
n +1

1 221. ∫ =  1 +  = 1 + −1
0   n + 1    n + 1 
1 1
0
1 1 1
lim lim  2
2 
n +1

2 n → ∞ ∫ = n → ∞  1 + − 1
0   n + 1  
−2 −1 0 1 2
 n +1 2
 
  lim2  2  
n → ∞  1 +  − 1 = e2 -1
   n + 1 
 
Graph of the curves have been drawn. The area re-  
quired is the shaded portion.
0, n even
1 1 p
1 − cosnp 
Required area = 4 × ×1 × = 1
2 2
222. ∫ sinnx dx = n
= 2
0  n ,n odd
dy ∞ ∞
2 2 5
219.
dx
− tan2x ⋅ sec2 x ⋅ y = cos 2 x ∑ I (5 ) = ∑ 5
n=0
n

n=0
n
=
1− 1
=
2
5
dy
+ Py = Q
dx 223. ydx - xdy = -xy2 dx
−2tan x y dx − xdy
∫ P ⋅ dx = ∫ 1 − tan 2
x
⋅ sec2 xdx = log(1 − tan2 x)
y2
= -xdx

e∫
Pdx
= 1 − tan2 x x  −x2 
d   = d
y  2 
∫ Pdx 1 − tan2 x sin2x
∫Q ⋅ e dx = ∫ 1 + tan2 x dx = ∫ cos2xdx = 2 x −x2
Integrating, = +c
1 y 2
\ y(1 - tan2x) = sin 2x + c
2 x = 1, y = 2 ⇒ c = 1
p 3 3 Solution curve is
if x = ,y =
6 8 x x2
= 1−
3 3 1 1 3 y 2
1−  = × + c⇒c=0
8  3 2 2 2 − x2
=
\ 2y(1 - tan2 x) = sin 2x 2
2x
220. xdy + ydx = d(xy) y=
y
(2 − x ) 2

xdy − ydx = x 2 d  
x we note that as x → ± 2,y→∞
y y y ⇒ choice (c) is true
x cos d(xy) = y ⋅ sin ⋅ x 2 d  
x x x Also, y = F(x) is an odd function
1 y y Choice (d)
⇒ d(xy) = tan ⋅ d  
xy x x tan x tan x
u 1 
y 224. ∫ 1+ u 2
dx =  log 1 + u 2 
2  1e
( )
log(xy) = log sec + log k 1
e
x
y 1 1  e2 + 1 
xy = k ⋅ sec = log sec2x - log  2 
x 2 2  e 
Integral Calculus  3.163

 e2 + 1  Eliminating c between (1) and (2), the differential


1
= log sec x - log  2   — (1) equation of the family of curves represented by (1)
2  e  is given by
3
cot x cot x dy  dy   dy 
2
dx 1 u  y = 3xy dx + 27 y  dx  + 9 y4   − 1
6
∫ u (1 + u ) ∫ 3 2
=  u −  dx
1
2
1 1 + u2   dx 
e e
which is first order, third degree
cot x
 1 
=  log u − log 1 + u 2 
 2  1e
( ) choice (c)

227. Set x = tan q


1 dx = sec2q dq
= log cot x - log cosec2x p
2 x = 0 → q = 0; x = ∞ → q =
2
 1  1 + e2  
 −  −1 − log  2   (tan q)(log tan q)sec2 qdq
p
∞ 2
2  e 
 ∫=
0

0 sec 4 q
1  1 + e2 
= log cos x + 1 + log  2   —(2) p
2  e  1 2
= ∫ sin2q log tan q dq = I (say)
(1) + (2) gives the value as 1 20
p
choice (d) 1 2
= ∫ sin2q log cot q dq (using the result
225. x(y log x - x) dy = y(x logy - y) dx 20
a a
x y
⇒ xy (logx -
y
) dy = yx (log y - ) dx
x
( )
∫ ˘˘˘˘ = ∫ ( − )
0 0
p
x y 1 2
⇒ (logx - ) dy = (log y - ) dx
y x 2I =
2 ∫ (sin2q) log tan q + log cot q dq
0
x y = 0 → I = 0
(log y) dx + dy = (log x) dy + dx
y x
choice (a)
⇒ d(× log y) = d(y log x)
228. From the given relation, we infer that the function is
Integrating, given by
x log y = y log x + c f(x) = xex
1 1
x = e, y = 1 gives
∫ x f(x)dx = ∫ x e dx
2 3 x

0 = 1 + c ⇒ c = -1 0 0

x log y - y log x + 1 = 0 = (x3ex - 3x2ex + 6xex - 6 )10


log yx = log xy - 1
= [ex (x3 - 3x2 + 6x - 6) ]10
x=1
= e(1 - 3 + 6 - 6) - [(-6) ]0
log y = 0 - 1
= 6 - 2e
1
y= choice (c)
e
choice (b) 1

229. Let ∫ y ( x ) dx = k
226. y3 = cx + c3 + c2 - 1  — (1) 0

Differentiating w r t x, The given relation may be rewritten as


dy dy
3y2 =c —(2) =x+k
dx dx
3.164  Integral Calculus

Integrating, d
x2
231.
dx
(xsin x + cos x ) = x cos x
y= + kx + l p p
2 4 4
x cos x
y(0) = 1 → 1 = 0 + 0 + l ⇒ l = 1 ∫= ∫ ( xsec x ). dx
(xsin x + cos x )
2
0 0
x2
therefore, y = + kx + 1 p
2 4  −1 
1 1
= ∫ ( xsec x ) d  xsin x + cos x 
 x2  0  
∫ y ( x ) dx = ∫0  2 + kx + 1 dx p
0
 − xsec x  4
=  
1 k  xsin x + cos x  0
k = + +1
6 2
(cos x + x sin x ) dx
p
4
k 7 7 −1
= →k=
2 6 3
= ∫ ( xsin x + cos x )
0
×
cos2 x
x 2 7x p × 2 p
4
y(x) = + +1 4
∫ sec
2
2 3 = + xdx
p 
 + 1  0
2
1 1
 x 3 7x 2  4 2
∫ xy ( x ) dx = ∫ +
 2 3
+ x  dx

0 0 −p 2
1 7 1 9 + 56 + 36 = 4 +1
= + + = (p + 4 )
8 9 2 72
101 4 2
=
72 −p 2 4 2
= × +1
choice (d) 4 (p + 4 )
230. Observe that g(2) =0 −2p
= +1
1
2+h (p + 4 )
g(2 + h) = ∫ 3t − 2g ' (t ) dt
( h)
2 + 2 =
4−p
4+p
2+h

g (2 + h ) − g (2) ∫ {3t − 2g ' (t )} dt choice (a)


= 2
232.
h h (2 + h ) y

g (2 + h ) − g (2)
lim
h→0 h 1 2 x
 3 (2 + h ) − 2g ' (2 + h )  0
= lim   y = x2 –1 –1
h→0
 2 + 2h 
πx
6 − 2g ' (2) y = cos
2
=
2
1 1
⇒ g’(2) = 3 - g’(2) px
Required area = ∫ cos ∫ (x )
2
dx - − 1 dx +
2
3 0 0
g’(2) =
2 2 2
px
∫ (x )
− 1 dx - ∫ cos
2
dx
choice (d) 1 1
2
Integral Calculus  3.165

1 1 y
2 px   x3 
=  sin  -  − x  +
p 2 0  3 0
2 2 52 4
 x3  2 px 
 3 − x  -  p sin 2  1 α 5 β x
1

2  2 8 1   −2 
= −  −  +  − 2 − + 1 −  
p  3 3 3   p
4 We have
= +2
p H’ (l) = f(l) - f(5 - l)
choice (c) H’(l) > 0 if f(l) > f (5 - l)
dy 5
233. We have = (1 + x) (1 + y + y2) 0<l<
dx 2
choice (c)
dy
= (1 + x) dx
1 + y + y2 n −1
 k
dy
235. ∑ log 1 − n 
k =0
2
= (1 + x) dx
 1  3   1  2
 y + 2  +  2  = log1 + log 1 −  + log 1 − n 
   n
dy  n − 1
= (1 + x) dx + ….+ log 1 −
 1  3 
2
 n 
 y + 2  +  2 
  lim 1 n −1
 k
Integrating both sides
n→∞
n
∑ log 1 − n 
k =0

lim
2  2y + 1  x 2
=n→∞
tan −1   = +x+c
3  3  2
 1

x = 0, y = 0  Riemann sum corresponding ∫ log (1 − x ) dx 
 0 
2 p p
⇒ × = c ⇒c= 1 1
3 6 3 3 = ∫ log (1 − x ) dx = ∫ log x dx
2  2y + 1  x2 p 0 0

Hence, tan-1  = +x+ = ( x log x − x )0


1
 3   2
3 3 3
lim
choice (a) = - 1 - x → 0 x logx = - 1 - 0 = -1
234. f(x) = x(2x2 - 15x + 24)
0 
2x2 - 15x + 24 = 0 236. lim  form 
x →0  0 
15 ± 225 − 192 15 ± 33
x= = sin x
4 4 = lim
x →0 2x cos x 2
Roots of f(x) = 0 are 0, a, b where 0 < a < 5 2 and b > 5
f ’(x) = 6x2 - 30x + 24  1  lim  sin x 
= lim   × x→0  x 
x → 0  2cos x 2 
= 6(x2 - 5x + 4)  

= 6(x - 1) (x - 4) 1 1
= ×1 =
f(x) is max at x = 1 and min at x = 4. 2 2
The graph of y = f(x) is roughly as follows choice (a)
3.166  Integral Calculus

237. e2
1
y
x = 4y − 2
= 2 × 2 - ∫ x d (f (x ))
1

 f x e 
2

 ( ) f (x )
e2

x
= 4 -    + ∫ 2
dx 
−1 0 2   x 1 1 x

 2 2
e2
f (x )
= 4 -  2 −  -
e 1 ∫ x2
dx
For the points of intersection, we solve x2 = 4y and 1

x = 4y - 2 2 1 23 2
= 6 - − = − 2
We get x = -1, 2 e2 4 4 e
2
 a + 2 x2  choice (a)
Required area = ∫  4 − 4  dx
−1  240. Putting x = tan q;
2 p
1 x x  2 3
∞ 2
dq
=  + 2x − 
4 2 3  −1 ∫=
0
∫ 1 + tan
0
9
q
9 p
= 2
cos9 q p
8
choice (d)
= ∫
0 sin9 q + cos9 q
=
4
p choice (c)
238. The two curves intersect at x =
4
1 1
y 241. 2f(x) + f(-x) = sin (x - )  — (1)
x x
change x to -x in the above;
1 1
2f(-x) + f(x) = - sin (x + )
x x
−π −π 0 π π π π 3π x 1 1
2 3 6 3 2 = sin (x - ) — (2)
2 x x
(1) × 2 — (2) gives
y = tanx y = cotx 1 1
3f(x) = sin (x - )
x x
p p
4 4 1 1
f(x) = sin (x - )
Required area = ∫ tan x dx +
0
∫ cot x dx
0
3x x
e e
1 1  1
= ( log sec x )0 + ( log sin x )p ∫ f(x)dx = ∫
p p
4 2 sin  x -  dx
4 1/e
3 1/e x  x
 1  1
1/e
1 -1
= log 2 - log 
 2  = x ∫ t x sin  - t  
 2  dt
3 e  t t
 
= 2 log 2 = log 2 1/e
1  -1   1  1
choice (c) = x ∫   sin  - t  dt t =
3 e  t  t  x
2
e e2 1/e

∫ f " ( x ) log x dx = ∫ (log x ) d (f ' ( x ))


239. 1 1  1
= x ∫ sin  t -  dt
1 1 3 e t  t
e2
e2
f ' (x ) -1 1  1 
e
=  f ' ( x ) log x 1 − ∫ dx = x ∫ sin  t -  dt
1
x 3 1/e t  t
Integral Calculus  3.167

e 2

∫ f(x)dx = 0 243. l = ∫ e t dt
2


1/e 1

Choice (d) put t2 = log x


1
x2 2tdt = dx
242. 2 x
(x − p2 )(x 2 − q 2 )(x 2 − r 2 )
Limits for x become e to e4
− p2 −q 2 e4
1
(q − p )(r − p ) (p − q 2 )(r 2 − q 2 )
2
2 2 2 2 l= ∫ e log x.
2xt
dx
= + e
(x 2 + p2 ) (x 2 + q 2 ) e4 e4
1
−r 2 = ∫
e
2 log x
dx = ∫ x.d 
e
log x 

(p − r )(q 2 − r 2 )
2 2
+ e4

( ) ∫
e4
(x 2 + r 2 ) = x log x - log x dx
 e
e
+ p2 +q 2 e4
(p2 − q 2 )(r 2 − p2 ) (q 2 − r 2 )(p2 − q 2 )
+ = 2e4 - e - ∫ log t dt
(x 2 + p2 ) (x 2 + q 2 ) e

e4
+r 2 ⇒ ∫ log t dt = 2e4 - e - l
(r − p )(q 2 − r 2 )
2 2 e
 +
(x 2 + r 2 ) choice (d)
p /4
x2
∫ (x 2 + p2 )(x 2 + q 2 )(x 2 + r 2 ) dx ∫ tan
n−2
244. In = x(sec2 x − 1)dx
0

p /4 p /4
p x  tan n −1 x 
 − ∫ tan x dx
n−2
tan −1   =
2 2 2 2
(p − q )(r − p ) p  (n − 1)  0 0

1
q x = − In−2
+ 2 2 2 2
tan −1   (n − 1)
(q − r )(p − q ) q 
1
In + In - 2 =
r x n −1
+ 2 tan −1  
2 2 2
(r − p )(q − r ) r 2n + 1
(2n +1) (In + In - 2)=

p p (n − 1)
∫ 2 ∑ (p
0
2
− q )(r 2 − p2 )
2
lim (2n +1) (In + In - 2) = 2
n →∞

p choice (b)
= x ∑ p(q 2 − r 2 )
2(p − q )(q − r 2 )(r 2 − p2 )
2 2 2
l 2 3 4

∫ = ∫ F (x)dx + ∫ 2F (x)dx + ∫ 3F (x)dx + ....


1 1 1
245.
 — (1)
1 1 2 3

Now, ∑ p (q - r ) = (p - q) (q - r) (r - p) substitut-
2 2 [l] l

∫ ([l] − 1)F1 (x)dx + ∫ [l ]F (x)dx


1
+
ing in (1),
[ l ] −1 [l]

p = {F(2) - F(1)} +2 {F(3) - F(2)}
∫0 = 2(p + q)(q + r)(r + p)
+3 {F(4) - F(3)} +….+ ([l] - 1) (F([l])
choice (b) - F([l}] - 1) + [l] (F(l) - F ([l]))
3.168  Integral Calculus

= - F(1) - F(2) - F(3) -… x = x0, y = e


- F([l] - 1) - F ([l]) + [l] F (l) x 20 1
[l] 1= −
2e2 2
= [l] F(l) - ∑ F(i)
i =1 x 20 = 3e2
choice (b)
x0 = e 3
246. y = x + sin x choice (d)
y’ = 1 + cos x ≥ 0
dy ( x + 1) + y − 3
2
y = x and y = x + sin x intersect at the points 249. =
x = 0 and x = p dx ( x + 1)
In [0, p] x + sin x ≥ x dy
p
(x + 1) = (x + 1)2 + y - 3
Therefore, required area = ∫ ( x + sin x ) − x  dx dx
0
dy 1 3
=2 − y = ( x + 1) −
dx ( x + 1) x +1
247. y > 0 → x + 2y =1 general solution is given by
y < 0 → x - 2y = 1
1  3 
y× = C + ∫ 1 −  dx
y x +2y = 1 ( x + 1)  ( x + 1) 
2

A
3
= C + x +
O C x x +1
B x = 2, y = 0 → C = -3
solution is
x − 2y = 1
y 3
= x−3+
1 1 1 x +1 x +1
Required area = 2 × x x1 =
2 2 2 y = (x - 3) (x - 1) + 3
choice (c) = x2 - 2x ⇒ y+ 1 = (x - 1)2
248. (x2 - y2) dy = xy dx
y
y2 dy = xy dx - x2 dy
= -x (x dy -y dx)
y 0 1
= -x3d   x
x
dy −x3  y  -1
= 3 d  
y y x
y 1
−d  
x Required area = 2 ∫ (2x − x 2 )dx
= 3 0
y
 x  4
=
3
x2
Integrating, log y = + +c choice (b)
2y 2
−1 y
x = 1, y = 1 → c = 250. y = tx → t =
2 x
x2 1 ay  y2 
solution curve is log y = - Therefore, y = 1− 2 
2y 2
2 x  x 
Integral Calculus  3.169

⇒ x3 = a (x2 - y2) p y


ay2 = ax2 - x3 = x2 (a - x) = sin   ⇒ k = 2
2xy x
x 2 (a − x)
y2 = choice (a)
a
252. y
y y = e–x
1

x x
−e 1−e
t = −1 t =−1
(x =−a) (x = a)

Area of a loop of the curve 1


x = log   → y = e-x
2
a y
=
a0
∫ x a − x dx The curves y = e-x and y = log (x + e) intersect at x = 0
0 ∞
x = a sin2q
dx = 2a sinq cosq dq
Required area = ∫
1− e
log(x + e)dx + ∫ e − x dx
0
p e
Required area
2 = ∫ log t dt +1 = (t log t − t)1e + 1 = 2
p /2 1
2
∫ (a sin q) a cos q x 2a sin q cos q dq
2
= choice (d)
a 0
p /2 1 + sin3x 1 + 3s − 4s3
253. = where s = sin x
= 4a 2 ∫0
sin3 q cos2 q dq 1 + 2sin x (1 + 2s)

2 8a 2 (s + 1 − 2s2 )(1 + 2s)


= 4a2 × = =
5x 3 15 (1 + 2s)
p /2
choice (a)
∫ (sin x + 1 − 2sin
2
⇒ x)dx
0
251. The equation may be rewritten as
p 1 p
y y y =1+ -2× × =1
(y cos ) × x2d   + x sin d (xy) = 0 2 2 2
x x x
choice (c)
y y d(xy)
⇒ cot d   + =0 254. When x = -2, f(x) = -5
x x xy
y  1
Integrating, log (sin ) + log (xy) = C 4(2x − 1) + 7, − 2 < x < 2
x 
 1
y g of (x) = 5(2x − 1)2 − (2x − 1) + 7, < x < 0
xy sin =C  2
x 2 2 2
5(3x − 2) − (3x − 2) + 7 x > 0
p p 
y (1) = ⇒ ×1=C
2 2
 1
p 8x + 3, 2 < x < 2
C = 
2  1
solution is = 20x 2 − 22x + 13, < x < 0
 2
4 2
y p 45x − 63x + 29, x > 0
xy sin = 
x 2
3.170  Integral Calculus

2
dy dy
256. = (y - 1)cosec2x ⇒ = cosec2x dx

−2
∫ (g of )(x)dx dx y −1
1/2 Integrating, ln(y - 1) = - cot x + C  — (1)
∫ (8x + 3)dx + p 
= −2 passes through  , 2 
0 2 
∫ (20x
2
− 22x + 13)dx
p
1/2 ⇒ ln 1 = -cot +C⇒C=0
2
2
 + ∫ (45x 4 − 63x 2 + 29)dx \ y = 1 + e-cot x
0
As cot x is not defined when x = np, f(x) is not con-
 20 x  3 0
tinuous at these points.
= (4x 2 + 3x)1/2
−2 +  − 11x 2 + 13x 
 3 1/2 1
5 3 2
257. Let I = ∫ x log ex e log e2 x e dx
 + (9x − 21x + 29x) 0
dx
−15 55 1991
= ∫ x log ex log e e2 x
= − + 178 = e
2 12 12
dx dt
choice (d) = ∫ x (1 + ln x )(2 + ln x ) = ∫ (1 + t )(2 + t )
2 2
dx
255. Let ∫ f(t)dt = A, ∫ t(f )dt = B (by taking lnx = t so that = dt)
0 0
x
Then, f(x) = 2 + Ax2 + B  —(1)  1 1   1 + lnx 
= ∫ −  dt = log  +C
Integrate (1) over (0, 2) w.r.t.x. 1 + t 2+ t   2 + lnx 

8A 5A ⇒ f(x) = lnx
A=4+ +2B⇒ +2 B= -4
3 3 f(1 + x) ln(1 + x)
⇒ lim = lim =1
x→ 0 x x →0 x
⇒ 5 A + 6 B = -2  —(2)
\ choice is (b)
Multiplying both sides of (1) by x and integrating
w.r.t.x. over (0, 2), 1
p
2

B=4+4A+2B 258. ∫ sin (1 − x ) dx + ∫ sin (x − 1) dx


0 1
⇒ 4 A + B  —(3)
 − cos (1 − x ) 
1
p
Solving (2) and (3), =   − cos ( x − 1)1
2

−12 −28  (−1)  0


A= ,B =
19 19  p  
= (1 - cos1) -  cos  − 1 − 1
−12x2
28  2  
Hence, f(x) = 2 - −
19 19 = 2 - cos1 - sin1
2
10 12x d  x  25 − 4x 5
= − 259. We have = >0
19 19 dx  x 5 + 25  (
x 5 + 25
2
)
1 1
 10 12x 2 
∫ f(x)dx = ∫  − dx (as 0 < x5 < 1 for 0 < x < 1)
0
19 19 
0
26x
⇒ f (x) = 5 is an increasing function
10 12 10 4 6 x + 25
= − = − =
19 57 19 19 19 on [0, 1]
choice (a) ⇒ Min. f(x) = f(0) = 0 (say, m) and
Integral Calculus  3.171

26 By symmetry, required area (shaded region)


Max. f(x) = f(1) = = 1(say, M)
26 = 2 × Area OABP
3
 2 x2 3
8 
= 2 ∫ f ( x ) dx = 2  ∫ dx + ∫ 2
1
dx 
\ m(1 - 0) < 26 ∫ f(x) dx < M(1 - 0) 0 0 4 2 x + 4 
0
  x3 2 8  −1 x  
3
⇒ 0 < I < 1 = 2   + tan 
 2  2 
  12  0 2 
260. D1 = Area of DOCD
Y 2  3 p 
1 m = 2  + 4  tan −1 −  
= × 1x |m| = B(1, 5) 3  2 4 
2 2
(0, 2) x=1 262. Let y 4 = p. Given differential equation becomes
D2 = Area OCBA C (1, 0) xp’ = p
1 0 X
d4 y
∫ (3x )
2 D
= + 2 dx = 3 (1, m) ⇒ p = C1x (ie) = C1x
0 y = mx
dx 4
Integrating successively we get
Given D1 = D2
C x5 C x3 C x2
We have m = ± 6 But m < 0 y= 1 + 2 + 3 + C4x + C5
120 6 2
⇒ m = -6
The curve is symmetric with respect to y-axis
⇒ C1 = C2 = C4 = 0
x2 C x2
261. We have y =  — (1)
4 \ y = 3 + C5 ⇒ y = C1x2 + C2
2
8
y = 2 — (2)
x +4 1− x
263. Let − x 2 + 3x − 2 = t(x − 2) ⇒ t =
x2 8 x−2
(1) - (2) ⇒ − 2 =0
4 x +4 1− x 2t 2 + 1 2tdt
⇒ t2 = ⇒x= ⇒ dx =
⇒ x + 4x - 32 = 0 ⇒ (x + 8)(x - 4) = 0
(1 + t )
4 2 2 2
x− 2 2
t +1 2 2

⇒ x2 - 4 = 0 ⇒ x = ± 2 2t.dt

Y (1 + t )
2 2
2 −2
x 2 \ I = ∫ t  −t 
2
= −∫
t 2
dt =
t
+C
2 y= 4
Q
P 8 t +1  1 + t 2 
2
D 1 B y = x2 + 4
C A
X
x−2
−4 −3 −2 −1 O 1 2 3 4 = −2 +C
1 −x

264. y = x + sinx
(1) represents a parabola and (2) represents the curve
witch of Agnesi.
 x2 8 
\ f(x) = min  , 2
 4 x + 4  y = f’(x)
[∵f (x) is image of f(x) w.r. to y = x]
–1
 8 A
 , x≤ − 2
2 0 2π 3π 4π
 4 2+ x
π
x
=  , − 2 < x< 2 p
4 Required Area = 8A = 8 ∫ ( x + sin x − x ) dx
 8 , x≥ 2 0
 4 + x2
 = 8 (-cos x) = 8(2) = 16
3.172  Integral Calculus

x
sin t 269. Statement 2 is true
265. lim ∫ dt = 0 dx tan −1 y − x
x →5
5
t =
dy 1 + y2
 x sin t 
∫ dt 

dx
+
x
=
tan −1 y
t
We may write limit = lim(x) × lim  5  dy 1 + y 2 1 + y2
x →5 x →5  x − 5 
  which is linear in x.
  Statement 1 is true and follows from statement 2
 sin x 
270. Since f(x) = -f(1 - x), statement 2 is true.
 x  3/2
= 5 × lim , by L’ Hospital’ s rule
x →5 1 Using Statement 2,
−1/2
∫ f(x)dx = 0
sin5 Choice (a)
=5× = sin5
5
271. Statement 2 is true. Consider Statement 1
p
266. x + 3 = t2, dx = 2t dt
∫ sin
2
mx cosnx dx
2t dt 2dt  t − 1 0
∫ (t 2 − 1)t = ∫ t 2 − 1 = log  t + 1  1
p

(1 − cos2mx ) cosnx dx
2 ∫0
=
 x + 3 − 1
= log  +C p
 x + 3 + 1 1
=
2 ∫0
(cosnx − cos2mx cosnx ) dx
dy p
267. + 2y tan x = sin x 1 1
2 ∫0
dx = cosnxdx −
4
General solution is
p

y sec2 x = ∫ sec2 x sin x dx + c = sec x + c = ∫ cos(2m + n)x + cos(2m − n)x  dx


0
p 0 - 0 + 0, since 2m ≠ n
When x = , y = 0 ⇒ c = -2.
3 Statement 1 is true. However, it does not follow from
Solution is y sec2 x = sec x - 2 Statement 2. Choice (b)
y = (sec x - 2) cos2 x
272. Statement 2 is true.
= (1 - 2 cos x) cos x = cos x - 2 cos2 x Consider Statement 1 The general equation of the
1 family of parabolas in this case is
= -2 {cos2 x - cos x}
2 y2 = k (x - l)
  1 1 
2
Where k and l are parameters using Statement 2, we
= −2 (  cos x-  −  note that the differential equation of the family is of
  4 16 
order 2 Choice (a)
2
1  1
= − 2  cos x −  273. Statement 2 is true.
8  4
Consider Statement 1
1 p
Max y = In (0, ), sin x lies between 0 and 1.
8 2
268. Statement 2 is not true sin x 1 p
Hence 7
< 7
in (0, )
p x +1 x +1 2
∫ cos xdx = (sin x)
p
=0 p /2 p /2
0
sin x 1 p
0

Statement 1 is true
∫x
0
7
+1
dx < ∫x
0
7
+1
dx <
2
Integral Calculus  3.173

1 278. y2= 8x
(since max of = 1) Statement 1 is false.
7
x +1 L
Choice (d)

274. Statement 2 is true. x


O S(2, 0)
Consider Statement 1: Integrating the given equation,
dy
= x3 + C
dx
dy
y1(0) = 2 ⇒ 2 = C = x3 + 2 The shaded area is revolved about the x -axis.
dx 2
Integrating again, w. r. t. x, Volume of the solid generated = p∫ y 2 dx
x4 0
y= + 2x + D 2
4
= p∫ 8xdx = p(4x 2 )20
y(0) = 1 gives D = 1 0

x 4
= 16p
Hence, y= + 2x + 1
4 279. y
y(2) = 4 + 4 + 1 = 9
Statement 1 is false
y = sin x
\ choice (d)
x
O π
3p 3p
2 p 2
275. ∫ [sin x]dx = ∫ sin x  dx + ∫ sin x  dx
p p p
2 2

3p
p 2
3p The shaded region is revolved about the x - axis
= ∫ 0dx + ∫ −1dx = 0 - ( x )p 2

p p
Volume of the solid generated
2
p

(
− 3p − p = − p
2 2 ) = p∫ sin2 xdx
0
p
Statement 1 is false and Statement 2 is true 2
= p × 2 ∫ sin2 xdx
276. Statement 2 is a valid statement 0

Statement 1: For 0 ≤ x ≤ 1, 0 ≤ x2 ≤ 1 1 p p2
= 2p × × =
2 2 2
2 2
⇒ e0 ≤ e x ≤ e ⇒ 1 ≤ e x ≤ e
1 1 1
280. y
∫ 1. dx ≤ ∫ e dx ≤ ∫ edx
x2
\
0 0 0 y = x2
⇒ 1 ≤ I ≤ e
O x=2 x
\ Statement 1 is true and follows from Statement 2
\ choice is ‘a’

277. Statement 2 is true Volume of the solid generated


Statement 1 is true and follows from Statement 2 as x 2
2
= p lies within the interval of integration.  x5  32p
= ∫ px 4 dx = p   =
\ choice is ‘a’ 0  5  0
5
3.174  Integral Calculus

281. y From DOPM, OP2 + PM2 = OM2 = a2


c2 + h 2

(a - h)2 + c2 = a2 ⇒ a =
cycloid 2h
Substituting in (1), required volume
x
O θ=0 θ = 2π
 h 2 (c2 + h 2 ) h 3  ph 2
= p − = (3c + h 2 )
 2h 3  6

Volume of the solid generated


2p 2p
283. I = ∫
1

0
dx (∫ 2

0
(x 2
+ y 2 dy ) )
= p ∫ y 2 dx = p ∫ [a(1 − cos q)]2 × a(1 − cos q)dq 1  2 y3  1  8
2

∫  x y +  dx = ∫ 0  2x +  dx
2
0 0 =
2p
0
 3  y =0 3
= pa 3 ∫ (2sin2 q)2 (2sin2 q)dq 1
0  2x 3 8  2 8 10
=  + x = + =
2p p
 3 3 0 3 3 3
∫ sin ∫ sin
3 6 3 6
= 8pa qdq = 16pa qdq
0 0 p
2

∫ sin ( x + y ) dy
p p
2
284. I =
5× 3×1 p
= 32pa 3 ∫ sin6 qdq = 32pa 3 × × 0

6×4×2 2 p
p
0 2 p

∫ (− sin y + cos y )dy = (cos y + sin y )


2

= 5p a
2 3 x= =
2 0
0

282. y = 1 - (1) = 0
p p
M = ∫ sin ( x + y )  x2= 0 dy = ∫ p (cos y − sin y ) dy
p

p 2
2

p
O P A = sin y + cos y  p = 0-1-(1+ 0) = -2
2

N 285. One of the limits is a function of q. \ it refers to r. The


other limit is for q.
Let a be the radius of the sphere. Then OA = a a(1+ cos q )
 r2 
p
Let PM = c, PA = h \ I = ∫ 0 dq  2 
r =0
The required spherical cap is obtained by revolving the
shaded region PAM about the x-axis. The equation of p a 2 (1 + cos q)2
the circular arc AM is x2 + y2 = a2
⇒ = ∫ 0 2
dq

Volume of the spherical cap of height is h and base a2 p q p


a a =
2 ∫ 0
4cos 4
2
dq = 2a2
∫ 2
cos 4 ϕ 2d ϕ
radius c = p ∫ y 2 dx = p ∫ (a 2 − x 2 )dx 0

a −h a −h q 3 1 p 3
a where, = ϕ ⇒ = 2a2 . . = pa 2
 x  3 2 2 2 2 4
= p a 2 x − 
 3 a −h
286. Y
 a3 (a − h)3  B
= p a 3 − − a 2 (a − h) +  x +y = 1
 3 3 
1−x
 h3 
= p ah 2 −  — (1) x
 3 O y=0 A

Integral Calculus  3.175

x+ y ≤ 1 is equivalent to 1
290. Given f ’(x) =
0≤x≤1 2
x +1 + x
0≤y≤1-x
1 x2 + 1 − x
1− x = ×
 xy 2 
1 x2 + 1 + x x2 + 1 − x
I= ∫ 0  2  dx
y =0 x2 + 1 − x
=
1 1 x2 + 1 − x2
2 ∫0
= x(1 − x)2 dx
= x2 × 1 − x

1 1 1 1 2 1
=
2 ∫0
(
x − 2x 2 + x 3 dx =  − + 
2 2 3 4
) f(x) = ∫ x 2 + 1dx - ∫ x dx
x 1 x2
1 f(x) = x2 + 1 + log x + x 2 + 1 - +c
= 2 2 2
24
1
f(0) = 0 + g + c = c → f(0) = 0 + c
 2
1 y3 
x
2
287. ∫0 
 x y +  dx
3 x 1− 2 1− 2
given f(0) = ⇒c=
2 2
1  x3 
5
1 3
= ∫  x 2 + x 2 − x 3 −  dx 1 1 1 1− 2
0  3 3 f(1) = 2 + log 1 + 2 - +
2 2 2 2
2 1 2 1 1 3
= + . − − =
7 3 5 4 12 35 =
1
log 1 + 2 = log
1+ 2 2 −1 ( )( )
288.
2 2 −1 ( )
Y
1 2 −1 1
a2 − x2 = log =- log ( 2 − 1 )
2 2 −1 2

O y=0 X 291. I = ∫ cosec2 x sec 4 x dx


A
∫ (1 + cot x )sec
2 4
= x dx
t = cot x, dt = -cosec x dx 2

The limits are 0 ≤ x ≤ a; 0 ≤ y ≤ a2 − x2 −dt


dx =
a
a2 − x2 1 + t2
I= ∫∫ y =0
xydy dx
cosec 4 x
x =0 sec4x =
cot 4 x
a2 − x2
 y2  1 a 2
( ) (1 + t ) 2 2
( )
a 2x1
= ∫ x 
2 ∫0
dx = a x − x 3 dx dt 1 + t2
x =0
2 (
=- ∫ 1 + t 2
) t4 1 + t2
= − ∫ t4 dt
a
1  2 x2 x 4  1  a 4 a 4  a 4 1 + 2t 2 + t 4
= a −  =  − = = −∫ dt
2 2 4 0 2  2 4  8 t4

1
= − ∫ t −4 dt − 2 ∫ t −2 dt - ∫ dt
3 2  z  2
289. I = ∫ dx ∫ dy  xz + yz +  t −3 t −1
0 0
 2 0 =− -2 -t+C
−3 −1
3 2  1 1 1 2
=∫ ∫  x + y + 2  dy = × 3
+ − cot x + C
0 0 3 cot x cot x
3 3 1
= ∫ (2x + 2 + 1)dx = x 2 + 3x  = 18 = tan3x + 2 tanx - cot x + C
0 0 3
3.176  Integral Calculus

 log (sec x + tan x ) + sec x  dx


log 4
∫e
x
292. 39
∫ (Ae )
2x
+ Be x dx =
f(x) = log (sec x + tan x) 0
2
1 A 2 log 4 A 39
f ’ (x) = (secx tanx + sec2x) e + Be log 4 − − B =
(sec x + tan x ) 2 2 2
sec x (tan x + sec x ) 15A + 6B = 39  —(3)
= = sec x
(sec x + tan x ) from (2)
A + B = -1
Integral is of the type ∫ e x ( f ( x ) + f '(x)) dx = ex f(x)
5A + 2B = 13 from (3)
\ Ans = ex, log (sec x + tan x) B = 6, A = 5, C = 3
 sec x − tan x 
= ex log [sec x + tan x ] × 296. For 0 < x < 1, x4 < x3
 (sec x − tan x )  for 1 < x < 2, x4 > x3
sec2 x − tan2 x
4 3 4 3
2 x < 2 x and 2 x > 2 x
= ex log +C
sec x − tan x 1 1
x3
∫ 2 dx < ∫2
x4
 1  \ dx
= exlog  +C
 sec x − tan x 
0 0
2 1
= -ex log (secx - tan x) + C and ∫ 2 x 4 dx > ∫ 2 x3 dx
1 2
x dx 3 3 1 ⇒ I2 < I1 and I4 > I3
293. I = ∫ 1 − x3
put x 2
= t , x3 = t2 , x 2 dx = dt
2
p

2
2
sin x dx
⇒ x dx = dt
3
297. Given A = ∫ sin x + cos x
0

2 dt 2 2 3 p 
I=
3 ∫ 1− t 2
=
3
sin-1 t = sin −1 x 2 + c
3
p
2
sin  − x  dx
2 
3
= ∫ p  p 
f(x) = sin x, g(x) = x x = x
-1 2 0
sin  − x  + cos  − x 
2  2 
3 3
(fog) (x) = f(x 2 ) = sin −1 x 2
p
2
cos x dx
x
Also given B = ∫ sin x + cos x
∫ 2 − t 2 dt 0
294. f(x) =
1 p p
2
sin x dx 2
cos x dx
f ’(x) = 2−x 2 \ A + B = ∫ sin x + cos x
0
+ ∫ sin x + cos x
0
x2 - f ’(x) = 0 gives p p
2 2
sin x + cos x p
∫ dx = [ x ]
p
x2 - 2 − x 2 = 0 = ∫ sin x + cos x
dx = 0
2
=
2
2 - x2 = x4 0 0

x4 + x2 - 2 = 0 p
Also, A = B =
4
x2 = 1, x = ± 1
1 1
295. f(x) = Ae2x + Bex + cx 298. , are the roots of 6x2 - 5x + 1 = 0
f ’(x) = 2 Ae2x + Bex + c a b
31 = 2 Ae2 log2 + Belog2 + c \ {a, b} = {2, 3}
8A + 2B + c = 31  —(1)  p 3p 
Since cosq = (a - b), q ∈  ,  ⇒ a = 2, b = 3
Also -1 = A + B  —(2) 2 2 
Integral Calculus  3.177

1 1 1 2 2 b

∫( )
2
a=1+ + + ....∞ = = = =2 (d) x − a + b − x dx
a a2 1 2 −1 2 −1
1− a
a 3
b 1 1
= 1 + + 2 + ....∞ =
1
=
3
=
3 = ∫  x − 2 + 3 − x + 2

(x − 2)(3 − x )  dx
a b b 1 3−1 2 2
1−
b 3 3

\ b = 3
= ∫ 1dx + 2 ∫ ( x − 2)(3 − x )dx
2 2
i.e., a = 2, b = 3, a = 2, b = 3 p
2
b
pdx dx
3 = 1 + 2 ∫ 2sin2 q cos2 q dq
(a) ∫ = p∫
a (
x + a )( x + b) 2 (
x + 2)( x + 3)
0

3 1 p 3
3 = 1 + 4 . . . = 1+ p
 1 1  4 2 2 4
= p∫  − dx
 x + 2 x + 3 
2
299. (a) Put 2x = t ⇒ 2x log2dx = dt
3
 x + 2   25  1 dt 1
= p log 
 x + 3   2
= p log  
 24  \ I =
log 2 ∫ 2
=
log 2
( )
sin −1 2 x + C
1− t
b 3
x−a x−2 dx −2tdt
(b) ∫a
b−x
dx = ∫
2
3−x
dx (b) 1 - x3 = 1 ⇒ - 3x2 dx = dt ⇒
x
=
3x 3
x = 3sin2q + 2cos2q −2 dt
3 ∫ 1 − t2
\ I =
p
2
sin2 q
= ∫
0 cos2 q
.2sin q cos qdq
=
−2 1
;log
1+ t
+C
3 2 1− t
dx = 2sinq cosq dq
p −1 1 + 1 − x3
1 p p 2
= log
= 2 ∫ sin2 qdq = 2. . = 3 1 − 1 − x3
0
2 2 2
when x = 2, ⇒ q = 0 1 1 − x3 − 1
= log +C
p 3 1 − x 3 +1
x=3⇒q=
2
1
x - 2 = sin2q \ k =
3
3 - x = cos2q
dx cos x dx
b
 x−a b−x (c) ∫ 1 + tan x = ∫ sin x + cos x
(c) ∫a  b − x + x − a 
dx
1
 x−2
3
3−x
2
= (
x + log (sin x + cos x ) + C )
=∫ + dx
2
3−x x − 2  1
\ k =
3
x−2+3−x 2
= ∫ (3 − x )(x − 2) dx  1 1 
2
(d) I = ∫  2 + 2  dx
3
dx  x − 1 x +4 
= ∫ (3 − x )(x − 2)
2 1  x + 1 1 x
= log  + tan −1   + C
p
2  x − 1  2 2
2
2sin q cos q
1
= ∫ sin q cos q
dq = p \ k =
0 2
3.178  Integral Calculus

2p
xsin2n x where, t = tan2x
300. (a) I = ∫ sin
0
2n
x + cos2n x
dx
=
p
(
tan −1 ∞ − tan −1 0 =
p2
)
4 8
a a

By property ∫ f(x)dx = ∫ f(a − x) dx p2


I =
0 0 16
2p
sin x 2n p
sin x
( )
dx = − p tan −1 (cos x )
p
2I = 2p ∫ dx (c) 2I = p ∫ 1 + cos
0 sin x + cos2n x
2n
0
2
x 0

p
2  p p  p2
sin2n x p 2I = - p  − −  =
\ I = 4p ∫0
2n
sin x + cos x2n
= 4 p × = p2
4
 4 4 2
p2
p
2
xsin x cos x \I=
4
(b) I= ∫ sin 4
x + cos 4 x
dx
x

∫ (tan x )
−1 2

p  p  p  0 ∞
p
2  2 − x  sin  2 − x  cos  2 − x  (d) lim  ∞ 
x→∞ 2
x +1
= ∫ 4 p  4 p 
dx
x
sin  − x  + cos  − x  d
∫ (tan x )
0 2
−1
2  2 
dx 0
p = lim
p 2
sin x cos x x →∞ 2x
=
2 ∫ sin
0
4
x + cos x 4
dx − I
2 x 2 +1

(tan x )
p 2
−1
p 2
tan xsec2 x x 2 +1
2I =
2 ∫ 1 + tan 4 x
dx = lim
x→∞ x

p dt 1 p2
( )
2
= ∫
4 0 1 + t2
= lim tan −1 x
x→∞
1+
x2
=
4

You might also like